Sie sind auf Seite 1von 664

G.R. No. L-24693: Ermita-Malate Hotel & Motel Operators Assoc.

, Inc vs Mayor of Manila

On 13 June 1963, the Manila Municipal Board enacted Ord 4760 and thesame was approved by then acting mayor Astorga. Ord 4760 sought toregulate hotels and motels. It classified them into 1st class (taxed at 6k/yr) and 2ndclass (taxed at 4.5k/yr). It also compelled hotels/motels to get thedemographics of anyone who checks in to their rooms. It compelledhotels/motels to have wide open spaces so as not to conceal theidentity of their patrons. Ermita-Malate impugned the validity of thelaw averring that such is oppressive, arbitrary and against dueprocess. The lower court as well as the appellate court ruled in favorof Ermita-Malate. ISSUE: Whether or not Ord 4760 is against the due process clause. HELD: TheSC ruled in favor of Astorga. There is a presumption that the lawsenacted by Congress (in this case Mun Board) is valid. W/o a showing ora strong foundation of invalidity, the presumption stays. As in thiscase, there was only a stipulation of facts and such cannot prevailover the presumption. Further, the ordinance is a valid exercise ofPolice Power. There is no question but that the challenged ordinancewas precisely enacted to minimize certain practices hurtful to publicmorals. This is to minimize prostitution. The increase in taxes notonly discourages hotels/motels in doing any business other than legalbut also increases the revenue of the lgu concerned. And taxation is avalid exercise of police power as well. The due process contention islikewise untenable, due process has no exact definition but has reasonas a standard. In this case, the precise reason why the ordinance wasenacted was to curb down prostitution in the city which is reasonenough and cannot be defeated by mere singling out of the provisions ofthe said ordinance alleged to be vague.
(White Light Corp. vs. City of ManilaG.R. No. 122846, January 20, 2009) Mayor Lim signed into law City Ordinance No. 7774 prohibiting short-time admission, short-time admission rates, and wash-up rate schemes in hotels, motels, inns, lodging houses, pension houses, and similar establishments in the city of manila. White Light Corporation and other operators of drive-inhotels and motels in Manila complained that the Ordinance is unconstitutional and void since it violates the right to privacy and the freedom of movement; an invalid exercise of police power; and an unreasonable and oppressive interference in their business. On the other hand, the City of Manila argued that the Ordinance is a valid police power measure. It asserts that the subject establishments have gained notoriety as venue of prostitution, adultery and fornications in Manila. Thus, it became the ideal haven for prostitutes and thrill-seekers. Is Ordinance No. 7774 constitutional? SUGGESTED ANSWER: No, Ordinance No. 7774 is unconstitutional. The SC ruled that the ordinance is an arbitrary and whimsical intrusion into the rights of the establishments as well as their patrons. The Ordinance needlessly restrains the operation of the businesses of the petitioners as well as restricting the rights of their patrons without sufficient justification. The Ordinance rashly equates wash rates and renting out a room more than twice a day with immorality without accommodating innocuous intentions.

That the Ordinance prevents the lawful uses of a wash rate depriving patrons of a product and the petitioners of lucrative business ties in with another constitutional requisite for the legitimacy of the Ordinance as a police power measure. It must appear that the interests of the public generally, as distinguished from those of a particular class, require an interference with private rights and the means must be reasonably necessary for the accomplishment of the purpose and not unduly oppressive of private rights. It must also be evident that no other alternative for the accomplishment of the purpose less intrusive of private rights can work. More importantly, a reasonable relation must exist between the purposes of the measure and the means employed for its accomplishment, for even under the guise of protecting the public interest, personal rights and those pertaining to private property will not be permitted to be arbitrarily invaded. Lacking a concurrence of these requisites, the police measure shall be struck down as an arbitrary intrusion into private rights. As held in Morfe v. Mutuc, the exercise of police power is subject to judicial review when life, liberty or property is affected. The behavior which the Ordinance seeks to curtail is in fact already prohibited and could in fact be diminished simply by applying existing laws. Less intrusive measures such as curbing the proliferation of prostitutes and drug dealers through active police work would be more effective in easing the situation. So would the strict enforcement of existing laws and regulations penalizing prostitution and drug use. These measures would have minimal intrusion on the businesses of the petitioners and other legitimate merchants. Further, it is apparent that the Ordinance can easily be circumvented by merely paying the whole day rate without any hindrance to those engaged in illicit activities. (visit fellester.blogspot.com) Moreover, drug dealers and prostitutes can in fact collect wash rates from their clientele by charging their customers a portion of the rent for motel rooms and even apartments.

White Light Corp., vs City of Manila


On 3 Dec 1992, then Mayor Lim signed into law Ord 7774 entitled An Ordinance prohibiting short time admission in hotels, motels, lodging houses, pension houses and similar establishments in the City of Manila. White Light Corp is an operator of mini hotels and motels who sought to have the Ordinance be nullified as the said Ordinance infringes on the private rights of their patrons. The RTC ruled in favor of WLC. It ruled that the Ordinance strikes at the personal liberty of the individual guaranteed by the Constitution. The City maintains that the ordinance is valid as it is a valid exercise of police power. Under the LGC, the City is empowered to regulate the establishment, operation and maintenance of cafes, restaurants, beerhouses, hotels, motels, inns, pension houses, lodging houses and other similar establishments, including tourist guides and transports. The CA ruled in favor of the City.

ISSUE: Whether or not Ord 7774 is valid. HELD: The SC ruled that the said ordinance is null and void as it indeed infringes upon individual liberty. It also violates the due process clause which serves as a guaranty for protection against arbitrary regulation or seizure. The said ordinance invades private rights. Note that not all who goes into motels and hotels for wash up rate are really there for obscene purposes only. Some are tourists who needed rest or to wash up or to freshen up. Hence, the infidelity sought to be avoided by the said ordinance is more or less subjected only to a limited group of people. The SC reiterates that individual rights may be adversely affected only to the extent that may fairly be required by the legitimate demands of public interest or public welfare.

City of Manila vs Judge Perfecto Laguio


Police Power On 30 Mar 1993, Mayor Lim signed into law Ord 7783 entitled AN ORDINANCE PROHIBITING THE ESTABLISHMENT OR OPERATION OF BUSINESSES PROVIDING CERTAIN FORMS OF AMUSEMENT, ENTERTAINMENT, SERVICES AND FACILITIES IN THE ERMITA-MALATE AREA, PRESCRIBING PENALTIES FOR VIOLATION THEREOF, AND FOR OTHER PURPOSES. It basically prohibited establishments such as bars, karaoke bars, motels and hotels from operating in the Malate District which was notoriously viewed as a red light district harboring thrill seekers. Malate Tourist Development Corporation avers that the ordinance is invalid as it includes hotels and motels in the enumeration of places offering amusement or entertainment. MTDC reiterates that they do not market such nor do they use women as tools for entertainment. MTDC also avers that under the LGC, LGUs can only regulate motels but cannot prohibit their operation. The City reiterates that the Ordinance is a valid exercise of Police Power as provided as well in the LGC. The City likewise emphasized that the purpose of the law is to promote morality in the City. ISSUE: Whether or not Ordinance 7783 is valid. HELD: The SC ruled that the said Ordinance is null and void. The SC noted that for an ordinance to be valid, it must not only be within the corporate powers of the local government unit to enact and must be passed according to the procedure prescribed by law, it must also conform to the following substantive requirements: (1) must not contravene the Constitution or any statute; (2) must not be unfair or oppressive; (3) must not be partial or discriminatory; (4) must not prohibit but may regulate trade; (5) must be general and consistent with public policy; and

(6) must not be unreasonable. The police power of the City Council, however broad and far-reaching, is subordinate to the constitutional limitations thereon; and is subject to the limitation that its exercise must be reasonable and for the public good. In the case at bar, the enactment of the Ordinance was an invalid exercise of delegated power as it is unconstitutional and repugnant to general laws.

JMM Promotion and Management vs Court of Appeals


Police Power Due to the death of one Maricris Sioson in 1991, Cory banned the deployment of performing artists to Japan and other destinations. This was relaxed however with the introduction of the Entertainment Industry Advisory Council which later proposed a plan to POEA to screen and train performing artists seeking to go abroad. In pursuant to the proposal POEA and the secretary of DOLE sought a 4 step plan to realize the plan which included an Artists Record Book which a performing artist must acquire prior to being deployed abroad. The Federation of Talent Managers of the Philippines assailed the validity of the said regulation as it violated the right to travel, abridge existing contracts and rights and deprives artists of their individual rights. JMM intervened to bolster the cause of FETMOP. The lower court ruled in favor of EIAC. ISSUE: Whether or not the regulation by EIAC is valid. HELD: The SC ruled in favor of the lower court. The regulation is a valid exercise of police power. Police power concerns government enactments which precisely interfere with personal liberty or property in order to promote the general welfare or the common good. As the assailed Department Order enjoys a presumed validity, it follows that the burden rests upon petitioners to demonstrate that the said order, particularly, its ARB requirement, does not enhance the public welfare or was exercised arbitrarily or unreasonably. The welfare of Filipino performing artists, particularly the women was paramount in the issuance of Department Order No. 3. Short of a total and absolute ban against the deployment of performing artists to high risk destinations, a measure which would only drive recruitment further underground, the new scheme at the very least rationalizes the method of screening performing artists by requiring reasonable educational and artistic skills from them and limits deployment to only those individuals adequately prepared for the unpredictable demands of employment as artists abroad. It cannot be gainsaid that this scheme at least lessens the room for exploitation by unscrupulous individuals and agencies.

Lao Ichong vs Jaime Hernandez


Treaties May Be Superseded by Municipal Laws in the Exercise of Police Power Lao Ichong is a Chinese businessman who entered the country to take advantage of business opportunities herein abound (then) particularly in the retail business. For some time he and his fellow Chinese businessmen enjoyed a monopoly in the local market in Pasay. Until in June 1954 when Congress passed the RA 1180 or the Retail Trade Nationalization Act the purpose of

which is to reserve to Filipinos the right to engage in the retail business. Ichong then petitioned for the nullification of the said Act on the ground that it contravened several treaties concluded by the RP which, according to him, violates the equal protection clause (pacta sund servanda). He said that as a Chinese businessman engaged in the business here in the country who helps in the income generation of the country he should be given equal opportunity. ISSUE: Whether or not a law may invalidate or supersede treaties or generally accepted principles. HELD: Yes, a law may supersede a treaty or a generally accepted principle. In this case, there is no conflict at all between the raised generally accepted principle and with RA 1180. The equal protection of the law clause does not demand absolute equality amongst residents; it merely requires that all persons shall be treated alike, under like circumstances and conditions both as to privileges conferred and liabilities enforced; and, that the equal protection clause is not infringed by legislation which applies only to those persons falling within a specified class, if it applies alike to all persons within such class, and reasonable grounds exist for making a distinction between those who fall within such class and those who do not. For the sake of argument, even if it would be assumed that a treaty would be in conflict with a statute then the statute must be upheld because it represented an exercise of the police power which, being inherent could not be bargained away or surrendered through the medium of a treaty. Hence, Ichong can no longer assert his right to operate his market stalls in the Pasay city market. Representatives Gerardo S. Espina, et al. vs. Hon. Ronaldo Zamora, Jr., et al. G.R. No. 143855, September 21, 2010. Constitutionality; Retail Trade Liberalization Act of 2000. The Court dismissed petitioners argument that Republic Act No. 8762, known as the Retail Trade Liberalization Act of 200, violates the mandate of the 1987 Constitution for the State to develop a self-reliant and independent national economy effectively controlled by Filipinos. The provisions of Article II of the 1987 Constitution, the declarations of principles and state policies, are not self-executing. Legislative failure to pursue such policies cannot give rise to a cause of action in the courts. Further, while Section 19, Article II of the 1987 Constitution requires the development of a selfreliant and independent national economy effectively controlled by Filipino entrepreneurs, it does not impose a policy of Filipino monopoly of the economic environment. The objective is simply to prohibit foreign powers or interests from maneuvering our economic policies and ensure that Filipinos are given preference in all areas of development. The 1987 Constitution takes into account the realities of the outside world as it requires the pursuit of a trade policy that serves the general welfare and utilizes all forms and arrangements of exchange on the basis of equality and reciprocity; and speaks of industries which are competitive in both domestic and foreign markets as well as of the protection of Filipino enterprises against unfair foreign competition and trade practices. Thus, while the Constitution mandates a bias in favor of Filipino goods, services, labor and enterprises, it also recognizes the need for business exchange with the rest of the world on the bases of equality and reciprocity and limits protection of Filipino enterprises only against foreign competition and trade practices that are unfair. In other

words, the 1987 Constitution does not rule out the entry of foreign investments, goods, and services. While it does not encourage their unlimited entry into the country, it does not prohibit them either. In fact, it allows an exchange on the basis of equality and reciprocity, frowning only on foreign competition that is unfair. The key, as in all economies in the world, is to strike a balance between protecting local businesses and allowing the entry of foreign investments and services. More important, Section 10, Article XII of the 1987 Constitution gives Congress the discretion to reserve to Filipinos certain areas of investments upon the recommendation of the National Economic and Development Authority and when the national interest requires. Thus, Congress can determine what policy to pass and when to pass it depending on the economic exigencies. It can enact laws allowing the entry of foreigners into certain industries not reserved by the Constitution to Filipino citizens. In this case, Congress has decided to open certain areas of the retail trade business to foreign investments instead of reserving them exclusively to Filipino citizens. The control and regulation of trade in the interest of the public welfare is of course an exercise of the police power of the State. A persons right to property, whether he is a Filipino citizen or foreign national, cannot be taken from him without due process of law. In 1954, Congress enacted the Retail Trade Nationalization Act (RA 1180) that restricts the retail business to Filipino citizens. In denying the petition assailing the validity of such Act for violation of the foreigners right to substantive due process of law, the Supreme Court held that the law constituted a valid exercise of police power. The State had an interest in preventing alien control of the retail trade and R.A. 1180 was reasonably related to that purpose. That law is not arbitrary. Here, to the extent that RA 8762 lessens the restraint on the foreigners right to property or to engage in an ordinarily lawful business, it cannot be said that the law amounts to a denial of the Filipinos right to property and to due process of law. Filipinos continue to have the right to engage in the kinds of retail business to which the law in question has permitted the entry of foreign investors. Certainly, it is not within the province of the Court to inquire into the wisdom of RA 8762 save when it blatantly violates the Constitution. But as the Court has said, there is no showing that the law has contravened any constitutional mandate. The Court is not convinced that the implementation of RA 8762 would eventually lead to alien control of the retail trade business. Petitioners have not mustered any concrete and strong argument to support its thesis. The law itself has provided strict safeguards on foreign participation in that business.

United States vs Luis Toribio


Police Power Sometime in the 1900s, Toribio applied for a license to have his carabao be slaughtered. His request was denied because his carabao is found not to be unfit for work. He nevertheless slaughtered his carabao without the necessary license. He was eventually sued and was sentenced by the trial court. His counsel in one way or the other argued that the law mandating that one should acquire a permit to slaughter his carabao is not a valid exercise of police power. ISSUE: Whether or not the said law is valid.

HELD: The SC ruled against Toribio. The SC explained that it is not a taking of the property for public use, within the meaning of the constitution, but is a just and legitimate exercise of the power of the legislature to regulate and restrain such particular use of the property as would be inconsistent with or injurious to the rights of the publics. All property is acquired and held under the tacit condition that it shall not be so used as to injure the equal rights of others or greatly impair the public rights and interests of the community.

TOMAS VELASCO vs ANTONIO VILLEGAS


This is an appeal from an order of the lower court dismissing a suit for declaratory relief challenging the constitutionality based on Ordinance No. 4964 of the City of Manila, the contention being that it amounts to a deprivation of property of petitioners-appellants of their means of livelihood without due process of law. The assailed ordinance is worded thus: It shall be prohibited for any operator of any barber shop to conduct the business of massaging customers or other persons in any adjacent room or rooms of said barber shop, or in any room or rooms within the same building where the barber shop is located as long as the operator of the barber shop and the room where massaging is conducted is the same person. As noted in the appealed order, petitioners-appellants admitted that criminal cases for the violation of this ordinance had been previously filed and decided. The lower court, therefore, held that a petition for declaratory relief did not lie, its availability being dependent on there being as yet no case involving such issue having been filed. Even if such were not the case, the attack against the validity cannot succeed. As pointed out in the brief of respondents-appellees, it is a police power measure. The objectives behind its enactment are: (1) To be able to impose payment of the license fee for engaging in the business of massage clinic under Ordinance No. 3659 as amended by Ordinance 4767, an entirely different measure than the ordinance regulating the business of barbershops and, (2) in order to forestall possible immorality which might grow out of the construction of separate rooms for massage of customers. This Court has been most liberal in sustaining ordinances based on the general welfare clause. As far back as U.S. v. Salaveria, a 1918 decision, this Court through Justice Malcolm made clear the significance and scope of such a clause, which delegates in statutory form the police power to a municipality. As above stated, this clause has been given wide application by municipal authorities and has in its relation to the particular circumstances of the case been liberally construed by the courts. Such, it is well to really is the progressive view of Philippine jurisprudence. As it was then, so it has continued to be. There is no showing, therefore, of the unconstitutionality of such ordinance.

Agustin vs Edu
Generally Accepted Principles of International Law Agustin is the owner of a Volkswagen Beetle Car. He is assailing the validity of Letter of Instruction No 229 which requires all motor vehicles to have early warning devices particularly to equip them with a pair of reflectorized triangular early warning devices. Agustin is arguing that this order is unconstitutional, harsh, cruel and unconscionable to the motoring public. Cars

are already equipped with blinking lights which is already enough to provide warning to other motorists. And that the mandate to compel motorists to buy a set of reflectorized early warning devices is redundant and would only make manufacturers and dealers instant millionaires. ISSUE: Whether or not the said is EO is valid. HELD: Such early warning device requirement is not an expensive redundancy, nor oppressive, for car owners whose cars are already equipped with 1) blinking-lights in the fore and aft of said motor vehicles, 2) battery-powered blinking lights inside motor vehicles, 3) built-in reflectorized tapes on front and rear bumpers of motor vehicles, or 4) well-lighted two (2) petroleum lamps (the Kinke) . . . because: Being universal among the signatory countries to the said 1968 Vienna Conventions, and visible even under adverse conditions at a distance of at least 400 meters, any motorist from this country or from any part of the world, who sees a reflectorized rectangular early warning device installed on the roads, highways or expressways, will conclude, without thinking, that somewhere along the travelled portion of that road, highway, or expressway, there is a motor vehicle which is stationary, stalled or disabled which obstructs or endangers passing traffic. On the other hand, a motorist who sees any of the aforementioned other built-in warning devices or the petroleum lamps will not immediately get adequate advance warning because he will still think what that blinking light is all about. Is it an emergency vehicle? Is it a law enforcement car? Is it an ambulance? Such confusion or uncertainty in the mind of the motorist will thus increase, rather than decrease, the danger of collision. The Letter of Instruction in question was issued in the exercise of the police power. That is conceded by petitioner and is the main reliance of respondents. It is the submission of the former, however, that while embraced in such a category, it has offended against the due process and equal protection safeguards of the Constitution, although the latter point was mentioned only in passing. The broad and expansive scope of the police power which was originally identified by Chief Justice Taney of the American Supreme Court in an 1847 decision, as nothing more or less than the powers of government inherent in every sovereignty was stressed in the aforementioned case of Edu v. Ericta thus: Justice Laurel, in the first leading decision after the Constitution came into force, Calalang v. Williams, identified police power with state authority to enact legislation that may interfere with personal liberty or property in order to promote the general welfare. Persons and property could thus be subjected to all kinds of restraints and burdens in order to secure the general comfort, health and prosperity of the state. Shortly after independence in 1948, Primicias v. Fugoso reiterated the doctrine, such a competence being referred to as the power to prescribe regulations to promote the health, morals, peace, education, good order or safety, and general welfare of the people. The concept was set forth in negative terms by Justice Malcolm in a pre-Commonwealth decision as that inherent and plenary power in the State which enables it to prohibit all things hurtful to the comfort, safety and welfare of society. In that sense it could be hardly distinguishable as noted by this Court in Morfe v. Mutuc with the totality of legislative power. It is in the above sense the greatest and most powerful attribute of government. It is, to quote Justice Malcolm anew, the most essential, insistent, and at least illimitable powers, extending as Justice Holmes aptly pointed out to all the great public needs. Its scope, ever expanding to meet the exigencies of the times, even to anticipate the future where it could be done, provides enough room for an efficient and flexible response to

conditions and circumstances thus assuring the greatest benefits. In the language of Justice Cardozo: Needs that were narrow or parochial in the past may be interwoven in the present with the well-being of the nation. What is critical or urgent changes with the time. The police power is thus a dynamic agency, suitably vague and far from precisely defined, rooted in the conception that men in organizing the state and imposing upon its government limitations to safeguard constitutional rights did not intend thereby to enable an individual citizen or a group of citizens to obstruct unreasonably the enactment of such salutary measures calculated to insure communal peace, safety, good order, and welfare. It was thus a heavy burden to be shouldered by petitioner, compounded by the fact that the particular police power measure challenged was clearly intended to promote public safety. It would be a rare occurrence indeed for this Court to invalidate a legislative or executive act of that character. None has been called to our attention, an indication of its being non-existent. The latest decision in point, Edu v. Ericta, sustained the validity of the Reflector Law, an enactment conceived with the same end in view. Calalang v. Williams found nothing objectionable in a statute, the purpose of which was: To promote safe transit upon, and avoid obstruction on roads and streets designated as national roads . . . As a matter of fact, the first law sought to be nullified after the effectivity of the 1935 Constitution, the National Defense Act, with petitioner failing in his quest, was likewise prompted by the imperative demands of public safety.
G.R. No. L-49112: Agustin vs Edu

Generally Accepted Principles of International Law Agustin is the owner of a Volkswagen Beetle Car. He is assailing the validity of Letter of Instruction No 229 which requires all motor vehicles to have early warning devices particularly to equip them with a pair of reflectorized triangular early warning devices. Agustin is arguing that this order is unconstitutional, harsh, cruel and unconscionable to the motoring public. Cars are already equipped with blinking lights which is already enough to provide warning to other motorists. And that the mandate to compel motorists to buy a set of reflectorized early warning devices is redundant and would only make manufacturers and dealers instant millionaires. ISSUE: Whether or not the said is EO is valid. HELD: Such early warning device requirement is not an expensive redundancy, nor oppressive, for car owners whose cars are already equipped with 1) 'blinking-lights in the fore and aft of said motor vehicles,' 2) 'battery-powered blinking lights inside motor vehicles,' 3) 'built-in reflectorized tapes on front and rear bumpers of motor vehicles,' or 4) 'well-lighted two (2) petroleum lamps (the Kinke) . . . because: Being universal among the signatory countries to the said 1968 Vienna Conventions, and visible even under adverse conditions at a distance of at least 400 meters, any motorist from this country or from any part of the world, who sees a reflectorized rectangular early warning device installed on the roads, highways or expressways, will conclude, without thinking, that somewhere along the travelled portion of that road, highway, or expressway, there is a motor vehicle which is stationary, stalled or disabled which obstructs or endangers passing traffic. On the other hand, a motorist who sees any of the aforementioned other built-in warning devices or the petroleum lamps will not immediately get

adequate advance warning because he will still think what that blinking light is all about. Is it an emergency vehicle? Is it a law enforcement car? Is it an ambulance? Such confusion or uncertainty in the mind of the motorist will thus increase, rather than decrease, the danger of collision.

Taxicab Operators vs Board of Transportation


Police Power

Petitioner Taxicab Operators of Metro Manila, Inc. (TOMMI) is a domestic corporation composed of taxicab operators, who are grantees of Certificates of Public Convenience to operate taxicabs within the City of Manila and to any other place in Luzon accessible to vehicular traffic. On October 10, 1977, respondent Board of Transportation (BOT) issued Memorandum Circular No. 77-42 which reads: SUBJECT: Phasing out and Replacement of Old and Dilapidated Taxis On January 27, 1981, petitioners filed a Petition with the BOT, docketed as Case No. 80-7553, seeking to nullify MC No. 77-42 or to stop its implementation; to allow the registration and operation in 1981 and subsequent years of taxicabs of model 1974, as well as those of earlier models which were phased-out, provided that, at the time of registration, they are roadworthy and fit for operation. ISSUE A. Did BOT and BLT promulgate the questioned memorandum circulars in accord with the manner required by Presidential Decree No. 101, thereby safeguarding the petitioners constitutional right to procedural due process? B. Granting arguendo, that respondents did comply with the procedural requirements imposed by Presidential Decree No. 101, would the implementation and enforcement of the assailed memorandum circulars violate the petitioners constitutional rights to. (1) Equal protection of the law; (2) Substantive due process; and (3) Protection against arbitrary and unreasonable classification and standard? HELD

As enunciated in the preambular clauses of the challenged BOT Circular, the overriding consideration is the safety and comfort of the riding public from the dangers posed by old and dilapidated taxis. The State, in the exercise of its police power, can prescribe regulations to promote the health, morals, peace, good order, safety and general welfare of the people. It can prohibit all things hurtful to comfort, safety and welfare of society. [5] It may also regulate property rights. [6] In the language of Chief Justice Enrique M. Fernando the necessities imposed by public welfare may justify the exercise of governmental authority to regulate even if thereby certain groups may plausibly assert that their interests are disregarded.

Taxicab Operators of Metro Manila Inc vs The Board of Transportation et al


Equal Protection Phasing Out of Old Taxis in MM but not Elsewhere On 10 Oct 1977, BOT issued Circ 77-42 which has for its purpose the phasing out of old and dilapidated taxis which are 6 years older. The law is set to be immediately implemented in Metro Manila first before it would be implemented elsewhere. Pursuant to this, the Director of the Bureau of Land Transportation issued Circ 52 which is the IRR of the law in the NCR. TOMMI assailed the constitutionality of the law. It avers, among other things, that the Circular in question violates their right to equal protection of the law because the same is being enforced in Metro Manila only and is directed solely towards the taxi industry. At the outset it should be pointed out that implementation outside Metro Manila is also envisioned in Memorandum Circular No. 77-42. ISSUE: Whether or not there is a violation of the equal protection clause by the implementation of the said circular. HELD: The SC held that Circ 77-42 is valid. BOTs reason for enforcing the Circular initially in Metro Manila is that taxicabs in this city, compared to those of other places, are subjected to heavier traffic pressure and more constant use. Thus is of common knowledge. Considering that traffic conditions are not the same in every city, a substantial distinction exists so that infringement of the equal protection clause can hardly be successfully claimed. In so far as the non-application of the assailed Circulars to other transportation services is concerned, it need only be recalled that the equal protection clause does not imply that the same treatment be accorded all and sundry. It applies to things or persons identically or similarly situated. It permits of classification of the object or subject of the law provided classification is reasonable or based on substantial distinction, which make for real differences, and that it must apply equally to each member of the class. What is required under the equal protection clause is the uniform operation by legal means so that all persons under identical or similar circumstance would be accorded the same treatment both in privilege conferred and the liabilities imposed. The challenged Circulars satisfy the foregoing criteria.

Bautista vs Juinio

Police Power The validity of an energy conservation measure, Letter of Instruction No. 869, issued on May 31, 1979 the response to the protracted oil crisis that dates back to 1974 is put in issue in this prohibition proceeding filed by petitioners, spouses Mary Concepcion Bautista and Enrique D. Bautista, for being allegedly violative of the due process and equal protection guarantees of the Constitution. The use of private motor vehicles with H and EH plates on week-ends and holidays was banned from [12:00] a.m. Saturday morning to 5:00 a.m. Monday morning, or 1:00 a.m. of the holiday to 5:00 a.m. of the day after the holiday. 2 Motor vehicles of the following classifications are exempted: (a) S (Service); (b) T (Truck); (c) DPL (Diplomatic); (d) CC (Consular Corps); (e) TC (Tourist Cars

This Court gave due course to the petition requiring respondent to answer. There was admission of the facts as substantially alleged except, as previously noted, that the ban starts at 12:00 a.m. rather than 1:00 a.m. of a Saturday or of a holiday and as to the mention of a Willys Kaiser jeep being registered in the name of a certain Teresita Urbina, about which respondents had no knowledge. There was a denial of the allegations that the classification of vehicles into heavy (H) and extra heavy (EH) on the other hand and light and bantam on the other hand was violative of equal protection and the regulation as to the use of the former cars on the dates specified a transgression of due process. The answer likewise denied that there was an undue delegation of legislative power, reference being made to the Land Transportation and Traffic Code. There was also a procedural objection raised, namely, that what is sought amounts at most to an advisory opinion rather than an adjudication of a case or controversy. It is true, of course, that there may be instances where a police power measure may, because of its arbitrary, oppressive or unjust character, be held offensive to the due process clause and, therefore, may, when challenged in an appropriate legal proceeding, be declared void on its face. This is not one of them. A recital of the whereas clauses of the Letter of Instruction makes it clear. Thus: [Whereas], developments in the international petroleum supply situation continue to follow a trend of limited production and spiralling prices thereby precluding the possibility of immediate relief in supplies within the foreseeable future; [Whereas], the uncertainty of fuel supply availability underscores a compelling need for the adoption of positive measures designed to insure the viability of the countrys economy and sustain its developmental growth; [Whereas], to cushion the effect of increasing oil prices and avoid fuel supply disruptions, it is imperative to adopt a program directed towards the judicious use of our energy resources complemented with intensified conservation efforts and efficient utilization thereof; . . .. What is undeniable is that the action taken is an appropriate response to a problem that presses urgently for solution. It may not be the only alternative, but its reasonableness is immediately apparent. Thus, to repeat, substantive due process, which is the epitome of reasonableness and fair play, is not ignored, much less infringed. In the interplay between such a fundamental right and police power, especially so where the assailed governmental action deals with the use of ones property, the latter is accorded much leeway. That is settled law. What is more, it is good law. Due process, therefore, cannot be

validly invoked. As stressed in the cited Ermita-Malate Hotel decision: To hold otherwise would be to unduly restrict and narrow the scope of police power which has been properly characterized as the most essential, insistent and the least limitable of powers, extending as it does to all the great public needs. It would be, to paraphrase another leading decision, to destroy the very purpose of the state if it could be deprived or allowed itself to be deprived of its competence to promote public health, public morals, public safety and the general welfare. Negatively put, police power is that inherent and plenary power in the State which enables it to prohibit all that is hurtful to the comfort, safety, and welfare of society.

Mary Concepcion Bautista et al vs Alfredo Juinio et al


Equal Protection Distinction Between Heavy and Extra Heavy Cars and Others Bautista is assailing the constitutionality of LOI 869 issued in 1979 which classified vehicles into Heavy and Extra heavy. The LOI further banned these vehicles during weekends and holidays that is from 5am Saturday until 5am Monday. Purpose of this law is to curb down petroleum consumption as bigger cars consume more oil. Bautista claimed the LOI to be discriminatory as it made an assumption that H and EH cars are Heavy on petroleum consumption when in fact there are smaller cars which are also big on oil consumption. Further, the law restricts their freedom to enjoy their car while others who have smaller cars may enjoy theirs. Bautista avers that there is no rational justification for the ban being imposed on vehicles classified as Heavy (H) and extra-Heavy (EH), for precisely those owned by them fall within such category. ISSUE: Whether or not the LOI violates equal protection. HELD: The SC held that Bautista was not able to make merit out of her contention. The classification on cars on its face cannot be characterized as an affront to reason. The ideal situation is for the laws benefits to be available to all, that none be placed outside the sphere of its coverage. Only thus could chance and favor be excluded and the affairs of men governed by that serene and impartial uniformity, which is of the very essence of the idea of law. The actual, given things as they are and likely to continue to be, cannot approximate the ideal. Nor is the law susceptible to the reproach that it does not take into account the realities of the situation. . . . To assure that the general welfare be promoted, which is the end of law, a regulatory measure may cut into the rights to liberty and property. Those adversely affected may under such circumstances invoke the equal protection clause only if they can show that the governmental act assailed, far from being inspired by the attainment of the common weal was prompted by the spirit of hostility, or at the very least, discrimination that finds no support in reason. It suffices then that the laws operate equally and uniformly on all persons under similar circumstances or that all persons must be treated in the same manner, the conditions not being different, both in the privileges conferred and the liabilities imposed. Favoritism and undue preference cannot be allowed. For the principle is that equal protection and security shall be given to every person under circumstances, which if not identical are analogous. If law be looked upon in terms of burden or charges, those that fall within a class should be treated in the same fashion, whatever restrictions cast on some in the group equally binding on the rest.
G.R. No. 78742: Association of Small Landowners vs Secretary of Agrarian Reform

Equal Protection

These are 3 cases consolidated questioning the constitutionalityof the Agrarian Reform Act. Article XIII on Social Justice and Human Rightsincludes a call for the adoption by the State of an agrarian reform program.The State shall, by law, undertake an agrarian reform program founded on theright of farmers and regular farmworkers, who are landless, to own directly orcollectively the lands they till or, in the case of other farmworkers, toreceive a just share of the fruits thereof. RA 3844, Agricultural Land ReformCode, had already been enacted by Congress on August 8, 1963. This wassubstantially superseded almost a decade later by PD 27, which was promulgatedon Oct 21, 1972, along with martial law, to provide for the compulsoryacquisition of private lands for distribution among tenant-farmers and tospecify maximum retention limits for landowners. On July 17, 1987, Cory issuedEO 228, declaring full land ownership in favor of the beneficiaries of PD 27and providing for the valuation of still unvalued lands covered by the decreeas well as the manner of their payment. This was followed on July 22, 1987 byPP 131, instituting a comprehensive agrarian reform program (CARP), and EO 229,providing the mechanics for its implementation. Afterwhich is the enactment ofRA 6657, Comprehensive Agrarian Reform Law of 1988, which Cory signed on June10. This law, while considerably changing the earlier mentioned enactments,nevertheless gives them suppletory effect insofar as they are not inconsistentwith its provisions. Inconsidering the rentals as advance payment on the land, the executive orderalso deprives the petitioners of their property rights as protected by dueprocess. The equal protection clause is also violated because the order placesthe burden of solving the agrarian problems on the owners only of agriculturallands. No similar obligation is imposed on the owners of other properties. Thepetitioners maintain that in declaring the beneficiaries under PD 27 to be theowners of the lands occupied by them, EO 228 ignored judicial prerogatives andso violated due process. Worse, the measure would not solve the agrarianproblem because even the small farmers are deprived of their lands and theretention rights guaranteed by the Constitution. In hiscomment the Sol-Gen asserted that the alleged violation of the equal protectionclause, the sugar planters have failed to show that they belong to a differentclass and should be differently treated. The Comment also suggests the possibilityof Congress first distributing public agricultural lands and scheduling theexpropriation of private agricultural lands later. From this viewpoint, thepetition for prohibition would be premature. ISSUE: Whether or notthere was a violation of the equal protection clause. HELD: The SC ruledaffirming the Sol-Gen. The argument of the small farmers that they have beendenied equal protection because of the absence of retention limits has alsobecome academic under Sec 6 of RA 6657. Significantly, they too have notquestioned the area of such limits.

There is also the complaint that theyshould not be made to share the burden of agrarian reform, an objection alsomade by the sugar planters on the ground that they belong to a particular classwith particular interests of their own. However, no evidence has been submittedto the Court that the requisites of a valid classification have been violated. Classification has been defined as the grouping of persons orthings similar to each other in certain particulars and different from eachother in these same particulars. To be valid, it must conform to the followingrequirements: (1) it must be based on substantial distinctions; (2) it must be germane to the purposes of the law; (3) it must not be limited to existing conditions only; and (4) it must apply equally to all the members of the class.

The Court finds that all these requisites have been met by themeasures here challenged as arbitrary and discriminatory. Equal protection simply means that all persons or thingssimilarly situated must be treated alike both as to the rights conferred andthe liabilities imposed. The petitioners have not shown that they belong to adifferent class and entitled to a different treatment. The argument that notonly landowners but also owners of other properties must be made to share theburden of implementing land reform must be rejected. There is a substantialdistinction between these two classes of owners that is clearly visible exceptto those who will not see. There is no need to elaborate on this matter. In anyevent, the Congress is allowed a wide leeway in providing for a validclassification. Its decision is accorded recognition and respect by the courtsof justice except only where its discretion is abused to the detriment of theBill of Rights.
DECS vs San Diego Facts: San Diego, private respondent graduate of UE (Zoology), took NMAT 3 times and flunked 3 times. The fourth time he will take NMAT, he was rejected by DECS and DCEM for the rule that: A student shall be allowed only three (3) chances to take the NMAT. After three (3) successive failures, a student shall not be allowed to take the NMAT for the fourth time. Ramon Guevarra then went to the Regional Trial Court of Valenzuela, Metro Manila, to compel his admission to the test. He invoked his constitutional rights to academic freedom and quality education. By agreement of the parties, herein defendant was allowed to take the NMAT scheduled on April 16, 1989, subject to the outcome of his petition.

In an amended petition filed with leave of court, San Diego squarely challenged the constitutionality of MECS Order No. 12, Series of 1972, containing the above-cited rule. The additional grounds raised were due process and equal protection. Respondent Judge Teresita Dizon-Capulong ruled that the MECS Order No. 12, Series of 1972 was invalid and held that the petitioner had been deprived of his right to pursue a medical education through an arbitrary exercise of the police power.

DECS VS. SAN DIEGO, 180 SCRA 533 FACTS: 1. Private respondent Roberto Rey San Diego graduated from the University of the East with a BS degree in Zoology. 2. He took the NMAT three times and flunked it as many times. 3. When he applied again petitioner rejected him because it contended that under the NMAT rule: a student shall be allowed to take 3 chances to take the NMAT. After three successive failures, a student shall not be allowed to take the NMAT for the fourth time. 4. Private respondent went to the RTC for a petition mandamus invoking his right to academic freedom and quality education. 5. Respondent Judge Teresita Dizon-Capulong declared the challenged order unconstitutional. 6. Respondent Judge held that San Diego has been deprived of his right to pursue a medical education through an arbitrary exercise of police power. ISSUE: Is person who has failed the NMAT three times entitled to take it again. HELD: The proper exercise of the police power of the State requires the concurrence of a lawful subject and a lawful method. The subject of the challenged regulation is certainly within the ambit of the police power. It is the right and indeed the responsibility of the State to insure that the medical profession is not infiltrated by incompetents to whom patients may unwarily entrust their lives and health. The method employed by the challenged regulation is not irrelevant to the purpose of the law nor is it arbitrary or oppressive. The thee-flunk rule is intended to insulate the medical schools and ultimately the medical profession from the intrusion of those not qualified to be doctors. While every person is entitled to aspire to be a doctor, he does not have a constitutional right to be a doctor. This is true of any other calling in which the public interest is involved, and the closer the line, the longer the bridge to one's ambition. The State has the responsibility to harness its human resources and to see to it that they are, not dissipated or, no less worse, not used at all. These resources must be applied in a manner that will best promote the common good while also giving the individual a sense of satisfaction.

The right to quality education is not absolute. The Constitution also provides that "every citizen has a right to choose a profession or course of study, subject to fair, reasonable and equitable

admission and academic requirements. {Art. XIV, Sec.5 (3)} The contention that the challenged rule violates the equal protection clause is not well taken. A law does not have to operate with equal force on all persons or things to be conformable to Art. III, Sec 1 of the Constitution. There would be unequal protection if some applicants who have passed the tests are admitted and others who have also qualified are denied entrance. In other words, what the equal protection requires is equality among equals. moral of the story: Try and try until you suceed, but if you fail deny you even tried.

VILLANUEVA, ET. AL. VS CASTAEDA, JR., ET. AL. G.R. No. L-61311 September 2l, 1987 (damnun absque injuria) Appeal from a decision of CFI Pampanga holding that the land in question, being public in nature, was beyond the commerce of man and therefore could not be the subject of private occupancy. CRUZ, J.: Facts: In the vicinity of the public market of San Fernando, Pampanga, there stands on a strip of land, a conglomeration of vendors stalls together. The petitioners claim they have a right to remain in and conduct business in this area by virtue of a previous authorization (Resolution no. 28) granted to them by the municipal government. The respondents deny this and justify the demolition of their stalls as illegal constructions on public property per municipal council Resolution G.R. No. 29, which declared the subject area as "the parking place and as the public plaza of the municipality, thereby impliedly revoking Resolution No. 218. Issue: WON petitioners have the right to occupy the subject land. Ruling: Petition Dismissed. It is a well-settled doctrine that the town plaza cannot be used for the construction of market stalls, and that such structures constitute a nuisance subject to abatement according to law. The petitioners had no right in the first place to occupy the disputed premises and cannot insist in remaining there now on the strength of their alleged lease contracts. Even assuming a valid lease of the property in dispute, the resolution could have effectively terminated the agreement for it is settled that the police power cannot be surrendered or bargained away through the medium of a contract. Hence, the loss or damage caused to petitioners, in the case at bar, does not constitute a violation of a legal right or amount to a legal wrong - damnum absque injuria. PRC vs. De Guzman Facts: The respondents are all graduates of the Fatima College of Medicine, Valenzuela City, Metro

Manila. They passed the Physician Licensure Examination conducted in February 1993 by the Board of Medicine (Board). Petitioner Professional Regulation Commission (PRC) then released their names as successful examinees in the medical licensure examination. Shortly thereafter, the Board observed that the grades of the seventy-nine successful examinees from Fatima College in the two most difficult subjects in the medical licensure exam, Biochemistry (Bio-Chem) and Obstetrics and Gynecology (OBGyne), were unusually and exceptionally high. Eleven Fatima examinees scored 100% in Bio-Chem and ten got 100% in OB-Gyne, another eleven got 99% in Bio-Chem, and twenty-one scored 99% in OB-Gyne. For its part, the NBI found that the questionable passing rate of Fatima examinees in the [1993] Physician Examination leads to the conclusion that the Fatima examinees gained early access to the test questions. Issue: Was the act pursuant to R.A. 2382 a valid exercise of police power Ruling: Yes, it is true that this Court has upheld the constitutional right of every citizen to select a profession or course of study subject to a fair, reasonable, and equitable admission and academic requirements. But like all rights and freedoms guaranteed by the Charter, their exercise may be so regulated pursuant to the police power of the State to safeguard health, morals, peace, education, order, safety, and general welfare of the people. Thus, persons who desire to engage in the learned professions requiring scientific or technical knowledge may be required to take an examination as a prerequisite to engaging in their chosen careers Chavez vs. COMELEC Fact: Petitioner Chavez, on various dates, entered into formal agreements with certain establishments to endorse their products. On August 18, 2003, he authorized a certain Andrew So to use his name and image for 96 North, a clothing company. Petitioner also signed Endorsement Agreements with Konka International Plastics Manufacturing Corporation and another corporation involved in the amusement and video games business, G-Box. These last two agreements were entered into on October 14, 2003 and November 10, 2003, respectively. Pursuant to these agreements, three billboards were set up along the Balintawak Interchange of the North Expressway. One billboard showed petitioner promoting the plastic products of Konka International Plastics Manufacturing Corporation, and the other two showed petitioner endorsing the clothes of 96 North. One more billboard was set up along Roxas Boulevard showing petitioner promoting the game and amusement parlors of G-Box. Issue: Is Section 32 of COMELEC Resolution No. 6520 an invalid exercise of police power? Ruling: No, Police power, as an inherent attribute of sovereign.

Didipio Earth Savers Multipurpose Association et al vs DENR Sec Elisea Gozun et al


Police Power Eminent Domain In 1987, Cory rolled out EO 279 w/c empowered DENR to stipulate with foreign companies when it comes to either technical or financial large scale exploration or mining. In 1995, Ramos signed into law RA 7942 or the Philippine Mining Act. In 1994, Ramos already signed an FTAA with Arimco Mining Co, an Australian company. The FTAA authorized AMC (later CAMC) to explore 37,000 ha of land in Quirino and N. Vizcaya including Brgy Didipio. After the passage

of the law, DENR rolled out its implementing RRs. Didipio petitioned to have the law and the RR to be annulled as it is unconstitutional and it constitutes unlawful taking of property. In seeking to nullify Rep. Act No. 7942 and its implementing rules DAO 96-40 as unconstitutional, petitioners set their sight on Section 76 of Rep. Act No. 7942 and Section 107 of DAO 96-40 which they claim allow the unlawful and unjust taking of private property for private purpose in contradiction with Section 9, Article III of the 1987 Constitution mandating that private property shall not be taken except for public use and the corresponding payment of just compensation. They assert that public respondent DENR, through the Mining Act and its Implementing Rules and Regulations, cannot, on its own, permit entry into a private property and allow taking of land without payment of just compensation. Traversing petitioners assertion, public respondents argue that Section 76 is not a taking provision but a valid exercise of the police power and by virtue of which, the state may prescribe regulations to promote the health, morals, peace, education, good order, safety and general welfare of the people. This government regulation involves the adjustment of rights for the public good and that this adjustment curtails some potential for the use or economic exploitation of private property. Public respondents concluded that to require compensation in all such circumstances would compel the government to regulate by purchase. ISSUE: Whether or not RA 7942 and the DENR RRs are valid. HELD: The SC ruled against Didipio. The SC noted the requisites of eminent domain. They are; (1) (2) (3) the expropriator must enter a private property; the entry must be for more than a momentary period. the entry must be under warrant or color of legal authority;

(4) the property must be devoted to public use or otherwise informally appropriated or injuriously affected; (5) the utilization of the property for public use must be in such a way as to oust the owner and deprive him of beneficial enjoyment of the property. In the case at bar, Didipio failed to show that the law is invalid. Indeed there is taking involved but it is not w/o just compensation. Sec 76 of RA 7942 provides for just compensation as well as section 107 of the DENR RR. To wit, Section 76. xxx Provided, that any damage to the property of the surface owner, occupant, or concessionaire as a consequence of such operations shall be properly compensated as may be provided for in the implementing rules and regulations. Section 107. Compensation of the Surface Owner and Occupant- Any damage done to the property of the surface owners, occupant, or concessionaire thereof as a consequence of the

mining operations or as a result of the construction or installation of the infrastructure mentioned in 104 above shall be properly and justly compensated. Further, mining is a public policy and the government can invoke eminent domain to exercise entry, acquisition and use of private lands.

City Government of QC vs Judge Ericta & Himlayang Pilipino


Police Power Not Validly Exercised Quezon City enacted an ordinance entitled ORDINANCE REGULATING THE ESTABLISHMENT, MAINTENANCE AND OPERATION OF PRIVATE MEMORIAL TYPE CEMETERY OR BURIAL GROUND WITHIN THE JURISDICTION OF QUEZON CITY AND PROVIDING PENALTIES FOR THE VIOLATION THEREOF The law basically provides that at least six (6) percent of the total area of the memorial park cemetery shall be set aside for charity burial of deceased persons who are paupers and have been residents of Quezon City for at least 5 years prior to their death, to be determined by competent City Authorities. QC justified the law by invoking police power. ISSUE: Whether or not the ordinance is valid. HELD: The SC held the law as an invalid exercise of police power. There is no reasonable relation between the setting aside of at least six (6) percent of the total area of all private cemeteries for charity burial grounds of deceased paupers and the promotion of health, morals, good order, safety, or the general welfare of the people. The ordinance is actually a taking without compensation of a certain area from a PRIVATE cemetery to benefit paupers who are charges of the municipal corporation. Instead of building or maintaining a public cemetery for this purpose, the city passes the burden to private cemeteries.
CITY GOVERNMENT OF QUEZON CITY VS. ERICTA [122 SCRA 759; G.R. No. L-34915; 24 Jun 1983]

Facts: Section 9 of Ordinance No. 6118, S-64, entitled "Ordinance Regulating The Establishment, Maintenance And Operation Of Private Memorial Type Cemetery Or Burial Ground Within The Jurisdiction Of Quezon City And Providing Penalties For The Violation Thereof" provides: Sec. 9. At least six (6) percent of the total area of the memorial park cemetery shall be set aside for charity burial of deceased persons who are paupers and have been residents of Quezon City for at least 5 years prior to their death, to be determined by competent City Authorities. The area so designated shall immediately be developed and should be open for operation not later than six months from the date of approval of the application. For several years, the aforequoted section of the Ordinance was not enforced but seven years after the enactment of the ordinance, the Quezon City Council passed a resolution to request the City Engineer, Quezon City, to stop

any further selling and/or transaction of memorial park lots in Quezon City where the owners thereof have failed to donate the required 6% space intended for paupers burial. The Quezon City Engineer then notified respondent Himlayang Pilipino, Inc. in writing that Section 9 of the ordinance would be enforced. Respondent Himlayang Pilipino reacted by filing a petition for declaratory relief, prohibition and mandamus with preliminary injunction seeking to annul Section 9 of the Ordinance in question. Respondent alleged that the same is contrary to the Constitution, the Quezon City Charter, the Local Autonomy Act, and the Revised Administrative Code.

Issue: Whether or Not Section 9 of the ordinance in question is a valid exercise of police power.

Held: Section 9 of the City ordinance in question is not a valid exercise of police power. Section 9 cannot be justified under the power granted to Quezon City to tax, fix the license fee, and regulate such other business, trades, and occupation as may be established or practiced in the City. Bill of rights states that 'no person shall be deprived of life, liberty or property without due process of law' (Art. Ill, Section 1 subparagraph 1, Constitution). On the other hand, there are three inherent powers of government by which the state interferes with the property rights, namely-. (1) police power, (2) eminent domain, (3) taxation. The police power of Quezon City is defined in sub-section 00, Sec. 12, Rep. Act 537 that reads as follows: To make such further ordinance and regulations not repugnant to law as may be necessary to carry into effect and discharge the powers and duties conferred by this act and such as it shall deem necessary and proper to provide for the health and safety, , and for the protection of property therein; and enforce obedience thereto with such lawful fines or penalties as the City Council may prescribe under the provisions of subsection (jj) of this section. The power to regulate does not include the power to prohibit. The power to regulate does not include the power to confiscate. The ordinance in question not only confiscates but also prohibits the operation of a memorial park cemetery, because under Section 13 of said ordinance, 'Violation of the provision thereof is punishable with a fine and/or imprisonment and that upon conviction thereof the permit to operate and maintain a private cemetery shall be revoked or cancelled. The confiscatory clause and the penal provision in effect deter one from operating a memorial park cemetery. Moreover, police power is defined by Freund as 'the power of promoting the public welfare by restraining and regulating the use of liberty and property'. It is usually exerted in order to merely regulate the use and enjoyment of property of the owner. If he is deprived of his property outright, it is not taken for public use but rather to destroy in order to promote the general welfare. It seems to the court that Section 9 of Ordinance No. 6118, Series of 1964 of Quezon City is not a mere police regulation but an outright confiscation. It deprives a person of his private property without due process of law, nay, even without compensation.

Restituto Ynot vs Intermediate Appellate Court


Police Power Not Validly Exercised There had been an existing law which prohibited the slaughtering of carabaos (EO 626). To strengthen the law, Marcos issued EO 626-A which not only banned the movement of carabaos from interprovinces but as well as the movement of carabeef. On 13 Jan 1984, Ynot was caught transporting 6 carabaos from Masbate to Iloilo. He was then charged in violation of EO 626-A. Ynot averred EO 626-A as unconstitutional for it violated his right to be heard or his right to due process. He said that the authority provided by EO 626-A to outrightly confiscate carabaos even without being heard is unconstitutional. The lower court ruled against Ynot ruling that the EO is a valid exercise of police power in order to promote general welfare so as to curb down the indiscriminate slaughter of carabaos. ISSUE: Whether or not the law is valid. HELD: The SC ruled that the EO is not valid as it indeed violates due process. EO 626-A ctreated a presumption based on the judgment of the executive. The movement of carabaos from one area to the other does not mean a subsequent slaughter of the same would ensue. Ynot should be given to defend himself and explain why the carabaos are being transferred before they can be confiscated. The SC found that the challenged measure is an invalid exercise of the police power because the method employed to conserve the carabaos is not reasonably necessary to the purpose of the law and, worse, is unduly oppressive. Due process is violated because the owner of the property confiscated is denied the right to be heard in his defense and is immediately condemned and punished. The conferment on the administrative authorities of the power to adjudge the guilt of the supposed offender is a clear encroachment on judicial functions and militates against the doctrine of separation of powers. There is, finally, also an invalid delegation of legislative powers to the officers mentioned therein who are granted unlimited discretion in the distribution of the properties arbitrarily taken.

Vicente De La Cruz vs Edgardo Paras


Subject Shall Be Expressed in the Title Police Power Not Validly Exercise De La Cruz et al were club & cabaret operators. They assail the constitutionality of Ord. No. 84, Ser. of 1975 or the Prohibition and Closure Ordinance of Bocaue, Bulacan. De la Cruz averred that the said Ordinance violates their right to engage in a lawful business for the said ordinance would close out their business. That the hospitality girls they employed are healthy and are not allowed to go out with customers. Judge Paras however lifted the TRO he earlier issued against Ord. 84 after due hearing declaring that Ord 84. is constitutional for it is pursuant to RA 938 which reads AN ACT GRANTING MUNICIPAL OR CITY BOARDS AND COUNCILS THE POWER TO REGULATE THE ESTABLISHMENT, MAINTENANCE AND OPERATION OF CERTAIN PLACES OF AMUSEMENT WITHIN THEIR RESPECTIVE TERRITORIAL JURISDICTIONS. Paras ruled that the prohibition is a valid exercise of police power to promote general welfare. De la Cruz then appealed citing that they were deprived of due process.

ISSUE: Whether or not a municipal corporation, Bocaue, Bulacan can, prohibit the exercise of a lawful trade, the operation of night clubs, and the pursuit of a lawful occupation, such clubs employing hostesses pursuant to Ord 84 which is further in pursuant to RA 938. HELD: The SC ruled against Paras. If night clubs were merely then regulated and not prohibited, certainly the assailed ordinance would pass the test of validity. SC had stressed reasonableness, consonant with the general powers and purposes of municipal corporations, as well as consistency with the laws or policy of the State. It cannot be said that such a sweeping exercise of a lawmaking power by Bocaue could qualify under the term reasonable. The objective of fostering public morals, a worthy and desirable end can be attained by a measure that does not encompass too wide a field. Certainly the ordinance on its face is characterized by overbreadth. The purpose sought to be achieved could have been attained by reasonable restrictions rather than by an absolute prohibition. Pursuant to the title of the Ordinance, Bocaue should and can only regulate not prohibit the business of cabarets.

El Banco ESPAOL-Filipino vs Vicente Palanca


Judicial Due Process Requisites

Engracio Palanca was indebted to El Banco and he had his parcel of land as security to his debt. His debt amounted to P218,294.10. His property is worth 75k more than what he owe. Due to the failure of Engracio to make his payments, El Banco executed an instrument to mortgage Engracios property. Engracio however left for China and he never returned til he died. Since Engracio is a non resident El Banco has to notify Engracio about their intent to sue him by means of publication using a newspaper. The lower court further orderdd the clerk of court to furnish Engracio a copy and that itd be sent to Amoy, China. The court eventually granted El Banco petition to execute Engracios property. 7 years thereafter, Vicente surfaced on behalf of Engracio as his administrator to petition for the annulment of the ruling. Vicente averred that there had been no due process as Engracio never received the summons. ISSUE: Whether or not due process was not observed. HELD: The SC ruled against Palanca. The SC ruled that the requisites for judicial due process had been met. The requisites are; 1. There must be an impartial court or tribunal clothed with judicial power to hear and decide the matter before it. 2. Jurisdiction must be lawfully acquired over the person of the defendant or over the property subject of the proceedings. 3. The defendant must be given the opportunity to be heard. 4. Judgment must be rendered only after lawful hearing.

Imelda Marcos vs Sandiganbayan


Due Process Imelda was charged together with Jose Dans for Graft & Corruption for a dubious transaction done in 1984 while they were officers transacting business with the Light Railway Transit. The case was raffled to the 1st Division of the Sandiganbayan. The division was headed by Justice Garchitorena with J Balajadia and J Atienza as associate justices. No decision was reached by the division by reason of Atienzas dissent in favor of Imeldas innocence. Garchitorena then summoned a special division of the SB to include JJ Amores and Cipriano as additional members. Amores then asked Garchitorena to be given 15 days to send in his manifestation. On the date of Amores request, Garchitorena received manifestation from J Balajadia stating that he agrees with J Rosario who further agrees with J Atienza. Garchitorena then issued a special order to immediately dissolve the special division and have the issue be raised to the SB en banc for it would already be pointless to wait for Amores manifestation granted that a majority has already decided on Imeldas favor. The SB en banc ruled against Imelda. ISSUE: Whether or not due process has been observed. HELD: The SC ruled that the ruling of the SB is bereft of merit as there was no strong showing of Imeldas guilt. The SC further emphasized that Imelda was deprived of due process by reason of Garchitorena not waiting for Amores manifestation. Such procedural flaws committed by respondent Sandiganbayan are fatal to the validity of its decision convicting petitioner. Garchitorena had already created the Special Division of five (5) justices in view of the lack of unanimity of the three (3) justices in the First Division. At that stage, petitioner had a vested right to be heard by the five (5) justices, especially the new justices in the persons of Justices Amores and del Rosario who may have a different view of the cases against her. At that point, Presiding Justice Garchitorena and Justice Balajadia may change their mind and agree with the original opinion of Justice Atienza but the turnaround cannot deprive petitioner of her vested right to the opinion of Justices Amores and del Rosario. It may be true that Justice del Rosario had already expressed his opinion during an informal, unscheduled meeting in the unnamed restaurant but as aforestated, that opinion is not the opinion contemplated by law. But what is more, petitioner was denied the opinion of Justice Amores for before it could be given, Presiding Justice Garchitorena dissolved the Special Division.
G.R. No. L-47276: Emma Delgado vs Court of Appeals

Due Process Delgado together with 3 others were charged for estafa causing the frustration of one medical student. Delgado was assisted by one Atty. Yco. The said lawyer has filed for multiple postponement of trial and one time he failed to appear in court by reason of him being allegedly sick. No medical certificate was furnished. The court was not impressed with such actuation and had considered the same as Delgados waiver of her right to trial. The lower court convicted her and the others. She appealed before the CA and the CA sustained the lower courts rule. Delgado later found out that Yco is not a member of the IBP.

ISSUE: Whether or not due process was observed. HELD: The SC ruled in favor of Delgado. An accused person is entitled to be represented by a member of the bar in a criminal case filed against her before the Regional Trial Court. Unless she is represented by a lawyer, there is great danger that any defense presented in her behalf will be inadequate considering the legal perquisites and skills needed in the court proceedings. This would certainly be a denial of due process.
G.R. No. 179907: Pedro Consulta vs People of the Philippines

Due Process Consulta is charged for stealing a gold necklace worth 3.5k owned by a certain Silvestre. He was convicted by the lower court. The court of appeals raised before the CA the issue that he was not properly arraigned and that he was represented by a non lawyer. ISSUE: Whether or not Consulta was denied of due process. HELD: The SC ruled that Consultas claim of being misrepresented cannot be given due course. He was assisted by two lawyers during the proceeding. In the earlier part, he was assisted by one Atty. Jocelyn Reyes who seemed not to be a lawyer. Granting that she indeed is not a lawyer, her withdrawal from the case in the earlier part of the case has cured the defect as he was subsequently assisted by a lawyer coming from the PAO. G.R. No. L-46272 June 13, 1986 PEOPLE OF THE PHILIPPINES, plaintiff-appellee, vs. ALBERTO OPIDA y QUIAMBAO and VIRGILIO MARCELO, accused-appellants.

CRUZ, J.: This is an automatic review of the Decision of the Circuit Criminal Court, Seventh Judicial District, imposing the death penalty upon Alberto Opida and Virgilio Marcelo for the crime of murder. Unlike the victim in this case, who died from only one stab wound, the decision under review suffers from several fatal flaws, all equally deadly. It suffices to discuss only one of them. Time and again this Court has declared that due process requires no less than the cold neutrality of an impartial judge. 1 Bolstering this requirement, we have added that the judge must not only be impartial but must also appear to be impartial, to give added assurance to the parties that his decision will be just. 2 The parties are entitled to no less

than this, as a minimum guaranty of due process. This guaranty was not observed in this case. On July 31, 1976, in Quezon City, several persons ganged up on Fabian Galvan, stoned and hit him with beer bottles until finally one of them stabbed him to death. The actual knife-wielder was identified as Mario del Mundo. 3 Nonetheless, Alberto Opida and Virgilio Marcelo were charged with murder as conspirators and, after trial, sentenced to death. 4 The basis of their conviction by the trial court was the testimony of two prosecution witnesses, neither of whom positively said that the accused were at the scene of the crime, their extrajudicial confessions, which were secured without the assistance of counsel, and corroboration of the alleged conspiracy under the theory of interlocking confession. 5 What is striking about this case is the way the trial judge conducted his interrogation of the two accused and their lone witness, Lilian Layug. It was hardly judicious and certainly far from judicial, at times irrelevant, at Worst malicious. Reading the transcript, one gathers the impression that the judge had allied himself with the prosecution to discredit at the outset the credibility of the witnesses for the defense. Opida is a police character, admittedly a member of the Commando gang and with a string of convictions for robbery, theft and vagrancy. 6 It is worth noting that the judge took special interest in his tattoos, required him to remove his shirt so they could be examined, and even described them in detail for the record. 7 Besides belaboring Opida's criminal activities and his tattoos, the judge asked him if he had "ever been convicted at the National Mental Hospital with what else but malice and suggested to him that his claim of manhandling by the police was a lie because investigators leave no mark when they torture a suspect. 8 This was a point that could have been validly raised by the prosecution but certainly not by the court. The judge also made it of record that the witness was gnashing his teeth, was showing signs of hostility, that he was uneasy and that he was restless. "Now, whom do you want to fool the judge asked, "the prosecutor, your lawyer, or the court? 9 In the hearing of September 22, 1976, the interrogation of Virgilio Marcelo, the other accused, was conducted almost wholly by the judge who started cross-examining the witness even before the defense counsel could ask his first question, and took over from the prosecution the task of impeaching Marcelo's credibility. 10 The judge asked him about his drug addiction, his membership in the Commando gang, his tattoos, his parentage, his activities, his criminal record all when he was supposed to be under direct examination by his own lawyer. Defense counsel could hardly put in a word edgewise because the judge kept interrupting to ask his own questions. 11 The questions were not clarificatory but adversary; and when they were not adversary, they were irrelevant, and sometimes also cruel. At one point, the judge drew from the

witness the statement that his mother was living with another man; forthwith he suggested that the mother was unfaithful to his father. 12 We deplore this sadistic treatment of the witness, especially as, for all his supposed "toughness," he could not answer back. We fail to see what possible connection the mother's infidelity could have had, by any stretch of the imagination, with the instant prosecution. But the judge was to save the best or worst of his spite for the third witness, Lilian Layug, a waitress in the restaurant where the appellant Opida was working as a cook. Noting at the outset that she spoke English, he wanted to know where she had learned it and asked in ill-concealed insinuation if she had worked in Angeles City or Olongapo or Sangley. 13 Because she was gesturing nervously, he asked, "Are you a conductor? 14 Of the two accused, he asked her, "They are very proud of belonging to the Commando gang to which the witness answered, putting him in his place, "That I do not know, Your Honor." 15 One cannot but note the mockery in the following questions put by the judge to the witness, who was probably wondering what the interrogation was all about
Court Q You are a very good friend of Alberto Opida? A Yes, Your Honor. Q You have known him for years? A One year only, Your Honor. Q He always feed you with his favorite menu? A Yes, Your Honor. Q He is a very good cook? A Yes, Your Honor. Q Because what he could cook, you could not cook? A I know also how to cook, Your Honor. Q Answer my question. A Yes, Your Honor. Q Whenever you try to cook what he cooked, you could not imitate it, because he is a good cook? A Yes, Your Honor. Q So, your admiration developed because of his cooking? A Yes, Your Honor. Q What favorite dish does he cook that you like, as far as you are concerned?

A Adobo, Your Honor. Q Most often you request him to cook adobo for you? A Yes, Your Honor. Q That is precisely one of the reasons why you also admire him? A That is also a part, Your Honor, Q Whenever you request him to cook adobo for you, he always accommodate you? A Yes, Your Honor. Q As a matter of fact, the moment that he starts cooking adobo, you could smell it already? A Yes, Your Honor, Q That starts your admiration for him. A Yes, Your Honor. Q And in return you reciprocate? A Yes, Your Honor. Q What kind of reciprocation do you give to Alberto Opida, whenever you admire his cooking of adobo for you, cooking just for you? A None, Your Honor. Q Whenever he cooks adobo, he was singing? A Sometimes, Your Honor. Q What kind of song? A He is singing a song with intended for Cora, Your Honor. Q And you were also affected by it? A No, Your Honor. Q You mean to say, you are not very fond of emotional songs? A I am not, because Cora is not minding him, Your Honor. Q But sometimes he sings in the absence of Cora because, as you said, he is cooking adobo for you? A Yes, Your Honor. Q What does he sings (sic) for you? A He sings many songs, Your Honor. Q For example, give the title

A Milagro, Your Honor. Q He also sings Diyos Lamang Ang Nakakaalam? A Sometimes, Your Honor. Q He also sings Kapantay ay Langit? A Yes, Your Honor. Q He also sings Sapagkat Tayo'y Tao Lamang? A I did not hear, Your Honor. Q But, you said he also sings even in the absence of Cora? A Yes, Your Honor. Q You smell adobo while he cooks and sings. So, you developed admiration also? A Little only, Your Honor. Q One way or another you have appreciated him, but the only thing, as you know, he is related to Cora in the same way? A Yes, Your Honor. Q That is why you are testifying in his favor? Because of the smell of adobo and his songs and it is an admiration. Therefore, there is that tendency to testify in his favor? A Yes, Your Honor. 16

On direct examination, Opida challenged his extrajudicial confession, claiming it had been obtained without observance of the rights available under Article IV, Section 20 of the Constitution, particularly the right to counsel. 17 Parenthetically, the extrajudicial confession of Marcelo was also made without assistance of counsel. 18 Opida also testified, under questioning from his counsel, that he had been repeatedly hit with a "dos por dos" by a police officer while he was being investigated. 19 We have consistently held that the rights guaranteed during a custodial investigation are not supposed to be merely communicated to the suspect, especially if he is unlettered, but must be painstakingly explained to him so he can understand their nature and significance. Moreover, manhandling of any sort will vitiate any extrajudicial confession that may be extracted from him and renders it inadmissible in evidence against him. 20 Those principles were given mere lip service by the judge, who did not bother to look deeper into the validity of the challenged confessions. Given the obvious hostility of the judge toward the defense, it was inevitable that all the protestations of the accused in this respect would be, as they in fact were, dismissed. And once the confessions were admitted, it was easy enough to employ them as corroborating evidence of the claimed conspiracy among the accused.

The accused are admittedly notorious criminals who were probably even proud of their membership in the Commando gang even as they flaunted their tattoos as a badge of notoriety. 21 Nevertheless, they were entitled to be presumed innocent until the contrary was proved and had a right not to be held to answer for a criminal offense without due process of law. 22 The judge disregarded these guarantees and was in fact all too eager to convict the accused, who had manifestly earned his enmity. When he said at the conclusion of the trial, "You want me to dictate the decision now?" 23, he was betraying a pre-judgment long before made and obviously waiting only to be formalized. The scales of justice must hang equal and, in fact, should even be tipped in favor of the accused because of the constitutional presumption of innocence. Needless to stress, this right is available to every accused, whatever his present circumstance and no matter how dark and repellent his past. Despite their sinister connotations in our society, tattoos are at best dubious adornments only and surely not under our laws indicia of criminality. Of bad taste perhaps, but not of crime. In any event, convictions are based not on the mere appearance of the accused but on his actual commission of crime, to be ascertained with the pure objectivity of the true judge who must uphold the law for all without favor or malice and always with justice. Accused-appellants Opida and Marcelo, who have been imprisoned since 1976, have sent us separate letters pleading for the resolution of their death sentences one way or the other once and for all. Considering the way they were tried, we now declare that they should not be detained in jail a minute longer. While this is not to say that the accused are not guilty, it does mean that, because their constitutional rights have been violated, their guilt, if it exists, has not been established beyond reasonable doubt and so cannot be pronounced. Due process has stayed the uneven hand of the quick condemnor and must set the defendants free. WHEREFORE, the conviction of Alberto Opida and Virgilio Marcelo is reversed and they are hereby ordered released immediately. No costs. SO ORDERED. Abad Santos, Yap, Narvasa, Melencio-Herrera, Alampay, Gutierrez, Jr. and Paras, JJ., concur. Feria and Fernan JJ., are on leave.

Separate Opinions

TEEHANKEE, C.J., concurring: I concur. I wish to state that some of us are not persuaded at all that the two herein accused should be held guilty of the single stab wound inflicted on the victim in what appears to have been a tumultuous affray. I hail the Court's ratio decidendi that prescinding therefrom, the accused's guilt, if it exists in reality, cannot be pronounced because of the violation of their basic constitutional rights of due process and of the constitutional provision outlawing uncounselled confessions. In my dissenting opinion in the habeas corpus case of Dr. Aurora Parong, 1 wrote that "the Court stands as the guarantor of the constitutional and human rights of all persons within its jurisdiction and must see to it that the rights are respected and enforced. It is settled in this jurisdiction that once a deprivation of a constitutional right is shown to exist, the court that rendered the judgment or before whom the case is pending is ousted of jurisdiction and habeas corpus is the appropriate remedy to assail the legality of the detention. 2 So accused persons deprived of the constitutional right of speedy trial have been set free. 3 And likewise persons detained indefinitely without charges so much so that the detention becomes punitive and not merely preventive in character are entitled to regain their freedom. The spirit and letter of our Constitution negates as contrary to the basic precepts of human rights and freedom that a person be detained indefinitely without any charges." I had stressed in another case that the plain mandate of the constitutional provision expressly adopted the exclusionary rule as the only practical means of enforcing the constitutional injunction against uncounselled confessions obtained in violation of one's constitutional rights by outlawing their admission in court. The outlawing of such confessions thereby removed the incentive on the part of military or police officers to disregard such basic constitutional rights, in the same manner that the exclusionary rule bars admission of illegally seized evidence. 4 This fundamental rule that the court that rendered the judgment or before whom the case is pending is ousted of jurisdiction upon showing of deprivation of a basic constitutional right was eroded during the past authoritarian regime. I hail its vigorous restatement in the ponencia of Mr. Justice Isagani A. Cruz.

People vs Mortera (Right Of The Accused To An Impartial Trial)

FACTS: This is an appeal from the January 23, 2009 Decision of the Court of Appeals which affirmed with modification the Decision of the Regional Trial Court in criminal case which found accused Benancio Mortera guilty beyond reasonable doubt of the crime of murder for the killing of one Robelyn Rojas. Prosecution witness Ramil Gregorio testified that one afternoon, he together with other men were drinking tuba. They have just started drinking when Benancio Mortera, Jr. arrived. He wanted to

hit Alberto Rojas with a Nescafe glass. Alberto Rojas ran away. Mortera said, "Sayang." He listened while the group of Ramil Gregorio were singing accompanied by a guitar. Jomer Diaz, brother-in-law of Alberto Diaz, arrived. Mortera said, "Here comes another Rojas." Gregorio and his companions told Jomer Diaz to run away. Mortera hurled a stone at Diaz but the latter was not hit. Mortera left but he said that he will return. After a few minutes, Mortera came back. When Jomer Diaz ran, Robelyn Rojas, brother of Alberto Rojas went to Jomer. Mortera met Robelyn at a distance of about seven meters from the place where the group were drinking. Mortera and Robelyn discussed with each other and later shook hands. Robelyn turned his face and Mortera suddenly stabbed Robelyn Rojas at the back. After stabbing Robelyn, Mortera ran away. Robelyn Rojas tried to chase Mortera but he was not able to catch up but he fell down mortally wounded. He was brought to the hospital by his brother but he was pronounced DOA at the hospital. Jovel Veales who was drinking together with Ramil Gregorio and others, corroborated Ramil Gregorio's testimony. Although the accused pleaded not guilty when arraigned, during the trial, he admitted having stabbed the victim whom he referred to as Tonying, but claimed self-defense. By his account he passed by a corner and saw a group of people drinking. They were Ramil Gregorio, Jonel Veales and Tonying. Upon seeing him, Tonying ran away and called his brother, Alberto Rojas. When the accused was about to reach the main road, Alberto Rojas, Tonying and a certain "Duk" (brother-in-law of Tonying) accosted him and asked him for liquor money. When he refused, the three men got angry. After telling them that he had to go, Tonying hit him with a spray gun (for painting), causing him to fall down. While he was in a supine position, Tonying attempted to hit him again. It was at that point that he was able to get hold of his knife and thrust it forward and hit someone. He did not know who got stabbed. He then immediately fled. On January 23, 2007, the RTC rendered judgment finding the accused guilty of murder. In rejecting the claim of self-defense, the trial court stated that it was not worthy of belief as it was belied by the credible testimonies of the prosecution witnesses. The accused appealed to the CA raising the issues of denial of due process of law and his right to an impartial trial. He claimed that the trial court judge, Judge Jesus Carbon, was hostile towards him and prejudged his guilt as could be inferred from his "prosecutor-like" conduct. The accused likewise reiterated his claim of self-defense. In its decision, the CA affirmed the decision of the RTC with modification as to the civil liabilities. The CA ruled that the trial judge did not transgress the standard of "cold neutrality" required of a magistrate and added that the questions he propounded were "substantially clarificatory." Still not satisfied, the accused now comes before the SC. ISSUE: WON the accused were denied of his right to have an impartial trial. HELD: As correctly pointed out by the CA, although the trial judge might have made improper remarks and comments, it did not amount to a denial of his right to due process or his right to an impartial trial. Upon perusal of the transcript as a whole, it cannot be said that the remarks were reflective of his partiality. Not only did the accused mislead the court by initially invoking a negative defense only to claim otherwise during trial, he was also not candid to his own lawyer, who was kept in the dark as to his intended defense. The invocation of Opida did not persuade the SC. In Opida, SC did not fail to notice the "malicious," "sadistic" and "adversarial" manner of questioning by the trial judge of the accused therein, including their defense witness. In Opida,

the accused never admitted the commission of the crime, and so the burden of proof remained with the prosecution.

G.R. No. 167139

February 25, 2010

SUSIE CHAN-TAN, Petitioner, vs. JESSE C. TAN, Respondent. DECISION CARPIO, J.: The Case This is a petition for review1 of (i) the 17 May 2004 Resolution2 amending the 30 March 2004 Decision3 and (ii) the 15 February 2005 Resolution4 of the Regional Trial Court of Quezon City, Branch 107, in Civil Case No. Q-01-45743. In its 30 March 2004 Decision, the trial court declared the marriage between petitioner Susie Chan-Tan and respondent Jesse Tan void under Article 36 of the Family Code. Incorporated as part of the decision was the 31 July 2003 Partial Judgment5 approving the Compromise Agreement6 of the parties. In its 17 May 2004 Resolution, the trial court granted to respondent custody of the children, ordered petitioner to turn over to respondent documents and titles in the latter's name, and allowed respondent to stay in the family dwelling. In its 15 February 2005 Resolution, the trial court denied petitioner's motion for reconsideration of the 28 December 2004 Resolution7 denying petitioner's motion to dismiss and motion for reconsideration of the 12 October 2004 Resolution,8 which in turn denied for late filing petitioner's motion for reconsideration of the 17 May 2004 resolution. The Facts Petitioner and respondent were married in June of 1989 at Manila Cathedral in Intramuros, Manila.9 They were blessed with two sons: Justin, who was born in Canada in 1990 and Russel, who was born in the Philippines in 1993.10 In 2001, twelve years into the marriage, petitioner filed a case for the annulment of the marriage under Article 36 of the Family Code. The parties submitted to the court a compromise agreement, which we quote in full: 1. The herein parties mutually agreed that the two (2) lots located at Corinthian Hills, Quezon City and more particularly described in the Contract to Sell, marked in open court as Exhibits "H" to "H-3" shall be considered as part of the presumptive legitimes of their two (2) minor children namely, Justin Tan born on October 12, 1990 and Russel Tan born on November 28, 1993. Copies of the Contract to Sell are hereto attached as Annexes "A" and "B" and made integral parts hereof.

2. Susie Tan hereby voluntarily agrees to exclusively shoulder and pay out of her own funds/assets whatever is the remaining balance or unpaid amounts on said lots mentioned in paragraph 1 hereof directly with Megaworld Properties, Inc., until the whole purchase or contract amounts are fully paid. Susie Tan is hereby authorized and empowered to directly negotiate, transact, pay and deal with the seller/developer Megaworld Properties, Inc., in connection with the Contract to Sell marked as Annexes "A" and "B" hereof. The property covered by CCT No. 3754 of the Registry of Deeds of Quezon City and located at Unit O, Richmore Town Homes 12-B Mariposa St., Quezon City shall be placed in co-ownership under the name of Susie Tan (1/3), Justin Tan (1/3) and Russel Tan (1/3) to the exclusion of Jesse Tan. The property covered by TCT No. 48137 of the Registry of Deeds of Quezon City and located at View Master Town Homes, 1387 Quezon Avenue, Quezon City shall be exclusively owned by Jesse Tan to the exclusion of Susie Tan. The undivided interest in the Condominium Unit in Cityland Shaw. Jesse Tan shall exclusively own blvd. to the exclusion of Susie Tan. The shares of stocks, bank accounts and other properties presently under the respective names of Jesse Tan and Susie Tan shall be exclusively owned by the spouse whose name appears as the registered/account owner or holder in the corporate records/stock transfer books, passbooks and/or the one in possession thereof, including the dividends/fruits thereof, to the exclusion of the other spouse. Otherwise stated, all shares, bank accounts and properties registered and under the name and/or in the possession of Jesse Tan shall be exclusively owned by him only and all shares, accounts and properties registered and/or in the possession and under the name of Susie Tan shall be exclusively owned by her only. However, as to the family corporations of Susie Tan, Jesse Tan shall execute any and all documents transferring the shares of stocks registered in his name in favor of Susie Tan, or Justin Tan/Russel Tan. A copy of the list of the corporation owned by the family of Susie Tan is hereto attached as Annex "C" and made an integral part hereof. The parties shall voluntarily and without need of demand turn over to the other spouse any and all original documents, papers, titles, contracts registered in the name of the other spouse that are in their respective possessions and/or safekeeping. 3. Thereafter and upon approval of this Compromise Agreement by the Honorable Court, the existing property regime of the spouses shall be dissolved and shall now be governed by "Complete Separation of Property". Parties expressly represent that there are no known creditors that will be prejudiced by the present compromise agreement.

The parties shall have joint custody of their minor children. However, the two (2) minor children shall stay with their mother, Susie Tan at 12-B Mariposa St., Quezon City. The husband, Jesse Tan, shall have the right to bring out the two (2) children every Sunday of each month from 8:00 AM to 9:00 PM. The minor children shall be returned to 12-B Mariposa Street, Quezon City on or before 9:00 PM of every Sunday of each month. The husband shall also have the right to pick up the two (2) minor children in school/or in the house every Thursday of each month. The husband shall ensure that the children be home by 8:00 PM of said Thursdays. During the summer vacation/semestral break or Christmas vacation of the children, the parties shall discuss the proper arrangement to be made regarding the stay of the children with Jesse Tan. Neither party shall put any obstacle in the way of the maintenance of the love and affection between the children and the other party, or in the way of a reasonable and proper companionship between them, either by influencing the children against the other, or otherwise; nor shall they do anything to estrange any of them from the other. The parties agreed to observe civility, courteousness and politeness in dealing with each other and shall not insult, malign or commit discourteous acts against each other and shall endeavor to cause their other relatives to act similarly. 4. Likewise, the husband shall have the right to bring out and see the children on the following additional dates, provided that the same will not impede or disrupt their academic schedule in Xavier School, the dates are as follows: a. Birthday of Jesse Tan b. Birthday of Grandfather and Grandmother, first cousins and uncles and aunties c. Father's Day d. Death Anniversaries of immediate members of the family of Jesse Tan e. During the Christmas seasons/vacation the herein parties will agree on such dates as when the children can stay with their father. Provided that if the children stay with their father on Christmas Day from December 24th to December 25th until 1:00 PM the children will stay with their mother on December 31 until January 1, 1:00 PM, or vice versa. The husband shall always be notified of all school activities of the children and shall see to it that he will exert his best effort to attend the same.

5. During the birthdays of the two (2) minor children, the parties shall as far as practicable have one celebration. Provided that if the same is not possible, the Husband (Jesse Tan) shall have the right to see and bring out the children for at least four (4) hours during the day or the day immediately following/or after the birthday, if said visit or birthday coincides with the school day. 6. The existing Educational Plans of the two children shall be used and utilized for their High School and College education, in the event that the Educational Plans are insufficient to cover their tuition, the Husband shall shoulder the tuition and other miscellaneous fees, costs of books and educational materials, uniform, school bags, shoes and similar expenses like summer workshops which are taken in Xavier School, which will be paid directly by Jesse Tan to the children's school when the same fall due. Jesse Tan, if necessary, shall pay tutorial expenses, directly to the tutor concerned. The husband further undertake to pay P10,000.00/monthly support pendente lite to be deposited in the ATM Account of SUSIE CHAN with account no. 3-189-53867-8 Boni Serrano Branch effective on the 15th of each month. In addition Jesse Tan undertakes to give directly to his two (2) sons every Sunday, the amount needed and necessary for the purpose of the daily meals of the two (2) children in school. 7. This Compromise Agreement is not against the law, customs, public policy, public order and good morals. Parties hereby voluntarily agree and bind themselves to execute and sign any and all documents to give effect to this Compromise Agreement.11 On 31 July 2003, the trial court issued a partial judgment12 approving the compromise agreement. On 30 March 2004, the trial court rendered a decision declaring the marriage void under Article 36 of the Family Code on the ground of mutual psychological incapacity of the parties. The trial court incorporated in its decision the compromise agreement of the parties on the issues of support, custody, visitation of the children, and property relations. Meanwhile, petitioner cancelled the offer to purchase the Corinthian Hills Subdivision Lot No. 12, Block 2. She authorized Megaworld Corp. to allocate the amount of P11,992,968.32 so far paid on the said lot in the following manner: (a) P3,656,250.04 shall be transferred to fully pay the other lot in Corinthian Hills on Lot 11, Block 2; (b) P7,783,297.56 shall be transferred to fully pay the contract price in Unit 9H of the 8 Wack Wack Road Condominium project; and (c) P533,420.72 shall be forfeited in favor of Megaworld Corp. to cover the marketing and administrative costs of Corinthian Hills Subdivision Lot 12, Block 2.13

Petitioner authorized Megaworld Corp. to offer Lot 12, Block 2 of Corinthian Hills to other interested buyers. It also appears from the records that petitioner left the country bringing the children with her. Respondent filed an omnibus motion seeking in the main custody of the children. The evidence presented by respondent established that petitioner brought the children out of the country without his knowledge and without prior authority of the trial court; petitioner failed to pay the P8,000,000 remaining balance for the Megaworld property which, if forfeited would prejudice the interest of the children; and petitioner failed to turn over to respondent documents and titles in the latter's name.1avvphi1 Thus, the trial court, in its 17 May 2004 resolution, awarded to respondent custody of the children, ordered petitioner to turn over to respondent documents and titles in the latter's name, and allowed respondent to stay in the family dwelling in Mariposa, Quezon City. Petitioner filed on 28 June 2004 a motion for reconsideration14 alleging denial of due process on account of accident, mistake, or excusable negligence. She alleged she was not able to present evidence because of the negligence of her counsel and her own fear for her life and the future of the children. She claimed she was forced to leave the country, together with her children, due to the alleged beating she received from respondent and the pernicious effects of the latter's supposed gambling and womanizing ways. She prayed for an increase in respondent's monthly support obligation in the amount of P150,000. Unconvinced, the trial court, in its 12 October 2004 Resolution,15 denied petitioner's motion for reconsideration, which was filed beyond the 15-day reglementary period. It also declared petitioner in contempt of court for non-compliance with the partial judgment and the 17 May 2004 resolution. The trial court also denied petitioner's prayer for increase in monthly support. The trial court reasoned that since petitioner took it upon herself to enroll the children in another school without respondent's knowledge, she should therefore defray the resulting increase in their expenses. On 4 November 2004, petitioner filed a motion to dismiss16 and a motion for reconsideration17 of the 12 October 2004 Resolution. She claimed she was no longer interested in the suit. Petitioner stated that the circumstances in her life had led her to the conclusion that withdrawing the petition was for the best interest of the children. She prayed that an order be issued vacating all prior orders and leaving the parties at the status quo ante the filing of the suit. In its 28 December 2004 Resolution,18 the trial court denied both the motion to dismiss and the motion for reconsideration filed by petitioner. It held that the 30 March 2004 decision and the 17 May 2004 resolution had become final and executory upon the lapse of the 15-day reglementary period without any timely appeal having been filed by either party. Undeterred, petitioner filed a motion for reconsideration of the 28 December 2004 resolution, which the trial court denied in its 15 February 2005 resolution.19 The trial court then issued a Certificate of Finality20 of the 30 March 2004 decision and the 17 May 2004 resolution.

The Trial Court's Rulings The 30 March 2004 Decision21 declared the marriage between the parties void under Article 36 of the Family Code on the ground of mutual psychological incapacity. It incorporated the 31 July 2003 Partial Judgment22 approving the Compromise Agreement23 between the parties. The 17 May 2004 Resolution24 amended the earlier partial judgment in granting to respondent custody of the children, ordering petitioner to turn over to respondent documents and titles in the latter's name, and allowing respondent to stay in the family dwelling in Mariposa, Quezon City. The 15 February 2005 Resolution25 denied petitioner's motion for reconsideration of the 28 December 2004 Resolution26 denying petitioner's motion to dismiss and motion for reconsideration of the 12 October 2004 Resolution,27 which in turn denied for late filing petitioner's motion for reconsideration of the 17 May 2004 resolution. The Issue Petitioner raises the question of whether the 30 March 2004 decision and the 17 May 2004 resolution of the trial court have attained finality despite the alleged denial of due process. The Court's Ruling The petition has no merit. Petitioner contends she was denied due process when her counsel failed to file pleadings and appear at the hearings for respondent's omnibus motion to amend the partial judgment as regards the custody of the children and the properties in her possession. Petitioner claims the trial court issued the 17 May 2004 resolution relying solely on the testimony of respondent. Petitioner further claims the trial court erred in applying to her motion to dismiss Section 7 of the Rule on the Declaration of Absolute Nullity of Void Marriages and Annulment of Voidable Marriages. Petitioner argues that if indeed the provision is applicable, the same is unconstitutional for setting an obstacle to the preservation of the family. Respondent maintains that the 30 March 2004 decision and the 17 May 2004 resolution of the trial court are now final and executory and could no longer be reviewed, modified, or vacated. Respondent alleges petitioner is making a mockery of our justice system in disregarding our lawful processes. Respondent stresses neither petitioner nor her counsel appeared in court at the hearings on respondent's omnibus motion or on petitioner's motion to dismiss. The issue raised in this petition has been settled in the case of Tuason v. Court of Appeals.28 In Tuason, private respondent therein filed a petition for the annulment of her marriage on the ground of her husband's psychological incapacity. There, the trial court rendered judgment declaring the nullity of the marriage and awarding custody of the children to private respondent therein. No timely appeal was taken from the trial court's judgment. We held that the decision annulling the marriage had already become final and executory when the husband failed to appeal during the reglementary period. The husband claimed that the decision of the trial court was null and void for violation of his right to due process. He argued

he was denied due process when, after failing to appear on two scheduled hearings, the trial court deemed him to have waived his right to present evidence and rendered judgment based solely on the evidence presented by private respondent. We upheld the judgment of nullity of the marriage even if it was based solely on evidence presented by therein private respondent. We also ruled in Tuason that notice sent to the counsel of record is binding upon the client and the neglect or failure of the counsel to inform the client of an adverse judgment resulting in the loss of the latter's right to appeal is not a ground for setting aside a judgment valid and regular on its face.29 In the present case, the 30 March 2004 decision and the 17 May 2004 resolution of the trial court had become final and executory upon the lapse of the reglementary period to appeal.30 Petitioner's motion for reconsideration of the 17 May 2004 resolution, which the trial court received on 28 June 2004, was clearly filed out of time. Applying the doctrine laid down in Tuason, the alleged negligence of counsel resulting in petitioner's loss of the right to appeal is not a ground for vacating the trial court's judgments. Further, petitioner cannot claim that she was denied due process. While she may have lost her right to present evidence due to the supposed negligence of her counsel, she cannot say she was denied her day in court. Records show petitioner, through counsel, actively participated in the proceedings below, filing motion after motion. Contrary to petitioner's allegation of negligence of her counsel, we have reason to believe the negligence in pursuing the case was on petitioner's end, as may be gleaned from her counsel's manifestation dated 3 May 2004: Undersigned Counsel, who appeared for petitioner, in the nullity proceedings, respectfully informs the Honorable Court that she has not heard from petitioner since Holy Week. Attempts to call petitioner have failed. Undersigned counsel regrets therefore that she is unable to respond in an intelligent manner to the Motion (Omnibus Motion) filed by respondent.31 Clearly, despite her counsel's efforts to reach her, petitioner showed utter disinterest in the hearings on respondent's omnibus motion seeking, among others, custody of the children. The trial judge was left with no other recourse but to proceed with the hearings and rule on the motion based on the evidence presented by respondent. Petitioner cannot now come to this Court crying denial of due process. As for the applicability to petitioner's motion to dismiss of Section 7 of the Rule on the Declaration of Absolute Nullity of Void Marriages and Annulment of Voidable Marriages, petitioner is correct. Section 7 of the Rule on the Declaration of Absolute Nullity of Void Marriages and Annulment of Voidable Marriages provides: SEC. 7. Motion to dismiss. - No motion to dismiss the petition shall be allowed except on the ground of lack of jurisdiction over the subject matter or over the parties; provided, however, that any other ground that might warrant a dismissal of the case may be raised as an affirmative defense in an answer. (Emphasis supplied)

The clear intent of the provision is to allow the respondent to ventilate all possible defenses in an answer, instead of a mere motion to dismiss, so that judgment may be made on the merits. In construing a statute, the purpose or object of the law is an important factor to be considered.32 Further, the letter of the law admits of no other interpretation but that the provision applies only to a respondent, not a petitioner. Only a respondent in a petition for the declaration of absolute nullity of void marriage or the annulment of voidable marriage files an answer where any ground that may warrant a dismissal may be raised as an affirmative defense pursuant to the provision. The only logical conclusion is that Section 7 of the Rule does not apply to a motion to dismiss filed by the party who initiated the petition for the declaration of absolute nullity of void marriage or the annulment of voidable marriage. Since petitioner is not the respondent in the petition for the annulment of the marriage, Section 7 of the Rule does not apply to the motion to dismiss filed by her. Section 7 of the Rule not being applicable, petitioner's claim that it is unconstitutional for allegedly setting an obstacle to the preservation of the family is without basis. Section 1 of the Rule states that the Rules of Court applies suppletorily to a petition for the declaration of absolute nullity of void marriage or the annulment of voidable marriage. In this connection, Rule 17 of the Rules of Court allows dismissal of the action upon notice or upon motion of the plaintiff, to wit: Section 1. Dismissal upon notice by plaintiff. - A complaint may be dismissed by the plaintiff by filing a notice of dismissal at any time before service of the answer or of a motion for summary judgment. Upon such notice being filed, the court shall issue an order confirming the dismissal. x xx Section 2. Dismissal upon motion of plaintiff. - Except as provided in the preceding section, a complaint shall not be dismissed at the plaintiff's instance save upon approval of the court and upon such terms and conditions as the court deems proper. x x x (Emphasis supplied) However, when petitioner filed the motion to dismiss on 4 November 2004, the 30 March 2004 decision and the 17 May 2004 resolution of the trial court had long become final and executory upon the lapse of the 15-day reglementary period without any timely appeal having been filed by either party. The 30 March 2004 decision and the 17 May 2004 resolution may no longer be disturbed on account of the belated motion to dismiss filed by petitioner. The trial court was correct in denying petitioner's motion to dismiss. Nothing is more settled in law than that when a judgment becomes final and executory, it becomes immutable and unalterable. The same may no longer be modified in any respect, even if the modification is meant to correct what is perceived to be an erroneous conclusion of fact or law.33 The reason is grounded on the fundamental considerations of public policy and sound practice that, at the risk of occasional error, the judgments or orders of courts must be final at some definite date fixed by law. Once a judgment has become final and executory, the issues there should be laid to rest.34 WHEREFORE, we DENY the petition for review. We AFFIRM the (i) 17 May 2004 Resolution amending the 30 March 2004 Decision and (ii) the 15 February 2005 Resolution of the Regional Trial Court of Quezon City, Branch 107, in Civil Case No. Q-01-45743.

Costs against petitioner. SO ORDERED. A.M. No. MTJ-09-1729 January 20, 2009 (Formerly OCA I.P.I. No. 07-1910-MTJ) NORYN S. TAN, Petitioner, vs. JUDGE MARIA CLARITA CASUGA-TABIN, Municipal Trial Court in Cities, Branch 4, Baguio City, Respondent. RESOLUTION AUSTRIA-MARTINEZ, J.: Noryn S. Tan (complainant) filed a Complaint dated April 2, 2007 against Judge Maria Clarita Casuga-Tabin (respondent) of the Municipal Trial Court in Cities (MTCC), Branch 4, Baguio City for denial of due process relative to Criminal Case No. 118628. Complainant avers: On November 9, 2006, the Philippine National Police (PNP) Quezon City Police District (QCPD) served her a warrant of arrest dated October 13, 2006, issued by the MTCC Baguio City, Branch 4, presided by respondent, relative to Criminal Case No. 118628 for alleged violation of Batas Pambansa Blg. 22. It was only then that she learned for the first time that a criminal case was filed against her before the court. She was detained at the Quezon City Hall Complex Police Office and had to post bail of P1,000.00 before the Office of the Executive Judge of the Regional Trial Court (RTC) of Quezon City for her temporary release. Upon verification, she learned that respondent issued on August 8, 2006 an Order directing her to appear before the court on October 10, 2006 for arraignment. It was sent by mail to PNP Quezon City for service to her. However, she did not receive any copy of the Order and up to the present has not seen the same; hence, she was not able to attend her arraignment. She also found out that there was no proof of service of the Order or any notice to her of the arraignment. This notwithstanding, respondent issued a warrant for her arrest. Complainant alleges that she was deeply aggrieved and embarrassed by the issuance of the warrant for her arrest despite the fact that she was never notified of her arraignment. Complainant prayed that the appropriate investigation be conducted as to the undue issuance of a warrant for her arrest.1 In her Comment2 dated July 5, 2007, respondent answered: She issued the warrant of arrest because when the case was called for appearance, the complainant, as accused therein, failed to appear. Prior to the issuance of the warrant of arrest, her staff sent by registered mail the court's Order dated August 8, 2006 addressed to complainant "through the Chief of Police, PNP, 1104, Quezon City" directing complainant to appear on October 10, 2006 at 8:30 a.m. for the arraignment and preliminary conference in Criminal Case No. 118628, as proven by Registry Receipt No. 0310. It is true that the return on the court's Order dated August 8, 2006 had not yet been made by the QC Police on or before October 10, 2006. Nonetheless, she issued the warrant of arrest in good faith and upon the following grounds: (a) under Sec. 3 of Rule 1313 of the Rules

of Court, the court was entitled to presume that on October 10, 2006, after the lapse of a little over two months, official duty had been regularly performed and a letter duly directed and mailed had been received in the regular course of mail; and (b) Sec. 12 of the 1983 Rule on Summary Procedure in Special Cases provides that bail may be required where the accused does not reside in the place where the violation of the law or ordinance was committed. The warrant of arrest she issued was meant to implement this provision, which was not repealed by the 1991 Revised Rule on Summary Procedure, since complainant is a resident of Quezon City and not of Baguio City. If her interpretation was erroneous, she (respondent) believes that an administrative sanction for such error would be harsh and unsympathetic. She has nothing personal against complainant and did not want to embarrass or humiliate her. She issued the warrant in the honest belief that her act was in compliance with the rules. She prays that the case against her be dismissed and that a ruling on the interpretation of Secs. 10 & 12, of the 1983 Rule on Summary Procedure in Special Cases, in relation to Sec. 16 of the 1991 Revised Rule on Summary Procedure be made for the guidance of the bench and bar.4 The OCA, in its agenda report dated September 28, 2007, recommended that the case be dismissed for lack of merit. It held: Prior to the filing of the information, a preliminary investigation was conducted by the provincial prosecutor resulting in the Resolution dated July 11, 2006 recommending the filing of the case; it was incredulous for complainant to claim that she came to learn for the first time of the filing of the criminal case when the warrant of arrest was served on her; furthermore, there was already a complete service of notice as contemplated in Sec. 10, Rule 135 of the Rules of Court; hence the requirement of notice was fully satisfied by the service of the Order dated August 8, 2006 and the completion of the service thereof.6 Adopting the recommendation of the OCA, the Court on November 12, 2007 issued a Resolution dismissing the case for lack of merit.7 Complainant filed a Motion for Reconsideration dated January 8, 2008 alleging: The issue in this case was not whether complainant was aware of the criminal complaint against her, but whether the issuance of a warrant of arrest against her despite the absence of notice should be administratively dealt with; complainant was never notified of the arraignment; thus, she was not able to attend the same; respondent admitted in her Comment that no return had yet been made on or before October 10, 2006, the date respondent ordered the warrant to be issued; her explanation of good faith was therefore unjustifiable; neither could respondent invoke the presumption of regularity of performance of official duty, since the complainant did not actually receive any notice; respondent in an Order dated March 14, 2007 admitted that since she did not usually wear eyeglasses during hearings, she thought that the acknowledgment receipt at the back of the Order referred to the copy sent to complainant; later scrutiny, however, showed that it pertained to the one sent to the prosecutor's office; Section 10, Rule 13 of the Rules of Court did not apply to the instant case; the Order was addressed and sent to PNP Quezon City; assuming that the Order was properly served on the PNP, it was not equivalent to a service on complainant; there was no actual delivery of the Order to the complainant; hence, there was no personal service; neither was it served by ordinary mail or by registered mail; thus, the rule on completeness of service had not been satisfied; complainant was not aware of and therefore did not attend the preliminary investigation of her case; no proof can be shown that she was ever notified of the said preliminary investigation, much less of the filing of the same.8

In a Resolution dated April 16, 2008, the Court required respondent to Comment on complainant's Motion for Reconsideration.9 Complainant filed a Comment stating: Complainant's motion did not raise any new issue or ground that would merit the reconsideration of the Court's November 12, 2007 Resolution; complainant failed to rebut the presumption that she was notified of the scheduled arraignment; what complainant propounded was a mere self-serving denial that she never received the subpoena intended for her; there was no explanation why she would be able to receive a warrant of arrest; which was coursed in the same manner as the subpoena, in a little less than a month, but allegedly to receive the subpoena in almost two months; if complainant's assertion was to be believed, the effect would be to paralyze the operation of courts in the provinces that had to inevitably rely on the police resources of Metro Manila; arraignments could not proceed and trials could not go on; it was reasonable to follow as a rule that once a pleading or any other official document was received in the ordinary course of sending them, it must be presumed that others of the same nature were also delivered to the named addressees; to believe otherwise would be to delay justice for those residing outside Metro Manila.10 The Court finds the Motion for Reconsideration to be impressed with merit. Whenever a criminal case falls under the Summary Procedure, the general rule is that the court shall not order the arrest of the accused, unless the accused fails to appear whenever required.11 This is clearly provided in Section 16 of the 1991 Revised Rule on Summary Procedure which states: Sec. 16. Arrest of accused. - The court shall not order the arrest of the accused except for failure to appear whenever required. Release of the person arrested shall either be in bail or on recognizance by a responsible citizen acceptable to the court. (Emphasis supplied) In this case, respondent claims that the issuance of a warrant for the arrest of complainant was justified, since complainant failed to appear during the arraignment in spite of an order requiring her to do so. Respondent admits, however, that a copy of the Order dated August 8, 2006, was sent to complainant "through the Chief of Police, PNP, 1104, Quezon City." While it is true that the Rules of Court provides for presumptions, one of which is that official duty has been regularly performed, such presumption should not be the sole basis of a magistrate in concluding that a person called to court has failed to appear as required, which in turn justifies the issuance of a warrant for her arrest, when such notice was not actually addressed to her residence but to the police in her city. So basic and fundamental is a person's right to liberty that it should not be taken lightly or brushed aside with the presumption that the police through which the notice had been sent, actually served the same on complainant whose address was not even specified. Respondent further admitted in her Comment dated July 5, 2007 that when she proceeded with the arraignment on October 10, 2006 as scheduled, no return had yet been made by the Quezon City Police.12 Nevertheless, she issued the warrant of arrest, arguing that she did so on the presumption that regular duty had been performed, and that the Order had been received in the

regular course of mail; and since Sec. 12 of the 1983 Rules on Summary Procedure provides that bail may be required where the accused does not reside in the place where the violation of the law or ordinance was committed, the warrant of arrest she issued was justified since complainant is a resident of Quezon City and not of Baguio City. The Court disagrees. Sections 10 and 12 of the 1983 Rules on Summary Procedure in Special Cases (As Amended) state: Sec. 10. Duty of the Court. - On the basis of the complaint of information and the affidavits accompanying the same, the court shall make a preliminary determination whether to dismiss the case outright for being patently without basis or merit, or to require further proceedings to be taken. In the latter case, the court may set the case for immediate arraignment of an accused under custody, and if he pleads guilty, may render judgment forthwith. If he pleads not guilty, and in all other cases, the court shall issue an order, accompanied by copies of all the affidavits submitted by the complainant, directing the defendant(s) to appear and submit his counteraffidavit and those of his witnesses at a specified date not later than ten (10) days from receipt thereof. Failure on the part of the defendant to appear whenever required, shall cause the issuance of a warrant for his arrest if the court shall find that a probable cause exists after an examination in writing and under oath or affirmation of the complainant and his witnesses. (Emphasis supplied) xxxx Sec. 12. Bail not required; Exception. --- No bail shall be required except when a warrant of arrest is issued in accordance with Section 10 hereon or where the accused (a) is a recidivist; (b) is fugitive from justice; (c) is charged with physical injuries; (d) does not reside in the place where the violation of the law or ordinance was committed, or (e) has no known residence. Section 12 of the 1983 Rules on Summary Procedure was not reproduced in the 1991 Revised Rules on Summary Procedure, while Section 10 was revised and portions thereof reproduced in Sections 1213 and 16 of the 1991 Rules on Summary Procedure. Granting, arguendo, that Sections 10 and 12 of the 1983 Rules on Summary Procedure in Special Cases were not repealed by the 1991 Revised Rules, still it does not justify the warrant of arrest issued in this case. Section 12 talks of instances when bails are required, one of which is when the accused does not reside in the place where the violation of the law or ordinance was committed. It does not state, however, that a warrant of arrest shall immediately issue even without actual notice to the accused. Respondent's interpretation ascribes to the rules those which were not expressly stated therein and unduly expands their meaning. The Court also notes that in an Order dated March 14, 2007, a copy of which was attached by complainant to her Motion for Reconsideration, respondent admitted that:

As a point of clarification, during the hearing on October 10, 2006, when the case was called and the accused failed to appear, the Court verified from the staff if the Accused was notified to which said staff answered in the affirmative, showing to the Court a copy of the Order dated August 8, 2006, setting this case for Appearance of the Accused on October 10, 2006. At the back of the Order was an attached Acknowledgment Receipt. A quick glance of the said receipt, and without eyeglasses of the Presiding Judge, as she does not usually wear one during Court sessions, made this Court believed that indeed, that was the Acknowledgment Receipt proving that the Accused was served with a copy of the said Order. The attention of the Court was called upon receipt of the Accused's Motion for Clarification and a closer look on the Acknowledgment Receipt shows that the same was for the City Prosecutor's Office. x x x14 (Emphasis supplied) From this, it can be inferred that respondent issued the warrant of arrest on the mistaken belief that complainant was actually notified of the arraignment. A closer scrutiny of the records however showed that the Acknowledgment Receipt pertained to the copy of the City Prosecutor's Office and not that of complainant's. Whatever the real reasons behind respondent's issuance of complainant's warrant of arrest -whether from the mistaken belief that complainant was actually notified, or the presumption that the police had served a copy of the order on complainant or that the rules allow immediate issuance of warrants of arrests whenever the accused does not reside in the locality where the crime was committed -- the fact is, respondent failed to uphold the rules, for which she should be held administratively liable. The Court has held that a judge commits grave abuse of authority when she hastily issues a warrant of arrest against the accused in violation of the summary procedure rule that the accused should first be notified of the charges against him and given the opportunity to file his counteraffidavits and countervailing evidence.15 While judges may not always be subjected to disciplinary action for every erroneous order or decision they render, that relative immunity is not a license to be negligent, abusive and arbitrary in their prerogatives. If judges wantonly misuse the powers vested in them by law, there will not only be confusion in the administration of justice but also oppressive disregard of the basic requirements of due process.16 While there appears to be no malicious intent on the part of respondent, such lack of intent, however, cannot completely free her from liability.17 When the law is sufficiently basic, a judge owes it to her office to know and simply apply it.18 Considering that this is respondent's first administrative infraction in her more than 8 years of service in the judiciary,19 which serves to mitigate her liability, the Court holds the imposition of a fine in the amount of P10,000.00 to be proper in this case.20 WHEREFORE, Judge Maria Clarita Casuga-Tabin, Municipal Trial Court in Cities, Branch 4, Baguio City is hereby found guilty of abuse of authority for which she is fined in the sum of P10,000.00.

SO ORDERED.

Sec of Justice vs Judge Ralph Lantion


Due Process

Mark Jimenez was charged of multiple crimes ranging from tax evasion to wire tapping to conspiracy to defraud the USA. Jimenez was then wanted in the US. The US government, pursuant to the RP-US extradition treaty requested to have Jimenez be extradited there. Jimenez requested for a copy of the complaint against him as well as the extradition request by the USA. The DOJ sec refused to provide him copy thereof advising that it is still premature to give him so and that it is not a preliminary investigation hence he is not entitled to receive such copies. Jimenez sued the DOJ Sec and the lower court ruled in favor of Jimenez. ISSUE: Whether or not Jimenez is deprived of due process. HELD: The SC affirmed the ruling of the lower court. The case against Jimenez refer to an impending threat of deprivation of ones property or property right. No less is this true, but even more so in the case before us, involving as it does the possible deprivation of liberty, which, based on the hierarchy of constitutionally protected rights, is placed second only to life itself and enjoys precedence over property, for while forfeited property can be returned or replaced, the time spent in incarceration is irretrievable and beyond recompense.

Development of the Bank of the Philippines vs Court of Appeals


Due Process Opportunity to be Heard In 1968 and 1969, Continental Cement Corp. entered into a loan contract with DBP. In 1979, CCC entered into a MOA with DBP restructuring its loans. In November 1985, DBP filed for a foreclosure against the assets of CCC. In December 1985, CCC petitioned before RTC Bulacan to enjoin DBP and the Sheriff of Bulacan from foreclosing its assets and praying further that its loan terms with DBP be restructured and that the interest rate terms in the promissory note be declared null and void. A TRO was issued in favor of CCC. In December 1986, PP 502 was issued transferring nonperforming assets of the govt to Asset Privatization Trust. One of those transferred was CCCs account. DBP filed a petition to dismiss the pending case as it CCC could no longer deal with DBP but rather with APT. The trial court denied the petition and has instead allowed APT to join the proceeding pursuant to PP 502 as amended. To determine CCCs indebtedness to DBP/APT, the RTC designated JC Laya (former BSP Gov and DepEd Sec) as chair of a fact finding commission. He was given 60 days to come up with a report and he was given a lot of extensions thereafter. After several months, he was able to come up with the report. The parties then filed their reactions to the report and during the trial they were given a chance to cross examine each others witnesses. After cross examination, they were ordered to submit their

position papers as to their calculation of the amount of indebtedness. CCCs computation is at P43.6M, the Commissioners computation is at P61.6M while DBP/APTs calculation is at P2.6B. In June 1992, 3 of CCCs witnesses were scheduled to be cross examined by APTs counsel as DBPs counsel had already done so. APTs counsel was not able to do so raising the issue that he just took over the case and needs time to prepare. The cross examination was reset to August 24-26, 1992 but counsel for APT failed to appear due to Dengue. The other counsel, Jaime Cruz, for DBP was likewise absent; hes also a witness. On Aug 25th, the RTC ordered that due to the foregoing the case is deemed submitted for decision. APT filed for a motion for reconsideration. It was denied and the RTC ruled that the indebtedness to be paid by CCC is the calculation came up with by the Commissioner. APT appealed before the CA averring that it was denied due process when it was not allowed to cross examine the witnesses of CCC nor was it allowed to present further witnesses. CCC averred that by the failure of APTs counsel to appear APT has waived such right. The CA sustained the RTCs decision. ISSUE: Whether or not APT was denied of due process. HELD: The SC sustained the CAs ruling. Long ingrained in jurisprudence is the principle that there can be no denial of due process where a party had the opportunity to participate in the proceedings but did not do so. The withdrawal of APTs previous counsel in the thick of the proceedings would be a reasonable ground to seek postponement of the hearing. However, such reason necessitates a duty, nay an obligation, on the part of the new counsel to prepare himself for the next scheduled hearing. The excuse that it was due to the former counsels failure to turn over the records of the case to APT, shows the negligence of the new counsel to actively recover the records of the case. Mere demands are not sufficient. Counsel should have taken adequate steps to fully protect the interest of his client, rather than pass the blame on the previous counsel. The due process requirement is satisfied where the parties are given the opportunity to submit position papers, as in this case. Both parties, CCC and DBP/APT, were given opportunity to submit their respective position papers after the Commissioner rendered his report. Contained in their position papers were their respective comments and objections to the said report. Furthermore, the parties were also given the chance to cross-examine the Commissioner and his representative. They were likewise granted opportunity to cross-examine the witnesses of the other party, however, like in APTs case, they were deemed to have waived their right, as previously discussed. The essence of due process is that a party be afforded a reasonable opportunity to be heard and to support any evidence he may have in support of his defense. What the law prohibits is absolute absence of the opportunity to be heard, hence, a party cannot feign denial of due process when he had been afforded the opportunity to present his side.

Matuguina Integrated Wood Products Inc. vs Court of Appeals


Due Process Not Being Party to a Case

In 1973, license was issued to Milagros Matuguina to operate logging businesses under her group Matuguina Logging Enterprises. MIWPI was established in 1974 with 7 stockholders. Milagros Matuguina became the majority stockholder later on. Milagros later petitioned to have MLE be transferred to MIWPI. Pending approval of MLEs petition, Davao Enterprises Corporation filed a complaint against MLE before the District Forester (Davao) alleging that MLE has encroached upon the area allotted for DAVENCORs timber concession. The Investigating Committee found MLE guilty as charged and had recommended the Director to declare that MLE has done so. MLE appealed the case to the Ministry of Natural Resources. During pendency, Milagrosa withdrew her shares from MIWPI. Later, MNR Minister Ernesto Maceda found MLE guilty as charged. Pursuant to the finding, DAVENCOR and Philip Co requested Maceda to order MLE and/or MIWPI to comply with the ruling to pay the value in pesos of 2352.04 m3 worth of timbers. The Minister then issued a writ of execution against MIWPI. MIWPI filed a petition for prohibition before the Davao RTC. The RTC ruled in favor of MIWPI and has ordered to enjoin the Minister from pursuing the execution of the writ. DAVENCOR appealed and the CA reversed the ruling of the RTC. MIWPI averred that it is not a party to the original case (as it was MLE that was sued a separate entity). That the issuance of the order of execution by the Minister has been made not only without or in excess of his authority but that the same was issued patently without any factual or legal basis, hence, a gross violation of MIWPIs constitutional rights under the due process clause. ISSUE: Whether or not MIWPIs right to due process has been violated. HELD: The SC ruled in favor of MIWPI. Generally accepted is the principle that no man shall be affected by any proceeding to which he is a stranger, and strangers to a case not bound by judgment rendered by the court. In the same manner an execution can be issued only against a party and not against one who did not have his day in court. There is no basis for the issuance of the Order of Execution against the MIWPI. The same was issued without giving MIWPI an opportunity to defend itself and oppose the request of DAVENCOR for the issuance of a writ of execution against it. In fact, it does not appear that MIWPI was at all furnished with a copy of DAVENCORs letter requesting for the Execution of the Ministers decision against it. MIWPI was suddenly made liable upon the order of execution by the respondent Secretarys expedient conclusions that MLE and MIWPI are one and the same, apparently on the basis merely of DAVENCORs letter requesting for the Order, and without hearing or impleading MIWPI. Until the issuance of the Order of execution, MIWPI was not included or mentioned in the proceedings as having any participation in the encroachment in DAVENCORs timber concession. This action of the Minister disregards the most basic tenets of due process and elementary fairness. The liberal atmosphere which pervades the procedure in administrative proceedings does not empower the presiding officer to make conclusions of fact before hearing all the parties concerned. (1996 Oct 24) PEOPLE VS CA, 262 SCRA 452

Evelio Javier vs COMELEC & Arturo Pacificador


Due Process impartial and competent court

Javier and Pacificador, a member of the KBL under Marcos, were rivals to be members of the Batasan in May 1984 in Antique. During election, Javier complained of massive terrorism, intimidation, duress, vote-buying, fraud, tampering and falsification of election returns under duress, threat and intimidation, snatching of ballot boxes perpetrated by the armed men of Pacificador. COMELEC just referred the complaints to the AFP. On the same complaint, the 2nd Division of the Commission on Elections directed the provincial board of canvassers of Antique to proceed with the canvass but to suspend the proclamation of the winning candidate until further orders. On June 7, 1984, the same 2nd Division ordered the board to immediately convene and to proclaim the winner without prejudice to the outcome of the case before the Commission. On certiorari before the SC, the proclamation made by the board of canvassers was set aside as premature, having been made before the lapse of the 5-day period of appeal, which the Javier had seasonably made. Javier pointed out that the irregularities of the election must first be resolved before proclaiming a winner. Further, Opinion, one of the Commissioners should inhibit himself as he was a former law partner of Pacificador. Also, the proclamation was made by only the 2nd Division but the Constitute requires that it be proclaimed by the COMELEC en banc. In Feb 1986, during pendency, Javier was gunned down. The Solicitor General then moved to have the petition close it being moot and academic by virtue of Javiers death. ISSUE: Whether or not there had been due process in the proclamation of Pacificador. HELD: The SC ruled in favor of Javier and has overruled the Sol-Gens tenor. The SC has repeatedly and consistently demanded the cold neutrality of an impartial judge as the indispensable imperative of due process. To bolster that requirement, we have held that the judge must not only be impartial but must also appear to be impartial as an added assurance to the parties that his decision will be just. The litigants are entitled to no less than that. They should be sure that when their rights are violated they can go to a judge who shall give them justice. They must trust the judge, otherwise they will not go to him at all. They must believe in his sense of fairness, otherwise they will not seek his judgment. Without such confidence, there would be no point in invoking his action for the justice they expect. Due process is intended to insure that confidence by requiring compliance with what Justice Frankfurter calls the rudiments of fair play. Fair play calls for equal justice. There cannot be equal justice where a suitor approaches a court already committed to the other party and with a judgment already made and waiting only to be formalized after the litigants shall have undergone the charade of a formal hearing. Judicial (and also extrajudicial) proceedings are not orchestrated plays in which the parties are supposed to make the motions and reach the denouement according to a prepared script. There is no writer to foreordain the ending. The judge will reach his conclusions only after all the evidence is in and all the arguments are filed, on the basis of the established facts and the pertinent law.

Pedro Azul vs Judge Jose Castro & Rosalinda Tecson


Due Process Impartial and Competent Court Azul owns and operates a construction shop. To finance it he entered a loan agreement with Tecson in the amount of P391k. Tecson was only able to collect P141k thus leaving about P250k

as a balance. She filed a petition for collection of sum of money before the Rizal RTC and the case was given to J Sarmiento. On 27 Mar 79, Azul received the copy of the complaint. On 10 Apr 79, Azul filed a motion for a 15 day extension to file for responsive pleading. Azul was unaware that J Sarmiento retired and was temporarily substituted by J Aover who granted the extension but only for 5 days starting the next day. But Azul only received the notice granting such on the 23rd of the same month way passed the 5 day period. On the 17th of April, Tecson already filed a motion to dismiss averring that Azuls 5 day extension has already lapsed. On the 18th of the same month, J Castro, the permanent judge to replace J Sarmiento took office and he ordered Azul to be in default due to the lapse of the 5 day extension. J Castro proceeded with the reception of evidence the next day and of course without Azuls evidence as he was still unaware of him being in default. On April 27th, J Castro ruled in favor Tecson. On May 2nd Azul, unaware that J Castro already decided the case appealed to remove his default status. On May 7th Azul received the decision rendered by the court on Apr 27th (but on record the date of receipt was May 5th). Azul filed a motion for new trial on June 6th. The lower court denied the same on the 20th of the same month. On Aug 1st, Azul filed a notice of appeal it was denied on the 3rd but was reconsidered on the 7th hence Azul filed his record on appeal on the 21st and J Castro approved it on the 27th but surprisingly upon motion of Tecson on the 30th, J Castro set aside its earlier decisaion on the 27th. Finally, J Castro denied the appeal on the 7th of September. ISSUE: Whether or not Azul has been denied due process. HELD: The SC agreed with the Azul that he was denied due process. The constitutional provision on due process commands all who wield public authority, but most peremptorily courts of justice, to strictly maintain standards of fundamental fairness and to insure that procedural safeguards essential to a fair trial are observed at all stages of a proceeding. It may be argued that when the Azuls counsel asked for a fifteen (15) day extension from April 11, 1979 to file his answer, it was imprudent and neglectful for him to assume that said first extension would be granted. However, the records show that Atty. Camaya personally went to the session hall of the court with his motion for postponement only to be informed that J Sarmiento had just retired but that his motion would be considered submitted for resolution. Since the sala was vacant and pairing judges in Quezon City are literally swamped with their own heavy loads of cases, counsel may be excused for assuming that, at the very least, he had the requested fifteen (15) days to file his responsive pleading. It is likewise inexplicable why J Aover, who had not permanently taken over the sala vacated by the retired judge, should suddenly rule that only a five-day extension would be allowed. And to compound the Azuls problems, the order was sent by mail and received only twelve (12) days later or after the five-day period. Before the much publicized Project Mercury of the Bureau of Posts, a court should have known that court orders requiring acts to be done in a matter of days should not be sent by mail. Meanwhile, the petitioner was declared in default. The motion to declare defendant in default is dated April 17, 1979. No copy was furnished the petitioner. It was acted upon on April 18, 1979, the very first day in office of J Castro in Quezon City.

Mayor Miguel Paderanga vs Judge Cesar Azura


Due Process Hostility Between the Judge and the Parties Inhibition

Paderanga was the mayor of Gingoog City, Misamis Oriental. He petitioned that J Azura inhibits himself from deciding on pending cases brought before him on the grounds that they have lost confidence in him, that he entertained tax suits against the city and had issued TROs on the sales of properties when it is clearly provided for by law (Sec 74 PD 464) that the remedy to stop auction is to pay tax, that J Azura is bias, oppressive and is abusive in his power. ISSUE: Whether or not J Azura should inhibit himself from the trial. HELD: The SC ruled that Azura must. As decided in the Pimentel Case (21 SCRA 160), All the foregoing notwithstanding, this should be a good occasion as any to draw attention of all judges to appropriate guidelines in a situation where their capacity to try and decide fairly and judiciously comes to the fore by way of challenge from any one of the parties. A judge may not be legally prohibited from sitting in a litigation But when suggestion is made of record that he might be induced to act in favor of one party or with bias or prejudice against a litigant arising out of circumstances reasonably capable of inciting such a state of mind, he should conduct a careful self-examination. He should exercise his discretion in a way that the peoples faith in the courts of justice is not impaired. . . . The reminder is also apropos that next in importance to the duty of rendering a righteous judgment is that of doing it in such a manner as will beget no suspicion of the fairness and integrity of the judge . . . .

Filemon David vs Judge Gregorio Aquilizan et al


Due Process Hearing David has a large parcel of land in Polomolok, Cotabato. He let Felomeno Jugar and Ricardo Jugar tend and caretake separate portions of his land in 1971. The land is estimated to be yielding 60-70 cavans of corn cobs an dthe share agreed upon is 50-50. In 1973, David withdrew the land from the brothers and has not allowed them to go back there. The brothers prayed for reinstatement but David refused to do so. David denied that the borthers were his tenants. He said that Ricardo was his tractor driver before but he resigned to take care of his dad and to work for DOLE. Fewlomeno on the other hand surrendered the portion of the land he was tending to continue his faith healing. J Aquilizan handled the case filed by the brothers against David and after three months he rendered a decision in favor of the brothers without any hearing. David averred he was denied due process. J Aquilizan admitted that there was indeed no hearing conducted but he said the decision has already become final and executory as the period for appeal has already lapsed. ISSUE: Whether or not David is entitled to an appeal. HELD: The SC ruled in favor of David. A decision rendered without a hearing is null and void and may be attacked directly or collaterally. The decision is null and void for want of due process. And it has been held that a final and executory judgment may be set aside with a view to the renewal of the litigation when the judgment is void for lack of due process of law. In legal contemplation, it is as if no judgment has been rendered at all.

Anita Lorenzana vs Polly Cayetano


Due Process Hearing Lorenzana was renting a parcel of land from the Manila Railroad Company (later from the Bureau of Lands). She later purchased the land (San Lazaro Estate). She had the property be rented to tenants occupying stalls. Due to nonpayment of rents, she filed 12 ejectment cases against her tenant. On the other hand, Cayetano was an occupant of a parcel of land adjacent to that of Lorenzanas land. Cayetano was renting the same from the Bureau of Lands. The lower court granted Lorenzanas ejectment cases. Lorenzana then secured a writ of execution to forcibly eject her tenants but she included to eject Cayetanos property. Cayetano was not a party to the ejectment cases so she prayed for the lower court that her property be not touched. The lower court denied Cayetanos petition. The CA, upon appeal, favored Cayetano. Lorenzana averred that Cayetano is now a party to the ejectment cases as she already brought herself to the Courts jurisdiction by virtue of her appeal. ISSUE: Whether or not Cayetanos right to due process has been violated. HELD: The SC ruled in favor of Cayetano and has affirmed the CA. It must be noted that respondent was not a party to any of the 12 ejectment cases wherein the writs of demolition had been issued; she did not make her appearance in and during the pendency of these ejectment cases. Cayetano only went to court to protect her property from demolition after the judgment in the ejectment cases had become final and executory. Hence, with respect to the judgment in said ejectment cases, Cayetano remains a third person to such judgment, which does not bind her; nor can its writ of execution be informed against her since she was not afforded her day in court in said ejectment cases.

Zambales Chromite Mining et al vs Court of Appeals


Due Process Administrative Due Process ZCM filed an administrative case before the Director of Mines Gozon to have them be declared the rightful and prior locators and possessors of 69 mining claims in Sta. Cruz, Zambales. They are asserting their claim against the group of Martinez and Pabiloa. Gozon decided in favor of Martinez et al. ZCM appealed the case before the Secretary of Agriculture and Natural Resources. During pendency, Gozon was assigned as the Sec of Agri. And Natural Resources. He did not inhibit himself from deciding on the appeal but he instead affirmed his earlier decision when he was still the director of mines. ZCM then appealed before the CFI of Zambales. The CFI affirmed the decision of Gozon. It held that the disqualification of a judge to review his own decision or ruling (Sec. 1, Rule 137, Rules of Court) does not apply to administrative bodies; that there is no provision in the Mining Law, disqualifying the Secretary of Agriculture and Natural Resources from deciding an appeal from a case which he had decided as Director of Mines; that delicadeza is not a ground for disqualification; that the ZCM did not seasonably seek to disqualify Gozon from deciding their appeal, and that there was no evidence that Gozon acted arbitrarily and with bias, prejudice, animosity or hostility to ZCM. ZCM

appealed the case to the CA. The CA reversed Gozons finding and declared that ZCM had the rights earlier attributed to Martinez et al by Gozon. Martinez et al appealed averring that the factual basis found by Gozon as Director of Mines be given due weight. The CA reconsidered after realizing that Gozon cannot affirm his own decision and the CA remanded the case to the Minister of Natural Resources. Now both parties appealed urging their own contentions; ZCM wants the CAs earlier decision to be reaffirmed while Martinez et al demanded that Gozons finding be reinstated. The CA denied both petition. ISSUE: Whether or not Gozon can validly affirm his earlier decision w/o disturbing due process. HELD: The SC annulled the decision of Gozon calling it as a mockery of justice. Gozon had acted with grave abuse of discretion. In order that the review of the decision of a subordinate officer might not turn out to be a farce, the reviewing officer must perforce be other than the officer whose decision is under review; otherwise, there could be no different view or there would be no real review of the case. The decision of the reviewing officer would be a biased view; inevitably, it would be the same view since being human, he would not admit that he was mistaken in his first view of the case. The SC affirmed the 2nd decision of the CA.

FELICIDAD ANZALDO vs JACOBO C. CLAVE et al


Due Process Administrative Due Process Dr Anzaldo , 55, had been working in the National Institute of Science and Technology for 28 years. She was holding the position Scientist Research Associate IV when she was appointed as Science Research Supervisor II. Her appointment was approved by the CSC in 1978. The position was previously held by Dr Kintanar who recommended Dr Venzon to his position. Dr Venzon contested the position. Dr Afable, the one who appointed Anzaldo, averred that Anzaldos appointment was approved by the NIST evaluation Committee which gave 88 points to Anzalado and 66 points to Venzon. The issue was elevated to the Office of the president by Venzon. Clave was then the Presidential Executive Assistant. Pursuant to PD 807 or the Civil Service Decree, Clave referred the issue to the CSC. Clave was also holding the chairmanship of the CSC. Clave issued Res 1178 appointing Venzon to the contested position. After the denial of her motion for the reconsideration of that resolution, or on January 5, 1980, Anzaldo appealed to the Office of the President of the Philippines. Since Clave was holding the office of PEA he just affirmed his decision as the CSC chairman. ISSUE: Whether or not there is due process in the case at bar. HELD: The SC ruled in favor of Anzaldo. When PEA Clave said in his decision that he was inclined to concur in the recommendation of the Civil Service Commission, what he meant was that he was concurring with Chairman Claves recommendation: he was concurring with himself. It is evident that Anzaldo was denied due process of law when Presidential Executive Assistant Clave concurred with the recommendation of (himself) Chairman Clave of the Civil Service Commission. Due process of law means fundamental fairness. It is not fair to Anzaldo

that PEA Clave should decide whether his own recommendation as Chairman of the CSC, as to who between Anzaldo and Venzon should be appointed Science Research Supervisor II, should be adopted by the President of the Philippines.

Miguel Singson vs National Labor Relations Commission & Philippine Airlines


Due Process Dismissal of Employees Singson was an employee of PAL. On 7 Jun 1991, a Japanese national alleged that Singson extorted money from her ($200.00) by accusing her of having excess baggage; and that to settle the issue she needs to pay said amount to him. Singson was later investigated and the investigating committee found him guilty. PAL then dismissed Singson from employment. Singson then filed a case before NLRC against PAL for illegal dismissal, attys fees and damages. Labor Arbiter Raul Aquino ruled in favor of Singson as he found PALs side insufficient to dismiss Singson. PAL appealed to the NLRC. The 2nd Division, composed of Calaycay, Rayala former Arbiter Raul Aquino, of the NLRC took cognizance of the case. NLRC reversed the decision of Aquino. Singson moved for reconsideration which was denied by NLRC, this time only Calaycay & Rayala voted. ISSUE: Whether or not Singson was denied of due process. HELD: The SC ruled that Singson was denied due process. The SC held that Singson was denied due process when Aquino participated, as presiding commissioner of the 2nd Division of the NLRC, in reviewing PALs appeal. He was reviewing his own decision as a former labor arbiter. Under Rule VII, Section 2 (b) of the New Rules of Procedure of the NLRC, each Division shall consist of one member from the public sector who shall act as the Presiding Commissioner and one member each from the workers and employers sectors, respectively. The composition of the Division guarantees equal representation and impartiality among its members. Thus, litigants are entitled to a review of three (3) commissioners who are impartial right from the start of the process of review. Commissioner Aquino can hardly be considered impartial since he was the arbiter who decided the case under review. He should have inhibited himself from any participation in this case. The infirmity of the resolution was not cured by the fact that the motion for reconsideration of Singson was denied by two commissioners and without the participation of Aquino. The right of petitioner to an impartial review of his appeal starts from the time he filed his appeal. He is not only entitled to an impartial tribunal in the resolution of his motion for reconsideration. Moreover, his right is to an impartial review of three commissioners. The denial of Singsons right to an impartial review of his appeal is not an innocuous error. It negated his right to due process.

NOTES: Requisites of Procedural Due Process in Administrative Proceedings

(1) the right to a hearing, which includes the right to present ones case and submit evidence in support thereof; (2) the tribunal must consider the evidence presented; (3) the decision must have something to support itself; (4) the evidence must be substantial; (5) the decision must be based on the evidence presented at the hearing, or at least contained in the record and disclosed to the parties affected; (6) the tribunal or body or any of its judges must act on its own independent consideration of the law and facts of the controversy, and not simply accept the views of a subordinate; (7) the Board or body should, in all controversial questions, render its decision in such manner that the parties to the proceeding can know the various issues involved, and the reason for the decision rendered.

Administrative Due Process Includes (a) the right to notice, be it actual or constructive, of the institution of the proceedings that may affect a persons legal right; (b) reasonable opportunity to appear and defend his rights and to introduce witnesses and relevant evidence in his favor; (c) a tribunal so constituted as to give him reasonable assurance of honesty and impartiality, and one of competent jurisdiction; and (d) a finding or decision by that tribunal supported by substantial evidence presented at the hearing or at least ascertained in the records or disclosed to the parties.

Mayor Bayani Alonte vs Judge Maximo Savellano, NBI & People of the Philippines
Due Process in Criminal Proceedings Waiver of Right to Due Process Alonte was accused of raping JuvieLyn Punongbayan with accomplice Buenaventura Concepcion. It was alleged that Concepcion befriended Juvie and had later lured her into Alonetes house who was then the mayor of Bian, Laguna. The case was brought before RTC Bian. The counsel and the prosecutor later moved for a change of venue due to alleged intimidation. While the change of venue was pending, Juvie executed an affidavit of desistance.

The prosecutor continued on with the case and the change of venue was done notwithstanding opposition from Alonte. The case was raffled to the Manila RTC under J Savellano. Savellano later found probable cause and had ordered the arrest of Alonte and Concepcion. Thereafter, the prosecution presented Juvie and had attested the voluntariness of her desistance the same being due to media pressure and that they would rather establish new life elsewhere. Case was then submitted for decision and Savellano sentenced both accused to reclusion perpetua. Savellano commented that Alonte waived his right to due process when he did not cross examine Juvie when clarificatory questions were raised about the details of the rape and on the voluntariness of her desistance. ISSUE: Whether or not Alonte has been denied criminal due process. HELD: The SC ruled that Savellano should inhibit himself from further deciding on the case due to animosity between him and the parties. There is no showing that Alonte waived his right. The standard of waiver requires that it not only must be voluntary, but must be knowing, intelligent, and done with sufficient awareness of the relevant circumstances and likely consequences. Mere silence of the holder of the right should not be so construed as a waiver of right, and the courts must indulge every reasonable presumption against waiver. Savellano has not shown impartiality by repeatedly not acting on numerous petitions filed by Alonte. The case is remanded to the lower court for retrial and the decision earlier promulgated is nullified.

Read full text here.

NOTES: Due process in criminal proceedings (a) that the court or tribunal trying the case is properly clothed with judicial power to hear and determine the matter before it; (b) that jurisdiction is lawfully acquired by it over the person of the accused; (c) that the accused is given an opportunity to be heard; and (d) that judgment is rendered only upon lawful hearing.

Section 3, Rule 119, of the Rules of Court

Sec. 3. Order of trial. The trial shall proceed in the following order:

(a) The prosecution shall present evidence to prove the charge and, in the proper case, the civil liability.

(b) The accused may present evidence to prove his defense, and damages, if any, arising from the issuance of any provisional remedy in the case.

(c) The parties may then respectively present rebutting evidence only, unless the court, in furtherance of justice, permits them to present additional evidence bearing upon the main issue.

(d) Upon admission of the evidence, the case shall be deemed submitted for decision unless the court directs the parties to argue orally or to submit memoranda.

(e) However, when the accused admits the act or omission charged in the complaint or information but interposes a lawful defense, the order of trial may be modified accordingly. G.R. No. 184869 September 21, 2010

CENTRAL MINDANAO UNIVERSITY, Represented by Officer-In-Charge Dr. Rodrigo L. Malunhao, Petitioner, vs. THE HONORABLE EXECUTIVE SECRETARY, THE HONORABLE SECRETARY OF THE DEPARTMENT OF ENVIRONMENT AND NATURAL RESOURCES, THE CHAIRPERSON AND COMMISSIONERS OF THE NATIONAL COMMISSION ON INDIGENOUS PEOPLES, and THE LEAD CONVENOR OF THE NATIONAL ANTIPOVERTY COMMISSION, Respondents. DECISION ABAD, J.: This case concerns the constitutionality of a presidential proclamation that takes property from a state university, over its objections, for distribution to indigenous peoples and cultural communities.

The Facts and the Case Petitioner Central Mindanao University (CMU) is a chartered educational institution owned and run by the State.1 In 1958, the President issued Presidential Proclamation 476, reserving 3,401 hectares of lands of the public domain in Musuan, Bukidnon, as school site for CMU. Eventually, CMU obtained title in its name over 3,080 hectares of those lands under Original Certificates of Title (OCTs) 0-160, 0-161, and 0-162. Meanwhile, the government distributed more than 300 hectares of the remaining untitled lands to several tribes belonging to the areas cultural communities. Forty-five years later or on January 7, 2003 President Gloria Macapagal-Arroyo issued Presidential Proclamation 310 that takes 670 hectares from CMUs registered lands for distribution to indigenous peoples and cultural communities in Barangay Musuan, Maramag, Bukidnon. On April 3, 2003, however, CMU filed a petition for prohibition against respondents Executive Secretary, Secretary of the Department of Environment and Natural Resources, Chairperson and Commissioner of the National Commission on Indigenous Peoples (NCIP), and Lead Convenor of the National Anti-Poverty Commission (collectively, NCIP, et al) before the Regional Trial Court (RTC) of Malaybalay City (Branch 9), seeking to stop the implementation of Presidential Proclamation 310 and have it declared unconstitutional. The NCIP, et al moved to dismiss the case on the ground of lack of jurisdiction of the Malaybalay RTC over the action, pointing out that since the act sought to be enjoined relates to an official act of the Executive Department done in Manila, jurisdiction lies with the Manila RTC. The Malaybalay RTC denied the motion, however, and proceeded to hear CMUs application for preliminary injunction. Meanwhile, respondents NCIP, et al moved for partial reconsideration of the RTCs order denying their motion to dismiss. On October 27, 2003, after hearing the preliminary injunction incident, the RTC issued a resolution granting NCIP, et als motion for partial reconsideration and dismissed CMUs action for lack of jurisdiction. Still, the RTC ruled that Presidential Proclamation 310 was constitutional, being a valid State act. The RTC said that the ultimate owner of the lands is the State and that CMU merely held the same in its behalf. CMU filed a motion for reconsideration of the resolution but the RTC denied the same on April 19, 2004. This prompted CMU to appeal the RTCs dismissal order to the Court of Appeals (CA) Mindanao Station.2 CMU raised two issues in its appeal: 1) whether or not the RTC deprived it of its right to due process when it dismissed the action; and 2) whether or not Presidential Proclamation 310 was constitutional.3 In a March 14, 2008 decision,4 the CA dismissed CMUs appeal for lack of jurisdiction, ruling that CMUs recourse should have been a petition for review on certiorari filed directly with this Court, because it raised pure questions lawbearing mainly on the constitutionality of Presidential Proclamation 310. The CA added that whether the trial court can decide the merits

of the case based solely on the hearings of the motion to dismiss and the application for injunction is also a pure question of law. CMU filed a motion for reconsideration of the CAs order of dismissal but it denied the same,5 prompting CMU to file the present petition for review. The Issues Presented The case presents the following issues: 1. Whether or not the CA erred in not finding that the RTC erred in dismissing its action for prohibition against NCIP, et al for lack of jurisdiction and at the same time ruling that Presidential Proclamation 310 is valid and constitutional; 2. Whether or not the CA correctly dismissed CMUs appeal on the ground that it raised purely questions of law that are proper for a petition for review filed directly with this Court; and 3. Whether or not Presidential Proclamation 310 is valid and constitutional. The Courts Rulings One. The RTC invoked two reasons for dismissing CMUs action. The first is that jurisdiction over the action to declare Presidential Proclamation 310 lies with the RTC of Manila, not the RTC of Malaybalay City, given that such action relates to official acts of the Executive done in Manila. The second reason, presumably made on the assumption that the Malaybalay RTC had jurisdiction over the action, Presidential Proclamation 310 was valid and constitutional since the State, as ultimate owner of the subject lands, has the right to dispose of the same for some purpose other than CMUs use. There is nothing essentially wrong about a court holding on the one hand that it has no jurisdiction over a case, and on the other, based on an assumption that it has jurisdiction, deciding the case on its merits, both with the same results, which is the dismissal of the action. At any rate, the issue of the propriety of the RTC using two incompatible reasons for dismissing the action is academic. The CA from which the present petition was brought dismissed CMUs appeal on some technical ground. Two. Section 9(3) of the Judiciary Reorganization Act of 19806 vests in the CA appellate jurisdiction over the final judgments or orders of the RTCs and quasi-judicial bodies. But where an appeal from the RTC raises purely questions of law, recourse should be by a petition for review on certiorari filed directly with this Court. The question in this case is whether or not CMUs appeal from the RTCs order of dismissal raises purely questions of law. As already stated, CMU raised two grounds for its appeal: 1) the RTC deprived it of its right to due process when it dismissed the action; and 2) Presidential Proclamation 310 was

constitutional. Did these grounds raise factual issues that are proper for the CA to hear and adjudicate? Regarding the first reason, CMUs action was one for injunction against the implementation of Presidential Proclamation 310 that authorized the taking of lands from the university. The fact that the President issued this proclamation in Manila and that it was being enforced in Malaybalay City where the lands were located were facts that were not in issue. These were alleged in the complaint and presumed to be true by the motion to dismiss. Consequently, the CMUs remedy for assailing the correctness of the dismissal, involving as it did a pure question of law, indeed lies with this Court. As to the second reason, the CMU claimed that the Malaybalay RTC deprived it of its right to due process when it dismissed the case based on the ground that Presidential Proclamation 310, which it challenged, was constitutional. CMU points out that the issue of the constitutionality of the proclamation had not yet been properly raised and heard. NCIP, et al had not yet filed an answer to join issue with CMU on that score. What NCIP, et al filed was merely a motion to dismiss on the ground of lack of jurisdiction of the Malaybalay RTC over the injunction case. Whether the RTC in fact prematurely decided the constitutionality of the proclamation, resulting in the denial of CMUs right to be heard on the same, is a factual issue that was proper for the CA Mindanao Station to hear and ascertain from the parties. Consequently, the CA erred in dismissing the action on the ground that it raised pure questions of law. Three. Since the main issue of the constitutionality of Presidential Proclamation 310 has been raised and amply argued before this Court, it would serve no useful purpose to have the case remanded to the CA Mindanao Station or to the Malaybalay RTC for further proceedings. Ultimately, the issue of constitutionality of the Proclamation in question will come to this Court however the courts below decide it. Consequently, the Court should, to avoid delay and multiplicity of suits, now resolve the same. The key question lies in the character of the lands taken from CMU. In CMU v. Department of Agrarian Reform Adjudication Board (DARAB),7 the DARAB, a national government agency charged with taking both privately-owned and government-owned agricultural lands for distribution to farmers-beneficiaries, ordered the segregation for this purpose of 400 hectares of CMU lands. The Court nullified the DARAB action considering the inalienable character of such lands, being part of the long term functions of an autonomous agricultural educational institution. Said the Court: The construction given by the DARAB to Section 10 restricts the land area of the CMU to its present needs or to a land area presently, actively exploited and utilized by the university in carrying out its present educational program with its present student population and academic facility overlooking the very significant factor of growth of the university in the years to come. By the nature of the CMU, which is a school established to promote agriculture and industry, the need for a vast tract of agricultural land for future programs of expansion is obvious. At the outset, the CMU was conceived in the same manner as land grant colleges in America, a type of educational institution which blazed the trail for the development of vast tracts of unexplored and undeveloped agricultural lands in the Mid-West. What we now know as

Michigan State University, Penn State University and Illinois State University, started as small land grant colleges, with meager funding to support their ever increasing educational programs. They were given extensive tracts of agricultural and forest lands to be developed to support their numerous expanding activities in the fields of agricultural technology and scientific research. Funds for the support of the educational programs of land grant colleges came from government appropriation, tuition and other student fees, private endowments and gifts, and earnings from miscellaneous sources. It was in this same spirit that President Garcia issued Proclamation No. 476, withdrawing from sale or settlement and reserving for the Mindanao Agricultural College (forerunner of the CMU) a land reservation of 3,080 hectares as its future campus. It was set up in Bukidnon, in the hinterlands of Mindanao, in order that it can have enough resources and wide open spaces to grow as an agricultural educational institution, to develop and train future farmers of Mindanao and help attract settlers to that part of the country. xxxx The education of the youth and agrarian reform are admittedly among the highest priorities in the government socio-economic programs. In this case, neither need give way to the other. Certainly, there must still be vast tracts of agricultural land in Mindanao outside the CMU land reservation which can be made available to landless peasants, assuming the claimants here, or some of them, can qualify as CARP beneficiaries. To our mind, the taking of the CMU land which had been segregated for educational purposes for distribution to yet uncertain beneficiaries is a gross misinterpretation of the authority and jurisdiction granted by law to the DARAB. The decision in this case is of far-reaching significance as far as it concerns state colleges and universities whose resources and research facilities may be gradually eroded by misconstruing the exemptions from the CARP. These state colleges and universities are the main vehicles for our scientific and technological advancement in the field of agriculture, so vital to the existence, growth and development of this country.8 It did not matter that it was President Arroyo who, in this case, attempted by proclamation to appropriate the lands for distribution to indigenous peoples and cultural communities. As already stated, the lands by their character have become inalienable from the moment President Garcia dedicated them for CMUs use in scientific and technological research in the field of agriculture. They have ceased to be alienable public lands.1avvphi1 Besides, when Congress enacted the Indigenous Peoples Rights Act (IPRA) or Republic Act 83719 in 1997, it provided in Section 56 that "property rights within the ancestral domains already existing and/or vested" upon its effectivity "shall be recognized and respected." In this case, ownership over the subject lands had been vested in CMU as early as 1958. Consequently, transferring the lands in 2003 to the indigenous peoples around the area is not in accord with the IPRA. Furthermore, the land registration court considered the claims of several tribes belonging to the areas cultural communities in the course of the proceedings for the titling of the lands in CMUs name. Indeed, eventually, only 3,080 hectares were titled in CMUs name under OCTs 0-160, 0-

161 and 0-162. More than 300 hectares were acknowledged to be in the possession of and subject to the claims of those tribes. WHEREFORE, the Court GRANTS the petition, SETS ASIDE the March 14, 2008 decision and September 22, 2008 resolution of the Court of Appeals in CA-G.R. SP 85456, and DECLARES Presidential Proclamation 310 as null and void for being contrary to law and public policy. SO ORDERED.

Romeo Erece vs Lyn Macalingay et al


Due Process Administrative Bodies Atty Erece was the Regional Director CHR Region 1. Macalingay et al were Ereces subordinates. Macalingay et al were complaining that Erece had continuously denied them from using the company vehicle. That Erece had been receiving his Representation and Transportation Allowance yet he prioritizes himself in the use of the vehicle. The issue reached the CSc proper which found Erece guilty as charged. Erece contends that he was denied due process as he was not afforded the right to cross-examine his accusers and their witnesses. He stated that at his instance, in order to prevent delay in the disposition of the case, he was allowed to present evidence first to support the allegations in his Counter-Affidavit. After he rested his case, respondents did not present their evidence, but moved to submit their position paper and formal offer of evidence, which motion was granted by the CSC over his objection. Macalingay et al then submitted their Position Paper and Formal Offer of Exhibits. Erece submits that although he was allowed to present evidence first, it should not be construed as a waiver of his right to crossexamine the complainants. Although the order of presentation of evidence was not in conformity with the procedure, still Erece should not be deemed to have lost his right to cross-examine his accusers and their witnesses. This may be allowed only if he expressly waived said right. ISSUE: Whether or not Erece had been denied due process. HELD: The SC agrees with the CA that petitioner was not denied due process when he failed to cross-examine the complainants and their witnesses since he was given the opportunity to be heard and present his evidence. In administrative proceedings, the essence of due process is simply the opportunity to explain ones side. Judicial Due Process vs Administrative Due Process Due process of law in administrative cases is not identical with judicial process for a trial in court is not always essential to due process. While a day in court is a matter of right in judicial proceedings, it is otherwise in administrative proceedings since they rest upon different principles. The due process clause guarantees no particular form of procedure and its requirements are not technical. Thus, in certain proceedings of administrative character, the right to a notice or hearing are not essential to due process of law. The constitutional requirement of due process is met by a fair hearing before a regularly established administrative agency or

tribunal. It is not essential that hearings be had before the making of a determination if thereafter, there is available trial and tribunal before which all objections and defenses to the making of such determination may be raised and considered. One adequate hearing is all that due process requires. . . . The right to cross-examine is not an indispensable aspect of due process. Nor is an actual hearing always essential. . . .

Datu Pax Pakung Mangudadatu vs HRET & Angelo Montilla


Due Process Administrative Bodies HRET Service of Summons Datu Pax and Montilla were rivals in the Congressional elections in 2007. Datu Pax won the elections. Montilla contested the results of the elections before the HRET. The Secretary of HRET the issued the summons to Datu Pax to his quite remote residence in Sultan Kudarat. Datu Pax was required to file a reply within ten days from receipt. The summons was received by a certain Aileen Baldenas. 43 days past and no answer was received from Datu Pax as he was unaware of the summons. HRET then considered such inaction as a general denial to the protest. Datu Pax later learned about the protest against him and he coordinated with his lawyers to appear on behalf of him and to present the answer as well as to file counter protest. He alleged that he does not know of a Baldenas nor was she a part of the household. HRET denied his motion and had proceeded to the recount as prayed for by Montilla. ISSUE: Whether or not Datu Pax was denied due process by reason that he did not receive the summons personally. HELD: The SC ruled in favor of Datu Pax. The summons to Datu Pax should not have been delivered via registered mail as the same is susceptible to fraud. The HRET should have mde use of its own servers to make sure that the summons is personally received by Datu Pax. The 1997 Rules of Civil Procedure (which is in one way or the other adopted by the 2004 HRET rules on summons) provides that: SEC. 6. Service in person on defendant. Whenever practicable, the summons shall be served handling a copy thereof to the defendant in person, or, if he refuses to receive and sign for it, by tendering it to him. SEC. 7. Substituted service. If, for justifiable causes, the defendant cannot be served within a reasonable time as provided in the preceding section, service may be effected (a) by leaving copies of the summons at the defendants residence with some person of suitable age and discretion then residing therein, or (b) by leaving copies at defendants office or regular place of business with some competent person in charge thereof. Indeed, if in ordinary civil cases (which involve only private and proprietary interests) personal service of summons is preferred and service by registered mail is not allowed on jurisdictional

and due process grounds, with more reason should election cases (which involve public interest and the will of the electorate) strictly follow the hierarchy of modes of service of summons under the Rules of Court.
G.R. No. 157784: Department of Education vs Godofredo Cuanan

Due Process Certiorari Service of Summons

In1996, Cuanan, while being a school principal in San Antonio, Nueva Ecija, wascharged for sexual harassment. DECS Region III created an investigatingcommittee and the latter found Cuanan guilty as charged. Regional DirectorLabrador the forced resignation of Cuanan. In 2000, then Sec Gonzales affirmedthe decision of Labrador and had denied Cuanans Motion for Reconsideration. CUananappealed to the CSC which reversed Gonzales decision in Jan 2003 and CSCissued a copy of the resolution to Cuanan and Dep Ed, however, it seems thatDep Ed was not able to receive the copy as it requested a copy thereof again.The next month, Cuanan requested to be reinstated which was indorsed by theSuperintendent. In Mar 2003, now Dep Ed Sec de Jesus received a copy of theresolution. In April, de Jesus filed a motion for reconsideration w/ofurnishing a copy to Cuanan. In July 2003, de Jesus filed a supplemental motionfor reconsideration; no copy was furnished to Cuanan again. In Oct 2004, CSCreversed its decision and found Cuanan guilty of sexual harassment. Cuananfiled a certiorari before the CA. He averred that the CSC decision in Jan 2003had already become final and executory. The CA reversed the decision of theCSC. DepEd averred that the proper remedy should have been a petition forreview.

ISSUE: Whether or not there is dueprocess in the case at bar.

HELD: The SC affirmed the ruling ofthe CA. It noted that DepEd has the power to file a motion for reconsiderationin the case at bar. The SC maintainedthat the disciplining authority qualifies as a party adversely affected by thejudgment, who can file an appeal of a judgment of exoneration in anadministrative case. CSC Resolution 021600 provides: Section 2. Coverage andDefinition of Terms. - x x x (l) PARTY ADVERSELY AFFECTED refers to therespondent against whom a decision in a disciplinary case has been rendered or tothe disciplining authority in an appeal from a decision exonerating the saidemployee.

The remedy of an aggrieved partyfrom a resolution issued by the CSC is to file a petition for review thereofunder Rule 43 of the Rules of Court within fifteen days from notice of theresolution. Recourse to a petition for certiorari under Rule 65 rendersthe petition dismissible for being the wrong remedy. Nonetheless, there areexceptions to this rule, to wit: (a) when public

welfare and the advancement ofpublic policy dictates; (b) when the broader interest of justice so requires; (c)when the writs issued are null and void; or (d) when the questioned orderamounts to an oppressive exercise of judicial authority. As will be shownforthwith, exception (c) applies to the present case.

Furthermore,while a motion for reconsideration is a condition precedent to the filing of apetition for certiorari, immediate recourse to the extraordinary remedyof certiorari is warranted where the order is a patent nullity, as wherethe court a quo has no jurisdiction; where petitioner was deprived of dueprocess and there is extreme urgency for relief; where the proceedings in thelower court are a nullity for lack of due process; where the proceeding was exparte or one in which the petitioner had no opportunity to object. These exceptions find application toCuanan's petition for certiorari in the CA.

At anyrate, Cuanan's petition for certiorari before the CA could be treated asa petition for review, the petition having been filed on November 22, 2004, orthirteen (13) days from receipt on November 9, 2004 of CSC Resolution No.041147, clearly within the 15-day reglementary period for the filing of apetition for review. Such move would be in accordance with the liberal spiritpervading the Rules of Court and in the interest of substantial justice.

Cuananundoubtedly was denied procedural due process. He had no opportunity toparticipate in the proceedings for the petition for review/ reconsiderationfiled by the DepEd, since no copy of the pleadings filed by the DepEd wereserved upon him or his counsel; nor was he even required by the CSC to file hiscomments thereon. Considering that pleadings filed by the DepEd were not servedupon Cuanan, they may be treated as mere scraps of paper which should not havemerited the attention or consideration of the CSC.

CERTIORARI, defined;

When anytribunal, board, or officer exercising judicial functions has acted without orin excess of its or his jurisdiction, or with grave abuse of discretion, andthere is no appeal, nor any plain, speedy, and adequate remedy in the ordinarycourse of law, a person aggrieved thereby may file a verified petition forcertiorari in the proper court alleging the facts with certainty and prayingthat judgment be rendered annulling or modifying the proceedings, as the lawrequires, of such tribunal, board or officer.

Ang Tibay vs Court of Industrial Relations

Due Process Admin Bodies CIR TeodoroToribio owns and operates Ang Tibay a leather company which supplies the Philippine Army. Due to alleged shortage of leather, Toribio caused the lay off of members of National Labor Union Inc. NLU averred that Toribios act is not valid as it is not within the CBA. That there are two labor unions in Ang Tibay; NLU and National Workers Brotherhood. That NWB is dominated by Toribio hence he favors it over NLU. That NLU wishes for a new trial as they were able to come up with new evidence/documents that they were not able to obtain before as they were inaccessible and they were not able to present it before in the CIR. ISSUE: Whether or not there has been a due process of law. HELD: The SC ruled that there should be a new trial in favor of NLU. The SC ruled that all administrative bodies cannot ignore or disregard the fundamental and essential requirements of due process. They are; (1) The right to a hearing which includes the right of the party interested or affected to present his own case and submit evidence in support thereof. (2) Not only must the party be given an opportunity to present his case and to adduce evidence tending to establish the rights which he asserts but the tribunal must consider the evidence presented. (3) While the duty to deliberate does not impose the obligation to decide right, it does imply a necessity which cannot be disregarded, namely, that of having something to support its decision. A decision with absolutely nothing to support it is a nullity, a place when directly attached. (4) Not only must there be some evidence to support a finding or conclusion but the evidence must be substantial. Substantial evidence is more than a mere scintilla It means such relevant evidence as a reasonable mind might accept as adequate to support a conclusion. (5) The decision must be rendered on the evidence presented at the hearing, or at least contained in the record and disclosed to the parties affected. (6) The Court of Industrial Relations or any of its judges, therefore, must act on its or his own independent consideration of the law and facts of the controversy, and not simply accept the views of a subordinate in arriving at a decision. (7) The Court of Industrial Relations should, in all controversial questions, render its decision in such a manner that the parties to the proceeding can know the vario issues involved, and the reasons for the decisions rendered. The performance of this duty is inseparable from the authority conferred upon it.

American Tobacco Company et al vs Director of Patents


Due Process

ATC et al filed before the Philippine Patent Office concerning the use of trade mark and trade name. ATC et al challenged the validity of Rule 168 of the Revised Rules of Practice before the Philippine Patent Office in Trademark Cases as amended, authorizing the Director of Patents to designate any ranking official of said office to hear inter partes proceedings. Said Rule likewise provides that all judgments determining the merits of the case shall be personally and directly prepared by the Director and signed by him. These proceedings refer to the hearing of opposition to the registration of a mark or trade name, interference proceeding instituted for the purpose of determining the question of priority of adoption and use of a trade-mark, trade name or service-mark, and cancellation of registration of a trade-mark or trade name pending at the Patent Office. Petitioners filed their objections to the authority of the hearing officers to hear their cases, alleging that the amendment of the Rule is illegal and void because under the law the Director must personally hear and decide inter partes case. Said objections were overruled by the Director of Patents, hence, the present petition for mandamus, to compel the Director of Patents to personally hear the cases of petitioners, in lieu of the hearing officers. ISSUE: Whether or not the hearing done by hearing officers are within due process. HELD: The SC ruled that the power to decide resides solely in the administrative agency vested by law, this does not preclude a delegation of the power to hold a hearing on the basis of which the decision of the administrative agency will be made. The rule that requires an administrative officer to exercise his own judgment and discretion does not preclude him from utilizing, as a matter of practical administrative procedure, the aid of subordinates to investigate and report to him the facts, on the basis of which the officer makes his decisions. It is sufficient that the judgment and discretion finally exercised are those of the officer authorized by law. Neither does due process of law nor the requirements of fair hearing require that the actual taking of testimony be before the same officer who will make the decision in the case. As long as a party is not deprived of his right to present his own case and submit evidence in support thereof, and the decision is supported by the evidence in the record, there is no question that the requirements of due process and fair trial are fully met. In short, there is no abnegation of responsibility on the part of the officer concerned as the actual decision remains with and is made by said officer. It is, however, required that to give the substance of a hearing, which is for the purpose of making determinations upon evidence the officer who makes the determinations must consider and appraise the evidence which justifies them.
G.R. No. 78763: Manila Electric Company vs National Labor Relations Commission & Apolinario Signo

Due Process

Signowas employed in Meralco as supervisor-leadman since Jan 1963.In 1981, hesupervised the installation of electricity in de Laras house in Antipolo. DeLaras house was not yet within the required 30-meter distance from the Meralcofacility hence he is not yet within the service scope of Meralco. As aworkaround, Signo had it be declared that a certain sarisari store nearer thefacility be declared as de Laras so as to facilitate the installation.Evertything would have been smooth thereafter but due to fault of the PowerSales Division of Meralco, de Lara was not

billed for a year. Investigation wasconducted and Meralco found out the irregularity in Signos work on de Laraselectricity installation. Signo was dismissed on May 18, 1983. Signo filed acase for illegal dismissal and for backwages. The Lanor Arbiter ruled thatthough there is a breach of trust in the actuations of Signo dismissal is aharsh penalty as Signo has been employed for more than 20 years by Meralco andhas been commended twice before for honesty. The NLRC affirmed the LaborArbiter. Meralco appealed.

ISSUE: Whether or not there has beendue process in the dismissal of Signo.

HELD: The SC sustained the decisionof the NLRC. Well-established is the principle that findings of administrativeagencies which have acquired expertise because their jurisdiction is confinedto specific matters are generally accorded not only respect but even finality.Judicial review by this Court on labor cases does not go so far as to evaluatethe sufficiency of the evidence upon which the proper labor officer or officebased his or its determination but is limited to issues of jurisdiction orgrave abuse of discretion.Notwithstanding the existence of a valid cause fordismissal, such as breach of trust by an employee, nevertheless, dismissalshould not be imposed, as it is too severe a penalty if the latter has beenemployed for a considerable length of time in the service of his employer. Reinstatementof respondent Signo is proper in the instant case, but without the award ofbackwages, considering the good faith of the employer in dismissing therespondent.
Diosdado Guzman vs. National University Facts: Petitioners Diosdado Guzman, Ulysses Urbiztondo and Ariel Ramacula, students of respondent National University, have come to this Court to seek relief from what they describe as their school's "continued and persistent refusal to allow them to enrol." In their petition "for extraordinary legal and equitable remedies with prayer for preliminary mandatory injunction" dated August 7, 1984, they alleged that they were denied due to the fact that they were active participation in peaceful mass actions within the premises of the University. The respondents on the other hand claimed that the petitioners failure to enroll for the first semester of the school year 1984-1985 is due to their own fault and not because of their alleged exercise of their constitutional and human rights. That as regards to Guzman, his academic showing was poor due to his activities in leading boycotts of classes. That Guzman is facing criminal charges for malicious mischief before the Metropolitan Trial Court of Manila in connection with the destruction of properties of respondent University. The petitioners have failures in their records, and are not of good scholastic standing.

Held:

Immediately apparent from a reading of respondents' comment and memorandum is the fact that they had never conducted proceedings of any sort to determine whether or not petitionersstudents had indeed led or participated "in activities within the university premises, conducted without prior permit from school authorities, that disturbed or disrupted classes therein" 3 or perpetrated acts of "vandalism, coercion and intimidation, slander, noise barrage and other acts showing disdain for and defiance of University authority." 4 Parenthetically, the pendency of a civil case for damages and a criminal case for malicious mischief against petitioner Guzman, cannot, without more, furnish sufficient warrant for his expulsion or debarment from re-enrollment. Also apparent is the omission of respondents to cite this Court to any duly published rule of theirs by which students may be expelled or refused re-enrollment for poor scholastic standing. There are withal minimum standards which must be met to satisfy the demands of procedural due process; and these are, that (1) the students must be informed in writing of the nature and cause of any accusation against them; (2) they shag have the right to answer the charges against them, with the assistance of counsel, if desired; (3) they shall be informed of the evidence against them; (4) they shall have the right to adduce evidence in their own behalf; and (5) the evidence must be duly considered by the investigating committee or official designated by the school authorities to hear and decide the case.

THE PETITION WAS GRANTED AND THE RESPONDENTS ARE DIRECTED TO ALLOW THE PETITIONERS TO RE-ENROLL WITHOUT PREJUDICE TO ANY DISCIPLINARY PROCEEDINGS.

G.R. No. L-58610 September 30, 1982 BABELO BERINA, MARILOU ELAGDON, ERNESTO ROBERTO and JESUS SORIAO, petitioners, vs. PHILIPPINE MARITIME INSTITUTE, TOMAS CLOMA and JAIME CLOMA, respondents.

ABAD SANTOS, J.: Babelo Beria, Marilou Elagdon, Ernesto Roberto and Jesus Soriao are students of the Philippine Maritime Institute, PMI for short. In their petition which is styled FOR EXTRAORDINARY AND EQUITABLE REMEDY WITH PRELIMINARY

INJUNCTION, they claim that PMI, five weeks after school had started, posted sometime in August, 1981, a notice that there would be a 15% increase in tuition fees retroactive to the start of the current semester; that the students met and took positive steps in respect of the problem; that their representatives held dialogues with the school administration; "that, in reaction to these legitimate student activities and without compliance with due process respondents commencing on October 15, 1981 issued expulsion orders against Jesus Soriao, Ernesto Roberto, and Babelo Berina and an indefinite suspension against Marilou Elagdon;" that the penalties were imposed without due process and had the effect of negating the petitioners' right to free speech, peaceful assembly and petition for redress of grievances. The petitioners pray that the expulsion and suspension orders be annulled and that while the case is pending resolution they be restored to their status as students of the PMI, On November 10, 1981, We required PMI and its officers who were included as respondents to comment on the petition. We also issued a temporary restraining order commanding the respondents to refrain from carrying out the expulsion and suspension orders. PMI filed its comment as required where it said that the 15% increase in tuition fee had been authorized by the Ministry of Education and Culture; and denied that the action taken against the petitioners was in response to their activities in connection with the tuition fee increase. The comment also advances the arguments that this Court lacks jurisdiction to entertain the petition because it involves "matters that are well within the competence and jurisdiction of the lower courts to pass upon, as even more serious matters and cases of greater consequences are normally brought before them at the first instance prior to any appeal to the Supreme Court, and there are no valid and impelling excuses to warrant a direct recourse to the Highest Tribunal in the judicial hierarchy." We are not called upon to determine the validity or propriety of the tuition fee increase of 15% five weeks after the classes for the current semester had started. The issue in this case is limited to the question as to whether or not the petitioners were denied by the respondents their constitutional rights to due process, free speech, peaceful assembly and petition to redress of grievances. Treating the petition as having been filed under Rule 65 of the Rules of Court as the petitioners assert, We have no doubt that there is no absence of jurisdiction. Typical of the expulsion orders is that which was issued to petitioner Jesus Soriao on October 15, 1981, which has been marked as Annex A of the petition and which reads as follows:
For conduct unbecoming as a Cadet, you are hereby dropped from the roll of students of the School. That your actuations and behavior as reported and seen leave no other recourse hence this action.

That on September 9th, you with another student was (sic) caught inside the STC Building, distributing leaflets, enticing and coercing other students to join the slated demonstration. In the subsequent days, you were caught again by the undersigned campaigning and distributing leaflets, enjoining other students to join the boycott. That all these actions are contrary to MEC regulations and directives that appropriate action had to be taken. For your guidance.

The suspension order which was issued for Marilou Elagdon on October 20, 1981, which has been marked Annex C of the petition reads:
Please be informed that C/miss ELAGDON, Marilou is hereby suspended from her classes for conduct unbecoming of a Cadetee as against the rules and regulation of the School. Let the above-named student see the undersigned and in the meantime she remained suspended until clearance is given by this office. For your guidance.

The comment does not positively assert that in imposing the expulsion and suspension orders there was observance of due process which simply means that the petitioners should have been given an opportunity to defend themselves. It was only after the petitioners had said in their reply that the respondents failed to traverse the denial of due process that the latter invoked the legal presumption "that the ordinary course of business has been followed" (Sec. 5(q), Rule 131, Rules of Court)." It is obvious from the expulsion and suspension orders that the petitioners were denied due process, res ipsa loquitur. For the orders are bereft of the sides of the petitioners. Hence the legal presumption of regularity cannot be availed in the instant case. WHEREFORE, the petition is granted; the expulsion and suspension orders are hereby set aside but without prejudice to the power of the respondents to formally charge the petitioners for violation(s) of reasonable school rules and regulations and after due notice to hear and decide the charge. No special pronouncement as to costs. SO ORDERED. G.R. No. 127980 December 19, 2007

DE LA SALLE UNIVERSITY, INC., EMMANUEL SALES, RONALD HOLMES, JUDE DELA TORRE, AMPARO RIO, CARMELITA QUEBENGCO, AGNES YUHICO and

JAMES YAP, petitioners, vs. THE COURT OF APPEALS, HON. WILFREDO D. REYES, in his capacity as Presiding Judge of Branch 36, Regional Trial Court of Manila, THE COMMISSION ON HIGHER EDUCATION, THE DEPARTMENT OF EDUCATION CULTURE AND SPORTS, ALVIN AGUILAR, JAMES PAUL BUNGUBUNG, RICHARD REVERENTE and ROBERTO VALDES, JR., respondents. DECISION REYES, R.T., J.: NAGTATAGIS sa kasong ito ang karapatang mag-aral ng apat na estudyante na nasangkot sa away ng dalawang fraternity at ang karapatang akademiko ng isang pamantasan. PRIVATE respondents Alvin Aguilar, James Paul Bungubung, Richard Reverente and Roberto Valdes, Jr. are members of Tau Gamma Phi Fraternity who were expelled by the De La Salle University (DLSU) and College of Saint Benilde (CSB)1 Joint Discipline Board because of their involvement in an offensive action causing injuries to petitioner James Yap and three other student members of Domino Lux Fraternity. This is the backdrop of the controversy before Us pitting private respondents' right to education vis-a-vis the University's right to academic freedom. ASSAILED in this Petition for Certiorari, Prohibition and Mandamus under Rule 65 of the Rules of Court are the following: (1) Resolution of the Court of Appeals (CA) dated July 30, 1996 dismissing DLSU's petition for certiorari against respondent Judge and private respondents Aguilar, Bungubung, Reverente, and Valdes, Jr.;2 (2) Resolution of the CA dated October 15, 1996 denying the motion for reconsideration;3 (3) Order dated January 7, 1997 of the Regional Trial Court (RTC), Branch 36 Manila granting private respondent Aguilar's motion to reiterate writ of preliminary injunction;4 and (4) Resolution No. 181-96 dated May 14, 1996 of the Commission on Higher Education (CHED) exonerating private respondent Aguilar and lowering the penalties for the other private respondents from expulsion to exclusion.5 Factual Antecedents Gleaned from the May 3, 1995 Decision of the DLSU-CSB Joint Discipline Board, two violent incidents on March 29, 1995 involving private respondents occurred: x x x From the testimonies of the complaining witnesses, it appears that one week prior to March 29, 1995, Mr. James Yap was eating his dinner alone in Manang's Restaurant near La Salle, when he overheard two men bad-mouthing and apparently angry at Domino Lux. He ignored the comments of the two. When he arrived at his boarding house, he mentioned the remarks to his two other brods while watching television. These two brods had earlier finished eating their dinner at Manang's. Then, the three, together with four other persons went back to Manang's and confronted the two who were still in the

restaurant. By admission of respondent Bungubung in his testimony, one of the two was a member of the Tau Gamma Phi Fraternity. There was no rumble or physical violence then. After this incident, a meeting was conducted between the two heads of the fraternity through the intercession of the Student Council. The Tau Gamma Phi Fraternity was asking for an apology. "Kailangan ng apology" in the words of respondent Aguilar. But no apology was made. Then, 5 members of the Tau Gamma Phi Fraternity went to the tambayan of the Domino Lux Fraternity in the campus. Among them were respondents Bungubung, Reverente and Papio. They were looking for a person whose description matched James Yap. According to them, this person supposedly "nambastos ng brod." As they could not find Mr. Yap, one of them remarked "Paano ba iyan. Pasensiya na lang." Came March 29, 1995 and the following events. Ten minutes before his next class at 6:00 p.m., Mr. James Yap went out of the campus using the Engineering Gate to buy candies across Taft Avenue. As he was about to recross Taft Avenue, he heard heavy footsteps at his back. Eight to ten guys were running towards him. He panicked. He did not know what to do. Then, respondent Bungubung punched him in the head with something heavy in his hands "parang knuckles." Respondents Reverente and Lee were behind Yap, punching him. Respondents Bungubung and Valdes who were in front of him, were also punching him. As he was lying on the street, respondent Aguilar kicked him. People shouted; guards arrived; and the group of attackers left. Mr. Yap could not recognize the other members of the group who attacked him. With respect to respondent Papio, Mr. Yap said "hindi ko nakita ang mukha niya, hindi ko nakita sumuntok siya." What Mr. Yap saw was a long haired guy also running with the group. Two guards escorted Mr. Yap inside the campus. At this point, Mr. Dennis Pascual was at the Engineering Gate. Mr. Pascual accompanied Yap to the university clinic; reported the incident to the Discipline Office; and informed his fraternity brods at their tambayan. According to Mr. Pascual, their head of the Domino Lux Fraternity said: "Walang gagalaw. Uwian na lang." Mr. Ericson Cano, who was supposed to hitch a ride with Dennis Pascual, saw him under the clock in Miguel Building. However, they did not proceed directly for home. With a certain Michael Perez, they went towards the direction of Dagonoy Street because Mr. Pascual was supposed to pick up a book for his friend from another friend who lives somewhere in the area. As they were along Dagonoy Street, and before they could pass the Kolehiyo ng Malate Restaurant, Mr. Cano first saw several guys inside the restaurant. He said not to mind

them and just keep on walking. However, the group got out of the restaurant, among them respondents Reverente, Lee and Valdes. Mr. Cano told Mr. Lee: "Ayaw namin ng gulo." But, respondent Lee hit Mr. Cano without provocation. Respondent Reverente kicked Mr. Pascual and respondent Lee also hit Mr. Pascual. Mr. Cano and Mr. Perez managed to run from the mauling and they were chased by respondent Lee and two others. Mr. Pascual was left behind. After respondent Reverente first kicked him, Mr. Pascual was ganged-upon by the rest. He was able to run, but the group was able to catch up with him. His shirt was torn and he was hit at the back of his head with a lead pipe. Respondent Lee who was chasing Cano and Perez, then returned to Mr. Pascual. Mr. Pascual identified respondents Reverente and Lee, as among those who hit him. Although Mr. Pascual did not see respondent Valdes hit him, he identified respondent Valdez (sic) as also one of the members of the group. In fact, Mr. Cano saw respondent Valdes near Mr. Pascual. He was almost near the corner of Leon Guinto and Estrada; while respondent Pascual who managed to run was stopped at the end of Dagonoy along Leon Guinto. Respondent Valdes shouted: "Mga putang-ina niyo." Respondent Reverente hit Mr. Pascual for the last time. Apparently being satisfied with their handiwork, the group left. The victims, Cano, Perez and Pascual proceeded to a friend's house and waited for almost two hours, or at around 8:00 in the evening before they returned to the campus to have their wounds treated. Apparently, there were three cars roaming the vicinity.6 The mauling incidents were a result of a fraternity war. The victims, namely: petitioner James Yap and Dennis Pascual, Ericson Cano, and Michael Perez, are members of the "Domino Lux Fraternity," while the alleged assailants, private respondents Alvin Aguilar, James Paul Bungubung, Richard Reverente and Roberto Valdes, Jr. are members of "Tau Gamma Phi Fraternity," a rival fraternity. The next day, March 30, 1995, petitioner Yap lodged a complaint7 with the Discipline Board of DLSU charging private respondents with "direct assault." Similar complaints8 were also filed by Dennis Pascual and Ericson Cano against Alvin Lee and private respondents Valdes and Reverente. Thus, cases entitled "De La Salle University and College of St. Benilde v. Alvin Aguilar (AB-BSM/9152105), James Paul Bungubung (AB-PSM/9234403), Robert R. Valdes, Jr. (BS-BS-APM/9235086), Alvin Lee (EDD/9462325), Richard Reverente (AB-MGT/9153837) and Malvin A. Papio (AB-MGT/9251227)" were docketed as Discipline Case No. 9495-3-25121. The Director of the DLSU Discipline Office sent separate notices to private respondents Aguilar, Bungubung and Valdes, Jr. and Reverente informing them of the complaints and requiring them to answer. Private respondents filed their respective answers.9 As it appeared that students from DLSU and CSB10 were involved in the mauling incidents, a joint DLSU-CSB Discipline Board11 was formed to investigate the incidents. Thus, petitioner Board Chairman Emmanuel Sales sent notices of hearing12 to private respondents on April 12, 1995. Said notices uniformly stated as follows:

Please be informed that a joint and expanded Discipline Board had been constituted to hear and deliberate the charge against you for violation of CHED Order No. 4 arising from the written complaints of James Yap, Dennis C. Pascual, and Ericson Y. Cano. You are directed to appear at the hearing of the Board scheduled on April 19, 1995 at 9:00 a.m. at the Bro. Connon Hall for you and your witnesses to give testimony and present evidence in your behalf. You may be assisted by a lawyer when you give your testimony or those of your witnesses. On or before April 18, 1995, you are further directed to provide the Board, through the Discipline Office, with a list of your witnesses as well as the sworn statement of their proposed testimony. Your failure to appear at the scheduled hearing or your failure to submit the list of witnesses and the sworn statement of their proposed testimony will be considered a waiver on your part to present evidence and as an admission of the principal act complained of. For your strict compliance.13 During the proceedings before the Board on April 19 and 28, 1995, private respondents interposed the common defense of alibi, summarized by the DLSU-CSB Joint Discipline Board as follows: First, in the case of respondent Bungubung, March 29, 1995 was one of the few instances when he was picked-up by a driver, a certain Romeo S. Carillo. Most of the time, respondent Bungubung goes home alone sans driver. But on this particular date, respondent Bungubung said that his dad asked his permission to use the car and thus, his dad instructed this driver Carillo to pick-up his son. Mr. Carillo is not a family driver, but works from 8:00 a.m. to 5:00 p.m. for the Philippine Ports Authority where the elder Bungubung is also employed. Thus, attempting to corroborate the alibi of respondent Bungubung, Mr. Carillo said that he arrived at La Salle at 4:56 p.m.; picked-up respondent at 5:02 p.m.; took the Roxas Blvd. route towards respondent's house in BF Paraaque (on a Wednesday in Baclaran); and arrived at the house at 6:15 p.m. Respondent Bungubung was dropped-off in his house, and taking the same route back, Mr. Carillo arrived at the South Harbor at 6:55 p.m. the Philippine Ports Authority is located at the South Harbor.14 xxxx Secondly, respondent Valdes said that he was with his friends at McDonald's Taft just before 6:00 p.m. of March 29, 1995. He said that he left McDonald at 5:50 p.m. together to get some medicine at the university clinic for his throat irritation. He said that he was at the clinic at 5:52 p.m. and went back to McDonald, all within a span of 3 or even 4 minutes.

Two witnesses, a certain Sharon Sia and the girlfriend of respondent Valdes, a certain Jorgette Aquino, attempted to corroborate Valdez' alibi.15 xxxx Third, respondent Reverente told that (sic) the Board that he was at his home at 5:00 p.m. of March 29, 1995. He said that he was given the responsibility to be the paymaster of the construction workers who were doing some works in the apartment of his parents. Although he had classes in the evening, the workers according to him would wait for him sometimes up to 9:00 p.m. when he arrives from his classes. The workers get paid everyday. Respondent Reverente submitted an affidavit, unsigned by the workers listed there, supposedly attesting to the fact that he paid the workers at the date and time in question.16 xxxx Fourth, respondent Aguilar "solemnly sw[ore] that [he] left DLSU at 5:00 p.m. for Camp Crame for a meeting with some of the officers that we were preparing."17 On May 3, 1995, the DLSU-CSB Joint Discipline Board issued a Resolution18 finding private respondents guilty. They were meted the supreme penalty of automatic expulsion,19 pursuant to CHED Order No. 4.20 The dispositive part of the resolution reads: WHEREFORE, considering all the foregoing, the Board finds respondents ALVIN AGUILAR (AB-BSM/9152105), JAMES PAUL BUNGUBUNG (AB-PSM/9234403), ALVIN LEE (EDD/94623250) and RICHARD V. REVERENTE (AB-MGT/9153837) guilty of having violated CHED Order No. 4 and thereby orders their automatic expulsion. In the case of respondent MALVIN A. PAPIO (AB-MGT/9251227), the Board acquits him of the charge. SO ORDERED.21 Private respondents separately moved for reconsideration22 before the Office of the Senior VicePresident for Internal Operations of DLSU. The motions were all denied in a Letter-Resolution23 dated June 1, 1995. On June 5, 1995, private respondent Aguilar filed with the RTC, Manila, against petitioners a petition for certiorari and injunction under Rule 65 of the Rules of Court with prayer for temporary restraining order (TRO) and/or writ of preliminary injunction. It was docketed as Civil Case No. 95-74122 and assigned to respondent Judge of Branch 36. The petition essentially sought to annul the May 3, 1995 Resolution of the DLSU-CSB Joint Discipline Board and the June 1, 1995 Letter-Resolution of the Office of the Senior Vice-President for Internal Affairs.

The following day, June 6, 1995, respondent Judge issued a TRO24 directing DLSU, its subordinates, agents, representatives and/or other persons acting for and in its behalf to refrain and desist from implementing Resolution dated May 3, 1995 and Letter-Resolution dated June 1, 1995 and to immediately desist from barring the enrollment of Aguilar for the second term of school year (SY) 1995. Subsequently, private respondent Aguilar filed an ex parte motion to amend his petition to correct an allegation in paragraph 3.2125 of his original petition. Respondent Judge amended the TRO26 to conform to the correction made in the amended petition.27 On June 7, 1995, the CHED directed DLSU to furnish it with copies of the case records of Discipline Case No. 9495-3-25121,28 in view of the authority granted to it under Section 77(c) of the Manual of Regulations for Private Schools (MRPS). On the other hand, private respondents Bungubung and Reverente, and later, Valdes, filed petitions-in-intervention29 in Civil Case No. 95-74122. Respondent Judge also issued corresponding temporary restraining orders to compel petitioner DLSU to admit said private respondents. On June 19, 1995, petitioner Sales filed a motion to dismiss30 in behalf of all petitioners, except James Yap. On June 20, 1995, petitioners filed a supplemental motion to dismiss31 the petitionsin-intervention. On September 20, 1995, respondent Judge issued an Order32 denying petitioners' (respondents there) motion to dismiss and its supplement, and granted private respondents' (petitioners there) prayer for a writ of preliminary injunction. The pertinent part of the Order reads: For this purpose, respondent, its agents, representatives or any and all other persons acting for and in its behalf is/are restrained and enjoined from 1. Implementing and enforcing the Resolution dated May 3, 1995 ordering the automatic expulsion of petitioner and the petitioners-in-intervention from the De La Salle University and the letter-resolution dated June 1, 1995, affirming the Resolution dated May 3, 1995; and 2. Barring the enrolment of petitioner and petitioners-in-intervention in the courses offered at respondent De La Salle University and to immediately allow them to enroll and complete their respective courses/degrees until their graduation thereat in accordance with the standards set by the latter. WHEREFORE, the ancillary remedy prayed for is granted. Respondent, its agents, representatives, or any and all persons acting for and its behalf are hereby restrained and enjoyed from:

1. Implementing and enforcing the Resolution dated May 3, 1995 ordering the automatic expulsion of petitioner and petitioners-in-intervention and the LetterResolution dated June 1, 1995; and 2. Barring the enrollment of petitioner and petitioners-in-intervention in the courses offered at respondent (De La Salle University) and to forthwith allow all said petitioner and petitioners-in-intervention to enroll and complete their respective courses/degrees until their graduation thereat. The Writ of Preliminary Injunction shall take effect upon petitioner and petitioners-inintervention posting an injunctive bond in the amount of P15,000.00 executed in favor of respondent to the effect that petitioner and petitioners-in-intervention will pay to respondent all damages that the latter may suffer by reason of the injunction if the Court will finally decide that petitioner and petitioners-in-intervention are not entitled thereto. The motion to dismiss and the supplement thereto is denied for lack of merit. Respondents are directed to file their Answer to the Petition not later than fifteen (15) days from receipt thereof. SO ORDERED.33 Despite the said order, private respondent Aguilar was refused enrollment by petitioner DLSU when he attempted to enroll on September 22, 1995 for the second term of SY 1995-1996. Thus, on September 25, 1995, Aguilar filed with respondent Judge an urgent motion to cite petitioners (respondents there) in contempt of court.34 Aguilar also prayed that petitioners be compelled to enroll him at DLSU in accordance with respondent Judge's Order dated September 20, 1995. On September 25, 1995, respondent Judge issued35 a writ of preliminary injunction, the relevant portion of which reads: IT IS HEREBY ORDERED by the undersigned of the REGIONAL TRIAL COURT OF MANILA that until further orders, you the said DE LA SALLE University as well as your subordinates, agents, representatives, employees and any other person assisting or acting for or on your behalf, to immediately desist from implementing the Resolution dated May 3, 1995 ordering the automatic expulsion of petitioner and the intervenors in DLSU, and the letter-resolution dated June 1, 1995 affirming the said Resolution of May 3, 1995 and to immediately desist from barring the enrolment of petitioner and intervenors in the courses offered at DLSU and to allow them to enroll and complete their degree courses until their graduation from said school.36 On October 16, 1995, petitioner DLSU filed with the CA a petition for certiorari37 (CA-G.R. SP No. 38719) with prayer for a TRO and/or writ of preliminary injunction to enjoin the enforcement of respondent Judge's September 20, 1995 Order and writ of preliminary injunction dated September 25, 1995. On April 12, 1996, the CA granted petitioners' prayer for preliminary injunction.

On May 14, 1996, the CHED issued its questioned Resolution No. 181-96, summarily disapproving the penalty of expulsion for all private respondents. As for Aguilar, he was to be reinstated, while other private respondents were to be excluded.38 The Resolution states: RESOLUTION 181-96 RESOLVED THAT THE REQUEST OF THE DE LA SALLE UNIVERSITY (DLSU), TAFT AVENUE, MANILA FOR THE APPROVAL OF THE PENALTY OF EXPULSION IMPOSED ON MR. ALVIN AGUILAR, JAMES PAUL BUNGUBUNG, ROBERT R. VALDES, JR., ALVIN LEE AND RICHARD V. REVERENTE BE, AS IT IS HEREBY IS, DISAPPROVED. RESOLVED FURTHER, THAT THE COMMISSION DIRECT THE DLSU TO IMMEDIATELY EFFECT THE REINSTATEMENT OF MR. AGUILAR AND THE LOWERING OF THE PENALTY OF MR. JAMES PAUL BUNGUBUNG, MR. ROBER R. VALDEZ, JR., (sic) MR. ALVIN LEE AND MR. RICHARD V. REVERENTE FROM EXPULSION TO EXCLUSION.39 Despite the directive of CHED, petitioner DLSU again prevented private respondent Aguilar from enrolling and/or attending his classes, prompting his lawyer to write several demand letters40 to petitioner DLSU. In view of the refusal of petitioner DLSU to enroll private respondent Aguilar, CHED wrote a letter dated June 26, 1996 addressed to petitioner Quebengco requesting that private respondent Aguilar be allowed to continue attending his classes pending the resolution of its motion for reconsideration of Resolution No. 181-96. However, petitioner Quebengco refused to do so, prompting CHED to promulgate an Order dated September 23, 1996 which states: Acting on the above-mentioned request of Mr. Aguilar through counsel enjoining De La Salle University (DLSU) to comply with CHED Resolution 181-96 (Re: Expulsion Case of Alvin Aguilar, et al. v. DLSU) directing DLSU to reinstate Mr. Aguilar and finding the urgent request as meritorious, there being no other plain and speedy remedy available, considering the set deadline for enrollment this current TRIMESTER, and in order to prevent further prejudice to his rights as a student of the institution, DLSU, through the proper school authorities, is hereby directed to allow Mr. Alvin Aguilar to provisionally enroll, pending the Commission's Resolution of the instant Motion for Reconsideration filed by DLSU. SO ORDERED.41 Notwithstanding the said directive, petitioner DLSU, through petitioner Quebengco, still refused to allow private respondent Aguilar to enroll. Thus, private respondent Aguilar's counsel wrote another demand letter to petitioner DLSU.42 Meanwhile, on June 3, 1996, private respondent Aguilar, using CHED Resolution No. 181-96, filed a motion to dismiss43 in the CA, arguing that CHED Resolution No. 181-96 rendered the CA case moot and academic.

On July 30, 1996, the CA issued its questioned resolution granting the motion to dismiss of private respondent Aguilar, disposing thus: THE FOREGOING CONSIDERED, dismissal of herein petition is hereby directed. SO ORDERED.44 On October 15, 1996, the CA issued its resolution denying petitioners' motion for reconsideration, as follows: It is obvious to Us that CHED Resolution No. 181-96 is immediately executory in character, the pendency of a Motion for Reconsideration notwithstanding. After considering the Opposition and for lack of merit, the Motion for Reconsideration is hereby denied. SO ORDERED.45 On October 28, 1996, petitioners requested transfer of case records to the Department of Education, Culture and Sports (DECS) from the CHED.46 Petitioners claimed that it is the DECS, not CHED, which has jurisdiction over expulsion cases, thus, necessitating the transfer of the case records of Discipline Case No. 9495-3-25121 to the DECS. On November 4, 1996, in view of the dismissal of the petition for certiorari in CA-G.R. SP No. 38719 and the automatic lifting of the writ of preliminary injunction, private respondent Aguilar filed an urgent motion to reiterate writ of preliminary injunction dated September 25, 1995 before respondent RTC Judge of Manila.47 On January 7, 1997, respondent Judge issued its questioned order granting private respondent Aguilar's urgent motion to reiterate preliminary injunction. The pertinent portion of the order reads: In light of the foregoing, petitioner Aguilar's urgent motion to reiterate writ of preliminary injunction is hereby granted, and respondents' motion to dismiss is denied. The writ of preliminary injunction dated September 25, 1995 is declared to be in force and effect. Let a copy of this Order and the writ be served personally by the Court's sheriff upon the respondents at petitioners' expense. SO ORDERED.48 Accordingly, private respondent Aguilar was allowed to conditionally enroll in petitioner DLSU, subject to the continued effectivity of the writ of preliminary injunction dated September 25, 1995 and to the outcome of Civil Case No. 95-74122.

On February 17, 1997, petitioners filed the instant petition. On June 15, 1998, We issued a TRO49 as prayed for by the urgent motion for the issuance of a TRO50 dated June 4, 1998 of petitioners, and enjoined respondent Judge from implementing the writ of preliminary injunction dated September 25, 1995 issued in Civil Case No. 95-74122, effective immediately and until further orders from this Court. On March 27, 2006, private respondent Aguilar filed his manifestation51 stating that he has long completed his course at petitioner DLSU. He finished and passed all his enrolled subjects for the second trimester of 1997-1998, as indicated in his transcript of records52 issued by DLSU. However, despite having completed all the academic requirements for his course, DLSU has not issued a certificate of completion/graduation in his favor. Issues We are tasked to resolve the following issues: 1. Whether it is the DECS or the CHED which has legal authority to review decisions of institutions of higher learning that impose disciplinary action on their students found violating disciplinary rules. 2. Whether or not petitioner DLSU is within its rights in expelling private respondents. 2.a Were private respondents accorded due process of law? 2.b Can petitioner DLSU invoke its right to academic freedom? 2.c Was the guilt of private respondents proven by substantial evidence? 3. Whether or not the penalty imposed by DLSU on private respondents is proportionate to their misdeed. Our Ruling Prefatorily, there is merit in the observation of petitioners53 that while CHED Resolution No. 181-96 disapproved the expulsion of other private respondents, it nonetheless authorized their exclusion from petitioner DLSU. However, because of the dismissal of the CA case, petitioner DLSU is now faced with the spectacle of having two different directives from the CHED and the respondent Judge CHED ordering the exclusion of private respondents Bungubung, Reverente, and Valdes, Jr., and the Judge ordering petitioner DLSU to allow them to enroll and complete their degree courses until their graduation. This is the reason We opt to decide the whole case on the merits, brushing aside technicalities, in order to settle the substantial issues involved. This Court has the power to take cognizance of the petition at bar due to compelling reasons, and the nature and importance of the issues raised warrant the immediate exercise of Our jurisdiction.54 This is in consonance with our case law

now accorded near-religious reverence that rules of procedure are but tools designed to facilitate the attainment of justice, such that when its rigid application tends to frustrate rather than promote substantial justice, this Court has the duty to suspend their operation.55 I. It is the CHED, not DECS, which has the power of supervision and review over disciplinary cases decided by institutions of higher learning. Ang CHED, hindi ang DECS, ang may kapangyarihan ng pagsubaybay at pagrepaso sa mga desisyong pandisiplina ng mga institusyon ng mas mataas na pag-aaral. Petitioners posit that the jurisdiction and duty to review student expulsion cases, even those involving students in secondary and tertiary levels, is vested in the DECS not in the CHED. In support of their stance, petitioners cite Sections 4,56 15(2) & (3),57 54,58 57(3)59 and 7060 of Batas Pambansa (B.P.) Blg. 232, otherwise known as the "Education Act of 1982." According to them, Republic Act (R.A.) No. 7722 did not transfer to the CHED the DECS' power of supervision/review over expulsion cases involving institutions of higher learning. They say that unlike B.P. Blg. 232, R.A. No. 7722 makes no reference to the right and duty of learning institutions to develop moral character and instill discipline among its students. The clear concern of R.A. No. 7722 in the creation of the CHED was academic, i.e., the formulation, recommendation, setting, and development of academic plans, programs and standards for institutions of higher learning. The enumeration of CHED's powers and functions under Section 8 does not include supervisory/review powers in student disciplinary cases. The reference in Section 3 to CHED's "coverage" of institutions of higher education is limited to the powers and functions specified in Section 8. The Bureau of Higher Education, which the CHED has replaced and whose functions and responsibilities it has taken over, never had any authority over student disciplinary cases. We cannot agree. On May 18, 1994, Congress approved R.A. No. 7722, otherwise known as "An Act Creating the Commission on Higher Education, Appropriating Funds Thereof and for other purposes." Section 3 of the said law, which paved the way for the creation of the CHED, provides: Section 3. Creation of the Commission on Higher Education. In pursuance of the abovementioned policies, the Commission on Higher Education is hereby created, hereinafter referred to as Commission. The Commission shall be independent and separate from the Department of Education, Culture and Sports (DECS) and attached to the office of the President for administrative purposes only. Its coverage shall be both public and private institutions of higher education as well as degree-granting programs in all post secondary educational institutions, public and private.

The powers and functions of the CHED are enumerated in Section 8 of R.A. No. 7722. They include the following: Sec. 8. Powers and functions of the Commission. The Commission shall have the following powers and functions: xxxx n) promulgate such rules and regulations and exercise such other powers and functions as may be necessary to carry out effectively the purpose and objectives of this Act; and o) perform such other functions as may be necessary for its effective operations and for the continued enhancement of growth or development of higher education. Clearly, there is no merit in the contention of petitioners that R.A. No. 7722 did not transfer to the CHED the DECS' power of supervision/review over expulsion cases involving institutions of higher learning. First, the foregoing provisions are all-embracing. They make no reservations of powers to the DECS insofar as institutions of higher learning are concerned. They show that the authority and supervision over all public and private institutions of higher education, as well as degreegranting programs in all post-secondary educational institutions, public and private, belong to the CHED, not the DECS. Second, to rule that it is the DECS which has authority to decide disciplinary cases involving students on the tertiary level would render nugatory the coverage of the CHED, which is "both public and private institutions of higher education as well as degree granting programs in all post secondary educational institutions, public and private." That would be absurd. It is of public knowledge that petitioner DLSU is a private educational institution which offers tertiary degree programs. Hence, it is under the CHED authority. Third, the policy of R.A. No. 772261 is not only the protection, fostering and promotion of the right of all citizens to affordable quality education at all levels and the taking of appropriate steps to ensure that education shall be accessible to all. The law is likewise concerned with ensuring and protecting academic freedom and with promoting its exercise and observance for the continued intellectual growth of students, the advancement of learning and research, the development of responsible and effective leadership, the education of high-level and middlelevel professionals, and the enrichment of our historical and cultural heritage. It is thus safe to assume that when Congress passed R.A. No. 7722, its members were aware that disciplinary cases involving students on the tertiary level would continue to arise in the future, which would call for the invocation and exercise of institutions of higher learning of their right to academic freedom.

Fourth, petitioner DLSU cited no authority in its bare claim that the Bureau of Higher Education, which CHED replaced, never had authority over student disciplinary cases. In fact, the responsibilities of other government entities having functions similar to those of the CHED were transferred to the CHED.62 Section 77 of the MRPS63 on the process of review in student discipline cases should therefore be read in conjunction with the provisions of R.A. No. 7722. Fifth, Section 18 of R.A. No. 7722 is very clear in stating that "[j]urisdiction over DECSsupervised or chartered state-supported post-secondary degree-granting vocational and tertiary institutions shall be transferred to the Commission [On Higher Education]." This provision does not limit or distinguish that what is being transferred to the CHED is merely the formulation, recommendation, setting and development of academic plans, programs and standards for institutions of higher learning, as what petitioners would have us believe as the only concerns of R.A. No. 7722. Ubi lex non distinguit nec nos distinguere debemus: Where the law does not distinguish, neither should we. To Our mind, this provision, if not an explicit grant of jurisdiction to the CHED, necessarily includes the transfer to the CHED of any jurisdiction which the DECS might have possessed by virtue of B.P. Blg. 232 or any other law or rule for that matter. IIa. Private respondents were accorded due process of law. Ang mga private respondents ay nabigyan ng tamang proseso ng batas. The Due Process Clause in Article III, Section 1 of the Constitution embodies a system of rights based on moral principles so deeply imbedded in the traditions and feelings of our people as to be deemed fundamental to a civilized society as conceived by our entire history.64 The constitutional behest that no person shall be deprived of life, liberty or property without due process of law is solemn and inflexible.65 In administrative cases, such as investigations of students found violating school discipline, "[t]here are withal minimum standards which must be met before to satisfy the demands of procedural due process and these are: that (1) the students must be informed in writing of the nature and cause of any accusation against them; (2) they shall have the right to answer the charges against them and with the assistance if counsel, if desired; (3) they shall be informed of the evidence against them; (4) they shall have the right to adduce evidence in their own behalf; and (5) the evidence must be duly considered by the investigating committee or official designated by the school authorities to hear and decide the case."66 Where a party was afforded an opportunity to participate in the proceedings but failed to do so, he cannot complain of deprivation of due process.67 Notice and hearing is the bulwark of administrative due process, the right to which is among the primary rights that must be respected even in administrative proceedings.68 The essence of due process is simply an opportunity to be heard, or as applied to administrative proceedings, an opportunity to explain one's side or an opportunity to seek reconsideration of the action or ruling complained of.69 So long as the party

is given the opportunity to advocate her cause or defend her interest in due course, it cannot be said that there was denial of due process.70 A formal trial-type hearing is not, at all times and in all instances, essential to due process it is enough that the parties are given a fair and reasonable opportunity to explain their respective sides of the controversy and to present supporting evidence on which a fair decision can be based.71 "To be heard" does not only mean presentation of testimonial evidence in court one may also be heard through pleadings and where the opportunity to be heard through pleadings is accorded, there is no denial of due process.72 Private respondents were duly informed in writing of the charges against them by the DLSUCSB Joint Discipline Board through petitioner Sales. They were given the opportunity to answer the charges against them as they, in fact, submitted their respective answers. They were also informed of the evidence presented against them as they attended all the hearings before the Board. Moreover, private respondents were given the right to adduce evidence on their behalf and they did. Lastly, the Discipline Board considered all the pieces of evidence submitted to it by all the parties before rendering its resolution in Discipline Case No. 9495-3-25121. Private respondents cannot claim that they were denied due process when they were not allowed to cross-examine the witnesses against them. This argument was already rejected in Guzman v. National University73 where this Court held that "x x x the imposition of disciplinary sanctions requires observance of procedural due process. And it bears stressing that due process in disciplinary cases involving students does not entail proceedings and hearings similar to those prescribed for actions and proceedings in courts of justice. The proceedings in student discipline cases may be summary; and cross examination is not, x x x an essential part thereof." IIb. Petitioner DLSU, as an institution of higher learning, possesses academic freedom which includes determination of who to admit for study. Ang petitioner DLSU, bilang institusyon ng mas mataas na pag-aaral, ay nagtataglay ng kalayaang akademiko na sakop ang karapatang pumili ng mga mag-aaral dito. Section 5(2), Article XIV of the Constitution guaranties all institutions of higher learning academic freedom. This institutional academic freedom includes the right of the school or college to decide for itself, its aims and objectives, and how best to attain them free from outside coercion or interference save possibly when the overriding public interest calls for some restraint.74 According to present jurisprudence, academic freedom encompasses the independence of an academic institution to determine for itself (1) who may teach, (2) what may be taught, (3) how it shall teach, and (4) who may be admitted to study.75 It cannot be gainsaid that "the school has an interest in teaching the student discipline, a necessary, if not indispensable, value in any field of learning. By instilling discipline, the school teaches discipline. Accordingly, the right to discipline the student likewise finds basis in the freedom "what to teach."76 Indeed, while it is categorically stated under the Education Act of 1982 that students have a right "to freely choose their field of study, subject to existing curricula and to continue their course therein up to graduation,"77 such right is subject to the established

academic and disciplinary standards laid down by the academic institution. Petitioner DLSU, therefore, can very well exercise its academic freedom, which includes its free choice of students for admission to its school. IIc. The guilt of private respondents Bungubung, Reverente and Valdes, Jr. was proven by substantial evidence. Ang pagkakasala ng private respondents na sina Bungubung, Reverente at Valdes, Jr. ay napatunayan ng ebidensiyang substansyal. As has been stated earlier, private respondents interposed the common defense of alibi. However, in order that alibi may succeed as a defense, "the accused must establish by clear and convincing evidence (a) his presence at another place at the time of the perpetration of the offense and (b) the physical impossibility of his presence at the scene of the crime."78 On the other hand, the defense of alibi may not be successfully invoked where the identity of the assailant has been established by witnesses.79 Positive identification of accused where categorical and consistent, without any showing of ill motive on the part of the eyewitness testifying, should prevail over the alibi and denial of appellants whose testimonies are not substantiated by clear and convincing evidence.80 Well-settled is the rule that denial and alibi, being weak defenses, cannot overcome the positive testimonies of the offended parties.81 Courts reject alibi when there are credible eyewitnesses to the crime who can positively identify the accused.82 Alibi is an inherently weak defense and courts must receive it with caution because one can easily fabricate an alibi.83 Jurisprudence holds that denial, like alibi, is inherently weak and crumbles in light of positive declarations of truthful witnesses who testified on affirmative matters that accused were at the scene of the crime and were the victim's assailants. As between categorical testimonies that ring of truth on one hand and a bare denial on the other, the former must prevail.84 Alibi is the weakest of all defenses for it is easy to fabricate and difficult to disprove, and it is for this reason that it cannot prevail over the positive identification of accused by the witnesses.85 The required proof in administrative cases, such as in student discipline cases, is neither proof beyond reasonable doubt nor preponderance of evidence but only substantial evidence. According to Ang Tibay v. Court of Industrial Relations,86 it means "such reasonable evidence as a reasonable mind might accept as adequate to support a conclusion." Viewed from the foregoing, We reject the alibi of private respondents Bungubung, Valdes Jr., and Reverente.1awphi1 They were unable to show convincingly that they were not at the scene of the crime on March 29, 1995 and that it was impossible for them to have been there. Moreover, their alibi cannot prevail over their positive identification by the victims. We hark back to this Court's pronouncement affirming the expulsion of several students found guilty of hazing:

No one can be so myopic as to doubt that the immediate reinstatement of respondent students who have been investigated and found guilty by the Disciplinary Board to have violated petitioner university's disciplinary rules and standards will certainly undermine the authority of the administration of the school. This we would be most loathe to do. More importantly, it will seriously impair petitioner university's academic freedom which has been enshrined in the 1935, 1973 and the present 1987 Constitution.87 Certainly, private respondents Bungubung, Reverente and Valdes, Jr. do not deserve to claim a venerable institution as their own, for they may foreseeably cast a malevolent influence on the students currently enrolled, as well as those who come after them.88 It must be borne in mind that universities are established, not merely to develop the intellect and skills of the studentry, but to inculcate lofty values, ideals and attitudes; nay, the development, or flowering if you will, of the total man.89 As for private respondent Aguilar, however, We are inclined to give credence to his alibi that he was at Camp Crame in Quezon City at the time of the incident in question on March 29, 1995. This claim was amply corroborated by the certification that he submitted before the DLSU-CSB Joint Discipline Board, to wit: CERTIFICATION TO WHOM THIS MAY CONCERN: We, the undersigned, hereby declare and affirm by way of this Certification that sometime on March 29, 1995, at about and between 4:30 P.M. and 5:30 P.M., we were together with Alvin A. Aguilar, at Kiangan Hall, inside Camp Crame, Quezon City, meeting in connection with an affair of our class known as Class 7, Batch 89 of the Philippine Constabulary discussing on the proposed sponsorship of TAU GAMMA PHI from said Batch '89 affair. That the meeting was terminated at about 6:30 P.M. that evening and Alvin Aguilar had asked our permission to leave and we saw him leave Camp Crame, in his car with the driver. April 18, 1995, Camp Crame, Quezon City.90 The said certification was duly signed by PO3 Nicanor R. Faustino (Anti-Organized Crime CIC, NCR), PO3 Alejandro D. Deluviar (ODITRM, Camp Crame, Quezon City), PO2 Severino C. Filler (TNTSC, Camp Crame, Quezon City), and PO3 Ireneo M. Desesto (Supply Center, PNPLSS). The rule is that alibi assumes significance or strength when it is amply corroborated by credible and disinterested witnesses.91 It is true that alibi is a weak defense which an accused can easily fabricate to escape criminal liability. But where the prosecution evidence is weak, and betrays lack of credibility as to the identification of defendant, alibi assumes commensurate strength. This is but consistent with the presumption of innocence in favor of accused.92

Alibi is not always undeserving of credit, for there are times when accused has no other possible defense for what could really be the truth as to his whereabouts at the crucial time, and such defense may, in fact, tilt the scales of justice in his favor.93 III. The penalty of expulsion imposed by DLSU on private respondents is disproportionate to their misdeed. Ang parusang expulsion na ipinataw ng DLSU sa private respondents ay hindi angkop sa kanilang pagkakasala. It is true that schools have the power to instill discipline in their students as subsumed in their academic freedom and that "the establishment of rules governing university-student relations, particularly those pertaining to student discipline, may be regarded as vital, not merely to the smooth and efficient operation of the institution, but to its very survival."94 This power, however, does not give them the untrammeled discretion to impose a penalty which is not commensurate with the gravity of the misdeed. If the concept of proportionality between the offense committed and the sanction imposed is not followed, an element of arbitrariness intrudes. That would give rise to a due process question.95 We agree with respondent CHED that under the circumstances, the penalty of expulsion is grossly disproportionate to the gravity of the acts committed by private respondents Bungubung, Reverente, and Valdes, Jr. Each of the two mauling incidents lasted only for few seconds and the victims did not suffer any serious injury. Disciplinary measures especially where they involve suspension, dismissal or expulsion, cut significantly into the future of a student. They attach to him for life and become a mortgage of his future, hardly redeemable in certain cases. Officials of colleges and universities must be anxious to protect it, conscious of the fact that, appropriately construed, a disciplinary action should be treated as an educational tool rather than a punitive measure.96 Accordingly, We affirm the penalty of exclusion97 only, not expulsion,98 imposed on them by the CHED. As such, pursuant to Section 77(b) of the MRPS, petitioner DLSU may exclude or drop the names of the said private respondents from its rolls for being undesirable, and transfer credentials immediately issued. WHEREFORE, the petition is PARTIALLY GRANTED. The Court of Appeals Resolutions dated July 30, 1996 and dated October 15, 1996, and Regional Trial Court of Manila, Branch 36, Order dated January 7, 1997 are ANNULLED AND SET ASIDE, while CHED Resolution 18196 dated May 14, 1996 is AFFIRMED. Petitioner DLSU is ordered to issue a certificate of completion/graduation in favor of private respondent Aguilar. On the other hand, it may exclude or drop the names of private respondents Bungubung, Reverente, and Valdes, Jr. from its rolls, and their transfer credentials immediately issued. SO ORDERED.

G.R. Nos. 164684-85 November 11, 2005 PHILIPPINE LONG DISTANCE TELEPHONE COMPANY, INC., Petitioner, vs. ANTONIO Q. TIAMSON, Respondent. DECISION CALLEJO, SR., J.: Being questioned in this petition for review on certiorari is the Decision1 of the Court of Appeals (CA) dated April 16, 2004 in CA-G.R. SP Nos. 51855 and 52247, and the Resolution dated July 27, 2004 denying the motion for reconsideration thereof. On April 16, 1986, the Philippine Long Distance Telephone Company, Inc. (PLDT) employed Antonio Q. Tiamson as a Radio Technician II (JG4). He was assigned at the companys North Luzon Toll Network Division, Clark Transmission Maintenance Center (Clark-TMC) in Pampanga. After the expiration of the probationary period, he was extended regular appointment for the same position. In a Letter2 dated July 29, 1994, Anthony Dy Dee, the President of the Angeles City Telephone System and Datelcom Corporation, informed PLDT of his complaint against its employees assigned in Clark-TMC, stating therein that he suspected them to be in cohorts with the local subscribers in effecting illegal overseas calls. Acting on the letter-complaint, PLDT immediately dispatched a team of inspectors and investigators from its Quality Control and Inspection Department (QCID) and Security Division to conduct surveillance operations in the area. On August 2, 1994, Vidal Busa, a radio technician, was caught in flagrante delicto while monitoring an illegally connected overseas call using the radio facilities of the companys Clark-TMC Radio Room.3 The QCID, likewise, requested the Switching Network Division at PLDTs Sampaloc National Toll Center to print the CAMA4 tape recording of all long distance calls originating from the PLDT Clark Exchange Traffic for the period of July 29 to August 2, 1994. The printout revealed that a total of 469 fraudulent overseas and local calls were connected and completed at the PLDT Clark-TMC Radio Room for the said period. Three overseas calls to Saudi Arabia made on August 1, 1994 were imputed to Tiamson who appeared to be on duty from 10:00 p.m. to 6:00 a.m.5 The QCID conducted its initial investigation on August 2, 1994, where Busa readily admitted his involvement in the illegal connection of overseas calls. In his sworn statement, he specifically named Arnel Cayanan, his Shift Supervisor, Antonio Tiamson and Paul Cruzada, both radio technicians, as the other employees actively engaged in the illegal practice. He stated that he knew about this because whenever he would relieve them from their tour of duty, he would see that the circuit was engaged.6

On August 3, 1994, during a confrontation between Busa and Tiamson, the former reiterated his earlier statement that the latter was involved in the illegal act of connecting overseas calls.7 For his part, Tiamson admitted that he knew how to make an overseas call using the companys radio equipment and that he learned how to do so through hands-on experimentation and intensive reading of operating manuals. He, however, denied having actually made an illegal connection of overseas calls. He declared that he knew of the wrongdoings of Busa and even disconnected the latters overseas telephone calls whenever he (Tiamson) was on duty. Tiamson claimed that he failed to report the actuations of Busa because the latter was his supervisor and was afraid to antagonize him.8 On August 5, 1994, there was another confrontation proceeding between Busa, Tiamson, Cruzada and Cayanan. In their sworn statements, Busa and Cruzada testified that, sometimes when they relieve Cayanan from his duty, they would discover an illegal connection and an ongoing conversation in the line.9 Tiamson maintained that he disconnected the illegal calls of Busa, while Cayanan implicated his subordinates. The QCID recommended that administrative action for serious misconduct be instituted against the said employees. Consequently, the company issued to Tiamson an Inter-Office Memorandum dated August 12, 1994, charging him with violation of the companys disciplinary rules and regulations. He was, likewise, required to explain within 72 hours why he should not be dismissed, thus: Investigation of the complaint indicated hereunder disclosed that: 1. Complainant Mr. Anthony Dy, President DATELCOM Corp. 2. The decrease of toll revenue for DATELCOM Angeles/Mabalacat Exchange due to fraudulent overseas call scam was complained and notified by Mr. A. Dy to Mrs. B. G. Gendrano Clark Exchange Division Head on July 26, 1994. 3. The complainant requested assistance to NBI and PLDT QCI to apprehend the personnel responsible for the illegal connection. 4. A clue was provided by Mr. Anthony Dy that the illegal overseas call was coming from ClarkTMC through taped and equipment monitoring. 5. In the QCI investigation, you were implicated by your fellow Radio Technician Mr. Vidal C. Busa as involved in the case. You admitted you know how to operate the Lenkurt 26600 Signalling Test Set to initiate a call but denied doing it for personal gain or interest but you failed to report the anomaly to your superior as one of your supervisors was involved in the fraudulent case. The acts described above are in violation of the Companys rules and regulations and is punishable with dismissal from the service.

In view of the above, please explain in writing within 72 hours from receipt hereof why you should not be dismissed from the service for the acts described above. You may elect to be heard if you so desire. 10 Meanwhile, Tiamson was placed under preventive suspension on August 16, 1994.11 On August 18, 1994, Tiamson submitted his written explanation denying any participation in the illegal activities at PLDTs Clark-TMC. He averred that Busas statement against him was malicious and untrue and that he was the one relieving Busa from his tour of duty and not the other way around. He insisted that on August 1, 1994, his tour of duty was from 6:00 a.m. to 10:00 p.m.12 PLDT found his explanation unsatisfactory and inadequate in substance. Thus, it issued an InterOffice Memo13 dated October 5, 1994, terminating Tiamsons employment effective October 7, 1994 on the ground of serious misconduct and/or fraud. Tiamson filed a complaint against PLDT for illegal suspension, illegal dismissal, damages and other monetary claims, docketed as NLRC Case No. RAB-III-07-6414-95. The Labor Arbiter resolved the case in favor of Tiamson: WHEREFORE, premises considered, judgment is hereby rendered declaring respondent PLDT guilty of illegal dismissal and it is hereby ordered to reinstate complainant to his former position without loss of seniority rights and with full backwages reckoned from the date of his dismissal up to his actual or payroll reinstatement at the option of the respondent, which as of this date is in the amount of Three Hundred Seventy-Two Thousand Eight Hundred Twenty-Five and 32/100 (P372,825.32) Pesos. Further, respondent is ordered to pay complainant attorneys fee in the amount of Thirty-Seven Thousand Two Hundred Eighty-Two and 53/100 (P37,282.53) Pesos. The claims for moral and exemplary damages are dismissed for lack of evidence. SO ORDERED.14 The Labor Arbiter declared that the complainant could not have made any illegal connection on August 1, 1994 from 10:00 p.m. to 6:00 a.m. because he was off-duty. PLDT elevated the case to the National Labor Relations Commission (NLRC). On August 31, 1998, the NLRC ruled that while there was just cause for Tiamsons dismissal, the penalty of dismissal was too harsh. Hence, the NLRC ordered that Tiamson be reinstated to his former position without loss of seniority rights, but without backwages.15 Both parties moved to reconsider the decision, but the NLRC denied the motions for lack of merit.16

PLDT filed a petition for certiorari before the CA, assailing the NLRCs order of reinstatement despite a categorical finding that Tiamson was guilty of illegal connection of overseas calls. The petition was docketed as CA-G.R. SP No. 51855. Tiamson filed a similar petition, assailing the deletion of the award of backwages and attorneys fees. This was docketed as CA-G.R. SP No. 52247. The CA, thereafter, ordered the consolidation of the two petitions. On April 16, 2004, the CA reinstated the decision of the Labor Arbiter, thus: WHEREFORE, the petition by the PLDT under CA-G.R. SP No. 51855 is DENIED DUE COURSE and DISMISSED while the petition by Antonio Tiamson under CA-G.R. SP No. 52247 is GIVEN DUE COURSE and GRANTED, and the Decision dated October 15, 1997 of the Labor Arbiter which was set aside by the NLRC, is hereby REINSTATED in its fullness and without modifications. SO ORDERED.17 The CA held that Busas sworn statement was not worthy of credence, a mere afterthought, the contents of which were seriously flawed. The appellate court found it difficult to believe Busas assertion that, on several occasions when he came to relieve the respondent, a circuit was in use which the latter would turn off before leaving. In this regard, the appellate court noted that Busas work shift preceded that of the respondent, such that it would be impossible for him to see the respondent make an illegal connection.18 The CA likewise opined that the respondent was denied due process when he was not apprised of nor given the opportunity to confute the charge that during his duty on August 1, 1994, three overseas calls to Saudi Arabia were recorded in the CAMA tape.19 The petitioner timely filed a motion for reconsideration, which the CA denied in its Resolution20 dated July 27, 2004. The petitioner now comes before this Court, alleging that: THE COURT OF APPEALS COMMITTED SERIOUS ERROR IN REINSTATING THE DECISION OF THE ARBITER A QUO AS SAID DECISION WAS NOT IN ACCORD WITH LAW AND CONTRARY TO THE EVIDENCE ON RECORD.21 The petitioner submits that it has presented more than substantial evidence to prove that the respondent was involved in the illegal connection of overseas calls. The petitioner avers that the CA erred in holding that Busas sworn statement was not credible. According to the CA, it would have been impossible for Busa to see the respondent making an illegal connection since his tour of duty preceded that of the respondent. The petitioner, however, asserts that there was a rotation of the employees tour of duty such that, at times, it was Busa who would take over from the respondent; hence, Busa had the occasion to personally see the respondent connecting illegal calls. In support of this, the petitioner proffers the copy of logbook entries from July 13 to August 3, 1994, which was attached to its Memorandum of Appeal filed with the NLRC. The logbook shows that on several occasions, it was Busa who took over from the respondent.22

The petitioner further asserts that the respondent failed to show that Busa was actuated and impelled by improper motive and bad faith in executing his sworn statement.23 The records show that Busa, from the very start, had categorically and unequivocally named the respondent as one of those engaged in the illegal connection of overseas calls.24 Moreover, Busas sworn statement had been corroborated by the printout of the CAMA tapes (which disclosed that during the respondents August 1, 1994 duty, three fraudulent calls to Saudi Arabia were illegally made),25 as well as Cayanans sworn statement implicating the respondent.26 The petitioner submits that the respondents offense was serious in character and merits the penalty of dismissal from employment. It contends that the respondent was accorded the full measure of due process before he was dismissed: he was given a notice which apprised him of the charge against him and required him to explain why he should not be dismissed, and later, a notice of termination. The petitioner claims that the Labor Code simply requires that the employee be given a written notice containing a statement of the causes of termination. It insists that the printout of the recording of the CAMA tapes showing that three illegal connections were made on August 1, 1994 is a mere evidentiary matter that need not be mentioned in the notice.27 For his part, the respondent avers that Busas statement was uncorroborated and hearsay for lack of cross-examination. He insists that Busa could not have seen him make illegal connections since the latters shift came before his.28 The petitioner replies that an affidavit may be admissible even if the witness is not presented during trial because technical rules are not strictly followed in proceedings before the Labor Arbiter and the NLRC.29 The petition has no merit. It is a settled rule that factual findings of labor officials, who are deemed to have acquired expertise in matters within their respective jurisdictions, are generally accorded not only respect but even finality.30 Moreover, in a petition for review on certiorari under Rule 45, the Supreme Court reviews only errors of law and not errors of facts.31 However, where there is divergence in the findings and conclusions of the NLRC, on the one hand, from those of the Labor Arbiter and the Court of Appeals, on the other, the Court is constrained to examine the evidence.32 In termination cases, the burden of proof rests upon the employer to show that the dismissal is for just and valid cause; failure to do so would necessarily mean that the dismissal was illegal.33 The employers case succeeds or fails on the strength of its evidence and not on the weakness of the employees defense. If doubt exists between the evidence presented by the employer and the employee, the scales of justice must be tilted in favor of the latter.34 Moreover, the quantum of proof required in determining the legality of an employees dismissal is only substantial evidence. Substantial evidence is more than a mere scintilla of evidence or relevant evidence as a reasonable mind might accept as adequate to support a conclusion, even if other minds, equally reasonable, might conceivably opine otherwise.35 In this case, the appellate court ruled for respondent Tiamson, ratiocinating as follows:

The issues posed by both parties involve the evaluation of the findings of facts by the agencies a quo. While the general rule is that factual issues could not be properly raised and considered in a petition for certiorari, it however admits of this exception that a disharmony between the factual findings of the Labor Arbiter and those of the NLRC opens the door to review thereof by the Supreme Court (Asuncion vs. National Labor Relations Commission, 362 SCRA 56), including, of course, the Court of Appeals. The crux of both petitions is whether the NLRC with its findings quoted below, was correct in setting aside the disposition of the Labor Arbiter: We disagree that respondent failed to present evidence linking complainant to the illegal connection scam. As pointed out by the respondent, co-employee Busa and Cayanan in the course of their investigation implicated complainants participation in illegal overseas connection. Complainant also failed to refute respondents evidence that on August 1, 1994, while he was on duty, three (3) overseas calls to Saudi Arabia were recorded in cama tape (Annex 4, p. 30, records). However, we consider the penalty of dismissal too harsh considering that respondent imposed a sixty (60)-day suspension on Paul Cruzada, a co-employee of complainant who submitted (sic) culpability. For where a lesser punitive penalty would suffice, the supreme penalty of dismissal should be visited (Almira vs. B.F. Goodrich, 58 SCRA 120). Under the circumstances, reinstatement but without backwages is appropriate (pp. 39-40, Rollo) Our review of the records reveals that among the three employees who issued sworn statements, namely, Busa, Cayanan and Cruzada, it was only Busa who directly implicated Tiamson and it was done inexplicably only in his second sworn statement. It does not inspire credence as it comes as an afterthought and the contents are seriously flawed on material points. Looming large is the claim of Busa that on several occasions when he came to relieve Tiamson, he observed that his circuit was logged on and in use, and Tiamson would then put it off before leaving. This is a canard because the shift of Busa was from 1:00 p.m. to 6:00 a.m. and of course ahead of the 6:00 a.m. to 2:00 p.m. shift of Tiamson who came in as his reliever. Their tours of duty was in the converse order of what Busa claimed, and so he spoke with a forked tongue when he stated that Tiamson at the preceding shift had his circuit logged on and switched this off when he left. A no less important point is the undisputed fact that Tiamson was not given the opportunity to confute the charge that on August 1, 1994 while he was on duty, three (3) overseas calls to Saudi Arabia were recorded in the cama tape. This was not indicated in the memorandum sent to him on August 12, 1994, the full text of which reads: August 12, 1994 TO : MR. ANTONIO Q. TIAMSON Radio Tech II Clark TMC FROM : Division Head, North Luzon Toll Network SUBJECT: ADMINISTRATIVE CASE

--------------------------------------------Investigation of the complaint indicated hereunder disclosed that: 1. Complainant Mr. Anthony Dy, President DATELCOM Corp. 2. The decrease of toll revenue for DATELCOM Angeles/Mabalacat Exchange due to fraudulent overseas call scam was complained and notified by Mr. A. Dy to Mrs. H. G. Gendrano Clark Exchange Division Head on July 26, 1994. 3. The complainant requested assistance to NBI and PLDT QCI to apprehend the personnel responsible for the illegal connection. 4. A clue was provided by Mr. Anthony Dy that the illegal overseas call was coming from ClarkTMC through taped and equipment monitoring. 5. In the QCI investigation, you were implicated by your fellow Radio Technician Mr. Vidal C. Busa as involved in the case. You admitted you know how to operate the Lenkurt 26600 Signalling Test Set to initiate a call but denied doing it for personal gain or interest but you failed to report the anomaly to your superior as one of your supervisors was involved in the fraudulent case. The acts described above are in violation of the Companys rules and regulations and is punishable with dismissal from the service. In view of the above, please explain in writing within 72 hours from receipt hereof why you should not be dismissed from the service for the acts described above. You may elect to be heard if you so desire. Please be informed also that you will be placed under preventive suspension which will take effect on August 16, 1994 pending resolution of the case. If no written explanation is received from you within the said period of 72 hours, this case will be decided on the basis of the evidence on hand. (p. 227, Rollo) (SGD.) ARMANDO A. ABESAMIS Procedural due process requires that an employee be apprised of the charge against him, given reasonable time to answer the same, allowed ample opportunity to be heard and defend himself, and assisted by a representative if the employee so desires (Concorde Hotel vs. Court of Appeals, 362 SCRA 583; underlining supplied). Procedural due process requires that the employer serve the employees to be dismissed two (2) written notices before the termination of their employment is effected: (a) the first, to apprise them of the particular acts or omission for which their dismissal is sought; and (b) second, to inform them of the decision of the employer

that they are being dismissed (Perpetual Help Credit Cooperative, Inc. vs. Faburada, 366 SCRA 693; underlining supplied). The Labor Arbiter, therefore, was correct in ruling that Tiamson was indeed illegally dismissed from his employment.36 The petitioner maintains that contrary to the findings and conclusions of the appellate court, it has established through substantial evidence that there was just cause for the respondents dismissal. To bolster such contention, the petitioner adduces the following documentary evidences: (1) the sworn statements of Vidal Busa specifically implicating the respondent; (2) the sworn statement of Arnel Cayanan; and (3) the printout of the CAMA tape, recording the unauthorized overseas calls originating from Clark-TMC during the respondents tour of duty. The respondent disputes the admissibility of Busas sworn statements for being hearsay since the latter was not presented for cross-examination. This argument, however, is not persuasive because the rules of evidence are not strictly observed in proceedings before administrative bodies like the NLRC where decisions may be reached on the basis of position papers only.37 The Court agrees with the contentions of the respondent and the findings and rulings of the CA. The petitioner indeed failed to adduce substantial evidence to prove that the dismissal of the respondent was for a just cause. In his first sworn statement, Busa implicated the respondent in the illegal connections of overseas calls in this manner: T 25 - Bukod sa iyo, sinu-sino pa sa mga kasamahan mo ang tinuruan ni Mr. Cayanan ng sistemang ito? S - Sina Antonio Tiamson at Paul Cruzada na pawang mga Radio Technicians din. T 26 - Ang ibig mo sabihin, ginagawa din nina Mr. Tiamson at Cruzada ang magpa-patch ng mga tawag sa abroad o overseas? S - Opo. T 27 - Paano mo naman nasisiguro ito? S - Nakikita ko po. T 28 - Paano mo naman nakita samantalang magka-iba ang tour of duty ninyo? S - Pag nag-relyebo kami ay naaabutan kong naka-engage ang circuit at pag tinanong ko ay sinasabi nga nilang may tawag sila at kasalukuyang nag-uusap ang magkabilang parties.38 During the confrontation between Busa and the respondent, the former likewise made the following statements:

T 3 - Ayon sa iyo, ginagawa rin ni Mr. Tiamson ang magku-kunekta ng mga illegal na tawag overseas sa pamamagitan ng pag-gamit ng inyong Radio Equipment. Tama ba ito? S - Tama po, Sir. T 4 - Paano mo nalaman na ginagawa rin ni Mr. Tiamson ito? S - Dahil nakikita ko siyang nagkukunekta at ilang beses ko ring nadatnan kapag nag-relyebo kami na gumagana ang circuit na ang ibig sabihin ay may nag-uusap. At bago siya aalis ay inilalagay niya sa normal position ang linyang ginamit niya. T 5 - Kailan pa ito gingawa ni Mr. Tiamson kung natatandaan mo pa? S - Sa natatandaan ko ginagawa niya ito magmula noong 1992 pa. T 6 - Ayon pa rin sa iyo, alam din ni Mr. Tiamson na ginagawa rin ni Mr. Cayanan itong mga illegal activities na ito. Paano mo nasabi na alam ni Mr. Tiamson itong ginagawa ni Mr. Cayanan S - Kasi magkakasama kami at kaming apat lang nina Mr. Cayanan, Mr.Tiamson, Mr. Cruzada at ako ang nakaka-alam niyang operation na iyan.39 On the other hand, during the confrontation among all four employees implicated in the matter, Cayanan testified that he was aware that his "subordinates" were engaged in illegal activities. However, he failed to specifically mention who these subordinates were.40 Although admissible in evidence, affidavits being self-serving must be received with caution. This is because the adverse party is not afforded any opportunity to test their veracity.41 By themselves, generalized and pro forma affidavits cannot constitute relevant evidence which a reasonable mind may accept as adequate.42 There must be some other relevant evidence to corroborate such affidavits. On this point, the petitioner submits that the printout of the CAMA tapes corroborated Busas sworn statement. A perusal of the printout, however, shows that it is not authenticated by the proper officer of the company. Moreover, the name of the respondent and the other annotations in the said printout are handwritten and unsigned. The ruling in Asuncion v. National Labor Relations Commission43 is instructive on how such document should be treated. In that case, the employer submitted a handwritten listing and computer printouts to establish the charges against the employee. The handwritten listing was not signed, and while there was a computer-generated listing, the entries of time and other annotations therein were also handwritten and unsigned. The Court ruled that the handwritten listing and unsigned computer printouts were unauthenticated, hence, unreliable. Mere selfserving evidence (of which the listing and printouts are of that nature) should be rejected as evidence without any rational probative value even in administrative proceedings.44

Thus, in Uichico v. National Labor Relations Commission,45 the Court elucidated the extent of the liberality of procedure in administrative actions: It is true that administrative and quasi-judicial bodies like the NLRC are not bound by the technical rules of procedure in the adjudication of cases. However, this procedural rule should not be construed as a license to disregard certain fundamental evidentiary rules. While the rules of evidence prevailing in the courts of law or equity are not controlling in proceedings before the NLRC, the evidence presented before it must at least have a modicum of admissibility for it to be given some probative value. 46 The decisions of this Court, while adhering to a liberal view in the conduct of proceedings before administrative agencies, have nonetheless consistently required some proof of authenticity or reliability as a condition for the admission of documents.47 Absent any such proof of authenticity, the printout of the CAMA tape should be considered inadmissible, hence, without any probative weight. To conclude, the petitioner has not established by substantial evidence that there was just cause for the respondents termination from his employment. The sworn statements of Busa and Cayanan alone are not sufficient to establish that the respondent was guilty of serious misconduct. In light of such finding, there is no need to delve into whether or not the respondent was afforded due process when he was dismissed by the petitioner. WHEREFORE, premises considered, the petition is DENIED DUE COURSE. The Decision of the Court of Appeals dated April 16, 2004, and its Resolution dated July 27, 2004 in CA-G.R. SP Nos. 51855 and 52247 are AFFIRMED. SO ORDERED.

(MGG Marine Services, Inc., et al. vs. NLRC and E.A. Molina, G.R. No. 114313, July 29, 1996) As Mr. Justice Panganiban, in an en banc decision, states: To constitute a completely valid and faultless dismissal, it is well-settled that the employer must show not only sufficient ground therefor, but it must also prove that it observed procedural due process by giving the employee two notices: one, of the intention to dismiss, indicating therein his acts or omissions complained against, and two, notice of the decision to dismiss; and an opportunity to answer and rebut the charges against him, in between such notices.

G.R. No. 111173 September 4, 1996 PHILIPPINE SAVINGS BANK, petitioner, vs. NATIONAL LABOR RELATIONS COMMISSION and VICTORIA T. CENTENO, respondents.

MENDOZA, J.: This is a petition for certiorari to annul the decision of the National Labor Relations Commission in NLRC Case No. RB-IV-2-1554-85, affirming the decision of the Labor Arbiter, which found petitioner guilty of illegal dismissal, and the resolution of the NLRC denying reconsideration. The facts are as follows: Private respondent Victoria T. Centeno started, as a bank teller of petitioner Philippine Savings Bank, on November 3, 1965. Through the years she was promoted, becoming on February 4, 1985, assistant cashier of petitioner's Taytay branch, at a salary of P2,672.00 a month. From September 17, 1984 to November 15, 1984, private respondent was acting branch cashier, substituting for Mrs. Victoria Ubaa, who had gone on maternity leave. As acting branch cashier, private respondent was in charge of the cash in the vault and the preparation of the daily cash proof sheet, which was a daily record of the cash in the vault and was used as basis in determining the starting balance on the next banking day. On November 16, 1984, Mrs. Victoria Ubaa reported back to work. Before turning over the cash to Mrs. Ubaa, private respondent Centeno deposited P356,400.00 in the Metropolitan Bank and Trust Co. (Metrobank). However, what appeared as amount deposited in the November 16, 1984 cash proof and batch sheets of the cashier and clearing clerk, was P371,400.00, and not P356,400.00 as shown in the Metrobank passbook. Petitioner later charged that private respondent falsified the deposit slip and made it appear that she had deposited P371,400.00 when actually she had deposited only P356,400.00. On December 18, 1984, the branch accounting clerk, Lolita Oliveros, discovered a discrepancy between the cash deposit recorded (P371,400.00) in the cash proof and batch sheets and the deposit actually made (P356,400.00) as reflected in the Metrobank passbook. She called the attention of the clearing clerk, Alberto C. Jose, to the matter. They reviewed the records and found that what had been attached to the debit ticket of Jose was a deposit slip for P356,400.00, and not for P371,400.00. An audit team reviewed the account of the branch and found a P15,000.00 shortage incurred on November 16, 1984, the day private respondent turned over her accountability to Mrs. Ubaa after the latter's maternity leave. A committee was formed to investigate the shortage. Private respondent, the branch manager, Eladio C. Laurena, the cashier, Victoria N. Ubaa, the clearing

clerk, Alberto C. Jose, and two other employees were called to the investigation. The committee found private respondent accountable for the shortage. 1 Hence, on January 7, 1985, private respondent was given a memorandum which stated:
In connection with the shortage of P15,000.00 at Taytay Branch which has been recently discovered by the Auditing Department which shortage appears to have been deliberately perpetuated through falsifications of various documents, all of which appear to have been done by you, you are hereby required to submit your explanation within seventy two (72) hours from receipt of this memo why no administrative and/or disciplinary action shall be taken against you. In the meantime, you are hereby preventively suspended for a period of thirty (30) days effective January 8, 1985. (Emphasis added)

The manager, cashier, clearing clerk and a teller, were also given "show-cause" memoranda, but only private respondent was placed under preventive suspension. All those required to show cause filed their respective answers, except private respondent. Instead she requested the bank's vice-president, Antonio Viray, on January 15, 1985, to give her until January 18, 1985 within which to file her answer on the ground that she needed to consult her lawyer. Her request was granted but private respondent nonetheless failed to answer the charges against her. On February 4, 1985, private respondent was dismissed by the bank. The memorandum to her read:
Memorandum To : MS. VICTORIA T. CENTENO Assistant Cashier Taytay Branch This is in connection with the shortage of P15,000.00 at Taytay Branch which was incurred while your were in charge of the vault. Immediately after the discovery of the shortage, through the memorandum of the undersigned dated January 7, 1985 addressed to you, we required you to explain within seventy two (72) hours from receipt of said memo why no administrative and/or disciplinary action should be taken against you. Despite the lapse of the extension period you requested within which to submit your explanation, and up this date, you have not submitted your explanation. After carefully evaluating the evidence presented and considering your failure to explain the shortage which tantamounts to admission of guilt, we have no alternative but to conclude, as we hereby conclude, that you were the one who misappropriated the shortage of P15,000.00. You have therefore forfeited the confidence that the Bank has reposed on you as an officer. IN VIEW OF THE FOREGOING, Management hereby dismisses you FOR CAUSE effective immediately with forfeiture of all benefits. The Bank reserves the right to take

such actions it may deem necessary for the recovery of the P15,000.00. (Emphasis added)

Private respondent sued petitioner for illegal dismissal before the Labor Arbiter. Aside from claiming that her dismissal was without basis, she claimed that she was denied due process because she had not been informed of the specific acts for which she was dismissed. She claimed that during her 19 years of service in petitioner bank, she never "[played] fast and loose with bank funds." Petitioner alleged that private respondent was dismissed for loss of trust and confidence as a result of the shortage, which, according to petitioner, she tried to conceal by falsifying the bank's cash proof sheet and teller's vale. Petitioner claimed that private respondent was accorded due process prior to her dismissal. On September 15, 1988, the Labor Arbiter found petitioner guilty of having illegally dismissed private respondent and of denying her due process. Accordingly the Labor Arbiter ordered:
WHEREFORE, responsive to the foregoing, judgment is as it is hereby entered in favor of complainant and against respondent: 1. Considering the termination of complainant illegal; 2. Ordering respondent to reinstate complainant to her former position or equivalent position with full backwages from the time of her unlawful termination and until actually reinstated without loss of seniority rights and other privileges appertaining to her position; 3. Ordering respondent to pay complainant moral and exemplary damages in the amounts of Fifty Thousand Pesos (P50,000.00) and Ten Thousand Pesos (P10,000.00), respectively; and, 4. Ordering respondent to pay complainant attorney's fees equivalent to ten (10%) per cent of the total award. SO ORDERED.

On appeal, the NLRC affirmed with modification thus:


PREMISES CONSIDERED, the Decision of September 15, 1988 is hereby MODIFIED with the deletion of awards representing moral/exemplary damages and attorney's fees. However, the award of backwages and other benefits shall not exceed three (3) years as laid down by the Supreme Court. Respondent is hereby directed to pay complainant backwages in the amount of NINETY SIX THOUSAND ONE HUNDRED NINETY TWO PESOS (P96,192.00) and/or other benefits due. The other findings stand AFFIRMED. SO ORDERED.

Both parties move for reconsideration, but their motions were denied by the NLRC in its resolution on July 8, 1993.

Hence this petition. Petitioner claims that the NLRC gravely abused its discretion in:
a) holding that private respondent Centeno was denied due process of law prior to her dismissal; and b) failing to fully discuss all the six (6) assigned errors raised by the petitioner in its appeal by ignoring: 1) the valid ground wherein petitioner based its termination of the service of private respondent, and that is loss of confidence; 2) the specific circumstances that led the petitioner to lose its trust and confidence on private respondent; and 3) the applicable settled law and jurisprudence that the private respondent, having been validly dismissed, is not entitled to reinstatement and backwages.

First. Contrary to the finding of the Labor Arbiter and the NLRC, private respondent was notified of the charge against her through a memorandum sent to her on January 7, 1985. Indeed she knew the reason for the "show-cause" order because before that, she and other employees had been asked to attend an investigation. The law requires that the employer must furnish the worker sought to be dismissed with two (2) written notices before termination may be validly effected: first, a notice apprising the employee of the particular acts or omission for which his dismissal is sought and, second, a subsequent notice informing the employee of the decision to dismiss him. 2 In accordance with this requirement, private respondent was given the required notices, on January 7, 1985 and then on February 4, 1985. The NLRC ruled that an investigation should have been conducted prior to private respondent's dismissal. As already noted, however, private respondent was informed of the charges against her and given an opportunity to answer the charges. Upon her request, she was given until January 18, 1985 within which to file her answer. But she failed to file her answer. Of course she later tried to explain that she did not find it necessary to do so because "there was, after all, no ground for any action against [her] . . . and [she] did not feel obligated, therefore, to dispute the action which was baseless and unfounded." 3 Furthermore, she claimed she thought "the Committee had prejudged the case against her." 4 Whatever her reason might have been, the fact is that petitioner waive the right to be heard in an investigation. Due process is not violated where a person is not heard because he has chosen not to give his side of the case. If he chooses to be silent when he has a right to speak, he cannot later be heard to complain that he was silenced. 5 Private respondent having chosen not to answer, should not

be allowed to turn the tables on her employer and claim that she was denied due process. Indeed, the requirement of due process is satisfied when a fair and reasonable opportunity to explain his side of the controversy is afforded the party. A formal or trial-type hearing is not at all times and in all circumstances essential, especially when the employee chooses not to speak. 6 Under the circumstance of this case, it is too much to require petitioner to hear private respondent before the latter can be dismissed. Happily, no liability was imposed on petitioner by either the Labor Arbiter or the NLRC despite the finding that petitioner had denied private respondent due process. Accordingly, all that we need to do in this case is to record our finding that petitioner fully complied with its duty under the law to accord due process to private respondent. Second. Petitioner also claims that the NLRC gravely abused its discretion in not passing upon three (3) errors assigned by it on appeal. We find the contention without merit. In affirming the Labor Arbiter, the NLRC found the evidence supporting the Labor Arbiter's factual findings to be substantial and for this reason apparently found it unnecessary to make a separate discussion. Factual findings of administrative agencies are generally accorded respect and even finality in this Court if they are supported by substantial evidence. 7 Petitioner makes a "reconstruction" of the facts which, according to it, shows how the shortage incurred on November 16, 1984 was concealed. The "reconstruction" is as follows:
A) During the turn-over of the cash in vault by Mrs. Victoria Centeno to Mrs. Victoria Ubaa, after counting the cash in vault, no formal recording of how much cash was actually turned over was done. However, from the Cash Proof in November 16, 1984, it could be reconstructed and determined whether there is a shortage or not by the following figures: Cash Balance, Nov. 15 '84 P589,572.02 Deduct: Cash Vales of Tellers at start of banking day Pico of Teller No. 1 P19,207.06 Pico of Teller No. 2 P21,666.21 Pico of Teller No. 3 P21,995.25 [P62,868.52] Balance: Paper Bills & Coins P536,703.50 Deduct: Deposit with Metrobank P356,400.00 Balance that should have been turned over P170,303.50 Additional Vale-Coins-Teller 3 P11.00 Balance P170,292.50

Add: Sorted Paper Bills turned over by the tellers to the cashiers: Teller No. 1 P100,000.00 Teller No. 1 P40,000.00 Teller No. 2 P147,200.00 [P46,202.64] [P333,402.64] CORRECT CASH BALANCE November 16, 1984 P503,695.14 Cash Balance per Cash Proof November 16, 1984 P488,695.14 SHORTAGE P15,000.00 B) The above P15,000.00 shortage was covered up in two (2) ways or stages: First: In the original or untampered cash proof, the deposit to Metrobank was written originally as P371,400.00 instead of the actual deposit of P356,400.00. The writing of the P371,400.00 deposit to Metrobank was based on a deposit slip for P371,400.00 given to Mrs. Victoria Ubaa by Mrs. Victoria Centeno. The same deposit slip for P371,400.00 was also given to Mr. Alberto Jose, the Clearing clerk, who used the same to enter the P371,400.00 deposit with Metrobank in the Batch Sheet, as well as in the preparation of the Debit and Credit Tickets. The P15,000.00 shortage, which is the difference between P371,400.00 and P356,400.00 was therefore concealed in the P371,400.00 deposit to Metrobank, which actually and truly was for P356,400.00 only. Second: When the genuine deposit slip to Metrobank for P356,400.00 was placed in the Cash Proof file and the spurious deposit slip for P371,400.00 was removed by whoever was responsible for the shortage, the cashproof will NOT BE BALANCED, so that the second step was to ADD P15,000.00 to the P11.00 cash vale of Teller No. 3 to make it appear as if the vale was for P15,011.00. In this way, the cash proof will again be balanced, since the decease of P15,000.00 in deposit with Metrobank from P371,400.00 to P356,400.00 was shifted to the P15,011.00 vale which was actually P11.00 only. Though the P371,400.00 deposit slip is now missing, the insertion of the P15,000.00 in the vale of Teller No. 3 is very apparent, since the duplicate vale in the possession of the Teller has not been tampered and remains as P11.00. Incidentally, it has not missed the petitioner's attention also that, by force of habit, Teller No. 3 was accustomed to placing a "hyphen" across the centavo figures in her Teller's vales when there was no centavo entry thereon; the added figures amounting to P15,000.00 on the other hand did not contain such a "hyphen" in the centavo of the vale, leading us to believe that the addition of P15,000.00 could not have been made by the Teller concerned. (Affidavit of Norberto Robleza date 09 October 1985, pp. 4-6)

Loss of trust and confidence is a cause for dismissing an employee who is entrusted with fiducial matters, or with the custody, handling or care and protection of the

employer's property. 8 There is no dispute about this. But the employer must clearly and convincingly establish the facts and incidents upon which its loss of confidence in the employee may be fairly made to rest, otherwise, the dismissal will be rendered illegal. 9 Petitioner's claim is that although private respondent deposited only P356,400.00 in the Metrobank, she filed up a deposit slip showing the deposit to be P371,400.00 and this amount was recorded in the cash proof sheet and batch sheet for November 16, 1984. But there is no evidence to show this. The falsified deposit slip allegedly made by privates respondent was not presented. Petitioner claimed it was missing. But as private respondent testified the amount of P356,400.00 which she deposited was recorded in the Metrobank passbook. She gave this passbook to Mrs. Ubaa on November 16, 1984. Yet the supposed discrepancy was not noticed by Mrs. Ubaa in preparing the cash proof sheet and the debit sheet who recorded P371,400.00 as having been deposited in Metrobank. Petitioner's allegation that Ms. Centeno misled the cashier and the clearing clerk into recording P371,400.00 cannot therefore be given credence. Indeed, private respondent denied that she gave Mrs. Ubaa a falsified deposit slip showing a deposit of P371,400.00 because after the Metrobank picked up the deposit she made, private respondent handed to Mrs. Ubaa the deposit slip of P356,400.00 together with the cash proof sheet of November 15, 1984 and the key to the vault. 10 Besides, Mrs. Ubaa as already stated, had the passbook. She could not have failed to notice that the amount deposited was P356,400.00 and not P371,400.00 as the bank now claims it was made to understand on November 16, 1984. Petitioner claims that the party responsible for concealing the shortage altered the teller's vale and made it appear that the vale of Teller 3 (Antonette Reyes) was P15,011.00 when the fact was that it was only for P11.00 as shown in the duplicate vale in the possession of Reyes. This claim is subject to two objections. First, it was not shown that private respondent had custody of the vale or, if she had access to the document, that private respondent was the only one who had such access to it, so as to make her the only possible author of the alteration. Second, the fact that the altered vale of Teller 3 in the possession of the bank was not in the teller's customary way of recording does not necessarily mean that the vale she had was the authentic vale while that given to the clearing clerk was falsified. She could have altered her usual practice of recording. It is noteworthy that the shortage was incurred on the day (November 16, 1984) the branch regular cashier, Mrs. Victoria Ubaa, reported for work. It was she in fact who prepared the cash proof sheet. The alteration in the cash proof sheet on that day could not have been made by private respondent. As an NBI handwriting expert stated under cross examination:
WITNESS A The supplemental report is also an answer to the first. The requested analysis could center on the handwritings of the two (2) persons, Mrs. Victoria Ubaa and Mrs. Victoria Centeno. In my first report dated

December 3, 1985 may findings are as follows: The no. 1 states that there are existing fundamental differences between the questioned handwritings or figures appearing on the questioned document and the standard handwritings/figures appearing on the standard documents marked as "SV-1" thru "SV-9" and those standards were the handwritings of one Victoria Ubaa. The result of which is that the questioned handwritings and the standard handwritings were not written by one and the same person. And then in statement that the submitted standards, signatures under the specimen named Victoria Centeno any findings whether Victoria Centeno or not is the writer of the questioned handwritings, so I made the supplemental report to make a definite answer that all the figures and handwritings appearing on the cash proof sheet which is being questioned were not written by Victoria Centeno to answer this phrase. (Emphasis supplied)

Furthermore, the cash proof sheet and the vale were kept in the bank's vault, the key to which was held only by Mrs. Ubaa, as cashier of the bank. 11 Any alteration in the documents by private respondent or by any party could, therefore, have easily been discovered by the cashier. Petitioner further claims that private respondent's accounting method did not correctly reflect the bills from previous banking days and that taking into account all the entries, the amount not reflected was equivalent to the shortage. This contention is without merit. While the accounting method adopted by private respondent was different from the method used by Mrs. Ubaa, private respondent's method was nonetheless an acceptable bank procedure according to Mrs. Robleza, petitioner's own witness. 12 The method adopted by private respondent was accurate, otherwise it could not have been allowed by the bank. Indeed private respondent was acting cashier for two months, from September 17, 1984 to November 15, 1984. During that period no shortage was ever reported. At the time the cash in the vault was turned over the Mrs. Ubaa, it was counted and the failure to record its amount at that time can only mean one thing: that the cash turned over to Mrs. Ubaa corresponded with the amount recorded in the cash proof sheet on November 15, 1984. Private respondent had faithfully served petitioner bank for 19 years. Starting as a bank teller, she steadily rose to the position of assistant branch cashier. Considering this fact, petitioner should have been more careful in determining liability for the loss rather than merely relying on what it calls circumstantial evidence of guilt. The fact that only private respondent did not answer the charge when required in the memorandum of petitioner is not an indication of her guilt. While we recognize that petitioner has a wide latitude in dismissing a bank officer, nonetheless, the evidence on which it acts must be substantial. As the dismissal of private respondent is illegal, she is entitled to reinstatement to her former position without loss of seniority rights and to the payment to her of backwages. 13 The NLRC correctly limited the award of backwages to three years, consistent with the rule at the time of private respondent's dismissal. 14

R.A. No. 6715, which amended Art. 279 of the Labor Code, awarding full backwages to illegally dismissed employees, cannot be retroactively applied to dismissals taking place before its effectivity on March 21, 1989. 15 WHEREFORE, the petition is DISMISSED. SO ORDERED. G.R. No. 114290 September 9, 1996 RAYCOR AIRCONTROL SYSTEMS, INC., petitioner, vs. NATIONAL LABOR RELATIONS COMMISSION and ROLANDO LAYA, et al., respondents.

PANGANIBAN, J.: Were private respondents, employed by petitioner in its business of installing airconditioning systems in buildings, project employees or regular employees? And were their dismissals "due to (petitioner's) present status" and effective the day following receipt of notice legal? Where both the petitioner and the respondents fail to present sufficient and convincing evidence to prove their respective claims, how should the case be decided? This Court answers the foregoing questions in resolving this petition for certiorari assailing the Decision 1 promulgated November 29, 1993 by the National Labor Relations Commission, 2 which set aside and reversed the decision of the labor arbiter dated 22 January 1993, as well as the subsequent order of respondent Commission denying petitioner's motion for reconsideration. The Facts Petitioner's sole line of business is installing airconditioning systems in the building of its clients. In connection with such installation work, petitioner hired private respondents Roberto Fulgencio, Rolando Laya, Florencio Espina, Romulo Magpili, Ramil Hernandez, Wilfredo Brun, Eduardo Reyes, Crisostomo Donompili, Angelito Realingo, Hernan Delima, Jaime Calipayan, Jorge Cipriano, Carlito de Guzman, Susano Atienza, and Gerardo de Guzman, who worked in various capacities as tinsmith, leadman, aircon mechanic, installer, welder and painter. Private respondents insist that they had been regular employees all along, but petitioner maintains that they were project employees who were assigned to work on specific projects of petitioner, and that the nature of petitioner's business mere installation (not manufacturing) of aircon systems and equipment in buildings of its clients prevented petitioner from hiring private respondents as regular employees. As found by the labor arbiter, their average length of

service with petitioner exceeded one year, with some ranging from two six years (but private respondents claim much longer tenures, some allegedly exceeding ten years). In 1991, private respondent Laya and fourteen other employees of petitioner filed NLRC NCR Case No. 00-03-02080-92 for their "regularization". This case was dismissed on May 20, 1992 for want of cause of action. 4 On different dates in 1992, they were served with uniformly-worded notices of "Termination of Employment" by petitioner "due to our present business status", which terminations were to be effective the day following the date of receipt of the notices. Private respondent felt they were given their walking papers after they refused to sign a "Contract Employment" providing for, among others, a fixed period of employment which "automatically terminates without necessity of further notice" or even earlier at petitioner's sole discretion. Because of the termination, private respondents filed three cases of illegal dismissal against petitioner, alleging that the reason given for the termination of their employment was not one of the valid grounds therefor under the Labor Code. They also claimed that the termination was without benefit of due process. The three separate cases filed by private respondents against petitioner, docketed as NLRC-NCR 00-03-05930-92, NLRC NCR 00-05-02789-92, and NLRC NCR 00-0703699-92, were subsequently consolidated. The parties were given opportunity to file their respective memoranda and other supplemental pleadings before the labor arbiter. On January 22, 1993, the Labor Arbiter issued his decision dismissing the complaints for lack of merit. He reasoned that the evidence showed that the individual complainants (private respondents) were project employees within the meaning of Policy Instructions No. 20 (series of 1977) 5 of the Department of Labor and Employment, having been assigned to work on specific projects involving the installation of air-conditioning units as covered by contracts between their employer and the latter's clients. Necessarily, the installation of airconditioning systems "must come to a halt as projects come and go", and "(o)f consequence, the [petitioner] cannot hire workers in perpetuity. And as project employees, private respondent would not be entitled to termination pay, separation pay, holiday premium pay, etc.; and neither is the employer required to secure a clearance from the Secretary of Labor in connection with such termination. Private respondents appealed to the respondent NLRC, which in its November 29, 1993 Decision reversed the arbiter and found private respondent to have been regular employees illegally dismissed. The respondent Commission made the following fourparagraph disquisition:
From the above rules, it can easily be gleaned that complainants belong to a work pool from which the respondent company drew its manpower requirements. This is buttressed by the fact that many of the complainants have been employed for long periods of time already.

We doubt respondent's assertion that complainants were really assigned to different projects. The "Contract Employment" which it submitted (see pp. 32-38, record) purporting to show particular projects are not reliable nay even appears to have been recently typewritten. In the "Contract Employment" submitted by complainants (see p. 65, record), no such name of project appears. Verily, complainants were non-project employees. Anent the dismissal of complainants, suffice it to state that the same was capricious and whimsical which as shown by the vague reason proffered by respondent for said dismissal which is "due to our present business state" (should read "status") is undoubtedly not one of the valid causes for termination of an employment. We are thus inclined to give credence to complainants' allegation that they were eased out of work for their refusal to sign the one-sided "Contract Employment". The fact that complainants were dismissed merely to spite them is made more manifest by respondent's failure to make a report of dismissal or secure a clearance from the Department of Labor (see pp. 196 and 197, record) as required under P.I. No 20 and their publication of an advertisement for replacements for the same positions held by complainants (see p. 198, record). Even assuming that complainants were project employees, their unceremonious dismissal coupled with the attempt to replace them via the newspaper advertisement entitles them to reinstatement with backwages under P.I. No. 20.

The dispositive portion followed immediately and read:


WHEREFORE, the appealed Decision is hereby SET ASIDE and a new one entered ordering respondent to: 1. Immediately reinstate complainants (private respondents) to their former positions without loss of seniority rights and privileges; and 2. Pay them full backwages from the time they were dismissed up to the time they are actually reinstated.

Petitioner's motion for reconsideration was denied by public respondent on February 23, 1994 for lack of merit. Hence, this petition. Issues Petitioner charges public respondent NLRC with grave abuse of discretion in finding private respondent to have been non-project employees and illegally dismissed, and in ordering their reinstatement with full backwages. For clarity's sake, let us re-state the pivotal question involved in the instant case as follows: whether private respondents were project employees or regular (non-project) employees, and whether or not they were legally dismissed. In support of its petition, petitioner reiterates the same points it raised before the tribunals below: that it is engaged solely in the business of installation of airconditioning units or systems in the building of its clients. It has no permanent clients with continuous projects where its workers could be assigned; neither is it a manufacturing firm. Most of

its projects last from two to three months. (The foregoing matters were never controverted by private respondents.) Thus, for petitioner, work is "not done in perpetuity but necessarily comes to a halt when the installation of airconditioning units is completed." On the basis of the foregoing, petitioner asserts that it could not have hired private respondents as anything other than project employees. It further insists that "(a)t the incipience of hiring, private respondents were appraised (sic) that their work consisted only in the installation of airconditioning units and that as soon as the installation is completed, their work ceases and that they have to wait for another installation projects (sic)." In other words, their work was co-terminous with the duration of the project, and was not continuous or uninterrupted as claimed by them. Petitioner also claim that the private respondents signed project contracts of employment indicating the names of the projects or buildings they were working on. And when between projects, these project employees were free to work elsewhere with other establishments. Private respondents controverted these assertions of petitioner, claiming that they had worked continuously for petitioner for several years, some of them as long as ten years, and thus, by operation of law had become regular employees. The Court's Ruling Ordinarily, the findings made by the NLRC are entitled to great respect and are even clothed with finality and deemed binding on this Court, except that when such findings are contrary to those of the labor arbiter, this Court may choose to re-examine the same, as we hereby do in this case now. The First Issue: Project Employees or Regular Employees? An Unfounded Conclusion We scoured the assailed Decision for any trace of arbitrariness, capriciousness or grave abuse discretion, and noted that the respondent Commission first cited the facts of the case, then quoted part of the arbiter's disquisition along with relevant portions of Policy Instructions No. 20, after which it immediately leapt to the conclusion that "(F)rom the above rules, it can easily be gleaned that complainants belong to a work pool from which the respondent company drew its manpower requirements. This is buttressed by the fact that many of the complainants have been employed for long periods of time already. " (emphasis supplied) By reason of such "finding", respondent NLRC concluded that private respondents were regular (not project) employees, but failed to indicate the basis for such finding and conclusion. For our part, we combed the Decision in search of such basis. However, repeated scrutiny of the provisions of Policy Instruction No. 20 pertaining to work pools merely raised further questions.
Members of a work pool from which a construction company draws its project employees, if considered employees of the construction company while in the work pool are nonproject employees or employees for an indefinite period. If they are employed in a

particular project, the completion of the project or of any phase thereof will not mean severance of employer-employee relationship. However, if the workers in the work pool are free to leave anytime and offer their services to other employers then they era project employees employed by a construction company in a particular project or in a phase thereof.

A careful reading of the aforequoted and preceding provisions establishes the fact that project employees may or may not be members of a work pool, (that is the employer may or may not have formed a work pool at all), and in turn, members of a work pool could be either project employees or regular employees. In the instant case, respondent NLRC did not indicate how private respondents came to be considered members of a work pool as distinguished from ordinary (non-work pool) employees. It did not establish that a work pool existed in the first place. Neither did it make any finding as to whether the herein private respondents were indeed free to leave anytime and offer their services to other employers, as vigorously contended by petitioner, despite the fact that such a determination would have been critical in defining the precise nature of private respondents' employment. Clearly, the NLRC's conclusion of regular employment has no factual support and is thus unacceptable. Conclusion Based on Unwarranted Assumption of Bad Faith Immediately thereafter, respondent Commission determined without sufficient basis that complainants were non-project employees. We quote:
We doubt respondent's (petitioner's) assertion that complainants (private respondents) were really assigned to different projects. The "Contract Employment" which it submitted (see pp. 32-38, record) purporting to show particular projects are not reliable nay even appears to have been contrived. The names of the projects clearly appears to have been contrived. The names of the projects clearly appear to have been recently typewritten. In the "Contract Employment" submitted by complainants (see p. 65, record), no such name of project appears. Verily, complainants were non-project employees." (emphasis supplied)

The basis for respondent NLRC's statement that the contracts were contrived was the fact that the names of projects clearly appeared to have been typed in only after the contracts had been prepared. However, our examination of the contracts (presented by petitioner as Annexes "A", "B", "B-1", "C", "D", "E" and "F" 6 to its Position Paper dated July 30, 1992 filed with the labor arbiter)did not lead inexorably to the conclusion that these were "contrived". Said Annexes were photocopies of photocopies of the original "Contract Employment's", 7 and the names of projects had been typed onto these photocopies, meaning that the originals of said contracts probably did not indicate the project names. But this alone did not automatically or necessarily mean that petitioner had committed any falsehood or fraud, or had any intent to deceive or impose upon tribunals below, because the names of the projects could have been typed/filed in good faith, nunc pro tunc, in order to supply the data which ought to have been indicated in

the originals at the time those were issued, but which for some reason or other were omitted in short, the names of projects could have been filled in simply in order to make the contracts speak the truth more clearly or completely. Notably, no reason was advanced for not according the petitioner the presumption of good faith. Respondent NLRC, then made an unwarranted assumption that bad faith and fraudulent intent attended the filling in of the project names in said Annexes. In any event, it can be easily and clearly established with the use of the naked eye that the dates and durations of the projects and/or work assignments had been typed into the original contracts, and therefore, petitioner's failure to indicate in the originals of the contracts the name(s) of the project(s) to which private respondents were assigned does not necessarily mean that they could not have been project employees. (Incidentally, we should make mention here that what is or is not stated in a contract does not control nor change the juridical nature of an employment relationship since the same is determined and fixed by law. As a matter of fact, we note that there is no requirement in Policy Instructions No. 20 that project employees should be issued written contracts of employment, let alone that a written contract should indicate the name of the project to which the employee concerned is being assigned.) Statutory Basis for Determining Nature of Employment The parties and their respective counsel, as well as respondent Commission and the Solicitor General, should have re-read and carefully studied ALU-TUCP vs. National Labor Relations Commission, 8 which is highly instructional on this question:
The law on the matter is Article 280 of the Labor Code which reads in full: Article 280. Regular and Casual Employment The provisions of the written agreement to the contrary notwithstanding and regardless of the oral agreement of the parties, an employment shall be deemed to be regular where the employee has been engaged to perform activities which are usually necessary or desirable in the usual business or trade of the employer, except where the employment has been fixed for a specific project or undertaking the completion or termination of which has been determined at the time of the engagement of the employee or where the work or services to be performed is seasonal in nature and the employment is for the duration of the season. An employment shall be deemed to be casual if it is not covered by the preceding paragraph: Provided, That, any employee who has rendered at least one year of service, whether such service is continuous or broken, shall be considered a regular employee with respect to the activity in which he is employed and his employment shall continue while such activity exists. . . . xxx xxx xxx . . . For, as is evident from the provisions of Article 280 of the Labor Code, quoted earlier, the principal test for determining whether particular employees are property characterized as "project employees" as distinguished from "regular employees," is whether or not the "project employees" were assigned to carry out a "specific project or undertaking," the

duration (and scope) of which were specified at the time the employees were engaged for that project. (emphasis ours) In the realm of business and industry, we note that "project" could refer to . . . a particular job or undertaking that is within the regular or usual business of the employer company, but which is distinct and separate and identifiable as such, from the other undertakings of the company. Such job or undertaking begins and ends at determined or determinable times. The typical example of this . . . type of project is a particular construction job or project of a construction company. A construction company ordinarily carried out two or more discrete identifiable construction projects: e.g., a twenty-five story hotel in Makati; a residential condominium building in Baguio City; and a domestic air terminal in ilolo City Employees who are hired for the carrying out of one of these separate projects, the scope and duration of which has been determined and made known to the employees at the time of employment, are properly treated as "project employees," and their services may be lawfully terminated at completion of the project.

The same decision goes on to say: 9


. . . The simple fact that the employment of petitioners as project employees had gone beyond one (1) year, does not detract from, or legally dissolve, their status as project employees. The second paragraph of Article 280 of the Labor Code, quoted above, providing that an employee who has served for at least one (1) year, shall be considered a regular employee, relates to casual employees, not to project employees. In the case of Mercado, Sr. vs. National Labor Relations Commission (201 SCRA 332 [1991]), this Court ruled that the proviso in the second paragraph of Article 280 relates only to casual employees and is not applicable to those who fall within the definition of said Article's first paragraph, i.e., project employees. . . .

Incidentally, we should that both respondent Commission and the Solicitor General were in error in concluding based on private respondents' claimed length of employment (allegedly for over ten years) that they were regular employees. Sad to state, the Solicitor General in his arguments tried to "force-fit" private respondents into the "regular employee" category and completedly disregarded the critical distinctions set forth in ALU-TUCP and earlier cases. Inconclusive Evidence Based on the foregoing considerations, it is patent that, in the instant case, there needs to be a finding as to whether or not the duration and scope of the projects were determined or specified and made known to herein private respondents at the time of their engagement. The labor arbiter tried to do this, relying heavily on the "Contract(s) Employment" presented in petitioner's Annexes as well as on private respondents' own Annex "A" 10 attached to their Position Paper, and citing the fact that the said contracts of employment indicated the duration of the projects to which the private respondents had been assigned. He then held that "(t)here is no denial that complainants were assigned to work in these projects," 11 and concluded that they were indeed project employees.

But the arbiter completed ignored the fact that all the "Contract(s) Employment" presented in evidence by both petitioner and private respondents had been signed only by petitioner's president and general manager, Luis F. Ortega, but not by the employees concerned, who had precisely refused to sign them. The said contracts therefore could in no wise be deemed conclusive evidence. Thus, private respondents faulted the labor arbiter for giving credence and probative value to said contracts. Besides, they claimed, only seven contracts in all were presented in evidence, pertaining to seven individual employees, while there are fifteen employees involved in the complaints. Moreover, these contracts, purportedly issued either in July or December of 1991, except for one dated May 1992, were all one-shot contracts of short duration, the longest being for about five months. Now, inasmuch as petitioner had not denied nor rebutted private respondent' allegations that they had each worked several years for the petitioner, the obvious question is, why didn't petitioner produce in evidence similar contracts for all the other years that private respondents had worked as project employees? To these points, petitioner offered no explanation whatsoever. Failure to Discharge Burden of Proof For that matter, it seems self-evident to this Court that, even if the contracts presented by petitioner had been signed by the employees concerned, still, they would not constitute conclusive proof of petitioner's claim. After all, in the usual scheme of things, contract terms are normally dictated by the employer and simply acceded to and accepted by the employee, who may be desperate for work and therefore in no position to bargain freely or negotiate terms to his liking. In any event, petitioner in the case undoubtedly could have presented additional evidence to buttress its claim. For instance, petitioner could have presented copies of its contracts with its clients, to show the time, duration and scope of past installation projects. The data from these contracts could then have been correlated to the data which could be found in petitioner's payroll records for, let us say, the past three years or so, 12 to show that private respondents had been working intermittently as and when they were assigned to said projects, and that their compensation had been computed on the basis of such work. But petitioner did not produce such additional evidence, and we find it failed to discharge its burden of proof. It is not so much that this Court cannot appreciate petitioner's contentions about the nature of its business and its inability to maintain a large workforce on its permanent payroll. Private respondents have admitted that petitioner is engaged only in the installation (not manufacture) of aircon systems or units in buildings, and since such a line of business would obviously be highly (if not wholly) dependent on the availability of buildings or projects requiring such installation services, which factor no businessman, no matter how savvy, can accurately forecast from year to year, it can be easily surmised that petitioner, aware that its

revenues and income would be unpredictable, would always try to keep its overhead costs to a minimum, and would naturally want to engage workers on a per-project or per-building basis only, retaining very few employees (if any) on its permanent payroll. It would also have been more than glad if its employees found other employment elsewhere, in between projects. To our mind, it appears rather unlikely that petitioner would keep private respondent all fifteen of them continuously on its permanent payroll for, say, ten or twelve years, knowing fully well that there would be periods (of uncertain duration) when no project can be had. To illustrate, let us assume that private respondents (who were each making about P118.00 to P119.50 per day in 1991) were paid only P100.00 per day. If the fifteen were, as they claimed, regular employees entitled to their wages regardless of whether or not they were assigned to work on any project, the overhead for their salaries alone computed at P100.00/day for 30 days in a month would come to no less than P45,000.00 a month, or P540,000.00 a year, not counting 13th month pay, Christmas bonus, SSS/Medicare premium payments, sick leaves and service incentive leaves, and so forth. Even if petitioner may have been able to afford such overhead costs, it certainly does not make business sense for it or anyone else to do so, and is in every sense contrary to human nature, not to mention common business practice. On this score alone, we believe that petitioner could have made out a strong case. Which is why we have difficulty understanding its failure to present clear and convincing evidence on this point, it being doctrinal that in illegal dismissal cases, the employer always has the burden of proof. 13 Petitioner's problem of weak evidence was further compounded by certain documentary evidence in the records below which controverted petitioner's position, or, at the very least, tended to confuse rather than clarify matters. For instance, we noted that in their Memorandum of Appeal dated February 17, 1993 filed with the respondent Commission, herein private respondents had attached as annexes thereto the following documents:
1. As Annex "B" thereof, a Certification dated January 28, 1992, signed by one Flora P. Perez, Administrative/Accountant of Raycor, certifying that " . . . Mr. Roberto B. Fulgencio (one of the private respondents) has been connected with the undersigned corporation (Raycor) from August 22, 1986 to May 18, 1991 and September 01, 1990 to January 25, 1992 as Aircon Installer"; 2. As Annex "C" thereof, a Certification dated May 7, 1985, signed by Luis F. Ortega, President and General Manger of herein petitioner corporation, to the effect that ". . . Mr. Jaime Calipayan (another one of the private respondents) has been connected with the undersigned corporation from June 18, 1982 up to present as a Mechanical Installer; and 3. As Annex "D" thereof, a Certification dated June 06, 1991, likewise signed by Luis G. Ortega, president and general manager of Raycor, certifying that ". . . Mr. Susano A. Atienza (still another of the private respondents) has been connected with the undersigned corporation from October 10, 1983 up to present as Aircon Mechanical/Technician".

Understandably, private respondents made big capital out of these certifications. But, while petitioner failed utterly to offer rebutting evidence, still and all, we are not prepared to conclude on the basis of these certifications alone that private respondents were indeed regular employees. First of all, said certifications refer only to three out of the fifteen private respondent, so what could be true of them any not necessarily apply with respect to the other twelve. Moreover, the certifications do not categorically state that the three employees had been permanent employees of Raycor. In other words, they do not necessarily overturn petitioner's contention that private respondents were project employees, since it is still possible to read the documents as saying that the named employees were working as project employees during the period therein specified. This is especially so since the said certifications were prepared by nonlawyers who in all likelihood were not aware of the potential legal implications and ramifications of what were ostensibly innocuous certifications. As held in one recent case, ". . . it is however not difficult to understand that ordinary business activities are performed in the normal course without anticipation nor foreknowledge of litigation, often with dispatch and usually with a minimum of documentation." 14 Nonetheless, all things considered, the certifications, issued by petitioner itself, tend to put its claims in serious doubt. This situation was still further aggravated by the manner in which petitioner dismissed private respondents. As found by respondent Commission, the reason given for the dismissals, i.e., "due to our present business status," is vague, to say the least, and unarguably is not one of the valid or just causes provided by law for termination of an employment, whatever its classification. But more significantly if indeed private respondents were project employees, there would have been no need to terminate them by sending them notices of termination, inasmuch as their employment ceases "as result of the completion of the project or any phase thereof in which they are employed," per Policy Instruction No. 20 itself. Thus, if petitioner resorted to such dismissals, there is the unavoidable inference that petitioner regarded the private respondents as regular employees after all. But again, this is inconclusive, since the notices of termination were signed, and in all likelihood prepared, by the president and general manager of petitioner, probably sans any legal advice or awareness of the implications of such a move. All the aforesaid conflicting data have the net effect of casting doubt upon and clouding the real nature of the private respondents' employment status. And we are mandated by law to resolve all doubts in favor of labor. For which reason, we hereby hold that private respondents were regular employees of the petitioner. Having arrived at basically the same results as respondent NLRC with respect to private respondents' employment status, did this Court waste its time and effort in re-examining the instant case? The answer is in the negative. This Court cannot affirm a decision or judgment based on erroneous findings and conclusions, for

justice can never be adequately dispensed to all parties if a judgment is not grounded on the truth. Second Issue: Terminations Illegal On the second issue of alleged illegality of the subject dismissals, we agree with respondent Commission when it held, as mentioned above, that "the same was capricious and whimsical as shown by the vague reason proffered by respondent for said dismissal which is 'due to our present state' (should read 'status') is undoubtedly not one of the valid causes for termination of an employment." True indeed, for neither the Labor Code nor Policy Instructions No. 20 allows termination on such ground. Even Art. 283 of the Labor Code as amended, which treats of retrenchments and closures due to business losses, requires that the employer first serve written notice on the workers and the Department of Labor at least one month before the intended date thereof; and in certain cases, separation pay must be paid. And it cannot be denied that in the instant case, petitioner did not afford them due process thru the twin requirements of notice and hearing, 15 as the terminations took effect the day following receipt of the notices of termination. Ineluctably, the said terminations are not in accordance with law and therefore illegal. On top of that, there is evidence of the bad faith of petitioner in terminating the private respondents. Petitioner placed an ad 16 in the classified ads section of the People's Journal, sometime in June 1992, 17 which read:
WANTED IMMEDIATELY MECHANICAL INSTALLERS TINSMITHS WELDERS/PIPEFITTERS APPLY IN PERSON: RAYCOR AIR CONTROL SYSTEMS, INC. RM 306 20TH CENTURY BLDG. 632 SHAW BLVD., MAND. METRO MANILA

Unmistakably, petitioner, in placing the ad, must have had at least one project, maybe more, "in the pipeline" at that time, and was clearly in need of replacements for private respondents whom it had just fired. Thus, the dismissals could hardly have been due to a valid cause, not even due to petitioner's alleged "present business status". On this court as well, the dismissals were illegal. And lastly, we should mention that an order for reinstatement with payment of backwages must be based on the correct premises. This point is best illustrated by considering the last ratiocination utilized by public respondent: "Even assuming that complainants were project employees, their unceremonious

dismissal coupled with the attempt to replace them via the newspaper advertisement entitles them to reinstatement with backwages under P.I. No. 20." There is a world of difference between reinstatement as project employees and reinstatement as regular employees, but the difference was obviously lost on the respondent NLRC. Conclusion We reiterate that this Court waded through the records of this case searching for solid evidence upon which to decide the case either way. But all told, neither party managed to make out a clear case. Therefore, considering that in illegal dismissal cases, the employer always has the burden of proof, and considering further that the law mandates that all doubts, uncertainties, ambiguities and insufficiencies be resolved in favor of labor, we perforce rule against petitioner and in favor of private respondents. WHEREFORE, the foregoing considered, the assailed Decision is hereby SET ASIDE and a new one rendered holding that petitioner has failed to discharge its burden of proof in the instant case and therefore ORDERING the reinstatement of private respondents as regular employees of petitioner, without loss of seniority rights and privileges and with payment of backwages from the day they were dismissed up to the time are actually reinstated. No casts. SO ORDERED. WALLEM MARITIME SERVICES vs. NLRC 263 SCRA 174FACTS: Joselito Macatuno, private respondent, was a seaman on board theM / T F o r t u n a o f L i b e r i a n r e g i s t r y . H e w a s h i r e d b y W a l l e m Shipmanagament Ltd. Thru its manning agent Wallem Maritime ServicesInc. Macatunos contract of employment covers 10 months.W h i l e t h e v e s s e l w a s b e r t h e d a t t h e p o r t o f K a w a s a k i , J a p a n , a n altercation took place between Macatuno and Gurimbao, a fellow Filipinoagainst a cadet/apprentice officer of the same nationality as the captaino f t h e v e s s e l . T h e c a d e t / a p p r e n t i c e o r d e r e d G u r i m b a o t o s h o v e a n d throw dirty and oily water at the port of Japan. The latter protested sinces u c h a c t i s a g a i n s t t h e l a w s o f J a p a n . H o w e v e r , t h e c a d e t / a p p r e n t i c e insisted on his orders so Gurimbao complied with it. Having finished his j o b , G u r i m b a o s o u g h t t h e a i d o f M a c a t u n o t o a p p r o a c h t h e cadet/apprentice about his improper and unauthorized act. When the t w o F i l i p i n o s a p p r o a c h e d t h e c a d e t / a p p r e n t i c e , t h e l a t t e r r e a c t e d violently so Macatuno LEBpushed the latter twice on his chest while Gurimbao mildly hit his arm. The captain witnessed the altercation and entered the incident inthe tankers logbook. He summoned the two Filipinos at his cabin. The c a p t a i n t o l d t h e m t o pack their things as their services are being t e r m i n a t e d . A s a c o n s e q u e n c e , t h e t w o w e r e r e p a t r i a t e d t o t h e Philippines where they lodged complaints for illegal dismissal with

theP O E A . P e t i t i o n e r s c o n t e n d t h a t t h e t w o F i l i p i n o s h a d b e e n delinquent on board the vessel as shown by the records of the captainslogbook T h e P O E A f o u n d t h a t t h e p r i v a t e r e s p o n d e n t s M a c a t u n o a n d Gurimabaos dismissals were illegal. The NLRC affirmed the decision of the POEA. Hence, the instant petition. ISSUE: Whether the dismissal of the two seamen were illegal. RULING/DECISION: The Court upheld the decision of the NLRC in finding that t h e private respondents were illegally dismissed.P e t i t i o n e r s d i d n o t s u b m i t a s e v i d e n c e t o t h e P O E A t h e l o g b o o k itself but was merely a typewritten collation of excerpts from what could be the logbook. Hence, as the typewritten excerpts from then logbookwere the only pieces of evidence presented by petitioners to support thed i s m i s s a l o f p r i v a t e r e s p o n d e n t h a v e n o p r o b a t i v e v a l u e a t a l l , petitioners cause must fail. That the workers involvement in the incident was mustered or c o n v e n e d thereafter b y the captain is inconsequential. It is sufficientcompliance with the law which, requires, as a vital component of due process, o b s e r v a n c e o f t h e t w i n r e q u i r e m e n t s o f n o t i c e a n d h e a r i n g before dismissing an employee.

G.R. No. 117582 December 23, 1996 CONRADO SAMILLANO and MYRNA V. SAMILLANO, petitioners, vs. NATIONAL LABOR RELATIONS COMMISSIONS, DAN-AG SA DAKBAYAN BROADCASTING CORPORATION-RADIO STATION DXDD, MSGR. JESUS DOSADO and SIMPLICIA NERI, respondents.

PADILLA, J.:p This petition for certiorari under Rule 65 of the Rules of Court refers to two (2) cases filed by petitioner-spouses Conrado and Myrna Samillano against private respondents Dan-ag sa Dakbayan Broadcasting Corporation-Radio Station DXDD and/or Msgr. Jesus Dosado and/or Simplicia Neri, Chairman of the Board and Manager respectively of said respondent corporation. The first case, filed by petitioner-spouses on 8 February 1991, is a complaint for illegal demotion while the second complaint filed on 20 May 1991 is for illegal dismissal, payment of backwages, commissions and other monetary claims.

The two (2) complaints before Regional Arbitration Branch No. 10, Cagayan de Oro City of the National Labor Relations Commission (NLRC) docketed as NLRC RAB Case Nos. 10-03-00195 and 10-06-00371-91 were later consolidated since they involve the same parties and issues. The undisputed facts of the two (2) cases are as follows: 1. Petitioner-spouses Conrado and Myrna Samillano were hired by private respondents on 1 October 1981 and 1 August 1983 respectively; 2. On 2 October 1990, Conrado Samillano was transferred to the Technical Department as an SSB Operator from his previous position as Traffic Supervisor of private respondent corporation. On the same day, his wife Myrna V. Samillano was transferred to the AM Production Department from her position as cashier of respondent corporation; 3. As a result of the transfers, the petitioner-spouses filed the complaint for illegal demotion contending that the transfer resulted in loss of commissions and violated their security of tenure. 4. On 20 May 1991, petitioner-spouses filed the complaint for illegal dismissal contending that private respondents terminated their employment on 23 April 1991 without any lawful cause; 5. Private respondents relied on allegations that petitioner-spouses misappropriated funds of the radio station and committed acts of insubordination which resulted in loss of trust and confidence, upon which their dismissal were based; 6. In a supplemental position paper, herein petitioners contended that their demotion and subsequent dismissal were retaliatory acts of private respondents for their having reported violations by private respondents of labor laws particularly underpayment/nonpayment of salaries and other benefits; Labor Arbiter Noel Augusto S. Magbanua, to whom the cases were assigned, found that sometime in July 1989, the Department of Labor and Employment conducted an inspection of the premises of private respondent corporation and initially found deficiencies in wages and other benefits given to employees. It was further determined that in March or April 1990, private respondents conducted meetings with their employees seeking a compromise of the unpaid benefits. Some employees executed waivers of further claims against private respondents. Herein petitioners refused to sign said waivers. The labor arbiter formulated the following issues for resolution: 1

1) whether complainants' demotion and subsequent termination of employment were retaliatory acts for complainants' having allegedly reported respondents' violations of labor laws; 2) whether complainants' demotions were illegal; and 3) whether complainants' terminations from employment were illegal. The labor arbiter resolved the first two (2) issues in the negative. He declared that no evidence was presented to show that the demotions of petitioners were linked to their reporting of alleged violations by private respondents of the Labor Code. The labor arbiter further upheld management's prerogative, in the absence of bad faith, to protect its rights in relation to the alleged offenses committed by petitioners. The demotions of petitioners were therefore upheld. With respect to the dismissal of petitioners from employment, however, the labor arbiter found that the alleged misappropriations of funds committed by petitioners were not adequately substantiated. Hence, the dismissal of petitioners was declared illegal. The labor arbiter ruled however that instead of reinstatement, it would be for the best interest of the parties considering the strained relations between them, to award petitioners separation pay equivalent to one (1) month salary for every year of service. Full backwages were not awarded based on findings that petitioners acted in an arrogant and uncooperative manner during the investigation of their case which could be a possible reason why private respondents were not able to prove the formers' involvement in the financial irregularities subject of this case. 2 Only six (6) months backwages were awarded to each of the complainants (herein petitioners). Finally, the labor arbiter denied petitioners' claims for unpaid commissions for lack of evidence. Appeal by private respondents to the NLRC was dismissed on 9 February 1994 for their failure to properly perfect their appeal. The NLRC found that private respondents had filed their notice of appeal without attaching thereto their appeal memorandum as required by Section 3 Rule VI of the Rules of Procedure of the NLRC. There was therefore failure to perfect the appeal within the reglementary period of ten (10) days from receipt of the assailed labor arbiter's decision. On 30 June 1994, the NLRC reinstated the appeal based on findings that while the notice of appeal and appeal memorandum were received by the NLRC on 15 July 1993 and 20 July 1993 respectively, or way beyond the period for appeal which expired on 3 July 1993, both pleadings were, however, actually mailed on 2 July 1993 as evidenced by Registry Receipt No. 77 of the Tangub Post Office. 3

On the merits of the appeal, the NLRC ruled that private respondents have substantiated their claim of having lost trust and confidence in petitioners due to serious irregularities in the performance of their duties. The NLRC held that, contrary to the findings of the labor arbiter, an audit report submitted by a certain Domeciano Adaya dated 17 September 1990 showed substantial evidence of petitioners' involvement in irregularities including misappropriations of funds, non-turnover of collections and misuse of funds for personal purposes. The NLRC relied on reports made by Janice Poncianos, the Finance Department Business Head of respondent corporation's radio station addressed to the station manager as well as the report of the station manager to the chairman of the board of respondent corporation on the alleged acts of herein petitioners. 4 Based on the above findings, the NLRC set aside the assailed decision and ruled that petitioners were validly dismissed. However, private respondents were ordered to indemnify petitioners the amount of P2,000.00 each for violation of the latters' right to due process. The NLRC agreed with the petitioners that there was no formal investigation wherein the latter were given the chance to defend themselves against the charges levelled against them. 5 In their petition before the Court, it is argued by petitioners that: 1. The NLRC gravely abused its discretion in holding that the dismissals of herein petitioners were valid; and 2. The NLRC gravely abused its discretion in merely imposing a sanction on private respondents for violation of petitioners' right to due process. 6 Before ruling on the merits of this petition, the Court takes notice of a peculiar circumstance regarding the appeal of the private respondents from the decision of the labor arbiter. In the resolution reinstating private respondents' appeal, the NLRC found that the notice of appeal and memorandum on appeal were received on 15 July 1993 and 20 July 1993 respectively. The reason for reinstating the appeal was the finding that both pleadings were actually mailed on 2 July 1993 as evidenced by Registry Receipt No. 77 postmarked on the same date at the Tangub City Post Office. It is unexplained however why two (2) pleadings mailed together using a single registry receipt and presumably contained in one (1) envelope would be received on two (2) different dates. It should be pointed out that in the motion for reconsideration of the resolution dismissing the appeal, herein private respondents averred mailing only the notice of appeal and a postal money order to cover appeal fees on 2 July 1993. Be that as it may, the Court shall proceed to resolve this case on the merits despite the possible technicality of the appeal being filed late with the NLRC. The NLRC is however

reminded to be more accurate in recording the dates of mailing and receipt of pleadings filed before it since this is essential in the speedy and correct disposition of cases. Petitioners do not dispute before this Court the validity of their re-assignments. It is clear that the re-assignments were a valid exercise of management prerogative pending investigation of the alleged irregularities. The purpose of the re-assignments is no different from that of preventive suspension which private respondents could likewise have validly imposed on petitioners; to protect the employer's property pending investigation of the alleged malfeasance or misfeasance committed by the employee. 7 In the present case, the labor arbiter correctly held that there is no evidence to show that the transfer of petitioners to other positions and the subsequent termination of their employment were retaliatory acts of private respondents for petitioners' reporting of the alleged violations by private respondents of the Labor Code. The legality of petitioners' dismissal would be determined based on whether or not private respondents have proved the basis for loss of trust and confidence upon which the dismissals are based. In China City Restaurant Corporation v. NLRC 8 the Court held thus:
For loss of trust and confidence to be a valid ground for the dismissal of employees, it must be substantial and not arbitrary, whimsical, capricious or concocted. Irregularities or malpractices should not be allowed to escape the scrutiny of this Court. Solicitude for the protection of the rights of the working class are of prime importance. Although this is not a license to disregard the rights of management, still the Court must be wary of the ploys of management to get rid of employees it considers as undesirable.

The NLRC based its decision upholding petitioners' dismissal on the conclusion that the irregularities involving petitioners were more than sufficient to make out a case of loss of trust and confidence. 9 Said irregularities allegedly involving petitioners were enumerated in An Updated Report dated 17 August 1990 submitted by the Finance Department Business Head Janice Procianos and various letter-memos to petitioners as well as the audit report dated 17 September 1990 submitted by Domeciano Adaya. But petitioners correctly argue that the above-mentioned documents do not provide enough basis for termination of their employment based on loss of trust and confidence. The Adaya audit report in part reads:
I am suggesting with a request that the above-mentioned observations be reviewed and confirmed by the Station Accountant, Bookkeeper, Collector and Cashier or Cash Custodian in my presence in fairness to everyone before I give conclusion, implication or opinion to these observations. They may also give comments or raise objections, if any. The comments or objections may be made orally or in writing.

In this connection, as I don't have line authority over the personnel concerned may I request you to ask them to review and confirm by observations.

There is no evidence to show that herein private respondents undertook to review and/or confirm the observations contained in the audit report as recommended by the audit report itself. On the contrary, even in their comment on the petition filed with this Court, which respondents' later adopted as their memorandum, the dismissal of herein petitioners is justified mainly on the basis of said audit report submitted by Domeciano Adaya. 10 It is, however, clear from the above-quoted portion of the audit report that the findings contained therein do not categorically find herein petitioners guilty of committing irregularities. The clear import of the said audit report is that further investigation and verification would be necessary to pinpoint the source of the irregularities. There is thus no evidence on record to show that any further investigation and verification were done by private respondents. What is apparent is that petitioners were made to answer charges of misconduct based on suspicions which lacked adequate basis. While the law and this Court recognize the right of an employer to dismiss an employee based on loss of trust and confidence, the former's evidence must clearly and convincingly establish the facts upon which the loss of trust and confidence in the employee is based. 11 In the present case, the unsubstantiated suspicions and baseless conclusions of private respondents do not provide legal justification for dismissing herein petitioners. The doubt in this case should be resolved in favor of labor pursuant to the social justice policy of labor laws and the Constitution. Finally, on petitioners' right to due process, we uphold the NLRC findings that no formal investigation was conducted prior to dismissal of petitioners. Private respondents thus failed to adequately comply with the requirement that an employee should be given the opportunity to be heard and to defend himself before he is dismissed. In San Antonio v. NLRC, 12 the Court stated that "Proper compliance with the twin requirements of notice and hearing are conditions sine qua non before a dismissal may be validly effected. . . Any procedural shortcut, that effectively allows an employer to assume the roles of both accuser and judge at the same time, should not be countenanced." (emphasis supplied). In the present case, the notices/memoranda to petitioners requiring explanations/answers to the charges against them were plainly meant to provide a semblance of compliance with the due process requirement which the NLRC correctly ruled to be inadequate.

The Court will not be deceived by schemes to circumvent the requirements of law and the Constitution. For failure to fully comply with the requirements of due process, private respondents should, as a matter of course, indemnify the petitioners but we refrain from awarding damages on this score since we are awarding separation pay and backwages due to petitioners' illegal dismissal. The above-finding that petitioners were illegally dismissed normally requires that they be reinstated to their former or equivalent positions with full backwages. In this case, however, the relationships between petitioners and private respondents have undoubtedly become very strained, hence, separation pay in lieu of reinstatement is proper. 13 However, as a consequence of petitioners' illegal dismissal, full backwages from date of dismissal to the finality of this decision are due the petitioners in line with the ruling in the Bustamante case. 14 WHEREFORE, the decision appealed from is hereby SET ASIDE and a new one entered: 1. DECLARING the dismissal from employment of petitioners NULL and VOID; 2. ORDERING private respondents to pay petitioners' separation pay at the rate of ONE-HALF (1/2) MONTH salary for every year of service; and 3. ORDERING private respondents to pay petitioners full backwages from date of illegal dismissal to the finality of this decision. SO ORDERED. G.R. No. 117565 November 18, 1997 ARSENIO P. LUMIQUED (deceased), Regional Director, DAR CAR, Represented by his Heirs, Francisca A. Lumiqued, May A. Lumiqued, Arlene A. Lumiqued and Richard A. Lumiqued, petitioners, vs. Honorable APOLONIO G. EXEVEA, ERDOLFO V. BALAJADIA and FELIX T. CABADING, ALL Members of Investigating Committee, created by DOJ Order No. 145 on May 30, 1992; HON. FRANKLIN M. DRILON, SECRETARY OF JUSTICE, HON. ANTONIO T. CARPIO, CHIEF Presidential Legal Adviser/Counsel; and HON. LEONARDO A. QUISUMBING, Senior Deputy Executive Secretary of the Office of the President, and JEANNETTE OBAR-ZAMUDIO, Private Respondent, respondents.

ROMERO, J.:

Does the due process clause encompass the right to be assisted by counsel during an administrative inquiry? Arsenio P. Lumiqued was the Regional Director of the Department of Agrarian Reform Cordillera Autonomous Region (DAR-CAR) until President Fidel V. Ramos dismissed him from that position pursuant to Administrative Order No. 52 dated May 12, 1993. In view of Lumiqued's death on May 19, 1994, his heirs instituted this petition for certiorari and mandamus, questioning such order. The dismissal was the aftermath of three complaints filed by DAR-CAR Regional Cashier and private respondent Jeannette Obar-Zamudio with the Board of Discipline of the DAR. The first affidavit-complaint dated November 16, 1989, 1 charged Lumiqued with malversation through falsification of official documents. From May to September 1989, Lumiqued allegedly committed at least 93 counts of falsification by padding gasoline receipts. He even submitted a vulcanizing shop receipt worth P550.00 for gasoline bought from the shop, and another receipt for P660.00 for a single vulcanizing job. With the use of falsified receipts, Lumiqued claimed and was reimbursed the sum of P44,172.46. Private respondent added that Lumiqued seldom made field trips and preferred to stay in the office, making it impossible for him to consume the nearly 120 liters of gasoline he claimed everyday. In her second affidavit-complaint dated November 22, 1989, 2 private respondent accused Lumiqued with violation of Commission on Audit (COA) rules and regulations, alleging that during the months of April, May, July, August, September and October, 1989, he made unliquidated cash advances in the total amount of P116,000.00. Lumiqued purportedly defrauded the government "by deliberately concealing his unliquidated cash advances through the falsification of accounting entries in order not to reflect on 'Cash advances of other officials' under code 8-70-600 of accounting rules." The third affidavit-complaint dated December 15, 1989, 3 charged Lumiqued with oppression and harassment. According to private respondent, her two previous complaints prompted Lumiqued to retaliate by relieving her from her post as Regional Cashier without just cause. The three affidavit-complaints were referred in due course to the Department of Justice (DOJ) for appropriate action. On May 20, 1992, Acting Justice Secretary Eduardo G. Montenegro issued Department Order No. 145 creating a committee to investigate the complaints against Lumiqued. The order appointed Regional State Prosecutor Apolinario Exevea as committee chairman with City Prosecutor Erdolfo Balajadia and Provincial Prosecutor Felix Cabading as members. They were mandated to conduct an investigation within thirty days from receipt of the order, and to submit their report and recommendation within fifteen days from its conclusion. The investigating committee accordingly issued a subpoena directing Lumiqued to submit his counter-affidavit on or before June 17, 1992. Lumiqued, however, filed instead an urgent motion to defer submission of his counter-affidavit pending actual

receipt of two of private respondent's complaints. The committee granted the motion and gave him a five-day extension. In his counter-affidavit dated June 23, 1992, 4 Lumiqued alleged, inter alia, that the cases were filed against him to extort money from innocent public servants like him, and were initiated by private respondent in connivance with a certain Benedict Ballug of Tarlac and a certain Benigno Aquino III. He claimed that the apparent weakness of the charge was bolstered by private respondent's execution of an affidavit of desistance. 5 Lumiqued admitted that his average daily gasoline consumption was 108.45 liters. He submitted, however, that such consumption was warranted as it was the aggregate consumption of the five service vehicles issued under his name and intended for the use of the Office of the Regional Director of the DAR. He added that the receipts which were issued beyond his region were made in the course of his travels to Ifugao Province, the DAR Central Office in Diliman, Quezon City, and Laguna, where he attended a seminar. Because these receipts were merely turned over to him by drivers for reimbursement, it was not his obligation but that of auditors and accountants to determine whether they were falsified. He affixed his signature on the receipts only to signify that the same were validly issued by the establishments concerned in order that official transactions of the DAR-CAR could be carried out. Explaining why a vulcanizing shop issued a gasoline receipt, Lumiqued said that he and his companions were cruising along Santa Fe, Nueva Vizcaya on their way to Ifugao when their service vehicle ran out of gas. Since it was almost midnight, they sought the help of the owner of a vulcanizing shop who readily furnished them with the gasoline they needed. The vulcanizing shop issued its own receipt so that they could reimburse the cost of the gasoline. Domingo Lucero, the owner of said vulcanizing shop, corroborated this explanation in an affidavit dated June 25, 1990. 6 With respect to the accusation that he sought reimbursement in the amount of P660.00 for one vulcanizing job, Lumiqued submitted that the amount was actually only P6.60. Any error committed in posting the amount in the books of the Regional Office was not his personal error or accountability. To refute private respondent's allegation that he violated COA rules and regulations in incurring unliquidated cash advances in the amount of P116,000.00, Lumiqued presented a certification 7 of DAR-CAR Administrative Officer Deogracias F. Almora that he had no outstanding cash advances on record as of December 31, 1989. In disputing the charges of oppression and harassment against him, Lumiqued contended that private respondent was not terminated from the service but was merely relieved of her duties due to her prolonged absences. While admitting that private respondent filed the required applications for leave of absence, Lumiqued claimed that the exigency of the service necessitated disapproval of her application for leave of absence. He allegedly rejected her second application for leave of absence in view of her failure to file the same immediately with the head office or upon her return to work.

He also asserted that no medical certificate supported her application for leave of absence. In the same counter-affidavit, Lumiqued also claimed that private respondent was corrupt and dishonest because a COA examination revealed that her cash accountabilities from June 22 to November 23, 1989, were short by P30,406.87. Although private respondent immediately returned the amount on January 18, 1990, the day following the completion of the cash examination, Lumiqued asserted that she should be relieved from her duties and assigned to jobs that would not require handling of cash and money matters. Committee hearings on the complaints were conducted on July 3 and 10, 1992, but Lumiqued was not assisted by counsel. On the second hearing date, he moved for its resetting to July 17, 1992, to enable him to employ the services of counsel. The committee granted the motion, but neither Lumiqued nor his counsel appeared on the date he himself had chosen, so the committee deemed the case submitted for resolution. On August 12, 1992, Lumiqued filed an urgent motion for additional hearing, 8 alleging that he suffered a stroke on July 10, 1992. The motion was forwarded to the Office of the State Prosecutor apparently because the investigation had already been terminated. In an order dated September 7, 1992, 9 State Prosecutor Zoila C. Montero denied the motion, viz:
The medical certificate given show(s) that respondent was discharged from the Sacred Heart Hospital on July 17, 1992, the date of the hearing, which date was upon the request of respondent (Lumiqued). The records do not disclose that respondent advised the Investigating committee of his confinement and inability to attend despite his discharge, either by himself or thru counsel. The records likewise do not show that efforts were exerted to notify the Committee of respondent's condition on any reasonable date after July 17, 1992. It is herein noted that as early as June 23, 1992, respondent was already being assisted by counsel. Moreover an evaluation of the counter-affidavit submitted reveal(s) the sufficiency, completeness and thoroughness of the counter-affidavit together with the documentary evidence annexed thereto, such that a judicious determination of the case based on the pleadings submitted is already possible. Moreover, considering that the complaint-affidavit was filed as far back as November 16, 1989 yet, justice can not be delayed much longer.

Following the conclusion of the hearings, the investigating committee rendered a report dated July 31, 1992, 10 finding Lumiqued liable for all the charges against him. It made the following findings:
After a thorough evaluation of the evidences (sic) submitted by the parties, this committee finds the evidence submitted by the complainant sufficient to establish the guilt of the respondent for Gross Dishonesty and Grave Misconduct.

That most of the gasoline receipts used by the respondent in claiming for the reimbursement of his gasoline expenses were falsified is clearly established by the 15 Certified Xerox Copies of the duplicate receipts (Annexes G-1 to G-15) and the certifications issued by the different gasoline stations where the respondent purchased gasoline. Annexes "G-1" to "G-15" show that the actual average purchase made by the respondent is about 8.46 liters only at a purchase price of P50.00, in contrast to the receipts used by the respondent which reflects an average of 108.45 liters at a purchase price of P550.00. Here, the greed of the respondent is made manifest by his act of claiming reimbursements of more than 10 times the value of what he actually spends. While only 15 of the gasoline receipts were ascertained to have been falsified, the motive, the pattern and the scheme employed by the respondent in defrauding the government has, nevertheless, been established. That the gasoline receipts have been falsified was not rebutted by the respondent. In fact, he had in effect admitted that he had been claiming for the payment of an average consumption of 108.45 liters/day by justifying that this was being used by the 4 vehicles issued to his office. Besides he also admitted having signed the receipts. Respondent's act in defrauding the government of a considerable sum of money by falsifying receipts constitutes not only Dishonesty of a high degree but also a criminal offense for Malversation through Falsification of Official Documents. This committee likewise finds that the respondent have (sic) unliquidated cash advances in the year 1989 which is in violation of established office and auditing rules. His cash advances totaling to about P116,000.00 were properly documented. The requests for obligation of allotments and the vouchers covering the amounts were all signed by him. The mere certification issued by the Administrative Officer of the DAR-CAR cannot therefore rebut these concrete evidences (sic). On the third complaint, this committee likewise believes that the respondent's act in relieving the complainant of her functions as a Regional Cashier on December 1, 1989 was an act of harassment. It is noted that this was done barely two weeks after the complainant filed charges against her (sic). The recommendation of Jose G. Medina of the Commission on Audit came only on May 11, 1990 or almost six months after the respondent's order relieving the complainant was issued. His act in harassing a subordinate employee in retaliation to a complaint she filed constitute(s) Gross Misconduct on the part of the respondent who is a head of office. The affidavits of Joseph In-uyay and Josefina Guting are of no help to the respondent. In fact, this only show(s) that he is capable of giving bribes if only to have the cases against him dismissed. He could not have given a certain Benigno Aquino III the sum of P10,000.00 for any other purpose.

Accordingly, the investigating committee recommended Lumiqued's dismissal or removal from office, without prejudice to the filing of the appropriate criminal charges against him. Acting on the report and recommendation, former Justice Secretary Franklin M. Drilon adopted the same in his Memorandum to President Fidel V. Ramos dated October 22, 1992. He added that the filing of the affidavit of desistance 11 would not prevent the issuance of a resolution on the matter considering that what was at stake was not only "the violation of complainant's (herein private respondent's) personal rights" but also "the competence and fitness of the respondent (Lumiqued) to remain in public office."

He opined that, in fact, the evidence on record could call for "a punitive action against the respondent on the initiative of the DAR." On December 17, 1992, Lumiqued filed a motion for reconsideration of "the findings of the Committee" with the DOJ. 12 Undersecretary Ramon S. Esguerra indorsed the motion to the investigating committee. 13 In a letter dated April 1, 1993, the threemember investigating committee informed Undersecretary Esguerra that the committee "had no more authority to act on the same (motion for reconsideration) considering that the matter has already been forwarded to the Office of the President" and that their authority under Department Order No. 145 ceased when they transmitted their report to the DOJ. 14 Concurring with this view, Undersecretary Esguerra informed Lumiqued that the investigating committee could no longer act on his motion for reconsideration. He added that the motion was also prematurely filed because the Office of the President (OP) had yet to act on Secretary Drilon's recommendation. 15 On May 12, 1993, President Fidel V. Ramos himself issued Administrative Order No. 52 (A.O. No. 52), 16 finding Lumiqued administratively liable for dishonesty in the alteration of fifteen gasoline receipts, and dismissing him from the service, with forfeiture of his retirement and other benefits. Thus:
That the receipts were merely turned over to him by his drivers and that the auditor and accountant of the DAR-CAR should be the ones to be held liable is untenable. The receipts in question were signed by respondent for the purpose of attesting that those receipts were validly issued by the commercial establishments and were properly disbursed and used in the official business for which it was intended. This Office is not about to shift the blame for all these to the drivers employed by the DAR-CAR as respondent would want us to do.

The OP, however, found that the charges of oppression and harassment, as well as that of incurring unliquidated cash advances, were not satisfactorily established. In a "petition for appeal" 17 addressed to President Ramos, Lumiqued prayed that A.O. No. 52 be reconsidered and that he be reinstated to his former position "with all the benefits accorded to him by law and existing rules and regulations." This petition was basically premised on the affidavit dated May 27, 1993, of a certain Dwight L. Lumiqued, a former driver of the DAR-CAR, who confessed to having authored the falsification of gasoline receipts and attested to petitioner Lumiqued's being an "honest man" who had no "premonition" that the receipts he (Dwight) turned over to him were "altered." 18 Treating the "petition for appeal" as a motion for reconsideration of A.O. No. 52, the OP, through Senior Deputy Executive Secretary Leonardo A. Quisumbing, denied the same on August 31, 1993. Undaunted, Lumiqued filed a second motion for reconsideration, alleging, among other things, that he was denied the constitutional right to counsel during the hearing. 19 On

May 19, 1994, 20 however, before his motion could be resolved, Lumiqued died. On September 28, 1994, 21 Secretary Quisumbing denied the second motion for reconsideration for lack of merit. Hence, the instant petition for certiorari and mandamus praying for the reversal of the Report and Recommendation of the Investigating Committee, the October 22, 1992, Memorandum of then Justice Secretary Drilon, A.O. No. 52 issued by President Ramos, and the orders of Secretary Quisumbing. In a nutshell, it prays for the "payment of retirement benefits and other benefits accorded to deceased Arsenio Lumiqued by law, payable to his heirs; and the backwages from the period he was dismissed from service up to the time of his death on May 19, 1994." 22 Petitioners fault the investigating committee for its failure to inform Lumiqued of his right to counsel during the hearing. They maintain that his right to counsel could not be waived unless the waiver was in writing and in the presence of counsel. They assert that the committee should have suspended the hearing and granted Lumiqued a reasonable time within which to secure a counsel of his own. If suspension was not possible, the committee should have appointed a counsel de oficio to assist him. These arguments are untenable and misplaced. The right to counsel, which cannot be waived unless the waiver is in writing and in the presence of counsel, is a right afforded a suspect or an accused during custodial investigation. 23 It is not an absolute right and may, thus, be invoked or rejected in a criminal proceeding and, with more reason, in an administrative inquiry. In the case at bar, petitioners invoke the right of an accused in criminal proceedings to have competent and independent counsel of his own choice. Lumiqued, however, was not accused of any crime in the proceedings below. The investigation conducted by the committee created by Department Order No. 145 was for the purpose of determining if he could be held administratively liable under the law for the complaints filed against him. The order issued by Acting Secretary of Justice Montenegro states thus:
In the interest of the public service and pursuant to the provisions of existing laws, a Committee to conduct the formal investigation of the administrative complaint for oppression, dishonesty, disgraceful and immoral conduct, being notoriously undesirable and conduct prejudicial to the best interest of the service against Mr. ARSENIO P. LUMIQUED, Regional Director, Department of Agrarian Reform, Cordillera Autonomous 24 Region, is hereby created . . .

As such, the hearing conducted by the investigating committee was not part of a criminal prosecution. This was even made more pronounced when, after finding Lumiqued administratively liable, it hinted at the filing of a criminal case for malversation through falsification of public documents in its report and recommendation. Petitioners' misconception on the nature of the investigation 25 conducted against Lumiqued appears to have been engendered by the fact that the DOJ conducted it. While it is true that under the Administrative Code of 1987, the DOJ shall "administer

the criminal justice system in accordance with the accepted processes thereof consisting in the investigation of the crimes, prosecution of offenders and administration of the correctional system, 26 conducting criminal investigations is not its sole function. By its power to "perform such other functions as may be provided by law," 27 prosecutors may be called upon to conduct administrative investigations. Accordingly, the investigating committee created by Department Order No. 145 was duty-bound to conduct the administrative investigation in accordance with the rules therefor. While investigations conducted by an administrative body may at times be akin to a criminal proceeding, the fact remains that under existing laws, a party in an administrative inquiry may or may not be assisted by counsel, irrespective of the nature of the charges and of the respondent's capacity to represent himself, and no duty rests on such a body to furnish the person being investigated with counsel. 28 In an administrative proceeding such as the one that transpired below, a respondent (such as Lumiqued) has the option of engaging the services of counsel or not. This is clear from the provisions of Section 32, Article VII of Republic Act No. 2260 29 (otherwise known as the Civil Service Act) and Section 39, paragraph 2, Rule XIV (on Discipline) of the Omnibus Rules Implementing Book V of Executive Order No. 292 30 (otherwise known as the Administrative Code of 1987). Excerpts from the transcript of stenographic notes of the hearings attended by Lumiqued 31 clearly show that he was confident of his capacity and so opted to represent himself . Thus, the right to counsel is not imperative in administrative investigations because such inquiries are conducted merely to determine whether there are facts that merit disciplinary measures against erring public officers and employees, with the purpose of maintaining the dignity of government service. Furthermore, petitioners' reliance on Resolution No. 94-0521 of the Civil Service Commission on the Uniform Procedure in the Conduct of Administrative Investigation stating that a respondent in an administrative complaint must be "informed of his right to the assistance of a counsel of his choice," 32 is inappropriate. In the first place, this resolution is applicable only to cases brought before the Civil Service Commission. 33 Secondly, said resolution, which is dated January 25, 1994, took effect fifteen days following its publication in a newspaper of general circulation, 34 much later than the July 1992 hearings of the investigating committee created by Department Order No. 145. Thirdly, the same committee was not remiss in the matter of reminding Lumiqued of his right to counsel. Thus, at the July 3, 1992, hearing, Lumiqued was repeatedly appraised of his option to secure the services of counsel:
RSP EXEVEA: This is an administrative case against Director Lumiqued. Director Lumiqued is present. The complainant is present, Janet Obar-Zamudio. Complainant has just been furnished with a copy of the counter-affidavit of the respondent. Do you have a counsel, Director? DIR. LUMIQUED:

I did not bring anybody, Sir, because when I went to see him, he told me, Sir, that he has already set a hearing, morning and afternoon today. RSP EXEVEA: So, we will proceed with the hearing even without your counsel? You are willing to proceed with the hearing even without your counsel? DIR. LUMIQUED: Yes, I am confident. . . CP BALAJADIA: You are confident that you will be able to represent yourself? DIR. LUMIQUED: That is my concern.
35

(Emphasis supplied)

In the course of private respondent's damaging testimony, the investigating committee once again reminded Lumiqued of his need for a counsel. Thus:
CP BALAJADIA: Q. (To Director Lumiqued) You really wish to go through with this even without your counsel? DIRECTOR LUMIQUED: A. I think so, Sir. CP BALAJADIA: Let us make it of record that we have been warning you to proceed with the assistance of counsel but you said that you can take care of yourself 36 so we have no other alternative but to proceed. (Emphasis supplied).

Thereafter, the following colloquies transpired:


CP BALAJADIA: We will suspend in the meantime that we are waiting for the supplemental affidavit you are going to present to us. Do you have any request from the panel of investigators, Director Lumiqued? DIRECTOR LUMIQUED: I was not able to bring a lawyer since the lawyer I requested to assist me and was the one who prepared my counter-affidavit is already engaged

for a hearing and according to him he is engaged for the whole month of July. RSP EXEVEA: We cannot wait . . . CP BALAJADIA: Why don't you engage the services of another counsel. The charges against you are quite serious. We are not saying you are guilty already. We are just apprehensive that you will go through this investigation without a counsel. We would like you to be protected legally in the course of this investigation. Why don't you get the services of another counsel. There are plenty here in Baguio . . . DIRECTOR LUMIQUED: I will try to see, Sir . . . CP BALAJADIA: Please select your date now, we are only given one month to finish the investigation, Director Lumiqued. RSP EXEVEA: We will not entertain any postponement. With or without counsel, we will proceed. CP BALAJADIA: Madam Witness, will you please submit the document which we asked for and Director Lumiqued, if you have other witnesses, please bring them but reduce their testimonies in affidavit form so that we can 37 expedite with the proceedings.

At the hearing scheduled for July 10, 1992, Lumiqued still did not avail of the services of counsel. Pertinent excerpts from said hearing follow:
FISCAL BALAJADIA: I notice also Mr. Chairman that the respondent is not being represented by a counsel. The last time he was asked to invite his lawyer in this investigation. May we know if he has a lawyer to represent him in this investigation? DIR. LUMIQUED: There is none Sir because when I went to my lawyer, he told me that he had set a case also at 9:30 in the other court and he told me if there is a

possibility of having this case postponed anytime next week, probably Wednesday so we will have good time (sic) of presenting the affidavit. FISCAL BALAJADIA: Are you moving for a postponement Director? May I throw this to the panel. The charges in this case are quite serious and he should be given a chance to the assistance of a counsel/lawyer. RSP EXEVEA: And is (sic) appearing that the supplemental-affidavit has been furnished him only now and this has several documents attached to it so I think we could grant him one last postponement considering that he has already asked for an extension. DIR. LUMIQUED: Furthermore Sir, I am now being bothered by my heart ailment.
38

The hearing was reset to July 17, 1992, the date when Lumiqued was released from the hospital. Prior to said date, however, Lumiqued did not inform the committee of his confinement. Consequently because the hearing could not push through on said date, and Lumiqued had already submitted his counter-affidavit, the committee decided to wind up the proceedings. This did not mean, however, that Lumiqued was shortchanged in his right to due process. Lumiqued, a Regional Director of a major department in the executive branch of the government, graduated from the University of the Philippines (Los Baos) with the degree of Bachelor of Science major in Agriculture, was a recipient of various scholarships and grants, and underwent training seminars both here and abroad. 39 Hence, he could have defended himself if need be, without the help of counsel, if truth were on his side. This, apparently, was the thought he entertained during the hearings he was able to attend. In his statement, "That is my concern," one could detect that it had been uttered testily, if not exasperatedly, because of the doubt or skepticism implicit in the question, "You are confident that you will be able to represent yourself?" despite his having positively asserted earlier, "Yes, I am confident." He was obviously convinced that he could ably represent himself. Beyond repeatedly reminding him that he could avail himself of counsel and as often receiving the reply that he is confident of his ability to defend himself, the investigating committee could not do more. One can lead a horse to water but cannot make him drink. The right to counsel is not indispensable to due process unless required by the Constitution or the law. In Nera v. Auditor General, 40 the Court said:
. . . There is nothing in the Constitution that says that a party in a non-criminal proceeding is entitled to be represented by counsel and that, without such representation, he shall not be bound by such proceedings. The assistance of lawyers; while desirable, is not indispensable. The legal profession was not engrafted in the due process clause such

that without the participation of its members, the safeguard is deemed ignored or violated. The ordinary citizen is not that helpless that he cannot validly act at all except only with a lawyer at his side.

In administrative proceedings, the essence of due process is simply the opportunity to explain one's side. One may be heard, not solely by verbal presentation but also, and perhaps even much more creditably as it is more practicable than oral arguments, through pleadings. 41 An actual hearing is not always an indispensable aspect of due process. 42 As long as a party was given the opportunity to defend his interests in due course; he cannot be said to have been denied due process of law, for this opportunity to be heard is the very essence of due process. 43 Moreover, this constitutional mandate is deemed satisfied if a person is granted an opportunity to seek reconsideration of the action or ruling complained of. 44 Lumiqued's appeal and his subsequent filing of motions for reconsideration cured whatever irregularity attended the proceedings conducted by the committee. 45 The constitutional provision on due process safeguards life, liberty and property. 46 In the early case of Cornejo v. Gabriel and Provincial Board of Rizal 47 the Court held that a public office is not property within the sense of the constitutional guarantee of due process of law for it is a public trust or agency. This jurisprudential pronouncement has been enshrined in the 1987 Constitution under Article XI, Section 1, on accountability of public officers, as follows:
Sec. 1. Public office is a public trust. Public officers and employees must at all times be accountable to the people, serve them with utmost responsibility, integrity, loyalty, and efficiency, act with patriotism and justice, and lead modest lives.

When the dispute concerns one's constitutional right to security of tenure, however, public office is deemed analogous to property in a limited sense; hence, the right to due process could rightfully be invoked. Nonetheless, the right to security of tenure is not absolute. Of equal weight is the countervailing mandate of the Constitution that all public officers and employees must serve with responsibility, integrity, loyalty and efficiency. 48 In this case, it has been clearly shown that Lumiqued did not live up to this constitutional precept. The committee's findings pinning culpability for the charges of dishonesty and grave misconduct upon Lumiqued were not, as shown above, fraught with procedural mischief. Its conclusions were founded on the evidence presented and evaluated as facts. Well-settled in our jurisdiction is the doctrine that findings of fact of administrative agencies must be respected as long as they are supported by substantial evidence, even if such evidence is not overwhelming or preponderant. 49 The quantum of proof necessary for a finding of guilt in administrative cases is only substantial evidence or such relevant evidence as a reasonable mind might accept as adequate to support a conclusion. 50 Consequently, the adoption by Secretary Drilon and the OP of the committee's recommendation of dismissal may not in any way be deemed tainted with arbitrariness

amounting to grave abuse of discretion. Government officials are presumed to perform their functions with regularity. Strong evidence is not necessary to rebut that presumption, 51 which petitioners have not successfully disputed in the instant case. Dishonesty is a grave offense penalized by dismissal under Section 23 of Rule XIV of the Omnibus Rules Implementing Book V of the Administrative Code of 1987. Under Section 9 of the same Rule, the penalty of dismissal carries with it "cancellation of eligibility, forfeiture of leave credits and retirement benefits, and the disqualification for reemployment in the government service." The instant petition, which is aimed primarily at the "payment of retirement benefits and other benefits," plus back wages from the time of Lumiqued's dismissal until his demise, must, therefore, fail. WHEREFORE, the instant petition for certiorari and mandamus is hereby DISMISSED and Administrative Order no. 52 of the Office of the President is AFFIRMED. Costs against petitioners. SO ORDERED.

Atienza vs. Comelec


FACTS: Private respondent Antonio G. Sia was elected mayor of theMunicipality of Madrilejos, Cebu in the 1988 local elections obtaininga plurality of 126 votes over his nearest rival, herein petitioner Lou A.Atienza. Following Sia's proclamation by the Municipal Board of Canvassers,petitioner filed an election protest with the Regional Trial Courtquestioning the results of the elections in a number of precincts in themunicipality. Consequently, in the revision ordered by the lower court, petitionerobtained a total of 2,826 votes, a plurality of 12 votes ove r theprivate respondent. On April 12, 1989 the Regional Trial Court rendered its decision declaring petitioner the winner of the municipal elections andordering the private respondent to reimburse petitioner the amountof P300,856.19 representing petitioner's expenses in the election protest. Private respondent appealed the trial court's decision to the COMELECraising as errors 1) the computation of the number of votes receivedby the candidates; and 2) the alleged award of "excessive damages"in favor of the petitioner. The case was docketed and assigned to theCOMELEC's Second Division. The COMELEC, en banc , issued an Order setting aside the preliminaryinjunction and thereby allowing petitioner to assume as mayor of theMunicipality of Madrilejos pending resolution of his appeal. However, following the synchronized elections of May 11, 1992, thePresiding Commissioner of the COMELEC's Second Division issued anOrder dated July 18, 1992 dismissing petitioner's appeal for being moot and academic pursuant to the Commission's decision inResolution No. 2494 declaring the election protest and appeal

as of June 30,1992. On January 28, 1993, respondent Commission en banc released itsquestioned resolution, the dispositive portion of which states:PREMISES CONSIDERED, the Commission RESOLVED, as it herebyRESOLVES, that the dismissal of the appeal by the Commission ( Second Division ) f o r b e i n g m o o t a n d a c a d e m i c b e c a u s e o f t h e expiration of the term of office of the contested position did not thereby revive the vacated judgment of the Regional Trial Court, saidappealed judgment to remain vacated, not having been resolved onthe merits by the Commission for or against any of the parties; andthe judgment directing theprotestee-appellant to reimburse theprotestant-appellee the amount of P300,856.19 representing his expenses in the election protest, is hereby REVERSED , said judgmentnot being in accordance with law in the absence of any evidence of a n y w r o n g f u l , o r n e g l i g e n t a c t o r o m i s s i o n o n t h e p a r t o f t h e protestee appellant to justify the award. ISSUE(S)/HELD: Whether the COMELEC acted with grave abuse of discretion when itissued its Resolution of January 28, 1993 reversing the lower court's judgmentawarding damages to herein petitioner after it had earlier dismissed for beingmoot and academic. NO.RATIO: The Omnibus Election Code provides: Actual or compensatorydamages may be granted in all election contests or in quo warranto proceedings in accordance with law. Provisions for actual or compensatory damages under the law are embodied in various Civil Code articles allowing claims for damagesunder specific circumstances. Thus, Article 2176 provides: Whoeverby act or omission causes damage to another, there being fault ornegligence, is obliged to pay for the damage done. Such fault or negligence, if there is no pre-existing contractual relation betweenthe parties is called a quasi delict , and is governed by the provisionsof this chapter. Specifically, Article 2199 of the Civil Code mandates that: Except asprovided by law or by stipulation, one is entitled to an adequate compensation only for such pecuniary loss suffered by him as he hasduly proved. Such compensation is referred to as actual orcompensatory damages. Given this setting, it would appear virtually impossible for a party inan election protest case to recover actual or compensatory damagesin the absence of the co nditions specified under Articles 2201 and 2202 of the Civil Code, or in the absence of a law expressly providingfor situations allowing for the recovery of the same . I t f o l l o w s , naturally, that in most election protest cases where the monetaryclaim does no t hinge on either a contract or quasi -contract or atortious act or omission, the claimant must be able to point out to aspecific provision of law authorizing a money claim for electionprotest expenses against the losing party . This, petitioner has beenunable to do. Section 259 of the Omnibus Election Code merely provides for thegranting of actual and compensatory damages in accordance with l a w . T h a t i t w a s t h e i n t e n t o f t h e l e g i s l a t u r e t o d o a w a y w i t h provisions indemnifying the victorious party for expenses incurred inan election contest in the absence of a wrongful act or omission clearly attributable to the losing party cannot be gainsaid. The intent,moreover, to do away with such provisions merely recognizes themaxim, settled in law that a

wrong without damage or damage without wrong neither constitutes a cause of action nor creates a civil obligation.

People of the Philippines vs Cayat


Equal Protection Requisites of a Valid Classification Bar from Drinking Gin In 1937, there exists a law (Act 1639) which bars native non-Christians from drinking gin or any other liquor outside of their customary alcoholic drinks. Cayat, a native of the Cordillera, was caught with an A-1-1 gin in violation of this Act. He was then charged and sentenced to pay P5.00 and to be imprisoned in case of insolvency. Cayat admitted his guilt but he challenged the constitutionality of the said Act. He averred, among others, that it violated his right to equal protection afforded by the constitution. He said this an attempt to treat them with discrimination or mark them as inferior or less capable race and less entitled will meet with their instant challenge. The law sought to distinguish and classify native non-Christians from Christians. ISSUE: Whether or not the said Act violates the equal protection clause. HELD: The SC ruled that Act 1639 is valid for it met the requisites of a reasonable classification. The SC emphasized that it is not enough that the members of a group have the characteristics that distinguish them from others. The classification must, as an indispensable requisite, not be arbitrary. The requisites to be complied with are; (1) must rest on substantial distinctions; (2) must be germane to the purposes of the law; (3) must not be limited to existing conditions only; and (4) must apply equally to all members of the same class. Act No. 1639 satisfies these requirements. The classification rests on real or substantial, not merely imaginary or whimsical, distinctions. It is not based upon accident of birth or parentage. The law, then, does not seek to mark the non-Christian tribes as an inferior or less capable race. On the contrary, all measures thus far adopted in the promotion of the public policy towards them rest upon a recognition of their inherent right to equality in the enjoyment of those privileges now enjoyed by their Christian brothers. But as there can be no true equality before the law, if there is, in fact, no equality in education, the government has endeavored, by appropriate measures, to raise their culture and civilization and secure for them the benefits of their progress, with the ultimate end in view of placing them with their Christian brothers on the basis of true equality.
G.R. No. 78742: Association of Small Landowners vs Secretary of Agrarian Reform

Equal Protection

These are 3 cases consolidated questioning the constitutionalityof the Agrarian Reform Act. Article XIII on Social Justice and Human Rightsincludes a call for the adoption by the State of an agrarian reform program.The State shall, by law, undertake an agrarian reform program founded on theright of farmers and regular farmworkers, who are landless, to own directly orcollectively the lands they till or, in the case of other farmworkers, toreceive a just share of the fruits thereof. RA 3844, Agricultural Land ReformCode, had already been enacted by Congress on August 8, 1963. This wassubstantially superseded almost a decade later by PD 27, which was promulgatedon Oct 21, 1972, along with martial law, to provide for the compulsoryacquisition of private lands for distribution among tenant-farmers and tospecify maximum retention limits for landowners. On July 17, 1987, Cory issuedEO 228, declaring full land ownership in favor of the beneficiaries of PD 27and providing for the valuation of still unvalued lands covered by the decreeas well as the manner of their payment. This was followed on July 22, 1987 byPP 131, instituting a comprehensive agrarian reform program (CARP), and EO 229,providing the mechanics for its implementation. Afterwhich is the enactment ofRA 6657, Comprehensive Agrarian Reform Law of 1988, which Cory signed on June10. This law, while considerably changing the earlier mentioned enactments,nevertheless gives them suppletory effect insofar as they are not inconsistentwith its provisions.

Inconsidering the rentals as advance payment on the land, the executive orderalso deprives the petitioners of their property rights as protected by dueprocess. The equal protection clause is also violated because the order placesthe burden of solving the agrarian problems on the owners only of agriculturallands. No similar obligation is imposed on the owners of other properties.

Thepetitioners maintain that in declaring the beneficiaries under PD 27 to be theowners of the lands occupied by them, EO 228 ignored judicial prerogatives andso violated due process. Worse, the measure would not solve the agrarianproblem because even the small farmers are deprived of their lands and theretention rights guaranteed by the Constitution.

In hiscomment the Sol-Gen asserted that the alleged violation of the equal protectionclause, the sugar planters have failed to show that they belong to a differentclass and should be differently treated.

The Comment also suggests the possibilityof Congress first distributing public agricultural lands and scheduling theexpropriation of private agricultural lands later. From this viewpoint, thepetition for prohibition would be premature.

ISSUE: Whether or notthere was a violation of the equal protection clause.

HELD: The SC ruledaffirming the Sol-Gen. The argument of the small farmers that they have beendenied equal protection because of the absence of retention limits has alsobecome academic under Sec 6 of RA 6657. Significantly, they too have notquestioned the area of such limits. There is also the complaint that theyshould not be made to share the burden of agrarian reform, an objection alsomade by the sugar planters on the ground that they belong to a particular classwith particular interests of their own. However, no evidence has been submittedto the Court that the requisites of a valid classification have been violated.

Classification has been defined as the grouping of persons orthings similar to each other in certain particulars and different from eachother in these same particulars. To be valid, it must conform to the followingrequirements:

(1) it must be based on substantial distinctions; (2) it must be germane to the purposes of the law; (3) it must not be limited to existing conditions only; and (4) it must apply equally to all the members of the class.

The Court finds that all these requisites have been met by themeasures here challenged as arbitrary and discriminatory.

Equal protection simply means that all persons or thingssimilarly situated must be treated alike both as to the rights conferred andthe liabilities imposed. The petitioners have not shown that they belong to adifferent class and entitled to a different treatment. The argument that notonly landowners but also owners of other properties must be made to share theburden of implementing land reform must be rejected. There is a substantialdistinction between these two classes of owners that is clearly visible exceptto those who will not see. There is no need to elaborate on this matter. In anyevent, the Congress is allowed a wide leeway in providing for a validclassification. Its decision is accorded recognition and respect by the courtsof justice except only where its discretion is abused to the detriment of theBill of Rights.

Quinto vs Comelec G. R. No. 189698 FACTS: Petitioners Eleazar P. Quinto and Gerino A. Tolentino, Jr. filed a petition for certiorari and prohibition against the COMELEC for issuing a resolution declaring appointive officials who filed their certificate of candidacy as ipso facto resigned from their positions. In this defense, the COMELEC avers that it only copied the provision from Sec. 13 of R.A. 9369. ISSUE: Whether or not the said COMELEC resolution was valid. HELD: NO. In the Fari case, the petitioners challenged Sec. 14 of RA. 9006 repealing Sec. 66 of the as Omnibus Election Code (OEC) for giving undue benefit to elective officials in comparison with appointive officials. Incidentally, the Court upheld the substantial distinctions between the two and pronounced that there was no violation of the equal protection clause. However in the present case, the Court held that the discussion on the equal protection clause was an obiter dictum since the issue raised therein was against the repealing clause. It didn squarely challenge Sec. 66. t Sec. 13 of RA. 9369 unduly discriminated appointive and elective officials. Applying the 4 requisites of a valid classification, the proviso does not comply with the second requirement that it must be germane to the purpose of the law. The obvious reason for the challenged provision is to prevent the use of a governmental position to promote one candidacy, or even to wield a dangerous or coercive influence of the electorate. The s measure is further aimed at promoting the efficiency, integrity, and discipline of the public service by eliminating the danger that the discharge of official duty would be motivated by political considerations rather than the welfare of the public. The restriction is also justified by the proposition that the entry of civil servants to the electorate arena, while still in office, could result in neglect or inefficiency in the performance of duty because they would be attending to their campaign rather than to their office work. Sec. 13 of RA. 9369 pertains to all civil servants holding appointive posts without distinction as to whether they occupy high positions in government or not. Certainly, a utility worker in the government will also be considered as ipso facto resigned once he files his certificate of candidacy for the election. This scenario is absurd for, indeed, it is unimaginable how he can use his position in the government to wield influence in the political world. The provision s directed to the activity any and all public offices, whether they be partisan or non partisan in character, whether they be in the national, municipal or barangay level. Congress has not shown a compelling state interest to restrict the fundamental right involved on such a sweeping scale. MOTION FOR RECONSIDERATION

FACTS: This is a motion for reconsideration filed by the Commission on Elections. The latter moved to question an earlier decision of the Supreme Court declaring Section 4 (a) of COMELEC Resolution No. 8678 unconstitutional. Section 4 (a) of COMELEC Resolution No. 8678 provides that, Any person holding a public appointive office or position including active members of the Armed Forces of the Philippines, and other officers and employees in government-owned or controlled corporations, shall be considered ipso facto resigned from his office upon the filing of his certificate of candidacy.Be it noted that petitioners of the above-entitled case are appointive officials who intend to be elected in the previously held 2010 elections and who felt aggrieved by the issuance of the questioned resolution. ISSUE: Whether or not Section 4 (a) of COMELEC Resolution No. 8678 is constitutional. RULING: The Supreme Court overruled its previous decision declaring the assailed resolution unconstitutional. Here, it strongly upholds the constitutionality of the resolution saying that it does not violate the equal protection clause. It is settled that the equal protection clause does not demand absolute equality; it merely requires that all persons shall be treated alike, under like circumstances and conditions both as to privileges conferred and liabilities enforced. The test used is reasonableness which requires that: 1. The classification rests on substantial distinctions; 2. It is germane to the purposes of the law; 3. It is not limited to existing conditions only; and 4. It applies equally to all members of the same class. In the case under consideration, there is a substantial distinction between public and elective officials which has been rendered moot and academic by the ruling made in the case of Farinas, etl. al. vs. Executive Secretary, et. al. Section 4 (a) of COMELEC Resolution No. 8678 is constitutional.

Roma Drug & Romeo Rodriguez vs RTC of Guagua, Pampanga et al


Equal Protection Access to Medicine Roma Drug, owned by Rodriguez, was raided by the NBI upon request of Smithkline a pharmaceutical company (now Glaxo Smithkline). RD is apparently one of 6 pharmacies who are directly importing 5 medicines produced by Smithknline from abroad. RD is not purchasing those medicines via local Smithkline the authorized distributor of Smithkline in the Philippines. Smithkline Phil avers that because the medicines were not purchased from a Philippine registered counterpart of Smithkline then the products imported by RD are considered

as counterfeit or unregistered imported drug product - as defined by RA 8203 Special Law on Counterfeit Drugs. Notwithstanding RDs motion for reconsideration, the provincial prosecutor recommended that Rodriguez be tried. Rodriguez assails the constitutionality of SLCD averring, among other things, that it has violated his right to equal protection as it banned him access from such medicines. ISSUE: Whether or not SLCD violates equal protection. HELD: The SC ruled in favor of RD. The SC denounced SLCD for it violated equal protection. It does not allow private 3rd parties to import such medicines abroad even in cases of life and death nor does it allow the importation by 3rd parties in cases wherein the stocks of such medicine would run out. It discriminates at the expense of Filipinos who cannot travel abroad to purchase such medicines yet need them badly. Nevertheless, the flawed intention of Congress had been abrogated by the passage of RA 9502 Universally Accessible Cheaper and Quality Medicines Act of 2008 and its IRR. This law does not expressly repeal SLCD but it emphasized that any medicine introduced into the Philippines by its patent holder be accessible to anyone. It provides that the right to import drugs and medicines shall be available to any government agency OR ANY PRIVATE 3rd PARTY. The SC noted that this law provided and recognized the constitutionally-guaranteed right of the public to health.
G.R. No. L-45685: People vs Vera

Delegation of Powers

Cu Unjieng was convicted by the trial court in Manila. He filed for reconsideration which was elevated to the SC and the SC remanded the appeal to the lower court for a new trial. While awaiting new trial, he appealed for probation alleging that the he is innocent of the crime he was convicted of. Judge Tuason of the Manila CFI directed the appeal to the Insular Probation Office. The IPO denied the application. However, Judge Vera upon another request by petitioner allowed the petition to be set for hearing. The City Prosecutor countered alleging that Vera has no power to place Cu Unjieng under probation because it is in violation of Sec. 11 Act No. 4221 which provides that the act of Legislature granting provincial boards the power to provide a system of probation to convicted person. Nowhere in the law is stated that the law is applicable to a city like Manila because it is only indicated therein that only provinces are covered. And even if Manila is covered by the law it is unconstitutional because Sec 1 Art 3 of the Constitution provides equal protection of laws. The said law provides absolute discretion to provincial boards and this also constitutes undue delegation of power. Further, the said probation law may be an encroachment of the power of the executive to provide pardon because providing probation, in effect, is granting freedom, as in pardon.

ISSUE: Whether or not there is undue delegation of power.

HELD: The act of granting probation is not the same as pardon. In fact it is limited and is in a way an imposition of penalty. There is undue delegation of power because there is no set standard provided by Congress on how provincial boards must act in carrying out a system of probation. The provincial boards are given absolute discretion which is violative of the constitution and the doctrine of the non delegability of power. Further, it is a violation of equity so protected by the constitution. The challenged section of Act No. 4221 in section 11 which reads as follows: This Act shall apply only in those provinces in which the respective provincial boards have provided for the salary of a probation officer at rates not lower than those now provided for provincial fiscals. Said probation officer shall be appointed by the Secretary of Justice and shall be subject to the direction of the Probation Office. This only means that only provinces that can provide appropriation for a probation officer may have a system of probation within their locality. This would mean to say that convicts in provinces where no probation officer is instituted may not avail of their right to probation.
G.R. No. 126594: Imelda Marcos vs Court of Appeals, Manila RTC Judge Guillermo Loja Sr., et al

EqualProtection

Marcos was charged for violating Central Bank Circ 960 whichbanned residents, firms, associations and corporations from maintaining foreignexchange accounts abroad w/o permission from the CB. The circular was issued in1983. Any violation thereof constitutes a criminal offense. In 1991, 8informations were filed against Marcos accusing her of maintaining a foreignaccount in Switzerland from 1968-1991. On 21 Dec 1991, 14 more informationswere filed against Marcos, Benedicto and Rivera for the same offense. InJanuary 1992, 11 more informations were filed. The RTC consolidated the casesand Marcos was arraigned in Feb 1992. During the pendency of these cases, CBCirc 1318 and CB Circ 1353 (Further Liberalizing Foreign Exchange Regulations)were issued which basically allowed residents, firms, associations andcorporations to maintain foreign exchange accounts abroad but the circularshave a saving clause excepting from the circular pending criminal actionsinvolving violations of CB Circ 960. Marcos filed a Motion to Quash based onthe new circular. The RTC denied the Motion so did the CA hence the appeal.Marcos averred that her right to equal protection has been violated, amongothers, as the new circular was purposedly designed to preserve the criminalcases lodged against her.

ISSUE: Whether or notImeldas right to equal protection had been violated by CB Circ 1353.

HELD: The SC ruled against Imelda. The SC said Herlamentations that the aforementioned provisions are discriminatory because theyare aimed at her and her co-accused do not assume the dignity of a legalargument since they are unwarranted conjectures belied by even the text of thecirculars alone. Hence, as respondent appellate court correctly concludes, theforegoing facts clearly disprove petitioner's claim that her constitutionalright to equal protection of the law was violated. Should she nonethelessdesire to pursue such objection, she may always adduce additional evidence atthe trial of these cases since that is the proper stage therefor, and not attheir present posture.

G.R. No. 113811: Ishmael Himagan vs People of the Philippines & Judge Hilario Mapayo

EqualProtection Suspension of PNP Members Charged with Grave Felonies

Himaganis a policeman assigned in Camp Catititgan, Davao City. He was charged for themurder of Benjamin Machitar Jr and for the attempted murder of Benjaminsyounger brother, Barnabe. Pursuant to Sec 47 of RA 6975, Himagan was placedinto suspension pending the murder case. The law provides that Upon the filingof a complaint or information sufficient in form and substance against a memberof the PNP for grave felonies where the penalty imposed by law is six (6) yearsand one (1) day or more, the court shall immediately suspend the accused fromoffice until the case is terminated. Such case shall be subject to continuoustrial and shall be terminated within ninety (90) days from arraignment of theaccused. Himagan assailed the suspension averring that Sec 42 of PD 807 of the Civil Service Decree,that his suspension should be limited to ninety (90) days. He claims that animposition of preventive suspension of over 90 days is contrary to the CivilService Law and would be a violation of his constitutional right to equalprotection of laws.

ISSUE: Whether or not Sec 47, RA 6975 violates equal protectionguaranteed by the Constitution.

HELD: The language of the first sentence of Sec 47 of RA 6975 isclear, plain and free from ambiguity. It gives no other meaning than that thesuspension from office of the member of the PNP charged with grave offensewhere the penalty is six years and one day or more shall last until thetermination of the case. The suspension cannot be lifted before the terminationof the case. The second sentence of the same Section providing that the trialmust be terminated within ninety (90) days from arraignment does not qualify orlimit the first sentence. The two can stand independently of each other. Thefirst refers to the period of suspension. The second deals with the time fromwithin which the trial should be finished.

The reason why members of thePNP are treated differently from the other classes of persons chargedcriminally or administratively insofar as the application of the rule onpreventive suspension is concerned is that policemen carry weapons and thebadge of the law which can be used to harass or intimidate witnesses againstthem, as succinctly brought out in the legislative discussions.

If a suspended policeman criminally charged with a seriousoffense is reinstated to his post while his case is pending, his victim and thewitnesses against him are obviously exposed to constant threat and thus easilycowed to silence by the mere fact that the accused is in uniform and armed. theimposition of preventive suspension for over 90 days under Sec 47 of RA 6975does not violate the suspended policeman's constitutional right to equalprotection of the laws.

Suppose the trial is not terminated within ninety days fromarraignment, should the suspension of accused be lifted?

The answer is certainly no. While the law uses the mandatory word "shall"before the phrase "be terminated within ninety (90) days", there isnothing in RA 6975 that suggests that the preventive suspension of the accusedwill be lifted if the trial is not terminated within that period. Nonetheless,the Judge who fails to decide the case within the period without justifiablereason may be subject to administrative sanctions and, in appropriate caseswhere the facts so warrant, to criminal or civil liability. If the trialis unreasonably delayed without fault of the accused such that he is deprivedof his right to a speedy trial, he is not without a remedy. He may ask for thedismissal of the case. Should the court refuse to dismiss the case, the accusedcan compel its dismissal by certiorari, prohibition or mandamus, or secure hisliberty by habeas corpus.
G.R. No. 105371: Philippine Judges Association et al vs DOTC Secretary Pete Prado et al

EqualProtection Franking Privilege of the Judiciary

Areport came in showing that available data from the Postal Service Office showthat from January 1988 to June 1992, the total volume of frank mails amountedto P90,424,175.00. of this amount, frank mails from the Judiciary and otheragencies whose functions include the service of judicial processes, such as theintervenor, the Department of Justice and the Office of the Ombudsman, amountedto P86,481,759. Frank mails coming from the Judiciary amounted to P73,574,864.00,and those coming from the petitioners reached the total amount ofP60,991,431.00. The postmaster's conclusion is that because of thisconsiderable volume of

mail from the Judiciary, the franking privilege must bewithdrawn from it. Acting from this, Prado implemented Circ. No. 9228 as theIRR for the said law. PJA assailed the said law complaining that the law wouldadversely impair the communication within the judiciary as it may impair thesending of judicial notices. PJA averred that the law is discriminatory as itdisallowed the franking privilege of the Judiciary but has not disallowed thefranking privilege of others such as the executive, former executives and theirwidows among others.

ISSUE: Whether or not there has been a violation of equal protectionbefore the law.

HELD: The SC ruled that there is a violation of the equalprotection clause. The judiciary needs the franking privilege so badly as it isvital to its operation. Evident to that need is the high expense allotted tothe judiciarys franking needs. The Postmaster cannot be sustained incontending that the removal of the franking privilege from the judiciary is inorder to cut expenditure. This is untenable for if the Postmaster would intendto cut expenditure by removing the franking privilege of the judiciary, thenthey should have removed the franking privilege all at once from all the otherdepartments. If the problem of the respondents is the loss of revenues from thefranking privilege, the remedy is to withdraw it altogether from all agenciesof the government, including those who do not need it. The problem is notsolved by retaining it for some and withdrawing it from others, especiallywhere there is no substantial distinction between those favored, which may ormay not need it at all, and the Judiciary, which definitely needs it. Theproblem is not solved by violating the Constitution.

The equal protection clause doesnot require the universal application of the laws on all persons or thingswithout distinction. This might in fact sometimes result in unequal protection,as where, for example, a law prohibiting mature books to all persons,regardless of age, would benefit the morals of the youth but violate theliberty of adults. What the clause requires is equality among equals asdetermined according to a valid classification. By classification is meant thegrouping of persons or things similar to each other in certain particulars anddifferent from all others in these same particulars.

In lumping the Judiciary with the other offices from whichthe franking privilege has been withdrawn, Sec 35 has placed the courts ofjustice in a category to which it does not belong. If it recognizes the need ofthe President of the Philippines and the members of Congress for the frankingprivilege, there is no reason why it should not recognize a similar and in factgreater need on the part of the Judiciary for such privilege.

Mario Gumabon et al vs Director of the Bureau of Prisons


Equal Protection Hernandez Doctrine Gumabon et al were charged for rebellion punished under Art 134 of the RPC. Their offense was complexed with multiple murder, robbery, arson, and kidnapping. They were all sentenced to reclusion perpetua. Their sentence had become final and executory when the Hernandez Doctrine was promulgated by the SC. Hernandez Doctrine simply states that murder cannot be complexed to rebellion as it is necessarily absorbed therein. Hence, the without such complexion the penalty must be lower than reclusion perpetua. Gumabon precisely assert a deprivation of a constitutional right, namely, the denial of equal protection. The petitioners were convicted by CFI for the very same rebellion for which Hernandez and others were convicted. The law under which they were convicted is the very same law under which the latter were convicted. It had not and has not been changed. For the same crime, committed under the same law, how can the SC, in conscience, allow petitioners to suffer life imprisonment, while others can suffer only prision mayor? ISSUE: Whether or not Gumabon et al is entitled to the effects of the Hernandez Doctrine. HELD: The SC ruled in favor of Gumabon et al. The continued incarceration after the twelveyear period when such is the maximum length of imprisonment in accordance with the controlling doctrine, when others similarly convicted have been freed, is fraught with implications at war with equal protection. That is not to give it life. On the contrary, it would render it nugatory. Otherwise, what would happen is that for an identical offense, the only distinction lying in the finality of the conviction of one being before the Hernandez ruling and the other after, a person duly sentenced for the same crime would be made to suffer different penalties. If Gumabon et al would continue to endure imprisonment, then this would be repugnant to equal protection, people similarly situated were not similarly dealt with. What is required under this constitutional guarantee is the uniform operation of legal norms so that all persons under similar circumstances would be accorded the same treatment both in the privileges conferred and the liabilities imposed. As was noted in a recent decision: Favoritism and undue preference cannot be allowed. For the principle is that equal protection and security shall be given to every person under circumstances, which if not identical are analogous. If law be looked upon in terms of burden or charges, those that fall within a class should be treated in the same fashion, whatever restrictions cast on some in the group equally binding on the rest.

G.R. No. 147780: Panfilo Lacson vs Sandiganbayan, Executive Secretary, et al

EqualProtection KBG Cases Before the Sandiganbayan

On18 May 1995, alleged members of the Kuratong Baleleng Gang were shot to death.The incident was later sensationalized as a rub out. This implicated Lacsonamong others as guilty for multiple murder. The case was raised before theSandiganbayan. In 1996, Lacson et al filed separate motions questioning the jurisdiction of theSandiganbayan, asserting that under the amended informations, the cases fallwithin the jurisdiction of the RTC pursuant to Sec 2 (par a and c) of RA 7975"An Act To Strengthen The Functional And Structural Organization Of TheSandiganbayan, Amending For That Purpose Presidential Decree 1606, As Amended.They contend that the said law limited the jurisdiction of the Sandiganbayan tocases where one or more of the "principal accused" are governmentofficials with Salary Grade (SG) 27 or higher, or PNP officials with the rankof Chief Superintendent (Brigadier General) or higher. The highest rankingprincipal accused in the amended informations has the rank of only a ChiefInspector, and none has the equivalent of at least SG 27. In 1997, RA 8249 waspassed which basically expanded the jurisdiction of the Sandiganbayan. The lawwas authored by Lagman and Neptali Gonzales. Lacson assailed the law as it was introduced by the authors thereof in badfaith as it was made to precisely suit the situation in which petitionerscases were in at the Sandiganbayan by restoring jurisdiction thereover to it,thereby violating his right to procedural due process and the equal protectionclause of the Constitution. Further, from the way the Sandiganbayan hasfootdragged for nine (9) months the resolution of a pending incident involvingthe transfer of the cases to the Regional Trial Court, the passage of the lawmay have been timed to overtake such resolution to render the issue thereinmoot, and frustrate the exercise of petitioners vested rights under the oldSandiganbayan law (RA 7975).

ISSUE: Whether or not the right to equal protection by Lacson etal has been violated with the passage of RA 8249.

HELD: The SC ruled that RA 8249 did not violate the right ofLacson et al to equal protection. No concrete evidence and convincing argumentwere presented to warrant a declaration of an act of the entire Congress andsigned into law by the highest officer of the co-equal executive department asunconstitutional. Every classification made by law is presumed reasonable.Thus, the party who challenges the law must present proof of arbitrariness. Itis an established precept in constitutional law that the guaranty of the equalprotection of the laws is not violated by a legislation based on reasonableclassification. The classification is reasonable and not arbitrary when thereis concurrence of four elements, namely:

(1) it must rest on substantial distinction;

(2) it must be germane to the purpose of thelaw;

(3) must not be limited to existing conditionsonly, and

(4) must apply equally to all members of thesame class

The classification between those pending cases involvingthe concerned public officials whose trial has not yet commenced and whosecases could have been affected by the amendments of the Sandiganbayanjurisdiction under R.A. 8249, as against those cases where trial had alreadystarted as of the approval of the law, rests on substantial distinction thatmakes real differences. In the first instance, evidence against them were notyet presented, whereas in the latter the parties had already submitted theirrespective proofs, examined witness and presented documents. Since it is withinthe power of Congress to define the jurisdiction of courts subject to theconstitutional limitations, it can be reasonably anticipated that an alterationof that jurisdiction would necessarily affect pending cases, which is why ithas to provide for a remedy in the form of a transitory provision. Thus, Lacsonet al cannot claim that Secs 4 and 7 placedthem under a different category from those similarly situated as them.

Precisely, par A of Sec 4 provides that it shall apply to"all cases involving" certain public officials and, under thetransitory provision in Sec 7, to "all cases pending in any court."Contrary to petitioner and intervenors arguments, the law is not particularlydirected only to the Kuratong Baleleng cases. The transitory provision does notonly cover cases which are in the Sandiganbayan but also in "any court."It just happened that the Kuratong Baleleng cases are one of those affected bythe law. Moreover, those cases where trial had already begun are not affectedby the transitory provision under Sec 7 of the new law (R.A. 8249).
G.R. No. 91649: Humberto Basco et al vs Philippine Amusements & Gaming Corporation

EqualProtection - Gambling

PAGCORwas created by virtue of PD 1067-A dated Jan1, 1977 and was granted a franchiseunder PD 1067-B also dated Jan 1, 1977 "to establish, operate and maintaingambling casinos on land or water within the territorial jurisdiction of thePhilippines." Its operation was originally conducted in the well knownfloating casino "Philippine Tourist." The operation was considered asuccess for it proved to be a potential source of revenue to fundinfrastructure and socioeconomic projects, thus, PD 1399 was passed on June 2,1978 for PAGCOR to fully attain this objective. Subsequently, on July 11, 1983,PAGCOR was created under PD 1869 to enable

the Government to regulate andcentralize all games of chance authorized by existing franchise or permitted bylaw, under the following declared policy: Section1. Declaration of Policy. It is hereby declared to be the policy ofthe State to centralize and integrate all games of chance not heretoforeauthorized by existing franchises or permitted by law.

Basco and other lawyers assailedthe validity of PAGCOR averring among others that it violates the equal protection clause ofthe constitution in that it legalizes PAGCOR conducted gambling, while most other forms of gambling are outlawed,together with prostitution, drug trafficking and other vices.

ISSUE: Whether or not the creation of PAGCOR violates the equalprotection clause.

HELD: The SC found Bascos petition to be devoid of merit. Justhow PD 1869 in legalizing gambling conducted by PAGCOR is violative of theequal protection is not clearly explained in their petition. The mere fact thatsome gambling activities like cockfighting (PD 449) horse racing (RA 306 asamended by RA 983), sweepstakes, lotteries and races (RA 1169 as amended by BP42) are legalized under certain conditions, while others are prohibited, doesnot render the applicable laws, PD. 1869 for one, unconstitutional.

Bascos posture ignores thewell-accepted meaning of the clause "equal protection of the laws."The clause does not preclude classification of individuals who may be accordeddifferent treatment under the law as long as the classification is notunreasonable or arbitrary. A law does not have to operate in equal force on allpersons or things to be conformable to Article III, Sec 1 of the Constitution.The "equal protection clause" does not prohibit the Legislature fromestablishing classes of individuals or objects upon which different rules shalloperate. The Constitution does not require situations which are different infact or opinion to be treated in law as though they were the same.
G.R. No. 124360: Francisco Tatad vs Secretary of Energy

One Title, One Subject Rule Oid Deregulation Law

Considering that oil is not endemic to this country, history shows that the government has always been finding ways to alleviate the oil industry. The government created laws accommodate these innovations in the oil industry. One such law is the Downstream Oil Deregulation Act of 1996 or RA 8180. This law allows that "any person or entity may import or purchase any quantity of crude oil and petroleum products from a foreign or domestic source, lease or own and operate refineries and other downstream oil facilities and market such crude oil or use the same for his own requirement," subject only to monitoring by the Department of Energy. Tatad assails the constitutionality of the law. He claims that section 5 (b) of R.A. No. 8180 violates the one title one rule of Sec 26, Art 6 of the Constitution. Section 5 (b) provides:

"b) Any law to the contrary notwithstanding and starting with the effectivity of this Act, tariff duty shall be imposed and collected on imported crude oil at the rate of three percent (3%) and imported refined petroleum products at the rate of seven percent (7%), except fuel oil and LPG, the rate for which shall be the same as that for imported crude oil: Provided, That beginning on January 1, 2004 the tariff rate on imported crude oil and refined petroleum products shall be the same: Provided, further, That this provision may be amended only by an Act of Congress."

The inclusion of the tariff provision in section 5(b) of R.A. No. 8180 violates Section 26(1) Article VI of the Constitution requiring every law to have only one subject which shall be expressed in its title. Petitioner contends that the imposition of tariff rates in section 5(b) of R.A. No. 8180 is foreign to the subject of the law which is the deregulation of the downstream oil industry.

ISSUE: Whether or not RA 8180 is constitutional.

HELD: The SC declared the unconstitutionality of RA 8180 not because it violated the one title one subject rule but rather because it violated Sec 19 of Art 12 of the Constitution. It violated that provision because it only strengthens oligopoly which is contrary to free competition. The SC emphasized that the provision of Sec 5 (b) of RA 8180 does not violate the one title one subject rule. The SC, as a policy, has adopted a liberal construction of the one title - one subject rule. The SC also emphasized that the title need not mirror, fully index or catalogue all contents and minute details of a law. A law having a single general subject indicated in the title may contain any number of provisions, no matter how diverse they may be, so long as they are not inconsistent with or foreign to the general subject, and may be considered in furtherance of such subject by providing for the method and means of carrying out the general subject. The SC held that section 5(b) providing for tariff differential is germane to the subject of RA 8180 which is the deregulation of the downstream oil industry. The section is supposed to sway prospective investors to put up refineries in our country and make them rely less on imported petroleum.

Francisco Tatad et al vs Secretary of Energy


Equal Protection Oil Deregulation Law Considering that oil is not endemic to this country, history shows that the government has always been finding ways to alleviate the oil industry. The government created laws accommodate these innovations in the oil industry. One such law is the Downstream Oil Deregulation Act of 1996 or RA 8180. This law allows that any person or entity may import or purchase any quantity of crude oil and petroleum products from a foreign or domestic source, lease or own and operate refineries and other downstream oil facilities and market such crude oil or use the same for his own requirement, subject only to monitoring by the Department of Energy. Tatad assails the constitutionality of the law. He claims, among others, that the imposition of different tariff rates on imported crude oil and imported refined petroleum products violates the equal protection clause. Tatad contends that the 3%-7% tariff differential unduly favors the three existing oil refineries and discriminates against prospective investors in the downstream oil industry who do not have their own refineries and will have to source refined petroleum products from abroad.3% is to be taxed on unrefined crude products and 7% on refined crude products. ISSUE: Whether or not RA 8180 is constitutional. HELD: The SC declared the unconstitutionality of RA 8180 because it violated Sec 19 of Art 12 of the Constitution. It violated that provision because it only strengthens oligopoly which is contrary to free competition. It cannot be denied that our downstream oil industry is operated and controlled by an oligopoly, a foreign oligopoly at that. Petron, Shell and Caltex stand as the only major league players in the oil market. All other players belong to the lilliputian league. As the dominant players, Petron, Shell and Caltex boast of existing refineries of various capacities. The tariff differential of 4% therefore works to their immense benefit. Yet, this is only one edge of the tariff differential. The other edge cuts and cuts deep in the heart of their competitors. It erects a high barrier to the entry of new players. New players that intend to equalize the market power of Petron, Shell and Caltex by building refineries of their own will have to spend billions of pesos. Those who will not build refineries but compete with them will suffer the huge disadvantage of increasing their product cost by 4%. They will be competing on an uneven field. The argument that the 4% tariff differential is desirable because it will induce prospective players to invest in refineries puts the cart before the horse. The first need is to attract new players and they cannot be attracted by burdening them with heavy disincentives. Without new players belonging to the league of Petron, Shell and Caltex, competition in our downstream oil industry is an idle dream. RA 8180 is unconstitutional on the ground inter alia that it discriminated against the new players insofar as it placed them at a competitive disadvantage vis--vis the established oil companies by requiring them to meet certain conditions already being observed by the latter.

Taxicab Operators vs Board of Transportation


Police Power

Petitioner Taxicab Operators of Metro Manila, Inc. (TOMMI) is a domestic corporation composed of taxicab operators, who are grantees of Certificates of Public Convenience to operate taxicabs within the City of Manila and to any other place in Luzon accessible to vehicular traffic.

On October 10, 1977, respondent Board of Transportation (BOT) issued Memorandum Circular No. 77-42 which reads:

SUBJECT: Phasing out and Replacement of Old and Dilapidated Taxis

On January 27, 1981, petitioners filed a Petition with the BOT, docketed as Case No. 80-7553, seeking to nullify MC No. 77-42 or to stop its implementation; to allow the registration and operation in 1981 and subsequent years of taxicabs of model 1974, as well as those of earlier models which were phased-out, provided that, at the time of registration, they are roadworthy and fit for operation.

ISSUE

A. Did BOT and BLT promulgate the questioned memorandum circulars in accord with the manner required by Presidential Decree No. 101, thereby safeguarding the petitioners constitutional right to procedural due process?

B. Granting arguendo, that respondents did comply with the procedural requirements imposed by Presidential Decree No. 101, would the implementation and enforcement of the assailed memorandum circulars violate the petitioners constitutional rights to.

(1) Equal protection of the law;

(2) Substantive due process; and

(3) Protection against arbitrary and unreasonable classification and standard?

HELD As enunciated in the preambular clauses of the challenged BOT Circular, the overriding consideration is the safety and comfort of the riding public from the dangers posed by old and dilapidated taxis. The State, in the exercise of its police power, can prescribe regulations to promote the health, morals, peace, good order, safety and general welfare of the people. It can prohibit all things hurtful to comfort, safety and welfare of society. [5] It may also regulate property rights. [6] In the language of Chief Justice Enrique M. Fernando the necessities imposed by public welfare may justify the exercise of governmental authority to regulate even if thereby certain groups may plausibly assert that their interests are disregarded.

Mary Concepcion Bautista et al vs Alfredo Juinio et al


Equal Protection Distinction Between Heavy and Extra Heavy Cars and Others Bautista is assailing the constitutionality of LOI 869 issued in 1979 which classified vehicles into Heavy and Extra Heavy. The LOI further banned these vehicles during weekends and holidays that is from 5am Saturday until 5am Monday. Purpose of this law is to curb down petroleum consumption as bigger cars consume more oil. Bautista claimed the LOI to be discriminatory as it made an assumption that H and EH cars are heavy on petroleum consumption when in fact there are smaller cars which are also big on oil consumption. Further, the law restricts their freedom to enjoy their car while others who have smaller cars may enjoy theirs. Bautista avers that there is no rational justification for the ban being imposed on vehicles classified as heavy (H) and extra-heavy (EH), for precisely those owned by them fall within such category. ISSUE: Whether or not the LOI violates equal protection. HELD: The SC held that Bautista was not able to make merit out of her contention. The classification on cars on its face cannot be characterized as an affront to reason. The ideal situation is for the laws benefits to be available to all, that none be placed outside the sphere of its coverage. Only thus could chance and favor be excluded and the affairs of men governed by that serene and impartial uniformity, which is of the very essence of the idea of law. The actual, given things as they are and likely to continue to be, cannot approximate the ideal. Nor is the law susceptible to the reproach that it does not take into account the realities of the situation. . . . To assure that the general welfare be promoted, which is the end of law, a regulatory measure may

cut into the rights to liberty and property. Those adversely affected may under such circumstances invoke the equal protection clause only if they can show that the governmental act assailed, far from being inspired by the attainment of the common weal was prompted by the spirit of hostility, or at the very least, discrimination that finds no support in reason. It suffices then that the laws operate equally and uniformly on all persons under similar circumstances or that all persons must be treated in the same manner, the conditions not being different, both in the privileges conferred and the liabilities imposed. Favoritism and undue preference cannot be allowed. For the principle is that equal protection and security shall be given to every person under circumstances, which if not identical are analogous. If law be looked upon in terms of burden or charges, those that fall within a class should be treated in the same fashion, whatever restrictions cast on some in the group equally binding on the rest.

Patricio Dumlao et al vs COMELEC


Equal Protection Eligibility to Office after Being 65 Dumlao was the former governor of Nueva Vizcaya. He has retired from his office and he has been receiving retirement benefits therefrom. He filed for reelection to the same office for the 1980 local elections. On the other hand, BP 52 was passed (par 1 thereof) providing disqualification for the likes of Dumlao. Dumlao assailed the BP averring that it is class legislation hence unconstitutional. His petition was joined by Atty. Igot and Salapantan Jr. These two however have different issues. The suits of Igot and Salapantan are more of a taxpayers suit assailing the other provisions of BP 52 regarding the term of office of the elected officials, the length of the campaign and the provision barring persons charged for crimes may not run for public office and that the filing of complaints against them and after preliminary investigation would already disqualify them from office. In general, Dumlao invoked equal protection in the eye of the law. ISSUE: Whether or not the there is cause of action. HELD: The SC pointed out the procedural lapses of this case for this case would never have been merged. Dumlaos cause is different from Igots. They have separate issues. Further, this case does not meet all the requisites so that itd be eligible for judicial review. There are standards that have to be followed in the exercise of the function of judicial review, namely: (1) the existence of an appropriate case; (2) an interest personal and substantial by the party raising the constitutional question; (3) the plea that the function be exercised at the earliest opportunity; and (4) the necessity that the constitutional question be passed upon in order to decide the case. In this case, only the 3rd requisite was met. The SC ruled however that the provision barring persons charged for crimes may not run for public office and that the filing of complaints against them and after preliminary investigation would already disqualify them from office as null and void. The assertion that Sec 4 of BP 52 is contrary to the safeguard of equal protection is neither well taken. The constitutional guarantee of equal protection of the laws is subject to rational classification. If the groupings are based on reasonable and real differentiations, one class can be treated and regulated differently from another class. For purposes of public service, employees

65 years of age, have been validly classified differently from younger employees. Employees attaining that age are subject to compulsory retirement, while those of younger ages are not so compulsorily retirable. In respect of election to provincial, city, or municipal positions, to require that candidates should not be more than 65 years of age at the time they assume office, if applicable to everyone, might or might not be a reasonable classification although, as the Solicitor General has intimated, a good policy of the law should be to promote the emergence of younger blood in our political elective echelons. On the other hand, it might be that persons more than 65 years old may also be good elective local officials. Retirement from government service may or may not be a reasonable disqualification for elective local officials. For one thing, there can also be retirees from government service at ages, say below 65. It may neither be reasonable to disqualify retirees, aged 65, for a 65-year old retiree could be a good local official just like one, aged 65, who is not a retiree. But, in the case of a 65-year old elective local official (Dumalo), who has retired from a provincial, city or municipal office, there is reason to disqualify him from running for the same office from which he had retired, as provided for in the challenged provision.

Villegas vs. Hui Chiong Tsai Pao Ho

FACTS: This case involves an ordinance prohibiting aliens from being employed or engage or participate in any position or occupation or business enumerated therein, whether permanent, temporary or casual, without first securing an employment permit from the Mayor of Manila and paying the permit fee of P50.00. Private respondent Hiu Chiong Tsai Pao Ho who was employed in Manila, filed a petition to stop the enforcement of such ordinance as well as to declare the same null and void. Trial court rendered judgment in favor of the petitioner, hence this case. ISSUE: WON said Ordinance violates due process of law and equal protection rule of the Constitution. HELD: Yes. The Ordinance The ordinance in question violates the due process of law and equal protection rule of the Constitution. Requiring a person before he can be employed to get a permit from the City Mayor who may withhold or refuse it at his will is tantamount to denying him the basic right of the people in the Philippines to engage in a means of livelihood. While it is true that the Philippines as a State is not obliged to admit aliens within its territory, once an alien is admitted, he cannot be deprived of life without due process of law. This guarantee includes the means of livelihood. The shelter of protection under the due process and equal protection clause is given to all persons, both aliens and citizens.
G.R. No. L-52304: Ramon Ceniza et al vs Commission on Elections, COA & National Treasurer

Equal Protection - Gerrymandering

**"Gerrymandering" is a "term employed to describe an apportionment of representative districts so contrived as to give an unfair advantage to the party inpower." **

Pursuant to Batas Blg 51 (enacted 22 Dec 1979), COMELEC adopted Resolution No. 1421 which effectively bars voters in chartered cities (unless otherwise provided by their charter), highly urbanized (those earning above P40M) cities, and component cities (whose charters prohibit them) from voting in provincial elections. The City of Mandaue, on the other hand, is a component city NOT a chartered one or a highly urbanized one. So when COMELEC added Mandaue to the list of 20 cities that cannot vote in provincial elections,Ceniza, in behalf of the other members of DOERS (Democracy or Extinction: Resolved to Succeed) questioned the constitutionality of BB 51 and the COMELEC resolution. They said that the regulation/restriction of voting being imposedis a curtailment of the right to suffrage. Further, petitioners claim that political and gerrymandering motives were behind the passage of Batas Blg. 51and Section 96 of the Charter of Mandaue City. They contend that the Provinceof Cebu is politically and historically known as an opposition bailiwick and ofthe total 952,716 registered voters in the province, close to one-third (1/3)of the entire province of Cebu would be barred from voting for the provincialofficials of the province of Cebu. Ceniza also said that the constituents of Mandaue never ratified their charter. Ceniza likewise aver that Sec 3 of BB 885 insofar as it classifies cities including Cebu City as highly urbanized as the only basis for not allowing its electorate to vote for the provincial officials is inherently and palpably unconstitutional in that such classification is not based on substantial distinctions germane to the purpose of the law which in effect provides for and regulates the exercise of the right of suffrage, and therefore such unreasonable classification amounts to a denial of equal protection.

ISSUE: Whether or notthere is a violation of equal protection.

HELD: The thrust of the1973 Constitution is towards the fullest autonomy of local government units. Inthe Declaration of Principles and State Policies, it is stated that "TheState shall guarantee and promote the autonomy of local government units to ensure their fullest development as selfreliant communities. The petitioner's allegation of gerrymandering is of no merit, it has no factual or legal basis. The Constitutional requirement that the creation, division, merger, abolition, oralteration of the boundary of a province, city, municipality, or barrio shouldbe subject to the approval by the majority of the votes cast in a plebiscite in the governmental unit or units affected is a new requirement that came intobeing only with the 1973 Constitution. It is prospective in character and therefore cannot affect the creation of the City of Mandaue which came into existence on 21 June 1969.

The classification of cities into highly urbanized cities and component cities on the basis of their regular annual income is based upon substantial distinction.The revenue of a city would show whether or not it is capable of existence and development as a relatively independent social, economic, and political unit. It would also show whether the city has sufficient economic or industrial activity as to warrant its independence from the province where it is geographically situated. Cities with smaller income need the continued supportof the provincial government thus justifying the continued participation of thevoters in the election of provincial officials in some instances.

The petitioners also contend that the voters in Mandaue City aredenied equal protection of the law since the voters in other component citiesare allowed to vote for provincial officials. The contention is without merit. The practice of allowing voters in one component city to vote for provincial officials and denying the same privilege to voters in another component city isa matter of legislative discretion which violates neither the Constitution northe voter's right of suffrage.
G.R. No. 56515: United Democratic Opposition vs Commission on Elections

EqualProtection Access to Media

In 1981, the BP proposed amendments to the 1973 Constitution. The amendments were to be placed to a plebiscite for the peoples approval. The YES vote was being advanced by KBL Marcos Party. While the NO vote was being advanced by UNIDO. To ensure parity and equality, COMELEC issued Resolutions 1467-1469 w/c basically provided that there be equal opportunity, equal time and equal space on media use for campaigns for both sides. On 12 Mar 1981, Marcos campaigned for the YES vote via TV and radio from 9:30pm to 11:30pm. Thesame was broadcasted live by 26 TV stations and 248 radio stations nationwide.UNIDO petitioned before the COMELEC that they be granted the same opportunity as Marcos had pursuant to Resns 1467-69. COMELEC denied the demand. UNIDO assailed the denial as a denial of equal protection before the laws.

ISSUE: Whether or notUNIDO was denied equal protection by virtue of COMELECs denial of their request.

HELD: The SC ruled that UNIDO was not denied due process nor were they not afforded equal protection. It is the considered view of the SC that when Marcos conducted his 'pulong-pulong' or consultation with the people on March 12, 1981, he did so in his capacity asPresident/Prime Minister of the Philippines and not as the head of any political party. Under the Constitution, the 'Prime Minister and the Cabinet shall be responsible . . . for the program of government and shall

determinethe guidelines of national policy'. In instances where the head of state is atthe same time the president of the political party that is in power, it does not necessarily follow that he speaks with two voices when he dialogues with the governed. The president is accorded certain privileges that the oppositionmay not have. Further, the SC cannot compel TV stations and radio stations to give UNIDO free air time as they are not party to this case. UNIDO must sought contract with these TV stations and radio stations at their own expense.

Rufino Nuez vs Sandiganbayan & the People of the Philippines


Equal Protection Creation of the Sandiganbayan Nuez assails the validity of the PD 1486 creating the Sandiganbayan as amended by PD 1606. He was accused before the Sandoganbayan of estafa through falsification of public and commercial documents committed in connivance with his other co-accused, all public officials, in several cases. It is the claim of Nuez that PD1486, as amended, is violative of the due process, equal protection, and ex post facto clauses of the Constitution. He claims that the Sandiganbayan proceedings violates Nuezs right to equal protection, because appeal as a matter of right became minimized into a mere matter of discretion; appeal likewise was shrunk and limited only to questions of law, excluding a review of the facts and trial evidence; and there is only one chance to appeal conviction, by certiorari to the SC, instead of the traditional two chances; while all other estafa indictees are entitled to appeal as a matter of right covering both law and facts and to two appellate courts, i.e., first to the CA and thereafter to the SC. ISSUE: Whether or not the creation of Sandiganbayan violates equal protection insofar as appeals would be concerned. HELD: The SC ruled against Nuez. The 1973 Constitution had provided for the creation of a special court that shall have original jurisdiction over cases involving public officials charged with graft and corruption. The constitution specifically makes mention of the creation of a special court, the Sandiganbayan, precisely in response to a problem, the urgency of which cannot be denied, namely, dishonesty in the public service. It follows that those who may thereafter be tried by such court ought to have been aware as far back as January 17, 1973, when the present Constitution came into force, that a different procedure for the accused therein, whether a private citizen as petitioner is or a public official, is not necessarily offensive to the equal protection clause of the Constitution. Further, the classification therein set forth met the standard requiring that it must be based on substantial distinctions which make real differences; it must be germane to the purposes of the law; it must not be limited to existing conditions only, and must apply equally to each member of the class. Further still, decisions in the Sandiganbayan are reached by a unanimous decision from 3 justices - a showing that decisions therein are more conceivably carefully reached than other trial courts. Justice Makasiar (concurring & dissenting)

Persons who are charged with estafa or malversation of funds not belonging to the government or any of its instrumentalities or agencies are guaranteed the right to appeal to two appellate courts first, to the CA, and thereafter to the SC. Estafa and malversation of private funds are on the same category as graft and corruption committed by public officers, who, under the decree creating the Sandiganbayan, are only allowed one appeal to the SC (par. 3, Sec. 7, P.D. No. 1606). The fact that the Sandiganbayan is a collegiate trial court does not generate any substantial distinction to validate this invidious discrimination. Three judges sitting on the same case does not ensure a quality of justice better than that meted out by a trial court presided by one judge. The ultimate decisive factors are the intellectual competence, industry and integrity of the trial judge. But a review by two appellate tribunals of the same case certainly ensures better justice to the accused and to the people. Then again, par 3 of Sec 7 of PD 1606, by providing that the decisions of the Sandiganbayan can only be reviewed by the SC through certiorari, likewise limits the reviewing power of the SC only to question of jurisdiction or grave abuse of discretion, and not questions of fact nor findings or conclusions of the trial court. In other criminal cases involving offenses not as serious as graft and corruption, all questions of fact and of law are reviewed, first by the CA, and then by the SC. To repeat, there is greater guarantee of justice in criminal cases when the trial courts judgment is subject to review by two appellate tribunals, which can appraise the evidence and the law with greater objectivity, detachment and impartiality unaffected as they are by views and prejudices that may be engendered during the trial. Limiting the power of review by the SC of convictions by the Sandiganbayan only to issues of jurisdiction or grave abuse of discretion, likewise violates the constitutional presumption of innocence of the accused, which presumption can only be overcome by proof beyond reasonable doubt (Sec. 19, Art. IV, 1973 Constitution).

Antero Sison Jr. vs Acting BIR Commissioner Ruben Ancheta et al


Equal Protection Sison assails the validity of BP 135 w/c further amended Sec 21 of the National Internal Revenue Code of 1977. The law provides that thered be a higher tax impost against income derived from professional income as opposed to regular income earners. Sison, as a professional businessman, and as taxpayer alleges that by virtue thereof, he would be unduly discriminated against by the imposition of higher rates of tax upon his income arising from the exercise of his profession visa-vis those which are imposed upon fixed income or salaried individual taxpayers. He characterizes the above section as arbitrary amounting to class legislation, oppressive and capricious in character. There is a transgression of both the equal protection and due process clauses of the Constitution as well as of the rule requiring uniformity in taxation. ISSUE: Whether the imposition of a higher tax rate on taxable net income derived from business or profession than on compensation is constitutionally infirm.

HELD: The SC ruled against Sison. The power to tax, an inherent prerogative, has to be availed of to assure the performance of vital state functions. It is the source of the bulk of public funds. Taxes, being the lifeblood of the government, their prompt and certain availability is of the essence. According to the Constitution: The rule of taxation shall be uniform and equitable. However, the rule of uniformity does not call for perfect uniformity or perfect equality, because this is hardly attainable. Equality and uniformity in taxation means that all taxable articles or kinds of property of the same class shall be taxed at the same rate. The taxing power has the authority to make reasonable and natural classifications for purposes of taxation. Where the differentiation complained of conforms to the practical dictates of justice and equity it is not discriminatory within the meaning of this clause and is therefore uniform. There is quite a similarity then to the standard of equal protection for all that is required is that the tax applies equally to all persons, firms and corporations placed in similar situation. What misled Sison is his failure to take into consideration the distinction between a tax rate and a tax base. There is no legal objection to a broader tax base or taxable income by eliminating all deductible items and at the same time reducing the applicable tax rate. Taxpayers may be classified into different categories. In the case of the gross income taxation embodied in BP 135, the discernible basis of classification is the susceptibility of the income to the application of generalized rules removing all deductible items for all taxpayers within the class and fixing a set of reduced tax rates to be applied to all of them. Taxpayers who are recipients of compensation income are set apart as a class. As there is practically no overhead expense, these taxpayers are not entitled to make deductions for income tax purposes because they are in the same situation more or less. On the other hand, in the case of professionals in the practice of their calling and businessmen, there is no uniformity in the costs or expenses necessary to produce their income. It would not be just then to disregard the disparities by giving all of them zero deduction and indiscriminately impose on all alike the same tax rates on the basis of gross income. There is ample justification then for the Batasang Pambansa to adopt the gross system of income taxation to compensation income, while continuing the system of net income taxation as regards professional and business income.

Citizens Surety & Insurance Co., Inc. vs Judge Ricardo Puno, Register of Deeds Manila
Equal Protection Purchase of Land Barrio Obrero In 1956, Resolution 542 was passed by the Register of Deeds Manila which provided that only Filipino laborers whose wages do not exceed P180.00/month or P6.00/day and at the same time residents of Manila may be allowed to purchase lands in Barrio Obrero, Tondo, Manila. On 10 Oct 1966, Maria Barcelon mortgaged her 180 sq. m. land located in Barrio Obrero to CSICI. CSICI foreclosed the property due to nonpayment and later bought the land. CSICI later sought to register and consolidate the land before the Register of Deeds but then Justice Puno denied the request pursuant to Resn 542 as CSICI does not meet the qualification. CSICI averred that Resn 542 is null and void. It averred: As may be seen from Sec 4 of Resn 542, only laborers earning not more than P180.00 a month, or P5.00 a day are qualified to buy Lands in Barrio Obrero. Employees working in offices or establishments and earning as much but who are not

laborers cannot buy lands in that area. Also persons who are engaged in some calling or occupation earning as much are not also qualified. It should not be overlooked that the intention of the pertinent provisions of the Charter of the City of Manila contained in Sections 97, 98 and 100 of said Charter is to help the poor people of Manila to acquire residential lands on easy terms. CSICI points out that there is no substantial difference between these laborers to those mentioned in the Resolution. ISSUE: Whether or not Resolution 542 violates equal protection. HELD: The SC ruled against CSICI. CSICI, which is a corporation and not a lowly paid worker, is not competent to raise this claim. For even if the SC sustain it, no benefit can accrue to CSICI who will nonetheless be disqualified to acquire the lot. Moreover, in the absence of manifest abuse of power, the SC not vent to substitute their judgment for that of the City of Manila which is tasked by its Charter to acquire private lands in the city and to subdivide the same into home lots for sale on easy terms to residents, giving first priority to the bona-fide tenants or occupants of said lands, and second priority to laborers and low-salaried employees. Obviously, the questioned resolution merely seeks to implement the Charter provision.

Peralta et al vs Commission on Elections et al


Equal Protection 1978 Election Code Block Voting Peralta was an independent candidate in the April 1978 Interim Batasang Pambansa Elections. He, along with others, assailed the constitutionality of PD 1269 or the 1978 Election Code. Secs140 and 155, sub-paragraphs 26 to 28, of the 1978 Election Code, grants the voter the option to vote either for individual candidates by filling in the proper spaces in the ballot the names of candidates he desires to elect, or to vote for all the candidates of a political party, group or aggrupation by simply writing in the space provided for in the ballot the name of the political party, group or aggrupation (office-block ballot). Peralta was vehement in contending that the optional block voting scheme is violative of this provision of the Constitution: Bona fide candidates for any public office shall be free from any form of harassment and discrimination. He sought the shelter of its protection for himself and other independent candidates who, according to him, would be thus made to suffer if the assailed provision is not nullified. Essentially, in terms of individual rights, he would raise a due process and equal protection question. The main objection of Peralta against the optional straight party voting provided for in the Code is that an independent candidate would be discriminated against because by merely writing on his ballot the name of a political party, a voter would have voted for all the candidates of that party, an advantage which the independent candidate does not enjoy. In effect, it is contended that the candidate who is not a party-member is deprived of the equal protection of the laws, as provided in Sec 1 of Article IV, in relation to Sec 9 of Article XII, of the 1973 Constitution. ISSUE: Whether or not the 1978 Election Code is violative of equal protection.

HELD: The SC ruled that the 1978 Election Code is valid. Before a voter prepares his ballot, the voter will be able to read all the names of the candidates. No candidate will receive more than one vote, whether he is voted individually or as a candidate of a party group or aggrupation. The voter is free to vote for the individual candidates or to vote by party, group or aggrupation. The choice is his. No one can compel him to do otherwise. In the case of candidates, the decision on whether to run as an independent candidate or to join a political party, group or aggrupation is left entirely to their discretion. Certainly, before filing his certificate of candidacy, a candidate is aware of the advantages under the law accruing to candidates of a political party or group. If he wishes to avail himself of such alleged advantages as an official candidate of a party, he is free to do so by joining a political party group or aggrupation. In other words, the choice is his. In making his decision, it must be assumed that the candidate had carefully weighed and considered the relative advantages and disadvantages of either alternative. So long as the application of the rule depends on his voluntary action or decision, he cannot, after exercising his discretion, claim that he was the victim of discrimination.

HAWAIIAN-PHILIPPINE COMPANY vs ASOCIACION DE HACENDEROS DE SILAY-SARAVIA, INC., et al


Equal Protection Sugar Cane Industry Hawaiian is a sugar milling company while Asociacion is a corporation organized to represent sugar cane planters in Negros Occidental. In 1953, both had a contract which would include 12 crop (years) until the 1963-1964 crop. For the 1st 6 years, the sharing would be 63% to 37%, Asociacion being the highest getter. And from the next 5 years itd be 63.5% to 36.5% and in the 63-64 crop year itd be 64% to 36%. In 1961, Asociacion made known its intention to buy out Hawaiian which was reluctant at first but later agreed at a selling price of $14M. The sale was never actualized and Asociacion initially sought to restructure the contract to a 70%-30% sharing. Eventually, Asociacion severed ties with Hawaiian and Asocicion established its own milling company, the Agricultural Industrial Development Company of Silay-Saravia. Apparently, Asociacion is authorized by law to break existing contracts by virtue of RA 809. Hawaiian assailed the constitutionality of the said law: Secs 1, 4 and 9 of RA 809, Secs 4 and 5 of RA 1825 and Sec 3 of RA 1072 amending Sec 9 of Act 4166, for being violative of the constitutional guarantees against impairment of the freedom of contracts, denial of equal protection of the laws, taking of private property for public use without due process and without just compensation and impairment of vested rights and (2) validity of: aforesaid laws for being violative of treaty commitments previously entered into by the Government of the Republic of the Philippines. ISSUE: Whether or not there has been a violation of equal protection. HELD: The SC ruled that there is no such violation. RA 809 is a social justice and police power measure for the promotion of labor conditions in sugar plantations, hence whatever rational degree of constraint it exerts on freedom of contract and existing contractual obligations is constitutionally permissible. RA 1825 and RA 1072 amending Act 4166 covering as they do the same subject, i.e. sugar production, partake of the same nature as RA 809 and for the same

reasons as above stated, cannot be considered constitutionally objectionable. Sugar production is one of the great industries of our nation, sugar occupying a leading position among its export products, that it gives employment to thousands of laborers in field and factories, that it is a great source of the states wealth, is one of the important sources of foreign exchange needed by our government, and is thus pivotal in the plans of a regime committed to a policy of currency stability. Its promotion, protection and advancement therefore, redounds greatly to the general welfare. Hence it was competent for the legislature to find that the general welfare demanded that the sugar industry should be stabilized in turn, and in the wide field of its police power, the lawmaking body could provide that the distribution of benefits therefrom be readjusted among its components.

Ormoc Sugar Company Inc. vs Ormoc City et al


Equal Protection In 1964, Ormoc City passed a bill which read: There shall be paid to the City Treasurer on any and all productions of centrifugal sugar milled at the Ormoc Sugar Company Incorporated, in Ormoc City a municipal tax equivalent to one per centum (1%) per export sale to the United States of America and other foreign countries. Though referred to as a production tax, the imposition actually amounts to a tax on the export of centrifugal sugar produced at Ormoc Sugar Company, Inc. For production of sugar alone is not taxable; the only time the tax applies is when the sugar produced is exported. Ormoc Sugar paid the tax (P7,087.50) in protest averring that the same is violative of Sec 2287 of the Revised Administrative Code which provides: It shall not be in the power of the municipal council to impose a tax in any form whatever, upon goods and merchandise carried into the municipality, or out of the same, and any attempt to impose an import or export tax upon such goods in the guise of an unreasonable charge for wharfage, use of bridges or otherwise, shall be void. And that the ordinance is violative to equal protection as it singled out Ormoc Sugar As being liable for such tax impost for no other sugar mill is found in the city. ISSUE: Whether or not there has been a violation of equal protection. HELD: The SC held in favor of Ormoc Sugar. The SC noted that even if Sec 2287 of the RAC had already been repealed by a latter statute (Sec 2 RA 2264) which effectively authorized LGUs to tax goods and merchandise carried in and out of their turf, the act of Ormoc City is still violative of equal protection. The ordinance is discriminatory for it taxes only centrifugal sugar produced and exported by the Ormoc Sugar Company, Inc. and none other. At the time of the taxing ordinances enactment, Ormoc Sugar Company, Inc., it is true, was the only sugar central in the city of Ormoc. Still, the classification, to be reasonable, should be in terms applicable to future conditions as well. The taxing ordinance should not be singular and exclusive as to exclude any subsequently established sugar central, of the same class as plaintiff, from the coverage of the tax. As it is now, even if later a similar company is set up, it cannot be subject to the tax because the ordinance expressly points only to Ormoc Sugar Company, Inc. as the entity to be levied upon.
Flores vs.

C o m elec Facts:The petitioner, Roque Flores, was proclaimed by theboard of canvassers as having received the highest number of votes for kagawad in Brgy. Poblacion, Tayum, Abra, andthus became punong barangay pursuant to Sec 5 RA 6679.He was voted punong barangay during the 1 9 8 2 elections, aseparate position as that of Kagawad. The private respondent, Nobelito Rapisora,protected the result and filed a protect before the MCTCTayum. He argued that the ballot, which only indicatedFlores, should be declared stray votes and should not bedivided equally to them.In his defense, the petitioner argued that inaccordance with the Omnibus Election Code, the 4questioned votes should be entitled to him under the equity of the incumbent rule, which states that if there are 2 or morecandidates with the same full name and one of them is anincumbent and the ballot is written only on such full name,the vote is counted in favor of the incumbent.The lower court sustained the contention of theprivate respondent and subsequently declared him as thepunong barangay.Hence this petition. The petitioner argued that bynot following the rule stated, he is deprived of his right toequal protection of the law since he is also an incumbent punong barangay running for election, thereby he should beentitled by the rule.Issue:Was the petitioner considered an incumbent to beentitled under the rule?Held:No. Under the new rule Resolution 2 0 22- A passedby the Comelec, Barangay Captains who filed their candidacy for the office of Kagawad, which is another office,shall be deemed resigned in their former office. In his filing of candidacy, it stated that he is runningfor kagawad and not as a punong barangay. Thus, pursuantto the resolution, he deemed to resign his position as punongbarangay when he filed for his candidacy as a kagawad.The rule cannot thus then be applied to thepetitioner since pursuant to the resolution, he is notconsidered as an incumbent punong barangay, he is notwithin the same class as that of the incumbents.The court cannot sustain the argument of thepetitioner that since RA 669 speaks of 7 candidates for kagawad, the foremost of them is the punong barangay, heshould be regarded as running for the same office. Constitutional Law II (Bill of Rights): Case Briefs: Dennis G. Libunao UC College of Law 17 There is a substantial distinction between Punongbarangay and kagawad. The former is vested with executivepower and the latter legislative power pursuant to the LGC.Note:In 1 9 8 2 procedures, there are separate election for Punong barngay and members of the Sanguniangpambarangay.In 1 9 8 9 elections, the only disputed position for direct election is the Kagawad and PB will be granted byoperation of law to the highest vote earned.

RA 9372 SEC. 18. Period of Detention Without Judicial Warrant of Arrest. The provisions of Article 125 of the Revised Penal Code to the contrary notwithstanding, any police or law enforcement personnel, who, having been duly authorized in writing by the Anti-Terrorism Council has taken custody of a person charged with or suspected of the crime of terrorism or the crime of conspiracy to commit terrorism shall, without incurring any criminal liability for delay in the

delivery of detained persons to the proper judicial authorities, deliver said charged or suspected person to the proper judicial authority within a period of three (3) days counted from the moment the said charged or suspected person has been apprehended or arrested, detained, and taken into custody by the said police, or law enforcement personnel: Provided, That the arrest of those suspected of the crime of terrorism or conspiracy to commit terrorism must result from the surveillance under Section 7 and examination of bank deposits under Section 27 of this Act. The police or law enforcement personnel concerned shall, before detaining the person suspected of the crime of terrorism, present him or her before any judge at the latters residence or office nearest the place where the arrest took place at any time of the day or night. It shall be the duty of the judge, among other things, to ascertain the identity of the police or law enforcement personnel and the person or persons they have arrested and presented before him or her, to inquire of them the reasons why they have arrested the person and determine by questioning and personal observation whether or not the suspect has been subjected to any physical, moral or psychological torture by whom and why. The judge shall then submit a written report of what he/she had observed when the subject was brought before him to the proper court that has jurisdiction over the case of the person thus arrested. the judge shall forthwith submit his/her report within three (3) calendar days from the time the suspect was brought to his/her residence or office. Immediately after taking custody of a person charged with or suspected of the crime of terrorism or conspiracy to commit terrorism, the police or law enforcement personnel shall notify in writing the judge of the court nearest the place of apprehension or arrest: Provided, That where the arrest is made during saturdays, sundays, holidays or after office hours, the written notice shall be served at the residence of the judge nearest the place where the accused was arrested. The penalty of ten (10) years and one day to twelve (12) years of imprisonment shall be imposed upon the police or law enforcement personnel who fails to notify any judge as provided in the preceding paragraph. SEC. 19. Period of Detention in the Event of an Actual or Imminent Terrorist Attack. In the event of an actual or imminent terrorist attack, suspects may not be detained for more than three (3) days without the written approval of a municipal, city, provincial or regional official of a Human Rights Commission or judge of the municipal, regional trial court, the Sandiganbayan or a justice of the Court of Appeals nearest the place of the arrest. If the arrest is made during Saturdays, Sundays, holidays or after office hours, the arresting police or law enforcement personnel shall bring the person thus arrested to the residence of any of the officials mentioned above that is nearest the place where the accused was arrested. The approval in writing of any of the said officials shall be secured by the police or law enforcement personnel concerned within five (5) days after the date of the detention of the persons concerned: Provided, however, That within three (3) days after the detention the suspects, whose connection with the terror attack or threat is not established, shall be released immediately.

SEC. 26. Restriction on Travel. In cases where evidence of guilt is not strong, and the person charged with the crime of terrorism or conspiracy to commit terrorism is entitled to bail and is granted the same, the court, upon application by the prosecutor, shall limit the right of travel of the accused to within the municipality or city where he resides or where the case is pending, in the interest of national security and public safety, consistent with Article III, Section 6 of the Constitution. Travel outside of said municipality or city, without the authorization of the court, shall be deemed a violation of the terms and conditions of his bail, which shall then be forfeited as provided under the Rules of Court. He or she may also be placed under house arrest by order of the court at his or her usual place of residence. While under house arrest, he or she may not use telephones, cellphones, e-mails, computers, the internet or other means of communications with people outside the residence until otherwise ordered by the court. The restrictions abovementioned shall be terminated upon the acquittal of the accused or of the dismissal of the case filed against him or earlier upon the discretion of the court on motion of the prosecutor or of the accused.

Frank Uy & Unifish Packing Corp. vs Bureau of Internal Revenue et al


Search and Seizure Requisites of a Valid Search Warrant In Sept 1993, Rodrigo Abos, a former employee of UPC reported to the BIR that Uy Chin Ho aka Frank Uy, manager of UPC, was selling thousands of cartons of canned cartons without issuing a report. This is a violation of Sec 253 & 263 of the Internal Revenue Code. In Oct 1993, the BIR requested before RTC Cebu to issue a search warrant. Judge Gozo-Dadole issued a warrant on the same day. A second warrant was issued which contains the same substance but has only one page, the same was dated Oct 1st 2003. These warrants were issued for the alleged violation by Uy of Sec 253. A third warrant was issued on the same day for the alleged violation of Uy of Sec 238 in relation to sec 263. On the strength of these warrants, agents of the BIR, accompanied by members of the PNP, on 2 Oct 1993, searched the premises of the UPC. They seized, among other things, the records and documents of UPC. A return of said search was duly made by Labaria with the RTC of Cebu. UPC filed a motion to quash the warrants which was denied by the RTC. They appealed before the CA via certiorari. The CA dismissed the appeal for a certiorari is not the proper remedy. ISSUE: Whether or not there was a valid search warrant issued. HELD: The SC ruled in favor of UPC and Uy in a way for it ordered the return of the seized items but sustained the validity of the warrant. The SC ruled that the search warrant issued has not met some basic requisites of validity. A search warrant must conform strictly to the

requirements of the foregoing constitutional and statutory provisions. These requirements, in outline form, are:

(1) the warrant must be issued upon probable cause; (2) the probable cause must be determined by the judge himself and not by the applicant or any other person; (3) in the determination of probable cause, the judge must examine, under oath or affirmation, the complainant and such witnesses as the latter may produce; and (4) the warrant issued must particularly describe the place to be searched and persons or things to be seized.

The SC noted that there has been inconsistencies in the description of the place to be searched as indicated in the said warrants. Also the thing to be seized was not clearly defined by the judge. He used generic itineraries. The warrants were also inconsistent as to who should be searched. One warrant was directed only against Uy and the other was against Uy and UPC. The SC however noted that the inconsistencies wered cured by the issuance of the latter warrant as it has revoked the two others.

Section 2, Article III of the Constitution guarantees the right of the people against unreasonable searches and seizures:

The right of the people to be secure in their persons, houses, papers, and effects against unreasonable searches and seizures of whatever nature and for any purpose shall be inviolable, and no search warrant or warrant of arrest shall issue except upon probable cause to be determined personally by the judge after examination under oath or affirmation of the complainant and the witnesses he may produce, and particularly describing the place to be searched and the persons or things to be seized.

NOTES

Rule 126 of the Rules of Court provides:

SEC. 3. Requisite for issuing search warrant. A search warrant shall not issue but upon probable cause in connection with one specific offense to be determined personally by the judge after examination under oath or affirmation of the complainant and the witnesses he may produce, and particularly describing the place to be searched and the things to be seized.

SEC. 4. Examination of complainant; record. The judge must, before issuing the warrant, personally examine in the form of searching questions and answers, in writing and under oath the complainant and any witnesses he may produce on facts personally known to them and attach to the record their sworn statements together with any affidavits submitted.

Frank Uy & Unifish Packing Corp. vs Bureau of Internal Revenue et al


Search and Seizure Requisites of a Valid Search Warrant In Sept 1993, Rodrigo Abos, a former employee of UPC reported to the BIR that Uy Chin Ho aka Frank Uy, manager of UPC, was selling thousands of cartons of canned cartons without issuing a report. This is a violation of Sec 253 & 263 of the Internal Revenue Code. In Oct 1993, the BIR requested before RTC Cebu to issue a search warrant. Judge Gozo-Dadole issued a warrant on the same day. A second warrant was issued which contains the same substance but has only one page, the same was dated Oct 1st 2003. These warrants were issued for the alleged violation by Uy of Sec 253. A third warrant was issued on the same day for the alleged violation of Uy of Sec 238 in relation to sec 263. On the strength of these warrants, agents of the BIR, accompanied by members of the PNP, on 2 Oct 1993, searched the premises of the UPC. They seized, among other things, the records and documents of UPC. A return of said search was duly made by Labaria with the RTC of Cebu. UPC filed a motion to quash the warrants which was denied by the RTC. They appealed before the CA via certiorari. The CA dismissed the appeal for a certiorari is not the proper remedy. ISSUE: Whether or not there was a valid search warrant issued. HELD: The SC ruled in favor of UPC and Uy in a way for it ordered the return of the seized items but sustained the validity of the warrant. The SC ruled that the search warrant issued has not met some basic requisites of validity. A search warrant must conform strictly to the requirements of the foregoing constitutional and statutory provisions. These requirements, in outline form, are:

(1) the warrant must be issued upon probable cause;

(2) the probable cause must be determined by the judge himself and not by the applicant or any other person; (3) in the determination of probable cause, the judge must examine, under oath or affirmation, the complainant and such witnesses as the latter may produce; and (4) the warrant issued must particularly describe the place to be searched and persons or things to be seized.

The SC noted that there has been inconsistencies in the description of the place to be searched as indicated in the said warrants. Also the thing to be seized was not clearly defined by the judge. He used generic itineraries. The warrants were also inconsistent as to who should be searched. One warrant was directed only against Uy and the other was against Uy and UPC. The SC however noted that the inconsistencies wered cured by the issuance of the latter warrant as it has revoked the two others.

Section 2, Article III of the Constitution guarantees the right of the people against unreasonable searches and seizures:

The right of the people to be secure in their persons, houses, papers, and effects against unreasonable searches and seizures of whatever nature and for any purpose shall be inviolable, and no search warrant or warrant of arrest shall issue except upon probable cause to be determined personally by the judge after examination under oath or affirmation of the complainant and the witnesses he may produce, and particularly describing the place to be searched and the persons or things to be seized.

NOTES

Rule 126 of the Rules of Court provides:

SEC. 3. Requisite for issuing search warrant. A search warrant shall not issue but upon probable cause in connection with one specific offense to be determined personally by the judge after examination under oath or affirmation of the complainant and the witnesses he may produce, and particularly describing the place to be searched and the things to be seized.

SEC. 4. Examination of complainant; record. The judge must, before issuing the warrant, personally examine in the form of searching questions and answers, in writing and under oath the complainant and any witnesses he may produce on facts personally known to them and attach to the record their sworn statements together with any affidavits submitted.

People of the Philippines vs Rosa Aruta y Menguin


Search and Seizure Informers Tip In the morning of 13 Dec 1988, the law enforcement officers received information from an informant named Benjie that a certain Aling Rosa would be leaving for Baguio City on 14 Dec 1988 and would be back in the afternoon of the same day carrying with her a large volume of marijuana; At 6:30 in the evening of 14 Dec 1988, Aruta alighted from a Victory Liner Bus carrying a travelling bag even as the informant pointed her out to the law enforcement officers; NARCOM officers approached her and introduced themselves as NARCOM agents; When asked by Lt. Abello about the contents of her travelling bag, she gave the same to him; When they opened the same, they found dried marijuana leaves; Aruta was then brought to the NARCOM office for investigation. ISSUE: Whether or not the conducted search and seizure is constitutional. HELD: The SC ruled in favor of Aruta and has noted that some drug traffickers are being freed due to technicalities. Aruta cannot be said to be committing a crime. Neither was she about to commit one nor had she just committed a crime. Aruta was merely crossing the street and was not acting in any manner that would engender a reasonable ground for the NARCOM agents to suspect and conclude that she was committing a crime. It was only when the informant pointed to Aruta and identified her to the agents as the carrier of the marijuana that she was singled out as the suspect. The NARCOM agents would not have apprehended Aruta were it not for the furtive finger of the informant because, as clearly illustrated by the evidence on record, there was no reason whatsoever for them to suspect that accused-appellant was committing a crime, except for the pointing finger of the informant. The SC could neither sanction nor tolerate as it is a clear violation of the constitutional guarantee against unreasonable search and seizure. Neither was there any semblance of any compliance with the rigid requirements of probable cause and warrantless arrests. Consequently, there was no legal basis for the NARCOM agents to effect a warrantless search of Arutas bag, there being no probable cause and the accused-appellant not having been lawfully arrested. Stated otherwise, the arrest being incipiently illegal, it logically follows that the subsequent search was similarly illegal, it being not incidental to a lawful arrest. The constitutional guarantee against unreasonable search and seizure must perforce operate in favor of accused-appellant. As such, the articles seized could not be used as evidence against accused-appellant for these are fruits of a poisoned tree and, therefore, must be rejected, pursuant to Article III, Sec. 3(2) of the Constitution. Read full text here.

NOTES:

When is a warrantless search allowed? 1. Warrantless search incidental to a lawful arrest recognized under Section 12, Rule 126 of the Rules of Court 8 and by prevailing jurisprudence;

2. Seizure of evidence in plain view, the elements of which are:

(a) a prior valid intrusion based on the valid warrantless arrest in which the police are legally present in the pursuit of their official duties;

(b) the evidence was inadvertently discovered by the police who had the right to be where they are;

(c) the evidence must be immediately apparent, and

(d) plain view justified mere seizure of evidence without further search;

3. Search of a moving vehicle. Highly regulated by the government, the vehicles inherent mobility reduces expectation of privacy especially when its transit in public thoroughfares furnishes a highly reasonable suspicion amounting to probable cause that the occupant committed a criminal activity;

4. Consented warrantless search;

5. Customs search;

6. Stop and Frisk; and

7. Exigent and Emergency Circumstances.

The People of the Philippines vs Ruben Montilla y Gatdula


Political Law Search and Seizure Informers Tip Warrantless Arrest On 19 June 1994 at about 2pm, police officers Talingting and Clarin were informed by an asset that a drug courier would be arriving from Baguio to Dasmarias carrying an undetermined amount of marijuana. The next day, the informant pointed at Montilla as the courier who was waiting in a waiting shed Brgy Salitran, Dasmarias. Montilla was then apprehended and he was caught in possession of a bag and a carton worth 28 kilos of marijuana. Montilla denied the allegation and he said he came to Cavite from Baguio for work and he does not have any effects with him at that time except for some pocket money. He was sentenced to death thereafter. He averred that the search and seizure conducted was illegal for there was no warrant and that he should have been given the opportunity to cross examine the informant. He said that if the informant has given the cops the information about his arrival as early as the day before his apprehension, the cops should have ample time to secure a search warrant. ISSUE: Whether or not the warrantless arrest conducted is legal. HELD: The SC ruled that the warrantless arrest is legal. Sec 2 Art 3 of the Constitution has its exception, they are: (1) customs searches; (2) searches of moving vehicles, (3) seizure of evidence in plain view; (4) consented searches; (5) searches incidental to a lawful arrest; (6) stop and frisk measures have been invariably recognized as the traditional exceptions. In the case at bar, it should be noted that the information relayed by informant to the cops was that there would be delivery of marijuana at Barangay Salitran by a courier coming from Baguio in the early morning of June 20, 1994. Even assuming that the policemen were not pressed for time, this would be beside the point for, under these circumstances, the information relayed was

too sketchy and not detailed enough for the obtention of the corresponding arrest or search warrant. While there is an indication that the informant knew the courier, the records do not reveal that he knew him by name. On such bare information, the police authorities could not have properly applied for a warrant, assuming that they could readily have access to a judge or a court that was still open by the time they could make preparations for applying therefor, and on which there is no evidence presented by the defense. In determining the opportunity for obtaining warrants, not only the intervening time is controlling but all the coincident and ambient circumstances should be considered, especially in rural areas. A legitimate warrantless arrest, as above contemplated, necessarily cloaks the arresting police officer with authority to validly search and seize from the offender (1) dangerous weapons, and (2) those that may be used as proof of the commission of an offense.

Harry Stonehill et al vs DOJ Secretary Jose Diokno et al


Search and Seizure General Warrants Abandonment of the Moncado Doctrine Stonehill et al and the corporation they form were alleged to have committed acts in violation of Central Bank Laws, Tariff and Customs Laws, Internal Revenue (Code) and Revised Penal Code. By the strength of this allegation a search warrant was issued against their persons and their corporation. The warrant provides authority to search the persons above-named and/or the premises of their offices, warehouses and/or residences, and to seize and take possession of the following personal property to wit: Books of accounts, financial records, vouchers, correspondence, receipts, ledgers, journals, portfolios, credit journals, typewriters, and other documents and/or papers showing all business transactions including disbursements receipts, balance sheets and profit and loss statements and Bobbins (cigarette wrappers). The documents, papers, and things seized under the alleged authority of the warrants in question may be split into (2) major groups, namely: (a) those found and seized in the offices of the aforementioned corporations and (b) those found seized in the residences of petitioners herein. Stonehill averred that the warrant is illegal for: (1) they do not describe with particularity the documents, books and things to be seized; (2) cash money, not mentioned in the warrants, were actually seized;

(3) the warrants were issued to fish evidence against the aforementioned petitioners in deportation cases filed against them; (4) the searches and seizures were made in an illegal manner; and (5) the documents, papers and cash money seized were not delivered to the courts that issued the warrants, to be disposed of in accordance with law. The prosecution counters, invoking the Moncado doctrine, that the defects of said warrants, if any, were cured by petitioners consent; and (3) that, in any event, the effects seized are admissible in evidence against them. In short, the criminal cannot be set free just because the government blunders. ISSUE: Whether or not the search warrant issue is valid. HELD: The SC ruled in favor of Stonehill et al. The SC emphasized however that Stonehill et al cannot assail the validity of the search warrant issued against their corporation for Stonehill are not the proper party hence has no cause of action. It should be raised by the officers or board members of the corporation. The constitution protects the peoples right against unreasonable search and seizure. It provides; (1) that no warrant shall issue but upon probable cause, to be determined by the judge in the manner set forth in said provision; and (2) that the warrant shall particularly describe the things to be seized. In the case at bar, none of these are met. The warrant was issued from mere allegation that Stonehill et al committed a violation of Central Bank Laws, Tariff and Customs Laws, Internal Revenue (Code) and Revised Penal Code. In other words, no specific offense had been alleged in said applications. The averments thereof with respect to the offense committed were abstract. As a consequence, it was impossible for the judges who issued the warrants to have found the existence of probable cause, for the same presupposes the introduction of competent proof that the party against whom it is sought has performed particular acts, or committed specific omissions, violating a given provision of our criminal laws. As a matter of fact, the applications involved in this case do not allege any specific acts performed by herein petitioners. It would be a legal heresy, of the highest order, to convict anybody of a violation of Central Bank Laws, Tariff and Customs Laws, Internal Revenue (Code) and Revised Penal Code, as alleged in the aforementioned applications without reference to any determinate provision of said laws or codes. The grave violation of the Constitution made in the application for the contested search warrants was compounded by the description therein made of the effects to be searched for and seized, to wit: Books of accounts, financial records, vouchers, journals, correspondence, receipts, ledgers, portfolios, credit journals, typewriters, and other documents and/or papers showing all business transactions including disbursement receipts, balance sheets and related profit and loss statements. Thus, the warrants authorized the search for and seizure of records pertaining to all business transactions of Stonehill et al, regardless of whether the transactions were legal or illegal. The

warrants sanctioned the seizure of all records of Stonehill et al and the aforementioned corporations, whatever their nature, thus openly contravening the explicit command of the Bill of Rights that the things to be seized be particularly described as well as tending to defeat its major objective: the elimination of general warrants. The Moncado doctrine is likewise abandoned and the right of the accused against a defective search warrant is emphasized.

Bache & Co. Inc. et al vs BIR Commissioner Vivencio Ruiz et al


Search and Seizure Personal Examination of the Judge

On 24 Feb 1970, Commissioner Vera of Internal Revenue, wrote a letter addressed to J Ruiz requesting the issuance of a search warrant against petitioners for violation of Sec 46(a) of the NIRC, in relation to all other pertinent provisions thereof, particularly Sects 53, 72, 73, 208 and 209, and authorizing Revenue Examiner de Leon make and file the application for search warrant which was attached to the letter. The next day, de Leon and his witnesses went to CFI Rizal to obtain the search warrant. At that time J Ruiz was hearing a certain case; so, by means of a note, he instructed his Deputy Clerk of Court to take the depositions of De Leon and Logronio. After the session had adjourned, J Ruiz was informed that the depositions had already been taken. The stenographer read to him her stenographic notes; and thereafter, J Ruiz asked respondent Logronio to take the oath and warned him that if his deposition was found to be false and without legal basis, he could be charged for perjury. J Ruiz signed de Leons application for search warrant and Logronios deposition. The search was subsequently conducted. ISSUE: Whether or not there had been a valid search warrant. HELD: The SC ruled in favor of Bache on three grounds. 1. J Ruiz failed to personally examine the complainant and his witness. Personal examination by the judge of the complainant and his witnesses is necessary to enable him to determine the existence or non-existence of a probable cause. 2. The search warrant was issued for more than one specific offense. The search warrant in question was issued for at least four distinct offenses under the Tax Code. As ruled in Stonehill Such is the seriousness of the irregularities committed in connection with the disputed search warrants, that this Court deemed it fit to amend Section 3 of Rule 122 of the former Rules of Court that a search warrant shall not issue but upon probable cause in connection with one specific offense. Not satisfied with this qualification, the Court added thereto a paragraph, directing that no search warrant shall issue for more than one specific offense. 3. The search warrant does not particularly describe the things to be seized.

The documents, papers and effects sought to be seized are described in the Search Warrant Unregistered and private books of accounts (ledgers, journals, columnars, receipts and disbursements books, customers ledgers); receipts for payments received; certificates of stocks and securities; contracts, promissory notes and deeds of sale; telex and coded messages; business communications, accounting and business records; checks and check stubs; records of bank deposits and withdrawals; and records of foreign remittances, covering the years 1966 to 1970. The description does not meet the requirement in Art III, Sec. 1, of the Constitution, and of Sec. 3, Rule 126 of the Revised Rules of Court, that the warrant should particularly describe the things to be seized. A search warrant may be said to particularly describe the things to be seized when the description therein is as specific as the circumstances will ordinarily allow or when the description expresses a conclusion of fact not of law by which the warrant officer may be guided in making the search and seizure or when the things described are limited to those which bear direct relation to the offense for which the warrant is being issued.

DOJ Sec Vicente Abad Santos vs CFI Benguet Judge Pio Marcos
Search and Seizure

On March 31, 1971, Amansec went to Baguio and passed by a house at 47 Ledesma Street, Baguio he was attracted by several persons inside the house; he peeped from outside the house and when the curtain was moved he saw a Buddha that was inside the house; he observed what was going on inside the house and he heard someone say that the golden Buddha was actually for sale and when he observed them closer he overheard that it was being offered for sale for 100,000 pesos by Rogelio Roxas; he saw the Buddha and firearms and some bullets inside the house. By these facts, Colonel Calano requested for a warrant from J Marcos at about 12 midnight on Apr 4, 1971. Due to the urgency he issued the warrant. And eventually the golden Buddha and some firearms were seized from Roxass house. Santos assailed the warrant averring that the search warrant was not limited to one offense covering both illegal possession of firearms and violation of Central Bank rules and regulations; that it did not particularly describe the property to be seized; that he did not carefully examine under oath the applicant and his witnesses; that articles not mentioned were taken; and that thereafter the return and the inventory although appearing to have been prepared on said date were not actually submitted to respondent Judge until April 13, 1971 and the objects seized delivered only about a week later on April 19. ISSUE: Whether or not the search warrant issued by Judge Marcos is valid. HELD: The SC ruled in favor Judge Marcos and had basically affirmed the decision of appellate Judge Gatamaitan. Taking into consideration to nature of the articles so described, it is clear that no other more adequate and detailed description could be given, particularly because it is

difficult to give a particular description of the contents thereof, The description so made substantially complies with the legal provisions because the officer of the law who executed the warrant was thereby placed in a position enabling him to Identify the articles in question, which he did, so that here, since certainly, no one would be mistaken in Identifying the Buddha, whose image is well known, and even the firearms and ammunition because these were those without permit to possess, and all located at 47 Ledesma St., Baguio City, so far as description was concerned, the search warrant perhaps could not be said to have suffered fatal defects.
Castro vs. Pabalan (70 SCRA 477) Posted by taxcasesdigest on Tuesday, July 14, 2009 Labels: constitutional law, search and seizure, search warrant Facts: Judge Pabalan ordered the issuance of a search warrant despite failure of the application of Lumang or the warrant itself to specify the offense, to examine the applicant as well as his witnesses on the part of the Judge, and to describe with particularity the place to be searched and the things to be seized. Judge never refuted the assertions when required to answer. Application alleged that applicants wee informed and claimed that they verified the report that Maria Castro and Co Ling are in possession of narcotics and other contraband in Barrio Padasil, Bangar, La Union without specifying the particular place in the Barrio. No complete description of the goods and inquiry was brief. Upon actual search, it turned out that it was in Barrio Ma. Cristina and not in Padasil. Issue: Whether or not the search warrant is validly issued. Held: Search warrant issued illegal for violation of the 1935 Constitution and the Rules of Court because the two basic requirements are not complied with: (a) no warrant shall issue but upon probable cause, (b) the warrant shall particularly describe the things to be seized, thus, a general warrant. However, things seized cannot be returned and shall be destroyed, except the liquors, playing cards, distilled water and five bottles of Streptomycin.

G.R. No. L-25232 December 20, 1973 ASIAN SURETY and INSURANCE COMPANY, INC., petitioner, vs. HON. JOSE HERRERA, as Judge, City Court of Manila, NBI Agent CELSO J. ZOLETA, JR. and MANUEL CUARESMA, respondents. Astraquillo, Laquio, Brillantes and Associates, Taada, Carmon and Taada and Alidio, Elegir, Anchete and Catipon petitioner. Assistant Solicitor General Pacifico P. de Castro and Solicitor Augusto M. Amores for respondent Celso J. Zoleta, Jr. Antonio Barredo for respondent Manuel Cuaresma.

ESGUERRA, J.: Petition to quash and annul a search warrant issued by respondent Judge Jose Herrera of the City Court of Manila, and to command respondents to return immediately the documents, papers, receipts and records alleged to have been illegally seized thereunder by agents of the National Bureau of Investigation (NBI) led by respondent Celso Zoleta, Jr. On October 27, 1965, respondent Judge Herrera, upon the sworn application of NBI agent Celso Zoleta, Jr. supported by the deposition of his witness, Manuel Cuaresma, issued a search warrant in connection with an undocketed criminal case for estafa, falsification, insurance fraud, and tax evasion, against the Asian Surety and Insurance Co., a corporation duly organized and existing under the laws of the Philippines, with principal office at Room 200 Republic Supermarket Bldg., Rizal Avenue, Manila. The search warrant is couched in the following language:
It appearing to the satisfaction of the undersigned, after examining under oath NBI Agent Celso J. Zoleta, Jr. and his witness Manuel Cuaresma that there are good and sufficient reasons to believe that Mr. William Li Yao or his employees has/have in his/their control in premises No. 2nd Floor Republic Supermarket Building, in Rizal Avenue district of Sta. Cruz, Manila, property (Subject of the offense; stolen or embezzled and proceeds or fruits of the offense used or intended to be used as the means of committing the offense) should be seized and brought to the undersigned. You are hereby commanded to make an immediate search at any time in the ----- of the premises above-described and forthwith seize and take possession of the following personal property to wit: Fire Registers, Loss Bordereau, Adjusters Report including subrogation receipt and proof of loss, Loss Registers, Books of Accounts, including cash receipts and disbursements and general ledger, check vouchers, income tax returns, and other papers connected therewith ... for the years 1961 to 1964 to be dealt with as the law directs.

Armed with the search warrant Zoleta and other agents assigned to the Anti-graft Division of the NBI entered the premises of the Republic Supermarket Building and served the search warrant upon Atty. Alidio of the insurance company, in the presence of Mr. William Li Yao, president and chairman of the board of directors of the insurance firm. After the search they seized and carried away two (2) carloads of documents, papers and receipts. Petitioner assails the validity of the search warrant, claiming that it was issued in contravention of the explicit provisions of the Constitution and the Rules of Court, particularly Section 1, of Art. III of the 1935 Constitution, now Section 3, of Art. IV of the new Constitution, and Sections 3, 5, 8 and 10 of Rule 126 of the Rules of Court, hereunder quoted for convenience of reference, viz:
Sec. 3 The rights of the people to be secure in their persons, houses, papers and effects, against unreasonable searches and seizures shall not be violated, and no

warrant shall issue but upon probable cause to be determined by the judge after examination under oath or affirmation of the complainant and the witnessed he may produce, and particularly describing the place to be searched, and the persons, or things to be seized." (Art. IV, Section 3, New Constitution) Sec. 3 Requisites for issuing search warrant A search warrant shall not issue but upon probable cause in connection with one specific offense to be determined by the judge or justice of the peace after examination under oath or affirmation of the complainant and the witnesses he may produce, and particularly describing the place to be searched and the persons or things to be seized. No search warrant shall issue for more than one specific offense. (Sec. 3, Rule 126, Rules of Court) Sec. 5 Issuance and form of search warrant If the judge or justice of the peace is thereupon satisfied of the existence of facts upon which the application is based, or that there is probable cause to believe that they exist, he must issue the warrant in the form prescribed by these rules. (Sec. 5, Rule 126) Sec. 8 Time of making search The warrant must direct that it be served in the day time, unless the affidavit asserts that the property is on the person or in the place ordered to be searched, in which case a direction may be inserted that it be served at any time of the night or day. (Sec. 8, Rule 126) Sec. 10 Receipt for property seized. The officer seizing property under the warrant must give a detailed receipt for the same to the person on whom or in whose possession it was found, or in the absence of any person, must, in the presence of at least one witness, leave a receipt in the place in which he found the seized property. (Sec. 10, Rule 126) .

"Of all the rights of a citizen, few are of greater importance or more essential to his peace and happiness than the right of personal security, and that involves the exemption of his private affairs, books, and papers from the inspection and scrutiny of others. 1 While the power to search and seize is necessary to the public welfare, still it must be exercised and the law enforced without transgressing the constitutional rights of the citizens, for the enforcement of no statute is of sufficient importance to justify indifference to the basic principles of government (People v. Elias, 147 N.E. 472)." I. In the case at bar, the search warrant was issued for four separate and distinct offenses of : (1) estafa, (2) falsification, (3) tax evasion and (4) insurance fraud, in contravention of the explicit command of Section 3, Rule 126, of the Rules providing that: "no search warrant shall issue for more than one specific offense." The aforequoted provision, which is found in the last paragraph of the same section, is something new. "There is no precedent on this amendment prohibition against the issuance of a search warrant for more than one specific offense either in the American books on Criminal procedure or in American decisions." 2 It was applied in the celebrated case of Harry S. Stonehill v. Secretary of Justice 3 where this Court said:

To uphold the validity of the warrants in question would be to wipe out completely one of the most fundamental rights guaranteed in our Constitution, for it would place the sanctity of the domicile and the privacy of communication and correspondence at the mercy of the whims, caprice or passion of peace officers. This is precisely the evil sought to be remedied by the constitutional provision abovequoted to outlaw the so-called general warrants. It is not difficult to imagine what would happen in times of keen political strife, when the party in power feels that the minority is likely to wrest it, even though by legal means. Such is the seriousness of the irregularities committed in connection with the disputed search warrants, that this Court deemed it fit to amend section 3 of Rule 122 of the former Rules of Court by providing in its counterpart, under the Revised Rules of Court, that a search warrant shall not issue but upon probable cause in connection with one specific offense. Not satisfied with this qualification, the court added thereto a paragraph, directing that no search warrant shall issue for more than one specific offense.

II. Petitioner likewise contests the validity of the search warrant on the ground that it authorized the search and seizures of personal properties so vaguely described and not particularized, thereby infringing the constitutional mandate requiring particular description of the place to be searched and the persons or things to be seized. It also assails the noncompliance with the above-requirement as likewise openly violative of Section 2 of Rule 126 which provides:
SEC. 2. A search warrant may be issued for the search and seizure of the following personal property: (a) Property subject of the offense; (b) Property stolen or embezzled and other proceeds or fruits of the offense; and (c) Property used or intended to be used as the means of committing an offense.

The search warrant herein involved reads in part: "... property (Subject of the offense, stolen or embezzled and proceeds or fruits of the offense used or intended to be used as the means of committing the offense) should be seized and brought to the undersigned." The claim of respondents that by not cancelling the description of one or two of the classes of property contained in the form when not applicable to the properties sought to be seized, the respondent judge intended the search to apply to all the three classes of property. This is a patent impossibility because the description of the property to be searched and seized, viz: Fire Registers, Loss Bordereau, Adjusters Report, including subrogation receipts and proof of loss, Loss Registers, Books of Accounts including cash receipts and disbursements and general ledger, etc. and the offenses alleged to have been committed by the corporation to wit: estafa, falsification, tax evasion and insurance fraud, render it impossible for Us to see how the abovedescribed property can simultaneously be contraband goods, stolen or embezzled and other proceeds or fruits of one and the same offense. What is plain and clear is the fact that the respondent Judge made no attempt to determine whether the property he authorized to be searched and seized pertains specifically to any one of the three

classes of personal property that may be searched and seized under a search warrant under Rule 126, Sec. 2 of the Rules. The respondent Judge simply authorized search and seizure under an omnibus description of the personal properties to be seized. Because of this all embracing description which includes all conceivable records of petitioner corporation, which if seized (as it was really seized in the case at bar), could possibly paralyze its business, 4 petitioner in several motions, filed for early resolution of this case, manifested that the seizure of TWO carloads of their papers has paralyzed their business to the grave prejudice of not only the company, its workers, agents, employees but also of its numerous insured and beneficiaries of bonds issued by it, including the government itself, and of the general public. 5 And correlating the same to the charges for which the warrant was issued, We have before Us the infamous general warrants of old. In the case of Uy Kheytin, et al., v. Villareal, 42 Phil. 896, cited with approval in the Bache case, supra, We had occasion to explain the purpose of the requirement that the warrant should particularly describe the place to be searched and the things to be seized, to wit:
"... Both the Jones Law (sec. 3) and General Orders No. 58 (sec. 97) specifically require that a search warrant should particularly describe the place to be searched and the things to be seized. The evident purpose and intent of this requirement is to limit the things to be seized to those, and only those, particularly described in the search warrant to leave the officers of the law with no discretion regarding what articles they shall seize, to the end that "unreasonable searches and seizures" may not be made. That this is the correct interpretation of this constitutional provision is borne out by American authorities."

The purpose as thus explained could, surely and effectively, be defeated under the search warrant issued in this case. III. Moreover, as contended by petitioner, respondents in like manner transgressed Section 10 of Rule 126 of the Rules for failure to give a detailed receipt of the things seized. Going over the receipts (Annexes "B", "B-1", B-2", "B-3" and "B-4" of the Petition) issued, We found the following: one bordereau of reinsurance, 8 fire registers, 1 marine register, four annual statements, folders described only as Bundle gm-1 red folders; bundle 17-22 big carton folders; folders of various sizes, etc., without stating therein the nature and kind of documents contained in the folders of which there were about a thousand of them that were seized. In the seizure of two carloads of documents and other papers, the possibility that the respondents took away private papers of the petitioner, in violation of his constitutional rights, is not remote, for the NBI agents virtually had a field day with the broad and unlimited search warrant issued by respondent Judge as their passport. IV. The search warrant violated the specific injunctions of Section 8 of Rule 126. 6 Annex "A" of the Petition which is the search warrant in question left blank the "time" for making search, while actual search was conducted in the evening of October 27, 1965, at 7:30 p.m., until the wee hours of the morning of October 28, 1965, thus causing

untold inconveniences to petitioners herein. Authorities 7 are of the view that where a search is to be made during the night time, the authority for executing the same at that time should appear in the directive on the face of the warrant. In their Memorandum 8 respondents, relying on the case of Moncado v. Peoples Court (80 Phil. 1), argued:
Even assuming that the search warrant in question is null and void, the illegality thereof would not render the incriminating documents inadmissible in evidence.

This Court has reverted to the old rule and abandoned the Moncado ruling (Stonehill case, supra). Most common law jurisdictions have already given up this approach and eventually adopted the exclusionary rule, realizing that this is the only practical means of enforcing the constitutional injunction against unreasonable searches and seizures. Thus the Supreme Court of the United States declared: 9
If letters and private documents can thus be seized and held and used in evidence against a citizen accused of an offense the protection of the 4th Amendment, declaring his right to be secured against such searches and seizures is of no value, and so far as those thus placed are concerned, might as well be stricken from the Constitution. The efforts of the courts and their officials to bring the guilty to punishment, praise-worthy as they are, are not to be aided by the sacrifice of those great principles established by years of endeavor and suffering which have resulted in their embodiment in the fundamental law of the land.

Moreover, the criminal charges filed by the NBI have all been dismissed and/or dropped by the Court or by the office of the City Fiscal of Manila in 1968, as manifested in the petition filed by petitioner dated October 24, 1972, for early resolution of this case. V. It has likewise been observed that the offenses alleged took place from 1961 to 1964, and the application for search warrant was made on October 27, 1965. The time of the application is so far remote in time as to make the probable cause of doubtful veracity and the warrant vitally defective. Thus Mr. Joseph Varon, an eminent authority on Searches, Seizures and Immunities, has this to say on this point:
From the examination of the several cases touching upon this subject, the following general rules are said to apply to affidavits for search warrants: (1) xxx xxx xxx (2) Such statement as to the time of the alleged offense must be clear and definite and must not be too remote from the time of the making of the affidavit and issuance of the search warrant. (3) There is no rigid rule for determining whether the stated time of observation of the offense is too remote from the time when the affidavit is made or the search warrant issued, but, generally speaking, a lapse of time of more than three weeks will be held not to invalidate the search warrant while a lapse of four weeks will be held to be so.

A good and practical rule of thumb to measure the nearness of time given in the affidavit as to the date of the alleged offense, and the time of making the affidavit is thus expressed: The nearer the time at which the observation of the offense is alleged to have been made, the more reasonable the conclusion of establishment of probable cause. [Emphasis Ours]

PREMISES CONSIDERED, petition is hereby granted; the search warrant of October 27, 1965, is nullified and set aside, and the respondents are hereby ordered to return immediately all documents, papers and other objects seized or taken thereunder. Without costs. Makalintal, C.J., Castro, Fernandez * and Muoz Palma, JJ., concur. Makasiar, J., concurs in the result.
A sian S urety vs. Herrera Facts:On October 1 965, upon a sworn application of NBIagent Celso Zoleta Jr. supported with the deposition of witness Manuel Cuaresma, the respondent Judge JoseHerrera, issued a search warrant against the petitioner for criminal case of Estafa, falsification, insurance fraud and taxevasion.By virtue of the search warrant, NBI agents seizedthe place in the office of the petitioner in Republi Market andcarried away two car loads of documents, papers andreceipt.The petitioners, then filed a suit assailing the validityof the SW, contending that it doesnot follow theConstitutional and statutory requirements of a valid SW.Issue:What are the violated procedures in the case atbar?Held: S ingle warrant single offense rule. General warrant. The constitution requires that a SW should beissued upon a probable cause in connection with one singleoffense.In the case at bar, the SW was issued for 4separate and distinct offenses. Estafa, falsification, taxevasion and insurance fraud. Therefore it is invalid for it is ageneral warrant. Particular description of the objects to be seized. The constitution mandates that objects to be seizedshould be couched not on generic but specific terms. S ection 2 provides that a S W may be issued for the search and seizure of the following personal properties.a) P roperty subject to the offenseb) P roperty stolen or embezzled and other proceeds or fruits of the offense c) P roperty used or intended to be used as themeans of committing the offense In the case at bar, the respondent judge usedall three of the description in relation to the things to beseized in the petitioner. Thus, they are all couched in Constitutional Law II (Bill of Rights): Case Briefs: Dennis G. Libunao UC College of Law 27 generic terms. The respondent judge did not bother tospecify the things to be seized that would be admitted asan evidence to the offense charged. The Rule on RR C that the S

W should be issued on dayti m e. In the case at bar, the SW was conducted eveningof Oct 27, 1 965 at 7:3 0 pm until morning. Re m oteness of the ti m e of the offense and theapplication of the S W . Joseph Varon provides rules to apply affidavits for SW. 1 ) Such statement as to the time of the alleged offensemust be clear and definite and must not be tooremote from the time of the making of the affidavitand issuance of the search warrant2) There is no rigid rule for determining whether thestated time of observation of the offense id tooremote from the time when the affidavit was madeor the search warrant issued but generally speaking,a lapse of time of less than three weeks will be heldnot to invalidate the search warrant, while the lapseof four weeks will be held to be so.Thus, the nearer the time at which the observation of anoffense is alleged to have been made, the more reasonablethe conclusion of establishment of a probable cause.In the case at bar, the alleged commission of thecrime is from 1 96 1 to 1 964 and the application for SW ismade 1 965. thus, there can be doubt as to the establishmentof a probable cause because of the remoteness of time.

Viduya vs. Berdiago


73 SCRA 553 (1976)

Except in the case of the search of a dwelling house, persons exercising police authority under the customs law may effect search and seizure without a search warrant in the enforcement of customs laws.

FACTS: Respondent Berdiago is the owner of a Rolls Royce car, Model 1966, which arrived in the Port of Manila on January 8, 1968. However, the petitioner, Jose Viduya, then Collector of Customs of Manila, obtained reliable intelligence that fraudulent documents were used by Berdiago in securing the release of the car from the Bureau of Customs, making it appear therein that the car was a 1961 model instead of a 1966 one, thus enabling respondent to pay a much lower customs duty.

There was, accordingly, a formal demand for the payment of the sum to cover the deficiency, respondent manifesting his willingness to do so but failing to live up to his promise. As the car was kept in a dwelling house at the Yabut Compound, two officials of the Customs Police Service as duly authorized agents of petitioner, applied to respondent Judge for a warrant to search said dwelling house and to seize the Rolls Royce car found therein. Berdiago filed a motion to quash the search warrant issued by the court based on lack of probable cause to issue the warrant. Collector Viduya opposed, alleging that Berdiago could not rely on the constitutional right against unreasonable search and seizure because it was not shown that he owned the dwelling house which was searched. Nonetheless, respondent Judge in the challenged order quashed such search warrant. Hence, this petition. ISSUE:

Whether or not respondent Judge committed grave abuse of discretion in quashing the warrant

HELD: The Court opined that except in the case of the search of a dwelling house, persons exercising police authority under the customs law may effect search and seizure without a search warrant in the enforcement of customs laws. There is justification then for the insistence on the part of private respondent that probable cause be shown. So respondent Judge found in issuing the search warrant. Apparently, he was persuaded to quash it when he noted that the warrant for seizure and detention came later than its issuance. In thus acting, respondent Judge apparently overlooked that long before the search warrant was applied for, to be specific on April 15, 1968, the misdeclaration and underpayment was already noted and that thereafter on April 24, 1968, private respondent himself agreed to make good the further amount due but not in the sum demanded. As the car was kept in a dwelling house, petitioner through two of his officers in the Customs Police Service applied for and was able to obtain the search warrant. Had there been no such move on the part of petitioner, the duties expressly enjoined on him by law assess and collect all lawful revenues, to prevent and suppress smuggling and other frauds and to enforce tariff and customs law would not have been performed. While therefore, it is to be admitted that his warrant of seizure and detention came later than the search warrant, there were indubitable facts in existence at that time to call for its issuance. Certainly

there was probable cause. There was evidently need for the issuance of a search warrant. It ought not to have been thereafter quashed.

October 20, 1925 G.R. No. L-23051 THE PEOPLE OF THE PHILIPPINES ISLANDS, plaintiff-appellant, vs. JOSE MA. VELOSO, defendant-appellant. Claro M. Recto for appellant. Attorney-General Villa-Real for appellee. Malcolm, J.: This is an appeal from a judgment of the Court of First Instance of Manila finding the accused, Jose Ma. Veloso, guilty of the crime of resistance of the agents of the authority, in violation of article 252 of the Penal Code, and sentencing him to four months and one day imprisonment, arresto mayor, with the accessory penalties, to pay a fine of P200, with the corresponding subsidiary imprisonment in case of insolvency, and to pay the costs. The errors assigned by counsel for the accused as appellant, go to the proposition that the resistance of the police was justifiable on account of the illegality of the John Doe search warrant. In May, 1923, the building located at No. 124 Calle Arzobispo, City of Manila, was used by an organization known as the Parliamentary Club. Jose Ma. Veloso was at that time a member of the House of Representative of the Philippine Legislature. He was also the manager of the club. The police of Manila had reliable information that the so-called Parliamentary Club was nothing more than a gambling house. Indeed, on May 19, 1923, J. F. Townsend, the chief of the gambling squad, had been to the club and verified this fact. As a result, on May 25, 1923, Detective Andres Geronimo of the secret service of the City of Manila, applied for, and obtained a search warrant from Judge Garduo of the municipal court. Thus provided, the police attempted to raid the Parliamentary Club a little after three in the afternoon of the date abovementioned. They found the doors to the premises closed and barred. Accordingly, one band of police including policeman Rosacker, ascended a telephone pole, so as to enter a window of the house. Other policemen, headed by Townsend, broke in the outer door. Once inside the Parliamentary Club, nearly fifty persons were apprehended by the police. One of them was the defendant Veloso. Veloso asked Townsend what he wanted, and the latter showed him the search warrant. Veloso read it and told Townsend that he was Representative Veloso and not John Doe, and that the police had no right to search the house. Townsend answered that Veloso was considered as John Doe. As Veloso's pocket was bulging, as if it contained gambling utensils, Townsend required Veloso to show him the evidence of the game. About five minutes was consumed in conversation between the policemen and the accused the policemen insisting on searching Veloso, and Veloso insisting in his refusal to submit to the search.

At last the patience of the officers was exhausted. So policeman Rosacker took hold of Veloso only to meet with his resistance. Veloso bit Rosacker in the right forearm, and gave him a blow in another part of the body, which injured the policeman quite severely. Through the combined efforts of Townsend and Rosacker, Veloso was finally laid down on the floor, and long sheets of paper, of reglas de monte, cards, cardboards, and chips were taken from his pockets. All of the persons arrested were searched and then conducted to the patrol wagons. Veloso again refused to obey and shouted offensive epithets against the police department. It was necessary for the policemen to conduct him downstairs. At the door, Veloso resisted so tenaciously that three policemen were needed to place him in the patrol wagon. In the municipal court of the City of Manila, the persons arrest in the raid were accused of gambling. All of them were eventually acquitted in the Court of First Instance for lack of proof, with the sole exception of Veloso, who was found guilty of maintaining a gambling house. This case reached the appellate court where the accused was finally sentenced to pay a fine of P500. (No. 22163. 1 ) The foregoing are the principal facts taken mainly from the findings of the trial judge, the Honorable Vicente Nepomuceno. Counsel for the appellant makes no effort to impugn these findings, except that he stresses certain points as more favorable to the case of his client. The defense, as previously indicated, is planted squarely on the contention that since the name of Veloso did not appear in the search warrant, but instead the pseudonym John Doe was used, Veloso had a legal right to resist the police by force. The nature of this defense makes it advisable to set forth further facts, relating particularly to the search warrant, before passing to the law. There are found in the record the application for search warrant, the affidavit for search warrant, and the search warrant. The application reads: UNITED STATES OF AMERICA PHILIPPINE ISLANDS IN THE MUNICIPAL COURT OF THE CITY OF MANILA THE PEOPLE OF THE PHILIPPINE ISLANDS, plaintiff, vs. JOHN DOE, Defendant. APPLICATION FOR (G) SEARCH WARRANT Testimony taken before Hon. L. Garduo, Judge, Municipal Court, Manila. Andres Geronimo, being duly sworn, testifies as follows: Q. What is your name, residence and occupation? A. Andres Geronimo, No. 47 Revellin,

detective. Q. Are you the applicant of this search warrant? A. Yes, sir. Q. Do you know the premises situated at No. 124 Calle Arzobispo, District of W. C., City of Manila? A. Yes. sir. Q. Do you know who occupies said premises? A. I do not know. According to the best of my information the house is occupied by John Doe. Q . What are your reasons for applying for this search warrant? A. It has been reported to me by a person whom I consider to be reliable that in said premises there are instruments and devices used in gambling games, such as cards, dice, chips, lottery tickets, lists of drawing and lists used in prohibited games kept. It has been reported to me by a person whom I consider to be reliable that there are or there will be gambling conducted in said premises. The aforesaid premises are known as gambling house. I have watched the foregoing premises and believed it to be a gambling house and a place where instruments and devices used in gambling games, such as cards, dice, chips, lottery tickets, lists of drawing and lists used in prohibited games are kept. I, Andres Geronimo, being duly sworn, depose and say that I have read the foregoing questions and answers and that I find the same to correct and true to the best of my knowledge and belief. (Sgd.) ANDRES GERONIMO Subscribed and sworn to before me this 25th day of May, 1923. (Sgd.) L. GARDUO Judge, Municipal Court

The affidavit and the search warrant are so nearly alike that it will suffice to copy the search warrant alone. This document reads: UNITED STATES OF AMERICA PHILIPPINE ISLANDS IN THE MUNICIPAL COURT OF THE CITY OF MANILA THE PEOPLE OF THE PHILIPPINE ISLANDS, Plaintiff, vs. JOHN DOE, Defendant. SEARCH WARRANT (G)

The People of the Philippine Islands, to any member of the Police Force of the City of Manila. GREETING: Proof by affidavit having this day been made before me by Andres Geronimo that he has good reason to believe and does believe that John Doe has illegally in his possession in the building occupied by him and which is under his control, namely in the building numbered 124 Calle Arzobispo, City of Manila, Philippines Islands, certain devices and effects used in violation of the Gambling Law, to wit: money, cards, chips, reglas, pintas, tables and chairs and other utensils used in connection with the game commonly known as monte and that the said John Doe keeps and conceals said devices and effects with the illegal and criminal intention of using them in violation of the Gambling Law. Now therefore, you are hereby commanded that at any time in the day or night within ten (10) days on or after this date to make a search on the person of said John Doe and in the house situated at No. 124 Calle Arzobispo, City of Manila, Philippine Islands, in quest of the above described devices and effects and if you find the same or any part thereof, you are commanded to bring it forthwith before me as provided for by law. Given under my hand, this 25th day of May, 1923. (Sgd.) L. GARDUO Judge, Municipal Court Coming now to the legal aspects of the case it is first worthy of mention that by reason of the Fourth Amendment to the United States Constitution and the eleventh and eighteenth paragraphs of the Philippine Bill of Rights, as found in the present Organic Act, the security of the dwelling and the person is guaranteed. The organic act provides "that the right to be secured against unreasonable searches and seizures shall not be violated." It further provides "that no warrant shall issue but upon probable cause, supported by oath or affirmation and particularly describing the place to be searched and the person or things to be seized." In the Philippine Code of Criminal Procedure are found provisions of the same import although naturally entering more into detail. It is therein provided, among other things, that "a search warrant shall not issue except for probable cause and upon application supported by oath particularly describing the place to be searched and the person of thing to be seized." (Section 97.) After the judge or justice shall have examined on oath the complainant and any witnesses he may produce, and shall have taken their depositions in writing (section 98), and after the judge or

justice is satisfied of the existence of facts upon which the application is based, or that there is probable cause to believe that they exist, he must issue the warrant which must be substantially in the following form: . . . You are, therefore, commanded, . . . to make immediate search on the person of ............................, or in the house situated ...................................... (describing it or any other place to be searched with reasonable particularity, as the case may be) for the following property: . . . ." (Section 99.) It is finally provided that "a person charged with a crime may be searched for dangerous weapons or anything which may be used as proof of the commission of the crime. (Section 105). A search warrant must conform strictly to the requirements of the constitutional and statutory provisions under which it is issued. Otherwise it has rightly been held, must be absolutely legal, "for there is not a description of process known to the law, the execution of which is more distressing to the citizen. Perhaps there is none which excites such intense feeling in consequence of its humiliating and degrading effect." The warrant will always be construed strictly without, however, going the full length of requiring technical accuracy. No presumptions of regularity are to be invoked in aid of the process when an officer undertakes to justify under it. (24 R. C. L., pp. 711, et seq.; Reed vs. Rice [1829], 2 J. J. Marshall [Ky.] 44; 19 Am. Dec., 122; Smith vs. McDuffee [1914], 72 Ore., 276; Ann. Cas. 1916 D, 947.) The search warrant has been likened to a warrant of arrest. Although apprehending that there are material differences between the two, in view of the paucity of authority pertaining to John Doe search warrants we propose to take into consideration the authorities relied upon by the appellant, thus following the precedent of Uy Kheytin vs. Villareal ([1920], 42 Phil., 886), where the regularity of the issuance of the search warrant was also questioned. In the lower court, and again in this court, the attorneys for the defense quoted from Wharton's Criminal Procedure. In that text at pages 51, 52, 54, 55, and 56 of volume 1 of the Tenth Edition, is found the following: Form and Sufficiency of Warrant. Technical accuracy is not required. . . uklVBdbi. xxxxxxxxx Name and description of the accused should be inserted in the body of the warrant and where the name is unknown there must be such a description of the person accused as will enable the officer to identify him when found 8UYIXlM. xxxxxxxxx

Warrant for apprehension of unnamed party, or containing a wrong name for the party to be apprehended is void, except in those cases where it contains a descriptio personae such as will enable the officer to identify the accused. xxxxxxxxx John Doe' Warrants. It follows, on principle, from what has already been said regarding the essential requirements of warrants for the apprehension of persons accused, and about blank warrants, that a warrant for the apprehension of a person whose true name is unknown, by the name of "John Doe" or "Richard Roe," "whose other or true name in unknown," is void, without other and further descriptions of the person to be apprehended, and such warrant will not justify the officer in acting under it. Such a warrant must, in addition, contain the best descriptio personae possible to be obtained of the person or persons to be apprehended, and this description must be sufficient to indicate clearly the proper person or persons upon whom the warrant is to be served; and should state his personal appearance and peculiarities, give his occupation and place of residence, and any other circumstances by means of which he can be identified. Person apprehended in act of committing a crime, under a "John Doe" warrant, on the other hand, the apprehension will not be illegal, or the officer liable, because under such circumstances it is not necessary that a warrant should have been issued. The authority most often cited to sustain the text, and quoted with approval by the United States Supreme Court, is the case of Commonwealth vs. Crotty ([1865], 10 Allen [Mass.], 403). It there appeared that one Peaslee had made a complaint to the police court Lee, charging that "John Doe or Richard Roe, whose other or true name is to your complainant unknown," had committed an assault and battery upon him; upon which complaint a warrant was issued against "John Doe or Richard Roe, whose other or true name is to your complainant unknown, named in the foregoing complaint." Neither the complaint nor the warrant contained any further description or means of identification of the person to be arrested. Crotty resisted the arrest upon the ground that the warrant was invalid. Mr. Chief Justice Bigelow, as the organ of the Supreme Court of Massachusetts, said: We cannot entertain a doubt that the warrant on which the officer attempted to arrest one of the defendant at the time of the alleged riot was insufficient, illegal and void. It did not contain the name of the defendant, nor any description or designation by which he could be known and identified as the person against whom it was issued. It was in effect a general warrant, upon which any other individual might as well have been arrested, as being included in the description, as the defendant himself. Such a warrant was contrary to elementary principles, and in direct violation of the constitutional right of the citizen, as set forth in the Declaration of

Rights, article 14, which declares that every subject has a right to be secure from all unreasonable searches and seizures of his person, and that all warrants, therefore, are contrary to this right, if the order in the warrant to a civil officer to arrest one or more suspected persons or to seize their property be not accompanied with a special designation of the persons or objects of search, arrest or seizure. This is in fact only a declaration of an ancient common law right. It was always necessary to express the name or give some description of a party to be arrested on a warrant; and if one was granted with the name in blank, and without other designation of the person to be arrested, it was void. (1 Hale P. C. 577. 2 Ib. 119. Foster, 312. 7 Dane Ab. 248. 1 Chit. Crim. Law, 39. Mead vs. Haws, 7 Cow., 332, and cases cited.) This rule or principle does not prevent the issue and service of a warrant against a party whose name is unknown. In such case the best description possible of the person to be arrested is to be given in the warrant; but it must be sufficient to indicate clearly on whom it is to be served, by stating his occupation, his personal appearance and peculiarities, the place of his residence, or other circumstances by which he can be identified. (1 Chit. Crim. Law, 39, 40.) The warrant being defective and void on its face, the officer had no right to arrest the person on whom he attempted to serve it. He acted without warrant and was a trespasser. The defendant whom he sought to arrest had a right to resist by force, using no more than was necessary to resist the unlawful acts of the officer . . . The defendants, therefore, in resisting the officer in making an arrest under the warrant in question, if they were guilty of no improper or excessive force or violence, did not do an unlawful act by lawful means, or a lawful act by unlawful means, and so could not be convicted of the misdemeanor of a riot, with which they are charged in the indictment. Appellant's argument, as based on these authorities, runs something like this. The law, constitutional and statutory, requires that the search warrant shall not issue unless the application "particularly" describe the person to be seized. A failure thus to name the person is fatal to the validity of the search warrant. To justify search and arrest, the process must be legal. Illegal official action may be forcibly resisted hi3Pq6RbJJ. For the prosecution, however, as the arguments are advanced by the Attorney-General, and as the law was summarized by the trial judge, there is much to be said. Careful and logical reflection brings forth certain points of paramount force and exercising a decisive influence. We will now make mention of them by correlating the facts and the law. In the first place, the affidavit for the search warrant and the search warrant itself described the building to be searched as "the building No. 124 Calle Arzobispo, City of Manila, Philippine Islands." This, without doubt, was a sufficient designation of the premises to be searched. It is

the prevailing rule that a description of a place to be searched is sufficient if the officer with the warrant can, with reasonable effort, ascertain and identify the place intended. (Steele vs. U. S. [1925], U. S. Supreme Court Advance Opinions 1924-1925; 69 Law. ed., 757). The police officers were accordingly authorized to break down the door and enter the premises of the building occupied by the so-called Parliamentary Club. When inside, they then had the right to arrest the persons presumably engaged in a prohibited game, and to confiscate the evidence of the commission of the crime. It has been held that an officer making an arrest may take from the person arrested any money or property found upon his person, which was used in the commission of the crime or was the fruit of the crime, or which may furnish the person arrested with the means of committing violence or of escaping, or which may be used as evidence on the trial of the cause, but not otherwise. (Moreno vs. Ago Chi [1909], 12 Phil., 439.) Proceeding along a different line of approach, it is undeniable that the application for the search warrant, the affidavit, and the search warrant failed to name Jose Ma. Veloso as the person to be seized. But the affidavit and the search warrant did state that "John Doe has illegally in his possession in the building occupied by him, and which is under his control, namely, in the building numbered 124 Calle Arzobispo, City of Manila, Philippine Islands, certain devices and effects used in violation of the Gambling Law." Now, in this connection, it must not be forgotten that the Organic Act requires a particular description of the place to be searched, and the person or things to be seized, and that the warrant in this case sufficiently described the place and the gambling apparatus, and, in addition, contained a description of the person to be seized. Under the authorities cited by the appellant, it is invariably recognized that the warrant for the apprehension of an unnamed party is void, "except in those cases where it contains a description personae such as will enable the officer to identify the accused." The description must be sufficient to indicate clearly the proper person upon whom the warrant is to be served. As the search warrant stated that John Doe had gambling apparatus in his possession in the building occupied by him at No. 124 Calle Arzobispo, City of Manila, and as this John Doe was Jose Ma. Veloso, the manager of the club, the police could identify John Doe as Jose Ma. Veloso without difficulty. Again, it must be remembered that No. 124 Calle Arzobispo was supposed to be used for club purposes. It was not the home of Veloso; not the place of abode of the family, which the law carefully protects in all of its sanctity. It was a club partially public in nature. It was, moreover, a camouflaged club with a high sounding name calculated to mislead the police, but intended for nefarious practices. In a club of such a character, unlike in the home, there would commonly be varying occupancy, a number of John Does and Richard Roes whose names would be unknown to the police. It is also borne out by the authorities that, in defense of himself, any member of his family or his dwelling, a man has a right to employ all necessary violence. But even in the home, and much

less so in a club or public place, the person sought to be arrested or to be searched should use no more force than is necessary to repel the unlawful act of the officers. To authorize resistance to the agents of the authority, the illegality of the invasion must be clearly manifest. Here, there was possibly a proper case for protest. There was no case for excessive violence to enforce the defendant's idea of a debatable legal question. (Commonwealth vs. Crotty, supra; People vs. Chan Fook [1921], 42 Phil., 230; 3 Groizard, Codigo Penal, pp. 456, 457.) The trial judge deduced from the searched warrant that the accused Veloso was sufficiently identified therein. Mention was made by his Honor of the code provision relating to a complaint or information, permitting a fictitious name to be inserted in the complaint or information, in lieu of the true name. The Attorney-General adds to this the argument that the police were authorized to arrest without a warrant since a crime was being committed. We find it unnecessary to comment on this contention. John Doe search warrants should be the exception and not the rule. The police should particularly describe the place to be searched and the person or things to be seized, wherever and whenever it is feasible. The police should not be hindered in the performance of their duties, which are difficult enough of performance under the best of conditions, by superficial adherence to technicality or far fetched judicial interference 9G8mA7jwJm. We agree with the trial judge and with the Attorney-General in their conclusions to the effect that the search warrant was valid, and that the defendant has been proved guilty beyond a reasonable doubt, of the crime of resistance of the agents of the authority. The information alleges that at the time of the commission of the crime, the accused was a member of the House of Representatives. The trial court was led to consider this allegation in relation with the facts as an aggravating circumstance, and to sentence the accused accordingly. We doubt, however, that advantage was taken by the offender of his public position when he resisted the officers of the law. The offender did not necessarily make use of the prestige of his office as a means to commit a crime. Undoubtedly, Jose Ma. Veloso, as Juan de la Cruz, would have resisted the police just as stoutly, as the Honorable Jose Ma. Veloso did. The penalty, accordingly, falls within the medium of that provided by the Penal Code. Finding present no reversible error, agreeing in all respects with the findings of facts as made by the trial judge, and concurring with the trial judge in his legal conclusion, with one exception, it results that the judgment appealed from must be, as it is hereby, affirmed, with the sole modification that the defendant and appellant shall be sentenced to two months and one day imprisonment, arresto mayor, with the costs of this instance against him. Let the corresponding order to carry this judgment into effect issue.

Avancea, C.J., Street, Villamor, Ostrand, Johns, and Romualdez, JJ., concur. Villa-Real, JJ., took no part. G.R. No. 141176 May 27, 2004

ELI LUI and LEO ROJAS, petitioners, vs. SPOUSES EULOGIO and PAULINA MATILLANO, respondents. DECISION CALLEJO, SR., J.: This is a petition for review on certiorari of the Decision1 of the Court of Appeals in CA-G.R. CV No. 44768 which reversed and set aside the decision of the Regional Trial Court of Bansalan, Davao del Sur, Branch 21.2 The Antecedents Sometime in September 1987, then seventeen-year-old Elenito Lariosa visited his aunt, his fathers older sister, Paulina Lariosa Matillano, at Lily Street, Poblacion Bansalan, Davao del Sur. On May 2, 1988, Lariosawas employed as a laborer at the Davao United Products Enterprise store, with a monthly salary of P800.00. The store was owned by Leong Shiu Ben and King Kiao and was located at the corner of Monteverde and Gempesaw Streets, Davao City. Lariosa was tasked to close the store during lunchtime and after store hours in the afternoon. Ben himself opened the store in the mornings and after lunchtime. Adjacent to the said store was another store owned by Kiaos son, Eli Lui, who also happened to be Bens nephew. Aside from Lariosa, Ben and Kiao employed Maximo Pagsa and Rene Malang. Lariosa chose to live in the house of Kiao. Lariosa fed the dogs of his employer every morning before going to work and in the afternoon, in exchange for free meals and lodging. There were occasions when Lariosa stayed in the house of Pagsa and Malang and left some of his things with them. Lariosa deposited his savings with the Mindanao Savings Bank in Bansalan. On October 17, 1988, Lariosa was taken ill and was permitted to take the day off. He went to the house of his aunt, Paulina Matillano, and her husband Eulogio Matillano in Bansalan City, where he rested until the next day, October 18, 1988. Lariosa reported for work the day after, or on October 19, 1988, but Kiao told him that his employment was terminated. Lariosa was not paid his salary for the month of October. Kiao warned Lariosa not to report the matter to the Department of Labor. Lariosa decided to return to Bansalan without retrieving his things from Kiaos house. On October 27, 1988, Lariosa returned to Davao City and was able to collect his backwages from Ben in the amount of P500.00. Lariosa withdrew his savings from the Mindanao Savings Bank in Bansalan City and on November 1, 1988, applied for a job at his cousins place, at Quimpo

Boulevard, Davao City. He bought a radio cassette for P2,500.00 and a pair of Rayban sunglasses for P900.00. On November 3, 1988, Lariosa went to the house of his fiancee, Nancy, at New Matina, Davao City, but returned to Bansalan on the same day. On November 4, 1988, he returned to Nancys house and stayed there until the next day, November 5, 1988. That day, Ben informed his nephew, Eli Lui, that he had lost P45,000.00 in cash at the store. Ben reported the matter to NBI Senior Agent Ruperto Galvez, and forthwith executed an affidavit wherein he alleged that after Lariosas employment was terminated on October 19, 1988, he discovered that he had lost P45,000.00 in cash. He suspected that Lariosa was the culprit because the latter, as a former employee, had a duplicate key to the side door of the United Products Enterprise Store. At 9:00 a.m. on November 6, 1988, a Sunday, Lariosa went to the house of Pagsa and Malang to retrieve his things. The two invited Lariosa to go with them to the beach, and when Lariosa agreed, they borrowed Luis Ford Fierra for their transportation. The vehicle stopped at the Almendras Hall where Pagsa alighted on the pretext that he was going to buy fish. Lariosa, Rene, and his wife remained in the Fierra. Pagsa contacted Lui and informed the latter that Lariosa was with him. After about an hour, Lui arrived on board a vehicle. With him were Pagsa and two others, Alan Mendoza and Henry Tan. Lui told Lariosa that he wanted to talk, and asked the latter to go with him. Pagsa urged Lariosa to go along with Lui. Lariosa agreed and boarded Luis vehicle. The car stopped in front of Luis house, where the latter alighted and went inside, while his companions and Lariosa remained in the car. When Lui returned, he was armed with a 9 mm. caliber gun and poked Lariosa with the weapon. He warned Lariosa not to run, otherwise, he would be killed. The group went to Bens house to get the keys to the store. Ben joined them as they drove towards the store. Lui mauled Lariosa and tried to force the latter to admit that he had stolen Bens money. Lariosa refused to do so. Lui then brought Lariosa to the comfort room of the store and pushed his face into the toilet bowl, in an attempt to force him into confessing to the crime. Lariosa still refused to admit to anything. Lui then made a telephone call to the Metrodiscom (PNP) based in Davao City. Sgt. Alberto Genise of the Metrodiscom (PNP) issued Mission Order No. MRF-A-004-88 dated November 6, 1988, directing Pat. Leo Rojas "to follow up a theft case committed in Davao City from 12:30 p.m. to 5:00 p.m." Rojas was directed to coordinate with the nearest PNP headquarters and/or stations. He was authorized to carry his firearm for the mission. He then left the police station on board a police car and proceeded to the corner of Magsaysay and Gempesaw Streets. In the meantime, a police car arrived at the store with two policemen on board. One of them handcuffed Lariosa at gunpoint and ordered him to open the store with the use of the keys. As Lariosa opened the lock as ordered, one of Luis companions took his picture. Another picture

was taken as Lariosa held the door knob to open the door. Lariosa was then boarded in the police car and brought to the corner of Magsaysay and Gemphesaw Streets where he was transferred to the police car driven by Rojas. He was brought to the Metrodiscom headquarters. Lui once more mauled Lariosa, still trying to force the latter to confess that he stole P45,000.00 from his uncle and to reveal what he did with the money. When a policeman asked him where he slept the night before, Lariosa replied that he spent the night in the house of his girlfriends parents at New Matina, Davao City. The policemen brought Lariosa there, where they asked Nancy if Lariosa had left anything while he slept thereat. Nancy replied that Lariosa had left a radio cassette and a pair of sunglasses. The policemen took these and brought Lariosa back to the Metrodiscom headquarters where Lui and his two companions were waiting. Lui asked Lariosa where he stayed when he went to Bansalan, and Lariosa replied that he used to stay in the house of his aunt and uncle, the Spouses Matillano, in Lily Street, Poblacion Bansalan. Rojas and Lui then brought Lariosa, with his hands still handcuffed, to a car. Luis companions, Alan Mendoza and Henry Tan boarded another car and proceeded to the Matillano residence. Without prior coordination with the Bansalan PNP, Rojas, who was in civilian clothes, Lui, Tan and Mendoza arrived at the house of the Spouses Matillano at about 3:00 p.m, with the handcuffed Lariosa in tow. With handguns drawn, they kicked the door to the kitchen and gained entry into the house. They then proceeded to the sala where they found Lariosas aunt, Paulina Matillano. In the adjacent room were Julieta, Lariosas sister, Paulinas daughter-in-law, Virginia, the latters sister, Erlinda, and a seven-month-old baby. Paulina was shocked. Rojas told Paulina, "Mrs., we are authorities. We are here to get something." Paulina remonstrated, "Why are you meddling (manghilabot)?" Lui poked his gun at Paulina and warned her not to talk anymore because something might happen. He then said, "All right, where is your aparador because we are getting something." Paulina told Lui to wait for her husband Eulogio. Lui ignored her protest and told her that they were in a hurry. Paulina was then impelled to bring Lui and his two companions, Mendoza and Tan, to the second floor where her aparador was located. Rojas and the handcuffed Lariosa remained in the sala. Lui and his two companions then took two mats and two pairs of ladies shoes belonging to Paulina and Eulogio, two pairs of pants, leather shoes, two t-shirts and two polo shirts which belonged to the latters children. They also ordered Paulina to open a chest and when she did, Lui and his companions took her old Bulova wristwatch, necklace, ring and old coins. Lui and his two companions then went down to the ground floor. When Julieta went out of the room, one of Luis companions recognized her as Lariosas sister. Lui and his companions brought her along with them as they left the house. Paulina was so unnerved by the incident. Her vision blurred, her stomach ached and she was on the verge of losing consciousness. Concerned, Erlinda massaged Paulinas stomach. However, Erlinda had to leave because she was worried about her mother. Paulina then went to the kitchen, prepared hot water and put a soothing ointment on her stomach to relieve the pain. In the meantime, Lui and his companions proceeded to the Bansalan Police Station and caused an entry in the police blotter at 3:20 p.m. that he had recovered the following items from the

Matillano residence -- one pair of colored blue pants valued at P89.00; one floor mat costing P290.00; a pair of black ladies shoes worth P126.00; and another pair of ladies shoes worth P69.00. At 4:30 p.m., Paulina reported to the barangay captain that persons identifying themselves as policemen had gained entry into their house and took the following: two polo shirts; two t-shirts; two pairs of pants; two floor mats; two pairs of ladies shoes; one Bulova wristwatch; one necklace; one ring; and old coins.3 At 7:35 p.m., Eulogio Matillano made an entry in the Bansalan police blotter that earlier that day, at 4:00 p.m., Rojas took the following from his house: two polo shirts; two t-shirts; 2 pairs of pants; two floor mats; two pairs of ladies shoes; 1 Bulova wristwatch; 1 necklace; one ring; and, old coins, without his and his wifes consent and without a search warrant.4 In the meantime, Doroteo Barawan, officer-in-charge of the Office of the Barangay Captain, filed a complaint against Kim Kiao, et al., based on the complaint of Paulina, docketed as Barangay Case No. 168.5 On November 8, 1988, Lariosa executed an uncounselled confession where he stated that he stole P40,000.00 on October 15, 1988 from the Davao United Products, and that he used part of the money to buy appliances, a Sony cassette tape-recorder, two pairs of ladies shoes, a Seiko wristwatch, two pairs of maong pants, Rayban sunglasses and floor mats.6 On November 16, 1988, an Information was filed in the Regional Trial Court of Davao City, charging Lariosa with robbery with force upon things. The case was docketed as Criminal Case No. 17,136,88.7 The trial court rendered judgment on June 14, 1989, acquitting Lariosa of the crime charged on reasonable doubt. The trial court held that Lui procured Lariosas confession through force and intimidation, in connivance with police authorities.8 The trial court, likewise, found that Lui had an ulterior motive for charging Lariosa of robbery: What would have been the possible motive of complainant in putting the burden of this charged against the accused despite want of any appreciable evidence, can be gathered in the record, as indicating the fear of complainant, that the accused will file a complaint against him in the Department of Labor for illegally dismissing him in his employment, without any sufficient legal grounds and basis. This unfounded complaint was intended to support complainants ground against any possible complaint, the accused might file against him with the Department of Labor by way of anticipation.9 On motion of Lariosa, the trial court ordered the return of the following exhibits: Accordingly and conformably with the judgment of this court dated June 14, 1989, one Eulogio Matillano, accuseds uncle, is hereby allowed to get or to retrieve exhibits "H," "I," "J," "K," "L," and "M," consisting of Sony Cassette with serial no. W3658; Rayban sunglasses; two (2) bundles of floor mat; two (2) pairs of pants; two (2) pairs of ladies shoes; and Seiko Actus wristwatch.10

Meanwhile, Paulina Matillano filed a criminal complaint for robbery against Lui, Peter Doe, John Doe and Alan Mendoza. An Information was, thereafter, filed against them in the Municipal Circuit Trial Court of Bansalan, Davao del Sur, and the case was docketed as Criminal Case No. 880-B. On December 13, 1988, the court issued a warrant for the arrest of the accused therein. Upon reinvestigation, however, the Provincial Prosecutor issued a Resolution dated March 31, 1989, recommending that the case be dismissed for insufficiency of evidence, but that the charges be forwarded to the Judge Advocate Generals Office for possible administrative sanctions against Rojas. WHEREFORE, in view of the foregoing, it is respectfully recommended that the complaint against the respondents Eli Lui be dismissed for insufficiency of evidence. Considering that Pat. Leo Rojas is a member of the Integrated National Police, this office is without jurisdiction to entertain the complaint against him pursuant to Presidential Decree No. 1850. Therefore, let the complaint against Pat. Leo Rojas, together with its annexes, including a copy of the resolution of the undersigned, be forwarded to the Judge Advocate Generals Office at Camp Catitipan, Davao City, for whatever action it may take.11 The complaint was docketed as Administrative Case No. 92-0020. The National Police Commission, thereafter, rendered a decision exonerating Rojas of administrative liability for the complainants failure to substantiate the charges.12 The Commission held that Rojas was merely complying with the mission order issued to him when he accompanied Lui and the latters two companions to the Matillano residence. In a Resolution dated August 25, 1989, then Secretary of Justice Silvestre H. Bello III dismissed the petition for review of the Provincial Prosecutors resolution filed by Paulina Matillano. The Secretary of Justice, likewise, denied a motion for reconsideration thereon. In a parallel development, Lariosas parents, as well as Paulina Matillano, filed a complaint for robbery, violation of domicile, unlawful arrest and/or arbitrary detention against Leo Rojas, Eli Lui, et al., with the Commission of Human Rights docketed as CHR Case No. RFO No. 880207-DS. In a Resolution dated December 4, 1989, the Regional Office of the Commission recommended, thus: WHEREFORE, premises considered, we are recommending that there is sufficient prima facie evidence: 1. to indict Eli Lui for unlawful arrest as defined under Art. 369 of the Revised Penal Code, as amended; and 2. to indict both Eli Lui and Pat. Leo Rojas liable for Violation of Domicile, as defined under Art. 128 of the same code.13 The Proceedings in the Trial Court On January 11, 1990, the spouses Eulogio and Paulina Matillano filed a civil complaint for damages in the Regional Trial Court of Davao del Sur against Eli Lui, Leo Rojas, Alan Mendoza

and Henry Tan. The case was docketed as Civil Case No. G-XXI-47(90). The plaintiffs therein alleged the following: 3. That plaintiffs are merchants by occupation and have been residing in Bansalan, Davao del Sur, for several years now. They are law-abiding and peaceful citizens in the community; 4. That at about 3:00 oclock in the afternoon of November 6, 1988, while plaintiff husband was away from his residential house at Lily St., Bansalan, Davao del Sur, and plaintiff wife was there tending the house, defendants, without any lawful search warrant, arrived and thru intimidation succeeded in searching the house owned by the plaintiff after which they brought with them two floor mats, two pairs of ladies shoes, two pairs of pants, two polo shirts, two T-shirts, one Relova wrist watch, one necklace (sinubong), one ring (sinubong) and several old coins, without the consent of the plaintiffs and without even giving any receipt for the items taken; 5. That the defendants allegedly wanted to recover the items taken by one Elinito Lariosa but defendants thru the use of naked power and brute force, illegally searched the house of the herein plaintiffs in gross violation of plaintiffs constitutional rights; 6. That what defendants did in conspiring and confederating to illegally search the house of plaintiffs and then taking with them the items mentioned above without even the benefit of any receipt is not only violative of Article 19 in relation to Article 21 of the Civil Code but also of Article 32 of the Civil Code; 7. That because of what defendants did, plaintiffs suffered mental anguishes, wounded feelings, deprivation of the properties taken, besmirched reputation, and fright for which reason defendants should be made to jointly and severally pay moral damages in the amount of P500,000.00; 8. That in order to deter others similarly bent and minded and by way of example or correction for the public good, defendants should be made to pay jointly and severally exemplary damages in the amount of P300,000.00; 9. That in the protection of their rights, plaintiffs engaged the services of counsel for an agreed attorneys fees equivalent to 25% of the total award plus per diem of P1,000.00 per court appearance; 10. That plaintiffs are bound to incur litigation expenses in an amount not less than P10,000.00;14 They prayed that, after due proceedings, judgment be rendered in their favor, viz: WHEREFORE, it is most respectfully prayed that after hearing judgment issue ordering the defendants to jointly and severally pay plaintiffs:

1. P500,000.00 as moral damages; 2. P300,000.00 as exemplary damages; 3. Litigation expenses of P10,000.00; 4. Attorneys fees equivalent to 25% of the total award; 5. Per diems to be proved during the trial of this case. Plaintiffs pray for other reliefs consistent with equity.15 In their Answer to the complaint, the defendants therein alleged, inter alia, that they did not conduct a search in the house of the plaintiffs and that plaintiff Paulina Matillano allowed them to enter the house and even brought out pairs of pants. They added that the other items were brought out by Lariosas sister and that they took only one (1) floor mat, two (2) pairs of ladies shoes, and one (1) pair of blue pants.16 The defendants adduced evidence that plaintiff Paulina Matillano allowed them to enter their house, and with Lariosas sister, voluntarily turned over the items declared in the complaint. They testified that no violence, threats or intimidation were even committed by them against Paulina Matillano. Defendant Rojas further testified that he was merely complying with the Mission Order issued to him when he entered the house of the plaintiffs in the company of the other defendants, and that he remained in the ground floor while the other defendants retrieved the goods from plaintiff Matillano in the second floor of the house. On August 18, 1993, the RTC rendered judgment, ordering the dismissal of the complaint for plaintiffs failure to prove their claims. The trial court also dismissed the defendants counterclaims. The trial court gave credence to the collective testimonies of the defendants, that plaintiff Paulina Matillano voluntarily allowed them to enter her house, and that the latter voluntarily turned over the subject items to them. The trial court took into account the findings of the Provincial Prosecutor, the Secretary of Justice, the National Police Commission, as well as the order of the Municipal Circuit Trial Court of Bansalan, dismissing Criminal Case No. 880-B. The Case on Appeal The decision of the trial court was elevated to the Court of Appeals where the appellants contended, thus: 1. THE LOWER COURT ERRED IN FINDING THAT APPELLANT PAULINA MATILLANO VOLUNTARILY ALLOWED APPELLEES TO ENTER THE HOUSE BECAUSE OF THE PRESENCE OF HER NEPHEW ELINITO LARIOSA WHO WAS HANDCUFFED;

2. THE LOWER COURT ERRED IN FINDING THAT MRS. PAULINA MATILLANO WAS THE ONE WHO REPORTED THE MATTER TO THE BANSALAN POLICE STATION. 3. THE LOWER COURT ERRED IN DISMISSING THE COMPLAINT DESPITE CLEAR PREPONDERANCE OF EVIDENCE AGAINST THE DEFENDANTS APPELLEES.17 On April 22, 1999, the Court of Appeals rendered judgment reversing the decision of the RTC. The decretal portion of the decision reads: IN VIEW OF ALL THE FOREGOING, the decision appealed from is hereby REVERSED and SET ASIDE and a new one entered ordering defendants-appellees jointly and severally: 1. To pay plaintiffs-appellants the amount of Fifty Thousand Pesos (P50,000.00) as moral damages and Fifteen Thousand Pesos (P15,000.00) as exemplary damages; and 2. Ten Thousand Pesos (P10,000.00), as attorneys fees; and 3. To pay the costs. SO ORDERED.18 The appellate court denied the appellees motion for reconsideration of the said decision. The appellees Mendoza and Tan no longer appealed the decision. Petitioners Eli Lui and Leo Rojas now assail the decision of the Court of Appeals contending that: I. THE HONORABLE COURT OF APPEALS DISREGARDED THE TIMEHONORED DOCTRINE LAID DOWN BY THIS HONORABLE COURT THAT FINDINGS OF TRIAL COURT ARE BINDING AND CONCLUSIVE AND DESERVE A HIGH DEGREE OF RESPECT, WHEN IT SET ASIDE THE FINDINGS OF FACTS AND ASSESSMENT OF THE REGIONAL TRIAL COURT THAT TRIED THE CASE; II. THE HONORABLE COURT OF APPEALS ERRONEOUSLY CONCLUDED THAT AN ILLEGAL SEARCH WAS CONDUCTED IN MRS. MATILLANOS RESIDENCE, IN DISREGARD OF THE EXCULPATORY FINDINGS OF THE TRIAL COURT THAT MRS. MATILLANO HAD VOLUNTARILY ALLOWED PETITIONERS ENTRY INTO HER HOUSE.19 The Issues

The issues in this case may be synthesized, thus: (a) whether or not respondent Paulina Matillano consented to the petitioners entry into her house, as well as to the taking of the clothes, shoes and pieces of jewelry owned by her and her family; (b) whether or not the petitioners are liable for damages to the respondents; and, (c) if so, the extent of the petitioners liability to the respondents. Considering that the assignments of errors are interrelated, this Court shall delve into and resolve them simultaneously. The Courts Ruling The petition has no merit. Admittedly, the issues in the case at bar are factual. Under Rule 45 of the Rules of Court, only questions of law may be raised in this Court in a petition for review on certiorari. However, the rule admits of some exceptions, such as a case where the findings of facts of the trial court are substantially different from those of the appellate court, and the resolution of such issues are determinative of the outcome of the petition.20 The petitioners aver that the Court of Appeals committed a reversible error in discarding the factual findings of the trial court. Contrary to the disquisitions of the appellate court, the petitioners assert that the inconsistencies between the testimonies of Rojas and Lui are peripheral. Lui did not conduct any search in the second floor of the respondents house and even if he did so, respondent Paulina Matillano waived her right against unreasonable search when she allowed the petitioners to enter. According to the petitioners, the respondents failed to prove that they forced their way into the house of the respondents, and that the facts and circumstances which the appellate court found the trial court to have overlooked are not, in fact, substantial enough to warrant a reversal of the factual findings of the court a quo. According to the petitioners, the appellate court failed to discern that the action filed by the respondents with the trial court was merely a leverage to the charge of robbery against Lariosa, the respondents nephew. On the other hand, the Court of Appeals gave credence and full probative weight to the evidence of the respondents. It stated in its decision that the trial court erred in giving credence and probative weight to the testimonies of the petitioners (the appellants therein). Moreover, the appellate court found that the trial court had overlooked facts and circumstances of substance, which, if considered, would have altered the courts decision. The appellate court gave weight to the findings of the trial court in Criminal Case No. 17,136,88.21 We agree with the Court of Appeals. The evidence of the respondents show that the petitioners, Tan and Mendoza, guns drawn and with the handcuffed Lariosa in tow, kicked the kitchen door and barged into the house of the respondents. They proceeded to the sala where respondent Paulina Matillano was. Over her vehement protests, and because of petitioner Luis warning that she might be harmed, respondent

Paulina Matillano was forced to accompany the petitioner and his cohorts to the second floor of their house. The foregoing was testified to by respondent Paulina Matillano, thus: ATTY. SUARIO: Q Mrs. Matillano, do you know the person of Eli Lui? A I know him. Q Why do you know Eli Lui? A Because he is from Bansalan. Q On November 6, 1988, where were you, Mrs. Matillano? A I was in our house. Q At about 3:00 oclock in the afternoon of November 6, 1988, did you notice any unusual incident that took place in your house? A There was. Q What incident was that, Mrs. Matillano? A There were five (5) persons who suddenly went inside our house. Q Where did they enter? A They entered through the kitchen. Q Now, where were you when they entered suddenly in your house? A I was in our sala. Q Now, what did you do when you saw these five (5) persons entered (sic) your house? A I was afraid. Q Aside from fear, what did you do? A One of them suddenly said, "Mrs., we are authorities." ATTY. TAN: Not responsive to the question, Your Honor.

ATTY. SUARIO: She is responding the question because my question is, "Aside from fear, what did you do?" and according to this witness, she was not able to do anything because one of those who entered(not continued) COURT: I think the answer is not responsive. Just reform the question. ATTY. SUARIO: Q What did these persons do when they entered your house? A One of them said, "Mrs., we are authorities. We are here to get something from your house." Q Do you know who this person was, this person who was talking that they were persons in authority? A That person when he first went to our house, I do not know him yet, but I know (sic) him later to be Leo Rojas. Q Why do you know him later to be Leo Rojas? A When the case was already being tried, he introduced himself as Leo Rojas. Q What was Leo Rojas wearing at that time? A He was in civilian clothes. Q Aside from Leo Rojas, who were the other persons who entered your house? A Aside from the two (2) persons whom I do not know, my nephew was also with them in the name of Elinito Lariosa. Q Who else, Mrs. Matillano? A Eli Lui. ATTY. SUARIO: At least, may we ask, Your Honor, that the word "manghilabot" be incorporated.

COURT: So, the word is "interfering" or "meddling." You record the word "manghilabot." ATTY. SUARIO: Q When you said "manghilabot," what do you mean, Mrs. Matillano? A Yes, because they said that they are taking some of our things and I said why are they doing that (manghilabot)? Q When you said those remarks, what else happened? A It was Eli Lui who answered, "Mrs., do not answer anymore because something might happen." (Basig madisgrasya). ATTY. SUARIO: "Madisgrasya," Your Honor, is more than something. ATTY. SUARIO: Q When you heard those words from Eli Lui, what else transpired? A He said, "All right, where is your aparador because we are getting something." And I even told him that we should wait for my husband but they did not agree because they said they are in a hurry. Q And after that, what else happened? A I accompanied him upstairs. Q You accompanied him upstairs, who are you referring to that you accompanied upstairs. A Eli Lui and his other two (2) companions. Q These two (2) companions whom you said you do not know their names? A Yes, sir.22 ATTY. TAN:

Q Now, you said on November 6, 1988, five (5) men suddenly entered your house. When you said suddenly, will you please describe how did they enter the house? A They passed through the kitchen and suddenly appeared inside the house. Q You mean to say that they did not knock at the door? A They did not. Q Who first entered the house among the five (5)? A What I first saw was that they immediately converged in the sala and whom I recognized was Eli Lui and my nephew who was in handcuffs. Q Was your door opened at that time? A It was closed but it was not locked. It can be kicked open. Q But you can open it without kicking the door? A Yes, sir. Q Now, you said that you were afraid, why were you afraid? A Why would you not be afraid when they were armed? Q Who were armed among the five (5)? A All of them except the one who was in handcuffs. Q You are very sure of that? A I am very sure.23 Respondent Paulina Matillano, likewise, testified that petitioner Lui and his cohorts took her personal things, and those of her familys, from the second floor of the house: Q Now, while you and Eli Lui with two (2) other companions were upstairs, what happened upstairs? A Upon reaching upstairs, they immediately rolled the two (2) floor mats, the pair of leather shoes, 2 pairs of pants, two (2) polo-shirts. They also let me open the chest and when it was already open they rummaged through it and they got my old Bulova watch, my necklace, my ring and a coinsita, old gold coins. Q When you said "coinsita," what is "coinsita"?

A Old coins. Q After taking all of these things, what else happened? A They went downstairs.24 Q Now, you mentioned in this affidavit that several properties were taken from your house, do you confirm that there were two (2) polo-shirts that were taken? A Yes. Q And there were also two (2) floor mats? A Yes, that is true. Q One (1) Bulova wristwatch? A Yes. Q One (1) necklace? A Yes. Q Two (2) pairs of lady (sic) shoes? A Yes. Q Two (2) pairs of pants? A Yes. Q One (1) ring? A Yes. Q Who owns these two (2) pairs of ladys (sic) shoes? A That was mine. Q What were the color of the shoes? A Black and dirty white (referring to the color of the rostrum). Q Where did you buy that shoes?

A In Davao City. Q What store in Davao City? A NCCC. Q What particular date when you bought that shoes? A I think it was in the month of November. Q 1988? A 1988. Q And who owns these two (2) polo-shirts? A My children. Q What are the names of your children? A Allan and Danilo. Q Where is Allan residing? A During the incident, Allan was still schooling in Tacloban. Q So, you mean to say, on November 6, 1988, he was no longer residing in Bansalan? A No more. Q How about Danilo, where was he residing in November 6, 1988? A He was living in Sta. Cruz. Q He has a family of his own at Sta. Cruz? A He was still single then. Q But he was residing in Sta. Cruz? A Yes. Q How about these two (2) pairs of pants, who owns these pants? A My children also.

Q You are referring to Allan and Danilo? A No, because I still have so many children. Q So, who owns these two (2) pants? A Also my children, Eulogio, Jr. and Allan. Q Now, Eulogio, Jr. where is (sic) he residing on November 6, 1988? A In our house. Q How about these two (2) t-shirts? A Also owned by my children. Q Are you referring to Allan and Danilo? A They used to wear that. Q How come that Allan has a polo-shirt in your house when you said he was then residing in Tacloban? ATTY. SUARIO: May we manifest, Your Honor, that he was schooling in Tacloban. COURT: All right. A They used to have a vacation during December and March and usually they left some of their clothes inside our aparador. Q These polo shirts were still new? A Already used. Q How about the pants? A The other one is already used and the other one is new. Q How about the floor mats? A That is mine.

Q Now, you claimed that these clothes were taken from the cabinet or aparador, is that correct? A Yes, that is true. Q Inside your aparador, how many pieces of clothes were stored therein? A Many. Q Could you say one (1) dozen? A It cannot be counted. Q Could you say three (3) dozens? A It is really full of dress. Q Would you say it is more than three (3) dozens? A More. Q And these more than three (3) dozens consists of polo shirts, t-shirts and pants? A Yes. Q And inspite (sic) the fact that there were more than three (3) dozens of clothes, pants, polo shirts and t-shirts only these two (2) pants, two (2) polo shirts and two (2) t-shirts w ere taken? A Only those things because they only selected the ones which were still usable the good ones. Q Now, you mentioned also in your affidavit that the group also searched your trunk? A I was ordered to open the trunk. Q Who particularly ordered you to open the trunk? A Eli Lui.25 The respondents immediately reported the matter to the Office of the Barangay Captain26 and filed a complaint against petitioner Lui and his cohorts.27 The petitioners claim that respondent Paulina Matillano allowed them and their cohorts inside the house and voluntarily gave their personal belongings is belied by the unshaken testimony of respondent Paulina Matillano, corroborated by Erlinda Clarin.

The petitioners attempt to project themselves to have acted with civility and courtesy to respondent Paulina Matillano is implausible, taking into account petitioner Luis state of mind before he and petitioner Rojas and their cohorts left the Metrodiscom Headquarters in Davao City, and proceeded to the house of the respondents in Bansalan. Before they left Davao City, Lui sadistically mauled Lariosa with the acquiescence of the police authorities, and forced him to give an uncounselled extrajudicial confession. This was the finding of the RTC in Criminal Case No. 17,136,88: Despite being mauled by Eli Lui and drowned in a toilet bowl, accused denied having anything to do with the lost money of the complainant. Later, he was turned over to the police for investigation and there without affording accused with his right to counsel, he was interrogated orally and was forced to admit that out of the money he stole, he bought items which the police later recovered at Bansalan. They also returned the accused to the complainants establishment and forced to do re-enactment of the act of robbery, without accused again afforded the right to counsel. Pictures were taken during the re-enactment while accused was handcuffed, as shown in the pictures taken by the police. Finally, the accused was forced to admit and sign his extrajudicial statement (Exhibit A), no longer able to bear the pain of the mauling to him by Eli Lui, who has the temerity of maltreating the accused even in the presence of the guards in the jail and seriously threatening accused to admit ownership of the recovered items at Bansalan and at New Matina, SIR, Davao City, otherwise he will be salvaged, along with the serious threatening words of accuseds companion in the jail, that if he will refuse to sign his alleged confession, he will be salvaged as directed by Eli Lui with the police. Indeed, in the records, it can be deduced with sufficient basis, that Eli Lui seems to have an open hand in the prosecution of accused. He was the one who called the police to arrest him, even without a warrant of arrest. Before his statement was obtained, policeman relied on him in the investigation and the filing of proper charges against accused. They rode in a car of Eli Lui, in taking accused from the Metrodiscom to the establishment of complainant during the re-enactment in going to Bansalan, to recover the items allegedly bought by accused out of the money allegedly stolen; all of these incidents shows (sic) [that] the police despite justification, that they do not have enough facilities (sic), [had] gone astray in conducting an impartial investigation, by submitting to any possible indiscretion of Eli Lui of making the scale of justice bend in his favor, by manifesting control over the police power of investigation highly and seriously prejudicial to the rights, and interests of the accused.28 If petitioner Lui was so brazen as to have mauled Lariosa in the presence of police authorities, he would not have cared a whit in barging into the respondents house with petitioner Rojas, a policeman of Davao City, and his cohorts, and divesting the respondents of their belongings. The petitioners and their cohorts wanted to insure that their caper would succeed. Hence, they did not coordinate with the Bansalan Police Station when they went to the respondents house with their intention to divest them of their belongings.

Petitioner Rojas reliance on Mission Order No. MRF-A-004-98 issued to him by Sergeant Alberto Genise is misplaced. It bears stressing that the petitioner was merely tasked in the said order to "follow up a theft case within the area of responsibility of the Metrodiscom, Davao City." The petitioner was not authorized, under the said order, to commit or tolerate the commission of a crime, such as violation of domicile as defined in Article 128 of the Revised Penal Code, viz: ART. 128. Violation of domicile The penalty of prision correccional in its minimum period shall be imposed upon any public officer or employee who, not being authorized by judicial order, shall enter any dwelling against the will of the owner thereof, search papers or other effects found therein without the previous consent of such owner, or, having surreptitiously entered said dwelling, and being required to leave the premises, shall refuse to do so. If the offense be committed in the nighttime, or if any papers or effects not constituting evidence of a crime be not returned immediately after the search made by the offender, the penalty shall be prision correccional in its medium and maximum periods. Although petitioner Rojas did not follow petitioner Lui and his cohorts to the second floor of the respondents house and himself conduct a search therein, he allowed them to search the premises without a warrant. The petitioners and their cohorts were not authorized to conduct a search in the house of the respondents, much less divest the latter of their personal belongings. As a police officer, it was petitioner Rojas duty to prevent the commission of crimes in his presence, and to arrest the persons committing such crimes. The trial court rejected the testimony of respondent Paulina Matillano on the following grounds: (a) she had known petitioner Lui for ten years as a businessman doing business in Bansalan; (b) the occupants of the respondents house when the petitioners and their cohorts arrived were all women; (c) the respondents failed to report the incident to the Bansalan police authorities; and, (d) the provincial prosecutors resolution recommending the dismissal of Criminal Case No. 880B for robbery against the petitioners, which was sustained by the Secretary of Justice, and the ruling of the National Police Commission exonerating petitioner Rojas from any liability. We find that the Court of Appeals was correct in overruling the trial court. First. Respondent Paulina Matillano testified that petitioner Lui did not stay permanently in Bansalan. He went there only to collect money from a certain Matura and other businessmen.29 She also testified that there were many cases against the petitioner, one of which was for arson. The case was dismissed, but one of her neighbors was rendered missing.30 If the petitioner, a businessman for ten years or so, had no qualms in torturing Lariosa under the very noses of police officers, he would, likewise, have no qualms about intimidating respondent Paulina Matillano and divesting her of her personal belongings. It must be stressed that petitioner Lui was in the company of petitioner Rojas, a police officer from Davao City.

Second. The petitioners and their cohorts had no foreknowledge that the occupants of the respondents house were all women. They must have believed that there were male occupants; hence, barged into the house with drawn guns. Third. As shown clearly in respondent Paulina Matillanos sworn statement before the Bansalan Police Station, she declared that the petitioners were armed with guns. They threatened her life and, without any search warrant therefor, divested her and her family of their personal belongings against their will.31 Fourth. In her complaint before the Office of the Barangay Captain, respondent Paulina Matillano declared that the petitioners entered their house, that petitioner Lui pointed a gun at her, and that the petitioners and their cohorts searched the house and carted away their personal belongings.32 That the report made before the Barangay Captain and petitioner Paulina Matillanos sworn statement are not as complete as her testimony before the trial court is understandable. Affidavits are usually taken ex parte and are almost always incomplete and inaccurate, but they do not detract from the credibility of the witness.33 An entry in the police blotter is usually incomplete and inaccurate for want of suggestions or inquiries, without the aid of which the victim may be unable to recall the connected collateral circumstances necessary for the correction of the first suggestion of his memory, and for his accurate recollection of all that pertain to the subject.34 The same principle applies to entries in the barangay blotter. Fifth. As correctly held by the trial court, the findings of administrative and quasi-administrative agencies are not binding on the courts. In the present case, the Office of the Provincial Prosecutor, as affirmed by the Secretary of Justice,35 found no probable cause for robbery against the petitioners because they had no intent to rob, but merely to recover the properties from the house of the respondents which petitioner Lui perceived to have been acquired by Lariosa with money stolen from his uncle, Ben.36 The decision of the National Police Commission absolving petitioner Rojas of grave misconduct was anchored on its finding that the petitioner was merely performing his duty as ordered by his superior officer.37 It was inevitable for the City Prosecutor to dismiss the complaint for violation of domicile filed against petitioner Rojas in I.S. No. 911488 because the crime of violation of domicile was committed in Bansalan and not in Davao City.38 In contrast, the Commission on Human Rights recommended the indictment of petitioner Lui for unlawful arrest and of petitioner Rojas for violation of domicile.39 Sixth. Under Articles 19 and 32, in relation to Article 21 of the New Civil Code, the dismissal of the complaint against the petitioners by the Provincial and City Prosecutors, the Municipal Trial Court and the National Police Commission are of no relevance to the civil complaint for damages filed by the respondents against the petitioners. The action of the respondents against the petitioners may still proceed despite the dismissal of the criminal and administrative actions against them. The petitioners contention that respondent Paulina Matillano waived her right against unreasonable search and seizure deserves scant consideration. Under Article III, Section 2 of the Constitution, "the right of the people to be secure in their persons, houses, papers and effects against unreasonable searches and seizures of whatever nature and for any purpose shall be inviolable." This provision protects not only those who appear to be innocent but also those who

appear to be guilty, who must nevertheless be presumed innocent until the contrary is proved.40 The general rule is that a search and seizure must be carried through or with judicial warrant; otherwise, such a search and seizure becomes unconstitutional within the context of the constitutional provision41 because a warrantless search is in derogation of a constitutional right. Peace officers who effect a warrantless search cannot invoke regularity in the performance of official functions.42 The right against unreasonable searches and seizures is a personal right which may be waived expressly or impliedly. But a waiver by implication cannot be presumed.43 There must be clear and convincing evidence of an actual intention to relinquish the right to constitute a waiver of a constitutional right. There must be proof of the following: (a) that the right exists; (b) that the person involved had knowledge, either actual or constructive, of the existence of such right; and, (c) that the said person had an actual intention to relinquish the right.44 The waiver must be made voluntarily, knowingly and intelligently. The Court indulges every reasonable presumption against any waiver of fundamental constitutional rights.45 The fact that the aggrieved person did not object to the entry into her house by the police officers does not amount to a permission to make a search therein.46 A peaceful submission to search and seizure is not a consent or an invitation thereto, but is merely a demonstration of regard for the supremacy of the law.47 In this case, the petitioners failed to prove, with clear and convincing evidence, that respondent Paulina Matillano waived her right against unreasonable search and seizure by consenting thereto, either expressly or impliedly. Admittedly, respondent Paulina Matillano did not object to the opening of her wooden closet and the taking of their personal properties. However, such failure to object or resist did not amount to an implied waiver of her right against unreasonable search and seizure. The petitioners were armed with handguns; petitioner Lui threatened and intimidated her. Respondent Eulogio Matillano, her husband, was out of the house when the petitioner and his cohorts conducted the search and seizure. He could, thus, not have waived his constitutional right. Furthermore, the petitioners claim that respondent Paulina Matillano voluntarily handed over the articles to petitioner Lui is incredible. There is no evidence that there was foreknowledge on the part of the petitioners of the articles they wanted to retrieve from the respondents house. Even if respondent Paulina Matillano did hand over the articles to the petitioner, it was only because the petitioner and his cohorts had earlier threatened and intimidated her into doing so. We agree with the ruling of the Court of Appeals that the petitioners are liable to the respondents for moral and exemplary damages in the amounts respectively awarded by it. Petitioner Rojas, a policeman of Davao City, conspired with petitioner Lui and, with drawn guns, gained entry into the respondents house, and threatened and intimidated respondent Paulina Matillano. Although petitioner Rojas did not himself conduct the search, he assented thereto by allowing petitioner Lui and his cohorts to go up to the second floor and divest the respondents of their belongings. The petitioners even left together after the incident. In MHP Garments, Inc. vs. Court of Appeals,48 we had the occasion to state:

In the case of Lim vs. Ponce de Leon, we ruled for the recovery of damages for violation of constitutional rights and liberties from public officer or private individual, thus: "ART. 32. Any public officer or employee, or any private individual, who directly or indirectly obstructs, defeats, violates or in any manner impedes or impairs any of the following rights and liberties of another person shall be liable to the latter for damages. "x x x "(9) the rights to be secure in ones persons, house, papers and effects against unreasonable searches and seizures. "x x x "The indemnity shall include moral damages. Exemplary damages may also be adjudged." "ART 2219. Moral damages may be recovered in the following and analogous cases: "x x x "(6) Illegal search; "(1) Acts and actions referred to in Articles 21, 26, 27, 28, 29, 30, 32, 34, and 35. "Pursuant to the foregoing provisions, a person whose constitutional rights have been violated or impaired is entitled to actual and moral damages from the public officer or employee responsible therefor. In addition, exemplary damages may also be awarded." xxx "The very nature of Article 32 is that the wrong may be civil or criminal. It is not necessary therefore that there should be malice or bad faith. To make such a requisite would defeat the main purpose of Article 32 which is the effective protection of individual rights. Public officials in the past have abused their powers on the pretext of justifiable motives or good faith in the performance of their duties. Precisely, the object of the Article is to put an end to official abuse by plea of the good faith. In the United States this remedy is in the nature of a tort." (emphasis supplied) In the subsequent case of Aberca vs. Ver, the Court En Banc explained the liability of persons indirectly responsible, viz: "[T]he decisive factor in this case, in our view, is the language of Article 32. The law speaks of an officer or employee or person directly or indirectly responsible for the violation of the constitutional rights and liberties of another. Thus, it is not the actor alone (i.e., the one directly responsible) who must answer for damages under Article 32; the

person indirectly responsible has also to answer for the damages or injury caused to the aggrieved party. xxx "While it would certainly be too nave to expect that violators of human rights would easily be deterred by the prospect of facing damage suits, it should nonetheless be made clear in no uncertain terms that Article 32 of the Civil Code makes the persons who are directly, as well as indirectly, responsible for the transgression joint tortfeasors. xxx [N]either can it be said that only those shown to have participated directly should be held liable. Article 32 of the Civil Code encompasses within the ambit of its provisions those directly, as well as indirectly, responsible for its violations." (emphasis supplied) Applying the aforecited provisions and leading cases, the respondent court correctly granted damages to private respondents. Petitioners were indirectly involved in transgressing the right of private respondents against unreasonable search and seizure. Firstly, they instigated the raid pursuant to their covenant in the Memorandum Agreement to undertake the prosecution in court of all illegal sources of scouting supplies. As correctly observed by respondent court: "Indeed, the acts committed by the PC soldiers of unlawfully seizing appellees (respondents) merchandise and of filing the criminal complaint for unfair competition against appellees (respondents) were for the protection and benefit of appellant (petitioner) corporation. Such being the case, it is, thus, reasonably fair to infer from those acts that it was upon appellant (petitioner) corporations instance that the PC soldiers conducted the raid and effected the illegal seizure. These circumstances should answer the trial courts query posed in its decision now under consideration as to why the PC soldiers immediately turned over the seized merchandise to appellant (petitioner) corporation." The raid was conducted with the active participation of their employee. Larry de Guzman did not lift a finger to stop the seizure of the boy and girl scout items. By standing by and apparently assenting thereto, he was liable to the same extent as the officers themselves. So with the petitioner corporation which even received for safekeeping the goods unreasonable seized by the PC raiding team and de Guzman, and refused to surrender them for quite a time despite the dismissal of its complaint for unfair competition.49 IN LIGHT OF ALL THE FOREGOING, the petition is DISMISSED. The Decision of the Court of Appeals is AFFIRMED in toto. Costs against the petitioners. SO ORDERED. G.R. No. 182677 August 3, 2010

JOSE ANTONIO C. LEVISTE, Petitioner, vs. HON. ELMO M. ALAMEDA, HON. RAUL M. GONZALEZ, HON. EMMANUEL Y. VELASCO, HEIRS OF THE LATE RAFAEL DE LAS ALAS, Respondents. DECISION CARPIO MORALES, J.: Jose Antonio C. Leviste (petitioner) assails via the present petition for review filed on May 30, 2008 the August 30, 2007 Decision1 and the April 18, 2008 Resolution2 of the Court of Appeals in CA-G.R. SP No. 97761 that affirmed the trial courts Orders of January 24, 31, February 7, 8, all in 2007, and denied the motion for reconsideration, respectively. Petitioner was, by Information3 of January 16, 2007, charged with homicide for the death of Rafael de las Alas on January 12, 2007 before the Regional Trial Court (RTC) of Makati City. Branch 150 to which the case was raffled, presided by Judge Elmo Alameda, forthwith issued a commitment order4 against petitioner who was placed under police custody while confined at the Makati Medical Center.5 After petitioner posted a P40,000 cash bond which the trial court approved,6 he was released from detention, and his arraignment was set on January 24, 2007. The private complainants-heirs of De las Alas filed, with the conformity of the public prosecutor, an Urgent Omnibus Motion7 praying, inter alia, for the deferment of the proceedings to allow the public prosecutor to re-examine the evidence on record or to conduct a reinvestigation to determine the proper offense. The RTC thereafter issued the (1) Order of January 24, 20078 deferring petitioners arraignment and allowing the prosecution to conduct a reinvestigation to determine the proper offense and submit a recommendation within 30 days from its inception, inter alia; and (2) Order of January 31, 20079 denying reconsideration of the first order. Petitioner assailed these orders via certiorari and prohibition before the Court of Appeals. Meantime, petitioner filed an Urgent Ex-Parte Manifestation and Motion before the trial court to defer acting on the public prosecutors recommendation on the proper offense until after the appellate court resolves his application for injunctive reliefs, or alternatively, to grant him time to comment on the prosecutors recommendation and thereafter set a hearing for the judicial determination of probable cause.10 Petitioner also separately moved for the inhibition of Judge Alameda with prayer to defer action on the admission of the Amended Information.11 The trial court nonetheless issued the other assailed orders, viz: (1) Order of February 7, 200712 that admitted the Amended Information13 for murder and directed the issuance of a warrant of arrest; and (2) Order of February 8, 200714 which set the arraignment on February 13, 2007. Petitioner questioned these two orders via supplemental petition before the appellate court.

The appellate court dismissed petitioners petition, hence, his present petition, arguing that: PRIVATE RESPONDENT DID NOT HAVE THE RIGHT TO CAUSE THE REINVESTIGATION OF THE CRIMINAL CASE BELOW WHEN THE CRIMINAL INFORMATION HAD ALREADY BEEN FILED WITH THE LOWER COURT. HENCE, THE COURT OF APPEALS COMMITTED A GRAVE ERROR IN FINDING THAT RESPONDENT JUDGE DID NOT ACT WITH GRAVE ABUSE OF DISCRETION IN GRANTING SUCH REINVESTIGATION DESPITE HAVING NO BASIS IN THE RULES OF COURT[;] RESPONDENT JUDGE ACTED WITH GRAVE ABUSE OF DISCRETION IN ADMITTING STATE PROSECUTOR VELASCOS AMENDED INFORMATION, ISSUING A WARRANT OF ARREST, AND SETTING THE CASE BELOW FOR ARRAIGNMENT, CONSIDERING THAT THE VALIDITY AND LEGALITY OF HIS ORDERS DATED 24 AND 31 JANUARY 2007, WHICH LED TO THE QUESTIONABLE REINVESTIGATION AND ILLEGAL AMENDED INFORMATION[,] ARE YET TO BE RESOLVED BY THIS HONORABLE COURT (sic); [AND] CONSIDERING THAT PROSECUTOR VELASCOS FINDINGS IN HIS RESOLUTION DATED 2 FEBRUARY 2007 ARE BLATANTLY BASED ON MERE SPECULATIONS AND CONJECTURES, WITHOUT ANY SUBSTANTIAL OR MATERIAL NEW EVIDENCE BEING ADDUCED DURING THE REINVESTIGATION, RESPONDENT JUDGE SHOULD HAVE AT LEAST ALLOWED PETITIONERS MOTION FOR A HEARING FOR JUDICIAL DETERMINATION OF PROBABLE CAUSE.15 (emphasis in the original omitted) Records show that the arraignment scheduled on March 21, 2007 pushed through during which petitioner refused to plead, drawing the trial court to enter a plea of "not guilty" for him. Prior thereto or on February 23, 2007, petitioner filed an Urgent Application for Admission to Bail Ex Abundanti Cautela16 which the trial court, after hearings thereon, granted by Order of May 21, 2007,17 it finding that the evidence of guilt for the crime of murder is not strong. It accordingly allowed petitioner to post bail in the amount of P300,000 for his provisional liberty. The trial court, absent any writ of preliminary injunction from the appellate court, went on to try petitioner under the Amended Information. By Decision of January 14, 2009, the trial court found petitioner guilty of homicide, sentencing him to suffer an indeterminate penalty of six years and one day of prision mayor as minimum to 12 years and one day of reclusion temporal as maximum. From the Decision, petitioner filed an appeal to the appellate court, docketed as CAG.R. CR No. 32159, during the pendency of which he filed an urgent application for admission to bail pending appeal. The appellate court denied petitioners application which this Court, in G.R. No. 189122, affirmed by Decision of March 17, 2010. The Office of the Solicitor General (OSG) later argued that the present petition had been rendered moot since the presentation of evidence, wherein petitioner actively participated, had been concluded.18

Waiver on the part of the accused must be distinguished from mootness of the petition, for in the present case, petitioner did not, by his active participation in the trial, waive his stated objections. Section 26, Rule 114 of the Rules of Court provides: SEC. 26. Bail not a bar to objections on illegal arrest, lack of or irregular preliminary investigation. An application for or admission to bail shall not bar the accused from challenging the validity of his arrest or the legality of the warrant issued therefor, or from assailing the regularity or questioning the absence of a preliminary investigation of the charge against him, provided that he raises them before entering his plea. The court shall resolve the matter as early as practicable but not later than the start of the trial of the case. By applying for bail, petitioner did not waive his right to challenge the regularity of the reinvestigation of the charge against him, the validity of the admission of the Amended Information, and the legality of his arrest under the Amended Information, as he vigorously raised them prior to his arraignment. During the arraignment on March 21, 2007, petitioner refused to enter his plea since the issues he raised were still pending resolution by the appellate court, thus prompting the trial court to enter a plea of "not guilty" for him. The principle that the accused is precluded after arraignment from questioning the illegal arrest or the lack of or irregular preliminary investigation applies "only if he voluntarily enters his plea and participates during trial, without previously invoking his objections thereto."19 There must be clear and convincing proof that petitioner had an actual intention to relinquish his right to question the existence of probable cause. When the only proof of intention rests on what a party does, his act should be so manifestly consistent with, and indicative of, an intent to voluntarily and unequivocally relinquish the particular right that no other explanation of his conduct is possible.20 From the given circumstances, the Court cannot reasonably infer a valid waiver on the part of petitioner to preclude him from obtaining a definite resolution of the objections he so timely invoked. Other than its allegation of active participation, the OSG offered no clear and convincing proof that petitioners participation in the trial was unconditional with the intent to voluntarily and unequivocally abandon his petition. In fact, on January 26, 2010, petitioner still moved for the early resolution of the present petition.21 Whatever delay arising from petitioners availment of remedies against the trial courts Orders cannot be imputed to petitioner to operate as a valid waiver on his part. Neither can the nonissuance of a writ of preliminary injunction be deemed as a voluntary relinquishment of petitioners principal prayer. The non-issuance of such injunctive relief only means that the appellate court did not preliminarily find any exception22 to the long-standing doctrine that injunction will not lie to enjoin a criminal prosecution.23 Consequently, the trial of the case took its course. The petition is now moot, however, in view of the trial courts rendition of judgment.

A moot and academic case is one that ceases to present a justiciable controversy by virtue of supervening events, so that a declaration thereon would be of no practical use or value.24 The judgment convicting petitioner of homicide under the Amended Information for murder operates as a supervening event that mooted the present petition. Assuming that there is ground25 to annul the finding of probable cause for murder, there is no practical use or value in abrogating the concluded proceedings and retrying the case under the original Information for homicide just to arrive, more likely or even definitely, at the same conviction of homicide. Mootness would have also set in had petitioner been convicted of murder, for proof beyond reasonable doubt, which is much higher than probable cause, would have been established in that instance. Instead, however, of denying the petition outright on the ground of mootness, the Court proceeds to resolve the legal issues in order to formulate controlling principles to guide the bench, bar and public.26 In the present case, there is compelling reason to clarify the remedies available before and after the filing of an information in cases subject of inquest. After going over into the substance of the petition and the assailed issuances, the Court finds no reversible error on the part of the appellate court in finding no grave abuse of discretion in the issuance of the four trial court Orders. In his first assignment of error, petitioner posits that the prosecution has no right under the Rules to seek from the trial court an investigation or reevaluation of the case except through a petition for review before the Department of Justice (DOJ). In cases when an accused is arrested without a warrant, petitioner contends that the remedy of preliminary investigation belongs only to the accused. The contention lacks merit. Section 6,27 Rule 112 of the Rules of Court reads: When a person is lawfully arrested without a warrant involving an offense which requires a preliminary investigation, the complaint or information may be filed by a prosecutor without need of such investigation provided an inquest has been conducted in accordance with existing rules. In the absence or unavailability of an inquest prosecutor, the complaint may be filed by the offended party or a peace officer directly with the proper court on the basis of the affidavit of the offended party or arresting officer or person. Before the complaint or information is filed, the person arrested may ask for a preliminary investigation in accordance with this Rule, but he must sign a waiver of the provisions of Article 125 of the Revised Penal Code, as amended, in the presence of his counsel. Notwithstanding the waiver, he may apply for bail and the investigation must be terminated within fifteen (15) days from its inception. After the filing of the complaint or information in court without a preliminary investigation, the accused may, within five (5) days from the time he learns of its filing, ask for a preliminary

investigation with the same right to adduce evidence in his defense as provided in this Rule. (underscoring supplied) A preliminary investigation is required before the filing of a complaint or information for an offense where the penalty prescribed by law is at least four years, two months and one day without regard to fine.28 As an exception, the rules provide that there is no need for a preliminary investigation in cases of a lawful arrest without a warrant29 involving such type of offense, so long as an inquest, where available, has been conducted.30 Inquest is defined as an informal and summary investigation conducted by a public prosecutor in criminal cases involving persons arrested and detained without the benefit of a warrant of arrest issued by the court for the purpose of determining whether said persons should remain under custody and correspondingly be charged in court.31 It is imperative to first take a closer look at the predicament of both the arrested person and the private complainant during the brief period of inquest, to grasp the respective remedies available to them before and after the filing of a complaint or information in court. BEFORE THE FILING OF COMPLAINT OR INFORMATION IN COURT, the private complainant may proceed in coordinating with the arresting officer and the inquest officer during the latters conduct of inquest. Meanwhile, the arrested person has the option to avail of a 15-day preliminary investigation, provided he duly signs a waiver of any objection against delay in his delivery to the proper judicial authorities under Article 125 of the Revised Penal Code. For obvious reasons, this remedy is not available to the private complainant since he cannot waive what he does not have. The benefit of the provisions of Article 125, which requires the filing of a complaint or information with the proper judicial authorities within the applicable period,32 belongs to the arrested person. The accelerated process of inquest, owing to its summary nature and the attendant risk of running against Article 125, ends with either the prompt filing of an information in court or the immediate release of the arrested person.33 Notably, the rules on inquest do not provide for a motion for reconsideration.34 Contrary to petitioners position that private complainant should have appealed to the DOJ Secretary, such remedy is not immediately available in cases subject of inquest. Noteworthy is the proviso that the appeal to the DOJ Secretary is by "petition by a proper party under such rules as the Department of Justice may prescribe."35 The rule referred to is the 2000 National Prosecution Service Rule on Appeal,36 Section 1 of which provides that the Rule shall "apply to appeals from resolutions x x x in cases subject of preliminary investigation/ reinvestigation." In cases subject of inquest, therefore, the private party should first avail of a preliminary investigation or reinvestigation, if any, before elevating the matter to the DOJ Secretary. In case the inquest proceedings yield no probable cause, the private complainant may pursue the case through the regular course of a preliminary investigation.

ONCE A COMPLAINT OR INFORMATION IS FILED IN COURT, the rules yet provide the accused with another opportunity to ask for a preliminary investigation within five days from the time he learns of its filing. The Rules of Court and the New Rules on Inquest are silent, however, on whether the private complainant could invoke, as respondent heirs of the victim did in the present case, a similar right to ask for a reinvestigation. The Court holds that the private complainant can move for reinvestigation, subject to and in light of the ensuing disquisition. All criminal actions commenced by a complaint or information shall be prosecuted under the direction and control of the public prosecutor.37 The private complainant in a criminal case is merely a witness and not a party to the case and cannot, by himself, ask for the reinvestigation of the case after the information had been filed in court, the proper party for that being the public prosecutor who has the control of the prosecution of the case.38 Thus, in cases where the private complainant is allowed to intervene by counsel in the criminal action,39 and is granted the authority to prosecute,40 the private complainant, by counsel and with the conformity of the public prosecutor, can file a motion for reinvestigation. In fact, the DOJ instructs that before the arraignment of the accused, trial prosecutors must "examine the Information vis--vis the resolution of the investigating prosecutor in order to make the necessary corrections or revisions and to ensure that the information is sufficient in form and substance."41 x x x Since no evidence has been presented at that stage, the error would appear or be discoverable from a review of the records of the preliminary investigation. Of course, that fact may be perceived by the trial judge himself but, again, realistically it will be the prosecutor who can initially determine the same. That is why such error need not be manifest or evident, nor is it required that such nuances as offenses includible in the offense charged be taken into account. It necessarily follows, therefore, that the prosecutor can and should institute remedial measures[.]42 (emphasis and underscoring supplied) The prosecution of crimes appertains to the executive department of the government whose principal power and responsibility is to see that our laws are faithfully executed. A necessary component of this power to execute our laws is the right to prosecute their violators. The right to prosecute vests the prosecutor with a wide range of discretion the discretion of what and whom to charge, the exercise of which depends on a smorgasbord of factors which are best appreciated by prosecutors.43 The prosecutions discretion is not boundless or infinite, however.44 The standing principle is that once an information is filed in court, any remedial measure such as a reinvestigation must be addressed to the sound discretion of the court. Interestingly, petitioner supports this view.45 Indeed, the Court ruled in one case that: The rule is now well settled that once a complaint or information is filed in court, any disposition of the case, whether as to its dismissal or the conviction or the acquittal of the accused, rests in the sound discretion of the court. Although the prosecutor retains the direction and control of the

prosecution of criminal cases even when the case is already in court, he cannot impose his opinion upon the tribunal. For while it is true that the prosecutor has the quasi-judicial discretion to determine whether or not a criminal case should be filed in court, once the case had already been brought therein any disposition the prosecutor may deem proper thereafter should be addressed to the court for its consideration and approval. The only qualification is that the action of the court must not impair the substantial rights of the accused or the right of the People to due process of law. xxxx In such an instance, before a re-investigation of the case may be conducted by the public prosecutor, the permission or consent of the court must be secured. If after such re-investigation the prosecution finds a cogent basis to withdraw the information or otherwise cause the dismissal of the case, such proposed course of action may be taken but shall likewise be addressed to the sound discretion of the court.46 (underscoring supplied) While Abugotal v. Judge Tiro47 held that to ferret out the truth, a trial is to be preferred to a reinvestigation, the Court therein recognized that a trial court may, where the interest of justice so requires, grant a motion for reinvestigation of a criminal case pending before it. Once the trial court grants the prosecutions motion for reinvestigation, the former is deemed to have deferred to the authority of the prosecutorial arm of the Government. Having brought the case back to the drawing board, the prosecution is thus equipped with discretion wide and far reaching regarding the disposition thereof,48 subject to the trial courts approval of the resulting proposed course of action. Since a reinvestigation may entail a modification of the criminal information as what happened in the present case, the Courts holding is bolstered by the rule on amendment of an information under Section 14, Rule 110 of the Rules of Court: A complaint or information may be amended, in form or in substance, without leave of court, at any time before the accused enters his plea. After the plea and during the trial, a formal amendment may only be made with leave of court and when it can be done without causing prejudice to the rights of the accused. However, any amendment before plea, which downgrades the nature of the offense charged in or excludes any accused from the complaint or information, can be made only upon motion by the prosecutor, with notice to the offended party and with leave of court. The court shall state its reasons in resolving the motion and copies of its order shall be furnished all parties, especially the offended party. If it appears at any time before judgment that a mistake has been made in charging the proper offense, the court shall dismiss the original complaint or information upon the filing of a new one charging the proper offense in accordance with section 11, Rule 119, provided the accused

would not be placed in double jeopardy. The court may require the witnesses to give bail for their appearance at the trial. (emphasis supplied) In fine, before the accused enters a plea, a formal or substantial amendment of the complaint or information may be made without leave of court.49 After the entry of a plea, only a formal amendment may be made but with leave of court and only if it does not prejudice the rights of the accused. After arraignment, a substantial amendment is proscribed except if the same is beneficial to the accused.50 It must be clarified though that not all defects in an information are curable by amendment prior to entry of plea. An information which is void ab initio cannot be amended to obviate a ground for quashal.51 An amendment which operates to vest jurisdiction upon the trial court is likewise impermissible.52 Considering the general rule that an information may be amended even in substance and even without leave of court at any time before entry of plea, does it mean that the conduct of a reinvestigation at that stage is a mere superfluity? It is not. Any remedial measure springing from the reinvestigation be it a complete disposition or an intermediate modification53 of the charge is eventually addressed to the sound discretion of the trial court, which must make an independent evaluation or assessment of the merits of the case. Since the trial court would ultimately make the determination on the proposed course of action, it is for the prosecution to consider whether a reinvestigation is necessary to adduce and review the evidence for purposes of buttressing the appropriate motion to be filed in court. More importantly, reinvestigation is required in cases involving a substantial amendment of the information. Due process of law demands that no substantial amendment of an information may be admitted without conducting another or a new preliminary investigation. In Matalam v. The 2nd Division of the Sandiganbayan,54 the Court ruled that a substantial amendment in an information entitles an accused to another preliminary investigation, unless the amended information contains a charge related to or is included in the original Information. The question to be resolved is whether the amendment of the Information from homicide to murder is considered a substantial amendment, which would make it not just a right but a duty of the prosecution to ask for a preliminary investigation. The Court answers in the affirmative. A substantial amendment consists of the recital of facts constituting the offense charged and determinative of the jurisdiction of the court. All other matters are merely of form. The following have been held to be mere formal amendments: (1) new allegations which relate only to the range of the penalty that the court might impose in the event of conviction; (2) an amendment which does not charge another offense different or distinct from that charged in the original one; (3) additional allegations which do not alter the prosecutions theory of the case so

as to cause surprise to the accused and affect the form of defense he has or will assume; (4) an amendment which does not adversely affect any substantial right of the accused; and (5) an amendment that merely adds specifications to eliminate vagueness in the information and not to introduce new and material facts, and merely states with additional precision something which is already contained in the original information and which adds nothing essential for conviction for the crime charged. The test as to whether a defendant is prejudiced by the amendment is whether a defense under the information as it originally stood would be available after the amendment is made, and whether any evidence defendant might have would be equally applicable to the information in the one form as in the other. An amendment to an information which does not change the nature of the crime alleged therein does not affect the essence of the offense or cause surprise or deprive the accused of an opportunity to meet the new averment had each been held to be one of form and not of substance.55 (emphasis and underscoring supplied) Matalam adds that the mere fact that the two charges are related does not necessarily or automatically deprive the accused of his right to another preliminary investigation. Notatu dignum is the fact that both the original Information and the amended Information in Matalam were similarly charging the accused with violation of Section 3(e) of the Anti-Graft and Corrupt Practices Act. In one case,56 it was squarely held that the amendment of the Information from homicide to murder is "one of substance with very serious consequences."57 The amendment involved in the present case consists of additional averments of the circumstances of treachery, evident premeditation, and cruelty, which qualify the offense charged from homicide to murder. It being a new and material element of the offense, petitioner should be given the chance to adduce evidence on the matter. Not being merely clarificatory, the amendment essentially varies the prosecutions original theory of the case and certainly affects not just the form but the weight of defense to be mustered by petitioner. The Court distinguishes the factual milieus in Buhat v. CA58 and Pacoy v. Cajigal,59 wherein the amendment of the caption of the Information from homicide to murder was not considered substantial because there was no real change in the recital of facts constituting the offense charged as alleged in the body of the Information, as the allegations of qualifying circumstances were already clearly embedded in the original Information. Buhat pointed out that the original Information for homicide already alleged the use of superior strength, while Pacoy states that the averments in the amended Information for murder are exactly the same as those already alleged in the original Information for homicide. None of these peculiar circumstances obtains in the present case. Considering that another or a new preliminary investigation is required, the fact that what was conducted in the present case was a reinvestigation does not invalidate the substantial amendment of the Information. There is no substantial distinction between a preliminary investigation and a reinvestigation since both are conducted in the same manner and for the same objective of determining whether there exists sufficient ground to engender a well-founded belief that a crime has been committed and the respondent is probably guilty thereof and should

be held for trial.60 What is essential is that petitioner was placed on guard to defend himself from the charge of murder61 after the claimed circumstances were made known to him as early as the first motion. Petitioner did not, however, make much of the opportunity to present countervailing evidence on the proposed amended charge. Despite notice of hearing, petitioner opted to merely observe the proceedings and declined to actively participate, even with extreme caution, in the reinvestigation. Mercado v. Court of Appeals states that the rules do not even require, as a condition sine qua non to the validity of a preliminary investigation, the presence of the respondent as long as efforts to reach him were made and an opportunity to controvert the complainants evidence was accorded him.62 In his second assignment of error, petitioner basically assails the hurried issuance of the last two assailed RTC Orders despite the pendency before the appellate court of the petition for certiorari challenging the first two trial court Orders allowing a reinvestigation. The Rules categorically state that the petition shall not interrupt the course of the principal case unless a temporary retraining order or a writ of preliminary injunction has been issued.63 The appellate court, by Resolution of February 15, 2007,64 denied petitioners application for a temporary restraining order and writ of preliminary injunction. Supplementary efforts to seek injunctive reliefs proved futile.65 The appellate court thus did not err in finding no grave abuse of discretion on the part of the trial court when it proceeded with the case and eventually arraigned the accused on March 21, 2007, there being no injunction order from the appellate court. Moreover, petitioner opted to forego appealing to the DOJ Secretary, a post-inquest remedy that was available after the reinvestigation and which could have suspended the arraignment.661avvphi1 Regarding petitioners protestations of haste, suffice to state that the pace in resolving incidents of the case is not per se an indication of bias. In Santos-Concio v. Department of Justice,67 the Court held: Speed in the conduct of proceedings by a judicial or quasi-judicial officer cannot per se be instantly attributed to an injudicious performance of functions. For ones prompt dispatch may be anothers undue haste. The orderly administration of justice remains as the paramount and constant consideration, with particular regard of the circumstances peculiar to each case. The presumption of regularity includes the public officers official actuations in all phases of work. Consistent with such presumption, it was incumbent upon petitioners to present contradictory evidence other than a mere tallying of days or numerical calculation. This, petitioners failed to discharge. The swift completion of the Investigating Panels initial task cannot be relegated as shoddy or shady without discounting the presumably regular performance of not just one but five state prosecutors.68 There is no ground for petitioners protestations against the DOJ Secretarys sudden designation of Senior State Prosecutor Emmanuel Velasco as Acting City Prosecutor of Makati City for the present case69 and the latters conformity to the motion for reinvestigation.

In granting the reinvestigation, Judge Alameda cannot choose the public prosecutor who will conduct the reinvestigation or preliminary investigation.70 There is a hierarchy of officials in the prosecutory arm of the executive branch headed by the Secretary of Justice71 who is vested with the prerogative to appoint a special prosecutor or designate an acting prosecutor to handle a particular case, which broad power of control has been recognized by jurisprudence.72 As for the trial courts ignoring the DOJ Secretarys uncontested statements to the media which aired his opinion that if the assailant merely intended to maim and not to kill the victim, one bullet would have sufficed the DOJ Secretary reportedly uttered that "the filing of the case of homicide against ano against Leviste lintek naman eh I told you to watch over that case there should be a report about the ballistics, about the paraffin, etc., then thats not a complete investigation, thats why you should use that as a ground" no abuse of discretion, much less a grave one, can be imputed to it. The statements of the DOJ Secretary do not evince a "determination to file the Information even in the absence of probable cause."73 On the contrary, the remarks merely underscored the importance of securing basic investigative reports to support a finding of probable cause. The original Resolution even recognized that probable cause for the crime of murder cannot be determined based on the evidence obtained "[u]nless and until a more thorough investigation is conducted and eyewitness/es [is/]are presented in evidence[.]"74 The trial court concluded that "the wound sustained by the victim at the back of his head, the absence of paraffin test and ballistic examination, and the handling of physical evidence,"75 as rationalized by the prosecution in its motion, are sufficient circumstances that require further inquiry. That the evidence of guilt was not strong as subsequently assessed in the bail hearings does not affect the prior determination of probable cause because, as the appellate court correctly stated, the standard of strong evidence of guilt which is sufficient to deny bail to an accused is markedly higher than the standard of judicial probable cause which is sufficient to initiate a criminal case.76 In his third assignment of error, petitioner faults the trial court for not conducting, at the very least, a hearing for judicial determination of probable cause, considering the lack of substantial or material new evidence adduced during the reinvestigation. Petitioners argument is specious. There are two kinds of determination of probable cause: executive and judicial. The executive determination of probable cause is one made during preliminary investigation. It is a function that properly pertains to the public prosecutor who is given a broad discretion to determine whether probable cause exists and to charge those whom he believes to have committed the crime as defined by law and thus should be held for trial. Otherwise stated, such official has the quasi-judicial authority to determine whether or not a criminal case must be filed in court. Whether that function has been correctly discharged by the public prosecutor, i.e., whether he

has made a correct ascertainment of the existence of probable cause in a case, is a matter that the trial court itself does not and may not be compelled to pass upon.77 The judicial determination of probable cause is one made by the judge to ascertain whether a warrant of arrest should be issued against the accused. The judge must satisfy himself that based on the evidence submitted, there is necessity for placing the accused under custody in order not to frustrate the ends of justice. If the judge finds no probable cause, the judge cannot be forced to issue the arrest warrant.78 Paragraph (a), Section 5,79 Rule 112 of the Rules of Court outlines the procedure to be followed by the RTC. To move the court to conduct a judicial determination of probable cause is a mere superfluity, for with or without such motion, the judge is duty-bound to personally evaluate the resolution of the public prosecutor and the supporting evidence. In fact, the task of the presiding judge when the Information is filed with the court is first and foremost to determine the existence or nonexistence of probable cause for the arrest of the accused.80 What the Constitution underscores is the exclusive and personal responsibility of the issuing judge to satisfy himself of the existence of probable cause. But the judge is not required to personally examine the complainant and his witnesses. Following established doctrine and procedure, he shall (1) personally evaluate the report and the supporting documents submitted by the prosecutor regarding the existence of probable cause, and on the basis thereof, he may already make a personal determination of the existence of probable cause; and (2) if he is not satisfied that probable cause exists, he may disregard the prosecutors report and require the submission of supporting affidavits of witnesses to aid him in arriving at a conclusion as to the existence of probable cause.81 (emphasis and underscoring supplied) The rules do not require cases to be set for hearing to determine probable cause for the issuance of a warrant of arrest of the accused before any warrant may be issued.82 Petitioner thus cannot, as a matter of right, insist on a hearing for judicial determination of probable cause. Certainly, petitioner "cannot determine beforehand how cursory or exhaustive the [judge's] examination of the records should be [since t]he extent of the judges examination depends on the exercise of his sound discretion as the circumstances of the case require."83 In one case, the Court emphatically stated: The periods provided in the Revised Rules of Criminal Procedure are mandatory, and as such, the judge must determine the presence or absence of probable cause within such periods. The Sandiganbayans determination of probable cause is made ex parte and is summary in nature, not adversarial. The Judge should not be stymied and distracted from his determination of probable cause by needless motions for determination of probable cause filed by the accused.84 (emphasis and underscoring supplied) Petitioner proceeds to discuss at length evidentiary matters, arguing that no circumstances exist that would qualify the crime from homicide to murder. The allegation of lack of substantial or material new evidence deserves no credence, because new pieces of evidence are not prerequisites for a valid conduct of reinvestigation. It is not

material that no new matter or evidence was presented during the reinvestigation of the case. It should be stressed that reinvestigation, as the word itself implies, is merely a repeat investigation of the case. New matters or evidence are not prerequisites for a reinvestigation, which is simply a chance for the prosecutor to review and re-evaluate its findings and the evidence already submitted.85 Moreover, under Rule 45 of the Rules of Court, only questions of law may be raised in, and be subject of, a petition for review on certiorari since this Court is not a trier of facts. The Court cannot thus review the evidence adduced by the parties on the issue of the absence or presence of probable cause, as there exists no exceptional circumstances to warrant a factual review.86 In a petition for certiorari, like that filed by petitioner before the appellate court, the jurisdiction of the court is narrow in scope. It is limited to resolving only errors of jurisdiction.1avvphi1 It is not to stray at will and resolve questions and issues beyond its competence, such as an error of judgment.87 The courts duty in the pertinent case is confined to determining whether the executive and judicial determination of probable cause was done without or in excess of jurisdiction or with grave abuse of discretion. Although it is possible that error may be committed in the discharge of lawful functions, this does not render the act amenable to correction and annulment by the extraordinary remedy of certiorari, absent any showing of grave abuse of discretion amounting to excess of jurisdiction.88 WHEREFORE, the petition is DENIED. The assailed Decision and Resolution of the Court of Appeals in CA-G.R. SP No. 97761 are AFFIRMED. SO ORDERED. G.R. No. 113930 March 5, 1996 PAUL G. ROBERTS, JR., RODOLFO C. SALAZAR, LUIS LORENZO, SR., LUIS LORENZO, JR., AMAURY R. GUTIERREZ, BAYANI N. FABIC, JOSE YULO, JR., ESTEBAN B. PALANNUAYAN, and WONG FONG FUI, petitioners, vs. THE COURT OF APPEALS, THE HON. MAXIMIANO ASUNCION, in his capacity as the Presiding Judge of the Regional Trial Court, Quezon City, Branch 104, HON. APOLINARIO G. EXEVEA, HON. HENRICK F. GINGOYON, and HON. PHILIP A. AGUINALDO, in their capacities as Members of the Department of Justice "349" Committee, and the CITY PROSECUTOR OF QUEZON CITY, respondents. J. ROBERT DELGADO, petitioner-Intervenor.

DAVIDE, JR., J.:p We are urged in this petition to set aside (a) the decision of the Court of Appeals of 28 September 1993 in CA-G.R. SP No. 31226, 1 which dismissed the petition therein on the

ground that it has been "mooted with the release by the Department of Justice of its decision . . . dismissing petitioners' petition for review"; (b) the resolution of the said court of 9 February 1994 2 denying the petitioners' motion to reconsider the decision; (c) the order of 17 May 1993 3 of respondent Judge Maximiano C. Asuncion of Branch 104 of the Regional Trial Court (RTC) of Quezon City in Criminal Case No. Q-93-43198 denying petitioners' motion to suspend proceedings and to hold in abeyance the issuance of the warrants of arrest and the public prosecutor's motion to defer arraignment; and (d) the resolutions of 23 July 1993 and 3 February 1994 4 of the Department of Justice (DOJ) dismissing petitioners' petition for the review of the Joint Resolution of the Assistant City Prosecutor of Quezon City and denying the motion to reconsider the dismissal, respectively. The petitioners rely on the following grounds for the grant of the reliefs prayed for in this petition:
I Respondent Judge acted with grave abuse of discretion when he ordered the arrest of the petitioners without examining the record of the preliminary investigation and in determining for himself on the basis thereof the existence of probable cause. II The Department of Justice "349" Committee acted with grave abuse of discretion when it refused to review the City Prosecutor's Joint Resolution and dismissed petitioner's appeal therefrom. III The Court of Appeals acted with grave abuse of discretion when it upheld the subject order directing the issuance of the warrants of arrest without assessing for itself whether based on such records there is probable cause against petitioners. IV The facts on record do not establish prima facie probable cause and Criminal Case No. 5 Q-93-43198 should have been dismissed.

The antecedents of this petition are not disputed. Several thousand holders 6 of "349" Pepsi crowns in connection with the Pepsi Cola Products Phils., Inc.'s (PEPSI's) Number Fever Promotion 7 filed with the Office of the City Prosecutor of Quezon City complaints against the petitioner's in their respective capacities as Presidents or Chief Executive Officers, Chairman of the Board, ViceChairman of the Board, and Directors of PEPSI, and also against other officials of PEPSI. The complaints respectively accuse the petitioners and the other PEPSI officials of the following crimes: (a) estafa; (b) violation of R.A. No. 7394, otherwise known as the Consumer Act of the Philippines; (c) violation of E.O. No. 913; 8 and (d) violation of

Act No. 2333, entitled "An Act Relative to Untrue, Deceptive and Misleading Advertisements," as amended by Act No. 3740. 9 After appropriate proceedings, the investigating prosecutor, Ramon M. Gerona, released on 23 March 1993 a Joint Resolution 10 where he recommended the filing of an information against the petitioners and others for the violation of Article 318 of the Revised Penal Code and the dismissal of the complaints for the violation of Article 315, 2(d) of the Revised Penal Code; R.A. No. 7394; Act No. 2333, as amended by Act No. 3740; and E.O. No. 913. The dispositive portion thereof reads as follows:
In view of all the foregoing, it is recommended that: 1. The attached information be filed against respondents Paul G. Roberts, Jr., Rodolfo C. Salazar, Rosemarie R. Vera, Luis F. Lorenzo, Sr., Luis P. Lorenzo, Jr., J. Roberto Delgado, Amaury R. Gutierrez, Bayani N. Fabic, Jose Yulo, Jr., Esteban B. Pacannuayan, Jr., Wong Fong Fui, Quintin J. Gomez, Jr. and Chito V. Gutierrez for estafa under Article 318, Revised Penal Code, while the complaint for violation of Article 315, 2(d), Revised Penal Code against same respondents Juanito R. Ignacio, R. Sobong, R.O. Sinsuan, M.P. Zarsadias, L.G. Dabao, Jr., R.L. Domingo, N.N. Bacsal, Jesus M. Manalastas, Janette P. Pio de Roda, Joaquin W. Sampaico, Winefreda O. Madarang, Jack Gravey, Les G. Ham, Corazon Pineda, Edward S. Serapio, Alex O. Caballes, Sandy Sytangco, Jorge W. Drysdale, Richard Blossom, Pablo de Borja, Edmundo L. Tan, Joseph T. Cohen, Delfin Dator, Zosimo B. San Juan, Joaquin Franco, Primitivo S. Javier, Jr., Luisito Guevarra, Asif H. Adil, Eugenio Muniosguren, James Ditkoff and Timothy Lane be dismissed; 2. The complaints against all respondents for violation of R.A. 7394 otherwise known as the Consumer Act of the Philippines and violation of Act 2333 as amended by Act 3740 and E.O. 913 be also dismissed for insufficiency of evidence, and 3. I.S. Nos. 92-7833; 92-8710 and 92-P-1065 involving Crowns Nos. 173; 401; and 117, 425, 703 and 373, respectively, alleged to be likewise winning ones be further 11 investigated to afford respondents a chance to submit their counter-evidence.

On 6 April 1993, City Prosecutor Candido V. Rivera approved the recommendation with the modification that Rosemarie Vera, Quintin Gomez, Jr., and Chito Gonzales be excluded from the charge on the ground of insufficiency of evidence. 12 The information for estafa attached to the Joint Resolution was approved (on 7 April 1993) by Ismael P. Casabar, Chief of the Prosecution Division, upon authority of the City Prosecutor of Quezon City, and was filed with the RTC of Quezon City on 12 April 1993. It was docketed as Criminal Case No. Q-93-43198. 13 The information reads as follows:
The undersigned 1st Assistant City Prosecutor accuses PAUL G. ROBERTS, JR. RODOLFO C. SALAZAR, LUIS F. LORENZO, SR., LUIS P. LORENZO, JR., J. ROBERTO DELGADO, AMAURY R. GUTIERREZ, BAYANI N. FABIC, JOSE YULO, JR., ESTEBAN B. PACANNUAYAN, JR. and WONG FONG FUI, of the crime of ESTAFA, committed as follows:

That in the month of February, 1992, in Quezon City, Philippines and for sometime prior and subsequent thereto, the above-named accused Paul G. Roberts, Jr. ) being then the Presidents Rodolfo G. Salazar ) and Executive Officers Luis F. Lorenzo, Sr. ) being then the Chairman of the Board of Directors Luis P. Lorenzo, Jr. ) being then the Vice Chairman of the Board J. Roberto Delgado ) Amaury R. Gutierrez ) being then Members of Bayani N. Fabic ) the Board Jose Yulo, Jr. ) Esteban B. Pacannuayan, ) Jr. and Wong Fong Fui ) OF THE PEPSI COLA PRODUCTS PHILIPPINES, INC., CONSPIRING with one another, with intent of gain, by means of deceit, fraudulent acts or false pretenses, executed prior to or simultaneously with the commission of the fraud, did then and there willfully, unlawfully and feloniously defraud the private complainants whose names with their prizes claimed appear in the attached lists marked as Annexes "A" to "A-46"; "B" to "-33"; "C" to "C-281"; "D" to "D-238"; "E" to "E-30" and "F" to "F-244" in the following manner: on the date and in the place aforementioned, said accused pursuant to their conspiracy, launched the Pepsi Cola Products Philippines, Inc. "Number Fever Promotion" from February 17 to May 8, 1992 later extended to May 11-June 12, 1992 and announced and advertised in the media that "all holders of crowns and/or caps of Pepsi, Mirinda, Mountain Dew and Seven-up bearing the winning 3-digit number will win the full amount of the prize printed on the crowns/caps which are marked with a sevendigit security code as a measure against tampering or faking of crowns and each and every number has its own unique matching security code", enticing the public to buy Pepsi softdrinks with aforestated alluring and attractive advertisements to become millionaires, and by virtue of such representations made by the accused, the said complainants bought Pepsi softdrinks, but, the said accused after their TV announcement on May 25, 1992 that the winning number for the next day was "349", in violation of their aforecited mechanics, refused as they still refuse to redeem/pay the said Pepsi crowns and/or caps presented to them by the complainants, who, among others, were able to buy Pepsi softdrinks with crowns/caps bearing number "349" with security codes L-2560FQ and L-3560-FQ, despite repeated demands made by the complainants, to their damage and prejudice to the extent of the amount of the prizes respectively due them

from their winning "349" crowns/caps, together with such amounts they spent in going to and from the Office of Pepsi to claim their prizes and such other amounts used in buying Pepsi softdrinks which the complainants normally would not have done were it not for the false, fraudulent and deceitful posters of Pepsi Cola Products Philippines, Inc. CONTRARY TO LAW.

On 14 April 1993, the petitioners filed with the Office of the City Prosecutor a motion for the reconsideration of the Joint Resolution 14 alleging therein that (a) there was neither fraud in the Number Fever Promotion nor deviation from or modification of the promotional rules approved by the Department of Trade and Industry (DTI), for from the start of the promotion, it had always been clearly explained to the public that for one to be entitled to the cash prize his crown must bear both the winning number and the correct security code as they appear in the DTI list; (b) the complainants failed to allege, much less prove with prima facie evidence, the specific overt criminal acts or omissions purportedly committed by each of the petitioners; (c) the compromise agreement entered into by PEPSI is not an admission of guilt; and (d) the evidence establishes that the promo was carried out with utmost good faith and without malicious intent. On 15 April 1993, the petitioners filed with the DOJ a Petition for Review 15 wherein, for the same grounds adduced in the aforementioned motion for reconsideration, they prayed that the Joint Resolution be reversed and the complaints dismissed. They further stated that the approval of the Joint Resolution by the City Prosecutor was not the result of a careful scrutiny and independent evaluation of the relevant facts and the applicable law but of the grave threats, intimidation, and actual violence which the complainants had inflicted on him and his assistant prosecutors. On that same date, the petitioners filed in Criminal Case No. Q-93-43198 Motions to Suspend Proceedings and to Hold in Abeyance Issuance of Warrants of Arrest on the ground that they had filed the aforesaid Petition for Review. 16 On 21 April 1993, acting on the Petition for Review, Chief State Prosecutor Zenon L. de Guia issued a 1st Indorsement, 17 directing the City Prosecutor of Quezon City to inform the DOJ whether the petitioners have already been arraigned, and if not, to move in court for the deferment of further proceedings in the case and to elevate to the DOJ the entire records of the case, for the case is being treated as an exception pursuant to Section 4 of Department Circular No. 7 dated 25 January 1990. On 22 April 1993, Criminal Case No. Q-93-41398 was raffle to Branch 104 of the RTC of Quezon City. 18 In the morning of 27 April 1993, private prosecutor Julio Contreras filed an Ex-Parte Motion for Issuance of Warrants of Arrest. 19 In the afternoon of that same day, petitioner Paul Roberts, Jr., filed a Supplemental Urgent Motion to Hold in Abeyance Issuance of Warrant of Arrest and to Suspend Proceedings. 20 He stressed that the DOJ had taken cognizance of the Petition for

Review by directing the City Prosecutor to elevate the records of I.S. No. P-4401 and its related cases and asserted that the petition for review was an essential part of the petitioners' right to a preliminary investigation. The next day, respondent Judge Asuncion, Presiding Judge of Branch 104 of the RTC of Quezon City, issued an order advising the parties that his court would "be guided by the doctrine laid down by the Supreme Court in the case of Crespo vs. Mogul, 151 SCRA 462 and not by the resolution of the Department of Justice on the petition for review undertaken by the accused." 21 On 30 April 1993, Assistant City Prosecutor Tirso M. Gavero filed with the trial court a Motion to Defer Arraignment wherein he also prayed that "further proceedings be held in abeyance pending final disposition by the Department of Justice." 22 On 4 May 1993, Gavero filed an Amended Information, 23 accompanied by a corresponding motion 24 to admit it. The amendments merely consist in the statement that the complainants therein were only "among others" who were defrauded by the accused and that the damage or prejudice caused amounted "to several billions of pesos, representing the amounts due them from their winning '349' crowns/caps." The trial court admitted the amended information on the same date. 25 Later, the attorneys for the different private complainants filed, respectively, an Opposition to Motion to Defer Arraignment, 26 and Objection and Opposition to Motion to Suspend Proceedings and to Hold in Abeyance the Issuance of Warrants of Arrest. 27 On 14 May 1993, the petitioners filed a Memorandum in Support of their Motion to Suspend Proceedings and to Hold in Abeyance the Issuance of the Warrants of Arrest.
28

On 17 May 1993, respondent Judge Asuncion issued the challenged order (1) denying the petitioners' Motion to Suspend Proceedings and to Hold in Abeyance Issuance of Warrants of Arrest and the public prosecutor's Motion to Defer Arraignment and (2) directing the issuance of the warrants of arrest "after June 1993" and setting the arraignment on 28 June 1993. 29 Pertinent portions of the order read as follows:
In the Motion filed by the accused, it is alleged that on April 15, 1993, they filed a petition for review seeking the reversal of the resolution of City Prosecutor of Quezon City approving the filing of the case against the accused, claiming that: 1. The resolution constituting [sic] force and duress; 2. There was no fraud or deceit therefore there can be no estafa; 3. No criminal overt acts by respondents were proved; 4. Pepsi nor the accused herein made no admission of guilt before the Department of Trade and Industry;

5. The evidence presented clearly showed no malicious intent on the part of the accused. Trial Prosecutor Tirso M. Gavero in his Motion to Defer Arraignment averred that there is a pending petition for review with the Department of Justice filed by the accused and the Office of the City Prosecutor was directed, among other things, to cause for the deferment of further proceedings pending final disposition of said Petition by the Department of Justice. The motions filed by the accused and the Trial Prosecutor are hereby DENIED. This case is already pending in this Court for trial. To follow whatever opinion the Secretary of Justice may have on the matter would undermine the independence and integrity of this Court. This Court is still capable of administering justice. The Supreme Court in the case of Crespo vs. Mogul (SCRA 151, pp. 471-472) stated as follows: In order therefor to avoid such a situation whereby the opinion of the Secretary of Justice who reviewed the action of the fiscal may be disregarded by the trial court, the Secretary of Justice should, as far as practicable, refrain from entertaining a petition for review or appeal from the action of the fiscal, when the complaint or information has already been filed in Court. The matter should be left entirely for the determination of the Court. WHEREFORE, let warrant of arrest be issued after June 21, 1993, and arraignment be set on June 28, 1993, at 9:30 in the morning.

On 7 June 1993, the petitioners filed with the Court of Appeals a special civil action for certiorari and prohibition with application for a temporary restraining order, 30 which was docketed as CA-G.R. SP No. 31226. They contended therein that respondent Judge Asuncion had acted without or in excess of jurisdiction or with grave abuse of discretion in issuing the aforementioned order of 17 May 1993 because
I. RESPONDENT JUDGE FAILED TO EXAMINE THE RECORD OF PRELIMINARY INVESTIGATION BEFORE ORDERING THE ARREST OF PETITIONERS. II. THERE IS NO PROBABLE CAUSE TO HOLD PETITIONERS CRIMINALLY LIABLE FOR ESTAFA, OTHER DECEITS, OR ANY OTHER OFFENSE. III. THE PROCEEDINGS BELOW SHOULD HAVE BEEN SUSPENDED TO AWAIT THE SECRETARY OF JUSTICE'S RESOLUTION OF PETITIONERS' APPEAL, AND IV. THERE IS NO OTHER PLAIN, SPEEDY AND ADEQUATE REMEDY IN THE ORDINARY COURSE OF LAW.

On 15 June 1993, the Court of Appeals issued a temporary restraining order to maintain the status quo. 31 In view thereof; respondent Judge Asuncion issued an order on 28 June 1993 32 postponing indefinitely the arraignment of the petitioners which was earlier scheduled on that date.

On 28 June 1993, the Court of Appeals heard the petitioners' application for a writ of preliminary injunction, granted the motion for leave to intervene filed by J. Roberto Delgado, and directed the Branch Clerk of Court of the RTC of Quezon City to elevate the original records of Criminal Case No. 4-93-43198. 33 Upon receipt of the original records of the criminal case, the Court of Appeals found that a copy of the Joint Resolution had in fact been forwarded to, and received by, the trial court on 22 April 1993, which fact belied the petitioners' claim that the respondent Judge had not the slightest basis at all for determining probable cause when he ordered the issuance of warrants of arrest. It ruled that the Joint Resolution "was sufficient in itself to have been relied upon by respondent Judge in convincing himself that probable cause indeed exists for the purpose of issuing the corresponding warrants of arrest"; and that the "mere silence of the records or the absence of any express declaration" in the questioned order as to the basis of such finding does not give rise to an adverse inference, for the respondent Judge enjoys in his favor the presumption of regularity in the performance of his official duty. The Court of Appeals then issued a resolution 34 denying the application for a writ of preliminary injunction. On 8 June 1993, the petitioners filed a motion to reconsider 35 the aforesaid resolution. The Court of Appeals required the respondents therein to comment on the said motion.
36

On 3 August 1993, the counsel for the private complainants filed in CA-G.R. SP No. 31226 a Manifestation 37 informing the court that the petitioners' petition for review filed with the DOJ was dismissed in a resolution dated 23 July 1993. A copy 38 of the resolution was attached to the Manifestation. On 21 September 1993, the public respondents filed in CA-G.R. SP No. 31226 a motion to dismiss the petition 39 on the ground that it has become moot and academic in view of the dismissal by the DOJ of the petitioners' petition to review the Joint Resolution. The dismissal by the DOJ is founded on the following exposition:
You questioned the said order of the RTC before the Court of Appeals and prayed for the issuance of a writ of preliminary injunction to restrain the Trial Judge from issuing any warrant of arrest and from proceeding with the arraignment of the accused. The appellate court in a resolution dated July 1, 1993, denied your petition. In view of the said developments, it would be an exercise in futility to continue reviewing the instant cases for any further action on the part of the Department would depend on the sound discretion of the Trial Court. The denial by the said court of the motion to defer arraignment filed at our instance was clearly an exercise of its discretion. With the issuance of the order dated May 17, 1993, Trial Court was in effect sending a signal to this Department that "the determination of the case is within its exclusive jurisdiction and competence." The rule is that ". . . once a complaint or information is filed in Court, any disposition of the case as to dismissal or the conviction or acquittal of the accused rests in the sound discretion of the Court. Although the fiscal retains the direction and control of the prosecution of criminal cases even while the case is already in Court, he cannot impose his opinion on the trial court. The court is the best and sole judge on what to do 40 with the case before it. . . ." (Crespo vs. Mogul, 151 SCRA 462).

On 28 September 1993, the Court of Appeals promulgated a decision 41 dismissing the petition because it had been "mooted with the release by the Department of Justice of its decision . . . dismissing petitioners' petition for review by inerrantly upholding the criminal court's exclusive and unsupplantable authority to control the entire course of the case brought against petitioners, reiterating with approval the dictum laid down in the 'Crespo' case." The petitioners filed a motion to reconsider the DOJ's dismissal of the petition citing therein its resolutions in other similar cases which were favorable to the petitioners and adverse to other "349" Pepsi crowns holders. In its resolution of 3 February 1994, the DOJ, through its "349" Committee, denied the motion and stated: "The instant petition is different from the other petitions resolved by this Department in similar cases from the provinces. In the latter petitions, the complaints against herein respondents [sic] 42 were dismissed inasmuch as the informations have not yet been filed or even if already filed in court, the proceedings have been suspended by the courts to await the outcome of the appeal pending with this Department." 43 The petitioners likewise filed a motion to reconsider 44 the aforesaid Court of Appeals' decision, which the said court denied in its resolution 45 of 9 February 1994. Hence, the instant petition. The First Division of this Court denied due course to this petition in its resolution of 19 September 1994. 46 On 7 October 1994, the petitioners filed a motion for the reconsideration 47 of the aforesaid resolution. Acting thereon, the First Division required the respondents to comment thereon. Later, the petitioners filed a supplemental motion for reconsideration 48 and a motion to refer this case to the Court en banc. 49 In its resolution of 14 November 1994, 50 the First Division granted the latter motion and required the respondents to comment on the supplemental motion for reconsideration. In the resolution of 24 November 1994, the Court en banc accepted the referral. On 10 October 1995, after deliberating on the motion for reconsideration and the subsequent pleadings in relation thereto, the Court en banc granted the motion for reconsideration; reconsidered and set aside the resolution of 19 September 1994; and reinstated the petition. It then considered the case submitted for decision, "since the parties have exhaustively discussed the issues in their pleadings, the original records of Criminal Case No. Q-93-43198 and of CA-G.R. SP No. 31226 had been elevated to this Court, and both the petitioners and the Office of the Solicitor General pray, in effect, that this Court resolve the issue of probable cause on the basis thereof."

The pleadings of the parties suggest for this Court's resolution the following key issues:
1. Whether public respondent Judge Asuncion committed grave abuse of discretion in denying, on the basis of Crespo vs. Mogul, the motions to suspend proceedings and hold in abeyance the issuance of warrants of arrest and to defer arraignment until after the petition for review filed with the DOJ shall have been resolved. 2. Whether public respondent Judge Asuncion committed grave abuse of discretion in ordering the issuance of warrants of arrest without examining the records of the preliminary investigation. 3. Whether the DOJ, through its "349" Committee, gravely abused its discretion in dismissing the petition for review on the following bases: (a) the resolution of public respondent Court of Appeals denying the application for a writ of preliminary injunction and (b) of public respondent Asuncion's denial of the abovementioned motions. 4. Whether public respondent Court of Appeals committed grave abuse of discretion (a) in denying the motion for a writ of preliminary injunction solely on the ground that public respondent Asuncion had already before him the Joint Resolution of the investigating prosecutor when he ordered the issuance of the warrants of arrest, and (b) in ultimately dismissing the petition on the ground of mootness since the DOJ had dismissed the petition for review. 5. Whether this Court may determine in this proceedings the existence of probable cause either for the issuance of warrants of arrest against the petitioners or for their prosecution for the crime of estafa.

We resolve the first four issues in the affirmative and the fifth, in the negative. I. There is nothing in Crespo vs. Mogul 51 which bars the DOJ from taking cognizance of an appeal, by way 'of a petition for review, by an accused in a criminal case from an unfavorable ruling of the investigating prosecutor. It merely advised the DOJ to, "as far as practicable, refrain from entertaining a petition for review or appeal from the action of the fiscal, when the complaint or information has already been filed in Court." More specifically, it stated:
In order therefore to avoid such a situation whereby the opinion of the Secretary of Justice who reviewed the action of the fiscal may be disregarded by the trial court, the Secretary of Justice should, as far as practicable, refrain from entertaining a petition for review or appeal from the action of the fiscal, when the complaint or information has already been filed in Court. The matter should, be left entirely for the determination of the 52 Court.

In Marcelo vs. Court of Appeals, 53 this Court explicitly declared:


Nothing in the said ruling forecloses the power or authority of the Secretary of Justice to review resolutions of his subordinates in criminal cases. The Secretary of Justice is only enjoined to refrain as far as practicable from entertaining a petition for review or appeal from the action of the prosecutor once a complaint or information is filed in court. In any

case, the grant of a motion to dismiss, which the prosecution may file after the Secretary of Justice reverses an appealed resolution, is subject to the discretion of the court.

Crespo could not have intended otherwise without doing violence to, or repealing, the last paragraph of Section 4, Rule 112 of the Rules of Court 54 which recognizes the authority of the Secretary of Justice to reverse the resolution of the provincial or city prosecutor or chief state prosecutor upon petition by a proper party. Pursuant to the said provision, the Secretary of Justice had promulgated the rules on appeals from resolutions in preliminary investigation. At the time the petitioners filed their petition for the review of the Joint Resolution of the investigating prosecutor, the governing rule was Circular No. 7, dated 25 January 1990. Section 2 thereof provided that only resolutions dismissing a criminal complaint may be appealed to the Secretary of Justice. Its Section 4, 55 however, provided an exception, thus allowing, upon a showing of manifest error or grave abuse of discretion, appeals from resolutions finding probable cause, provided that the accused has not been arraigned. The DOJ gave due course to the petitioners' petition for review as an exception pursuant to Section 4 of Circular No. 7. Meanwhile, the DOJ promulgated on 30 June 1993 Department Order No. 223 56 which superseded Circular No. 7. This Order, however, retained the provisions of Section 1 of the Circular on appealable cases and Section 4 on the non-appealable cases and the exceptions thereto. There is nothing in Department Order No. 223 which would warrant a recall of the previous action of the DOJ giving due course to the petitioners' petition for review. But whether the DOJ would affirm or reverse the challenged Joint Resolution is still a matter of guesswork. Accordingly, it was premature for respondent Judge Asuncion to deny the motions to suspend proceedings and to defer arraignment on the following grounds:
This case is already pending in this Court for trial. To follow whatever opinion the Secretary of Justice may have on the matter would undermine the independence and integrity of this Court. This Court is still capable of administering justice.

The real and ultimate test of the independence and integrity of his court is not the filing of the aforementioned motions at that stage of the proceedings but the filing of a motion to dismiss or to withdraw the information on the basis of a resolution of the petition for review reversing the Joint Resolution of the investigating prosecutor. Before that time, the following pronouncement in Crespo did not yet truly become relevant or applicable:
The rule therefore in this jurisdiction is that once a complaint or information is filed in Court any disposition of the case as its dismissal or the conviction or acquittal of the accused rests in the sound discretion of the court. Although the fiscal retains the direction and control of the prosecution of criminal cases even while the case is already in court he cannot impose his opinion on the trial court. The court is the best and sole judge on what to do with the case before it. The determination of the case is within its exclusive jurisdiction and competence. A motion to dismiss the case filed by the fiscal should be addressed to the Court who has the option to grant or deny the same. It does not matter

if this is done before or after the arraignment of the accused or that the motion was filed after a reinvestigation or upon instructions of the Secretary of Justice who reviewed the 5 records of the investigation. 7

However, once a motion to dismiss or withdraw the information is filed the trial judge may grant or deny it, not out of subservience to the Secretary of Justice, but in faithful exercise of judicial prerogative. This Court pertinently stated so in Martinez vs. Court of Appeals: 58
Whether to approve or disapprove the stand taken by the prosecution is not the exercise of discretion required in cases like this. The trial judge must himself be convinced that there was indeed no sufficient evidence against the accused, and this conclusion can be arrived at only after an assessment of the evidence in the possession of the prosecution. What was imperatively required was the trial judge's own assessment of such evidence, it not being sufficient for the valid and proper exercise of judicial discretion merely to accept the prosecution's word for its supposed insufficiency. As aptly observed the Office of the Solicitor General, in failing to make an independent finding of the merits of the case and merely anchoring the dismissal on the revised position of the prosecution, the trial judge relinquished the discretion he was duty bound to exercise. In effect, it was the prosecution, through the Department of Justice which decided what to do and not the court which was reduced to a mere rubber stamp in violation of the ruling in Crespo vs. Mogul.

II. Section 2, Article III of the present Constitution provides that no search warrant or warrant of arrest shall issue except upon probable cause to be determined personally by the judge after examination under oath or affirmation of the complainant and the witnesses he may produce. Under existing laws, warrants of arrest may be issued (1) by the Metropolitan Trial Courts (MeTCs) except those in the National Capital Region, Municipal Trial Courts (MTCs), and Municipal Circuit Trial Courts (MCTCs) in cases falling within their exclusive original jurisdiction; 59 in cases covered by the rule on summary procedure where the accused fails to appear when required; 60 and in cases filed with them which are cognizable by the Regional Trial Courts (RTCs); 61 and (2) by the Metropolitan Trial Courts in the National Capital Region (MeTCs-NCR) and the RTCs in cases filed with them after appropriate preliminary investigations conducted by officers authorized to do so other than judges of MeTCs, MTCs and MCTCs. 62 As to the first, a warrant can issue only if the judge is satisfied after an examination in writing and under oath of the complainant and the witnesses, in the form of searching questions and answers, that a probable cause exists and that there is a necessity of placing the respondent under immediate custody in order not to frustrate the ends of justice. As to the second, this Court held in Soliven vs. Makasiar 63 that the judge is not required to personally examine the complainant and the witnesses, but

[f]ollowing established doctrine and procedure, he shall: (1) personally evaluate the report and supporting documents submitted by the fiscal regarding the existence of probable cause and, on the basis thereof; issue a warrant of arrest; or (2) if on the basis thereof he finds no probable cause, he may disregard the fiscal's report and require the submission of supporting affidavits of witnesses to aid him in arriving at a conclusion as to the 64 existence of probable cause.

Sound policy supports this procedure, "otherwise judges would be unduly laden with the preliminary examination and investigation of criminal complaints instead of concentrating on hearing and deciding cases filed before their courts." It must be emphasized that judges must not rely solely on the report or resolution of the fiscal (now prosecutor); they must evaluate the report and the supporting document. In this sense, the aforementioned requirement has modified paragraph 4(a) of Circular No. 12 issued by this Court on 30 June 1987 prescribing the Guidelines on Issuance of Warrants of Arrest under Section 2, Article III of the 1987 Constitution, which provided in part as follows:
4. In satisfying himself of the existence of a probable cause for the issuance of a warrant of arrest, the judge, following established doctrine and procedure, may either: (a) Rely upon the fiscal's certification of the existence of probable cause whether or not the case is cognizable only by the Regional Trial Court and on the basis thereof, issue a warrant of arrest. . . .

This requirement of evaluation not only of the report or certification of the fiscal but also of the supporting documents was further explained in People vs. Inting, 65 where this Court specified what the documents may consist of, viz., "the affidavits, the transcripts of stenographic notes (if any), and all other supporting documents behind the Prosecutor's certification which are material in assisting the Judge to make his determination" of probable cause. Thus:
We emphasize the important features of the constitutional mandate that ". . . no search warrant or warrant of arrest shall issue except upon probable cause to be determined personally by the judge . . ." (Article III, Section 2, Constitution). First, the determination of probable cause is a function of the Judge. It is not for the Provincial Fiscal or Prosecutor nor the Election Supervisor to ascertain. Only the Judge and the Judge alone makes this determination. Second, the preliminary inquiry made by a Prosecutor does not bind the Judge. It merely assists him to make the determination of probable cause. The Judge does not have to follow what the Prosecutor presents to him. By itself, the Prosecutor's certification of probable cause is ineffectual. It is the report, the affidavits, the transcripts of stenographic notes (if any), and all other supporting documents behind the Prosecutor's certification which are material in assisting the Judge to make his determination.

In adverting to a statement in People vs. Delgado 66 that the judge may rely on the resolution of the Commission on Elections (COMELEC) to file the information by the same token that it may rely on the certification made by the prosecutor who conducted

the preliminary investigation in the issuance of the warrant of arrest, this Court stressed in Lim vs. Felix 67 that
Reliance on the COMELEC resolution or the Prosecutor's certification presupposes that the records of either the COMELEC or the Prosecutor have been submitted to the Judge and he relies on the certification or resolution because the records of the investigation sustain the recommendation. The warrant issues not on the strength of the certification standing alone but because of the records which sustain it.

And noting that judges still suffer from the inertia of decisions and practice under the 1935 and 1973 Constitutions, this Court found it necessary to restate the rule "in greater detail and hopefully clearer terms." It then proceeded to do so, thus:
We reiterate the ruling in Soliven vs. Makasiar that the Judge does not have to personally examine the complainant and his witnesses. The Prosecutor can perform the same functions as a commissioner for the taking of the evidence. However, there should be a report and necessary documents supporting the Fiscal's bare certification. All of these should be before the Judge. The extent of the Judge's personal examination of the report and its annexes depends on the circumstances of each case. We cannot determine beforehand how cursory or exhaustive the Judge's examination should be. The Judge has to exercise sound discretion for, after all, the personal determination is vested in the Judge by the Constitution. It can be as brief as or detailed as the circumstances of each case require. To be sure, the Judge must go beyond the Prosecutor's certification and investigation report whenever necessary. He should call for the complainant and witnesses themselves to answer the court's probing questions when the circumstances of the case so require.

This Court then set aside for being null and void the challenged order of respondent Judge Felix directing the issuance of the warrants of arrest against petitioners Lim, et al., solely on the basis of the prosecutor's certification in the informations that there existed probable cause "without having before him any other basis for his personal determination of the existence of a probable cause." In Allado vs. Diokno, 68 this Court also ruled that "before issuing a warrant of arrest, the judge must satisfy himself that based on the evidence submitted there is sufficient proof that a crime has been committed and that the person to be arrested is probably guilty thereof." In the recent case of Webb vs. De Leon, 69 this Court rejected the thesis of the petitioners of absence of probable cause and sustained the investigating panel's and the respondent Judge's findings of probable cause. After quoting extensively from Soliven vs. Makasiar, 70 this Court explicitly pointed out:
Clearly then, the Constitution, the Rules of Court, and our case law repudiate the submission of petitioners that respondent judges should have conducted "searching examination of witnesses" before issuing warrants of arrest against them. They also reject petitioners' contention that a judge must first issue an order of arrest before issuing a warrant of arrest. There is no law or rule requiring the issuance of an Order of Arrest prior to a warrant of arrest.

In the case at bar, the DOJ Panel submitted to the trial court its 26-page report, the two (2) sworn statements of Alfaro and the sworn statements of Carlos Cristobal and Lolita Birrer as well as the counter-affidavits of the petitioners. Apparently, the painstaking recital and analysis of the parties' evidence made in the DOJ Panel Report satisfied both judges that there is probable cause to issue warrants of arrest against petitioners. Again, we stress that before issuing warrants of arrest, judges merely determine personally the probability, not the certainty of the guilt of an accused. In doing so, judges do not conduct a de novo hearing to determine the existence of probable cause. They just personally review the initial determination of the prosecutor finding a probable cause to see if it is supported by substantial evidence. The sufficiency of the review process cannot be measured by merely counting minutes and hours. The fact that it took the respondent judges a few hours to review and affirm the probable cause determination of the DOJ Panel does not mean they made no personal evaluation of the evidence attached to the records of the case. (emphasis supplied)

The teachings then of Soliven, Inting, Lim, Allado, and Webb reject the proposition that the investigating prosecutor's certification in an information or his resolution which is made the basis for the filing of the information, or both, would suffice in the judicial determination of probable cause for the issuance of a warrant of arrest. In Webb, this Court assumed that since the respondent Judges had before them not only the 26-page resolution of the investigating panel but also the affidavits of the prosecution witnesses and even the counter-affidavits of the respondents, they (judges) made personal evaluation of the evidence attached to the records of the case. Unfortunately, in Criminal Case No. Q-93-43198, nothing accompanied the information upon its filing on 12 April 1993 with the trial court. As found by the Court of Appeals in its resolution of 1 July 1993, a copy of the Joint Resolution was forwarded to, and received by, the trial court only on 22 April 1993. And as revealed by the certification 71 of Branch Clerk of Court Gibson Araula, Jr., no affidavits of the witnesses, transcripts of stenographic notes of the proceedings during the preliminary investigation, or other documents submitted in the course thereof were found in the records of Criminal Case No. Q-93-43198 as of 19 May 1993. Clearly, when respondent Judge Asuncion issued the assailed order of 17 May 1993 directing, among other things, the issuance of warrants of arrest, he had only the information, amended information, and Joint Resolution as bases thereof. He did not have the records or evidence supporting the prosecutor's finding of probable cause. And strangely enough, he made no specific finding of probable cause; he merely directed the issuance of warrants of arrest "after June 21, 1993." It may, however, be argued that the directive presupposes a finding of probable cause. But then compliance with a constitutional requirement for the protection of individual liberty cannot be left to presupposition, conjecture, or even convincing logic. III. As earlier stated, per its 1st Indorsement of 21 April 1993, the DOJ gave due course to the petitioners' petition for review pursuant to the exception provided for in Section 4 of Circular No. 7, and directed the Office of the City Prosecutor of Quezon City to forward to the Department the records of the cases and to file in court a motion for the deferment of the proceedings. At the time it issued the indorsement, the DOJ already

knew that the information had been filed in court, for which reason it directed the City Prosecutor to inform the Department whether the accused have already been arraigned and if not yet arraigned, to move to defer further proceedings. It must have been fully aware that, pursuant to Crespo vs. Mogul, a motion to dismiss a case filed by the prosecution either as a consequence of a reinvestigation or upon instructions of the Secretary of Justice after a review of the records of the investigation is addressed to the trial court, which has the option to grant or to deny it. Also, it must have been still fresh in its mind that a few months back it had dismissed for lack of probable cause other similar complaints of holders of "349" Pepsi crowns. 72 Thus, its decision to give due course to the petition must have been prompted by nothing less than an honest conviction that a review of the Joint Resolution was necessary in the highest interest of justice in the light of the special circumstances of the case. That decision was permissible within the "as far as practicable" criterion in Crespo. Hence, the DOJ committed grave abuse of discretion when it executed on 23 July 1993 a unilateral volte-face, which was even unprovoked by a formal pleading to accomplish the same end, by dismissing the petition for review. It dismissed the petition simply because it thought that a review of the Joint Resolution would be an exercise in futility in that any further action on the part of the Department would depend on the sound discretion of the trial court, and that the latter's denial of the motion to defer arraignment filed at the instance of the DOJ was clearly an exercise of that discretion or was, in effect, a signal to the Department that the determination of the case is within the court's exclusive jurisdiction and competence. This infirmity becomes more pronounced because the reason adduced by the respondent Judge for his denial of the motions to suspend proceedings and hold in abeyance issuance of warrants of arrest and to defer arraignment finds, as yet, no support in Crespo. IV. If the only issue before the Court of Appeals were the denial of the petitioners' Motion to Suspend Proceedings and to Hold in Abeyance Issuance of Warrants of Arrest and the public prosecutor's Motion to Defer Arraignment, which were both based on the pendency before the DOJ of the petition for the review of the Joint Resolution, the dismissal of CA-G.R. SP No. 31226 on the basis of the dismissal by the DOJ of the petition for review might have been correct. However, the petition likewise involved the issue of whether respondent Judge Asuncion gravely abused his discretion in ordering the issuance of warrants of arrest despite want of basis. The DOJ's dismissal of the petition for review did not render moot and academic the latter issue. In denying in its resolution of 1 July 1993 the petitioners' application for a writ of preliminary injunction to restrain respondent Judge Asuncion from issuing warrants of arrest, the Court of Appeals justified its action in this wise:
The Joint Resolution was sufficient in itself to have been relied upon by respondent judge in convincing himself that probable cause indeed exists for the purpose of issuing the corresponding warrants of arrest. The mere silence of the records or the absence of any express declaration in the questioned Order of May 17, 1993 as to where the respondent

Judge based his finding of probable cause does not give rise to any adverse inference on his part. The fact remains that the Joint Resolution was at respondent Judge's disposal at the time he issued the Order for the issuance of the warrants of arrest. After all, respondent Judge enjoys in his favor the presumption of regularity in the performance of official actuations. And this presumption prevails until it is overcome by clear and convincing evidence to the contrary. Every reasonable intendment will be made in support of the presumption, and in case of doubt as to an officer's act being lawful or unlawful it should be construed to be lawful. (31 C.J.S., 808-810. See also Mahilum, et al. vs. Court of Appeals, 17 SCRA 482; People vs. Cortez, 21 SCRA 1228; Government of the P.I. vs. Galarosa, 36 Phil. 338).

We are unable to agree with this disquisition, for it merely assumes at least two things: (1) that respondent Judge Asuncion had read and relied on the Joint Resolution and (2) he was convinced that probable cause exists for the issuance of the warrants of arrest against the petitioners. Nothing in the records provides reasonable basis for these assumptions. In his assailed order, the respondent Judge made no mention of the Joint Resolution, which was attached to the records of Criminal Case No. Q-93-43198 on 22 April 1993. Neither did he state that he found probable cause for the issuance of warrants of arrest. And, for an undivinable reason, he directed the issuance of warrants of arrest only "after June 21, 1993." If he did read the Joint Resolution and, in so reading, found probable cause, there was absolutely no reason at all to delay for more than one month the issuance of warrants of arrest. The most probable explanation for such delay could be that the respondent Judge had actually wanted to wait for a little while for the DOJ to resolve the petition for review. It is, nevertheless, contended in the dissenting opinion of Mr. Justice Reynato S. Puno that whatever doubts may have lingered on the issue of probable cause was dissolved when no less than the Court of Appeals sustained the finding of probable cause made by the respondent Judge after an evaluation of the Joint Resolution. We are not persuaded with that opinion. It is anchored on erroneous premises. In its 1 July 1993 resolution, the Court of Appeals does not at all state that it either sustained respondent Judge Asuncion's finding of probable cause, or found by itself probable cause. As discussed above, it merely presumed that Judge Asuncion might have read the Joint Resolution and found probable cause from a reading thereof. Then too, that statement in the dissenting opinion erroneously assumes that the Joint Resolution can validly serve as sufficient basis for determining probable cause. As stated above, it is not. V. In criminal prosecutions, the determination of probable cause may either be an executive or a judicial prerogative. In People vs. Inting, 73 this Court aptly stated:
And third, Judges and Prosecutors alike should distinguish the preliminary inquiry which determines probable cause for the issuance of a warrant of arrest from a preliminary investigation proper which ascertains whether the offender should be held for trial or released. Even if the two inquiries are conducted in the course of one and the same proceeding, there should be no confusion about the objectives. The determination of probable cause for the warrant of arrest is made by the Judge. The preliminary investigation proper whether or not there is reasonable ground to believe that the

accused is guilty of the offense charged and, therefore, whether or not he should be subjected to the expense, rigors and embarrassment of trial is the function of the Prosecutor. .... We reiterate that preliminary investigation should be distinguished as to whether it is an investigation for the determination of a sufficient ground for the filing of the information or it is an investigation for the determination of a probable cause for the issuance of a warrant of arrest. The first kind of preliminary investigation is executive in nature. It is part of the prosecution's job. The second kind of preliminary investigation which is more properly called preliminary examination is judicial in nature and is lodged with the judge. . ..

Ordinarily, the determination of probable cause is not lodged with this Court. Its duty in an appropriate case is confined to the issue of whether the executive or judicial determination, as the case may be, of probable cause was done without or in excess of jurisdiction or with grave abuse of discretion amounting to want of jurisdiction. This is consistent with the general rule that criminal prosecutions may not be restrained or stayed by injunction, preliminary or final. There are, however, exceptions to this rule. Among the exceptions are enumerated in Brocka vs. Enrile 74 as follows:
a. To afford adequate protection to the constitutional rights of the accused (Hernandez vs. Albano, et al., L-19272, January 25, 1967, 19 SCRA 95); b. When necessary for the orderly administration of justice or to avoid oppression or multiplicity of actions (Dimayuga, et al. vs. Fernandez, 43 Phil. 304; Hernandez vs. Albano, supra; Fortun vs. Labang, et al., L-38383, May 27, 1981, 104 SCRA 607); c. When there is a pre-judicial question which is sub judice (De Leon vs. Mabanag, 70 Phil. 202); d. When the acts of the officer are without or in excess of authority (Planas vs. Oil, 67 Phil. 62); e. Where the prosecution is under an invalid law, ordinance or regulation (Young vs. Rafferty, 33 Phil. 556; Yu Cong Eng vs. Trinidad, 47 Phil. 385, 389); f. When double jeopardy is clearly apparent (Sangalang vs. People and Avendia, 109 Phil. 1140); g. Where the court has no jurisdiction over the offense (Lopez vs. City Judge, L-25795, October 29, 1966, 18 SCRA 616); h. Where it is a case of persecution rather than prosecution (Rustia vs. Ocampo, CA-G.R. No. 4760, March 25, 1960); i. Where the charges are manifestly false and motivated by the lust for vengeance (Recto vs. Castelo, 18 L.J. [1953], cited in Raoa vs. Alvendia, CA-G.R. No. 30720-R, October 8, 1962; Cf. Guingona, et al. vs. City Fiscal, L-60033, April 4, 1984, 128 SCRA 577); and

j. When there is clearly no prima facie case against the accused and a motion to quash on that ground has been denied (Salonga vs. Pao, et al., L- 59524, February 18, 1985, 134 SCRA 438). 7. Preliminary injunction has been issued by the Supreme Court to prevent to threatened unlawful arrest of petitioners (Rodriguez vs. Castelo, L- 6374, August 1, 1953). (cited in Regalado, Remedial Law Compendium, p. 188, 1988 Ed.)

In these exceptional cases, this Court may ultimately resolve the existence or non-existence of probable cause by examining the records of the preliminary investigation, as it did in Salonga vs. Pao, 75 Allado, and Webb. There can be no doubt that, in light of the several thousand private complainants in Criminal Case No. Q-93-43198 and several thousands more in different parts of the country who are similarly situated as the former for being holders of "349" Pepsi crowns, any affirmative holding of probable cause in the said case may cause or provoke, as justly feared by the petitioners, the filing of several thousand cases in various courts throughout the country. Inevitably, the petitioners would be exposed to the harassments of warrants of arrest issued by such courts and to huge expenditures for premiums on bailbonds and for travels from one court to another throughout the length and breadth of the archipelago for their arraignments and trials in such cases. Worse, the filing of these staggering number of cases would necessarily affect the trial calendar of our overburdened judges and take much of their attention, time, and energy, which they could devote to other equally, if not more, important cases. Such a frightful scenario would seriously affect the orderly administration of justice, or cause oppression or multiplicity of actions a situation already long conceded by this Court to be an exception to the general rule that criminal prosecutions may not be restrained or stayed by injunction. 76 We shall not, however, reevaluate the evidence to determine if indeed there is probable cause for the issuance of warrants of arrest in Criminal Case No. Q-93-43298. For, as earlier stated, the respondent Judge did not, in fact, find that probable cause exists, and if he did he did not have the basis therefor as mandated by Soliven, Inting, Lim, Allado, and even Webb. Moreover, the records of the preliminary investigation in Criminal Case No. Q-93-43198 are not with this Court. They were forwarded by the Office of the City Prosecutor of Quezon City to the DOJ in compliance with the latter's 1st Indorsement of 21 April 1993. The trial court and the DOJ must be required to perform their duty. WHEREFORE, the instant petition is GRANTED and the following are hereby SET ASIDE: (a) Decision of 28 September 1993 and Resolution of 9 February 1994 of respondent Court of Appeals in CA-G.R. SP No. 31226; (b) The Resolutions of the "349" Committee of the Department of Justice of 23 July 1993 dismissing the petitioners' petition for review and of 3 February 1994 denying the motion to reconsider the dismissal; and

(c) The Order of respondent Judge Maximiano C. Asuncion of 17 May 1993 in Criminal Case No. Q-93-43198. The Department of Justice is DIRECTED to resolve on the merits, within sixty (60) days from notice of this decision, the petitioners' petition for the review of the Joint Resolution of Investigating Prosecutor Ramon Gerona and thereafter to file the appropriate motion or pleading in Criminal Case No. Q-93-43198, which respondent Judge Asuncion shall then resolve in light of Crespo vs. Mogul, Soliven vs. Makasiar, People vs. Inting, Lim vs. Felix, Allado vs. Diokno, and Webb vs. De Leon. In the meantime, respondent Judge Asuncion is DIRECTED to cease and desist from further proceeding with Criminal Case No. Q-93-43198 and to defer the issuances of warrants of arrest against the petitioners. No pronouncement as to costs. SO ORDERED.

PEOPLE VS. DEL ROSARIO [234 SCRA 246; G.R. NO. 109633; 20 JUL 1994] Wednesday, February 04, 2009 Posted by Coffeeholic Writes Labels: Case Digests, Political Law

Facts: Accused was charged and convicted by the trial court of illegal possession of firearms and illegal possession and sale of drugs, particularly methamphetamine or shabu. After the issuance of the search warrant, which authorized the search and seizure of an undetermined quantity of methamphetamine and its paraphernalias, an entrapment was planned that led to the arrest of del Rosario and to the seizure of the shabu, its paraphernalias and of a .22 caliber pistol with 3 live ammunition.

Issue:

Whether

or

Not

the

seizure

of

the

firearms

was

proper.

Held: No. Sec 2 art. III of the constitution specifically provides that a search warrant must particularly describe the things to be seized. In herein case, the only objects to be seized that the warrant determined was the methamphetamine and the paraphernalias therein. The seizure of the firearms was unconstitutional.

Wherefore the decision is reversed and the accused is acquitted. Probable Cause, Defined

Such facts and Circumstances which would lead areasonably prudent man to believe that an offense has beencommitted and the objects sought in connection with theoffense are in the place sought to be searched (20 thCentury Fox Film. Corp. vs. CA) De Los Santos vs. Montesa Facts:This case came from the Criminal Case for themurder of Reyes et. al in San Juan Del Monte Bulacan. After the submission of an investigation by the StateProsecutor, RTC of Malolos Bulacan issued a warrant of Arrest. First, the accused filed a petition to grant bail and toReduce Bail, subsequently withdrawn and rather filed anurgent motion to quash warrants of arrest for lack of existence of probable cause. After an examination of the lower court of the caseand documents forwarded to him by the prosecution, the trialcourt found the existence of probable cause , but instead of issuing of correspondent Arrest Warrant for acquiring jurisdiction of the persons involved in the crime, therespondent judge granted the petition for bail despite of theits withdrawal and lack of hearing where the prosecutionwould have been accorded the right to present evidenceshowing the evidence of guilt is strong.Thus the petitioner filed an administrative complaintcharging the respondent judge with gross ignorance of thelaw and evident dishonesty in his exercise of his function.Issue:What should be the procedure in the determinationof the existence of a probable cause? Held:What the respondent judge had in mind in the caseat bar is that, since he believed that the evidence agisnt theaccused are purely circumstantial and weak, he resolved togranting the petition of the accused to grant bail in order for the court to acquire jurisdiction over these persons instead of issuing warrant of arrest and set hearing for the prosecutionspresentation of evidence.He is thus then confused in the procedure of determining the existence of probable cause in the issuanceof arrest warrant and proceedings for admission to bail.What the court is given prerogative by theconstitution in accordance to Sec. 2 Art III is that, it issufficient that he personally evaluates the report andsupporting documents submitted by the prosecution indetermining probable cause, hearing is not necessary. Insatisfying the existence of probable cause, the judge shalleither 1. Personally evaluate the report and thesupporting documents submitted by the prosecutor regarding the existence of probablecause, and in basis thereof, issue arrest warrant and 2. If there is no sufficient establishment of probable cause, he may disregard the prosecutors certification and require thesubmission of the supporting affidavits of witnesses to aid him in arriving at a conclusionas to the existence of probable cause .The judge is not tasked to review in detail theevidence submitted during the preliminary investigation.Thus, once a judge found the probable cause, whathe should have to do is to issue warrants of arrest andadmission to bail shall only be granted once persons areapprehended and are under their jurisdiction. Lim vs. Felix Facts:The petitioners. Lim et al, was charged of the crimeof multiple murder and frustrated murder of CongressmanEspinosa of Masbate among others.Private respondent, Alfane was designated toreview the case and was raffled to RTC Makati Br. 56 of therespondent judge, Nemesio Felix. After transmittal of the case, the respondent Judgeissued warrant of Arrest against the accused by virtue of theprosecutors certification in each submitted informationrecommending the existence of a probable cause.Issue:Whether or not a judge may issue a warrant of arrest without bail by simply relying on the prosecutionscertification and recommendation that a probable causeexists.Held:Yes. But by itself, it does not bind judges to comeout with the warrant of arrest.Issuance of warrants calls for the exercise of judicialdiscretion on the part of the issuing judge. If the judge issatisfied from the preliminary examination conducted by himor by the investigating officer than an offense complained of has been committed and that there is a reasonable groundsto believe that the accused has committed it, he must issue awarrant or order for an arrest. A judge is not required to personally examine thecomplainants and witnesses, what the constitution mandatesin satisfying the existence of probable cause, the judge shalleither; 1. Personally evaluate the report and thesupporting documents submitted by the prosecutor regarding the existence of probablecause, and in basis thereof, issue arrest warrant and 2. If there is no sufficient establishment of probable cause, he may disregard the prosecutors certification and require thesubmission of the supporting affidavits of witnesses to aid him in arriving at a conclusionas to the existence of probable cause .Moreover, the constitution pursuant to Sec 2 Art IIIalso mandates that x x x probable cause should bepersonally determined by the judgex x x. This means that 1 . The determination of probable cause is a function of the judge.2. Preliminary inquiry made by a prosecutor does notbind the judge. 3. Judges and prosecutors alike should distinguish thepreliminary inquiry, which determines probablecause for the issuance of a warrant of arrest fromthe preliminary investigation proper, whichascertains whether the offender should be held for trial or release.In the case at bar, the only basis of the respondent judgein issuing warrants of arrest is only the certification of theprosecutor, without personally examining the information(

which still in Masbate, and wherein the respondent denied the motion for transmittal of such records of the cases in theground that certification id enough ground for thedetermination of probable cause and issuance of warrant ).Thus, there is no personal examination conducted by the judge to establish the existence of probable cause, thereby,the respondent committed abuse of discretion. Note: Preliminary investigation for the determination of sufficient ground for filing of information and investigation for the determination of a probable cause for the issuance of awarrant of arrest , Distinguished. T he former is executive in nature and part of aPROSECU T ORS JOB. While the latter aka preliminary examination is judicial in nature and is lodged to the JUDGE.

Amargavs.Abbas
Facts:The respondent Judge, Macapanton Abbas, after receiving; 1 ) An information with a certification stating that the petitioner fiscal, Amarga has conducted a sufficient preliminaryinvestigation pursuant to the provision of RA 732, and2) One supporting affidavit of one witness (Jubair) statingthat he saw the deceased Dugusan Paspasan was shot andkilled by three gunmen,Dismissed the criminal handled by the petitioner against Appang et. al on the ground that; 1 ) The affidavit of Jubair is hearsay and does notpossess gravity for the establishment of theexistence of probable cause.2) So as the certification of the petitioner fiscal wontwarrant the existence of probable cause.Therefore, there can be no prima facie evidence asto necessity for the issuance of warrant of arrest against theaccused.Thus, the petitioner filed a petitioner for certiorariand mandamus contending that petitioner has alreadyconducted a preliminary examination and thus it is ministerialfunction for the respondent to issue arrest warrants.Upon the other hand, the respondent judge arguesthat the issuance of a warrant of arrest involves a judicialpower which necessarily imposes upon him the legal duty of first satisfying himself that there is probable cause,independently of and notwithstanding the preliminaryinvestigation made by the provincial fiscal under Republic ActNo. 732; and to that end he may require the fiscal to submitsuch evidence as may be sufficient to show at least a primafacie case.Issue:Whether or not the certification of a prosecutor issufficient to issue a warrant of arrest.Held:No.The constitution mandates that the determination of probable cause depends upon the judgment and discretion of the judge or magistrate in issuing warrant of arrest.It simply means that sufficient facts must bepresented to the judge or magistrate issuing the warrant toconvince him, not that the particular person has committedthe crime, but that there is probable cause for believing thatthe person whose arrest is sought committed the crimechargedIn the case at bar, the petitioners certification thathe had already conducted a preliminary investigation in thecase does not sufficiently warrant the existence of probablecause. Nor the single affidavit submitted to the respondent asit is not enough for the respondent judge to exercise his judicial function to determine the existence of probablecause.However, the petition is granted to continue thehearing of the case in the ground of lack of prosecution andthat refusal of the prosecution to submit additional affidavit isnot a valid ground for the dismissal of a case.

20th Century Fox vs. CA


Facts: After surveillance and investigation, petitioner, withthe NBI apllied for three separate search warrants againstthe video outlets owned by private respondents, Barretto etal, and was subsequently granted and issued by the lower court.The applicants did not present the master tapes andthe pirated tapes upon their application of search warrant.Instead, only the deposition of two NBI witnesses served asthe basis for the existence of probable cause.Subsequently, the lower court lifted the three issuedsearch warrants in the ground that the articles seized, whichwere still under the custody of NBI, could not be a basis for any criminal prosecution.Issue:Was there an establishment of probable cause for the issuance of search warrant against the respondent?Held:None. The probable cause is wanting in theissuance of the search warrant. As ingrained by law and jurisprudence, probablecause is defined as such facts and circumstances whichwould lead a reasonably discreet and prudent man tobelieve that an offense has been committed and that theobjects sought in connection with the offenses are in the place sought to be searched. The existence of probable cause requires that thereshould be a personal knowledge by the complainant and witnesses of the facts upon which the issuance of asearch warrant may be justified .

In the case at bar, the lower court lifted the issuedsearch warrant on the ground of the discovery that the NBIwitnesses does not have personal knowledge that the crimeof Piracy has been committed. The deposition of suchwitnesses cannot provide a sufficient existence of probablecause necessary for the issuance of search warrant.Moreover, in accordance with PD 49 (Anti FilmPiracy), the essence of such law is the similarity of thepirated and the copyrighted work. Thus, the applicants mustpresent the court the copyrighted films and compare themwith the evidence of the video tapes allegedly pirated todetermine whether or not the crim has been violated.In the case at bar, the applicants does not provideany evidences 9master tapes and pirated tapes) that wouldconvince the judge that violation of PD 49 has been Constitutional Law II (Bill of Rights): Case Briefs: Dennis G. Libunao UC College of Law 19 committed, and therefore for the existence of probablecause.The articles included in the search warrant iscouched in general term, making it a general warrant which isprohibited under the Constitution. Note: S earch and S eizure, guaranteed rights. Protects a citizen against unreasonable searchersinvasion of his privacy and liberty as to his person, houses,papers and effects. The privacy of a person must not bedisturbed except in case of overriding social need and onlyunder stringent procedural safeguards.Thus, the government so as not to make arrest,search and seizure unreasonable should strictly followconstitutional and statutory guidelines.

Quintero vs. NBI ( M arcos m anipulation) Facts: After the expose of the petitioner, Eduardo Quinteroof the 1 st district of Leyte in the 1 97 1 Con-con alleging thatsome delegates, including him, in the Con-con are under thepayroll of the first lady and then President Marcos, NBIagents raided the houses of the petitioner by virtue of thesearch warrant issued by the respondent Judge Elias Asuncion. The raid confiscated an amount of moneyamounting to 379K.The said search warrant was base from theapplication of two persons. One is from the affidavit of Congressman Artemio Mate, also from 1 st district of Leyteand an NBI Agent Samuel Castro, alleging that the petitioner committed the crime of bribery.The affidavit of the latter applicant however showedthat he has no personal knowledge about the allegationsagainst the petitioner.The affidavit of the statements of the Congressmanalso shows that his allegations are anchored uponspeculations.No sufficient evidence is presented to therespondent judge.Issue:Was there an establishment of the existence of aprobable cause?Held:No. The allegations and the statements of theapplicants are merely anchored on hearsay andspeculations. As ingrained under jurisprudence (Roan vs.Gonzales as cited), in application for a search warrant, if based on hearsay, sannot, standing alone justify theissuance of search warrant. Thus, it is indispensable that theapplicants should have personal knowledge of the crimecommitted.In the case at bar, the deposition of the NBI agent isbased on hearsay in which only and anchored on theinformation given by Cong. Mata.Moreover, the deposition of Cong. Mata cannotestablish that he has a direct personal knowledge of

thealleged bribery of the petitioner since his affidavit shows thatit is only based on speculation. The element of directnessand definiteness is wanting so as to establish his personalknowledge. Not to mention, there is no concrete evidencethat would support their accusation, so as to validly establishprobable cause.Irregularities: 1 ) Moreover, there is also irregularity in the printing of the search warrant, wherein the crime of bribery Art2 10 of RPC was superimposed by ink, which wasoriginally Art. 2 8 2.In the case at bar, the search warrantpresented was in the case of grave threatsdirected against the nephew of thepetitioner (nephew), thus, the confiscationof he money is not related to the articlesseized.2) The search team also violated statutory guidelinesfor a lawful search since there is no members of thehousehold present while others are searching thepremises. Thus it is planted and orchestratedsearch.3) Also the respondents also violated the statutoryguidelines that they should issue a detailed receiptof articles seized.The lifting of the respondent judge of the search warrantare null and void. Presidential A ntiD ollar vs. C A (quasi-judicial body daw) Facts:The petitioner, PADS, is the presidents armassigned to investigate and prosecute :dollar-saltingactivities in the country pursuant to PD 1 936.Sometime 1 9 8 5, PADS issued a search warrantagainst respondent Karamfil Import-Export Co. et al. Atty.Gatmaitan of Bureau of Customs applied for a Searchwarrant, a deputized member of PADS with attached affidavitby Castro, an investigator and operative of PADS. After the search procedure, the respondentcontested the search warrant and subsequently declared bythe lower court null and void.The respondent CA initially favored the petitioner stating that it is a quasi judicial body that ranks with RTC andthat, lower courts has no jurisdiction to declare the issuedsearch warrants of the petitioners null and void. A motion for reconsideration was filed by thepetitioner Karamfil and subsequently, November of the sameyear, the respondent CA reversed itself.Thus, the petitioner filed an appeal alleging that therespondent CA committed grave abuse of discretion andacted in excess of its appellate jurisdiction by validating therestraining of the lawful orders or decrees issued by thepetitioner as a quasi judicial body by the lower Court. This isso since the petitioner contends that they are quasi-judicialbody that ranks with the RTC.Issue: As we have observed, the question is whether or not the Presidential Anti-Dollar Salting Task Force is, in thefirst place, a quasi-judicial body, and one whose decisionsmay not be challenged before the regular courts, other thanthe higher tribunals ---- the Court of Appeals and this Court.Held:Quasi-judicial body has been defined as "an organof government other than a court and other than a legislature,which affects the rights of private parties through either adjudication or rule making." Thus, these bodies has thebasic function to adjudicate claims and/or to determine rights,unless its decisions are appealed to proper reviewingauthorities.In the case at bar, the

PADS is not meant toexercise quasi judicial function, that is to decide and tryclaims and execute its judgments, its only task is to handlethe prosecution of salting or black marketing activities andnothing more. Constitutional Law II (Bill of Rights): Case Briefs: Dennis G. Libunao UC College of Law 20 However, the enabling statute of the petitioner empowers them to determine the existence of probablecause and therefore power to issue warrants of arrest or search and seizure however it does not make them co-equalwith RTC nor make this agency semi-court. Note: K inds of quasi-Judicial agencies: ( 1 ) Agencies created to function in situations wherein thegovernment is offering some gratuity, grant, or special privilege, like the defunct Philippine Veterans Board, Board on Pensions for Veterans, and NARRA, and PhilippineVeterans Administration. (2 ) Agencies set up to function in situations wherein thegovernment is seeking to carry on certain government functions, like the Bureau of Immigration, the Bureau of Internal Revenue, the Board of Special Inquiry and Board of Commissioners, the Civil Service Commission, the Central Bank of the Philippines. (3 ) Agencies set up to function in situations wherein thegovernment is performing some business service for the public, like the Bureau of Posts, the Postal Savings Bank,Metropolitan Waterworks & Sewerage Authority, PhilippineNational Railways, the Civil Aeronautics Administration. (4 ) Agencies set up to function in situations wherein thegovernment is seeking to regulate business affected with public interest, like the Fiber Inspections Board, thePhilippine Patent Office, Office of the InsuranceCommissioner. ( 5 ) Agencies set up to function in situations wherein thegovernment is seeking under the police power to regulate private business and individuals, like the Securities &Exchange Commission, Board of Food Inspectors, the Board of Review for Moving Pictures, and the Professional Regulation Commission. ( 6 ) Agencies set up to function in situations wherein thegovernment is seeking to adjust individual controversiesbecause of some strong social policy involved, such as theNational Labor Relations Commission, the Court of AgrarianRelations, the Regional Offices of the Ministry of Labor, theSocial Security Commission, Bureau of Labor Standards,Women and Minors Bureau. 3 1 P D 1936, the enabling statute of P A

DS isunconstitutional, reason. Under the constitution, the existence of probablecause is under the sole responsibility and discretion of a judge, who, must be neutral and prudent enough for hisexercise of conducting preliminary examination of the factsand circumstances of the case submitted by the fiscal.Presidential Anti-Dollar Salting Task Forceexercises, or was meant to exercise, prosecutorial powers,and on that ground, it cannot be said to be a neutral anddetached "judge" to determine the existence of probablecause for purposes of arrest or search. Unlike a magistrate, aprosecutor is naturally interested in the success of his case. Although his office "is to see that justice is done and notnecessarily to secure the conviction of the person accused,"he stands, invariably, as the accused's adversary and hisaccuser. To permit him to issue search warrants and indeed,warrants of arrest, is to make him both judge and jury in hisown right, when he is neither . That makes, to our mind and tothat extent, Presidential Decree No. 1 936 as amended byPresidential Decree No. 2 00 2, unconstitutional. S oliven vs. M akasiar ( A controversial case) T his case softens the doctrine laid down by the B A CHE case where the court states that the judge should personally depose the complainants and witnessesunder oath and in writing in determining the existence of probable cause. T his is a responsibility that should not be delegated to clerk of court or other authority. Facts:This is a consolidated petition for certiorari andprohibition to review the decision of the respondent JudgeRamon Makasiar.In the case filed by Beltran, he alleged that therespondent judge committed grave abuse of discretionamounting to lack or excess of jurisdiction when therespondent judge issued a warrant of arrest against thepetitioner for the crime of libel, without the respondent judgepersonally examining the complainant and witnesses for thedetermination of probable cause.The petitioner contend that the constitution requiresthat the judge should personally examine the complainantand/or witness for the determination of probable cause andtherefore issue an arrest warrant.Issue:Was the contention correct?Held:No. (Sadly)What the Constitution underscores is the exclusiveand personal responsibility of the issuing judge to satisfyhimself the existence of probable cause. In satisfying himself of the existence of probable cause for the issuance of awarrant of arrest, the judge is not required to personallyexamine the complainant and his witnesses. Followingestablished doctrine and procedure, he shall:( 1 ) Personally evaluate the report and the supportingdocuments submitted by the fiscal regarding theexistence of probable cause and, on the basisthereof, issue a warrant of arrest; or (2) If on the basis thereof he finds no probable cause,he may disregard the fiscal's report and requirethe submission of supporting affidavits of witnesses to aid him in arriving at a conclusion asto the existence of probable cause.Sound policy dictates this procedure,

otherwise judges would be unduly laden with the preliminaryexamination and investigation of criminal complaints insteadof concentrating on hearing and deciding cases filed beforetheir courts.On June 3 0 , 1 9 8 7, the Supreme Court unanimouslyadopted Circular No. 1 2, setting down guidelines for theissuance of warrants of arrest. The procedure thereinprovided is reiterated and clarified in this resolution.It has not been shown that respondent judge hasdeviated from the prescribed procedure. Thus, with regard tothe issuance of the warrants of arrest, a finding of graveabuse of discretion amounting to lack or excess of jurisdictioncannot be sustained. Pendon vs. C A ( S i m ilar C ase with the 20 th C enturyFox) Facts: Constitutional Law II (Bill of Rights): Case Briefs: Dennis G. Libunao UC College of Law 21 Sometime February of 1 9 8 7, 1 st Lt. Felipe Rojas,Officer in charge of the Philippine Constabulary - CriminalInvestigation Service (PC-CIS), Bacolod City, filed anapplication for a search warrant, alleging that KENERTrading is the possession of some NAPOCOR Properties,contrary to anti-fencing law.His application was subscribed before JudgeDemosthenes Magallanes of MTC Bacolod City supported bythe joint deposition of two (2) witnesses, Ignacio L. Reyes, anemployee of NAPOCOR (National Power Corporation) andIAI Eduardo Abaja of the CIS of Bacolod City.Search warrant was issued by the lower court andthen proceeded the search operation, seizing some articlesfrom the premises.The petitioner, Pendon, filed an application for thereturn of the articles seized by the PCCIS contending thatthe search warrant is illegally issued, which, however deniedby the lower court.The petitioner

subsequently filed a petition for mandamus certiorari and prohibition in the CA assailing thelegality of the Search Warrant, which also was denied by theCA stating that there is an existence of a probable cause.The petitioner contend that the applicants for theassailed warrants has no personal knowledge that thearticles sought to be seized were stolen.Moreover, he also alleged that there is irregularitieswith the contents of the joint deposition of the two witnessesand that there is no personal examination conducted by the judge as required by the law and the rules.Issue:Whether or not the search warrant is illegally issued.Held:Yes. The assailed search warrant was illegallyissued.The constitution provides that search warrants areissued based solely on probable cause. And in determiningprobable cause, it is required that 1 ) T he judge ( or) officer must examine thewitnesses personally;2) T he examination must be under oath; and 3) T he examination must be reduced to writing inthe form of searching questions and answers And also, such finding of probable cause should besubstantiated by the records.In the case at bar, We find that the requirementmandated by the law and the rules that the judge must personally examine the applicant and his witnesses in theform of searching questions and answers before issuing thewarrant , was not sufficiently complied with. The applicanthimself was not asked any searching question by JudgeMagallanes. The records disclose that the only part played bythe applicant, Lieutenant Rojas was to subscribe theapplication before Judge Magallanes . T he applicationcontained pre-typed questions, none of which stated that applicant had personal knowledge of a robbery or a theft and that the proceeds thereof are in the possession and control of the person against whom the search warrant was sought tobe issued . Affidavits of the complainants and witness are notsufficient for determining a probable cause.Pursuant to the Anti fencing law, the records alsoshows that the questions asked during the deposition cannotsupport the finding of the probable cause T here was also nostatement in the joint deposition that the articles sought to beseized were derived from the proceeds of the crime of robbery or a theft or that applicants have any knowledge that a robbery or theft was committed and the articles sought tobe seized were the proceeds thereof .Moreover, the items to be confiscated listed in theSW is couched on general terms, therefore considered as aprohibited general warrants. ( Galvanized bolts, aluminumwires and other Napocor tower and line parts and accessories) The items confiscated does not also fall under thepersonal properties that are allowed to be seized under thelaw.Thus the petition is granted and the articles areordered to be return since possession thereof is notprohibited by the law. Note: A

nti Fencing Law, commission of, in the case at bar, The offense which petitioner was sought to becharged was violation of the anti-fencing law which punishesthe act of any person who, with intent to gain for himself or for another, shall buy, receive, possess, keep, acquire,conceal, sell or dispose of, or shall buy or sell, or in any other manner deal in any article, item, object or anything of valuewhich he knows, or should have known to him, to have beenderived from the proceeds of the crime of robbery or theft ( Sec. 2 a, P.D. 1 6 1 2 ) P ersonal properties that may be confiscated inaccordance with the law, enumerate Section 2, Rule 1 26 of the Rules on Criminal Procedure a) T he subject of an offense;b) S tolen or embezzled property and other proceeds or fruits of an offense; and c) Used or intended to be used as a means of committing an offense. If the law does not prohibit the possession of the articlessought, it should be returned to the owner. People vs. Inting (si m ilar to D eLos S antos case)Preli m inary inquiry of the prosecutor does not bind the judge for the deter m ination of probable cause Facts:Sometime 1 9 88 , Editha Barba filed a letter complaint against OIC Mayor of Tanjay, Negros oriental withthe Comelec for transferring her to a remote barangaywithout prior clearance from the Comelec.The complaint was handled by the provincialelection Supervisor of Dumaguete City, atty. GerardoLituanas . after his investigation, he found prima facie

evidence and then filed to the respondent Trial court under Judge Enrique Inting a criminal case in violation of Sec. 26 1 of the Omnibus Election Code against the OIC Mayor. An arrest warrant was issued but later set aside onthe ground that Atty. Lituanas is not authorized to determinethe existence of probable cause. Hence this petition.Issue:Does the Provincial Election Supervisor of theComelec has the jurisdiction to find the existence of aprobable cause?Held:No. the phrase under Sec 2 Art II of the Constitutionwhich reads and such other responsible officer as may beauthorized by law has been deleted, making thedetermination for the existence of a probable cause under the sole responsibility and discretion of a Judge. Constitutional Law II (Bill of Rights): Case Briefs: Dennis G. Libunao UC College of Law 22 It should be distinguished that preliminaryinvestigation for the determination of a probable cause isvested with the authority of Judges (J udicial in nature) andwhile preliminary investigation in determining whether or notthe accused is guilty of the offense charged and thereforeshould be subjected to litigation is vested to the authority of the prosecutor ( executive in nature). The separate kinds of preliminary investigation hasbeen delineated and that the latter kind has been deleted tothe authorities of judges ( preliminary investigation proper). What the constitution vested the Comelec is toinvestigate and prosecute cases in violation of election laws(Art. IX Sec 2), however, this does not mean that the power to determine the existence of probable cause is within thescope of their authority, but rather, they are empowered onlyin purpose in assisting the Judge to determine the probablecause and for filing for an information. Note: The Prosecutor cannot assume roles in theprosecution of election offenses, if he has, it is because he isdeputized by the Comelec to handle such election offenses. U m il vs. Ra m os Facts:This is a consolidated petition for habeas corpus where the petitioners alleged that their detention is illegal andunlawful as their arrests were made without warrant and thatno preliminary investigation was first conducted, making theinformations filed against them are null and void.The respondents contends otherwise.In this consolidated case, all of the petitioners arecharged under the Anti Subversion Law, with an exception tothe case of Enrile vs. Lim (Inciting to sedition) and Nazarenovs. Station Commander.The rest are charged guilty of rebellion, a crimeagainst the State, and is a continuing crimes

in nature. Theywere found of the possession of unlicensed firearms andammunitions as well as subversive documents.Issue:Was warrantless arrest in the case at bar illegal, asthe arrest was not made pursuant to the constitutional andstatutory guidelines for the issuance of warrantless arrest?Held:Warrantless arrest conducted in the case at bar islawful.Pursuant to 5 Rule 11 3 of RRC, arrest of a personwithout a warrant of arrest or previous complaint isrecognized by law. The instances where a valid warrantlessarrest may be effected are the following. S ec5. A rrest without Warrant: When lawful A peaceofficer or a private person may, without warrant, arrest a person: ( a) When in his presence, the person to be arrested has committees, is actually committing, or isattempting to commit an offense. ( b) When an offense has in fact just beencommitted, and he has personal knowledge of the facts indicating that the person to bearrested has committed it; and (c) When the person to be arrested is a prisoner who has escaped from a penal establishment or place where he is serving final judgment or temporarily confined while his case is pending,or has escaped while being transferred from oneconfinement or another. Thus, pursuant to the RRC, warrantless arrest is justifiedwhen a person arrested is caught in flagranti delicto or inthe act of co mm itting an offense; or when an offense has just been co mm itted and the person m aking the arresthas personal knowledge of the facts indicating that theperson arrested has co mm itted it. The rational behind the concept o warrantless arrestis laid down in the PPI vs. Malasugui, that to sustain thatarresting a person without warrant illegal would leave thesociety, into a large extent, at the mercy of the shrewdest,the most expert and the most depraved of criminals,facilitating their escape in many instances. In the case at bar, the petitioners had freshlycommitted or were actually committing an offense.In the case of the petitioners who were charged of rebellion and inciting to sedition, the court held that they arelawfully detained and the informations filed against them arevalid.This is since the crime in which they are arrestedare continuous crime which is against the State, thus, thecontinued possession of subversive materials and unlicensedfirearms and ammunitions, even without preliminaryinvestigation and without warrant, as long as the authoritieshave confiscated such prohibited materials under their possession and that in the case of Enrile vs. Lim

, they hadactually done what is prohibited by law as long as they arepositively identified by a witness having a personalknowledge of the committed crime, their arrest are lawful.This is justified since under the doctrine of Garcia-Padilla vs. Enrile, persons arrested of rebellion does notneed to follow strict procedures, since their crimescommitted are violence against the S tate, whichconcerns the very survival of the society and government. In the case of Nazareno vs. station Commander, thewarrantless arrest is justified since there was a prior information filed against the petitioner for the same offense,and that he was positively identified by a witness who has apersonal knowledge about the crime he committed. He wasarrested while he was at large. Paderangs vs. D rilon Facts:Felizardo Roxas, was included as a co-accused inan amended information for the murder of Bucag et. al.The respondent state prosecutor Henrick Gingoyanwas designated by the DOJ to conduct the preliminaryinvestigation and directed to include the petitioner Roxas asa co-accused in the criminal case.Contending that the preliminary investigation wasnot yet completed so as to include Roxas as a Co-accused inthe case and that he was deprived of his right to present acorresponding counter-affidavit and additional evidencecrucial to the determination of the allege linkage to the crimecharged, the petitioner filed a Motion for reconsideration, butwas later denied. A petition for Review was subsequentlyfiled in the DOJ but was also denied.Thus, the petitioner filed a suit before the SCcontending that preliminary investigation is not yet completedthereby, there is no existing prima facie evidence or probablecause that would justify the petitioners inclusion to the crimecharged.Issue:Whether or not the preliminary investigation is theproper forum to present evidence to prove or disprove theguilt of the party.Held:No. A preliminary investigation is defined as an inquiry or proceeding for the purpose of determining whether there is Constitutional Law II (Bill of Rights): Case Briefs: Dennis G. Libunao UC College of Law 23 sufficient ground to engender a well founded belief that acrime cognizable by the Regional T rial Court has beencommitted and that the respondent is probably guilty thereof,and should be held for trial. The quantum of evidence now required inpreliminary investigation is such evidence sufficient to"engender a well founded belief" as to the fact of thecommission of a crime and the respondent's probable guiltthereof. A preliminary investigation is not the occasion for the full and exhaustive display of the parties' evidence; it isfor the presentation of such evidence only as may engender a well-grounded belief that an offense has been committed and that the accused is probably guilty thereof Preliminary investigation is generally inquisitorial,and it is often the only means of discovering the persons whomay be reasonably charged with a crime, to enable the fiscalto prepare his complaint or information. It is not a trial of thecase on the merits and has no purpose except that of determining whether a crime has been committed andwhether there is probable cause to believe that the accusedis guilty thereof, and it does not place the person againstwhom it is taken in jeopardy.

It does not mean absolute certainty. Innocenceof a party, whether merely proclaimed or realdoes not necessarily preclude a finding of PC.

[DOH v. Sy Chi Siong (1989) ]


People vs. Villanueva A clear difference between the 197 3 and the 1987 Constitutional provision on Sec 2 Art III ( responsible officers authorized by law) Facts:Sometime 1 9 80 , the petitioners, city fiscal andassistant City fiscal filed an information against Rogelim Yeewith serious slander by deed. The petitioners certified thatthey have already conducted a preliminary investigation andfind probable cause.Instead of issuing an arrest warrant, the respondentJudge, Napoleon Villanueva conducted an ex parte preliminary examination for scanning the records todetermine the existence of probable cause. After such examination, the respondent judge foundout that the crime committed may wither be slander by deedor slight physical injury. However, since the information wasfiled 64 days after the commission of the crime, the judgedismissed the case by prescription.Hence, this petition for certiorari and mandamus onthe ground that the respondent committed grave abuse of discretion amounting to lack or excess of jurisdiction.In his defense, the respondent judge contends thathe was not satisfied in the existence of probable cause andthat he needed to conduct a separate examination for itsdetermination and that, the case was dismissed because of prescription.Issue:Whether or not the judge may dismiss the caseoutright upon the absence of probable cause.Held:No.Under the 1

973 constitution, the judge has thepower and legal duty to determine the existence of probablecause, also, in cases where he is not satisfied with thecertification of the prosecutor in the information, he mayconduct preliminary investigation authorized under Sec 6Rule 11 2 of the RRC.But that power does not include the authority todismiss outright the information if the judge believes thatthere is no probable cause. The judge should require thefiscal to present additional evidence to show probable cause.If the fiscal refuses to do so, then the case may be dismissedfor "lack of prosecution" as also stated under the case of Amarga vs. Abbas.The fiscal is a "responsible officer authorized bylaw" within the meaning of Section 3 of the Bill of Rights. Hisdetermination of probable cause is a sufficient justification for the issuance of a warrant of arrest.Thus, it was held that "in a clash of views betweenthe judge who did not investigate and the fiscal who did, or between the fiscal and the offended party or the defendant,those of the fiscal's should normally prevail"Thus, during the 73 constitution, it is a rule that thecertification that probable cause exist by the preliminaryinvestigation of the prosecutor is a sufficient ground for a judge to issue probable cause. The judge, therefore shouldntconduct another examination.Note:Fiscals certification is sufficient, reason:The time-saving practice has been for the judge(municipal, city or Court of First Instance) to rely on thepreliminary investigation conducted by the fiscal as the basisfor issuing the warrant of arrest. That practice is supported bythe presumption that the fiscal performed his duties regularlyand competently (Edillon vs. Narvios, Administrative CaseNo. 1 753, August 2 1 , 1 9 80 , 99 SCRA 1 74). And that practiceexisted even under the old Constitution when Section 1 (3) of the Bill of Rights did not contain the terms "warrant of arrest"and "such other responsible officer as may be authorized bylaw."We hold that, as a rule, a trial judge should not holdanother preliminary examination to determine probable causein case the fiscal has filed an information and certified that hehas conducted the requisite preliminary investigation. Thatcertification means that there is a prima facie case againstthe accused and that the issuance of a warrant of arrest is justified. Placer vs. Villanueva Conflicting decision as far as the fiscals certification in theexistence of probable cause is concerned Facts:The petitioners, Fiscal and assistant City Fiscal, intheir submission of ten information in the court of therespondent, provides certification that probable cause existsand that there is a reasonable ground to believe that crimehas been committed and the accused are probably guiltythereof and therefore be subjected to arduous litigationprocess.(existence of probable cause and prima facie evidence)Following receipt of said informations, respondent judge issued an order setting on April 5, 1 9 8 2 the hearing of said criminal cases for the purpose of determining thepropriety of issuing the corresponding warrants of arrest. After said hearing, respondent issued the questioned ordersdated April

1 3, 1 5, 1 6 and 1 9, 1 9 8 2, requiring petitioners tosubmit to the court the affidavits of the prosecution witnesses Constitutional Law II (Bill of Rights): Case Briefs: Dennis G. Libunao UC College of Law 28 and other documentary evidence in support of theinformations to aid him in the exercise of his power of judicial review of the findings of probable cause by petitioners. Petitioners filed two separate motions for reconsideration of said orders, contending that under P.D.Nos. 77 and 9 11 , they are authorized to determine theexistence of a probable cause in a preliminaryexamination/investigation, and that their findings as to theexistence thereof constitute sufficient basis for the issuanceof warrants of arrest by the court. On April 2 8 , 1 9 8 2,respondent judge denied said motions and reiterated hisorder to petitioners to submit the supporting affidavits andother documents within five (5) days from notice.Hence this petition for certiorari and mandamus wasfiled.Issue:Whether or not the respondent city judge may, for the purpose of issuing a warrant of arrest, compel the fiscalto submit to the court the supporting affidavits and other documentary evidence presented during the preliminaryinvestigation.Held:NO. As dictated in the Amarga case, the certification of the fiscal may relied upon by the judge bit however notbinding for an automatic issuance of arrest warrant, this is so,since the issuance of warrants is not ministerial function, itcalls for the exercise of judicial discretion on the part of theissuing magistrate.Under Section 6, Rulle 11 2 of RRC the judge must satisfy himself of the existence of probable cause beforeissuing a warrant or order of arrest . I f on the face of theinformation the judge finds no probable cause , he maydisregard the fiscal's certification and require the submissionof the affidavits of witnesses to aid him in arriving at aconclusion as to the existence of a probable cause. This hasbeen the rule since U. S

. vs. Ocampo and A marga vs. A bbas . And this evidently is the reason for the issuance byrespondent of the questioned orders of April 1 3, 1 5, 1 6, 1 9, 1 9 8 2 and July 1 3, 1 9 8 2. Without the affidavits of theprosecution witnesses and other evidence which, as a matter of long-standing practice had been attached to theinformations filed in his sala, respondent found theinformations inadequate bases for the determination of probable cause. For as the ensuing events would show, after petitioners had submitted the required affidavits, respondentwasted no time in issuing the warrants of arrest in the caseswhere he was satisfied that probable cause existed. Note: C ases subjected to S u mm ary Procedures, reason for requiring the sub m ission of affidavits of the co m plainantand witnesses To enable the court to determine whether to dismissthe case or require further proceedings.Under the Rule on Summary Procedure in SpecialCases, the respondent judge has the power to order theoutright dismissal of the charge if, from the information andthe affidavits attached thereto, he finds the same to bepatently without basis or merit. I m portant: Re

m edial Law; C ri m inal ProcedureOn Probable cause:To be deter m ined by the Judge, The probable cause here to bedetermine by the judge is to whether to issuea search or arrest warrant or not . Notehowever that, though the judge finds probablecause, this does not mean that he shouldautomatically issue the same, in cases of anarrest warrant, he should determine whether there is a necessity to arrest the accused soas not to frustrate the ends of justice. Otherwise, the judge may refuse to issue anarrest warrant. To be deter m ined by the prosecutor The probable cause here is todetermine whether or not there is a crimecommitted and that the accused is probablyguilty thereof. This is not tantamount to thedetermination of guilt of the accused, what isimportant is that he is probably guiltythereof. Tolentino vs. Villaluz Facts:Sometime 1 973, the respondent Fiscal Mojica fileda complaint against the petitioners Bayot, Parra and Castillobefore the Circuit Criminal Court Court of Pasig under therespondent Judge, Onofre Villaluz, for violation of the Anti-Graft and Corrupt Practices Act. After preliminary examination and investigation, therespondent judge issued a resolution stating that; 1 . There exists a prima facie case against thepetitioners.2. Ordering as arrest warrant against the accused-petitioners.3. Ordering respondent Fiscal to conduct apreliminary examination and investigation in thiscase to determine the criminal liability of all themembers of the said City Council and thereafter tofile the corresponding information in the court of competent jurisdiction, if evidence so warrant. A subsequent motion to Dismiss was filed by thepetitioners but however was denied by the respondent court.Hence a petition for certiorari was filed by the petitionersenjoining the respondent Judge to take cognizance in thecriminal case filed against them and to declare all theproceedings undertaken and orders issued by the lower courtnull and void.They contend that the respondent judge has limited jurisdiction as a judge of the Circuit Criminal Court to try anddecided specific criminal cases. They allege that therespondent has no authority to conduct preliminaryinvestigations pursuant to RA 5 1

9.Issue:Does the limited jurisdiction of the respondentJudge also limits its authority as to the issuance of warrantsof arrest, determination of probable cause and conductingpreliminary investigation?Held:No.What is limited by RA 5 1 79 is only the scope of thecases tat maybe tried by Circuit Criminal Courts.These courts cannot try all criminal cases fallingunder the jurisdiction of the CFIs as courts of general jurisdiction. They can only try cases provided under section 1 of the said law. However, these does not follow that judgesunder these courts also has limited power and authority.They have the same authority and powers as those conferredupon regular CFIs. Constitutional Law II (Bill of Rights): Case Briefs: Dennis G. Libunao UC College of Law 29 Thus, the judges under these courts can alsoconduct preliminary investigations for the determination of prima facie case and of probable cause for the issuance of warrants. Note:Reason for the establishment of Circuit Criminal Courts. To alleviate the burden of the CFI and expedite thedisposition of case. Limiting the powers conferred to judgesof these courts would defeat such purpose, as the judges of the CFIs will still carry the burden to try and decide thesecases. C ruz vs. Gatan Facts:Serafin G. Cruz was arrested by PC agents on August 3 0 , 1 976, at the Baguio Checkpoint along KennonRoad, Baguio City, and brought to Camp Olivas, SanFernando, Pampanga, under the command of respondentGen. Romeo Gatan, for custodial interrogation, where he ispresently detained.On October 22, 1 976, a petition for the issuance of a writ of habeas corpus was filed in his behalf wherein it wasclaimed that the said Serafin Cruz is held incommunicado;that he is restrained of his liberty without due process of lawand is in the custody of the respondent not by virtue of a judgment or court order; that he is not a member of anysubversive organization covered by Proclamation No. 1081 and falls within the clays of persons to whom the privilege of the writ of habeas corpus has not been suspended.The petitioner is said to be an over-all Commander and Contractor General of the Bataan Defenders Command,an unregistered veterans outfit. He is thus allegedly violated Art. 1 47 of the RPC (Illegal Associations).Subsequently, the counsel for the petitioner however calimed after a discussion with the petitioner claimed that his continued detention is the free will andvolition of the petitioner who expressed fears that he mightbe harmed or injured by some members of the "BataanDefenders Command" if he were free from custody while themastermind and legal counsel of the association, one Atty.Cecilio Baylon Buenafe, has not yet been arrested.Issue:Was the petitioner illegally detained?Held:No. The privilege of writ of habeas corpus wassuspended by virtue of the declaration of PD 1081

.Moreover, the case (petition for habeas corpus) becamemoot and academic since the petitioner voluntarily give itsconsent for its detention due to threats.The petitioner in the instant case was arrested anddetained by virtue of an Arrest, Search, and Seizure Order issued by the Secretary of National Defense for violation of Article 1 47 of the Revised Penal Code pursuant to theaforequoted General Order No. 2-A, as amended; hence, hisarrest and continued detention is legal. The declaration of martial law and the consequent suspension of the privilege of the writ of habeas corpus with respect to persons reasonablybelieved or charged to be engaged in the disorder or infomenting it having been settled in the case of Aquino, Jr. vs.Ponce Enrile etc., et al., any inquiry by this Court into thecontinued detention of the petitioner would be purposeless. Geroni m ovs.Ra m os Facts:On January 8 , 1 9 80 , private respondent JulianPendre filed a petition with the Commission on Elections(COMELEC) to disqualify petitioner Meliton C. Geronimofrom running as a candidate for the mayorship of Baras, Rizalon the ground of political turncoatism. After hearing, the Comelec declared his adversaryfor the position of mayor in the said town Bayani Ferrera asduly elected mayor.The decision of the Comelec resulted into anger anddiscontent from the supporters of the petitioners, theyentered the municipal hall and stayed until May 1 3, 1 9 8 2.Subsequently, Pendre filed a petition before the SCpraying to cite the petitioner in contempt. After hearing, theComelec granted the petition, sentencing the petitioner tosuffer 5-month imprisonment with fine. As a result, the PC forcibly arrest the petitioner, acommotion between the PC and supporters of the petitionershappened on May 1 4, 1 9 8 2. The incident resulted to theissuance of 5 warrants of arrest against the supporters of thepetitioner for the following violations: Usurpation of Authority [Art. 177, Revised Penal Code ( RPC)]; Violation of Usurpation of Authority of Official ( sic) [Art. 177, Revised Penal Code

( RPC)]; T umultuous Affray [Art. 1 5 3 , RPC];Sedition ( Art. 1 3 9, RPC); Illegal Possession of Firearms;Disobedience to a Person in Authority or the Agent of suchPerson ( Art. 1 5 1, RPC) and Alarm and Scandal ( Art. 1 55 ,RPC). Thus three separate petitions were filed by thepetitioner. One of these prays for the dismissal of the 75complaints filed against the petitioner and his supporters.Issue:Was the issuance of 75 warrant in a single day validas a matter of procedure?Held:No.It is impossible for one Judge to finish thepreliminary investigation/examination of 75 persons in oneday. Similar caution is warranted for the issuance of warrants of arrest. In the case at bar and especially considering the background circumstances which led to thefiling of charges, we find it highly improbable for the judge tobe able to determine the existence of reasonable grounds tobelieve that the offenses have been committed and that eachand everyone of the seventy-six ( 7 6 ) persons are probably guilty thereof in a matter of a few hours and to proceed withthe issuance of the warrants of arrest also on the same day.It should be remembered and the judge should have takeninto account that all the offenses which were allegedly committed were only the product and result of the outburst of the feelings and emotions of the people of Baras due to thehighly tense situation in the municipality, which culminated with the May 1 4 , 198 2 incident. T he judge, therefore, inconducting his preliminary investigation should haveascertained with double care if, indeed, there was ampleevidence to warrant the issuance of arrest warrants and eventually the filing of criminal informations against such abig number of persons, most of whom were impelled by different motivations and whose respective participationswere of varying natures and degrees. One of the crimescharged was sedition, a particularly grave offense not to belightly treated by any prosecuting officer or judge. T

he possibility of prolonged detention because of the chargeshould have been considered. Thus, such a hasty and manifestly haphazardmanner of conducting the preliminary examination todetermine probable cause for the issuance of the warrants of Constitutional Law II (Bill of Rights): Case Briefs: Dennis G. Libunao UC College of Law 30 arrest and eventually for the filing of the necessaryinformation cannot be sanctioned by the Supreme Court. It is an undisputed fact that all the criminal chargeswere the result of the events that transpired before and until the May 1 4 , 198 2 incident, when Mr. Geronimo was forcibly taken out of the municipal building of Baras by the military. T he charges were also filed almost successively: one onMarch 20 ; one on April 1 2 ; one on April 1 4 ; two on May 4 ,two on May 1 4 , and one on May 19, - and with the samecourt and presided over by the same judge. In one of thecriminal complaints wherein about 7 5 people were charged,the warrants of arrest were issued on the same day that the preliminary examination was conducted. A judge ( Ricardo Javier in the case at bar) must firstsatisfy himself of the existence of probable cause beforeissuing a warrant or order of arrest. The requirements arestrict. The examination must be legitimate and not a feignedone intended to justify a course of action alreadypredetermined.Thus, because of the inherent impossibility of a judge to conduct preliminary examination to all 75 complaintsand finding probable cause to all of it resulting to hisissuance of warrants is hasty and haphazard. Note (V ery I mportant): Existing Ruling on determination of probable cause: Issuance of; S earch and

S eizure W arrant: The judge should personally examine the witnessesand complainant under oath or affirmance for thedetermination of probable cause. This is the doctrineembodied under Sec 2 Art III of the Constitution. A rrest W arrant: The judge is not required to personally examine thecomplainant and witnesses to determine the probable cause.It is enough that(a) He examines the report and supporting documentsprovided by the fiscal in the determination of probable cause, and in basis thereof, issue arrestwarrant.(b) If there is no sufficient establishment of probablecause, he may disregard the prosecutorscertification and require the submission of thesupporting affidavits of witnesses to aid him inarriving at a conclusion as to the existence of probable cause.

Juan Ponce Enrile vs Judge Salazar


Habeas Corpus Right to Bail Rebellion SC Cannot Change Law

In February 1990, Sen Enrile was arrested. He was charged together with Mr. & Mrs. Panlilio, and Honasan for the crime of rebellion with murder and multiple frustrated murder which allegedly occurred during their failed coup attempt. Enrile was then brought to Camp Karingal. Enrile later filed for the habeas corpus alleging that the crime being charged against him is nonexistent. That he was charged with a criminal offense in an information for which no complaint was initially filed or preliminary investigation was conducted, hence was denied due process; denied his right to bail; and arrested and detained on the strength of a warrant issued without the judge who issued it first having personally determined the existence of probable cause. ISSUE: Whether or not the court should affirm the Hernandez ruling. HELD: Enrile filed for habeas corpus because he was denied bail although ordinarily a charge of rebellion would entitle one for bail. The crime of rebellion charged against him however is complexed with murder and multiple frustrated murders the intention of the prosecution was to make rebellion in its most serious form so as to make the penalty thereof in the maximum. The SC ruled that there is no such crime as Rebellion with murder and multiple frustrated murder. What Enrile et al can be charged of would be Simple Rebellion because other crimes such as murder or all those that may be necessary to the commission of rebellion is absorbed hence he should be entitiled for bail. The SC however noted that a petition for habeas corpus was not the proper remedy so as to avail of bail. The proper step that should have been taken was for Enrile to file a petition to be admitted for bail. He should have exhausted all other efforts before petitioning for habeas corpus. The Hernandez ruling is still valid. All other crimes committed in

carrying out rebellion are deemed absorbed. The SC noted, however, that there may be a need to modify the rebellion law. Considering that the essence of rebellion has been lost and that it is being used by a lo t of opportunists to attempt to grab power.

PEOPLE V. MENGOTE [210 SCRA 174; G.R. NO. 87059; 22 JUN 1992] Friday, February 06, 2009 Posted by Coffeeholic Writes Labels: Case Digests, Political Law Facts: The Western Police District received a telephone call from an informer that there were three suspicious looking persons at the corner of Juan Luna and North Bay Boulevard in Tondo, Manila. A surveillance team of plainclothesmen was forthwith dispatched to the place. The patrolmen saw two men looking from side to side, one of whom holding his abdomen. They

approached the persons and identified themselves as policemen, whereupon the two tried to run but unable to escape because the other lawmen surrounded them. The suspects were then searched. One of them the accused-appellant was found with a .38 caliber with live ammunitions in it, while his companion had a fan knife. The weapons were taken from them and they were turned over to the police headquarters for investigation. An information was filed before the RTC convicting the accused of illegal possession of firearm arm. A witness testified that the weapon was among the articles stolen at his shop, which he reported to the police including the revolver. For his part, Mengote made no effort to prove that he owned the fire arm or that he was licensed to possess it but instead, he claimed that the weapon was planted on him at the time of his arrest. He was convicted for violation of P.D.1866 and was sentenced to reclusion perpetua. In his appeal he pleads that the weapon was not admissible as evidence against him because it had been illegally seized and therefore the fruit of a poisonous tree.

Issue: Whether or not the warrantless search and arrest was illegal.

Held: An evidence obtained as a result of an illegal search and seizure inadmissible in any proceeding for any purpose as provided by Art. III sec 32 of the Constitution. Rule 113 sec.5 of the Rules of Court, provides arrest without warrant lawful when: (a) the person to be arrested has committed, is actually committing, or is attempting to commit an offense, (b) when the offense in fact has just been committed, and he has personal knowledge of the facts indicating the person arrested has committed it and (c) the person to be arrested has escaped from a penal establishment or a place where he is serving final judgment or temporarily confined while his case is pending, or has escaped while being transferred from one confinement to another. These requirements have not been established in the case at bar. At the time of the arrest in question, the accused appellant was merely looking from side to side and holding his abdomen, according to the arresting officers themselves. There was apparently no offense that has just been committed or was being actually committed or at least being attempt by Mengote in their presence. Moreover a person may not be stopped and

frisked in a broad daylight or on a busy street on unexplained suspicion. Judgment is reversed and set aside. Accused-appellant is acquitted. Warrantless Search and Seizure-Whenvalid or not? People vs. Go Facts: After an information was relayed to the intelligenceand Follow Up Unit of Calamba Police that shabu or methamphetamine hydrochloride is supplied in Crossing,Calamba Laguna, agents went in the place for follow up. A police agent, Panuringan, then reported that hesaw the accused, Luisito Go, enter a disco House with twowomen with a gun tucked in the accused waist.Three policemen entered the disco House for the Operation Bakal to search for illegally possessed firearms.Thus, when the accused was ordered to stand-up, policeagents saw a revolver tucked in his waist, they asked for itslicensed but the accused never showed it, instead, he gavehis drivers license bearing other name. The police agentsconfiscated the gun with 10 live ammunitions.The agents also confiscated glass toothers and tinfoils in the accused-appellants car together with an allegedshabu wrapped in cellophane. The accused was arrestedwithout search and arrest warrants.Thus, two information were filed against theaccused, one in violation of the Dangerous Drugs Act (RA6452) and violation to PD1866 or the Illegal Possession of firearms and ammunitions.The accused assails the validity of his arrest and hissubsequent convictions for two crimes.Issue:Was the warrantless arrest valid?Held:Yes. It is valid.Rules of Court and Jurisprudence recognizeexceptional cases where an arrest may be effected withoutwarrant, to wit;(a) In the presence of a peace officer, the person to bearrested has committed, is actually committing, or isattempting to commit an offense(b) When an offense has in fact just been committedand the arresting officer has personal knowledge of facts indicating that the person to be arrested hascommitted it.(c) When the accused is a fugitive from law whileserving his sentence from a final conviction.In the case at bar, it is clear that the policemen sawa gun tucked in his waist when he was arrested. The gunis plainly visible and no corresponding license wasfound. Thus, it is a valid search under the RRC ( Plainview doctrine) no arrest warrant is necessary.The case also falls under the recognize instancesprovided under the RRC ( Search incidental to lawfularrest), thus, the police has the authority to search theaccused for dangerous drugs or weapons that is used asproof to the commission of the offense. The incidentaldiscovery and confiscation of the drug paraphernalia tohis car is lawful. Note: Bail; Criminal Procedure: Admission for bail does not preclude the accused toraise question on the validity of his arrest, as long as heraised the same before he enters plea, otherwise, it will bar him from questioning the same as he place himself alreadyunder the jurisdiction of the court. Absence of Preliminary investigation is not a groundto quash the information.

PITA VS CA

FACTS: Pursuant to the Anti-Smut Campaign of Mayor Ramon Bagatsng, policemen seized and confiscated from dealers, distributors, newsstand owners and peddlers along Manila sidewalks, magazines, publications and other reading materials believed to be obscene, pornographic, and indecent and later burned the seized materials in public. Among the publications seized and later burned was "Pinoy Playboy" magazines published and co-edited by plaintiff Leo Pita. After his injunctive relief was dismissed by the RTC and his appeal rejected by CA, he seeks review with SC, invoking the guaranty against unreasonable searches and seizure. Issue: W/N the search and seizure was illegal HELD: YES. It is basic that searches and seizure may be done only through a judicial warrant , otherwise, they become unreasonable and subject to challenge. In Burgos v Chief of Staff (133 SCRA 800) , the SC countermanded the orders of the RTC authorizing the serach of the premises WE Forum and Metropolitan Mail, two Metro Manila Dailies, by reason of a defective warrant. There is

a greater reason in this case to reprobate the questioned raid, in the complete absence of a warrant, valid or invalid. The fact that the instant case involves an obscenity rap makes it no different from Burgos, a political case, because speech is speech, whether political or "obscene". The authorities must apply for the issuance of the a search warrant from the judge , if in their opinion, an obscenity rap is in order. They must convince the court that the materials sought to be seized are "obscene" and pose a clear and present danger of an evil substantive enough to warrant State interference and action. The judge must determine WON the same are indeed "obscene": the question is to be resolved on a case-to-case basis and on the judge's sound discretion. If probable cause exist, a search warrant will issue.
G.R. No. 163087 February 20, 2006 SILAHIS INTERNATIONAL HOTEL, INC. and JOSE MARCEL PANLILIO, Petitioners, vs. ROGELIO S. SOLUTA, JOSELITO SANTOS, EDNA BERNATE, VICENTA DELOLA, FLORENTINO MATILLA, and GLOWHRAIN-SILAHIS UNION CHAPTER, Respondents. FACTS: Loida Somacera (Loida), a laundrywoman of the hotel, stayed overnight at the female locker room at the basement of the hotel. At dawn, she heard pounding sounds outside, she saw five men in barong tagalog whom she failed to recognize but she was sure were not employees of the hotel, forcibly opening the door of the union office. In the morning, as union officer Soluta was trying in vain to open the door of the union office, Loida narrated to him what she had witnessed at dawn. Soluta immediately lodged a complaint before the Security Officer. And he fetched a locksmith. At that

instant, men in barong tagalog armed with clubs arrived and started hitting Soluta and his companions. Panlilio thereupon instructed Villanueva to force open the door, and the latter did. Once inside, Panlilio and his companions began searching the office, over the objection of Babay who even asked them if they had a search warrant. A plastic bag was found containing marijuana flowering tops. As a result of the discovery of the presence of marijuana in the union office and after the police conducted an investigation of the incident, a complaint against the 13 union officers was filed before the Fiscals Office of Manila. RTC acquitted the accused. On appeal, the CA affirmed with modification the decision of the trial court. ISSUE: Whether respondent individual can recover damages for violation of constitutional rights. RULING: Article 32, in relation to Article 2219(6) and (10) of the Civil Code, allows so. ART. 32. Any public officer or employee, or any private individual, who directly or indirectly obstructs, defeats, violates or in any manner impedes or impairs any of the following rights and liberties of another person shall be liable to the latter for damages: x x x x In the present case, petitioners had, by their own claim, already received reports in late 1987 of illegal activities and Maniego conducted surveillance. Yet, in the morning of January 11, 1988, petitioners and their companions barged into and searched the union office without a search warrant, despite ample time for them to obtain one. The course taken by petitioners and company stinks in illegality. Petitioners violation of individual respondents constitutional right against unreasonable search thus furnishes the basis for the award of damages under Article 32 of the Civil Code. For respondents, being the lawful occupants of the office had the right to raise the question of validity of the search and seizure. Article 32 speaks of an officer or employee or person "directly or indirectly" responsible for the violation of the constitutional rights and liberties of another. Hence, it is not the actor alone who must answer for damages under Article 32; the person indirectly responsible has also to answer for the damages or injury caused to the aggrieved party. Such being the case, petitioners, together with Maniego and Villanueva, the ones who orchestrated the illegal search, are jointly and severally liable for actual, moral and exemplary damages to herein individual respondents in accordance with the earlier-quoted pertinent provision of Article 32, in relation to Article 2219(6) and (10) of the Civil Code which provides: Art. 2219. Moral damages may be recovered in the following and analogous cases, among others, (6) Illegal search and (10) Acts and action referred to in Articles 21, 26, 27, 28, 29, 30, 32, 34 and 35. DECISION: Denied.

PEOPLE OF THE PHILIPPINES, plaintiff-appellee vs ANDRE MARTI,accused-appellant . G.R.No.81561 January 18, 1991 FACTS: Accused-appellant went to a forwarding agency to send four packages to a friend inZurich. Initially, the accused was asked by the proprietress if the packages can beexamined. However, he refused. Before delivering said packages to the Bureau of Customs and the Bureau of Posts, the husband of the proprietress opened said boxes for final inspection. From that inspection, included in the standard operating procedure andout of curiosity, he took several grams of its contents.He brought a letter and the said sample to the National Bureau of Investigation. When the NBI was informed that the rest of the shipment was still in his office, three agents went back with him. In their presence, the husband totally opened the packages. Afterwards,the NBI took custody of said packages. The contents, after examination by forensicchemists, were found to be marijuana flowering tops.The appellant, while claiming his mail at the Central Post Office, was invited by theagents for questioning. Later on, the trial court found him guilty of violation of theDangerous Drugs Act. ISSUES: 1.Whether or not the items admitted in the searched illegally searched and seized.2. Whether or not custodial investigation was not properly applied.3. Whether or not the trial court did not give credence to the explanation of the appellanton how said packages came to his possession. HELD: 1. No. The case at bar assumes a peculiar character since the evidence sought to beexcluded was primarily discovered and obtained by a private person, acting in a private capacity and without the intervention and participation of State authorities.Under the circumstances, can accused/appellant validly claim that his constitutionalright against unreasonable searches and seizure has been violated. Stated otherwise,may an act of a private individual, allegedly in violation of appellant's constitutionalrights, be invoked against the State. In the absence of governmental interference, theliberties guaranteed by the Constitution cannot be invoked against the State. It wasMr. Job Reyes, the proprietor of the forwarding agency, who made search/inspectionof the packages. Said inspection was reasonable and a standard operating procedureon the part of Mr. Reyes as a precautionary measure before delivery of packages tothe Bureau of Customs or the Bureau of Posts. Second, the mere presence of the NBIagents did not convert the reasonable search effected by Reyes into a warrantlesssearch and seizure proscribed by the Constitution. Merely to observe and look at thatwhich is in plain sight is not a search. Having observed that which is open, where notrespass has been committed in aid thereof, is not search.2. No. The law enforcers testified that accused/appellant was informed of hisconstitutional rights. It is presumed that they have regularly performed their duties(See. 5(m), Rule 131) and their testimonies should be given full faith and credence,there being no evidence to the contrary.3. No. Appellant signed the contract as the owner and shipper thereof giving moreweight to the presumption that things which a person possesses, or exercises acts of ownership over, are

owned by him (Sec. 5 [j], Rule 131). At this point, appellant istherefore estopped to claim otherwise.

G.R. No. 185011

December 23, 2009

PEOPLE OF THE PHILIPPINES, Plaintiff-Appellee, vs. SPO3 SANGKI ARA y MIRASOL, MIKE TALIB y MAMA, and JORDAN MUSA y BAYAN, Accused-Appellants. DECISION VELASCO, JR., J.: This is an appeal from the December 13, 2007 Decision of the Court of Appeals (CA) in CAG.R. CR-H.C. No. 00025B entitled People of the Philippines v. SPO3 Sangki Ara y Mirasol, Mike Talib y Mama, Jordan Musa y Bayan, which affirmed the Decision of the Regional Trial Court (RTC), Branch 9 in Davao City, convicting accused-appellants of violation of Republic Act No. (RA) 9165 or the Comprehensive Dangerous Drugs Act of 2002. The Facts Three Informations charged accused-appellants Sangki Ara, Mike Talib, and Jordan Musa, as follows: Criminal Case No. 51,471-2002 against Ara That on or about December 20, 2002, in the City of Davao, Philippines and within the jurisdiction of this Honorable Court, the above-named accused, without being authorized by law, willfully, unlawfully and consciously traded, transported and delivered 26.6563 grams of Methamphetamine Hydrochloride or "shabu," which is a dangerous drug, with the aggravating circumstance of trading, transporting and delivering said 26.6563 grams of "shabu" within 100 meters from [the] school St. Peter's College of Toril, Davao City. CONTRARY TO LAW.1 Criminal Case No. 51,472-2002 against Talib That on or about December 20, 2002, in the City of Davao, Philippines and within the jurisdiction of this Honorable Court, the above-named accused, without being authorized by law, willfully, unlawfully and consciously had in his possession and control one (1) plastic sachet of Methamphetamine Hydrochloride or "shabu," weighing 0.3559 gram, which is a dangerous drug. CONTRARY TO LAW.2

Criminal Case No. 51,473-2002 against Musa That on or about December 20, 2002, in the City of Davao, Philippines, and within the jurisdiction of this Honorable Court, the above-mentioned accused, without being authorized by law, willfully, unlawfully and consciously had in his possession and control five (5) big plastic sachet[s] of Methamphetamine Hydrochloride or "shabu" weighing 14.2936 grams, which is a dangerous drug. CONTRARY TO LAW.3 During their arraignment, accused-appellants all gave a "not guilty" plea. Version of the Prosecution At the trial, the prosecution presented the following witnesses: Forensic Chemist Noemi Austero, PO2 Ronald Lao, SPO1 Bienvenido Furog, PO1 Enrique Ayao, Jr., SPO4 Rodrigo Mallorca, and PO2 Jacy Jay Francia. In the morning of December 20, 2002, a confidential informant (CI) came to the Heinous Crime Investigation Section (HCIS) of the Davao City Police Department and reported that three (3) suspected drug pushers had contacted him for a deal involving six (6) plastic sachets of shabu. He was instructed to go that same morning to St. Peter's College at Toril, Davao City and look for an orange Nissan Sentra car.4 Police Chief Inspector Fulgencio Pavo, Sr. immediately formed a buy-bust team composed of SPO3 Reynaldo Capute, SPO4 Mario Galendez, SPO3 Antonio Balolong, SPO2 Arturo Lascaos, SPO2 Jim Tan, SPO1 Rizalino Aquino, SPO1 Bienvenido Furog, PO2 Vivencio Jumawan, Jr., PO2 Ronald Lao, and PO1 Enrique Ayao, Jr., who would act as poseur-buyer.5 The team proceeded to the school where PO1 Ayao and the CI waited by the gate. At around 8:45 a.m., an orange Nissan Sentra bearing plate number UGR 510 stopped in front of them. The two men approached the vehicle and the CI talked briefly with an old man in the front seat. PO1 Ayao was then told to get in the back seat as accused-appellant Mike Talib opened the door. The old man, later identified as accused-appellant SPO3 Ara, asked PO1 Ayao if he had the money and the latter replied in the positive. Ara took out several sachets with crystalline granules from his pocket and handed them to PO1 Ayao, who thereupon gave the pre-arranged signal of opening the car door. The driver of the car, later identified as accused-appellant Jordan Musa, tried to drive away but PO1 Ayao was able to switch off the car engine in time. The back-up team appeared and SPO1 Furog held on to Musa while PO2 Lao restrained Talib. PO1 Ayao then asked Ara to get out of the vehicle.6 Recovered from the group were plastic sachets of white crystalline substance: six (6) big sachets, weighing 26.6563 grams, from Ara by PO1 Ayao; five (5) big sachets, weighing 14.2936 grams, from Musa by SPO1 Furog; and a small sachet, weighing 0.3559 gram, from Talib by PO2 Lao.7

The three suspects were brought to the HCIS and the seized items indorsed to the Philippine National Police (PNP) Crime Laboratory for examination. Forensic Chemist Austero, who conducted the examination, found that the confiscated sachets all tested positive for shabu.8 Version of the Defense The defense offered the sole testimony of Ara, who said that he had been a member of the PNP for 32 years, with a spotless record. On December 20, 2002, SPO3 Ara was in Cotabato City, at the house of his daughter Marilyn, wife of his co-accused Musa. He was set to go that day to the Ombudsman's Davao City office for some paperwork in preparation for his retirement on July 8, 2003. He recounted expecting at least PhP 1.6 million in retirement benefits.9 Early that morning, past three o'clock, he and Musa headed for Davao City on board the latter's car. As he was feeling weak, Ara slept in the back seat. Upon reaching Davao City, he was surprised to see another man, Mike Talib, in the front seat of the car when he woke up. Musa explained that Talib had hitched a ride on a bridge they had passed.10 When they arrived in Toril, Ara noticed the car to be overheating, so they stopped. Ara did not know that they were near St. Peter's College since he was not familiar with the area. Talib alighted from the car and Ara transferred to the front seat. While Talib was getting into the back seat, PO1 Ayao came out of nowhere, pointed his .45 caliber pistol at Ara even if he was not doing anything, and ordered him to get off the vehicle. He saw that guns were also pointed at his companions. As the group were being arrested, he told PO1 Ayao that he was also a police officer. Ara insisted that he was not holding anything and that the shabu taken from him was planted. He asserted that the only time he saw shabu was on television.11 The Ruling of the Trial Court The RTC pronounced accused-appellants guilty of the crimes charged. In its Decision dated March 1, 2003, the trial court held that the prosecution was able to establish the quantum of proof showing the guilt of accused-appellants beyond reasonable doubt. It further ruled that the "intercept operation" conducted by the buy-bust team was valid. The dispositive portion of the RTC Decision reads: WHEREFORE, premised on the foregoing the Court finds the following: In Criminal Case No. 51,471-2002, the accused herein SANGKI ARA Y MASOL, Filipino, 55 years old, widower, a resident of Kabuntalan, Cotabato City, is hereby found GUILTY beyond reasonable doubt, and is CONVICTED of the crime of violation of Sec. 5, 1st paragraph of Republic Act 9165. He is hereby imposed the DEATH PENALTY and FINE of TEN MILLION PESOS (PhP 10,000,000) with all the accessory penalties corresponding thereto, including absolute perpetual disqualification from any public office, in view of the provision of section 28 of RA 9165 quoted above.

Since the prosecution proved beyond reasonable doubt that the crime was committed in the area which is only five (5) to six (6) meters away from the school, the provision of section 5 paragraph 3 Article II of RA 9165 was applied in the imposition of the maximum penalty against the herein accused. In Criminal Case No. 51,472-2002, the accused herein MIKE TALIB y MAMA, Filipino, of legal age, single and a resident of Parang, Cotabato, is found GUILTY beyond reasonable doubt, and is CONVICTED of the crime of violation of Sec. 11, 3rd paragraph, Article II of Republic Act 9165. He is hereby imposed a penalty of Imprisonment of SIXTEEN (16) YEARS and a fine of THREE HUNDRED THOUSAND PESOS (PhP 300,000) with all the accessory penalties corresponding thereto. In Criminal Case No. 51,473-2002 the accused herein JORDAN MUSA Y BAYAN, Filipino, 30 years old, married and a resident of Cotabato City, is hereby found GUILTY beyond reasonable doubt and is CONVICTED of the crime for Violation of Sec. 11, 1st paragraph, Article II of Republic Act No. 9165. He is hereby sentenced to suffer a penalty of LIFE IMPRISONMENT and FINE of FOUR HUNDRED THOUSAND PESOS (PhP 400,000) with all the accessory penalties corresponding thereto. SO ORDERED.12 As the death penalty was imposed on Ara, the case went on automatic review before this Court. Conformably with People v. Mateo,13 we, however, ordered the transfer of the case to the CA. The Ruling of the Appellate Court Contesting the RTC Decision, accused-appellants filed separate appeals before the CA. Talib claimed that it was erroneous for the trial court to have used the complaining witnesses' affidavits as basis for ruling that their arrest was valid. He also cited as erroneous the trial court's refusal to rule that the prosecution's evidence was inadmissible. Lastly, he questioned the failure of the buy-bust team to follow the requirements of RA 9165 on proper inventory of seized drugs. Ara and Musa filed a joint brief, alleging the following: (1) the trial court erred in denying the Motion to Suppress and/or exclude illegally obtained evidence; (2) the trial court erred in denying the Demurrer to Evidence; (3) the trial court failed to consider that the criminal informations did not allege conspiracy among the accused; and (4) the trial court erred in ruling that the "intercept operation" was valid. The CA affirmed the trial court's decision with some modifications on the penalty imposed. It ruled that a majority of the errors raised in the appeal referred to technicalities in the conduct of buy-bust operations that did not invalidate the police officers' actions. On the issue of the evidence presented, the CA held that the presumption that police officers performed their duties in a regular manner was not overturned. The appellate court resolved the issue of the validity of the buy-bust operation by stating that the law requires no specific method of conducting such an operation. It ruled that to require a

warrant of arrest would not accomplish the goal of apprehending drug pushers in flagrante delicto. The CA's Decision emphasized that all the elements necessary for the prosecution of illegal sale of drugs were established. The fallo of the December 13, 2007 CA Decision reads: WHEREFORE, premises foregoing, the appeal is hereby DISMISSED and the appealed March 1, 2003 Decision is hereby AFFIRMED subject to the modification insofar as the death penalty imposed upon accused SPO3 Sangki Ara is concerned. Accordingly, his penalty is hereby reduced to life imprisonment pursuant to Republic Act No. 9346. SO ORDERED.14 On December 17, 2008, this Court required the parties to submit supplemental briefs if they so desired. The parties, save for Musa, manifested their willingness to forego the filing of additional briefs. The Issues Reiterating the matters raised before the CA, accused-appellants alleged the following: I Whether the Court of Appeals erred in holding that the arrest of the accused-appellants was valid based on the affidavits of the complaining witnesses II Whether the Court of Appeals erred in disregarding the apparent defects and inconsistencies in the affidavits of the complaining witnesses III Whether the Court of Appeals erred in refusing to consider the suppression or exclusion of evidence IV Whether the Court of Appeals erred in not holding that the prosecution miserably failed to prove the guilt of the accused beyond reasonable doubt Talib also raises the following grounds for his acquittal: I Whether the arrest of Talib was illegal and the evidence confiscated from him illegally obtained

II Whether the police officers who conducted the illegal search and arrest also deliberately failed and/or violated the provisions of RA 9165 III Whether the testimonies of the prosecution's witnesses and their respective affidavits were gravely inconsistent Ara and Musa additionally raise the following issues: I Whether the trial court erred in denying the Demurrer to Evidence II Whether the trial court failed to consider that the criminal informations did not allege conspiracy among the accused III Whether the trial court erred in ruling that the "intercept operation" was valid Accused-appellant Musa also avers that the CA erred in convicting him since the prosecution failed to prove the corpus delicti of the offense charged. The Ruling of this Court What are mainly raised in this appeal are (1) whether the buy-bust conducted was valid; (2) whether the crimes of illegal sale and illegal possession of drugs were sufficiently established; and (3) whether the chain of custody over the shabu was unbroken. Warrantless Arrest and Seizure Valid In calling for their acquittal, accused-appellants decry their arrest without probable cause and the violation of their constitutional rights. They claim that the buy-bust team had more than a month to apply for an arrest warrant yet failed to do so. Owing to the special circumstances surrounding the drug trade, a buy-bust operation has long been held as a legitimate method of catching offenders. It is a form of entrapment employed as an effective way of apprehending a criminal in the act of commission of an offense.15 We have ruled that a buy-bust operation can be carried out after a long period of planning. The period of planning for such operation cannot be dictated to the police authorities who are to undertake such operation.16 It is unavailing then to argue that the operatives had to first secure a warrant of arrest

given that the objective of the operation was to apprehend the accused-appellants in flagrante delicto. In fact, one of the situations covered by a lawful warrantless arrest under Section 5(a), Rule 113 of the Rules of Court is when a person has committed, is actually committing, or is attempting to commit an offense in the presence of a peace officer or private person. It is erroneous as well to argue that there was no probable cause to arrest accused-appellants. Probable cause, in warrantless searches, must only be based on reasonable ground of suspicion or belief that a crime has been committed or is about to be committed. There is no hard and fast rule or fixed formula for determining probable cause, for its determination varies according to the facts of each case.17 Probable cause was provided by information gathered from the CI and from accused-appellants themselves when they instructed PO1 Ayao to enter their vehicle and begin the transaction. The illegal sale of shabu inside accused-appellants' vehicle was afterwards clearly established. Thus, as we have previously held, the arresting officers were justified in making the arrests as accused-appellants had just committed a crime when Ara sold shabu to PO1 Ayao.18 Talib and Musa were also frisked for contraband as it may be logically inferred that they were also part of Ara's drug activities inside the vehicle. This inference was further strengthened by Musa's attempt to drive the vehicle away and elude arrest. Moreover, the trial court correctly denied the Motion to Suppress or Exclude Evidence. We need not reiterate that the evidence was not excluded since the buy-bust operation was shown to be a legitimate form of entrapment. The pieces of evidence thus seized therein were admissible. As the appellate court noted, it was within legal bounds and no anomaly was found in the conduct of the buy-bust operation. There is, therefore, no basis for the assertion that the trial court's order denying said motion was biased and committed with grave abuse of discretion. Prosecution Established Guilt Beyond Reasonable Doubt For the successful prosecution of the illegal sale of shabu, the following elements must be established: (1) the identity of the buyer and the seller, the object of the sale, and the consideration; and (2) the delivery of the thing sold and its payment. What is material is the proof that the transaction or sale actually took place, coupled with the presentation in court of the corpus delicti as evidence.19 All these requisites were met by the prosecution. In contrast, Ara, the sole defense witness, could only proffer the weak defenses of denial and alibi. He expressed surprise at having Talib in his car and claimed he was framed and that the shabu confiscated from him was planted. According to the trial court, however, Ara's lying on the witness stand "was so intense as he tried very hard in vain to win the Court's sympathy."20 Given the prosecution's evidence, we rule that the presumption of regularity in the performance of official duties has not been overturned. The presumption remains because the defense failed to present clear and convincing evidence that the police officers did not properly perform their duty or that they were inspired by an improper motive.21 Ara could not explain why his fellow police officers, who did not know him prior to his arrest, would frame him for such a serious offense. Validity of Buy-Bust Operation

Likewise questioned by the defense in the affidavits of the police officers was the allegation that there was a legitimate buy-bust operation. No marked money was presented to back up the police officers' claims. This argument lacks basis, however. There are requirements that must be complied with in proving the legitimacy of drug buy-bust operations. Nevertheless, this Court has ruled that presentation of the marked money used is not such a requirement. In the prosecution for the sale of dangerous drugs, the absence of marked money does not create a hiatus in the evidence for the prosecution, as long as the sale of dangerous drugs is adequately proved and the drug subject of the transaction is presented before the court.22 In the instant case, the police officers' testimonies adequately established the illegal sale of shabu. The shabu was then presented before the trial court. The non-presentation of the marked money may, thus, be overlooked as a peripheral matter. Talib further contends that it is incredible that a shabu transaction would be carried out in a very open and public place. Contrary to Talib's claim, however, judicial experience has shown that drug transactions have been conducted without much care for an inconspicuous location. Thus, we observed in People v. Roldan: Drug pushing when done on a small level x x x belongs to that class of crimes that may be committed at anytime and at any place. After the offer to buy is accepted and the exchange is made, the illegal transaction is completed in a few minutes. The fact that the parties are in a public place and in the presence of other people may not always discourage them from pursuing their illegal trade these factors may even serve to camouflage the same. Hence, the Court has sustained the conviction of drug pushers caught selling illegal drugs in a billiard hall, in front of a store, along a street at 1:45 p.m., and in front of a house.23 It is also argued as impossible to believe that even if there was already a deal between the informant and accused-appellants, it was the apprehending police officer who acted as the buyer and that he requested to see the shabu first before showing the money. These claims by Talib are similarly undeserving of consideration. First, there is no uniform method by which drug pushers and their buyers operate. Second, the choice of effective ways to apprehend drug dealers is within the ambit of police authority. Police officers have the expertise to determine which specific approaches are necessary to enforce their entrapment operations.24 Third, as long as they enjoy credibility as witnesses, the police officers' account of how the buy-bust operation transpired is entitled to full faith and credit. 25 Lastly, these arguments are merely incidental and do not affect the elements of the crime which have been, in the instant case, sufficiently established. Talib also alleges that during his testimony, SPO1 Furog was not certain as to the reason he was apprehending Musa. Another claim is that SPO1 Furog, when examined by the prosecutor and two different defense lawyers, allegedly made relevant inconsistencies in his testimony. The pertinent exchange reads: Direct Examination of SPO1 Furog: Prosecutor Weis:

Q What was your basis for stopping [Musa] from letting the car go? A I made him [stop] the car[.] [W]e [had] to check them first because I think Ayao saw [that] Ara [had] the suspected shabu. Cross-Examination of SPO1 Furog: Atty. Estrada Q When you arrested Musa as you said, it was because he attempted to drive the car away, that was it? A The most, when SPO3 Sangki Ara told us that he was a PNP member and when we saw the substances from the two of them first. xxxx Q You are referring to Musa and Ara? A Yes sir.26 xxxx Atty. Javines Q Ayao did not arrest [Ara] inside the vehicle? A Only I rushed to the vehicle. I don't know if he directly arrested him when he saw the substance and [got] out of the vehicle but I saw him get out from the vehicle.27 The alleged inconsistencies in SPO1 Furog's "reason for apprehending Musa" are, however, insignificant and do not merit much consideration as well. The questioned parts in the testimony of SPO1 Furog do not dent the totality of evidence against accused-appellants. To repeat, the elements of the crime of illegal sale of drugs and illegal possession of drugs were both sufficiently established. Although SPO1 Furog was not categorical in explaining his basis for apprehending Musa, the arrest of the latter must be considered as part of a legitimate buy-bust operation which was consummated. Musa's arrest came after the pre-arranged signal was given to the back-up team and this served as basis for the police officers to apprehend all those in the vehicle, including Musa. Denial of Demurrer to Evidence Although alleged by accused-appellants Ara and Musa, no reason was given in the appeal as to why the trial court erred in denying their Demurrer to Evidence. Whatever their basis may be, an action on a demurrer or on a motion to dismiss rests on the sound exercise of judicial discretion.28 In Gutib v. CA,29 we explained that:

A demurrer to evidence is an objection by one of the parties in an action, to the effect that the evidence which his adversary produced is insufficient in point of law, whether true or not, to make out a case or sustain the issue. The party demurring challenges the sufficiency of the whole evidence to sustain a verdict. The court, in passing upon the sufficiency of the evidence raised in a demurrer, is merely required to ascertain whether there is competent or sufficient evidence to sustain the indictment or to support a verdict of guilt. Here, the trial court found competent and sufficient evidence to support a conviction of all three accused-appellants. We see no reason to overturn the trial court's finding. Allegation of Conspiracy in Information Not Necessary We find no merit in accused-appellants' insistence that conspiracy should have been alleged in the separate Informations indicting them. We agree with the appellate court, which succinctly stated that conspiracy was not alleged "precisely because they were charged with different offenses for the distinct acts that each of them committed. One's possession of an illegal drug does not need to be conspired by another who, on his part, also possessed an illegal drug."30 The three separate indictments against Ara, Musa, and Talib do not need to allege conspiracy, for the act of conspiring and all the elements of the crime must be set forth in the complaint or information only when conspiracy is charged as a crime.31 Requirements of RA 9165 on Proper Inventory Musa contends that since the markings on the seized items were only made at the police station, there is a great possibility that these were replaced. The result, he argues, would be a lack of guarantee that what were inventoried and photographed at the crime laboratory were the same specimens confiscated from the accused. As recently highlighted in People v. Cortez32 and People v. Lazaro, Jr.,33 RA 9165 and its subsequent Implementing Rules and Regulations (IRR) do not require strict compliance as to the chain of custody rule. The arrest of an accused will not be invalidated and the items seized from him rendered inadmissible on the sole ground of non-compliance with Sec. 21, Article II of RA 9165. We have emphasized that what is essential is "the preservation of the integrity and the evidentiary value of the seized items, as the same would be utilized in the determination of the guilt or innocence of the accused." Briefly stated, non-compliance with the procedural requirements under RA 9165 and its IRR relative to the custody, photographing, and drug-testing of the apprehended persons, is not a serious flaw that can render void the seizures and custody of drugs in a buy-bust operation.34 The chain of custody in the instant case did not suffer from serious flaws as accused-appellants argue. The recovery and handling of the seized drugs showed that, as to Ara, first, PO1 Ayao recovered six plastic sachets of white crystalline substance from Ara and marked them with both his and Ara's initials. Second, the sachets were likewise signed by property custodian PO3 Pelenio. Third, PO1 Ayao signed a Request for Laboratory Examination then personally

delivered the sachets to the PNP Crime Laboratory for examination. Fourth, SPO4 Mallorca then received the sachets at the crime laboratory. As to Musa, first, SPO1 Furog seized the sachets from Musa and marked each with his own initials. Second, an Inventory of Property Seized was then made by SPO4 Galendez. Lastly, SPO1 Furog later submitted a Request for Laboratory Examination of the five (5) sachets weighing a total of 14.2936 grams to the PNP Crime Laboratory. As to Talib, first, PO2 Lao seized a small sachet from Talib during the buy-bust operation. Second, PO2 Lao delivered a Request for Laboratory Examination of one (1) sachet of suspected shabu weighing 0.3559 gram. Third, SPO4 Mallorca also received the items at the PNP Crime Laboratory. Forensic Chemist Noemi Austero's examination of the sachets confiscated from all accusedappellants showed that these were positive for shabu. During trial, the seized items were identified in court. The five (5) sachets taken from Musa were marked Exhibits "A-1" to "A-5," while the sachet seized from Talib was marked Exhibit "B." The six (6) sachets taken from Ara were marked Exhibits "B1-B6." We are, thus, satisfied that the prosecution was able to preserve the integrity and evidentiary value of the shabu in all three criminal cases against accused-appellants. The rest of the arguments interposed are evidently without merit and do not warrant discussion. Penalties Imposed Criminal Case No. 51,472-2002 against Talib The crime of illegal possession of drugs is punishable by Sec. 11 of RA 9165, as follows: Sec. 11. Possession of Dangerous Drugs. - x x x xxxx 3) Imprisonment of twelve (12) years and one (1) day to twenty (20) years and a fine ranging from Three hundred thousand pesos (P300,000.00) to Four hundred thousand pesos (P400,000.00), if the quantities of dangerous drugs are less than five (5) grams of x x x methamphetamine hydrochloride x x x. Talib was sentenced to imprisonment of sixteen (16) years and a fine of PhP 300,000. Criminal Case No. 51,473-2002 against Musa The provision Musa was charged of violating provides the following penalty:

(1) Life imprisonment and a fine ranging from Four hundred thousand pesos (P400,000.00) to Five hundred thousand pesos (P500,000.00), if the quantity of methamphetamine hydrochloride or "shabu" is ten (10) grams or more but less than fifty (50) grams; Musa was sentenced to life imprisonment and a fine of PhP 400,000. Criminal Case No. 51,471-2002 against Ara The crime of illegal sale of shabu is penalized by Sec. 5, Art. 11 of RA 9165: SEC. 5. Sale, Trading, Administration, Dispensation, Delivery, Distribution and Transportation of Dangerous Drugs and/or Controlled Precursors and Essential Chemicals. - The penalty of life imprisonment to death and a fine ranging from Five Hundred Thousand Pesos (P500,000.00) to Ten Million Pesos (P10,000,000.00) shall be imposed upon any person, who, unless authorized by law, shall sell, trade, administer, dispense, deliver, give away to another, distribute, dispatch in transit or transport any dangerous drug, including any and all species of opium poppy regardless of the quantity and purity involved, or shall act as a broker in any of such transactions. The same section contains the following provision: If the sale, trading, administration, dispensation, delivery, distribution or transportation of any dangerous drug and/or controlled precursor and essential chemical transpires within one hundred (100) meters from the school, the maximum penalty shall be imposed in every case. Since the sale of shabu was within five (5) to six (6) meters from St. Peter's College, the maximum penalty of death should be imposed on Ara. Pursuant to RA 9346 or "An Act Prohibiting the Imposition of Death Penalty in the Philippines," however, only life imprisonment and a fine shall be meted on him. Ara was sentenced to life imprisonment and a fine of PhP 10,000,000. He, however, is no longer eligible for parole. What distinguishes this case from others is that one of the accused-appellants was a police officer himself who should have known better than to break the law he was duty-bound to enforce. What is more, he is charged with the crime of selling illegal drugs, an offense so horrendous for destroying the lives of its victims and their families that the penalty of death used to be imposed on its perpetrators. No one could have been more deserving of such a punishment than someone who should be enforcing the law but caught pushing drugs instead. As it was, the death penalty was indeed originally imposed on SPO3 Ara, who had been in the service for more than 30 years.1avvphi1 The ill effects of the use of illegal drugs are too repulsive and shocking to enumerate. Thus, once the charges of sale and possession of said drugs are established in cases such as this, any errors or technicalities raised by the suspects should not be allowed to invalidate the actions of those involved in curtailing their illegal activities. The punishments given to drug pushers should serve

as deterrent for others not to commit the same offense. No price seems high enough for drug dealers to pay; it is just unfortunate that the penalty of death can no longer be imposed because it has been abolished. As the penalties meted out to all three accused-appellants are within the range provided by RA 9165, we affirm the CA's sentence. WHEREFORE, the appeal is DENIED. The CA Decision in CA-G.R. CR-H.C. No. 00025B entitled People of the Philippines v. SPO3 Sangki Ara y Mirasol, Mike Talib y Mama, Jordan Musa y Bayan is AFFIRMED with the modification that accused-appellant Sangki Ara is not eligible for parole. SO ORDERED. G.R. No. 175604 April 10, 2008

THE PEOPLE OF THE PHILIPPINES, appellee, vs. SALVADOR PEAFLORIDA, JR., Y CLIDORO, appellant. DECISION TINGA, J.: Subject of this appeal is the Decision1 of the Court of Appeals in CA-G.R. CR No. 01219, dated 31 July 2006, affirming in toto the judgment2 of the Regional Trial Court of Camarines Sur, Branch 30, in Criminal Case No. T-1476. The trial court found appellant Salvador Peaflorida y Clidoro guilty of transporting marijuana and sentenced him to suffer the penalty of reclusion perpetua and to pay a fine of one million pesos. The Information against appellant reads: That on or about the 7th day of June, 1994, in the afternoon thereat, at Barangay Huyonhuyon, Municipality of Tigaon, Province of Camarines Sur, Philippines and within the jurisdiction of this Honorable Court, the above-named accused, with intent to sell, possess and to deliver with the use of a bicycle, did then and there, willfully, unlawfully and feloniously have in his possession, control and custody, [o]ne bundle estimated to be one (1) kilo more or less, of dried marijuana leaves (Indian Hemp) without the necessary license, permit or authority to sell, administer, deliver, give away to another, distribute, dispatch in transit or transport any prohibited drug from a competent officer as required by law. ACTS CONTRARY TO LAW.3 Upon arraignment, appellant pleaded not guilty. Trial ensued.

Two police officers and one forensic chemist testified for the prosecution. SPO3 Vicente Competente (Competente) narrated that in his capacity as chief of the Investigation and Operation Division of the Philippine National Police (PNP) station in Tigaon, Camarines Sur, that he received a tip from an asset that a bundle of marijuana was being transported by appellant to Huyon-huyon from another barangay in Tigaon, Camarines Sur.4 Major Domingo Agravante (Agravante), chief of police of Tigaon, then organized a team composed of Competente as team leader, SPO2 Ricardo Callo (Callo), SPO1 Portugal, PO3 Pillos and PO2 Edgar Latam. The team boarded the police mobile car and proceeded to Sitio Nasulan in Barangay Huyon-huyon.5 They overtook appellant who was on a bicycle. The police officers flagged appellant down and found marijuana wrapped in a cellophane and newspaper together with other grocery items. The amount of P1550.00 was also found in appellant's possession. The police officers confiscated these items and took photographs thereof. Appellant was then brought to the headquarters where he was booked. 6 Callo, who was the chief intelligence officer of Tigaon PNP, recounted that at around 1:00 p.m. on 7 June 1994, he was called by Competente and was briefed about the operation. While they were in Nasulan, the members of the police team caught a man riding a bicycle who turned out to be appellant. Callo saw the marijuana wrapped in a cellophane and newspaper in the bicycle of appellant so the latter was brought to the police headquarters and turned over to the desk officer.
7

Major Lorlie Arroyo (Arroyo), a forensic chemist at the PNP Crime Laboratory Regional Office No. V, was presented as an expert witness to identify the subject marijuana leaves. She related that after taking a representative sample from the 928-gram confiscated dried leaves, the same was tested positive of marijuana. The findings were reflected in Chemistry Report No. D-26-94 dated 9 June 1994.8 Appellant denied the accusations against him. Appellant, who is a resident of Huyon-huyon, Tigaon, Camarines Sur, testified that in the morning of 7 June 1994, he first went to the house of Igmidio Miranda (Miranda) in Sagnay, Camarines Sur. The latter accompanied appellant to the house of Arnel Dadis in San Francisco, Tigaon to buy a dog. They, however, failed to get the dog; prompting them to leave. On their way home, they met Boyet Obias (Obias) who requested appellant to bring a package wrapped in a newspaper to Jimmy Gonzales (Gonzales).9 Appellant placed it in the basket in front of his bicycle and Gonzales proceeded to the Tiagon town proper. He and Miranda parted ways when they reached the place. Appellant dropped by the grocery store and the blacksmith to get his scythe. On his way home, he was flagged down by the police and was invited to go with them to the headquarters. Upon inspection of the package in his bicycle, the police discovered the subject marijuana. Appellant tried to explain that the package was owned by Obias but the police did not believe him. He was sent to jail.10 Miranda corroborated the testimony of appellant that the two of them went to San Francisco, Tigaon, Camarines Sur in the morning of 7 June 1994 to buy a dog. On their way back to the town proper of Tigaon, they met Obias who requested appellant to bring a package, which Miranda thought contained cookies, to Gonzales. Upon reaching the town proper, they parted ways.11

On 26 October 1998, the trial court rendered judgment finding appellant guilty beyond reasonable doubt of transporting a prohibited drug, a violation of Section 4, Article II of Republic Act (R.A.) No. 6425, otherwise known as The Dangerous Drugs Act of 1972, as amended by R.A. No. 7659. The dispositive portion of the decision reads: WHEREFORE, the accused Salvador Peaflorida[,Jr.] is hereby sentenced to suffer the penalty of imprisonment of reclusion perpetua and to pay a fine of One Million (P1,000,000.00) Pesos, with subsidiary imprisonment in accordance with law, in case of insolvency for the fine and for him to pay the costs. The accused Salvador Peaflorida[,Jr.] shall be entitled to full credit of his preventive imprisonment if he agreed to abide with the rules imposed upon convicted person, otherwise, he shall be entitled to four-fifth (4/5) credit thereof. The subject marijuana consisting of 928 grams, possession thereof being mala prohibita, the court hereby orders its confiscation in favor of the Government to be destroyed in accordance with law. This court, however, hereby recommends to His Excellency, the President of the Philippines, through the Honorable Secretary of Justice to commute the above penalty herein imposed, being too harsh; accordingly, the said penalty imposed to accused Salvador Peaflorida[,Jr] shall be six (6) years of prision correccional, as minimum, to eight (8) years and one (1) day of prision mayor, as maximum. SO ORDERED.12 In convicting appellant, the trial court lent credence to the testimonies of the police officers, thus: Now going over the evidence adduced, the court is convinced that the accused Salvador Peaflorida[,Jr.] committed the offense of illegal possession of 928 grams of marijuana, if not, of transporting it, as charged. This is so, because it appears undisputed that on June 7, 1994, at about 1:00 o'clock in the afternoon police officers Vicente Competente and his four (4) other co-police officers apprehended the accused Salvador Peaflorida[,Jr.] on the roadside at Nasulan, Huyon-huyon, Tigaon, Camarines Sur [,] then riding on his bicycle and placed on the still structure at its front, a thing wrapped in a newspaper and found to be 928 grams of marijuana. No ill-motive has been presented by the defense against the police officers Vicente Competente and companions by falsely testifying against the accused Salvador Peaflorida, Jr. So, the conclusion is inevitable that the presumption that the police officers were in the regular performance of their duties apply. The confiscation of the marijuana subject of the instant case and the arrest of the accused Salvador Peaflorida[,Jr.] by the said police officers being lawful, having been caught in flagrante delicto, there is no need for the warrant for the seizure of the fruit of the crime, the same being incidental to the lawful arrest. Rightly so, because a person caught illegally possessing or transporting drugs is subject to the warrantless search. Besides, object in the "plain view" of an officer who has the right to be in the position to have that view are subject to seizure and may be presented as evidence.13

In view of the penalty imposed, the case was directly appealed to this Court on automatic review. Pursuant to our decision in People v. Mateo,14 however, this case was referred to the Court of Appeals. The appellate court affirmed appellant's conviction on 31 July 2006. In a Resolution15 dated 14 February 2007, the parties were given to file their supplemental briefs, if they so desire. Both parties manifested their intention not to file any supplemental brief since all the issues and arguments have already been raised in their respective briefs.16 Hence, the instant case is now before this Court on automatic review. In assailing his conviction, appellant submits that there is doubt that he had freely and consciously possessed marijuana. First, he claims that the alleged asset did not name the person who would transport the marijuana to Huyon-huyon. In view of the "vague" information supplied by the asset, the latter should have been presented in court. Second, upon receipt of the information from the asset, the police officers should have first investigated and tried to obtain a warrant of arrest against appellant, instead of arbitrarily arresting him. Third, appellant maintains that he is not aware of the contents of the package. Fourth, upon arrival at the headquarters, the police did not determine the contents and weight of the package. Fifth, appellant argues that the findings of the forensic expert are questionable because there is doubt as to the identity of the package examined.17 Prefatorily, factual findings of the trial courts, including their assessment of the witness' credibility are entitled to great weight and respect by this Court, particularly when the Court of Appeals affirm the findings.18 Indeed, the trial court is in the best position to assess the credibility of witnesses since it has observed firsthand their demeanor, conduct and attitude under grilling examination.19 After a review of the records of this case, we find no cogent reason to disregard this time-honored principle. We shall retrace the series of events leading to the arrest of appellant and resolve the issues raised by him. Acting on an asset's tip, a police team was organized to apprehend appellant who was allegedly about to transport the subject marijuana. Appellant is wrong in concluding that the asset did not name appellant. As early as 16 November 1996, appellant through counsel had already conceded in his Memorandum20 filed with the trial court that based on the tip, he was about to transport the contraband. It further cited excerpts from the result of the preliminary investigation conducted by the judge on Competente, and we quote: Q: Did your [a]sset tell you the place and the person or persons involved? A: Yes[,]sir. Q: Where and who? A: He said that marijuana is being transported from Tigaon town to Bgy. Huyon-huyon by Salvador Peaflorida, Jr.21

Moreover, on cross-examination, the defense counsel even assumed that according to the asset's tip it was appellant who was assigned to deliver the contraband. And the witness under crossexamination affirmed it was indeed appellant who would be making the delivery according to the tip: Q: Will you inform this Honorable Court who has given you the tip that the accused was going to deliver that marijuana[?] [W]ho is [this] person? A: It was a confidential tip. Q: Now, but [sic] on June 1 you were in your office? A: Yes[,] sir[.] I was in the office. Q: Since your office is just near the Municipal Trial Court of Tigaon and you were given a tip that Salvador Peaflorida[,Jr.] will be delivering marijuana, why did you not get a [w]arrant of [a]rrest? xxx Q: The tip that was given to you that it was Salvador Peaflorida [who] will be dealing marijuana on that date and according to you Salvador was to travel from a certain town to Tigaon, is that the tip? A: Yes[,] sir[.] That he would deliver marijuana. Q: So, at the time that you form[ed] a team, Salvador was nowhere to be seen, you have not seen the shadow of Salvador? A: When the tip was given to us[,] I have not seen him[.] [B]ut the tip is he will deliver from Tigaon to Huyon-huyon, that is why we chased him.22 [Emphasis supplied] Prescinding from the above argument, appellant insists that the asset should have been presented in court. He invoked the court ruling in People v. Libag,23 wherein the non-presentation of the informant was fatal to the case of the prosecution. Libag cannot find application in this case. In that case, the crime charged was the sale of shabu where the informant himself was a poseurbuyer and a witness to the transaction. His testimony as a poseur-buyer was indispensable because it could have helped the trial court in determining whether or not the appellant had knowledge that the bag contained marijuana, such knowledge being an essential ingredient of the offense for which he was convicted.24 In this case, however, the asset was not present in the police operation. The rule is that the presentation of an informant in an illegal drugs case is not essential for conviction nor is it indispensable for a successful prosecution because his testimony would merely be corroborative and cumulative. Informants are generally not presented in court because of the need to hide their identity and preserve their invaluable service to the police.25

Competente testified that his team caught up with appellant who was riding a bicycle. He saw the marijuana in a package which appellant was carrying inside his basket, thus: Q: And so as the team leader x x x and in connection with the instruction of Chief Domingo Agravante, what did you do? A: We used the mobile and proceeded to the place, to the route where the marijuana was being transported. Q: When you said we to whom are you referring to? A: The team. Q: Were you able to go to the place as you said? A: Yes, sir. Q: So, upon reaching the place, [sic] what place was that? A: Sitio Nasulan, Barangay Huyon-huyon, Tigaon, Camarines Sur. Q: And upon reaching the place together with the other member of the team, what did you find if you found any? A: We overtook our suspect while riding in a bicycle and we stopped him. Q: And did the suspect stop? A: Yes[,] sir. Q: Tell us the name of your suspect? A: Salvador Peaflorida[,] Jr. y Clidoro. Q: And after stopping the accused in this case, what else did you do[,] if any[,] together with the team? A: When we saw the marijuana and other groceries in his bicycle we invited him to the headquarters.26 Callo also confirmed that he saw appellant transporting and in possession of the subject marijuana: Q: When you reached there[,] what happened next?

A: We have not reached yet [sic] the Huyon-huyon proper. [W]e are in Nasulan when we met the man who had with him the marijuana. xxx Q: After you talked with the person with marijuana[,] what happened next? A: We saw on his bicycle a wrap[ped] marijuana. Q: Who was in possession of that? A: Salvador Peaflorida[,] Jr. Q: How is that person related to the accused in this case now? A: He is the one, sir. Q: Kindly describe to us the marijuana that you are able to tell that it was marijuana? A: It was wrapped on [cellophane] and newspaper. We saw the edges of the marijuana. Q: For the [record], kindly describe to us the edges of the marijuana[;] its appearance and color. A: It was like a shape of ream of coupon bond and the color is green.27 These positive and categorical declarations of two police officers deserve weight and credence in light of the presumption of regularity accorded to them and the lack of motive on their part to falsely testify against appellant. Appellant resorts to a challenge on the validity of his arrest predicated on lack of a warrant of arrest. The OSG correctly justifies the failure to apply for an arrest warrant because at that point, time was of the essence in appellant's apprehension, noting in the same breath that there is no law requiring investigation and surveillance upon receipt of tips from assets before conducting police operations.28 The police officers succinctly testified on this point when cross-examined, viz: Q: Will you inform this Honorable Court who has given you the tip that the accused was going to deliver that marijuana, who is that person? A: It was a confidential tip. Q: Now, but [sic] on June 1 you were in your office? A: Yes[,] sir[.] I was in the office.

Q: Since your office is just near the Municipal Trial Court of Tigaon and you were given a tip that Salvador Peaflorida[,Jr.] will be delivering marijuana, why did you not get a [w]arrant of [a]rrest from the court? A: There was no time to apply for a search warrant because just after the information was received, we proceeded. xxx Q: If that is true, Mr. Competente that you were given a tip, the most that you will do is first see the Judge of Tigaon in as much as you have not seen yet [sic] the said person carrying marijuana? A: There was no time for us to apply, because the marijuana is being delivered so we have no more time to see the Judge. xxx Q: Are you aware of the law that illegally confiscated marijuana cannot be used in court? FISCAL SOLANO: Conclusion of law. A: Yes, sir[.] [I]f it is illegally confiscated it cannot be used in court. ATTY. CLEDERA: Despite that prohibition under the rules[,] you insisted in apprehending Salvador Peaflorida[,Jr.] without warrant of arrest inspite of the fact that you know that restriction? A: Our apprehension was in plain view. Q: How can you see that it was in open view when according to you the house of Salvador is 120 meters[?] [H]ow can you see that distance? A: I could see that because the marijuana was carried in his bicycle, we have seen it. Q: In what street? A: Huyon-huyon[,] Sitio Nasulan, Tigaon, Camarines Sur. Q: About what time did you see him? A: 1:00 o'clock sir. x x x29

The police was tipped off at around 1:00 p.m. that appellant was transporting marijuana to Huyon-huyon. Certainly, they had no time to secure an arrest warrant as appellant was already in transit and already committing a crime. The arrest was effected after appellant was caught in flagrante delicto. He was seen riding his bicycle and carrying with him the contraband, hence, demonstrating that a crime was then already being committed. Under the circumstances, the police had probable cause to believe that appellant was committing a crime. Thus, the warrantless arrest is justified. Article II, Section 4 of R.A. No. 6425, as amended by R.A. No. 7659, states: SEC. 4. Sale, Administration, Delivery, Distribution and Transportation of Prohibited Drugs. The penalty of reclusion perpetua to death and a fine ranging from five hundred thousand pesos to ten million pesos shall be imposed upon any person who, unless authorized by law, shall sell, administer, deliver, give away to another, distribute, dispatch in transit or transport any prohibited drug, or shall act as broker in any of such transactions. x x x. Jurisprudence defines "transport" as "to carry or convey from one place to another."30 In the instant case, appellant was riding his bicycle when he was caught by the police. He admitted that he was about to convey the package, which contained marijuana, to a certain Jimmy Gonzales. Appellant, however, denies any knowledge that the package in his possession contained marijuana. But the trial court rejected his contention, noting that it was impossible for appellant not to be aware of the contents of the package because "marijuana has a distinct sweet and unmistakable aroma x x x which would have alarmed him."31 Taking one step further, the appellate court went on to declare that being mala prohibita, one commits the crime under R.A. No. 6425 by mere possession of a prohibited drug without legal authority. Intent, motive or knowledge thereof is not necessary.32 Appellant, in the main, asserts that he did not freely and consciously possess marijuana.33 In criminal cases involving prohibited drugs, there can be no conviction unless the prosecution shows that the accused knowingly possessed the prohibited articles in his person, or that animus possidendi is shown to be present together with his possession or control of such article. Animus possidendi is only prima facie. It is subject to contrary proof and may be rebutted by evidence that the accused did not in fact exercise power and control over the thing in question, and did not intend to do so. The burden of evidence is thus shifted to the possessor to explain absence of animus possidendi.34 Knowledge refers to a mental state of awareness of a fact. Since courts cannot penetrate the mind of an accused and thereafter state its perceptions with certainty, resort to other evidence is necessary. Animus possidendi, as a state of mind, may be determined on a case-to-case basis by taking into consideration the prior or contemporaneous acts of the accused, as well as the surrounding circumstances. Its existence may and usually must be inferred from the attendant events in each particular case.35

Appellant failed to satisfactorily establish his lack of knowledge of possession in the instant case. First, the marijuana was found in the bicycle he himself was driving. Second, the police officers first readily saw in plain view the edges of the marijuana leaves jutting out of the package. Third, it is incredulous that appellant did not ask Obias what the package contained when the latter requested him to do the delivery errand since the package was wrapped in a newspaper and weighed almost one kilogram. The same observation was reached by the trial court: Finally, it is very hard for the court to accept the claim of the accused Salvador Peaflorida[,Jr.] that he does not know that the thing wrapped in a newspaper which Boyet Obias, now dead, requested the accused Peaflorida[,Jr.] would deliver to a certain Jimmy Gonzales whose present whereabouts is not known, was a marijuana. Its odor is different especially from tobacco. This was observed by the court during the trial of the case, everytime the wrapper containing the subject marijuana with a volume of 928 grams is brought to court its odor is noticeable. For the accused Peaflorida[,Jr.], not to notice it is hard to believe. Rightly so, because marijuana has a distinct sweet and unmistakable aroma very different from (and not nauseating) unlike tobacco. This aroma would have alarmed him.36 Furthermore, it appeared from the cross-examination of appellant that Obias was an acquaintance. In the ordinary course of things, one is expected to inquire about the contents of a wrapped package especially when it is a mere acquaintance who requests the delivery and, more so, when delivery is to a place some distance away. Anent appellant's claim that the package examined by Arroyo was not the one confiscated from him, the appellate court had this to say: SPO3 Competente testified that marijuana was confiscated from appellant. The pictures of appellant, together with the items seized from him, depict a package containing dry leaves suspected to be marijuana. On the other hand, Forensic Chemist Arroyo testified that the specimen she examined was delivered to her by Major Agravante on June 9, 1994 or two days after the apprehension. From these series of events, it can be inferred that the package confiscated from appellant and the specimen delivered to Forensic Chemist Arroyo for laboratory examination were one and the same.37 Despite intense grilling from the defense counsel, Arroyo never faltered and was in fact consistent in declaring that she received the specimen from Agravante on 9 June 1994 and immediately conducted the laboratory test. Finally, the lower courts correctly sentenced appellant to suffer the penalty of reclusion perpetua and to pay a fine of one million pesos by virtue of the amendment to Section 4, R.A. No. 6425 by R.A. No. 7659.38 WHEREFORE, in view of the foregoing, the decision of the Regional Trial Court of San Jose, Camarines Sur, Branch 30 in Criminal Case No. T-1476, finding appellant Salvador Peaflorida y Clidoro guilty beyond reasonable doubt of violation of Section 4, Article II of R.A. No. 6425

(Dangerous Drugs Act) as amended, and sentencing him to suffer the penalty of reclusion perpetua and to pay a fine of One Million Pesos (P1,000,000.00), is AFFIRMED in toto. SO ORDERED. Morenovs.AgoChi Facts:Sometime May1904, the defendant (Ago Chi),represented by the plaintiff was charged and was convictedby the CFI Manila in the crime of assassination. He wassentenced to capital punishment but was later reduced to 20 years of reclusion temporal upon appeal in the SC.Upon the defendants arrest, the arresting officer confiscated his money amounting to P700and was depositedin the clerk of court.The plaintiff Moreno, filed a petition to the CFIManila under Judge Quintero praying for the court to pay theplaintiff of his legal services to the defendant in the amount of P600that will be taken from the confiscated money of thedefendant.The lower court only granted P50as legal servicesto the plaintiff. Hence this petition. Issue:Whether or not the search and confiscation of thedefendant is a valid search.Can the plaintiff validly claim any lien from theconfiscated money?Held:No.Under the Rules of Criminal procedure, an officer making an arrest may take from the person arrested anymoney or property found upon his person which was used inthe commission of the crime or was the fruit of the crime or which might furnish the prisoner with the means of committing violence or of escaping, or which may be used asevidence in the trial of the cause.In the case at bar, the defendant was convicted inthe crime of assassination, thus, the confiscation of hismoney amounting to P7 00 cannot in no way be connectedwith the crime where he was charged or in any casesmentioned.To deprive the defendant of his money or propertyunder other circumstances than those mentioned above is todeprive him, perhaps, of the lawful means of defense.Thus, the money is illegally searched andconfiscated and cannot be admitted as evidence. Since itwas illegally confiscated, third persons cannot validly claimany liens from the confiscated money and should remainunder the custody of the defendant.Case remanded to the lower court. People vs. Ang ChunKit Facts: ANG CHUN KIT, a Chinese national and reputed tobe a member of a Hong Kong-based drug syndicateoperating in Metro Manila, was collared by NARCOMoperatives in a buy-bust operation after he sold to anundercover agent for P4 00 , 000 . 00 a kilo of methamphetamine hydrochloride known as shabu. His car also yielded more of the regulated drug neatly tucked in aKleenex box. After tipping an information from a ConfidentialInformer, Chief investigator Avelino Razon arranged a buy-bust operation using operatives and marked money,scheduled to be operated in Cardinal Santos Hospital.The buy-bust operation was successful, the moneywas confiscated and nearly a kilo of shabu was seized in thedefendants car.He was then brought to the Camp Crame, chargedwith violation to the Dangerous drugs Act. He was convictedby RTC Pasig Br. 1 55 in violation of RA 6425, Sec 1

5 Art III.Hence This petition.He argued that the confiscated articles areinadmissible as evidence since it was procured through anillegal search and seizure and that the drugs are not found inhis person but in his car.Issue:Was the defendats contention correct?Held:No. It was a search and seizure incidental to a validarrest.Under the constitutional and statutory provision, avalid warrantless arrest may be effected when the accused isactually committed, committing or about to commit a crime. And that a warrantless search and seizure can be effectedwhen the search is incidental to a valid arrest. Moreover, awarrantless search and seizure as an incident to a lawfularrest may extend beyond the person of the one arrested toinclude the premises or surroundings under his immediatecontrol.In the case at bar, RA 6425 actually punishes evena mere possession of regulated drugs, thus the accused is inthe process of committing a crime during the buy bustoperation. It is thus then a valid warrantless arrest.Since it is a valid, the warrantless search mayextend to the accused surrounding under his immediatecontrol, thus the search seizure of shabu in his car is a validseizure. Note: In the case at bar . the Court however agree withthe accused that his signature on the receipt or lists of itemsconfiscated from him is inadmissible in evidence as there isno showing that he was then assisted by counsel. In Peoplev. Mauyao we said that "conformance to these documentsare declarations against interest and tacit admissions of thecrime charged, since merely unexplained possession of prohibited drugs is punished by law. They have beenobtained in violation of his right as a person under custodialinvestigation for the commission of an offense, there beingnothing in the records to show that he was assisted bycounsel."With regard to the Booking Sheet and ArrestReport, we already said in People v. Morico that "when anarrested person signs a Booking Sheet and Arrest Report ata police station he does not admit the commission of anoffense nor confess to any incriminating circumstance. TheBooking Sheet is merely a statement of the accused's beingbooked and of the date which accompanies the fact of anarrest. It is a police report and may be useful in charges of arbitrary detention against the police themselves. It is not anextra-judicial statement and cannot be the basis of a judgment of conviction." People vs. Lua Facts:The defendant, Rolando Chekwa Lua, was arrestedby a buy bust operation under the Oplan Saturn by theCaloocan police operatives in Bagong Silang Caloocan City.In the buy bust operation, after theinformant/operative successfully bought some marijuanafrom the accused, the police subsequently acted andarrested Lua, confiscated the marked money, the regulateddrug marijuana and a .3 8 cal paltik and live bulletswhich wasfound in the accused waistline. These articles are soughtoutside the accuseds houseThe police operatives also found and confiscated abrick of marijuana inside the accuseds house.The RTC Br. 1 24 of Caloocan City convicted theaccused in violation of RA 6425. Hence this petition.Issue:Was there a valid warrantless search and seizure?Held:Yes. The search and seizure is lawful, that being awarrantless search and seizure incidental to a lawful arrest,however, the articles (brick of marijuana) seized inside theaccuseds house is inadmissible evidence.Having settled the issues raised by appellant, theequally important matter as regards admissibility of theevidence should likewise be passed upon. The buy-bustoperation conducted by the police operatives is a form of entrapment allowed by law. The arrest of the appellant waslawful having been caught in flagrante delicto

. Consequently,there is no need for a warrant for the seizure of the 3 tea Constitutional Law II (Bill of Rights): Case Briefs: Dennis G. Libunao UC College of Law 32 bags of marijuana (5.3934 grams) the same being the fruit of the crime. With respect to the body search made by Puno,the same was valid being incidental to a lawful arrest.Therefore, the .3 8 cal. paltik and the two (2) live bullets andthe empty shell found in the cylinder are admissible inevidence. As regards the brick of marijuana found inside theappellant's house, the trial court correctly ignored itapparently in view of its inadmissibility.While initially the arrest as well as the body searchwas lawful, the warrantless search made inside appellant'shouse became unlawful since the police operatives were notarmed with a search warrant. Such search cannot fall under "search made incidental to a lawful arrest," the same beinglimited to body search and to that point within reach or control of the person arrested, or that which may furnish himwith the means of committing violence or of escaping. In thecase at bar, appellant was admittedly outside his house whenhe was arrested. Hence, it can hardly be said that the inner portion of his house was within his reach or control.In sum, this court finds accused-appellant RolandoLua guilty beyond reasonable doubt of violating Sec. 4, Art.II, of R.A. 6425, as amended, under which the penalty of lifeimprisonment to death and a fine ranging from twentythousand to thirty thousand pesos shall be imposed. People vs. Figueroa Facts:The accused was charged with Illegal Possession of Firearms and Ammunitions and and of RA 645 andsubsequently convicted by the RTC Br. 23 of Trece Martiresin Cavite.While serving the warrant of arrest, the officersnoticed, strewn around, aluminum foil packages of differentsizes in the sala. Suspecting thus the presence of "shabu" inthe premises, the arresting officers requested appellant, aswell as his brother and sister, to acquiesce to a search of thehouse. The search yielded a .45 caliber pistol, a magazine,seven live ammunitions, and a match box containing analuminum foil package with "shabu." Confronted, Figueroadenied ownership of the items. An inventory was conductedby the PC team, attested to by Barangay Captain Bigornia, of the seized items.The accused questions the admissibility in evidenceof the firearm and confiscated ammunition for it wasdiscovered during a warrantless search.Issue:Was their an unlawful warrantless search andseizure.Held:No. the search and seizure of the articles sought isa valid being a search incidental to an arrest.The .45 caliber pistol, magazine and rounds of ammunition were not unlawfully obtained. While the SC mightconcede difficulty in readily accepting the statement of theprosecution that the search was conducted with consentfreely given by appellant and members of his household, itshould be pointed out, in any case, that the search andseizure was done admittedly on the occasion of a lawfularrest. A significant exception from the necessity for asearch warrant is when the search and seizure is effected asan incident to a lawful arrest. As a doctrine in jurisprudence, the warrantlesssearch and seizure, as an incident to a suspect's lawfularrest, may extend beyond the person of the one arrested toinclude the premises or surrounding under his immediatecontrol. Objects in the `plain view' of an officer who has theright to be in the position to have that view are subject toseizure and may be presented as evidence." Nolasco vs. Pano Conflicting views between the ponente Justice Melencio-Herrera and Justices Teehankee and Cuevas concerning thevalidity place of arrest and search of the accused inconnection with Lawful Search Incidental to an Arrest Facts:The petitioners, Nolasco, Aguilar-Roque andTolentino were charged of the crime of Rebellion, Subversionand/or Conspiracy o commit Rebellion/Subversion and wasarrested by Constabulary Security Group (CSG) on August6 that11:30AM at the intersection of Mayon St. and P.Margali St, Quezon City. At 1

2 noon of that same day, a search wasconducted at the residence of the petitioner Aguilar-Roque at239 B Mayon St. QC. The CSG confiscated 42 8 writtendocuments, typewriter and 2 wooden boxes.The petitioners alleged that the search warrant andarrest warrants issued was void in the ground that there is noestablished existing probable cause and that the warrant is ageneral in nature, violative of their constitutional right.Moreover, they contend that the articles confiscatedin inadmissible as evidence as the search is illegal not beinga search incidental to an arrest.Issue:Was the search cannot be qualified as a lawfulsearch incidental to lawful arrest?Held:No. The Search is lawful.UNDER Sec 12, rule126 of the RRC,Section 12 . Search without warrant of personarrested. - A person charged with an offense may besearched for dangerous weapons or anything which may beused as proof of the commission of the offense." The provision is declaratory in the sense that it isconfined to the search, without a search warrant, of a personwho had been arrested. It is also a general rule that, as anincident of an arrest, the place or premises where the arrestwas made can also be search without a search warrant. Inthis latter case, "the extent and reasonableness of the searchmust be decided on its own facts and circumstances, and ithas been stated that, in the application of general rules, thereis some confusion in the decisions as to what constitutes theextent of the place or premises which may be searched"."What must be considered is the balancing of the individual'sright to privacy and the public's interest in the prevention of crime and the apprehension of criminals."Considering that AGUILAR-ROQUE has beencharged with Rebellion, which is a crime against public order;that the warrant for her arrest has not been served for aconsiderable period of time; that she was arrested within thegeneral vicinity of her dwelling; and that the search of her dwelling was made within a half hour of her arrest, we are of the opinion that, in her respect, the search at No. 239-BMayon Street, Quezon City, did not need a search warrant ;this, for possible effective results in the interest of publicorder.The search in the residence of the petitioner Aguilar-Roque is valid since in relation with the nature of thecrime of rebellion, which is a crime against public order, it is asearch incidental to an arrest. The immediate vicinity of thearrest is not controlling as far as the publics interest in theprevention of the crime and apprehension of the criminals areconcerned. Separate and Dissenting opinion of justices T eehankeeand Cuevas Justice Teehankee The exception of Rule 1 26, sec. 1 2 which allows awarrantless search of a person who is lawfully arrested isabsolutely l imited to his person, at the time of and incident tohis arrest and to "dangerous weapons or anything which may be used as proof of the commission of the offense." Suchwarrantless search obviously cannot be made in a placeother than the place of arrest . In this case, petitioner Aguilar-Roque was arrested at 11 :3 0

a.m. on board a public vehicleon the road (at Mayon and P. Margall Streets). To hold thather dwelling could "later on the same day" be searchedwithout warrant is to sanction an untenable violation, if notnullification, of the cited basic constitutional rights againstunreasonable searches and seizures. Justice C uevas The lawful arrest being the sole justification for thevalidity of the warrantless search under the statutoryprovision (Sec. 1 2, Rule 1 26) the same must be limited toand circumscribed by, the subject, time, and place of said arrest . A s to subject , the warrantless search is sanctionedonly with respect to the person of the suspect, and things thatmay be seized from him are limited to "dangerous weapons"or "anything which may be used as proof of the commissionof the offense.""An officer making an arrest may take from theperson arrested any money or property found upon hisperson which was used in the commission of the crime or might furnish the prisoner with the means of committingviolence or escaping or which may be used as evidence inthe trial of the cause. W ith respect to the ti m e and place of thewarrantless search allowed by law , it must becontemporaneous with the lawful arrest. Stated otherwise, tobe valid, the search must have been conducted at about thetime of the arrest or immediately thereafter and only at theplace where the suspect was arrested."The right without a search warrant contemporaneously tosearch a person lawfully arrested while committing a crimeand to search the place where the arrest is made in order tofind and seize things connected with the crime as its fruits or as the means by which it was committed, as well as weaponsor other things to effect an escape from custody is not to bedoubted. But the right does not extend to other places The second element which must exist in order tobring the case within the exception to the general rule is that,in addition to a lawful arrest, t he search m ust be incident tothe arrest. "The search m ust be m ade at the place of thearrest, otherwise, it is not incident to the arrest. AGNELLO vs. U.S. supra. In this latter case, 269 U.S. 2 0 at3

0 , it is said that the officers have a right to make a searchcontemporaneously with the arrest. And if the purpose of theofficers in making their entry is not to make an arrest, but tomake a search to obtain evidence for some future arrest,then search is not incidental to arrest. BYARS vs. U.S. 273U.S., 2 8 ET AL." (Papani vs. U.S., 8 4F2d 1 6 0 , 1 63)In the instant case, petitioners were arrested at theintersection of Mayon St. and P. Margall St. at 11 :3 0 A.M. of August 6, 1 976. The search, on the other hand, wasconducted after the arrest, that was at around 1 2: 00 noon of the same day or "late that same day (as respondents claim intheir "COMMENT") at the residence of petitioner AGUILAR-ROQUE in 239B Mayon St., Quezon City. How far or howmany kilometers is that place from the place where petitioner was arrested do not appear shown by the record, But whatappears undisputed is that the search was made in a placeother than the place of arrest and, not on the occasion of nor immediately after the arrest. It cannot be said, therefore, thatsuch a search was incidental to the arrest of the petitioners.Not being an incident of a lawful arrest, the search of thepremises at 239B Mayon St., Quezon City WITHOUT A VALIDSEARCHWARRANT is ILLEGAL and violative of the constitutional rights of the respondent . The thingsand properties seized on the occasion of said illegal searchare therefore INADMISSIBLE in evidence under theexclusionary rule.

G.R. No. 188611

June 16, 2010

PEOPLE OF THE PHILIPPINES, Appellee, vs. BELEN MARIACOS, Appellant. DECISION NACHURA, J.:

Before this Court is an appeal from the Decision1 of the Court of Appeals (CA) in CA-G.R. CRHC No. 02718, which affirmed the decision2 of the Regional Trial Court (RTC), Branch 29, San Fernando City, La Union, in Criminal Case No. 7144, finding appellant Belen Mariacos guilty of violating Article II, Section 5 of Republic Act (R.A.) No. 9165, or the Comprehensive Dangerous Drugs Act of 2002. The facts of the case, as summarized by the CA, are as follows: Accused-appellant Belen Mariacos was charged in an Information, dated November 7, 2005 of violating Section 5, Article II of Republic Act [No.] 9165, allegedly committed as follows: "That on or about the 27th day of October, 2005, in the Municipality of San Gabriel, Province of La Union, Philippines, and within the jurisdiction of this Honorable Court, the above-named accused, did then and there willfully, unlawfully and feloniously transport, deliver 7,030.3, (sic) grams of dried marijuana fruiting tops without the necessary permit or authority from the proper government agency or office. CONTRARY TO LAW." When arraigned on December 13, 2005, accused-appellant pleaded not guilty. During the pretrial, the following were stipulated upon: "1. Accused admits that she is the same person identified in the information as Belen Mariacos; 2. That accused is a resident of Brgy. Lunoy, San Gabriel, La Union; 3. That at the time of the arrest of the accused, accused had just alighted from a passenger jeepney; 4. That the marijuana allegedly taken from the possession of the accused contained in two (2) bags were submitted for examination to the Crime Lab; 5. That per Chemistry Report No. D-109-2005, the alleged drug submitted for examination gave positive result for the presence of marijuana; 6. That the drugs allegedly obtained from the accused contained (sic) and submitted for examination weighed 7,030.3 grams; 7. The Prosecutor admits the existence of a counter-affidavit executed by the accused; and 8. The existence of the affidavits executed by the witnesses of the accused family (sic): Lyn Punasen, Mercedes Tila and Magdalena Carino." During the trial, the prosecution established the following evidence:

On October 26, 2005, in the evening, the San Gabriel Police Station of San Gabriel, La Union, conducted a checkpoint near the police station at the poblacion to intercept a suspected transportation of marijuana from Barangay Balbalayang, San Gabriel, La Union. The group at the checkpoint was composed of PO2 Lunes B. Pallayoc ("PO2 Pallayoc"), the Chief of Police, and other policemen. When the checkpoint did not yield any suspect or marijuana, the Chief of Police instructed PO2 Pallayoc to proceed to Barangay Balbalayang to conduct surveillance operation (sic). At dawn on October 27, 2005, in Barangay Balbalayang, PO2 Pallayoc met with a secret agent of the Barangay Intelligence Network who informed him that a baggage of marijuana had been loaded on a passenger jeepney that was about to leave for the poblacion. The agent mentioned three (3) bags and one (1) blue plastic bag. Further, the agent described a backpack bag with an "O.K." marking. PO2 Pallayoc then boarded the said jeepney and positioned himself on top thereof. While the vehicle was in motion, he found the black backpack with an "O.K." marking and peeked inside its contents. PO2 Pallayoc found bricks of marijuana wrapped in newspapers. He then asked the other passengers on top of the jeepney about the owner of the bag, but no one knew. When the jeepney reached the poblacion, PO2 Pallayoc alighted together with the other passengers. Unfortunately, he did not notice who took the black backpack from atop the jeepney. He only realized a few moments later that the said bag and three (3) other bags, including a blue plastic bag, were already being carried away by two (2) women. He caught up with the women and introduced himself as a policeman. He told them that they were under arrest, but one of the women got away. PO2 Pallayoc brought the woman, who was later identified as herein accused-appellant Belen Mariacos, and the bags to the police station. At the police station, the investigators contacted the Mayor of San Gabriel to witness the opening of the bags. When the Mayor arrived about fifteen (15) minutes later, the bags were opened and three (3) bricks of marijuana wrapped in newspaper, two (2) round bundles of marijuana, and two (2) bricks of marijuana fruiting tops, all wrapped in a newspaper, were recovered. Thereafter, the investigators marked, inventoried and forwarded the confiscated marijuana to the crime laboratory for examination. The laboratory examination showed that the stuff found in the bags all tested positive for marijuana, a dangerous drug. When it was accused-appellants turn to present evidence, she testified that: On October 27, 2005, at around 7:00 in the morning, accused-appellant, together with Lani Herbacio, was inside a passenger jeepney bound for the poblacion. While the jeepney was still at the terminal waiting for passengers, one Bennie Lao-ang ("Lao-ang"), her neighbor, requested her to carry a few bags which had been loaded on top of the jeepney. At first, accused-appellant refused, but she was persuaded later when she was told that she would only be carrying the bags. When they reached the poblacion, Lao-ang handed accused-appellant and her companion, Lani Herbacio, the bags, and then Lao-ang suddenly ran away. A few moments later, PO2 Pallayoc was upon them, arresting them. Without explanation, they were brought to the police station.

When they were at the police station, Lani Herbacio disappeared. It was also at the police station that accused-appellant discovered the true contents of the bags which she was asked to carry. She maintained that she was not the owner of the bags and that she did not know what were contained in the bags. At the police station (sic) she executed a Counter-Affidavit.3 On January 31, 2007, the RTC promulgated a decision, the dispositive portion of which states: WHEREFORE, the Court finds the accused Belen Mariacos GUILTY as charged and sentences here (sic) to suffer the penalty of life imprisonment and to pay a fine of P500,000.00. The 7,030.3 grams of marijuana are ordered confiscated and turned over to the Philippine Drug Enforcement Agency for destruction in the presence of the Court personnel and media. SO ORDERED.4 Appellant appealed her conviction to the CA. She argued that the trial court erred in considering the evidence of the prosecution despite its inadmissibility.5 She claimed that her right against an unreasonable search was flagrantly violated by Police Officer (PO)2 Pallayoc when the latter searched the bag, assuming it was hers, without a search warrant and with no permission from her. She averred that PO2 Pallayocs purpose for apprehending her was to verify if the bag she was carrying was the same one he had illegally searched earlier. Moreover, appellant contended that there was no probable cause for her arrest.6 Further, appellant claimed that the prosecution failed to prove the corpus delicti of the crime.7 She alleged that the apprehending police officers violated Dangerous Drugs Board Regulation No. 3, Series of 1979, as amended by Board Regulation No. 2, Series of 1990, which prescribes the procedure in the custody of seized prohibited and regulated drugs, instruments, apparatuses, and articles. The said regulation directs the apprehending team having initial custody and control of the drugs and/or paraphernalia, immediately after seizure or confiscation, to have the same physically inventoried and photographed in the presence of appellant or her representative, who shall be required to sign copies of the inventory. The failure to comply with this directive, appellant claimed, casts a serious doubt on the identity of the items allegedly confiscated from her. She, likewise, averred that the prosecution failed to prove that the items allegedly confiscated were indeed prohibited drugs, and to establish the chain of custody over the same. On the other hand, the People, through the Office of the Solicitor General (OSG), argued that the warrantless arrest of appellant and the warrantless seizure of marijuana were valid and legal,8 justified as a search of a moving vehicle. It averred that PO2 Pallayoc had reasonable ground to believe that appellant had committed the crime of delivering dangerous drugs based on reliable information from their agent, which was confirmed when he peeked into the bags and smelled the distinctive odor of marijuana.9 The OSG also argued that appellant was now estopped from questioning the illegality of her arrest since she voluntarily entered a plea of "not guilty" upon arraignment and participated in the trial and presented her evidence.10 The OSG brushed aside appellants argument that the bricks of marijuana were not photographed and inventoried in her presence or that of her counsel immediately after confiscation, positing that physical inventory

may be done at the nearest police station or at the nearest office of the apprehending team, whichever was practicable.11 In a Decision dated January 19, 2009, the CA dismissed appellants appeal and affirmed the RTC decision in toto.12 It held that the prosecution had successfully proven that appellant carried away from the jeepney a number of bags which, when inspected by the police, contained dangerous drugs. The CA ruled that appellant was caught in flagrante delicto of "carrying and conveying" the bag that contained the illegal drugs, and thus held that appellants warrantless arrest was valid. The appellate court ratiocinated: It must be stressed that PO2 Pallayoc had earlier ascertained the contents of the bags when he was aboard the jeep. He saw the bricks of marijuana wrapped in newspaper. That said marijuana was on board the jeepney to be delivered to a specified destination was already unlawful. PO2 Pallayoc needed only to see for himself to whom those bags belonged. So, when he saw accusedappellant carrying the bags, PO2 Pallayoc was within his lawful duty to make a warrantless arrest of accused-appellant. xxxx Firstly, this Court opines that the invocation of Section 2, Article III of the Constitution is misplaced. At the time, when PO2 Pallayoc looked into the contents of the suspicious bags, there was no identified owner. He asked the other passengers atop the jeepney but no one knew who owned the bags. Thus, there could be no violation of the right when no one was entitled thereto at that time. Secondly, the facts of the case show the urgency of the situation. The local police has been trying to intercept the transport of the illegal drugs for more than a day, to no avail. Thus, when PO2 Pallayoc was tipped by the secret agent of the Barangay Intelligence Network, PO2 Pallayoc had no other recourse than to verify as promptly as possible the tip and check the contents of the bags. Thirdly, x x x the search was conducted in a moving vehicle. Time and again, a search of a moving vehicle has been justified on the ground that the mobility of motor vehicles makes it possible for the vehicle to move out of the locality or jurisdiction in which the warrant must be sought. Thus, under the facts, PO2 Pallayoc could not be expected to secure a search warrant in order to check the contents of the bags which were loaded on top of the moving jeepney. Otherwise, a search warrant would have been of no use because the motor vehicle had already left the locality.13 Appellant is now before this Court, appealing her conviction. Once again, we are asked to determine the limits of the powers of the States agents to conduct searches and seizures. Over the years, this Court had laid down the rules on searches and seizures, providing, more or less, clear parameters in determining which are proper and which are not.1avvphi1

Appellants main argument before the CA centered on the inadmissibility of the evidence used against her. She claims that her constitutional right against unreasonable searches was flagrantly violated by the apprehending officer. Thus, we must determine if the search was lawful. If it was, then there would have been probable cause for the warrantless arrest of appellant. Article III, Section 2 of the Philippine Constitution provides: Section 2. The right of the people to be secure in their persons, houses, papers, and effects against unreasonable searches and seizures of whatever nature and for any purpose shall be inviolable, and no search warrant or warrant of arrest shall issue except upon probable cause to be determined personally by the judge after examination under oath or affirmation of the complainant and the witnesses he may produce, and particularly describing the place to be searched and the persons or things to be seized. Law and jurisprudence have laid down the instances when a warrantless search is valid. These are: 1. Warrantless search incidental to a lawful arrest recognized under Section 12 [now Section 13], Rule 126 of the Rules of Court and by prevailing jurisprudence; 2. Seizure of evidence in "plain view," the elements of which are: (a) a prior valid intrusion based on the valid warrantless arrest in which the police are legally present in the pursuit of their official duties; (b) the evidence was inadvertently discovered by the police who had the right to be where they are; (c) the evidence must be immediately apparent[;] and; (d) "plain view" justified mere seizure of evidence without further search. 3. Search of a moving vehicle. Highly regulated by the government, the vehicle's inherent mobility reduces expectation of privacy especially when its transit in public thoroughfares furnishes a highly reasonable suspicion amounting to probable cause that the occupant committed a criminal activity; 4. Consented warrantless search; 5. Customs search; 6. Stop and Frisk; and 7. Exigent and Emergency Circumstances.14

Both the trial court and the CA anchored their respective decisions on the fact that the search was conducted on a moving vehicle to justify the validity of the search. Indeed, the search of a moving vehicle is one of the doctrinally accepted exceptions to the Constitutional mandate that no search or seizure shall be made except by virtue of a warrant issued by a judge after personally determining the existence of probable cause.15 In People v. Bagista,16 the Court said: The constitutional proscription against warrantless searches and seizures admits of certain exceptions. Aside from a search incident to a lawful arrest, a warrantless search had been upheld in cases of a moving vehicle, and the seizure of evidence in plain view. With regard to the search of moving vehicles, this had been justified on the ground that the mobility of motor vehicles makes it possible for the vehicle to be searched to move out of the locality or jurisdiction in which the warrant must be sought. This in no way, however, gives the police officers unlimited discretion to conduct warrantless searches of automobiles in the absence of probable cause. When a vehicle is stopped and subjected to an extensive search, such a warrantless search has been held to be valid only as long as the officers conducting the search have reasonable or probable cause to believe before the search that they will find the instrumentality or evidence pertaining to a crime, in the vehicle to be searched. It is well to remember that in the instances we have recognized as exceptions to the requirement of a judicial warrant, it is necessary that the officer effecting the arrest or seizure must have been impelled to do so because of probable cause. The essential requisite of probable cause must be satisfied before a warrantless search and seizure can be lawfully conducted.17 Without probable cause, the articles seized cannot be admitted in evidence against the person arrested.18 Probable cause is defined as a reasonable ground of suspicion supported by circumstances sufficiently strong in themselves to induce a cautious man to believe that the person accused is guilty of the offense charged. It refers to the existence of such facts and circumstances that can lead a reasonably discreet and prudent man to believe that an offense has been committed, and that the items, articles or objects sought in connection with said offense or subject to seizure and destruction by law are in the place to be searched.19 The grounds of suspicion are reasonable when, in the absence of actual belief of the arresting officers, the suspicion that the person to be arrested is probably guilty of committing the offense is based on actual facts, i.e., supported by circumstances sufficiently strong in themselves to create the probable cause of guilt of the person to be arrested. A reasonable suspicion therefore must be founded on probable cause, coupled with good faith on the part of the peace officers making the arrest.20 Over the years, the rules governing search and seizure have been steadily liberalized whenever a moving vehicle is the object of the search on the basis of practicality. This is so considering that

before a warrant could be obtained, the place, things and persons to be searched must be described to the satisfaction of the issuing judge a requirement which borders on the impossible in instances where moving vehicle is used to transport contraband from one place to another with impunity.21 This exception is easy to understand. A search warrant may readily be obtained when the search is made in a store, dwelling house or other immobile structure. But it is impracticable to obtain a warrant when the search is conducted on a mobile ship, on an aircraft, or in other motor vehicles since they can quickly be moved out of the locality or jurisdiction where the warrant must be sought.22 Given the discussion above, it is readily apparent that the search in this case is valid. The vehicle that carried the contraband or prohibited drugs was about to leave. PO2 Pallayoc had to make a quick decision and act fast. It would be unreasonable to require him to procure a warrant before conducting the search under the circumstances. Time was of the essence in this case. The searching officer had no time to obtain a warrant. Indeed, he only had enough time to board the vehicle before the same left for its destination. It is well to remember that on October 26, 2005, the night before appellants arrest, the police received information that marijuana was to be transported from Barangay Balbalayang, and had set up a checkpoint around the area to intercept the suspects. At dawn of October 27, 2005, PO2 Pallayoc met the secret agent from the Barangay Intelligence Network, who informed him that a baggage of marijuana was loaded on a passenger jeepney about to leave for the poblacion. Thus, PO2 Pallayoc had probable cause to search the packages allegedly containing illegal drugs. This Court has also, time and again, upheld as valid a warrantless search incident to a lawful arrest. Thus, Section 13, Rule 126 of the Rules of Court provides: SEC. 13. Search incident to lawful arrest.A person lawfully arrested may be searched for dangerous weapons or anything which may have been used or constitute proof in the commission of an offense without a search warrant.23 For this rule to apply, it is imperative that there be a prior valid arrest. Although, generally, a warrant is necessary for a valid arrest, the Rules of Court provides the exceptions therefor, to wit: SEC. 5. Arrest without warrant; when lawful.A peace officer or a private person may, without a warrant, arrest a person: (a) When, in his presence, the person to be arrested has committed, is actually committing, or is attempting to commit an offense; (b) When an offense has just been committed and he has probable cause to believe based on personal knowledge of facts or circumstances that the person to be arrested has committed it; and

(c) When the person to be arrested is a prisoner who has escaped from a penal establishment or place where he is serving final judgment or is temporarily confined while his case is pending, or has escaped while being transferred from one confinement to another. In cases falling under paragraphs (a) and (b) above, the person arrested without a warrant shall be forthwith delivered to the nearest police station or jail and shall be proceeded against in accordance with section 7 of Rule 112.24 Be that as it may, we have held that a search substantially contemporaneous with an arrest can precede the arrest if the police has probable cause to make the arrest at the outset of the search.25 Given that the search was valid, appellants arrest based on that search is also valid. Article II, Section 5 of the Comprehensive Dangerous Drugs Act of 2002 states: SEC. 5 Sale, Trading, Administration, Dispensation, Delivery, Distribution and Transportation of Dangerous Drugs and/or Controlled Precursors and Essential Chemicals. The penalty of life imprisonment to death and a fine ranging from Five hundred thousand pesos (P500,000.00) to Ten million pesos (P10,000,000.00) shall be imposed upon any person, who, unless authorized by law, shall sell, trade, administer, dispense, deliver, give away to another, distribute, dispatch in transit or transport any dangerous drug, including any and all species of opium poppy regardless of the quantity and purity involved, or shall act as a broker in any of such transactions. The penalty of imprisonment ranging from twelve (12) years and one (1) day to twenty (20) years and a fine ranging from One hundred thousand pesos (P100,000.00) to Five hundred thousand pesos (P500,000.00) shall be imposed upon any person who, unless authorized by law, shall sell, trade, administer, dispense, deliver, give away to another, distribute, dispatch in transit or transport any controlled precursor and essential chemical, or shall act as a broker in such transactions. In her defense, appellant averred that the packages she was carrying did not belong to her but to a neighbor who had asked her to carry the same for him. This contention, however, is of no consequence. When an accused is charged with illegal possession or transportation of prohibited drugs, the ownership thereof is immaterial. Consequently, proof of ownership of the confiscated marijuana is not necessary.26 Appellants alleged lack of knowledge does not constitute a valid defense. Lack of criminal intent and good faith are not exempting circumstances where the crime charged is malum prohibitum, as in this case.27 Mere possession and/or delivery of a prohibited drug, without legal authority, is punishable under the Dangerous Drugs Act.28 Anti-narcotics laws, like anti-gambling laws, are regulatory statutes. They are rules of convenience designed to secure a more orderly regulation of the affairs of society, and their

violation gives rise to crimes mala prohibita. Laws defining crimes mala prohibita condemn behavior directed not against particular individuals, but against public order.29 Jurisprudence defines "transport" as "to carry or convey from one place to another."30 There is no definitive moment when an accused "transports" a prohibited drug. When the circumstances establish the purpose of an accused to transport and the fact of transportation itself, there should be no question as to the perpetration of the criminal act.31 The fact that there is actual conveyance suffices to support a finding that the act of transporting was committed and it is immaterial whether or not the place of destination is reached.32 Moreover, appellants possession of the packages containing illegal drugs gave rise to the disputable presumption33 that she is the owner of the packages and their contents.34 Appellant failed to rebut this presumption. Her uncorroborated claim of lack of knowledge that she had prohibited drug in her possession is insufficient. Appellants narration of facts deserves little credence. If it is true that Bennie Lao-ang merely asked her and her companion to carry some baggages, it is but logical to first ask what the packages contained and where these would be taken. Likewise, if, as appellant said, Lao-ang ran away after they disembarked from the jeepney, appellant and her companion should have ran after him to give him the bags he had left with them, and not to continue on their journey without knowing where they were taking the bags. Next, appellant argues that the prosecution failed to prove the corpus delicti of the crime. In particular, she alleged that the apprehending police officers failed to follow the procedure in the custody of seized prohibited and regulated drugs, instruments, apparatuses, and articles. In all prosecutions for violation of the Dangerous Drugs Act, the existence of all dangerous drugs is a sine qua non for conviction. The dangerous drug is the very corpus delicti of that crime.35 Thus, Section 21 of R.A. No. 9165 prescribes the procedure for custody and disposition of seized dangerous drugs, to wit: Section 21. Custody and Disposition of Confiscated, Seized, and/or Surrendered Dangerous Drugs, Plant Sources of Dangerous Drugs, Controlled Precursors and Essential Chemicals, Instruments/Paraphernalia and/or Laboratory Equipment. The PDEA shall take charge and have custody of all dangerous drugs, plant sources of dangerous drugs, controlled precursors and essential chemicals, as well as instruments/paraphernalia and/or laboratory equipment so confiscated, seized and/or surrendered, for proper disposition in the following manner: (1) The apprehending team having initial custody and control of the drugs shall, immediately after seizure and confiscation, physically inventory and photograph the same in the presence of the accused or the person/s from whom such items were confiscated and/or seized, or his/her representative or counsel, a representative from the media and the Department of Justice (DOJ), and any elected public official who shall be required to sign the copies of the inventory and be given a copy thereof.

The Implementing Rules and Regulations (IRR) of R.A. No. 9165 further provides: SECTION 21. Custody and Disposition of Confiscated, Seized and/or Surrendered Dangerous Drugs, Plant Sources of Dangerous Drugs, Controlled Precursors and Essential Chemicals, Instruments/Paraphernalia and/or Laboratory Equipment. The PDEA shall take charge and have custody of all dangerous drugs, plant sources of dangerous drugs, controlled precursors and essential chemicals, as well as instruments/paraphernalia and/or laboratory equipment so confiscated, seized and/or surrendered, for proper disposition in the following manner: (a) The apprehending officer/team having initial custody and control of the drugs shall, immediately after seizure and confiscation, physically inventory and photograph the same in the presence of the accused or the person/s from whom such items were confiscated and/or seized, or his/her representative or counsel, a representative from the media and the Department of Justice (DOJ), and any elected public official who shall be required to sign the copies of the inventory and be given a copy thereof: Provided, that the physical inventory and photograph shall be conducted at the place where the search warrant is served; or at the nearest police station or at the nearest office of the apprehending officer/team, whichever is practicable, in case of warrantless seizures; Provided, further, that non-compliance with these requirements under justifiable grounds, as long as the integrity and the evidentiary value of the seized items are properly preserved by the apprehending officer/team, shall not render void and invalid such seizures of and custody over said items. PO2 Pallayoc testified that after apprehending appellant, he immediately brought her to the police station. At the station, the police requested the Mayor to witness the opening of the bags seized from appellant. When the Mayor arrived, he opened the bag in front of appellant and the other police officers. The black bag yielded three bricks of marijuana wrapped in newspaper, while the plastic bag yielded two bundles of marijuana and two bricks of marijuana fruiting tops.36 PO2 Pallayoc identified the bricks. He and PO3 Stanley Campit then marked the same. Then the seized items were brought to the PNP Crime Laboratory for examination. It is admitted that there were no photographs taken of the drugs seized, that appellant was not accompanied by counsel, and that no representative from the media and the DOJ were present. However, this Court has already previously held that non-compliance with Section 21 is not fatal and will not render an accuseds arrest illegal, or make the items seized inadmissible. What is of utmost importance is the preservation of the integrity and evidentiary value of the seized items.37 Based on the testimony of PO2 Pallayoc, after appellants arrest, she was immediately brought to the police station where she stayed while waiting for the Mayor. It was the Mayor who opened the packages, revealing the illegal drugs, which were thereafter marked and sent to the police crime laboratory the following day. Contrary to appellants claim, the prosecutions evidence establishes the chain of custody from the time of appellants arrest until the prohibited drugs were tested at the police crime laboratory. While it is true that the arresting officer failed to state explicitly the justifiable ground for noncompliance with Section 21, this does not necessarily mean that appellants arrest was illegal or

that the items seized are inadmissible. The justifiable ground will remain unknown because appellant did not question the custody and disposition of the items taken from her during the trial.38 Even assuming that the police officers failed to abide by Section 21, appellant should have raised this issue before the trial court. She could have moved for the quashal of the information at the first instance. But she did not. Hence, she is deemed to have waived any objection on the matter. Further, the actions of the police officers, in relation to the procedural rules on the chain of custody, enjoyed the presumption of regularity in the performance of official functions. Courts accord credence and full faith to the testimonies of police authorities, as they are presumed to be performing their duties regularly, absent any convincing proof to the contrary.39 In sum, the prosecution successfully established appellants guilt. Thus, her conviction must be affirmed. WHEREFORE, the foregoing premises considered, the appeal is DISMISSED. The Decision of the Court of Appeals in CA-G.R. CR-HC No. 02718 is AFFIRMED. SO ORDERED.

Espano vs.CourtofAppeals Facts:Sometime July of 1991 , the narcotics division of theWPD conducted a by bust operation in Zamora andPandacan Streets in Manila after the confirmation of drugpushing reports in the same area.During the operation in the area, after the policeofficers saw the accused selling something to a buyer, theyapproached Espano and frisked him. The officers seized twoplastic tea bags of marijuana from the accused. They thenlater asked the accused if there are more marijuana in him,the Espano replied that there are more in his house. Thus,the arresting officers went to Espanos house andsubsequently confiscated ten more plastic tea bags of marijuana.Espano was brought to the police, and charged inviolation with RA 6425. The RTC Br.1of Manila convictedhim in the charges whish was later affirmed in totoby therespondent appellate court. Hence, this petition for review. As defense, the petitioner contends that the drugsconfiscated are inadmissible evidence against him beingsearch and seized illegally.Issue:Was the contention correct?Held:Partly, the drugs confiscated in his person isadmissible, however, the articles sought in his residencecannot be admitted as evidence by the prosecution beingunlawfully seized.Rule 113 Section 5(a) of the Rules of Court providesthat an officer may without a warrant arrest a person when inhis presence:(a) Has committed(b) Is actually committing(c) Or is about to commit a crimeIn the case at bar, petitioner's arrest falls squarely under the aforecited rule. He was caught in flagranti as a result of abuy-bust operation conducted by police officers on the basisof information received regarding the illegal trade of drugswithin the area of Zamora and Pandacan Streets, Manila.The police officer saw petitioner handing over something toan alleged buyer. After the buyer left, they searched him anddiscovered two cellophanes of marijuana. His arrest was,therefore, lawful and the two cellophane bags of marijuanaseized were admissible in evidence, being the fruits of thecrime. As for the ten cellophane bags of marijuana found atpetitioner's residence, however, the same inadmissiblein evidence.The1987 Constitution guarantees freedom againstunreasonable searches and seizures under Article III,Section 2 which provides:

The right of the people to be secure in their persons,houses, papers and effects against unreasonablesearches and seizures of whatever nature and for anypurposes shall be inviolable, and no search warrant or warrant of arrest shall issue except upon probable causeto be determined personally by the judge after examination under oath or affirmation of the complainantand the witnesses he may produce, and particularlydescribing the place to be searched and the persons or things to be seized." An exception to the said rule is a warrantless searchincidental to a lawful arrest for dangerous weapons or anything, which may be used as proof of the commission of an offense. It may extend beyond the person of the onearrested to include the premises or surroundings under hisimmediate control. In this case, the ten cellophane bags of marijuana seized at petitioner's house after his arrest atPandacan and Zamora Streets do not fall under the saidexceptions.The articles seized from petitioner during his arrestwere valid under the doctrine of search made incidental to alawful arrest. The warrantless search made in his house,however, which yielded ten cellophane bags of marijuanabecame unlawful since the police officers were not armedwith a search warrant at the time. Moreover, it was beyondthe reach and control of petitioner.
Carroll v. United States, 267 U.S. 132 (1925), was a decision by the United States Supreme Court which upheld that the warrantless search of an automobile is known as the automobile exception. The case has also been used to increase the scope of warrantless searches.

Federal prohibition officers arranged an undercover buy of liquor from George Carroll, an illicit dealer under investigation, but the transaction was not completed. They later saw Carroll and one Kiro driving on the highway from Detroit to Grand Rapids, which they regularly patrolled. They gave chase, pulled them over, and searched the car, finding illegal liquor behind the rear seat. The National Prohibition Act provided that officers could make warrantless searches of vehicles, boats, or airplanes when they had reason to believe illegal liquor was being transported.[1]

Opinion of the court


The Court noted that Congress early obviated the need for a warrant in border search situations[2], and Congress always recognized a necessary difference between searches of buildings and vehicles for contraband goods, where it is not practical to secure a warrant, because the vehicle can be quickly moved out of the locality or jurisdiction in which the warrant must be sought.[3] The warrantless search under these circumstances was thus valid. The Court held, however, that
[i]t would be intolerable and unreasonable if a prohibition agent were authorized to stop every automobile on the chance of finding liquor, and thus subject all persons lawfully using the highways to the inconvenience and indignity of such a search... . [T]hose lawfully within the country, entitled to use the public highways, have a right to free passage without interruption or search unless there is known to a competent official, authorized to search, probable cause for believing that their vehicles are carrying contraband or illegal merchandise.[4]

The Court added that where the securing of a warrant is reasonably practicable, it must be used.[5]

This became known as the Carroll doctrine: a vehicle could be searched without a warrant if there was probable cause to believe that evidence is present in the vehicle, coupled with exigent circumstances to believe that the vehicle could be removed from the area before a warrant could be obtained.
People v. Lo Ho Wing, 193 SCRA 122 F: Peter Lo , together with co-accused Lim Cheng Huat alias Antonio Lim and Reynaldo Tia, were charged with a violation of the Dangerous Drugs Act, for the transport of metamphetamine hydrochloride, otherwise known as "shabu". The drug was contained in tea bags inside tin cans which were placed inside their luggages. Upon arrival from Hongkong, they boarded the taxis at the airport which were apprehended by CIS operatives. Their luggages were subsequently searched where the tea bags were opened and found to contain shabu. Only Lo and Lim were convicted. Tia was discharged as a state witness, who turned out to be a " deep penetration agent" of the CIS in its mission to bust the drug syndicate . Issue: W/N the search and seizure was legal. HELD: YES That search and seizure must be supported by a valid warrant is not an absolute rule. One of the exceptions thereto is a search of a moving vehicle. The circumstance of the case clearly show that the serach in question was made as regards a moving vehicle. Ads by Google Full-Text Online Library Research online. Books, journals, articles at Questia Online Library. www.Questia.com Buy Bag-kok Online Bags of Top International Brands Free Shipping and 7 Days Return! www.zalora.com.ph/Bag-kok Therefore, a valid warrant was not necessary to effect the search on appellant and his co-accused. It was firmly established from the factual findings of the court that the authorities had reasonable ground to believe that appellant would attempt to bring in contraband and transport within the country. The belief was based on intelligence reports gathered from surveillance activities on the suspected syndicate, of which appellant was touted to be amember. Aside from this, they were also certain as to the expected date and time of arrival of the accused from China via Hongkong. But such knowledge was insufficient to enable them to fulfill the requiremnents for the issuance of a search warrant. Still and all, the important thing is that there was probable cause to conduct the warrantless search, which must still be present in the case.

Source: http://www.shvoong.com/law-and-politics/1767269-case-digest-people-vs-lo/#ixzz1sFwkqx6K

People vs.Malsmedt With substantial dissenting opinions of Justices Narvasa ( CJ)and Cruz

Facts:Upon reports that vehicles coming from Sagada aretransporting marijuana and other prohibited drugs, the Firstregional NARCOM was ordered to set-up a temporarycheckpoint area to monitor all vehicle coming from Sagada.The accused, Mikael Malmstedt, a Swedish nationalwas boarding in the rear end of the bus when the officers areinspecting. One officer saw a bulging object in the waist of the accused and asked Malmstedt to show his passport andother identification documents. The accused failed to complyand was asked to bring out whatever the object bulging in hisstomach. It turns out to be haishish a derivative of marijuana.The accused was invited to step out of the bus for questioning. Before doing so, the accused first get his twotraveling bags from the luggage carrier of the bus. Theofficers inspected the bags and saw a suspicious teddy bear,which turned out to be containing the same materialsconfiscated from the accused.The RTC of La Trinidad Benguet later convicted theaccused with the violation of RA 6425. hence this petition for reversal.The accused contended that his arrest and seizureof his personal effects are illegal there being conductedwithout warrant, therefore follows that articles confiscated isinadmissible evidence against him.Issue:Were the arrest, search and seizure illegal?Held:No. It is legal, being one of the instances providedby statutory provision that warrantless arrest and seizure canbe effected. It was search pursuant to a lawful arrest andsearch in a moving vehicle. Accused was searched and arrested whiletransporting prohibited drugs (hashish). A crime was actuallybeing committed by the accused and he was caught in flagrante delicto . Thus, the search made upon his personaleffects falls squarely under paragraph ( 1 ) of the foregoingprovisions of law ( committed a crime) , which allow awarrantless search incident to a lawful arrest.Note: Dissenting Opinions of Justices Narvasa and Cruz Justice Narvasa: If, on the other, a person is searched without awarrant, or under circumstances other than those justifyingan arrest without warrant in accordance with law, supra,merely on suspicion that he is engaged in some feloniousenterprise, and in order to discover if he has indeedcommitted a crime, it is not only the arrest which is illegal butalso, the search on the occasion thereof, as being "the fruit of the poisonous tree." In that event, any evidence taken, evenif confirmatory of the initial suspicion, is inadmissible "for anypurpose in any proceeding." They are merely fishing for evidence to corroborate their speculations.The search was not made by virtue of a warrant or as an incident of a lawful warrantless arrest, i.e., under circumstances sufficient to engender a reasonable belief thatsome crime was being or about to be committed, or had justbeen committed. There was no intelligent and intentionalwaiver of the right against unreasonable searches andseizure.The search was therefore illegal, since the lawrequires that there first be a lawful arrest of an individualbefore a search of his body and his belongings may licitly bemade. The process cannot be reversed, i.e., a search be firstundertaken, and then an arrest effected, on the strength of the evidence yielded by the search. An arrest made in thatcase would be unlawful, and the search undertaken as anincident of such an unlawful arrest, also unlawful.Search may extend to the area "within his immediate control,defined:The area from which said person arrested mightgain possession of a weapon or destructible evidence.

Mustang Lumber vs.CA A consolidated petitionFacts:First Case:On1April1990 , acting on an information that ahuge stockpile of narra flitches, shorts, and slabs were seeninside the lumberyard of the petitioner in Valenzuela, MetroManila, the SAID organized a team of foresters andpolicemen and sent it to

conduct surveillance at the saidlumberyard. In the course thereof, the team members sawcoming out from the lumberyard the petitioner's truck, withPlate No. CCK-322, loaded with lauan and almaciga lumber of assorted sizes and dimensions. Since the driver could notproduce the required invoices and transport documents, theteam seized the truck together with its cargo and impoundedthem at the DENR compound at Visayas Avenue, QuezonCity. The team was not able to gain entry into the premisesbecause of the refusal of the owner. 2The special Action and nvestigation Division of DENr also procured a search warrant from Jusge AdrianoOsorio of RTC Valenzuela, by virtue of the warrant, the teamseized for truckloads of narra woods including almaciagaand supa. Moreover, the lumberyard of the petitioner wasalso placed under administrative seizure. For failure toproduce certificates of lumber origins, auxiliary invoices, tallysheets and delivery receipts.Subsequently, the Sec of DENR Factoran issued anorder confiscating the woods seized in the truck of thepetitioner as well as those found in their lumberyard. Thus, the petitioner filed a petitioner for certiorariand prohibition contending that the search and seizureoperation by the respondents is a violation under Sec 2 Art IIIof the Constitution for not having a valid search warrant. Second case: PP vs. C apulongetal This case deals with whether the Forestry Codewhere the petitioner allegedly violated refers to either timber or lumber Issue:Was the warrantless search and seizure invalid?Held:No. It is a valid warrantless search being one of thestatutory instances that accepted.Search of a moving vehicle is one of the fivedoctrinally accepted exceptions to the constitutional mandatethat no search and seizure shall be made except by a virtueof a warrant. Thus a search could be lawfully conducted on amoving vehicle without a search warrant.In the case at bar, the conducted search andseizure is indeed without a valid warrant, however, it wasconducted to search the materials that can be found in amoving vehicle, which is the truck of the first case.

Papa vs.Mago Facts:Petitioner Martin Alagao, head of the counter-intelligence unit of the Manila Police Department, acting upona reliable information received on November 3,1966 to theeffect that a certain shipment of personal effects, allegedlymisdeclared and undervalued, would be released thefollowing day from the customs zone of the port of Manilaand loaded on two trucks, and upon orders of petitioner Ricardo Papa, Chief of Police of Manila and a duly deputizedagent of the Bureau of Customs, conducted surveillance atgate No.1of the customs zone.When the trucks left gate No.1at about 4:30in theafternoon of November 4,1966, elements of the counter-intelligence unit went after the trucks and intercepted them atthe Agrifina Circle, Ermita, Manila. The load of the two trucks,consisting of nine bales of goods, and the two trucks, wereseized on instructions of the Chief of Police. Uponinvestigation, a person claimed ownership of the goods andshowed to the policemen a "Statement and Receipts of Duties Collected on Informal Entry No.147-5501", issued bythe Bureau of Customs in the name of a certain BienvenidoNaguit.The respondent Mago, filed a petition for mandamusand certiorari before the CFI Manila contending that thesearch and seizure is illegal for lack of a valid warrant.Moreover, she also contends that such articles sought fromher is not included by the law for prohibited importation andthat it no longer under the control of the Tariff and Customscode for it (articles) were already sold to the petitioner.She also contends that the search seizureconducted by the respondents are illegally being madeoutside the jurisdiction of the BOC and that the subsequentsearch warrant issued by the collector of customs is not validbeing not issued by a judge.The

respondent Mago filed an ex-parte motion torelease the confiscated articles upon her posting a bond.This motion was then granted by the respondent JudgeJarencio.Issue:Was the seizure of the goods unlawful? And that theBOC has no jurisdiction over the articles sought?Held:No. it is a valid seizure.The Chief of the Manila Police Department, RicardoG. Papa, having been deputized in writing by theCommissioner of Customs, could, for the purposes of theenforcement of the customs and tariff laws, effect searches,seizures, and arrests, and it was his duty to make seizure,among others, of any cargo, articles or other movableproperty when the same may be subject to forfeiture or liablefor any fine imposed under customs and tariff laws. He couldlawfully open and examine any box, trunk, envelope or other container wherever found when he had reasonable cause tosuspect the presence therein of dutiable articles introducedinto the Philippines contrary to law; and likewise to stop,search and examine any vehicle, beast or person reasonablysuspected of holding or conveying such article as aforesaid.It cannot be doubted, therefore, that petitioner Ricardo G.Papa, Chief of Police of Manila, could lawfully effect thesearch and seizure of the goods in question. The Tariff andCustoms Code authorizes him to demand assistance of anypolice officer to effect said search and seizure, and the latter has the legal duty to render said assistance. This was whathappened precisely in the case of Lt. Martin Alagao who, withhis unit, made the search and seizure of the two trucksloaded with the nine bales of goods in question at the Agrifina Circle. He was given authority by the Chief of Policeto make the interception of the cargo.Petitioner Martin Alagao and his companionpolicemen had authority to effect the seizure without anysearch warrant issued by a competent court.The Tariff and Customs Code does not require saidwarrant in the instant case. The Code authorizes personshaving police authority under Section 2203 of the Tariff andCustoms Code to enter, pass through or search any land,inclosure, warehouse, store or building, not being a dwellinghouse; and also to inspect, search and examine any vesselor aircraft and any trunk, package, box or envelope or anyperson on board, or stop and search and examine anyvehicle, beast or person suspected of holding or conveyingany dutiable or prohibited article introduced into thePhilippines contrary to law, without mentioning the need of asearch warrant in said cases.It is our considered view, therefore, that except inthe case of the search of a dwelling house, personsexercising police authority under the customs law may effectsearch and seizure without a search warrant in theenforcement of customs laws. Note: The Bureau of Customs has the duties, powers and jurisdiction, among others, (1) to assess and collect all lawfulrevenues from imported articles, and all other dues, fees,charges, fines and penalties, accruing under the tariff andcustoms laws; (2) to prevent and suppress smuggling andother frauds upon the customs; and (3) to enforce tariff andcustoms laws.The goods in question were imported from Hongkong, asshown in the "Statement and Receipts of Duties Collected onInformal Entry." As long as the importation has not beenterminated the imported goods remain under the jurisdictionof the Bureau of Customs. I m portation is deemed terminated only upon thepayment of the duties, taxes and other charges upon thearticles, or secured to be paid, at the port of entry and thelegal permit for withdrawal shall have been granted. The payment of the duties, taxes, fees and other charges must be in full.

Pacis vs. Pa

m aran Facts:The petitioner, Pedro Pacis, acting Collector of Customs in the Port of Manila, issued a warrant of seizureand detention for an automobile (Mercury 1 957) owned bythe respondent Ricardo Santos, who, by the records of hisoffice, have not fully paid the customs duty collectible fromthe car.The respondent Santos filed a suit against thepetitioner on the ground usurpation of justice, for thepetitioner has no power granted by the constitution to issue awarrant, which, is s judicial function of a judge.Issue:Whether or not the petitioner is clothe with jurisdiction to issue a warrant.Held:Yes. The Collector of customs may order seizure of untaxed goods being without being liable for usurpation of judicial function as provided under the tariff and customsCode. Hizon vs. C A Facts:Sometime 1 992, the Maritime Command of the PNPPalawan arrested the petitioner who are allegedly conducting muro-ami a prohibited system of fishing while on board FBrobinson.The PNP filed a complaint against the petitioners(3 1 of 35) charging them in violation of the Fishery Laws inthe Philippines, by using sodium Cyanide in catching fishes.The RTC Puerto princesa convicted the petitionersand was affirmed by the CA., hence this petition. As defense, the petitioners alleged that the searchand seizure is illegal for the absence of a warrant and thefishes sought cannot be admitted as evidence against them.Issue:Was the arrest, search and seizure invalid?Held:No. it is a valid being a search in a moving motor vehicle.Our Constitution proscribes search and seizure andthe arrest of persons without a judicial warrant. 1 6 As ageneral rule, any evidence obtained without a judicial warrantis inadmissible for any purpose in any proceeding. The ruleis, however, subject to certain exceptions. Some of theseare:( 1 ) A search incident to a lawful of arrest;(2) Seizure of evidence in plain view;(3) Search of a moving motor vehicle;and(4) Search in violation of customs laws.Search and seizure without search warrant of vessels and aircrafts for violations of customs laws havebeen the traditional exception to the constitutionalrequirement of a search warrant. It is rooted on therecognition that a vessel and an aircraft, like motor vehicles,can be quickly moved out of the locality or jurisdiction inwhich the search warrant must be sought and secured.Yielding to this reality, judicial authorities have not required asearch warrant of vessels and aircrafts before their searchand seizure can be constitutionally effected.The same exception ought to apply to seizures of fishing vessels and boats breaching our fishery laws. Thesevessels are normally powered by high-speed motors thatenable them to elude arresting ships of the Philippine Navy,the Coast Guard and other government authorities enforcingour fishery laws.Thus, in the case at bar, the warrantless search onthe F/B Robinson, a fishing boat suspected of havingengaged in illegal fishing is valid. The fish and other evidenceseized in the course of the search were properly admitted bythe trial court. People vs. Que Facts: After receiving information that a truck loader withplate number PAD 54 8

contains illegally cut lumber and willpass through Ilocos Norte, the PNP then proceeded andpatrol along the vicinity of General Segundo Ave. Laoag City.Sometime March 1 994, the officers saw the truckwith the plate number indicated, they followed and thenapprehended in Marcos bridge. The officers then arrested thepetitioners for failure to show the necessary documents suchas ( 1 ) Certificate of Lumber Origin (2) Certificate of Transport Agreement (3) Auxiliary invoice and others.The petitioners were charged for violating theForestry Code and was convicted by RTC Laoag.The petitioners filed an appeal before the SCcontending that the search and seizure is illegal and theconfiscated articles cannot be used as evidence againstthem being fruits of a poisonous tree.Issue:Was there an illegal search?Held:No. The search is legal being conducted in searchof a moving vehicle. As held in the PP vs. Bagista explaining theexception and rational of a validity in search of a movingvehicle;With regard to the search of moving vehicles, thishad been justified on the ground that the mobility of motor vehicles makes it possible for the vehicle to be searched tomove out of the locality or jurisdiction in which the warrantmust be sought.This in no way, however, gives the police officersunlimited discretion to conduct warrantless searches of automobiles in the absence of probable cause. When avehicle is stopped and subjected to an extensive search,such a warrantless search has been held to be valid as longas the officers conducting the search have reasonable or probable cause to believe before search that they will find theinstrumentality or evidence pertaining to a crime, in thevehicle to be searched. (citations omitted; emphasissupplied)The police officers in the case at bar had probablecause to search appellant's truck. A member of the ProvincialTask Force on Illegal Logging received a reliable informationthat a ten-wheeler truck bearing plate number PAD-54 8 loaded with illegal lumber would pass through Ilocos Norte.Two weeks later, while members of the Provincial Task Forcewere patrolling along General Segundo Avenue, they saw theten-wheeler truck described by the informant. When theyapprehended it at the Marcos Bridge, accused-appellant, theowner of the truck and the cargo, admitted that there weresawn lumber in between the coconut slabs. When the policeofficers asked for the lumber's supporting documents,accused-appellant could not present any. The foregoingcircumstances are sufficient to prove the existence of probable cause which justified the extensive search of appellant's truck even without a warrant. Thus, the 25 8 pieces of tanguile lumber were lawfully seized and were thus Constitutional Law II (Bill of Rights): Case Briefs: Dennis G. Libunao UC College of Law 37 properly admitted as evidence to prove the guilt of accused-appellant.

G.R. No. 190889

January 10, 2011

ELENITA C. FAJARDO, Petitioner, vs. PEOPLE OF THE PHILIPPINES, Respondent. DECISION

NACHURA, J.: At bar is a Petition for Review on Certiorari under Rule 45 of the Rules of Court, seeking the reversal of the February 10, 2009 Decision1 of the Court of Appeals (CA), which affirmed with modification the August 29, 2006 decision2 of the Regional Trial Court (RTC), Branch 5, Kalibo, Aklan, finding petitioner guilty of violating Presidential Decree (P.D.) No. 1866, as amended. The facts: Petitioner, Elenita Fajardo, and one Zaldy Valerio (Valerio) were charged with violation of P.D. No. 1866, as amended, before the RTC, Branch 5, Kalibo, Aklan, committed as follows: That on or about the 28th day of August, 2002, in the morning, in Barangay Andagao, Municipality of Kalibo, Province of Aklan, Republic of the Philippines, and within the jurisdiction of this Honorable Court, the above-named accused, conspiring, confederating and mutually helping one another, without authority of law, permit or license, did then and there, knowingly, willfully, unlawfully and feloniously have in their possession, custody and control two (2) receivers of caliber .45 pistol, [M]odel [No.] M1911A1 US with SN 763025 and Model [No.] M1911A1 US with defaced serial number, two (2) pieces short magazine of M16 Armalite rifle, thirty-five (35) pieces live M16 ammunition 5.56 caliber and fourteen (14) pieces live caliber .45 ammunition, which items were confiscated and recovered from their possession during a search conducted by members of the Provincial Intelligence Special Operation Group, Aklan Police Provincial Office, Kalibo, Aklan, by virtue of Search Warrant No. 01 (9) 03 issued by OIC Executive Judge Dean Telan of the Regional Trial Court of Aklan.3 When arraigned on March 25, 2004, both pleaded not guilty to the offense charged.4 During pretrial, they agreed to the following stipulation of facts: 1. The search warrant subject of this case exists; 2. Accused Elenita Fajardo is the same person subject of the search warrant in this case who is a resident of Sampaguita Road, Park Homes, Andagao, Kalibo, Aklan; 3. Accused Zaldy Valerio was in the house of Elenita Fajardo in the evening of August 27, 2002 but does not live therein; 4. Both accused were not duly licensed firearm holders; 5. The search warrant was served in the house of accused Elenita Fajardo in the morning of August 28, 2002; and 6. The accused Elenita Fajardo and Valerio were not arrested immediately upon the arrival of the military personnel despite the fact that the latter allegedly saw them in possession of a firearm in the evening of August 27, 2002.5

As culled from the similar factual findings of the RTC and the CA,6 these are the chain of events that led to the filing of the information: In the evening of August 27, 2002, members of the Provincial Intelligence Special Operations Group (PISOG) were instructed by Provincial Director Police Superintendent Edgardo Mendoza (P/Supt. Mendoza) to respond to the complaint of concerned citizens residing on Ilang-Ilang and Sampaguita Roads, Park Homes III Subdivision, Barangay Andagao, Kalibo, Aklan, that armed men drinking liquor at the residence of petitioner were indiscriminately firing guns. Along with the members of the Aklan Police Provincial Office, the elements of the PISOG proceeded to the area. Upon arrival thereat, they noticed that several persons scampered and ran in different directions. The responding team saw Valerio holding two .45 caliber pistols. He fired shots at the policemen before entering the house of petitioner. Petitioner was seen tucking a .45 caliber handgun between her waist and the waistband of her shorts, after which, she entered the house and locked the main door. To prevent any violent commotion, the policemen desisted from entering petitioners house but, in order to deter Valerio from evading apprehension, they cordoned the perimeter of the house as they waited for further instructions from P/Supt. Mendoza. A few minutes later, petitioner went out of the house and negotiated for the pull-out of the police troops. No agreement materialized. At around 2:00 a.m. and 4:00 a.m. of August 28, 2002, Senior Police Officer 2 Clemencio Nava (SPO2 Nava), who was posted at the back portion of the house, saw Valerio emerge twice on top of the house and throw something. The discarded objects landed near the wall of petitioners house and inside the compound of a neighboring residence. SPO2 Nava, together with SPO1 Teodoro Neron and Jerome T. Vega (Vega), radio announcer/reporter of RMN DYKR, as witness, recovered the discarded objects, which turned out to be two (2) receivers of .45 caliber pistol, model no. M1911A1 US, with serial number (SN) 763025, and model no. M1911A1 US, with a defaced serial number. The recovered items were then surrendered to SPO1 Nathaniel A. Tan (SPO1 Tan), Group Investigator, who utilized them in applying for and obtaining a search warrant. The warrant was served on petitioner at 9:30 a.m. Together with a barangay captain, barangay kagawad, and members of the media, as witnesses, the police team proceeded to search petitioners house. The team found and was able to confiscate the following: 1. Two (2) pieces of Short Magazine of M16 Armalite Rifle; 2. Thirty five (35) pieces of live M16 ammos 5.56 Caliber; and 3. Fourteen (14) pieces of live ammos of Caliber 45 pistol. Since petitioner and Valerio failed to present any documents showing their authority to possess the confiscated firearms and the two recovered receivers, a criminal information for violation of P.D. No. 1866, as amended by Republic Act (R.A.) No. 8294, was filed against them.

For their exoneration, petitioner and Valerio argued that the issuance of the search warrant was defective because the allegation contained in the application filed and signed by SPO1 Tan was not based on his personal knowledge. They quoted this pertinent portion of the application: That this application was founded on confidential information received by the Provincial Director, Police Supt. Edgardo Mendoza.7 They further asserted that the execution of the search warrant was infirm since petitioner, who was inside the house at the time of the search, was not asked to accompany the policemen as they explored the place, but was instead ordered to remain in the living room (sala). Petitioner disowned the confiscated items. She refused to sign the inventory/receipt prepared by the raiding team, because the items allegedly belonged to her brother, Benito Fajardo, a staff sergeant of the Philippine Army. Petitioner denied that she had a .45 caliber pistol tucked in her waistband when the raiding team arrived. She averred that such situation was implausible because she was wearing garterized shorts and a spaghetti-strapped hanging blouse.8 Ruling of the RTC The RTC rejected the defenses advanced by accused, holding that the same were already denied in the Orders dated December 31, 2002 and April 20, 2005, respectively denying the Motion to Quash Search Warrant and Demurrer to Evidence. The said Orders were not appealed and have thus attained finality. The RTC also ruled that petitioner and Valerio were estopped from assailing the legality of their arrest since they participated in the trial by presenting evidence for their defense. Likewise, by applying for bail, they have effectively waived such irregularities and defects. In finding the accused liable for illegal possession of firearms, the RTC explained: Zaldy Valerio, the bodyguard of Elenita Fajardo, is a former soldier, having served with the Philippine Army prior to his separation from his service for going on absence without leave (AWOL). With his military background, it is safe to conclude that Zaldy Valerio is familiar with and knowledgeable about different types of firearms and ammunitions. As a former soldier, undoubtedly, he can assemble and disassemble firearms. It must not be de-emphasize[d] that the residence of Elenita Fajardo is definitely not an armory or arsenal which are the usual depositories for firearms, explosives and ammunition. Granting arguendo that those firearms and ammunition were left behind by Benito Fajardo, a member of the Philippine army, the fact remains that it is a government property. If it is so, the residence of Elenita Fajardo is not the proper place to store those items. The logical explanation is that those items are stolen property. xxxx

The rule is that ownership is not an essential element of illegal possession of firearms and ammunition. What the law requires is merely possession which includes not only actual physical possession but also constructive possession or the subjection of the thing to ones control and management. This has to be so if the manifest intent of the law is to be effective. The same evils, the same perils to public security, which the law penalizes exist whether the unlicensed holder of a prohibited weapon be its owner or a borrower. To accomplish the object of this law[,] the proprietary concept of the possession can have no bearing whatsoever. xxxx x x x. [I]n order that one may be found guilty of a violation of the decree, it is sufficient that the accused had no authority or license to possess a firearm, and that he intended to possess the same, even if such possession was made in good faith and without criminal intent. xxxx To convict an accused for illegal possession of firearms and explosive under P.D. 1866, as amended, two (2) essential elements must be indubitably established, viz.: (a) the existence of the subject firearm ammunition or explosive which may be proved by the presentation of the subject firearm or explosive or by the testimony of witnesses who saw accused in possession of the same, and (b) the negative fact that the accused has no license or permit to own or possess the firearm, ammunition or explosive which fact may be established by the testimony or certification of a representative of the PNP Firearms and Explosives Unit that the accused has no license or permit to possess the subject firearm or explosive (Exhibit G). The judicial admission of the accused that they do not have permit or license on the two (2) receivers of caliber .45 pistol, model M1911A1 US with SN 763025 and model M1911A1 of M16 Armalite rifle, thirty-five (35) pieces live M16 ammunition, 5.56 caliber and fourteen (14) pieces live caliber .45 ammunition confiscated and recovered from their possession during the search conducted by members of the PISOG, Aklan Police Provincial Office by virtue of Search Warrant No. 01 (9) 03 fall under Section 4 of Rule 129 of the Revised Rules of Court.9 Consequently, petitioner and Valerio were convicted of illegal possession of firearms and explosives, punishable under paragraph 2, Section 1 of P.D. No. 1866, as amended by R.A. No. 8294, which provides: The penalty of prision mayor in its minimum period and a fine of Thirty thousand pesos (P30,000.00) shall be imposed if the firearm is classified as high powered firearm which includes those with bores bigger in diameter than .38 caliber and 9 millimeter such as caliber .40, .41, .44, .45 and also lesser calibered firearms but considered powerful such as caliber .357 and caliber .22 center-fire magnum and other firearms with firing capability of full automatic and by burst of two or three: Provided, however, That no other crime was committed by the person arrested. Both were sentenced to suffer the penalty of imprisonment of six (6) years and one (1) day to twelve (12) years of prision mayor, and to pay a fine of P30,000.00.

On September 1, 2006, only petitioner filed a Motion for Reconsideration, which was denied in an Order dated October 25, 2006. Petitioner then filed a Notice of Appeal with the CA. Ruling of the CA The CA concurred with the factual findings of the RTC, but disagreed with its conclusions of law, and held that the search warrant was void based on the following observations: [A]t the time of applying for a search warrant, SPO1 Nathaniel A. Tan did not have personal knowledge of the fact that appellants had no license to possess firearms as required by law. For one, he failed to make a categorical statement on that point during the application. Also, he failed to attach to the application a certification to that effect from the Firearms and Explosives Office of the Philippine National Police. x x x, this certification is the best evidence obtainable to prove that appellant indeed has no license or permit to possess a firearm. There was also no explanation given why said certification was not presented, or even deemed no longer necessary, during the application for the warrant. Such vital evidence was simply ignored.10 Resultantly, all firearms and explosives seized inside petitioners residence were declared inadmissible in evidence. However, the 2 receivers recovered by the policemen outside the house of petitioner before the warrant was served were admitted as evidence, pursuant to the plain view doctrine. Accordingly, petitioner and Valerio were convicted of illegal possession of a part of a firearm, punishable under paragraph 1, Section 1 of P.D. No. 1866, as amended. They were sentenced to an indeterminate penalty of three (3) years, six (6) months, and twenty-one (21) days to five (5) years, four (4) months, and twenty (20) days of prision correccional, and ordered to pay a P20,000.00 fine. Petitioner moved for reconsideration,11 but the motion was denied in the CA Resolution dated December 3, 2009.12 Hence, the present recourse. At the onset, it must be emphasized that the information filed against petitioner and Valerio charged duplicitous offenses contrary to Section 13 of Rule 110 of the Rules of Criminal Procedure, viz.: Sec. 13. Duplicity of offense. A complaint or information must charge but one offense, except only in those cases in which existing laws prescribe a single punishment for various offenses. A reading of the information clearly shows that possession of the enumerated articles confiscated from Valerio and petitioner are punishable under separate provisions of Section 1, P.D. No. 1866, as amended by R.A. No. 8294.13 Illegal possession of two (2) pieces of short magazine of M16 Armalite rifle, thirty-five (35) pieces of live M16 ammunition 5.56 caliber, and fourteen (14) pieces of live caliber .45 ammunition is punishable under paragraph 2 of the said section, viz.:

The penalty of prision mayor in its minimum period and a fine of Thirty thousand pesos (P30,000.00) shall be imposed if the firearm is classified as high powered firearm which includes those with bores bigger in diameter than .38 caliber and 9 millimeter such as caliber .40, 41, .44, .45 and also lesser calibered firearms but considered powerful such as caliber .357 and caliber .22 center-fire magnum and other firearms with firing capability of full automatic and by burst of two or three: Provided, however, That no other crime was committed by the person arrested.14 On the other hand, illegal possession of the two (2) receivers of a .45 caliber pistol, model no. M1911A1 US, with SN 763025, and Model M1911A1 US, with a defaced serial number, is penalized under paragraph 1, which states: Sec. 1. Unlawful manufacture, sale, acquisition, disposition or possession of firearms or ammunition or instruments used or intended to be used in the manufacture of firearms or ammunition. The penalty of prision correccional in its maximum period and a fine of not less than Fifteen thousand pesos (P15,000.00) shall be imposed upon any person who shall unlawfully manufacture, deal in, acquire, dispose, or possess any low powered firearm, such as rimfire handgun, .380 or .32 and other firearm of similar firepower, part of firearm, ammunition, or machinery, tool or instrument used or intended to be used in the manufacture of any firearm or ammunition: Provided, That no other crime was committed.15 This is the necessary consequence of the amendment introduced by R.A. No. 8294, which categorized the kinds of firearms proscribed from being possessed without a license, according to their firing power and caliber. R.A. No. 8294 likewise mandated different penalties for illegal possession of firearm according to the above classification, unlike in the old P.D. No. 1866 which set a standard penalty for the illegal possession of any kind of firearm. Section 1 of the old law reads: Section 1. Unlawful Manufacture, Sale, Acquisition, Disposition or Possession of Firearms or Ammunition or Instruments Used or Intended to be Used in the Manufacture of Firearms of Ammunition. The penalty of reclusion temporal in its maximum period to reclusion perpetua shall be imposed upon any person who shall unlawfully manufacture, deal in, acquire dispose, or possess any firearms, part of firearm, ammunition, or machinery, tool or instrument used or intended to be used in the manufacture of any firearm or ammunition. (Emphasis ours.) By virtue of such changes, an information for illegal possession of firearm should now particularly refer to the paragraph of Section 1 under which the seized firearm is classified, and should there be numerous guns confiscated, each must be sorted and then grouped according to the categories stated in Section 1 of R.A. No. 8294, amending P.D. No. 1866. It will no longer suffice to lump all of the seized firearms in one information, and state Section 1, P.D. No. 1866 as the violated provision, as in the instant case,16 because different penalties are imposed by the law, depending on the caliber of the weapon. To do so would result in duplicitous charges. Ordinarily, an information that charges multiple offenses merits a quashal, but petitioner and Valerio failed to raise this issue during arraignment. Their failure constitutes a waiver, and they could be convicted of as many offenses as there were charged in the information.17 This accords propriety to the diverse convictions handed down by the courts a quo.

Further, the charge of illegal possession of firearms and ammunition under paragraph 2, Section 1 of P.D. No. 1866, as amended by R.A. No. 8294, including the validity of the search warrant that led to their confiscation, is now beyond the province of our review since, by virtue of the CAs Decision, petitioner and Valerio have been effectively acquitted from the said charges. The present review is consequently only with regard to the conviction for illegal possession of a part of a firearm. The Issues Petitioner insists on an acquittal and avers that the discovery of the two (2) receivers does not come within the purview of the plain view doctrine. She argues that no valid intrusion was attendant and that no evidence was adduced to prove that she was with Valerio when he threw the receivers. Likewise absent is a positive showing that any of the two receivers recovered by the policemen matched the .45 caliber pistol allegedly seen tucked in the waistband of her shorts when the police elements arrived. Neither is there any proof that petitioner had knowledge of or consented to the alleged throwing of the receivers. Our Ruling We find merit in the petition. First, we rule on the admissibility of the receivers. We hold that the receivers were seized in plain view, hence, admissible. No less than our Constitution recognizes the right of the people to be secure in their persons, houses, papers, and effects against unreasonable searches and seizures. This right is encapsulated in Article III, Section 2, of the Constitution, which states: Sec. 2. The right of the people to be secure in their persons, houses, papers, and effects against unreasonable searches and seizures of whatever nature and for any purpose shall be inviolable, and no search warrant or warrant of arrest shall issue except upon probable cause to be determined personally by the judge after examination under oath or affirmation of the complainant and the witnesses he may produce, and particularly describing the place to be searched and the persons or things to be seized. Complementing this provision is the exclusionary rule embodied in Section 3(2) of the same article (2) Any evidence obtained in violation of this or the preceding section shall be inadmissible for any purpose in any proceeding. There are, however, several well-recognized exceptions to the foregoing rule. Thus, evidence obtained through a warrantless search and seizure may be admissible under any of the following circumstances: (1) search incident to a lawful arrest; (2) search of a moving motor vehicle; (3) search in violation of custom laws; (4) seizure of evidence in plain view; and (5) when the accused himself waives his right against unreasonable searches and seizures.18

Under the plain view doctrine, objects falling in the "plain view" of an officer, who has a right to be in the position to have that view, are subject to seizure and may be presented as evidence.19 It applies when the following requisites concur: (a) the law enforcement officer in search of the evidence has a prior justification for an intrusion or is in a position from which he can view a particular area; (b) the discovery of the evidence in plain view is inadvertent; and (c) it is immediately apparent to the officer that the item he observes may be evidence of a crime, contraband, or otherwise subject to seizure. The law enforcement officer must lawfully make an initial intrusion or properly be in a position from which he can particularly view the area. In the course of such lawful intrusion, he came inadvertently across a piece of evidence incriminating the accused. The object must be open to eye and hand, and its discovery inadvertent.20 Tested against these standards, we find that the seizure of the two receivers of the .45 caliber pistol outside petitioners house falls within the purview of the plain view doctrine. First, the presence of SPO2 Nava at the back of the house and of the other law enforcers around the premises was justified by the fact that petitioner and Valerio were earlier seen respectively holding .45 caliber pistols before they ran inside the structure and sought refuge. The attendant circumstances and the evasive actions of petitioner and Valerio when the law enforcers arrived engendered a reasonable ground for the latter to believe that a crime was being committed. There was thus sufficient probable cause for the policemen to cordon off the house as they waited for daybreak to apply for a search warrant. Secondly, from where he was situated, SPO2 Nava clearly saw, on two different instances, Valerio emerge on top of the subject dwelling and throw suspicious objects. Lastly, considering the earlier sighting of Valerio holding a pistol, SPO2 Nava had reasonable ground to believe that the things thrown might be contraband items, or evidence of the offense they were then suspected of committing. Indeed, when subsequently recovered, they turned out to be two (2) receivers of .45 caliber pistol. The pertinent portions of SPO2 Navas testimony are elucidating: Q When you arrived in that place, you saw policemen? A Yes, sir. Q What were they doing? A They were cordoning the house. Q You said that you asked your assistant team leader Deluso about that incident. What did he tell you? A Deluso told me that a person ran inside the house carrying with him a gun. Q And this house you are referring to is the house which you mentioned is the police officers were surrounding?

A Yes, sir. Q Now, how long did you stay in that place, Mr. Witness? A I stayed there when I arrived at past 10:00 oclock up to 12:00 oclock the following day. Q At about 2:00 oclock in the early morning of August 28, 2002, can you recall where were you? A Yes, sir. Q Where were you? A I was at the back of the house that is being cordoned by the police. Q While you were at the back of this house, do you recall any unusual incident? A Yes, sir. Q Can you tell the Honorable Court what was that incident? A Yes, sir. A person went out at the top of the house and threw something. Q And did you see the person who threw something out of this house? A Yes, sir. xxxx Q Can you tell the Honorable Court who was that person who threw that something outside the house? A It was Zaldy Valerio. COURT: (to witness) Q Before the incident, you know this person Zaldy Valerio? A Yes, sir. Q Why do you know him? A Because we were formerly members of the Armed Forces of the Philippines. xxxx

PROS. PERALTA: Q When you saw something thrown out at the top of the house, did you do something if any? A I shouted to seek cover. xxxx Q So, what else did you do if any after you shouted, "take cover?" A I took hold of a flashlight after five minutes and focused the beam of the flashlight on the place where something was thrown. Q What did you see if any? A I saw there the lower [part] of the receiver of cal. 45. xxxx Q Mr. Witness, at around 4:00 oclock that early morning of August 28, 2002, do you recall another unusual incident? A Yes, sir. Q And can you tell us what was that incident? A I saw a person throwing something there and the one that was thrown fell on top of the roof of another house. Q And you saw that person who again threw something from the rooftop of the house? A Yes, sir. Q Did you recognize him? A Yes, sir. Q Who was that person? A Zaldy Valerio again. xxxx Q Where were you when you saw this Zaldy Valerio thr[o]w something out of the house?

A I was on the road in front of the house. Q Where was Zaldy Valerio when you saw him thr[o]w something out of the house? A He was on top of the house. xxxx Q Later on, were you able to know what was that something thrown out? A Yes, sir. Q What was that? A Another lower receiver of a cal. 45. xxxx Q And what did he tell you? A It [was] on the wall of another house and it [could] be seen right away. xxxx Q What did you do if any? A We waited for the owner of the house to wake up. xxxx Q Who opened the fence for you? A It was a lady who is the owner of the house. Q When you entered the premises of the house of the lady, what did you find? A We saw the lower receiver of this .45 cal. (sic)21 The ensuing recovery of the receivers may have been deliberate; nonetheless, their initial discovery was indubitably inadvertent. It is not crucial that at initial sighting the seized contraband be identified and known to be so. The law merely requires that the law enforcer observes that the seized item may be evidence of a crime, contraband, or otherwise subject to seizure. Hence, as correctly declared by the CA, the two receivers were admissible as evidence. The liability for their possession, however, should fall only on Valerio and not on petitioner.1avvphil

The foregoing disquisition notwithstanding, we find that petitioner is not liable for illegal possession of part of a firearm. In dissecting how and when liability for illegal possession of firearms attaches, the following disquisitions in People v. De Gracia22 are instructive: The rule is that ownership is not an essential element of illegal possession of firearms and ammunition. What the law requires is merely possession which includes not only actual physical possession but also constructive possession or the subjection of the thing to one's control and management. This has to be so if the manifest intent of the law is to be effective. The same evils, the same perils to public security, which the law penalizes exist whether the unlicensed holder of a prohibited weapon be its owner or a borrower. To accomplish the object of this law the proprietary concept of the possession can have no bearing whatsoever. But is the mere fact of physical or constructive possession sufficient to convict a person for unlawful possession of firearms or must there be an intent to possess to constitute a violation of the law? This query assumes significance since the offense of illegal possession of firearms is a malum prohibitum punished by a special law, in which case good faith and absence of criminal intent are not valid defenses. When the crime is punished by a special law, as a rule, intent to commit the crime is not necessary. It is sufficient that the offender has the intent to perpetrate the act prohibited by the special law. Intent to commit the crime and intent to perpetrate the act must be distinguished. A person may not have consciously intended to commit a crime; but he did intend to commit an act, and that act is, by the very nature of things, the crime itself. In the first (intent to commit the crime), there must be criminal intent; in the second (intent to perpetrate the act) it is enough that the prohibited act is done freely and consciously. In the present case, a distinction should be made between criminal intent and intent to possess. While mere possession, without criminal intent, is sufficient to convict a person for illegal possession of a firearm, it must still be shown that there was animus possidendi or an intent to possess on the part of the accused. Such intent to possess is, however, without regard to any other criminal or felonious intent which the accused may have harbored in possessing the firearm. Criminal intent here refers to the intention of the accused to commit an offense with the use of an unlicensed firearm. This is not important in convicting a person under Presidential Decree No. 1866. Hence, in order that one may be found guilty of a violation of the decree, it is sufficient that the accused had no authority or license to possess a firearm, and that he intended to possess the same, even if such possession was made in good faith and without criminal intent. Concomitantly, a temporary, incidental, casual, or harmless possession or control of a firearm cannot be considered a violation of a statute prohibiting the possession of this kind of weapon, such as Presidential Decree No. 1866. Thus, although there is physical or constructive possession, for as long as the animus possidendi is absent, there is no offense committed.23 Certainly, illegal possession of firearms, or, in this case, part of a firearm, is committed when the holder thereof:

(1) possesses a firearm or a part thereof (2) lacks the authority or license to possess the firearm.24 We find that petitioner was neither in physical nor constructive possession of the subject receivers. The testimony of SPO2 Nava clearly bared that he only saw Valerio on top of the house when the receivers were thrown. None of the witnesses saw petitioner holding the receivers, before or during their disposal. At the very least, petitioners possession of the receivers was merely incidental because Valerio, the one in actual physical possession, was seen at the rooftop of petitioners house. Absent any evidence pointing to petitioners participation, knowledge or consent in Valerios actions, she cannot be held liable for illegal possession of the receivers. Petitioners apparent liability for illegal possession of part of a firearm can only proceed from the assumption that one of the thrown receivers matches the gun seen tucked in the waistband of her shorts earlier that night. Unfortunately, the prosecution failed to convert such assumption into concrete evidence. Mere speculations and probabilities cannot substitute for proof required to establish the guilt of an accused beyond reasonable doubt. The rule is the same whether the offenses are punishable under the Revised Penal Code, which are mala in se, or in crimes, which are malum prohibitum by virtue of special law.25 The quantum of proof required by law was not adequately met in this case in so far as petitioner is concerned. The gun allegedly seen tucked in petitioners waistband was not identified with sufficient particularity; as such, it is impossible to match the same with any of the seized receivers. Moreover, SPO1 Tan categorically stated that he saw Valerio holding two guns when he and the rest of the PISOG arrived in petitioners house. It is not unlikely then that the receivers later on discarded were components of the two (2) pistols seen with Valerio. These findings also debunk the allegation in the information that petitioner conspired with Valerio in committing illegal possession of part of a firearm. There is no evidence indubitably proving that petitioner participated in the decision to commit the criminal act committed by Valerio. Hence, this Court is constrained to acquit petitioner on the ground of reasonable doubt. The constitutional presumption of innocence in her favor was not adequately overcome by the evidence adduced by the prosecution. The CA correctly convicted Valerio with illegal possession of part of a firearm. In illegal possession of a firearm, two (2) things must be shown to exist: (a) the existence of the subject firearm; and (b) the fact that the accused who possessed the same does not have the corresponding license for it.26

By analogy then, a successful conviction for illegal possession of part of a firearm must yield these requisites: (a) the existence of the part of the firearm; and (b) the accused who possessed the same does not have the license for the firearm to which the seized part/component corresponds. In the instant case, the prosecution proved beyond reasonable doubt the elements of the crime. The subject receivers - one with the markings "United States Property" and the other bearing Serial No. 763025 - were duly presented to the court as Exhibits E and E-1, respectively. They were also identified by SPO2 Nava as the firearm parts he retrieved af ter Valerio discarded them.27 His testimony was corroborated by DYKR radio announcer Vega, who witnessed the recovery of the receivers.28 Anent the lack of authority, SPO1 Tan testified that, upon verification, it was ascertained that Valerio is not a duly licensed/registered firearm holder of any type, kind, or caliber of firearms.29 To substantiate his statement, he submitted a certification30 to that effect and identified the same in court.31 The testimony of SPO1 Tan, or the certification, would suffice to prove beyond reasonable doubt the second element.32 WHEREFORE, premises considered, the February 10, 2009 Decision of the Court of Appeals is hereby REVERSED with respect to petitioner Elenita Fajardo y Castro, who is hereby ACQUITTED on the ground that her guilt was not proved beyond reasonable doubt. SO ORDERED.

PP. vs. Valdez Facts: After a tip from a source hat there is a plantation of marijuana owned by the appellant at Sitio Bulan VillaverdeNV, the PNP Nueva Vizcaya immediately formed anoperation to verify the report with an instruction to uprootsaid marijuana plants and arrest the cultivator of the same.Upon reaching their destination, the police sayseven five-foot height marijuana flowering tops situatedapproximately 25 meters from the accused. Valdez admittedthat the plants are his, thus, the police uprooted the plantsand subject it to forensic analysis which was later foundpositive to be a marijuana. The accused was arrestedThe lower court was convicted and sentenced tosuffer the capital punishment of death. Hence this automaticappeal.The petitioner through counsel contended that thesearch is unlawful since the officers has ample time to securea search warrant, thereby the evidences cannot be admittedas evidence against him being a fruit of a proverbial poisonedtree.The respondent, through the solicitor General aversthat the search is lawful since it is one of the considered asplain view doctrine/ search.Issue:Whether the search is in accordance with the plainview doctrine.Held:The search is unlawful and cannot be classified as aplain view search. As ingrained in jurisprudence, if the arrestingofficers has ample time to secure a arrest warrant, speciallywhen they all have the information needed to identify theplace and the person sought to be searched and/or arrested,search warrant is needed. However, statutory andconstitutional provisions also allows instances wherewarrants are dispensable. One of which is search of plainview or the plain view doctrine.For the doctrine to apply, the following requisitesshould be present.(a) A prior valid intrusion based on thevalid warrantless arrest in

which thepolice are legally present in the pursuitof their official duties;(b) The evidence was inadvertentlydiscovered by the police who have theright to be where they are; and(c) The evidence must be immediatelyapparent; and(d) Plain view justified mere seizure of evidence without further search.Thus, the plain view doctrine applies only where thepolice officer is not searching for evidence against theaccused, but inadvertently comes across an incriminatingobject.In the case at bar, their discovery of the plant is notinadvertent. It is clear from the records and the orders of theofficer that they have to uproot the cannabis plants whenseen and that the officers testified that they first located themarijuana before the arrest of Valdez.Thus, the search is illegal and that the confiscatedplants cannot be admitted as evidence being a fruit of apoisonous tree. Note: The guarantee refers to "the right of personalsecurity" of the individual. As appellant correctly points out,what is sought to be protected against the State's unlawfulintrusion are persons, not places. To conclude otherwisewould not only mean swimming against the stream, it wouldalso lead to the absurd logic that for a person to be immuneagainst unreasonable searches and seizures, he must be inhis home or office, within a fenced yard or a private place.The Bill of Rights belongs as much to the person in the streetas to the individual in the sanctuary of his bedroom.
Harris v. United States, 390 U.S. 234 (1968)

Harris v. United States No. 92 Argued January 18, 1968 Decided March 5, 1968 390 U.S. 234 Syllabus Pursuant to a departmental regulation, a police officer searched a impounded car held as evidence of a robbery. The search completed, the officer opened the car door for the purpose of rolling up a window and thus protecting the car and its contents. On opening the door, the officer saw, exposed to plain view, the automobile registration card belonging to the victim of the robbery. This card was used as evidence in petitioner's trial. Petitioner's conviction was affirmed by the Court of Appeals over his contention that the card had been illegally seized following a warrantless search. Held: The card was subject to seizure and introducible in evidence, since it was not discovered by means of a search in the technical sense, but was plainly visible to the officer who had a right to be in a position of viewing it. 125 U.S.App.D.C. 231, 370 F.2d 477, affirmed.

Official Supreme Court caselaw is only found in the print version of the United States Reports. Justia caselaw is provided for general informational purposes only, and may not reflect current legal developments, verdicts or settlements. We make no warranties or guarantees about the accuracy, completeness, or adequacy of the information contained on this site or information linked to from this site. Please check official sources.

People vs. Damaso Facts: After a sequence of arrest among members of theNPA, the Philippine Constabulary officers of LingayenPangasinan, the apprehended NPA (Luz Tancianco)members pointed out to the PC the house of the AppellantDamaso where the same is leasing.Being pointed out that the appellant is the lessee of the house, the police went inside and eventually saw M-14rifles, radio sets, subversive materials and pamphlets, maps,computer machines, bullets and ammunitions. These articlesare confiscated to use as evidence against the accused inthe crime of Subversion.He was then convicted by the RTC Dagupan in thecrime of subversion, hence this appeal.The appellant contends that the seizure in his houseis illegal for absence of a search warrant.Issue:Was the search unlawful?Held:Yes. The search n the house of the appellant isillegal because of absence of search warrant and evenprobable cause for the issuance of the same.In the testimony of the witness by the prosecution,he has no personal knowledge making its testimonieshearsay and weak to establish the existence of a probablecause.Moreover, the search conducted is not one beingqualified as a valid search without warrant(1) Search incidental to an arrest.(2) Customs search.(3) Consented search(4) Search of a moving vehicle(5) Stop frisk(6) Plain view search.(Apply theMendoza doctrine- incidental discovery of incriminating evidence to qualify as a plain view search)Moreover, the rights granted under the Bill of Rights ispersonal and that cannot be waived by anyone else rather than the person whose rights is invaded or one whoexpressly to do so in his behalf.In the case at bar, the prosecution did notestablished that Luz Tancianco was authorized by theappellant to allow the officers to enter the appellants houseand seized the effects of the appellant upon seeing it inpresumption that it is connected with the crime of subversion.Thereby violating the appellants rights of privacy and securityof house and effects.Thus, the prosecutions evidences are weak andthat the search being illegal, the articles sought areinadmissible as evidences being a fruit of poisonous tree. People vs. Velasco Facts: A close surveillance by the Narcotics unit of theWPD confirmed that the Accused, Yolanda Velasco akaShabu Queen is indeed engage in peedling shabu, aregulated drug, along Quirada St. in Manila. After the confirmation, the WPD organized a buy-bust operation involving a poseur and marked money. Theoperation was successful and the appellant was caught inflagranti delicto, was arrested and brought to precinct for aninvestigation. The confiscated five decks of shabu werefound in the accuseds pockets.Hence this petition, arguing that the articles soughtcannot be used as an evidence since the search was illegalbeing obtained through a warrantless arrest. She also allegethat she was framed-up by the officers.Issue:Was there an illegal warrantless arrest?Held:No. it being one of the exceptions provided bystatutory provision.The RRC provides that an officer, in his presence,can arrest even without a search or arrest warrant when theperson arrested actually committed, committing or attemptingto commit a crime.In the case at bar, the accused was caught inflagranti delicto (in the act of committing the crime), thus it justified the warrantless arrest conducted by the officer, thus,debunk the petitioner contention of frame up andinadmissibility of evidence against her.No evidence presented by the petitioner that thearresting officers are engage in unscrupulous discharge of their duty to back up her contention of a frame up. People vs. Leangsiri Facts:Suchinda Leangsiri was arrested in the NAIA in theact of bringing into the country approximately more than * kiloof heroin. In his arrest, he informed the arresting officers thatthe heroin is meant to deliver to three other

people in LasPalmas Hotel in Manila.Immediately, the NARCOM formed a group for afollow up operation in the said Hotel. In the accusedscooperation, he was allowed to check in to Room 5 0 4, wherethe others will meet him to give the drugs. Around 10 pm, Amidu two other co-appellantsentered Room 5 0 4 and Leagsiri gave them the drugs, beforethe appellants leave the room, the NARCOM officers bargedin and arrested the appellants. Amidu, told the officers that he is staying in Rm 4 1 3and that the two others are in royal Palm Hotel. The officersthen went to the room of Amidu, searching for evidence andsubsequently confiscated a telephone address bearing thename of Leangsiri, other possessions and documents of Amidu were also confiscated.In the case of the two other, the police confiscated asuit case and masking tape and empty transparent bag,allegedly will be use in transporting the drugs.The appellants were charged and was convicted inconspiring to transport heroin violative of RA 6425. Hencethis petition, alleging that the search is illegal beingconducted not in the direct premises of the arrest.Issue:Whether the articles sought in the other room andhotel, outside the direct premises of the arrest admissible asevidence?Held:No. those article are inadmissible as evidence as itwas obtained not in plan view nor within the direct premisesof the arrest.The plain view doctrin applies to OBJECTS OF THEPLAIN VIEW OF AN OFFICER WHO HAS THE RIGHT TOBE IN THE POSITION TO HAVE THAT VIEW ARESUBJECT TO SEIZURE AND MAYBE PRESENTED ASEVIDENCE.Thus, what can be admitted are evidences seizedwithin the direct premise where the accused has animmediate control which should only be Rm. 5 0 9.In the case at bar, appellants were arrested inRoom 5 0 4 of the Las Palmas Hotel. The piece of paper bearing Leangsiri's name was obtained through awarrantless search of Room 4 1 3 of the same hotel, andfound tucked within the pages of appellant Amidu's telephoneand address book. Clearly, the warrantless search is illegaland the piece of paper bearing Leangsiri's name cannot beadmitted as evidence against appellants. The inadmissibilityof this evidence will not, however, exculpate appellants. Itsexclusion does not destroy the prosecution's case againstappellants. The remaining evidence still established their guiltbeyond reasonable doubt. Note:Plain view" doctrine is usually applied where apolice officer is not, searching for evidence against theaccused, but nonetheless inadvertently comes across anincriminating object.Furthermore, the U.S. Supreme Court stated thefollowing limitations on the application of the doctrine. W hat the "plain view" cases have in co mm on i sthat the police officer in each of them had a prior justificationfor an intrusion in the course of which he came inadvertentlyacross a piece of evidence incriminating the accused. Thedoctrine serves to supplement the prior justification whether it be a warrant for another object, hot pursuit, search incidentto lawful arrest, or some other legitimate reason for beingpresent unconnected with a search directed against theaccused and permits the warrantless seizure. Of course, theextension of the original justification is legitimate only whereit is immediately

apparent to the police that they haveevidence before them; the "plain view" doctrine may not beused to extend a general exploratory search from one objectto another until something incriminating at last emerges. W hen there is waiver of right or gives hisconsent A Valid waiver of right in Sec 2 art III, elements( 1 ) The right to be waived is existing(2) The person waiving it had knowledge, actual or constructive(3) He or she has actual intention to relinquish the right.

People vs. Baula Facts: After the gruesome killing of Patronicia Caburao inthe municipality of Sual, Pangasinan, the investigating policewent to the residence of the accused-appellant, Baula et al.In the process of questioning the appellants, thepolice saw bloodstained bolo, short pants, polo shirts andwas subsequently confiscated without search warrant anddirected to the NBI for forensic exams. The exam resultedthat the bloods found in the confiscated articles bears thesame blood type O as that of the victim. Thus, the accused were arrested, charged and wasconvicted in the crime of murder by the RTC Lingayen andsentenced to suffer RP. Hence this appeal for review on thedecision of the lower court in the ground that the articlessought (bloodstained bolo, shirt and short pants) cannot beadmitted as evidence against the accused since it wasseized without a valid search and seizure warrant.Issue:Whether the articles are validly seized even withouta valid search warrant and therefore admitted as evidence inthe case at bar.Whether the articles when with consent given to theofficers can be admitted as an evidence?Held:The articles are unlawfully searched and seized. A search incidental to a valid arrest is one of thestatutory exceptions to the constitutional mandate that nosearch and seizure shall be effected without a valid warrant.In this instance, the arrest should be lawful before searchand seizure by the arresting officer would be conducted. Awarrantless arrest may be effected by the arresting officer when in his presence the person arrested is have committed,committing or attempting to commit the crime. It cannot bereversed; otherwise, it would unlawful and unconstitutionaland the seized article would be inadmissible evidence.In the case at bar, Accused-appellants were notbeing arrested at the time that the subject articles wereallegedly taken from them but were just being questioned bythe police officers conducting the investigation about thedeath of Patrocinia Caburao. The investigating officers hadno personal knowledge of facts indicating that the accusedhad committed the crime. Being in no position to effect awarrantless arrest, the police officers were thus likewisebarred from effecting a warrantless search and seizure. An illegal search cannot be undertaken and then anarrest effected on the strength of the evidence yielded by thatsearch.The Court finds it less than credible the stance of the prosecution that the polo shirt and short pants have beenvoluntarily given. An alleged consent to a warrantless searchand seizure cannot be based merely on the presumption of regularity in the performance of duty. This presumption, byitself, cannot prevail against the constitutionally protectedrights of an individual, and zeal in the pursuit of criminalscannot ennoble the use of arbitrary methods that theConstitution itself abhors.Thus, the bloodstained polo, bolo and shorts areinadmissible as evidence. Veroy vs. Layaguwe Facts:The petitioners, Leopoldo Veroy has a residentialhouse in K-8th St. Quezon City where it is under the care of two houseboys and a certain Soquilon. The bedrooms in thishouse are constantly locked and access to the kitchen is theonly key to the caretakers.Sometime1990 , a directive issue was ordered toCapt. Obrero to search the house of the petitioner in thereport that their residence is a hideout and recruiting site of rebel soldiers.They were able to enter the yard with the help of thecaretakers but did not enter the house since the owner wasnot present and they did not have a search warrant.Petitioner Ma. Luisa

was contacted by telephone in her Quezon City residence by Capt. Obrero to ask permission tosearch the house in Davao City as it was reportedly beingused as a hideout and recruitment center of rebel soldiers.Petitioner Ma. Luisa Veroy responded that she is flying toDavao City to witness the search but relented if the searchwould not be conducted in the presence of Major ErnestoMacasaet, an officer of the PC/INP, Davao City and a longtime family friend of the Veroys.The authority given by Ma. Luisa Veroy was relayedby Capt. Obrero to Major Macasaet who answered that Ma.Luisa Veroy has called him twice by telephone on the matter and that the permission was given on the condition that thesearch be conducted in his presence.The officers in the following day with Maj. Macasaet,by virtue of the authority granted by the petitioner, gained anentrance to the house and first search the masters andchildrens bedroom, where they saw and confiscated .45 calhandgun with live bullets, printed materials of RAM-SFP,traveling bag with assorted clothes, telescope, map, a book,medicines and religious pamphlets.The petitioners were then charged with a criminalcomplaint in the crime of illegal possession of Firearms and Ammunitions in furtherance with Rebellion, and that theprosecution recommended no bail. Due to stress and anxiety,the petitioners were confined in a hospital and filed Motionsfor Bail and Hospital Confinement under the respondent judge Layague, which was denied.Then an order was issued by the respondent judgeto transfer the petitioners from St Lukes to Camp Crame.Hence this petition. The petitioners question the admissibilityin evidence of the articles seized in violation of their constitutional right against unreasonable searches andseizure.Petitioners aver that while they concede that Capt.Obrero had permission from Ma. Luisa Veroy to break openthe door of their residence, it was merely for the purpose of ascertaining thereat the presence of the alleged "rebel"soldiers. The permission did not include any authority toconduct a room-to-room search once inside the house. Theitems taken were, therefore, products of an illegal search,violative of their constitutional rights. As such, they areinadmissible in evidence against them.Issue:Does the articles seized unlawful and cannot beadmitted as evidence? or Does the consent given by the petitioners make thesearch and seizure lawful?Held:The articles are illegally seized thereforeinadmissible as evidence.The Constitution guarantees the right of the peopleto be secure in their persons, houses, papers and effectsagainst unreasonable searches and seizures (Article III,Section 2 of the1987 Constitution).However, the rule that searches and seizures mustbe supported by a valid warrant is not an absolute one. Among the recognized exceptions thereto are: (1)a searchincidental to an arrest; (2)a search of a moving vehicle; and (3)seizure of evidence in plain viewNone of these exceptions pertains to the case atbar. The reason for searching the house of herein petitioner is that it was reportedly being used as a hideout andrecruitment center for rebel soldiers. While Capt. Obrero wasable to enter the compound, he did not enter the housebecause he did not have a search warrant and the ownerswere not present. This shows that he himself recognized theneed for a search warrant, hence, he did not persist inentering the house but rather contacted the Veroys to seekpermission to enter the same.Permission was indeed granted by Ma. Luisa Veroyto enter the house but only to ascertain the presence of rebelsoldiers. Under the circumstances it is undeniable that the police officers had ample time to procure a search warrantbut did not.In a number of cases decided by this ,Court,(Guazon v. De Villa, supra.; People v. Aminnudin), warrantless searches were declared illegal because theofficials conducting the search had every opportunity tosecure a search warrant . The objects seized, beingproducts of illegal searches, were inadmissible in evidence inthe criminal actions subsequently instituted against theaccused-appellantsUndeniably, the offense of illegal possession of firearms is malum prohibitum but it does not follow that thesubject thereof is necessarily illegal per se.Motive is immaterial in mala prohibita but thesubjects of this kind of offense may not be summarily seizedsimply because they are prohibited. A search warrant is stillnecessary. Hence, the rule having been violated and noexception being applicable, the articles seized

wereconfiscated illegally and are therefore protected by theexclusionary principle. They cannot be used as evidenceagainst the petitioners in the criminal action against them for illegal possession of firearms.

De Garcia vs. Locsin Facts:Mariano Almeda, an agent of Anti-usury Boardapplied for a search warrant before the respondent judgeagainst the petitioner, believing that there is a probablecause that the petitioner, Leona De Garcia, keeps andconseals in her house and store at Victoria Tarlac certainbooks, lists, chits, receipts and documents relating to her activities as usurer, contrary to law.The search warrant was granted and with officers of the law, Almeda et al search the person and house/store of the petitioner to seized evidence related to usury.The search proceeded without the presence of thepetitioner, and thus, packages of records were confiscated,turned over to the fiscal by the Anti-Usury Board and waslater filed six separate complaint against the petitioner for violation of the Anti-usury law.Thus, a motion was filed by the petitioner demanding the respondent Board to return the articles seizedin the search in the ground of its invalidity. The respondent judge denied such motion and contends that even though thesearch warrant is invalid, the articles are still admissible asevidence since there is a waiver of right on the part of thepetitioner.Issue:Was there a waiver of right?If there is, would it make the search valid andtherefore articles seized are admissible evidence?Held:No. There is no waiver of right and that the articlesare inadmissible evidence.The constitutional immunity against unreasonablesearches and seizures is a personal right which may bewaived.The waiver may be either express or implied.In the case at bar, no express waiver has beenmade. It is urged, however, that there has been a waiver byimplication. It is well-settled that to constitute a waiver of aconstitutional right, it must appear,( 1 ) First, that the right exists;(2) Secondly, that the person involved hadknowledge, either actual or constructive, of theexistence of such right; and,(3) Lastly, that said person had an actual intentionto relinquish the right.It is true that the petitioner did not object to thelegality of the search when it was made. She could not haveobjected because she was sick and was not present whenthe warrant was served upon Alfredo Salas. Certainly, theconstitutional immunity from unreasonable searches andseizures, being a personal one, cannot be waived by anyoneexcept the person whose rights are invaded or one who isexpressly authorized to do so in his or her behalf.Of course, the petitioner came to know later of theseizure of some of her papers and documents. But this wasprecisely the reason why she sent her attorneys to the officeof the Anti-Usury Board to demand the return of thedocuments seized. In any event, the failure on the part of thepetitioner and her bookkeeper to resist or object to theexecution of the warrant does not constitute an impliedwaiver of constitutional right. As the constitutional guaranty is not dependentupon any affirmative act of the citizen, the courts do not placethe citizen in the position of either contesting an officer'sauthority by force, or waiving his constitutional rights; butinstead they hold that a peaceful submission to a search or seizure is not a consent or an invitation thereto, but is merelya demonstration of regard for the supremacy of the law.Moreover, the invalidity of the search warrant notonly anchored in the mentioned ground, but also, therespondent judge did not follow the constitutionalrequirement in determining probable cause to issue warrants,such that, it is the applicant that determines the probablecause in the case at bar. In 35 constitution, it mandated the judge to personally determine the existence of probablecause and no other. Lopez and Velasco vs. Commissioner of Customs Confusing case where it is adverse to later jurisprudence that a waiver of right can only be waived by the person whoseright against unreasonable search and seizure was invaded. Facts: After the surveillance conducted by the respondentsNBI officers, NBI and PC Davao officers went to the room(Rm 22 0

) rented by the petitioner, Tomas Velasco, to searchand seized articles papers and documents including a .45 calpistol, that became evidence that commodities confiscated inthe wharf (MV Jojo Lema) allegedly smuggled fromIndonesia to the country (sacks of coffee beans and copra).The search was without a search warrant, however,the officers have successfully confiscated the articles byvirtue of the consent of the petitioners wife (Teofila ibanez)who also is an occupant of the room rented by the petitioner.The petitioner contends that the consent given byIbanez cannot be regarded since she was not the legal wifeof the petitioner Velasco, but a certain Corazon Velasco.The decision of CTA is adverse by the petitioner,affirmed by the SC.Issue:Was the consent valid to justify the warrantlesssearch and seizure?Held: As far as the decision is concerned, yes, theconsent given by Ibanez is a valid so as to dispense thenecessity of a search warrant.The court ruled that the mere fact that Ibanez ispresent in a room rented by the petitioner, her consent wouldlead to belief that her consent as an alleged wife of thepetitioner and that it would be an act on behalf of thepetitioner. Note: Exact decision penned by J. LaurelThere was an attempt on the part of petitioners tocounteract the force of the above recital by an affidavit of oneCorazon Y. Velasco, who stated that she is the legal wife of petitioner Tomas Velasco, and another by such petitioner himself reiterating such a fact and that the person who waspresent at his hotel room was one Teofila Ibaez, "amanicurist by occupation." Their effort appurtenant thereto isdoomed to failure. If such indeed were the case, then it ismuch more easily understandable why that person, TeofilaIbaez, who could be aptly described as the wrong person atthe wrong place and at the wrong time, would have signifiedher consent readily and immediately.Under the circumstances, that was the most prudentcourse of action. It would save her and even petitioner Velasco himself from any gossip or innuendo. Nor could theofficers of the law be blamed if they would act on theappearances. There was a person inside who from allindications was ready to accede to their request. Evencommon courtesy alone would have precluded them frominquiring too closely as to why she was there. Under all thecircumstances, therefore, it can readily be concluded thatthere was consent sufficient in law to dispense with the needfor a search warrant. The petition cannot, therefore, prevail.

Stop and Frisk Doctrine Defined: Stop-and-frisk was defined as the vernacular designation of the right of a police officer to stop a citizen onthe street, interrogate him, and pat him for weapon

G.R. No. 182010

August 25, 2010

SUSAN ESQUILLO Y ROMINES, Petitioner, vs. PEOPLE OF THE PHILIPPINES, Respondent. DECISION CARPIO MORALES, J.: Via petition erroneously captioned as one for Certiorari, Susan Esquillo y Romines (petitioner) challenges the November 27, 2007 Decision1 of the Court of Appeals in CA-G.R. CR No. 27894 which affirmed the July 28, 2003 Decision of Branch 116 of the Regional Trial Court (RTC) of

Pasay City in Criminal Case No. 02-2297 convicting Susan Esquillo y Romines (petitioner) for violating Section 11, Article II of Republic Act (R.A.) No. 9165 (the Comprehensive Dangerous Drugs Act of 2002) possession of methamphetamine hydrochloride or shabu. The accusatory portion of the Information dated December 12, 2002 indicting petitioner reads: That on or about the 10th day of December, 2002 in Pasay City, Metro Manila, Philippines and within the jurisdiction of this Honorable Court, the above-named accused, without authority of law, did then and there willfully, unlawfully and feloniously have in her possession, custody and control 0.1224 gram of Methylamphetamine Hydrochloride (shabu).2 (underscoring supplied) At the trial, petitioner admitted the genuineness and due execution of the documentary evidence of the prosecution, particularly the Dangerous Drugs and Toxicology Reports issued by National Bureau of Investigation (NBI) Forensic Chemist Antonino de Belen (de Belen),3 subject to her defenses, to thus dispense with the testimony of de Belen. De Belen recorded the results of the laboratory examination of the contents of the sachet in Dangerous Drugs Report No. DD-02-613,4 viz: xxxx SPECIMEN: White crystalline substance contained in a heat-sealed transparent plastic sachet marked "SRE" and further placed in bigger marked transparent plastic sachet. xxxx F I N D I N G S: Net Weight of specimen = 0.1224 gram Examinations conducted on the above-mentioned specimen gave POSITIVE RESULTS for METHAMPHETAMINE HYDROCHLORIDE, a dangerous drug. x x x x x x x (emphasis and underscoring supplied) With respect to the examination of the urine of petitioner, de Belen recorded the results thereof in Toxicology Report No. TDD-02-41285 reading: xxxx SPECIMEN: Urine of one SUSAN ESQUILLO Y ROMINES. 37 y/o, married, jobless, of no. 1159 Bo. Bayanihan, Maricaban, Pasay City.

xxxx F I N D I N G S: Volume of urine = 60 mL. pH of urine = 5.0 Appearance = yellow orange, turbid Examinations conducted on the above-mentioned specimen gave POSITIVE RESULTS for the presence of METHAMPHETAMINE HYDROCHLORIDE, and its metabolite AMPHETAMINE. x x x x x x x (emphasis and underscoring supplied) Based on its documentary evidence and the testimony of PO1 Alvin Cruzin (PO1 Cruzin),6 a member of the Pasay City Police Station Special Operations Group (SOG), the prosecution established its version as follows: On the basis of an informants tip, PO1 Cruzin, together with PO2 Angel Aguas (PO2 Aguas), proceeded at around 4:00 p.m. on December 10, 2002 to Bayanihan St., Malibay, Pasay City to conduct surveillance on the activities of an alleged notorious snatcher operating in the area known only as "Ryan." As PO1 Cruzin alighted from the private vehicle that brought him and PO2 Aguas to the target area, he glanced in the direction of petitioner who was standing three meters away and seen placing inside a yellow cigarette case what appeared to be a small heat-sealed transparent plastic sachet containing white substance. While PO1 Cruz was not sure what the plastic sachet contained, he became suspicious when petitioner started acting strangely as he began to approach her. He then introduced himself as a police officer to petitioner and inquired about the plastic sachet she was placing inside her cigarette case. Instead of replying, however, petitioner attempted to flee to her house nearby but was timely restrained by PO1 Cruzin who then requested her to take out the transparent plastic sachet from the cigarette case. After apprising petitioner of her constitutional rights, PO1 Cruzin confiscated the plastic sachet7 on which he marked her initials "SRE." With the seized item, petitioner was brought for investigation to a Pasay City Police Station where P/Insp. Aquilino E. Almanza, Chief of the Drug Enforcement Unit, prepared a memorandum8 dated December 10, 2002 addressed to the Chief Forensic Chemist of the NBI in Manila requesting for: 1) a laboratory examination of the substance contained in the plastic sachet to determine the presence of shabu, and 2) the conduct of a drug test on the person of petitioner. PO1 Cruzin and PO2 Aguas soon executed a Joint Affidavit of Apprehension9 recounting the details of their intended surveillance and the circumstances leading to petitioners arrest. Repudiating the charges, petitioner10 gave the following tale:

At around 1:00 to 2:00 p.m. of the date in question, while she was sick and resting at home, several policemen in civilian garb with guns tucked in their waists barged in and asked her whether she knew one named "Ryan" who they claimed was a notorious snatcher operating in the area, to which she replied in the negative. The police officers then forced her to go with them to the Pasay City Police Station-SOG office where she was detained. While she was under detention, the police officers were toying with a wallet which they claimed contained shabu and recovered from her. In fine, petitioner claimed that the evidence against her was "planted," stemming from an all too obvious attempt by the police officers to extort money from her and her family. Two other witnesses for the defense, petitioners daughter Josan Lee11 and family friend Ma. Stella Tolentino,12 corroborated petitioners account. They went on to relate that the police officers never informed them of the reason why they were taking custody of petitioner. By Decision13 of July 28, 2003, the trial court found petitioner guilty of illegal possession of Methylamphetamine Hydrochloride or shabu, disposing as follows: WHEREFORE, in light of the foregoing premises and considerations, this Court hereby renders judgment finding the accused Susan Esquillo y Romines GUILTY beyond reasonable doubt of the crime of Violation of par. 3 of Section 11, Article II of R. A. 9165, otherwise known as the Comprehensive Dangerous Drugs Act of 2002, and absent any modifying circumstance to either aggravate or mitigate the criminal liability of the same accused, and furthermore, applying the provisions of the Indeterminate Sentence Law, the same accused is hereby sentenced to suffer the penalty of imprisonment ranging from Eight (8) years and One (1) day, as minimum, to Fourteen (14) years, Eight (8) months and One (1) day, as maximum, and to pay a fine of P350,000.00, Philippine Currency, plus costs. The 0.1224 gram of Methylamphetamine Hydrochloride or "Shabu" involved in this case is declared forfeited in favor of the Government and ordered to be turned over to the Philippine Drug Enforcement Agency (PDEA) for proper and appropriate disposition in accordance with the provisions of the law.14 (underscoring supplied) Before the Court of Appeals, appellant questioned as illegal her arrest without warrant to thus render any evidence obtained on the occasion thereof inadmissible. In its challenged Decision affirming petitioners conviction, the appellate court, citing People v. Chua,15 held that the police officers had probable cause to search petitioner under the "stop-andfrisk" concept, a recognized exception to the general rule prohibiting warrantless searches.16 Brushing aside petitioners defense of frame-up, the appellate court noted that petitioner failed to adduce evidence that the arresting officers were impelled by any evil motive to falsely charge her, and that she was even found positive for substance abuse.1wphi1

In her present petition, petitioner assails the appellate courts application of the "stop-and-frisk" principle in light of PO1 Cruzins failure to justify his suspicion that a crime was being committed, he having merely noticed her placing something inside a cigarette case which could hardly be deemed suspicious. To petitioner, such legal principle could only be invoked if there were overt acts constituting unusual conduct that would arouse the suspicion.17 Respondent, through the Office of the Solicitor General, prays for the affirmance of the appealed decision but seeks a modification of the penalty to conform to the pertinent provisions of R.A. No. 9165. Appellants conviction stands. Petitioner did not question early on her warrantless arrest before her arraignment. Neither did she take steps to quash the Information on such ground. Verily, she raised the issue of warrantless arrest as well as the inadmissibility of evidence acquired on the occasion thereof for the first time only on appeal before the appellate court.18 By such omissions, she is deemed to have waived any objections on the legality of her arrest.19 Be that as it may, the circumstances under which petitioner was arrested indeed engender the belief that a search on her was warranted. Recall that the police officers were on a surveillance operation as part of their law enforcement efforts. When PO1 Cruzin saw petitioner placing a plastic sachet containing white crystalline substance into her cigarette case, it was in his plain view. Given his training as a law enforcement officer, it was instinctive on his part to be drawn to curiosity and to approach her. That petitioner reacted by attempting to flee after he introduced himself as a police officer and inquired about the contents of the plastic sachet all the more pricked his curiosity. That a search may be conducted by law enforcers only on the strength of a valid search warrant is settled. The same, however, admits of exceptions, viz: (1) consented searches; (2) as an incident to a lawful arrest; (3) searches of vessels and aircraft for violation of immigration, customs, and drug laws; (4) searches of moving vehicles; (5) searches of automobiles at borders or constructive borders; (6) where the prohibited articles are in "plain view;" (7) searches of buildings and premises to enforce fire, sanitary, and building regulations; and (8) "stop and frisk" operations.20 (emphasis underscoring supplied) In the instances where a warrant is not necessary to effect a valid search or seizure, the determination of what constitutes a reasonable or unreasonable search or seizure is purely a judicial question, taking into account, among other things, the uniqueness of the circumstances involved including the purpose of the search or seizure, the presence or absence of probable cause, the manner in which the search and seizure was made, the place or thing searched, and the character of the articles procured.21 Elucidating on what includes "stop-and-frisk" operation and how it is to be carried out, the Court in People v. Chua22 held:

. . . the act of a police officer to stop a citizen on the street, interrogate him, and pat him for weapon(s) or contraband. The police officer should properly introduce himself and make initial inquiries, approach and restrain a person who manifests unusual and suspicious conduct, in order to check the latters outer clothing for possibly concealed weapons. The apprehending police officer must have a genuine reason, in accordance with the police officers experience and the surrounding conditions, to warrant the belief that the person to be held has weapons (or contraband) concealed about him. It should therefore be emphasized that a search and seizure should precede the arrest for this principle to apply. This principle of "stop-and-frisk" search was invoked by the Court in Manalili v. Court of Appeals. In said case, the policemen chanced upon the accused who had reddish eyes, walking in a swaying manner, and who appeared to be high on drugs. Thus, we upheld the validity of the search as akin to a "stop-and-frisk." In People v. Solayao, we also found justifiable reason to "stop-and-frisk" the accused after considering the following circumstances: the drunken actuations of the accused and his companions, the fact that his companions fled when they saw the policemen, and the fact that the peace officers were precisely on an intelligence mission to verify reports that armed persons w[h]ere roaming the vicinity. (emphasis and underscoring supplied; citations omitted)1wphi1 What is, therefore, essential is that a genuine reason must exist, in light of the police officers experience and surrounding conditions, to warrant the belief that the person who manifests unusual suspicious conduct has weapons or contraband concealed about him. Such a "stop-andfrisk" practice serves a dual purpose: (1) the general interest of effective crime prevention and detection, which underlies the recognition that a police officer may, under appropriate circumstances and in an appropriate manner, approach a person for purposes of investigating possible criminal behavior even without probable cause; and (2) the more pressing interest of safety and self-preservation which permit the police officer to take steps to assure himself that the person with whom he deals is not armed with a deadly weapon that could unexpectedly and fatally be used against the police officer.23 From these standards, the Court finds that the questioned act of the police officers constituted a valid "stop-and-frisk" operation. The search/seizure of the suspected shabu initially noticed in petitioners possession - later voluntarily exhibited24 to the police operative - was undertaken after she was interrogated on what she placed inside a cigarette case, and after PO1 Cruzin introduced himself to petitioner as a police officer. And, at the time of her arrest, petitioner was exhibiting suspicious behavior and in fact attempted to flee after the police officer had identified himself. It bears recalling that petitioner admitted the genuineness and due execution of the Dangerous Drugs and Toxicology Reports, subject, however, to whatever available defenses she would raise. While such admissions do not necessarily control in determining the validity of a warrantless search or seizure, they nevertheless provide a reasonable gauge by which petitioners credibility as a witness can be measured, or her defense tested. It has not escaped the Courts attention that petitioner seeks exculpation by adopting two completely inconsistent or incompatible lines of defense. On one hand, she argues that the "stop-

and-frisk" search upon her person and personal effects was unjustified as it constituted a warrantless search in violation of the Constitution. In the same breadth, however, she denies culpability by holding fast to her version that she was at home resting on the date in question and had been forcibly dragged out of the house by the police operatives and brought to the police station, for no apparent reason than to try and extort money from her. That her two witnesses a daughter and a friend who were allegedly present at the time of her arrest did not do anything to report it despite their claim that they were not informed why she was being arrested, should dent the credibility of their testimony. Courts have tended to look with disfavor on claims of accused, such as those of petitioners, that they are victims of a frame-up. The defense of frame-up, like alibi, has been held as a shop-worn defense of the accused in drug-related cases, the allegation being easily concocted or contrived. For this claim to prosper, the defense must adduce clear and convincing evidence to overcome the presumption of regularity of official acts of government officials. This it failed to do. Absent any proof of motive to falsely accuse petitioner of such a grave offense, the presumption of regularity in the performance of official duty and the findings of the trial court with respect to the credibility of witnesses prevail over that of petitioner.25 A word on the penalty. While the appellate court affirmed the trial courts decision, it overlooked the error in the penalty imposed by the trial court. The trial court, applying the provisions of the Indeterminate Sentence Law, sentenced petitioner to "suffer the penalty of imprisonment ranging from Eight (8) years and One (1) day, as minimum, to Fourteen (14) years, Eight (8) months and One (1) day, as maximum." Article II, Section 11 of R.A. No. 9165 provides, however: Section 11. Possession of Dangerous Drugs. xxxx Otherwise, if the quantity involved is less than the foregoing quantities, the penalties shall be graduated as follows: xxxx (3) Imprisonment of twelve (12) years and one (1) day to twenty (20) years and a fine ranging from Three hundred thousand pesos (P300,000) to Four hundred thousand pesos (P400,000), if the quantities of dangerous drugs are less than five (5) grams of opium, morphine, heroin, cocaine or cocaine hydrochloride, marijuana resin or marijuana resin oil, metamphetamine hydrochloride or "shabu" or other dangerous drugs such as, but not limited to MDMA or "ecstacy," PMA, TMA, LSD, GHB and those similarly designed or newly introduced drugs and their derivatives, without having any therapeutic value or if the quantity

possesses is far behind therapeutic requirements; or less than three hundred (300) grams of marijuana. (emphasis and underscoring supplied) Section 1 of the Indeterminate Sentence Law provides that when the offense is punished by a law other than the Revised Penal Code, "the court shall sentence the accused to an indeterminate sentence, the maximum term of which shall not exceed the maximum fixed by law and the minimum shall not be less than the minimum term prescribed by the same." The prayer of the Office of the Solicitor General for a modification of the penalty is thus in order. The Court, therefore, imposes on petitioner the penalty of imprisonment of twelve (12) years and one (1) day, as minimum, to fourteen (14) years, as maximum. WHEREFORE, the assailed decision of the Court of Appeals is AFFIRMED, with the MODIFICATION that the penalty of imprisonment shall be twelve (12) years and one (1) day, as minimum, to fourteen (14) years, as maximum. In all other respects, the decision of the RTC in Criminal Case No. 02-2297 is AFFIRMED. SO ORDERED.

People vs. Posadas Facts:On October 16,1986 at about10:00o'clock in themorning Pat. Ursicio Ungab and Pat. Umbra Umpar, bothmembers of the Integrated National Police (INP) of theDavao Metrodiscom assigned with the Intelligence TaskForce, were conducting a surveillance along MagallanesStreet, Davao City. While they were within the premises of the Rizal Memorial Colleges they spotted petitioner carryinga "buri" bag and they noticed him to be acting suspiciously.They approached the petitioner and identifiedthemselves as members of the INP. Petitioner attempted toflee but his attempt to get away was thwarted by the twonotwithstanding his resistance.They then checked the "buri" bag of the petitioner where they found one (1) caliber .38Smith & Wessonrevolver with Serial No. 770196 two (2) rounds of liveammunition for a .38caliber gun, a smoke (tear gas) grenadea and two (2) live ammunitions for a .22 caliber gun.They brought the petitioner to the police station for further investigation. In the course of the same, the petitioner was asked to show the necessary license or authority topossess firearms and ammunitions found in his possessionbut he failed to do so.He was prosecuted for illegal possession of firearmsand ammunitions in the Regional Trial Court of Davao Citywherein after a plea of not guilty and trial on the merits adecision was rendered on October 8,1987 finding petitioner guilty of the offense. The CA affirmed the decision in toto.Thus, this appeal was filed by the appellant in theground that the search and seizure is unlawful and thearticles sought are inadmissible evidence against him.Issue:Was the warrantless search valid?Held:Yes. The warrantless search is valid.Under the statutory provision (RRC) warrantlessarrest t may be effected by a peace officer or private person,among others, when in his presence the person to bearrested has committed, is actually committing, or isattempting to commit an offense; or when an offense has infact just been committed, and he has personal knowledge of the facts indicating that the person arrested has committed it.The Solicitor General argues that when the twopolicemen approached the petitioner, he was actuallycommitting or had just committed the offense of illegalpossession of firearms and ammunitions in the presence of the police officers and consequently the search and seizureof the contraband was incidental to the lawful arrest inaccordance with

Section12, Rule126 of the1985 Rules onCriminal ProcedureHowever, in the case at bar, the record does notshow that when the officers arrested the petitioner whoattempted to flee, there is no probable cause nor a personalknowledge by the officers that the appellant is actuallycommitted or committing a crime, the apprehension was onlydue to the fact of suspicion by the officers that he is hidingsomething in the bag. Thus, it does justified a warrantlessarrest.However, this does not make the arrest invalid asthe case at bar falls with the exclusionary rule of stop andfrisk doctrine.Jurisprudence has already dictate that warrantlessarrest, search and seizure by checkpoints of military andpolice is justified by the stop and frisk method.Thus, as between a warrantless search and seizureconducted at military or police checkpoints and the searchthereat in the case at bar, there is no question that, indeed,the latter is more reasonable considering that unlike in theformer, it was effected on the basis of a probable cause.The probable cause is that when the petitioner acted suspiciously and attempted to flee with the buri bagthere was a probable cause that he was concealingsomething illegal in the bag and it was the right and duty of the police officers to inspect the same.It would be useless and too late for the officers if they should first procure a warrant before searching the bagof a suspicious person. Manalili vs. People Facts:Policemen from the Anti-Narcotics Unit of theCaloocan City Police Station were conducting a surveillancealong A. Mabini street, Caloocan City, in front of theCaloocan City Cemetery.The surveillance was being made because of information that drug addicts were roaming the area in frontof the Caloocan City Cemetery.Upon reaching the Caloocan City Cemetery, thepolicemen alighted from their vehicle. They then chancedupon a male person in front of the cemetery who appearedhigh on drugs. The male person was observed to havereddish eyes and to be walking in a swaying manner. Whenthis male person tried to avoid the policemen, the latter approached him and introduced themselves as policeofficers. The policemen then asked the male person what hewas holding in his hands. The male person tried to resist.Pat. Romeo Espiritu asked the male person if hecould see what said male person had in his hands. The latter showed the wallet and allowed Pat. Romeo Espiritu toexamine the same. Pat. Espiritu took the wallet and examined it. He found suspected crushed marijuana residueinside. He kept the wallet and its marijuana contents.The confiscated residue was tested positive fir marijuana.However, the petitioner rebutted the information innarrating a different facts in the case. The petitioner filed anappeal questioning the admissibility of the evidencepresented by the prosecution.Issue:Was the search qualified as a stop and friskmeasure therefore justified as a valid warrantless arrest andsearch?Held:Yes. The search is valid being conducted through astop and frisk method.Stop-and-frisk was defined as the vernacular designation of the right of a police officer to stop a citizen onthe street, interrogate him, and pat him for weapon. Thismethod allows police officer to approach a person, inappropriate circumstances and manner, for purposes of investigating possible criminal behavior even though there isinsufficient probable cause to make an actual arrest.In the case at hand, Patrolman Espiritu and hiscompanions observed during their surveillance that appellanthad red eyes and was wobbling like a drunk along theCaloocan City Cemetery, which according to policeinformation was a popular hangout of drug addicts.From his experience as a member of the Anti-Narcotics Unit of the Caloocan City Police, such suspiciousbehavior was characteristic of drug addicts who were "high."The policemen therefore had sufficient reason tostop petitioner to investigate if he was actually high on drugs.During such investigation, they found marijuana inpetitioner's possession. Issue on waiver of rights: The Sc also ruled for the Solicitor General'scontention that petitioner effectively waived the inadmissibilityof any evidence illegally obtained when he filed to raise thisissue or to object thereto during the trial. A valid waiver of a right, more particularly of theconstitutional right against unreasonable search, requires theconcurrence of the following requirements:( 1 ) The right to be waived existed;(2) The person waiving it had knowledge, actual or constructive, thereof; and(3) He or she had an actual intention to relinquishthe right.Otherwise, the Courts will indulge every

reasonablepresumption against waiver of fundamental safeguards andwill not deduce acquiescence from the failure to exercise thiselementary right.In the present case, however, petitioner is deemedto have waived such right for his failure to raise its violationbefore the trial court. In petitions under Rule 45, asdistinguished from an ordinary appeal of criminal caseswhere the whole case is opened for review, the appeal isgenerally limited to the errors assigned by petitioner. Issuesnot raised below cannot be pleaded for the first time onappeal. Malacat vs.CA Facts:The arresting officers conducted a foot patrol due tothe report that a grioup of Muslim extremists was going toexplode a grenade somewhere on the vicinity of PlazaMiranda.While on patrol, they chanced upon two groups of Muslim-looking men, with each group, comprised of three tofour men, posted at opposite sides of the corner of QuezonBoulevard near the Mercury Drug Store. These men wereacting suspiciously with "[t]heir eyes. . . moving very fast."Yu and his companions positioned themselves atstrategic points and observed both groups for about thirtyminutes. The police officers then approached one group of men, who then fled in different directions. As the policemen gave chase, Yu caught up withand apprehended petitioner. Upon searching petitioner, Yufound a fragmentation grenade tucked inside petitioner's"front waist line." Yu's companion, police officer RogelioMalibiran, apprehended Abdul Casan from whom a .38 caliber revolver was recovered. Petitioner and Casan werethen brought to Police Station No. 3 where Yu placed an "X"mark at the bottom of the grenade and thereafter gave it tohis commander.The lower court then charged the petitioners andwas convicted holding that the search is valid being a searchincidental to a lawful arrest.The petitioner filed an appeal in the ground that theCA erred in ruling that the search is valid since they areabout to or attempting to commit a crime due to their attemptto flee when the officers approached them.Issue:Was the search qualified as a search incidental tovalid arrest or stop frisk measure?Held:None. The lower and appellate court erred in thedecision in such a way the the case at bar is neither a lawfulsearch incidental to valid arrest nor a valid warrantlesssearch by stop frisk methodThe search is not qualified as a search incidental toa valid arrest.In the instant petition, the trial court validated thewarrantless search as a "stop and frisk" with "the seizure of the grenade from the accused [as an appropriate incident tohis arrest," hence necessitating a brief discussion on thenature of these exceptions to the warrant requirement. At the outset, we note that the trial court confusedthe concepts of a "stop-and-frisk" and of a search incidentalto a lawful arrest. These two types of warrantless searchesdiffer in terms of the requisite quantum of proof before they may be validly effected and in their allowable scope . In a search incidental to a lawful arrest , as theprecedent arrest determines the validity of the incidentalsearch, the legality of the arrest is questioned in a largemajority of these cases, e.g., whether an arrest was merelyused as a pretext for conducting a search. 36 In thisinstance, the law requires that there first be a lawful arrestbefore a search can be made the process cannot bereversed. At bottom, assuming a valid arrest, the arrestingofficer may search the person of the arrestee and the areawithin which the latter may reach for a weapon or for evidence to destroy, and seize any money or property foundwhich was used in the commission of the crime, or the fruit of the crime, or that which may be used as evidence, or whichmight furnish the arrestee with the means of escaping or committing violence.Here, there could have been no valid in flagrantedelicto or hot pursuit arrest preceding the search in light of the lack of personal knowledge on the part of Yu, thearresting officer, or an overt physical act, on the part of petitioner, indicating that a crime had just been committed,was being committed or was going to be committed.

Having thus shown the invalidity of thewarrantless arrest in this case, plainly, the searchconducted on petitioner could not have been oneincidental to a lawful arrest. On the other hand, stop and frisk is limitedprotective search of outer clothing for weapons or any other incriminating evidence against the arrestee, which does notrequire probable cause but the existence of a genuinereason. A "stop-and-frisk" serves a two-fold interest:( 1 ) The general interest of effective crimeprevention and detection, which underlies therecognition that a police officer may, under appropriate circumstances and in an appropriatemanner, approach a person for purposes of investigating possible criminal behavior evenwithout probable cause; and(2) The more pressing interest of safety andself-preservation which permit the police officer totake steps to assure himself that the person withwhom he deals is not armed with a deadly weaponthat could unexpectedly and fatally be used againstthe police officer.The court did not qualify the case at bar as a validwarrantless search through stop and frisk method. For thefollowing reasons: First , we harbor grave doubts as to Yu's claim thatpetitioner was a member of the group which attempted tobomb Plaza Miranda two days earlier.This claim is neither supported by any police reportor record nor corroborated by any other police officer whoallegedly chased that group. Aside from impairing Yu'scredibility as a witness, this likewise diminishes theprobability that a genuine reason existed so as to arrest andsearch petitioner.If only to further tarnish the credibility of Yu'stestimony, contrary to his claim that petitioner and hiscompanions had to be chased before being apprehended,the affidavit of arrest (Exh. "A") expressly declares otherwise,i.e., upon arrival of five (5) other police officers, petitioner andhis companions were "immediately collared." Second, there was nothing in petitioner's behavior or conduct which could have reasonably elicited even meresuspicion other than that his eyes were "moving very fast"an observation which leaves us incredulous since Yu and histeammates were nowhere near petitioner and it was already6:30 p.m., thus presumably dusk. Petitioner and hiscompanions were merely standing at the corner and were notcreating any commotion or trouble. Third , there was at all no ground, probable or otherwise, to believe that petitioner was armed with a deadlyweapon. None was visible to Yu, for as he admitted, thealleged grenade was "discovered" "inside the front waistline"of petitioner, and from all indications as to the distancebetween Yu and petitioner, any telltale bulge, assuming thatpetitioner was indeed hiding a grenade, could not have beenvisible to Yu. In fact, as noted by the trial court:When the policemen approached the accused andhis companions, they were not yet aware that a handgrenade was tucked inside his waistline. They did not seeany bulging object in [sic] his person.What is unequivocal then in this case are blatantviolations of petitioner's rights solemnly guaranteed inSections 2 and12(1) of Article III of the Constitution.Lastly, the search is illegal since the officers hasample time to procure a valid search and arrest warrant. In the issue of admissibility of conducted testimony andinvestigation by NBI: Serapio conducted the custodial investigation onpetitioner the day following his arrest. No lawyer was presentand Serapio could not have requested a lawyer to assistpetitioner as no PAO lawyer was then available. Thus, even if petitioner consented to the investigation and waived hisrights to remain silent and to counsel, the waiver was invalidas it was not in writing, neither was it executed in thepresence of counsel.Contrary to Sec12 Art III involving right to acompetent counsel and that any admission or admissionobtained in violation of this or Section17 shall beinadmissible in evidence against him.
GUANZON VS. DE VILLA [181 SCRA 623; G.R. 80508; 30 JAN 1990] Friday, February 06, 2009 Posted by Coffeeholic Writes Labels: Case Digests, Political Law

Facts:

The 41 petitioners alleged that the "saturation

drive" or "aerial target zoning" that were conducted in their place (Tondo Manila) were unconstitutional. They alleged that there is no specific target house to be search and that there is no search warrant or warrant of arrest served. Most of the policemen are in their civilian clothes and without nameplates or identification cards. The residents were rudely rouse from their sleep by banging on the walls and windows of their houses. The residents were at the point of high-powered guns and herded like cows. Men were ordered to strip down to their briefs for the police to examine their tattoo marks. The residents complained that they're homes were ransacked, tossing their belongings and destroying their valuables. Some of their money and valuables had disappeared after the operation. The residents also reported incidents of maulings, spot-beatings and maltreatment. Those who were detained also suffered mental and physical torture to extract confessions and tactical informations. The respondents said that such accusations were all lies. Respondents contends that the Constitution grants to government the power to seek and cripple subversive movements for the maintenance of peace in the state. The aerial target zoning were intended to flush out subversives and criminal elements coddled by the communities were the said drives were conducted. They said that they have intelligently and carefully planned months ahead for the actual operation and that local and foreign media joined the operation to witness and record

such

event.

Issue: Whether
consisted of

or Not the saturation drive committed violation of human rights.

Held: It

is not the police action per se which should be

prohibited rather it is the procedure used or the methods which "offend even hardened sensibilities" .Based on the facts stated by the parties, it appears to have been no impediment to securing search warrants or warrants of arrest before any houses were searched or individuals roused from sleep were arrested. There is no showing that the objectives sought to be attained by the "aerial zoning" could not be achieved even as th rights of the squatters and low income families are fully protected. However, the remedy should not be brought by a tazpaer suit where not one victim complaints and not one violator is properly charged. In the circumstances of this taxpayers' suit, there is no erring soldier or policeman whom the court can order prosecuted. In the absence of clear facts no permanent relief can be given. In the meantime where there is showing that some abuses were committed, the court temporary restraint the alleged violations which are shocking to the senses. Petition is remanded to the RTC of Manila.
UMIL VS. RAMOS [187 SCRA 311; G.R. NO. 81567; 3 OCT 1991] Wednesday, February 04, 2009 Posted by Coffeeholic Writes Labels: Case Digests, Political Law

Facts:

On 1 February 1988, military agents were

dispatched to the St. Agnes Hospital, Roosevelt Avenue, Quezon City, to verify a confidential information which was received by their office, about a "sparrow man" (NPA member) who had been admitted to the said hospital with a gunshot wound. That the wounded man in the said hospital was among the five (5) male "sparrows" who murdered two (2) Capcom mobile patrols the day before, or on 31 January 1988 at about 12:00 o'clock noon, before a road hump along Macanining St., Bagong Barrio, Caloocan City. The wounded man's name was listed by the hospital management as "Ronnie Javellon," twenty-two (22) years old of Block 10, Lot 4, South City Homes, Bian, Laguna however it was disclosed later that the true name of the wounded man was Rolando Dural. In view of this verification, Rolando Dural was transferred to the Regional Medical Servicesof the CAPCOM, for security reasons. While confined thereat, he was positively identified by the eyewitnesses as the one who murdered the 2 CAPCOM mobile patrols.

Issue: Whether

or Not Rolando was lawfully arrested.

Held: Rolando Dural was arrested for being a member of


the NPA, an outlawed subversive organization. Subversion being a continuing offense, the arrest without warrant is

justified as it can be said that he was committing as offense when arrested. The crimes rebellion, subversion, conspiracy or proposal to commit such crimes, and crimes or offenses committed in furtherance therefore in connection therewith constitute direct assaults against the state and are in the nature of continuing crimes.

G.R. No. 78606 September 26, 1988 GELACIO V. SAMULDE, in his official capacity as Municipal Judge, petitioner, vs. RAMON M. SALVANI, Jr., in his official capacity as Provincial fiscal of Antique, substituted by LEOPOLDO O. VILLAVERT, respondent.

GRIO-AQUINO, J.: This case involves a disagreement between an investigating judge and the provincial fiscal on whether it is mandatory for the former to issue a warrant for the arrest of the accused in view of his finding, after conducting a preliminary investigation, that there exists prima facie evidence that the accused committed the crime charged. Municipal Judge Gelacio Samulde of Patnogon, Antique, conducted a preliminary investigation of Pelayo Arangale upon a complaint for robbery filed on October 29, 1985 by Maria Magbanua, alleging that Arangale harvested palay from a portion of her land directly adjoining Arangale's land (Crim. Case No. 2046-B, entitled "People of the Philippines vs. Pelayo Arangale"). After making a preliminary investigation based on the affidavits of the complainant and her witnesses and counter-affidavits of the respondent and his witnesses, as provided in Section 3, Rule 112 of the 1985 Rules on Criminal Procedure, Judge Samulde transmitted the records of the case to Provincial Fiscal Ramon Salvani with his finding that "there is prima facie evidence of robbery as charge in the complaint" (Annex A of Answer). The fiscal returned the records to Judge Samulde on the ground that the transmittal of the records to his office was "premature" because Judge Samulde failed to include the warrant of arrest against the accused as provided in Section 5, Rule 112 of the 1985 Rules on Criminal Procedure.

Judge Samulde sent back the records to Fiscal Salvani. He pointed out that under Section 6, Rule 112, he may issue a warrant of arrest if he is satisfied "that a probable cause exists and that there is a necessity of placing the respondent under immediate custody in order not to frustrate the ends of justice, " implying thereby that, although he found that a probable cause existed, he did not believe that Arangale should be immediately placed under custody so as not to frustrate the ends of justice. Hence, he refused to issue a warrant of arrest. On October 9, 1986, a special civil action of mandamus was filed in the Regional Trial Court of Antique by Provincial Fiscal Salvani against Judge Samulde to compel the latter to issue a warrant for the arrest of Arangale. During the pendency of the case in the lower court, Fiscal Salvani was replaced by Fiscal Leopoldo Villavert as provincial fiscal of Antique. Believing that the procedural question involved is important, Fiscal Villavert manifested to the Court that there is need to continue the case begun by his predecessor. On February 12, 1987, the Regional Trial Judge Pedro Icamina dismissed the petition for mandamus on the ground that "the petitioner had not shown that he has a clear, legal right to the performance of the act to be required of respondent and that the latter had an imperative duty to perform (it)," citing this Court's decision in Felix Morada vs. Hon. Hermogenes Caluag, 5 SCRA 1128. Nevertheless, he ordered Judge Samulde to issue a warrant for the arrest of Arangale in Crim. Case No. 2046-B in view of his (Judge Samulde's) resolution dated May 22, 1986, and to transmit the warrant, if the arrest is by virtue of a warrant, to the Provincial Fiscal for appropriate action in accordance with the provisions of Section 5, Rule 112 of the 1985 Rules on Criminal Procedure. He further advised the Municipal Judge "that henceforth he adheres to the same rule in similar cases where he conducts a preliminary investigation with a finding of a prima facie or probable cause." (pp. 23-32, Records.) Unconvinced, Judge Samulde appealed to this Court. The issue posed by this case necessitates an examination of the history and development of the rule on preliminary investigation. Section 13 of General Orders No. 58 dated April 23, 1900 of the U.S. Military Governor in the Philippines was the original source of the rule on preliminary investigation. It provided
SEC. 13. When a complaint or information alleging the commission of a crime is laid before a magistrate, he must examine, on oath, the informant or prosecutor and the witnesses produced, and take their depositions in writing, causing them to be subscribed by the parties making them. If the magistrate be satisfied from the investigation that the crime complained of has been committed, and that there is reasonable ground to believe that the party charged has committed it, he must issue an order for his arrest. If the offense be bailable, and the defendant offers a sufficient security, he shall be admitted to bail; otherwise he shall be committed to prison. (General Orders & Circulars, issued by the Office of U.S. Military Governor in the Philippine Islands, 1900, p. 3, Emphasis supplied.)

It was amended by Act 194 of the Public Laws enacted by the Philippine Commission with Amendments indicated, Vol. I, p. 527, which authorized every justice of the peace to conduct such investigation and order the arrest of the accused if he believed the complaint to be well founded.
SEC. 1. Every justice of the peace in the Philippine Islands is hereby invested with authority to make preliminary investigation of any crime alleged to have been committed within his municipality, jurisdiction to hear and determine which is by law now vested in the judges of Courts of First Instance. It shall be the duty of every justice of the peace, when written complaint under oath has been made to him that a crime has been committed within his municipality and there is reason to believe that any person has committed the same, which complaint the justice believes to be well founded, or when he has knowledge of facts tending to show the commission of a crime within his municipality by any person, to issue an order for the arrest of the accused and have him brought before the justice of the peace for such preliminary examination. (Emphasis supplied.)

The rule was substantially unchanged under Rule 108 of the Rules of Court except that municipal judges and city fiscals were also authorized to conduct the preliminary investigation of offenses committed within their municipality or city cognizable by the Court of First Instance. Under Rule 112 of the 1964 Rules of Court, the provincial fiscal was added to the enumeration of persons authorized to conduct a preliminary investigation. In the 1985 Rules on Criminal Procedure, Section 2, Rule 112, the list grew even longer to include: (a) provincial or city fiscals and their assistants; (b) Judges of the Municipal Trial Courts and Municipal Circuit Trial Courts; (c) National and Regional state prosecutors; and (d) other officers authorized by law. * Both the 1940 and 1964 Rules of Court provided for two (2) stages of the preliminary investigation, to wit: (1) the "previous inquiry or examination" of the complainant and his witnesses to determine whether a warrant of arrest should issue against the defendant, and (2) the preliminary investigation proper of the defendant himself to determine if he should be held for trial. Thus, the preliminary investigation was defined as:
... a previous inquiry or examination made before the arrest of the defendant by the judge . . . for the purpose of determining whether there is a reasonable ground to believe that an offense has been committed and the defendant is probably guilty thereof, so as to issue a warrant of arrest and to hold him for trial. (Sec. 1, Rule 108, 1940 Rules of Court. Emphasis supplied.)

In Section 1, Rule 112 of the 1964 Rules of Court, the distinction between a preliminary examination and preliminary investigation was more clearly defined by using the term "preliminary examination" in Section I of the Rule to differentiate the first stage of the preliminary investigation (where only the testimonies of the complainant and his witnesses were taken), from the second stage where, after the arrest of the defendant, he was informed of the complaint against him and given a chance to testify and present his evidence (Sec. 10, Rule 112, 1964 Rules of Court). The purpose of the preliminary examination was still to determine "whether there is a reasonable ground to believe that

an offense has been committed and the accused is probably guilty thereof, so that a warrant of arrest may be issued and the accused held for trial. " (Sec. 1, Rule 112, 1964 Revised Rules of Court.) In both the 1940 and 1964 Rules of Court, it was mandatory upon the investigating judge to issue a warrant for the arrest of the accused, if he was satisfied that the offense charged was committed and that the accused probably committed it. Accordingly, Section 7, Rule 108 of the 1940 Rules of Court provided:
SEC. 7. Warrant of arrest, when issued. If the judge be satisfied from the preliminary investigation conducted by him that the offense complained of has been committed and that there is reasonable ground to believe that the defendant has committed it, he must issue a warrant or order for his arrest. (Emphasis ours.)

Section 6, Rule 112 of the 1964 Rules of Court similary provided:


SEC. 6. Warrant of arrest, when issued. If the judge be satisfied from the preliminary examination conducted by him or by the investigating officer that the offense complained of has been committed and that there is reasonable ground to believe that the accused has committed it, he must issue a warrant or order for his arrest. (Emphasis supplied.)

Because the arrest of the accused was mandatory, the records to be transmitted by the investigating judge to the clerk of the Court of First Instance upon the conclusion of the preliminary investigation, included the warrant of arrest (Sec. 13, Rule 108,1940 Rules of Court; Sec. 12, Rule 112,1964 Revised Rules of Court). However, the rule on preliminary investigation underwent some modifications in the 1985 Rules on Criminal Procedure, which is the applicable rule in this case. Under Section 1 of the present rule, the definition of the purpose of a preliminary investigation, does not contemplate the issuance of a warrant of arrest by the investigating judge or officer:
SECTION 1. Definition. Preliminary investigation is an inquiry or proceeding for the purpose of determining whether there is sufficient ground to engender a well founded belief that a crime cognizable by the Regional Trial Court has been committed and that the respondent is probably guilty thereof, and should be held for trial.

The mandatory provision that the investigating judge "must issue a warrant of arrest" if he finds probable cause that the respondent committed the crime charged, found in all previous rules of criminal procedure, from General Orders No. 58 down to Rule 112 of the 1964 Revised Rules of Court, is absent in Section 1 of the 1985 Rules on Criminal Procedure. Another significant change is that under the 1985 Rules on Criminal Procedure there is only one (1) way of conducting a preliminary investigation, and that is by affidavits and counter-affidavits submitted by the parties to the investigating judge under Section 3, Rule 112. On the basis of the affidavits, the investigating judge shall "determine whether or not there is sufficient ground to hold the respondent for trial' (subpar. f ). Gone is the

requirement in the 1940 and 1964 Rules of Court that "he must issue a warrant or order" for the arrest of the defendant. To determine whether a warrant of arrest should issue against the accused, the investigating judge must examine the complainant and his witnesses "in writing and under oath ... in the form of searching questions and answers." When he is "satisfied that a probable cause exists, and that there is a necessity of placing the respondent under immediate custody in order not to frustrate the ends of justice," he may issue the warrant as provided in Section 6, par. b, of the 1985 Rules on Criminal Procedure.
SEC. 6 When warrant of arrest may issue. (a) By the Regional Court..... (b) By the Municipal Trial Court. -If the municipal trial judge conducting the preliminary investigation is satisfied after an examination in writing and under oath of the complainant and his witnesses in the form of searching questions and answers, that a probable cause exists and that there is a necessity of placing the respondent under immediate custody in order not to frustrate the ends of justice, he shall issue a warrant of arrest.

As correctly argued by the petitioner Judge Samulde, three (3) conditions must concur for the issuance of the warrant of arrest. The investigating judge must:
(a) have examined in writing and under oath the complainant and his witnesses by searching questions and answers; (b) be satisfied that a probable cause exists; and (c) that there is a need to place the respondent under immediatecustody in order not to frustrate the ends of justice.

It is an entirely new rule, and it is plain to see that it is not obligatory, but merely discretionary, upon the investigating judge to issue a warrant for the arrest of the accused, even after having personally examined the complainant and his witnesses in the form of searching questions and answers, for the determination of whether a probable cause exists and whether it is necessary to arrest the accused in order not to frustrate the ends of justice, is left to his sound judgment or discretion. In this particular case, since the robbery charge was the offshoot of a boundary dispute between two property owners, the investigating judge did not believe there was any danger of the accused absconding before the filing of the information against him by the fiscal, hence, he found no need to place him under immediate custody. The provincial fiscal anchored his action for mandamus on Section 5, Rule 112 of the 1985 Rules on Canal Procedure which provides that upon the termination of the preliminary investigation, the investigating judge should transmit to the provincial fiscal (instead of the clerk of Court of the CFI as provided in the 1940 and 1964 Rules of Court) the warrant of arrest and other records of the preliminary investigation. From that

he deduced that the investigating judge must issue a warrant for the arrest of the accused upon the conclusion of the preliminary investigation. That inference is not correct. The provision of Section 5, Rule 112 simply means that the warrant of arrest, if one was issued, shall be transmitted to the fiscal with the records of the preliminary investigation. If the investigating judge, in the exercise of his sound discretion, decides not to issue a warrant of arrest, then none need be transmitted to the fiscal, and he may not be compelled by mandamus to issue it (Vda. de Crisologo vs. Court of Appeals, 137 SCRA 231; Pio vs. Marcos, 56 SCRA 725; PAL Employees Assn. vs. PAL, Inc., III SCRA 215). The fiscal's speedy and adequate remedy, if he believes that the accused should be immediately placed under custody so as not to frustrate the ends of justice, is not to file a mandamus action (which may take two years or more to finally resolve, as happened in this case), but as sensibly indicated by the petitioner, to immediately file the information so that the Regional Trial Court may issue a warrant for the arrest of the accused (Sec. 6, par. a, Rule 112,1985 Rules on Criminal Procedure). WHEREFORE, the appealed decision in Civil Case No. 2145 is set aside. No costs. SO ORDERED. Narvasa, Cruz, Gancayco and Medialdea JJ., concur.

People vs Court of Appeals (291 SCRA 400) FACTS A petition for certiorari has been filed to invalidate the order of Judge Casanova which quashed search warrant issued by Judge Bacalla and declared inadmissible for any purpose the items seized under the warrant. >An application for a search warrant was made by S/Insp Brillantes against Mr. Azfar Hussain who had allegedly in his possession firearms and explosives at Abigail Variety Store, Apt 1207 Area F. Bagon Buhay Avenue, Sarang Palay, San Jose Del Monte, Bulacan. The following day Search Warrant No. 1068 was issued but was served not at Abigail Variety Store but at Apt. No. 1, immediately adjacent to Abigail Variety Store resulting in the arrest of 4 Pakistani nationals and the seizure of a number of different explosives and firearms. ISSUE: WON a search warrant was validly issued as regard the apartment in which private respondents were then actually residing, or more explicitly, WON that particular apartment had been specifically described in the warrant. HELD: The ambiguity lies outside the instrument, arising from the absence of a meeting of minds as to the place to be searched between the applicants for the warrant and the Judge issuing the same; and what was done was to substitute for the place that the Judge had written down in the warrant, the premises that the executing officers had in their mind. This should not have been done. It is neither fair nor licit to

allow police officers to search a place different from that stated in the warrant on the claim that the place actually searched although not that specified in the warrant is exactly what they had in view when they applied for the warrant and had demarcated in their supporting evidence. What is material in determining the validity of a search is the place stated in the warrant itself, not what the applicants had in their thoughts, or had represented in the proofs they submitted to the court issuing the warrant. The place to be searched, as set out in the warrant, cannot be amplified or modified by the officers' own personal knowledge of the premises, or the evidence they adduced in support of their application for the warrant. Such a change is proscribed by the Constitution which requires inter alia the search warrant to particularly describe the place to be searched as well as the persons or things to be seized. It would concede to police officers the power of choosing the place to be searched, even if it not be that delineated in the warrant. It would open wide the door to abuse of the search process, and grant to officers executing a search warrant that discretion which the Constitution has precisely removed from them. The particularization of the description of the place to be searched may properly be done only by the Judge, and only in the warrant itself; it cannot be left to the discretion of the police officers conducting the search.

Prudente vs. Dayrit, GR No.82870


(Constitutional Law Search Warrant, Probable Cause) Facts: Based on an information from a verified source that petitioner violated the Dangerous Drug Act of 1972, police officers applied for a search warrant which was subsequently issued by respondent judge. Petitioner assailed the validity of the search warrant on the ground that there was no probable cause for the issuance of search warrant. Issue: Whether or not a warrant issued based on verified information to the police is valid. Held: No. Probable cause must be shown to be within the personal knowledge of the complainant or the witnesses and not simply on hearsay. G.R. No. 82870 December 14, 1989 DR. NEMESIO E. PRUDENTE, petitioner, vs. THE HON. EXECUTIVE JUDGE ABELARDO M. DAYRIT, RTC Manila, Branch 33 and PEOPLE OF THE PHILIPPINES, respondents. Francisco SB Acejas III, Oscar S. Atencio, Rodolfo M. Capocyan, Ernesto P. Fernandez, Romulo B. Macalintal, Rodrigo H. Melchor, Rudegelio D. Tacorda Virgilio L. Valle and Luciano D. Valencia for petitioner.

PADILLA, J.: This is a petition for certiorari to annul and set aside the order of respondent Judge dated 9 March 1988 which denied the petitioner's motion to quash Search Warrant No. 87-14, as well as his order dated 20 April 1988 denying petitioner's motion for reconsideration of the earlier order. It appears that on 31 October 1987, P/Major Alladin Dimagmaliw, Chief of the Intelligence Special Action Division (ISAD) of the Western Police District (WPD) filed with the Regional Trial Court (RTC) of Manila, Branch 33, presided over by respondent Judge Abelardo Dayrit, now Associate Justice of the Court of Appeals. an application 1 for the issuance of a search warrant, docketed therein as SEARCH WARRANT NO. 8714, for VIOLATION OF PD NO. 1866 (Illegal Possession of Firearms, etc.) entitled "People of the Philippines, Plaintiff, versus Nemesis E. Prudente, Defendant." In his application for search warrant, P/Major Alladin Dimagmaliw alleged, among others, as follows:
1. That he has been informed and has good and sufficient reasons to believe that NEMESIO PRUDENTE who may be found at the Polytechnic University of the Philippines, Anonas St. Sta. Mesa, Sampaloc, Manila, has in his control or possession firearms, explosives handgrenades and ammunition which are illegally possessed or intended to be used as the means of committing an offense which the said NEMESIO PRUDENTE is keeping and concealing at the following premises of the Polytechnic University of the Philippines, to wit: a. Offices of the Department of Military Science and Tactics at the ground floor and other rooms at the ground floor; b. Office of the President, Dr. Nemesio Prudente at PUP, Second Floor and other rooms at the second floor; 2. That the undersigned has verified the report and found it to be a fact, and therefore, believes that a Search Warrant should be issued to enable the undersigned or any agent of the law to take possession and bring to this Honorable Court the following described properties: a. M 16 Armalites with ammunitions; b. .38 and .45 Caliber handguns and pistols; c. explosives and handgrenades; and, d. assorted weapons with ammunitions.

In support of the application for issuance of search warrant, P/Lt. Florenio C. Angeles, OIC of the Intelligence Section of (ISAD) executed a "Deposition of Witness" dated 31 October 1987, subscribed and sworn to before respondent Judge. In his deposition, P/Lt. Florenio Angeles declared, inter alia, as follows:
Q: Do you know P/Major Alladin Dimagmaliw, the applicant for a Search Warrant?

A: Yes, sir, he is the Chief, Intelligence and Special Action Division, Western Police District. Q: Do you know the premises of Polytechnic University of the Philippines at Anonas St., Sta. Mesa, Sampaloc, Manila A: Yes, sir, the said place has been the subject of our surveillance and observation during the past few days. Q: Do you have personal knowledge that in the said premises is kept the following properties subject of the offense of violation of PD No. 1866 or intended to be used as a means of committing an offense:

a. M 16 Armalites with ammunitions; b. .38 and 45 Caliber handguns and pistols; c. explosives and handgrenades; and d. Assorted weapons with ammunitions?
A: Yes sir. Q: Do you know who is or who are the person or persons who has or have control of the abovedescribed premises? A: Yes sir, it is Dr. Nemesio Prudente, President of the Polytechnic University of the Philippines. Q: How do you know that said property is subject of the offense of violation of Pres. Decree No. 1866 or intended to be used as the means of committing an offense? A: Sir, as a result of our continuous surveillance conducted for several days, we gathered information from verified sources that the holder of said firearms and explosives as well as ammunitions aren't licensed to possess said firearms and ammunition. Further, the premises is a school and the holders of these firearms are not students who were not supposed to possess firearms, explosives and ammunition.

On the same day, 31 October 1987, respondent Judge issued Search Warrant No. 8714, 3 the pertinent portions of which read as follows:
It appearing to the satisfaction of the undersigned, after examining under oath applicant ALLADIN M. DIMAGMALIW and his witness FLORENIO C. ANGELES that there are good and sufficient reasons to believe (probable cause) that NEMESIO PRUDENTE has in his control in the premises of Polytechnic University of the Philippines, Anonas St., Sta. Mesa, Sampaloc, Manila, properties which are subject of the above offense or intended to be used as the means of committing the said offense. You are hereby commanded to make an immediate search at any time in the day or night of the premises of Polytechnic University of the Philippines, more particularly (a) offices of the Department of Military Science and Tactics at the ground floor and other rooms at the ground floor; (b) office of the President, Dr. Nemesio Prudente at PUP, Second Floor and other rooms at the second floor, and forthwith seize and take possession of the following personal properties, to wit: a. M 16 Armalites with ammunition; b. .38 and .45 Caliber handguns and pistols; c. explosives and hand grenades; and

d. assorted weapons with ammunitions. and bring the above described properties to the undersigned to be dealt with as the law directs.

On 1 November 1987, a Sunday and All Saints Day, the search warrant was enforced by some 200 WPD operatives led by P/Col. Edgar Dula Torre, Deputy Superintendent, WPD, and P/Major Romeo Maganto, Precinct 8 Commander. In his affidavit, 4 dated 2 November 1987, Ricardo Abando y Yusay, a member of the searching team, alleged that he found in the drawer of a cabinet inside the wash room of Dr. Prudente's office a bulging brown envelope with three (3) live fragmentation hand grenades separately wrapped with old newspapers, classified by P/Sgt. J.L. Cruz as follows (a) one (1) pc.M33 Fragmentation hand grenade (live); (b) one (11) pc.M26 Fragmentation hand grenade (live); and (c) one (1) pc.PRB423 Fragmentation hand grenade (live). On 6 November 1987, petitioner moved to quash the search warrant. He claimed that (1) the complainant's lone witness, Lt. Florenio C. Angeles, had no personal knowledge of the facts which formed the basis for the issuance of the search warrant; (2) the examination of the said witness was not in the form of searching questions and answers; (3) the search warrant was a general warrant, for the reason that it did not particularly describe the place to be searched and that it failed to charge one specific offense; and (4) the search warrant was issued in violation of Circular No. 19 of the Supreme Court in that the complainant failed to allege under oath that the issuance of the search warrant on a Saturday was urgent. 5 The applicant, P/Major Alladin Dimagmaliw thru the Chief, Inspectorate and Legal Affairs Division, WPD, opposed the motion. 6 After petitioner had filed his reply 7 to the opposition, he filed a supplemental motion to quash. 8 Thereafter, on 9 March 1988, respondent Judge issued an order, 9denying the petitioner's motion and supplemental motion to quash. Petitioner's motion for reconsideration 10 was likewise denied in the order 11 dated 20 April 1988. Hence, the present recourse, petitioner alleging that respondent Judge has decided a question of substance in a manner not in accord with law or applicable decisions of the Supreme Court, or that the respondent Judge gravely abused his discretion tantamount to excess of jurisdiction, in issuing the disputed orders. For a valid search warrant to issue, there must be probable cause, which is to be determined personally by the judge, after examination under oath or affirmation of the complainant and the witnesses he may produce, and particularly describing the place to be searched and the persons or things to be seized. 12 The probable cause must be in connection with one specific offense 13 and the judge must, before issuing the warrant, personally examine in the form of searching questions and answers, in writing and under oath, the complainant and any witness he may produce, on facts personally

known to them and attach to the record their sworn statements together with any affidavits submitted. 14 The "probable cause" for a valid search warrant, has been defined "as such facts and circumstances which would lead a reasonably discreet arid prudent man to believe that an offense has been committed, and that objects sought in connection with the offense are in the place sought to be searched." 15 This probable cause must be shown to be within the personal knowledge of the complainant or the witnesses he may produce and not based on mere hearsay. 16 Petitioner assails the validity of Search Warrant No. 87-14 on the ground that it was issued on the basis of facts and circumstances which were not within the personal knowledge of the applicant and his witness but based on hearsay evidence. In his application for search warrant, P/Major Alladin Dimagmaliw stated that "he has been informed" that Nemesio Prudente "has in his control and possession" the firearms and explosives described therein, and that he "has verified the report and found it to be a fact." On the other hand, in his supporting deposition, P/Lt. Florenio C. Angeles declared that, as a result of their continuous surveillance for several days, they "gathered informations from verified sources" that the holders of the said fire arms and explosives are not licensed to possess them. In other words, the applicant and his witness had no personal knowledge of the facts and circumstances which became the basis for issuing the questioned search warrant, but acquired knowledge thereof only through information from other sources or persons. While it is true that in his application for search warrant, applicant P/Major Dimagmaliw stated that he verified the information he had earlier received that petitioner had in his possession and custody the t there is nothing in the record to show or indicate how and when said applicant verified the earlier information acquired by him as to justify his conclusion that he found such information to be a fact. He might have clarified this point if there had been searching questions and answers, but there were none. In fact, the records yield no questions and answers, whether searching or not, vis-a-vis the said applicant. What the records show is the deposition of witness, P/Lt. Angeles, as the only support to P/Major Dimagmaliw's application, and the said deposition is based on hearsay. For, it avers that they (presumably, the police authorities) had conducted continuous surveillance for several days of the suspected premises and, as a result thereof, they "gathered information from verified sources" that the holders of the subject firearms and explosives are not licensed to possess them. In Alvarez vs. Court of First Instance, 17 this Court laid the following test in determining whether the allegations in an application for search warrant or in a supporting deposition, are based on personal knowledge or not
The true test of sufficiency of a deposition or affidavit to warrant issuance of a search warrant is whether it has been drawn in a manner that perjury could be charged thereon and the affiant be held liable for damage caused. The oath required must refer to the

truth of the facts within the personal knowledge of the applicant for search warrant, and/or his witnesses, not of the facts merely reported by a person whom one considers to be reliable.

Tested by the above standard, the allegations of the witness, P/Lt. Angeles, in his deposition, do not come up to the level of facts of his personal knowledge so much so that he cannot be held liable for perjury for such allegations in causing the issuance of the questioned search warrant. In the same Alvarez case, 18 the applicant stated that his purpose for applying for a search warrant was that: "It had been reported to me by a person whom I consider to be reliable that there are being kept in said premises books, documents, receipts, lists, chits and other papers used by him in connection with his activities as a money lender, challenging usurious rate of interests, in violation of law." The Court held that this was insufficient for the purpose of issuing a search warrant. In People vs. Sy Juco, 19 where the affidavit contained an allegation that there had been a report to the affiant by a person whom lie considered reliable that in said premises were "fraudulent books, correspondence and records," this was likewise held as not sufficient for the purpose of issuing a search warrant. Evidently, the allegations contained in the application of P/ Major Alladin Dimagmaliw and the declaration of P/Lt. Florenio C. Angeles in his deposition were insufficient basis for the issuance of a valid search warrant. As held in the Alvarez case:
The oath required must refer to the truth of the facts within the personal knowledge of the petitioner or his witnesses, because the purpose thereof is to convince the committing magistrate, not the individual making the affidavit and seeking the issuance of the warrant, of the existence of probable cause.

Besides, respondent Judge did not take the deposition of the applicant as required by the Rules of Court. As held in Roan v. Gonzales, 20 "(m)ere affidavits of the complainant and his witnesses are thus not sufficient. The examining Judge has to take depositions in writing of the complainant and the witnesses he may produce and attach them to the record." Moreover, a perusal of the deposition of P/Lt. Florenio Angeles shows that it was too brief and short. Respondent Judge did not examine him "in the form of searching questions and answers." On the contrary, the questions asked were leading as they called for a simple "yes" or "no" answer. As held in Quintero vs. NBI," 21 the questions propounded by respondent Executive Judge to the applicant's witness are not sufficiently searching to establish probable cause. Asking of leading questions to the deponent in an application for search warrant, and conducting of examination in a general manner, would not satisfy the requirements for issuance of a valid search warrant." Manifestly, in the case at bar, the evidence failed to show the existence of probable cause to justify the issuance of the search warrant. The Court also notes post facto that the search in question yielded, no armalites, handguns, pistols, assorted weapons or

ammunitions as stated in the application for search warrant, the supporting deposition, and the search warrant the supporting hand grenades were itself Only three (3) live fragmentation found in the searched premises of the PUP, according to the affidavit of an alleged member of the searching party. The Court avails of this decision to reiterate the strict requirements for determination of "probable cause" in the valid issuance of a search warrant, as enunciated in earlier cases. True, these requirements are stringent but the purpose is to assure that the constitutional right of the individual against unreasonable search and seizure shall remain both meaningful and effective. Petitioner also assails the validity of the search warrant on the ground that it failed to particularly describe the place to be searched, contending that there were several rooms at the ground floor and the second floor of the PUP. The rule is, that a description of a place to be searched is sufficient if the officer with the warrant can, with reasonable effort, ascertain and Identify the place intended . 22 In the case at bar, the application for search warrant and the search warrant itself described the place to be searched as the premises of the Polytechnic University of the Philippines, located at Anonas St., Sta. Mesa, Sampaloc, Manila more particularly, the offices of the Department of Military Science and Tactics at the ground floor, and the Office of the President, Dr. Nemesio Prudente, at PUP, Second Floor and other rooms at the second floor. The designation of the places to be searched sufficiently complied with the constitutional injunction that a search warrant must particularly describe the place to be searched, even if there were several rooms at the ground floor and second floor of the PUP. Petitioner next attacks the validity of the questioned warrant, on the ground that it was issued in violation of the rule that a search warrant can be issued only in connection with one specific offense. The search warrant issued by respondent judge, according to petitioner, was issued without any reference to any particular provision of PD No. 1866 that was violated when allegedly P.D. No. 1866 punishes several offenses. In Stonehill vs. Diokno, 23 Where the warrants involved were issued upon applications stating that the natural and juridical persons therein named had committed a "violation of Central Bank Laws, Tariff and Customs Laws, Internal Revenue Code and Revised Penal Code," the Court held that no specific offense had been alleged in the applications for a search warrant, and that it would be a legal hearsay of the highest order to convict anybody of a "Violation of Central Bank Laws, Tariff and Customs Laws, Internal Revenue Code and Revised Penal Code" without reference to any determinate provision of said laws and codes. In the present case, however, the application for search warrant was captioned: "For Violation of PD No. 1866 (Illegal Possession of Firearms, etc.) While the said decree punishes several offenses, the alleged violation in this case was, qualified by the phrase "illegal possession of firearms, etc." As explained by respondent Judge, the term "etc."

referred to ammunitions and explosives. In other words, the search warrant was issued for the specific offense of illegal possession of firearms and explosives. Hence, the failure of the search warrant to mention the particular provision of PD No. 1-866 that was violated is not of such a gravity as to call for its invalidation on this score. Besides, while illegal possession of firearms is penalized under Section 1 of PD No. 1866 and illegal possession of explosives is penalized under Section 3 thereof, it cannot be overlooked that said decree is a codification of the various laws on illegal possession of firearms, ammunitions and explosives; such illegal possession of items destructive of life and property are related offenses or belong to the same species, as to be subsumed within the category of illegal possession of firearms, etc. under P.D. No. 1866. As observed by respondent Judge: 24
The grammatical syntax of the phraseology comparative with the title of PD 1866 can only mean that illegal possession of firearms, ammunitions and explosives, have been codified under Section 1 of said Presidential Decree so much so that the second and third are forthrightly species of illegal possession of firearms under Section (1) thereof It has long been a practice in the investigative and prosecution arm of the government, to designate the crime of illegal possession of firearms, ammunitions and explosives as 'illegal possession of firearms, etc.' The Constitution as well as the Rules of Criminal Procedure does not recognize the issuance of one search warrant for illegal possession of firearms, one warrant for illegal possession of ammunitions, and another for illegal possession of explosives. Neither is the filing of three different informations for each of the above offenses sanctioned by the Rules of Court. The usual practice adopted by the courts is to file a single information for illegal possession of firearms and ammunitions. This practice is considered to be in accordance with Section 13, Rule 110 of the 1985 Rules on Criminal Procedure which provides that: 'A complaint or information must charge but one offense, except only in those cases in which existing laws prescribe a single punishment for various offenses. Describably, the servers did not search for articles other than firearms, ammunitions and explosives. The issuance of Search Warrant No. 87-14 is deemed profoundly consistent with said rule and is therefore valid and enforceable. (Emphasis supplied)

Finally, in connection with the petitioner's contention that the failure of the applicant to state, under oath, the urgent need for the issuance of the search warrant, his application having been filed on a Saturday, rendered the questioned warrant invalid for being violative of this Court's Circular No. 19, dated 14 August 1987, which reads:
3. Applications filed after office hours, during Saturdays, Sundays and holidays shall likewise be taken cognizance of and acted upon by any judge of the court having jurisdiction of the place to be searched, but in such cases the applicant shall certify and state the facts under oath, to the satisfaction of the judge, that the issuance is urgent.

it would suffice to state that the above section of the circular merely provides for a guideline, departure from which would not necessarily affect the validity of an otherwise valid search warrant. WHEREFORE, all the foregoing considered, the petition is GRANTED. The questioned orders dated 9 March 1988 and 20 April 1988 as well as Search Warrant No. 87-14 are hereby ANNULLED and SET ASIDE.

The three (3) live fragmentation hand grenades which, according to Ricardo Y. Abando, a member of the searching team, were seized in the washroom of petitioner's office at the PUP, are ordered delivered to the Chief, Philippine Constabulary for proper disposition. SO ORDERED.

G.R. No. 121234 August 23, 1995 HUBERT J. P. WEBB, petitioner, vs. HONORABLE RAUL E. DE LEON, the Presiding Judge of the Regional Trial Court of Paraaque, Branch 258, HONORABLE ZOSIMO V. ESCANO, the Presiding Judge of the Regional Trial Court of Paraaque, Branch 259, PEOPLE OF THE PHILIPPINES, ZENON L. DE GUIA, JOVENCITO ZUO, LEONARDO GUIYAB, JR., ROBERTO LAO, PABLO FORMARAN, and NATIONAL BUREAU OF INVESTIGATION, and HONORABLE AMELITA G. TOLENTINO, the Presiding Judge of the Regional Trial Court of Paraaque, Branch 274, respondents, LAURO VIZCONDE, intervenor. G.R. No. 121245 August 23, 1995 MICHAEL A. GATCHALIAN, petitioner, vs. HONORABLE RAUL E. DE LEON, the Presiding Judge of the Regional Trial Court of Paraaque, Branch 258, HONORABLE ZOSIMO V. ESCANO, the Presiding Judge of the Regional Trial Court of Paraaque, Branch 259, PEOPLE OF THE PHILIPPINES, ZENON L. DE GUIYAB, JR., ROBERTO LAO, PABLO FORMARAN, and NATIONAL BUREAU OF INVESTIGATION, and HONORABLE AMELITA G. TOLENTINO, the Presiding Judge of the Regional Trial Court of Paraaque, Branch 274, respondents. G.R. No. 121297 August 23, 1995 ANTONIO L. LEJANO, petitioner, vs. HONORABLE RAUL E. DE LEON, the Presiding Judge of the Regional Trial Court of Paraaque, Branch 258, HONORABLE ZOSIMO V. ESCANO, the Presiding Judge of the Regional Trial Court of Paraaque, Branch 259, PEOPLE OF THE PHILIPPINES, ZENON L. DE GUIA, JOVENCITO ZUO, LEONARDO GUIYAB, JR., ROBERTO LAO, PABLO FORMARAN, and NATIONAL BUREAU OF INVESTIGATION, and HONORABLE AMELITA G. TOLENTINO, the Presiding Judge of the Regional Trial Court of Paraaque, Branch 274, respondents.

PUNO, J.: Before the Court are petitions for the issuance of the extraordinary writs of certiorari, prohibition and mandamus with application for temporary restraining order and preliminary injunction to: (1) annul and set aside the Warrants of Arrest issued against petitioners by respondent Judges Raul E. de Leon and Amelita Tolentino in Criminal Case No. 95-404; (2) enjoin the respondents from conducting any proceeding in the aforementioned criminal case; and (3) dismiss said criminal case or include Jessica Alfaro as one of the accused therein. 1 From the records of the case, it appears that on June 19, 1994, the National Bureau of Investigation (NBI) filed with the Department of Justice a letter-complaint charging petitioners Hubert Webb, Michael Gatchalian, Antonio J. Lejano and six (6) other persons, 2 with the crime of Rape with Homicide. Forthwith, the Department of Justice formed a panel of prosecutors headed by Assistant Chief State Prosecutor Jovencio R. Zuo to conduct the preliminary investigation 3 of those charged with the rape and killing on June 30, 1991 of Carmela N. Vizconde; 4 her mother Estrellita Nicolas-Vizconde, 5 and her sister Anne Marie Jennifer 6 in their home at Number 80 W. Vinzons, St., BF Homes, Paraaque, Metro Manila. During the preliminary investigation, the NBI presented the following: (1) the sworn statement dated May 22, 1995 of their principal witness, Maria Jessica M. Alfaro who allegedly saw the commission of the crime; 7 (2) the sworn statements of two (2) of the former housemaids of the Webb family in the persons of Nerissa E. Rosales and Mila S. Gaviola; 8 (3) the sworn-statement of Carlos J. Cristobal who alleged that on March 9, 1991 he was a passenger of United Airlines Flight No. 808 bound for New York and who expressed doubt on whether petitioner Webb was his co-passenger in the trip; (4) the sworn statement of Lolita Birrer, a former live-in partner of Gerardo Biong, who narrated the manner of how Biong investigated and tried to cover up the crime at bar; 9 (5) the sworn statements of Belen Dometita and Teofilo Minoza, two of the Vizconde maids, and the sworn statements of Normal White, a security guard and Manciano Gatmaitan, an engineer. The autopsy reports of the victims were also submitted and they showed that Carmela had nine (9) stab wounds, Estrellita twelve (12) and Jennifer nineteen (19). 10 The genital examination of Carmela confirmed the presence of spermatozoa. 11 Before submitting his counter-affidavit, petitioner Webb filed with the DOJ Panel a Motion for Production And Examination of Evidence and Documents for the NBI to produce the following:
(a) Certification issued by the U.S. Federal Bureau of Investigation on the admission to and stay of Hubert Webb in the United States from March 9, 1991 to October 22, 1992; (b) Laboratory Report No. SN-91-17 of the Medico Legal Officer, Dr. Prospero A. Cabanayan, M.D.;

(c) Sworn Statements of Gerardo C. Biong (other than his Sworn Statement dated October 7, 1991); (d) Photographs of fingerprints lifted from the Vizconde residence taken during the investigation; (e) Investigation records of NBI on Engr. Danilo Aguas, et al.; (f) List of names of 135 suspects/persons investigated by the NBI per Progress Report dated September 2, 1991 submitted by Atty. Arlis Vela, Supervising Agent; (g) Records of arrest, interview, investigation and other written statements of Jessica Alfaro (other than the May 22, 1995 Sworn Statement) conducted by the NBI and other police agencies; (h) transmittal letter to the NBI, including the report of the investigation conducted by Superintendent Rodolfo C. Sison, Regional Deputy Director, NCRC; (i) The names of NBI officials/agents composing the Task Force Jecares, including their respective positions and duties; (j) Statements made by other persons in connection with the crime charged.

The motion was granted by the DOJ Panel and the NBI submitted photocopies of the documents. It alleged it lost the original of the April 28, 1995 sworn statement of Alfaro. This compelled petitioner Webb to file Civil Case No. 951099 in the Regional Trial Court (RTC) of Makati, Br. 63, for the purpose, among others, of obtaining the original of said sworn statement. He succeeded, for in the course of its proceedings, Atty. Arturo L. Mercader, Jr., produced a copy of said original in compliance with a subpoena duces tecum. The original was then submitted by petitioner Webb to the DOJ Panel together with his other evidence. It appears, however, that petitioner Webb failed to obtain from the NBI the copy of the Federal Bureau of Investigation (FBI) Report despite his request for its production. Petitioner Webb claimed during the preliminary investigation that he did not commit the crime at bar as he went to the United States on March 1, 1991 and returned to the Philippines on October 27, 1992. 12 His alibi was corroborated by Honesto Aragon, Lecinia Edrosolano, Sylvia Climaco, Gina Roque, Sonia Rodriguez, Edgardo Venture and Pamela Francisco. 13 To further support his defense, he submitted documentary evidence that he bought a bicycle and a 1986 Toyota car while in the United States on said dates 14 and that he was issued by the State of California Driver's License No. A8818707 on June 14, 1991. 15 Petitioner Webb likewise submitted the letter dated July 25, 1995 of Mr. Robert Heafner, Legal Attache of the US Embassy, citing certain records tending to confirm, among others, his arrival at San Francisco, California on March 9, 1991 as a passenger in United Airlines Flight No. 808. The other respondents Hospicio "Pyke" Fernandez, Michael Gatchalian, Antonio "Tony Boy" Lejano, Peter Estrada, Miguel Rodriguez and Gerardo Biong submitted sworn statements, responses, and a motion to dismiss denying their complicity in the rape-killing of the Vizcondes. 16 Only the respondents Joey Filart and Artemio "Dong"

Ventura failed to file their counter-affidavits though they were served with subpoena in their last known address. 17 In his sworn statement, petitioner Gatchalian alleged that from 11 o'clock in the evening of June 29, 1991 until 3 o'clock in the morning of the following day, he was at the residence of his friends, Carlos and Andrew Syyap, at New Alabang Village, Muntinlupa watching video tapes. He claimed that his co-petitioner Lejano was with him. On August 8, 1995, the DOJ Panel issued a 26-page Resolution "finding probable cause to hold respondents for trial" and recommending that an Information for rape with homicide be filed against petitioners and their co-respondents, 18 On the same date, it filed the corresponding Information 19 against petitioners and their co-accused with the Regional Trial Court of Paraaque. The case was docketed as Criminal Case No. 95404 and raffled to Branch 258 presided by respondent judge Zosimo V. Escano. It was, however, the respondent judge Raul de Leon, pairing judge of Judge Escano, who issued the warrants of arrest against the petitioners. On August 11, 1995, Judge Escano voluntarily inhibited himself from the case to avoid any suspicion about his impartiality considering his employment with the NBI before his appointment to the bench. The case was re-raffled to Branch 274, presided by Judge Amelita Tolentino who issued new warrants of arrest against the petitioners and their co-accused. On August 11, 1995, petitioner Webb voluntarily surrendered to the police authorities at Camp Ricardo Papa Sr., in Bicutan, Taguig. Petitioners Gatchalian and Lejano likewise gave themselves up to the authorities after filing their petitions before us. In their petitions at bar, petitioners contend: (1) respondent Judges de Leon and Tolentino gravely abused their discretion when they failed to conduct a preliminary examination before issuing warrants of arrest against them: (2) the DOJ Panel likewise gravely abused its discretion in holding that there is probable cause to charge them with the crime of rape with homicide; (3) the DOJ Panel denied them their constitutional right to due process during their preliminary investigation; and (4) the DOJ Panel unlawfully intruded into judicial prerogative when it failed to charge Jessica Alfaro in the Information as an accused. We find the petitions bereft of merit. I Petitioners fault the DOJ Panel for its finding of probable cause. They insist that the May 22, 1995 sworn statement of Jessica Alfaro is inherently weak and uncorroborated. They hammer on alleged material inconsistencies between her April 28, 1995 and May 22, 1995 sworn statements. They assail her credibility for her misdescription of petitioner Webb's hair as semi-blonde. They also criticize the procedure followed by the DOJ Panel when it did not examine witnesses to clarify the alleged incredulities and inconsistencies in the sworn statements of the witnesses for the NBI.

We start with a restatement of the purpose of a preliminary investigation. Section 1 of Rule 112 provides that a preliminary investigation should determine " . . . whether there is a sufficient ground to engender a well-grounded belief that a crime cognizable by the Regional Trial Court has been committed and that the respondent is probably guilty thereof, and should be held for trial." Section 3 of the same Rule outlines the procedure in conducting a preliminary investigation, thus:
Sec. 3. Procedure. Except as provided for in Section 7 hereof, no complaint or information for an offense cognizable by the Regional Trial Court shall be filed without a preliminary investigation having been first conducted in the following manner: (a) The complaint shall state the known address of the respondent and be accompanied by affidavits of the complainant and his witnesses as well as other supporting documents, in such number of copies as there are respondents, plus two (2) copies for the official file. The said affidavits shall be sworn to before any fiscal, state prosecutor or government official authorized to administer oath, or, in their absence or unavailability, a notary public, who must certify that he personally examined the affiants and that he is satisfied that they voluntarily executed and understood their affidavits. (b) Within ten (10) days after the filing of the complaint, the investigating officer shall either dismiss the same if he finds no ground to continue with the inquiry, or issue a subpoena to the respondent, attaching thereto a copy of the complaint, affidavits and other supporting documents. Within ten (10) days from receipt thereof, the respondent shall submit counter-affidavits and other supporting documents. He shall have the right to examine all other evidence submitted by the complainant. (c) Such counter-affidavits and other supporting evidence submitted by the respondent shall also be sworn to and certified as prescribed in paragraph (a) hereof and copies thereof shall be furnished by him to the complainant. (d) If the respondent cannot be subpoenaed, or if subpoenaed, does not submit counteraffidavits within the ten (10) day period, the investigating officer shall base his resolution on the evidence presented by the complainant. (e) If the investigating officer believes that there are matters to be clarified, he may set a hearing to propound clarificatory questions to the parties or their witnesses, during which the parties shall be afforded an opportunity to be present but without the right to examine or cross-examine. If the parties so desire, they may submit questions to the investigating officer which the latter may propound to the parties or witnesses concerned. (f) Thereafter, the investigation shall be deemed concluded, and the investigating officer shall resolve the case within ten (10) days therefrom. Upon the evidence thus adduced, the investigating officer shall determine whether or not there is sufficient ground to hold the respondent for trial.

Section 4 of Rule 112 then directs that "if the investigating fiscal finds cause to hold the respondent for trial, he shall prepare the resolution and corresponding information. He shall certify under oath that he, or as shown by the record, an authorized officer, has personally examined the complainant and his witnesses, that there is reasonable ground to believe that a crime has been committed and that the accused is probably guilty thereof . . ."

The need to find probable cause is dictated by the Bill of Rights which protects "the right of the people to be secure in their persons . . . against unreasonable searches and seizures of whatever nature . . ." 20 An arrest without a probable cause is an unreasonable seizure of a person, and violates the privacy of persons which ought not to be intruded by the State. 21 Probable cause to warrant arrest is not an opaque concept in our jurisdiction. Continuing accretions of case law reiterate that they are facts and circumstances which would lead a reasonably discreet and prudent man to believe that an offense has been committed by the person sought to be arrested. 22 Other jurisdictions utilize the term man of reasonable caution 23 or the term ordinarily prudent and cautious man. 24 The terms are legally synonymous and their reference is not to a person with training in the law such as a prosecutor or a judge but to the average man on the street. 25 It ought to be emphasized that in determining probable cause, the average man weighs facts and circumstances without resorting to the calibrations of our technical rules of evidence of which his knowledge is nil. Rather, he relies on the calculus of common sense of which all reasonable men have an abundance. Applying these basic norms, we are not prepared to rule that the DOJ Panel gravely abused its discretion when it found probable cause against the petitioners. Petitioners belittle the truthfulness of Alfaro on two (2) grounds: (a) she allegedly erroneously described petitioner Webb's hair as semi-blond and (b) she committed material inconsistencies in her two (2) sworn statement, thus: 26
xxx xxx xxx To illustrate, the following are some examples of inconsistencies in the two sworn statements of Alfaro: On whether Alfaro knew Carmela before the incident in question First Affidavit: She had NOT met Carmela before June 29, 1991. Second Affidavit: "I met her in a party sometime in February, 1991." On whether Alfaro saw the dead bodies First Affidavit: She did not see the three dead persons on that night. She just said "on the following day I read in the newspaper that there were three persons who were killed . . ." Second Affidavit: "I peeped through the first door on the left. I saw two bodies on top of the bed, bloodied, and in the floor, I saw Hubert on top of Carmela." On the alleged rape of Carmela Vizconde First Affidavit: She did not see the act of rape.

Second Affidavit: She saw Hubert Webb "with bare buttocks, on top of Carmela and pumping, her mouth gagged and she was moaning and I saw tears on her eyes." On how Webb, Lejano, and Ventura entered the Vizconde house First Affidavit: "by jumping over the fence, which was only a little more than a meter high." Second Affidavit: They "entered the gate which was already open." On whether Alfaro entered the Vizconde house First Affidavit: She never entered the house. Second Affidavit: "I proceeded to the iron grill gate leading to the dirty kitchen."

In its Resolution, the DOJ Panel ruled that these alleged misdescription and inconsistencies did not erode the credibility of Alfaro. We quote the pertinent ruling, viz.: 27
xxx xxx xxx As regards the admissibility of Alfaro's statements, granting for purposes of argument merely that she is a co-conspirator, it is well to note that confessions of a co-conspirator may be taken as evidence to show the probability of the co-conspirator's participation in the commission of the crime (see People vs. Lumahang, 94 Phil. 1084). Furthermore, it is a well-established doctrine that conspiracy need not be proved by direct evidence of prior agreement to commit the crime. Indeed, "only rarely would such a prior agreement be demonstrable since, in the nature of things, criminal undertakings are only rarely documented by agreements in writing. Thus, conspiracy may be inferred from the conduct of the accused before, during and after the commission of the crime, showing that the several accused had acted in concert or in unison with each other, evincing a common purpose or design." (Angelo vs. Court of Appeals, 210 SCRA 402 [1992], citations omitted; People vs. Molleda, 86 SCRA 699). Neither can we discredit Alfaro merely because of the inconsistencies in her two sworn statements. In Angelo, the Court refused to discredit the testimony of a witness accusing therein petitioner for the slaying of one Gaviano Samaniego even though said witness failed to name Angelo in his affidavit which was executed five (5) months earlier. Granting, the Court continued, that a part of the witness' testimony is untrue, such circumstance is not sufficient to discredit the entire testimony of the witness. On August 7, 1995, another counsel for respondent Webb submitted his memorandum suggesting that the instant complaint "should not be decided within the month to give time to the NBI to coordinate with the FBI on the latter's inquiry into the whereabouts of Hubert Webb . . . and to check on our U.S.-based witnesses."

In said memorandum, counsel for respondent Webb calls for the application of the maxim falsus in uno, falsus in omnibus arising from the inconsistencies of Alfaro's statements, among others. This is untenable. As held in Angelo: There is no rule of law which prohibits a court from crediting part of the testimony of a witness as worthy of belief and from simultaneously rejecting other parts which the court may find incredible or dubious. The maxim falsus in uno, falsus in omnibus is not a rule of law, let alone a general rule of law which is universally applicable. It is not a legal presumption either. It is merely a latinism describing the conclusion reached by a court in a particular case after ascribing to the evidence such weight or lack of weight that the court deemed proper. In the case before us, complainant reasoned out that Alfaro was then having reservations when she first executed the first statement and held back vital information due to her natural reaction of mistrust. This being so, the panel believes that the inconsistencies in Alfaro's two sworn statements have been sufficiently explained especially specially so where there is no showing that the inconsistencies were deliberately made to distort the truth. Consequently, the probative value of Alfaro's testimony deserves full faith and credit. As it has been often noted, ex parte statements are generally incomplete because they are usually executed when the affiant's state of mind does not give her sufficient and fair opportunity to comprehend the import of her statement and to narrate in full the incidents which transpired (People vs. Sarellana, 233 SCRA 31 [1994]; Angelo vs. Court of Appeals, supra). In the case at bar, there is no dispute that a crime has been committed and what is clear before us is that the totality of the evidence submitted by the complainant indicate a prima facie case that respondents conspired in the perpetration of the imputed offense.

We note that the May 22, 1995 sworn statement of Alfaro was given with the assistance of counsel 28 and consists of six (6) pages, in single space reciting in rich details how the crime was planned and then executed by the petitioners. In addition, the DOJ Panel evaluated the supporting sworn statements of Nerissa Rosales and Mila Gaviola, former housemaids of the Webbs, Carlos J. Cristobal, a passenger in United Airlines Flight No. 808 and Lolita Birrer, a paramour of Gerardo Biong. The Panel assayed their statements as follows: 29
xxx xxx xxx According to Nerissa E. Rosales, a former housemaid of the Webb family, on June 29, 1991, between 7:00 o'clock and 8:00 o'clock in the evening, Hubert was at home inside his room with two male visitors. She knew it because she and her co-housemaid, Loany, were instructed by Hubert to bring them three glasses of juice. It was the last time she saw Hubert and was later told by then Congressman Webb that Hubert was in the United States. While Mila S. Gaviola, another former housemaid of the Webb family and who served as a laundry woman, claims, aside from corroborating the statement of Nerissa Rosales, that on June 30, 1991, she woke up at around 4:00 in the morning and as what she used to do, she entered the rooms of the Webbs to get their clothes to be washed. As a matter of fact, in that early morning, she entered Hubert's room and saw Hubert, who was only wearing his pants, already awake and smoking while he was sitting on his bed. She picked up Hubert's scattered clothes and brought them together with the clothes of the other members of the family to the laundry area. After taking her breakfast, she began

washing the clothes of the Webbs. As she was washing the clothes of Hubert Webb, she noticed fresh bloodstains in his shirt. After she finished the laundry, she went to the servant's quarters. But feeling uneasy, she decided to go up to the stockroom near Hubert's room to see what he was doing. In the said stockroom, there is a small door going to Hubert's room and in that door there is a small opening where she used to see Hubert and his friends sniffing on something. She observed Hubert was quite irritated, uneasy, and walked to and from inside his room. On that day, she noticed Hubert left the house at around 1:00 in the afternoon and came back at around 4:00 in the same afternoon and went inside his room using the secret door of the house. It was the last time that she saw Hubert until she left the Webb family. On the other hand, Carlos J. Cristobal alleged that on March 9, 1991, at about 10:00 in the morning, he was at the Ninoy Aquino International Airport as he was then scheduled to take the United Airlines Flight No. 808 at 2:00 in the afternoon for New York. At the airport's lobby, he saw then Congressman Freddie Webb with a male companion. He greeted him and Webb answered: "Mabuti naman, at ito, ihahatid ko ang anak ko papuntang Florida." He knew Freddie Webb because he often watched him then in a television show "Chicks to Chicks." He observed that the man whom Freddie Webb referred to as his son, was of the same height as Freddie. The son referred to has fair complexion with no distinguishing marks on his face. He (son of Webb) was then wearing a striped white jacket. When he and his children were already inside the plane, he did not see Freddie anymore, but he noticed his son was seated at the front portion of the economy class. He never noticed Freddie Webb's son upon their arrival in San Francisco. He claims that, while watching the television program "DONG PUNO LIVE" lately, he saw the wife of Freddie Webb with her lawyer being interviewed, and when she described Hubert as "moreno" and small built, with a height of five feet and seven inches tall, and who was the one who left for United States on March 9, 1991, he nurtured doubts because such description does not fit the physical traits of the son of Freddie, who left with him for United States on the same flight and date. Lolita Birrer, alleged that she know Gerardo Biong because she had an affair with him for almost three (3) years and in fact, she had a child with him who is now four (4) years old. Their relationship started in February, 1991 until she broke up with him in September 1993. She recalls that on June 29, 1991, at around 6:00 p.m., Biong invited her to play mahjong at the canteen of a certain Aling Glo located at the back of the Paraaque Municipal Hall. At about 2:30, in the early morning of January 30, 1991, the radio operator of the Paraaque police told Biong that he has a phone call. Before Biong went to the radio room, she was instructed to take him over and after somebody won the game, she followed Biong at the radio room where she overheard him uttering, "Ano?, Saan? Mahirap yan, Paano, o sige, aantayin kita, O ano?, dilaw na taxi, o sige." When he put the phone down, Biong told her, "Mayroon lang akong rerespondehan, ikaw muna ang maupo" and then, he went outside the canteen apparently waiting for somebody. Twenty minutes later, a taxi, colored yellow, arrived with a male passenger sitting at the backseat and parked near the canteen. After it made some signals by blinking its headlight, Biong rode thereat at the front seat beside the driver and then, they left. She was not able to recognize the male passenger because the window of the taxi was tinted. Biong came back at around 7:00 of the same morning and when he arrived, he immediately washed his hands and face, and took his handkerchief from his pocket which he threw at the trash can. She asked him why he threw his handkerchief and he answered, "Hmp . . . amoy tae." She inquired what happened in BF Homes and he replied, "Putang inang mga batang iyon, pinahirapan nila ako."

Biong later invited her for breakfast, but they first went to his office where she observed him doing something in his steel cabinet while he appeared to be uneasy. Moments later, Galvan, another policeman of Paraaque, arrived and said, "Oy Biong, may tatlong patay sa BF, imbestigahan mo" to which Biong answered, "Oo susunod na ako." Biong went to the office of Capt. Don Bartolome who offered to accompany him and with whom she asked permission to go with them. Before they proceeded to the place where the killings happened, she asked Biong if he knew the exact address and the latter immediately responded, "Alam ko na yon." She was surprised because Galvan never told him the place of the incident. As soon as they arrived at the Vizconde's residence, Biong instructed the housemaids to contact the victim's relatives, while the security guard fetched the barangay chairman and the president of the Homeowners Association. When all these persons were already in the house, Biong started recording the wounds of the victim. Inside the master's bedroom, she saw Biong took a watch from the jewelry box. Because she could not tolerate the foul odor, she and Capt. Bartolome went out of the room and proceeded to the dining area. On top of the dining table, she saw the scattered contents of a shoulder bag. Moments later, Biong came out from the room and proceeded to the front door to remove the chain lock; asked the keys from the housemaid and it was only then that the main door was opened. Biong noticed a stone in front of the broken glass of the door and requested Capt. Bartolome to go inside the servant's quarters as he doubted the housemaids' claim that they heard nothing unusual. Using the handle of his gun, Biong broke the remaining glass of the door panel. Bartolome then came out of the room and told Biong that he can hear the sound of the glass being broken. At the garage, Biong also noticed same marks on the hood of the car. On the following day, at around 12:00 noon, Biong arrived in her house together with the Vizconde housemaids. When Biong was preparing to take a bath, she saw him remove from his pocket the things she also saw from Vizconde's residence, to wit: calling cards, driver's license, ATM card, a crossed check worth P80,000.00, earrings, a ring, bracelet, necklace, and the watch he took from the jewelry box inside the room of the Vizcondes. These jewelry items were later pawned by Biong for P20,000.00 at a pawnshop in front of Chow-Chow restaurant in Santos Avenue, Paraaque. The next day, she saw Biong took from his locker at the Paraaque Police Station an imported brown leather jacket, which the latter claimed to have been given to him by the person who called him up in the early morning of June 30, 1991. Since then, Biong has been wearing said jacket until they broke up sometime in 1993. She observed that Biong seemed not interested in pursuing the investigation of the Vizconde case. In fact, when Biong and this group picked up Mike Gatchalian and brought him to the Paraaque Police Station, she was surprised that Biong halted the investigation when Gatchalian was profusely sweating while being interrogated. After the father of Gatchalian talked to Colonel Pureza, the latter called up and instructed Biong to bring Gatchalian to him (Colonel Pureza) and that was the last thing she remembered regarding this case.

The DOJ Panel then weighed these inculpatory evidence against the exculpatory evidence of petitioners. It ruled: 30
xxx xxx xxx The voluminous number of exhibits submitted by respondent Webb to support his defense of denial and alibi notwithstanding, the panel, after a careful and thorough evaluation of the records, believes that they cannot outweigh the evidence submitted by

the complainant. Alibi cannot prevail over the positive identification made by a prosecution witness. Verily, alibi deserves scant consideration in the face of positive identification especially so where the claim of alibi is supported mainly by friends and relatives (People vs. Apolonia, 235 SCRA 124 [1994]; People vs. Lucas, 181 SCRA 316 and a long line of cases). Similarly, denial is a self-serving negative which cannot be given greater evidentiary weight than the declaration of a credible witness who testified on affirmative matters (People vs. Carizo, 233 SCRA 687 [1994]). Indeed, denial, like alibi, is weak and becomes even more weaker when arrayed against the positive identification by the witness for the prosecution (People vs. Onpaid, 233 SCRA 62 [1994]). Surprisingly, Gatchalian's defense of alibi was not corroborated by Lejano, whom he claimed was with him watching video tapes at the Syyap residence. Other than claiming that he "was not and could not have been at or near the area of the Vizconde residence at the time of the alleged commission of the crime," respondent Lejano proffered no evidence to substantiate his claim of alibi. xxx xxx xxx On the other hand, respondent Webb seeks to enhance the acceptability of his alibi in the form of documents tending to show that he was thousands of miles away when the incident occurred. We have carefully deliberated and argued on the evidence submitted by respondent Webb in support of his absence from the country since March 9, 1991 to October 26, 1992 and found the same wanting to exonerate him of the offense charged. The material dates in this case are June 29 and 30, 1991. While respondent Webb may have submitted proof tending to show that he was issued a California driver's license on June 14, 1991, there is no showing that he could not have been in the country on the dates above mentioned. Neither do we find merit in the allegation that respondent Webb personally bought a bicycle on June 30, 1991 in California in view of his positive identification by Alfaro and the two (2) househelps of the Webb family who testified that he was here in the country on said dates. Additionally, the issuance of receipt evidencing the purchase of a bicycle in California is no conclusive proof that the name appearing thereon was the actual buyer of the merchandise.

Given these conflicting pieces of evidence of the NBI and the petitioners, we hold that the DOJ Panel did not gravely abuse its discretion when it found probable cause against the petitioners. A finding of probable cause needs only to rest on evidence showing that more likely than not a crime has been committed and was committed by the suspects. Probable cause need not be based on clear and convincing evidence of guilt, neither on evidence establishing guilt beyond reasonable doubt and definitely, not on evidence establishing absolute certainty of guilt. As well put in Brinegar v. United States, 31 while probable cause demands more than "bare suspicion," it requires "less than evidence which would justify . . . conviction." A finding of probable cause merely binds over the suspect to stand trial. It is not a pronouncement of guilt. Considering the low quantum and quality of evidence needed to support a finding of probable cause, we also hold that the DOJ Panel did not, gravely abuse its discretion in refusing to call the NBI witnesses for clarificatory questions. The decision to call witnesses for clarificatory questions is addressed to the sound

discretion of the investigator and the investigator alone. If the evidence on hand already yields a probable cause, the investigator need not hold a clarificatory hearing. To repeat, probable cause merely implies probability of guilt and should be determined in a summary manner. Preliminary investigation is not a part of trial and it is only in a trial where an accused can demand the full exercise of his rights, such as the right to confront and cross-examine his accusers to establish his innocence. In the case at bar, the DOJ Panel correctly adjudged that enough evidence had been adduced to establish probable cause and clarificatory hearing was unnecessary. II We now come to the charge of petitioners that respondent Judge Raul de Leon and, later, respondent Judge Amelita Tolentino issued warrants of arrest against them without conducting the required preliminary examination. Petitioners support their stance by highlighting the following facts: (1) the issuance of warrants of arrest in a matter of few hours; (2) the failure of said judges to issue orders of arrest; (3) the records submitted to the trial court were incomplete and insufficient from which to base a finding of probable cause; and (4) that even Gerardo Biong who was included in the Information as a mere accessory had a "NO BAIL" recommendation by the DOJ Panel. Petitioners postulate that it was impossible to conduct a "searching examination of witnesses and evaluation of the documents" on the part of said judges. The issuance of a warrant of arrest interferes with individual liberty and is regulated by no less than the fundamental law of the land. Section 2 of Article III of the Constitution provides:
Sec. 2. The right of the people to be secure in their persons, houses, papers, and effects against unreasonable searches and seizures of whatever nature and for any purpose shall be inviolable, and no search warrant or warrant of arrest shall issue except upon probable cause to be determined personally by the judge after examination under oath or affirmation of the complainant and the witnesses he may produce and particularly describing the place to be searched and the persons or things to be seized.

The aforequoted provision deals with the requirements of probable cause both with respect to issuance of warrants of arrest or search warrants. The similarities and differences of their requirements ought to be educational. Some of them are pointed out by Professors LaFave and Israel, thus: 32 "It is generally assumed that the same quantum of evidence is required whether one is concerned with probable cause to arrest or probable cause to search. But each requires a showing of probabilities as to somewhat different facts and circumstances, and thus one can exist without the other. In search cases, two conclusions must be supported by substantial evidence: that the items sought are in fact seizable by virtue of being connected with criminal activity, and that the items will be found in the place to be searched. It is not also necessary that a particular person be implicated. By comparison, in arrest cases there must be probable cause that a

crime has been committed and that the person to be arrested committed it, which of course can exist without any showing that evidence of the crime will be found at premises under that person's control." Worthy to note, our Rules of Court do not provide for a similar procedure to be followed in the issuance of warrants of arrest and search warrants. With respect to warrants of arrest, section 6 of Rule 112 simply provides that "upon filing of an information, the Regional Trial Court may issue a warrant for the arrest of the accused." In contrast, the procedure to be followed in issuing search warrants is more defined. Thus, Sections 3, 4 and 5 of Rule 126 provide:
xxx xxx xxx Sec. 3. Requisites for issuing search warrant. A search warrant shall not issue but upon probable cause in connection with one specific offense to be determined personally by the judge after examination under oath or affirmation of the complainant and the witnesses he may produce, and particularly describing the place to be searched and the things to be seized. Sec. 4. Examination of complainant; record. The judge must, before issuing the warrant, personally examine in the form of searching questions and answers, in writing and under oath the complainant and any witnesses he may produce on facts personally known to them and attach to the record their sworn statements together with any affidavits submitted. Sec. 5. Issuance and form of search warrant. If the judge is thereupon satisfied of the facts upon which the application is based, or that there is probable cause to believe that they exist, he must issue the warrant, which must be substantially in the form prescribed by these Rules.

We discussed the difference in the Procedure of issuing warrants of arrest and search warrants in Soliven vs. Makasiar, 33 thus:
xxx xxx xxx The second issue, raised by Beltran, calls for an interpretation of the constitutional provision on the issuance of warrants of arrest. The pertinent provision reads: Art. III, Sec. 2. The right of the people to be secure in their persons, houses, papers and effects against unreasonable searches and seizures of whatever nature and for any purpose shall be inviolable, and no search warrant or warrant of arrest shall issue except upon probable cause to be determined personally by the judge after examination under oath or affirmation of the complainant and the witnesses he may produce, and particularly describing the place to be searched and the persons or things to be seized. The addition of the word "personally" after the word "determined" and the deletion of the grant of authority by the 1973 Constitution to issue warrants to "other responsible officers as may be authorized by law," has apparently convinced petitioner Beltran that the Constitution now requires the judge to personally examine the complainant and his witnesses in his determination of probable cause for the issuance of warrants of arrest. This is not an accurate interpretation.

What the Constitution underscores is the exclusive and personal responsibility of the issuing judge to satisfy himself of the existence of probable cause. In satisfying himself of the existence of probable cause for the issuance of a warrant of arrest, the judge is not required to personally examine the complainant and his witnesses. Following established doctrine and procedure, he shall: (1) personally evaluate the report and the documents submitted by the fiscal regarding the existence of probable cause and, on the basis thereof, issue a warrant; or (2) if on the basis thereof he finds no probable cause, he may disregard the fiscal's report and require the submission of supporting affidavits of witnesses to aid him in arriving at a conclusions as to the existence of probable cause. Sound policy dictates this procedure, otherwise judges would be unduly laden with the preliminary examination and investigation of criminal complaints instead of concentrating on hearing and deciding cases filed before their courts.

Clearly then, the Constitution, the Rules of Court, and our case law 34 repudiate the submission of petitioners that respondent judges should have conducted "searching examination of witnesses" before issuing warrants of arrest against them. They also reject petitioners' contention that a judge must first issue an order of arrest before issuing a warrant of arrest. There is no law or rule requiring the issuance of an Order of Arrest prior to a warrant of arrest. In the case at bar, the DOJ Panel submitted to the trial court its 26-page report, the two (2) sworn statements of Alfaro and the sworn statements of Carlos Cristobal and Lolita Birrer 35 as well as the counter-affidavits of the petitioners. Apparently, the painstaking recital and analysis of the parties' evidence made in the DOJ Panel Report satisfied both judges that there is probable cause to issue warrants of arrest against petitioners. Again, we stress that before issuing warrants of arrest, judges merely determine personally the probability, not the certainty of guilt of an accused. In doing so, judges do not conduct a de novo hearing to determine the existence of probable cause. They just personally review the initial determination of the prosecutor finding a probable cause to see if it is supported by substantial evidence. The sufficiency of the review process cannot be measured by merely counting minutes and hours. The fact that it took the respondent judges a few hours to review and affirm the probable cause determination of the DOJ Panel does not mean they made no personal evaluation of the evidence attached to the records of the case. 36 Petitioners' reliance on the case of Allado vs. Diokno 37 is misplaced. Our Allado ruling is predicated on the utter failure of the evidence to show the existence of probable cause. Not even the corpus delicti of the crime was established by the evidence of the prosecution in that case. Given the clear insufficiency of the evidence on record, we stressed the necessity for the trial judge to make a further personal examination of the complainant and his witnesses to reach a correct assessment of the existence or non-existence of probable cause before issuing warrants of arrest against the accused. The case at bar, however, rests on a different factual setting. As priorly discussed, the various types of evidence extant in the records of the case provide substantial basis for a finding of probable cause against the petitioner. The corpus delicti of the crime is a given

fact. There is an eyewitness account of the imputed crime given by Alfaro. The alibi defense of petitioner Webb is also disputed by sworn statements of their former maids. It was therefore unnecessary for the respondent judges to take the further step of examining ex parte the complainant and their witnesses with searching questions. III Petitioners also complain about the denial of their constitutional right to due process and violation of their right to an impartial investigation. They decry their alleged hasty and malicious prosecution by the NBI and the DOJ Panel. They also assail the prejudicial publicity that attended their preliminary investigation. We reject these contentions. The records will show that the DOJ Panel did not conduct the preliminary investigation with indecent haste. Petitioners were given fair opportunity to prove lack of probable cause against them. The fairness of this opportunity is well stressed in the Consolidated Comment of the Solicitor General, viz.:
Again, there is no merit in this contention. Petitioners were afforded all the opportunities to be heard. Petitioner Webb actively participated in the preliminary investigation by appearing in the initial hearing held on June 30, 1995 and in the second hearing on July 14, 1995; and by filing a "Motion for Production and Examination of Evidence and Documents" on June 27, 1995 (p. 4, Petition), a "Reply to the compliance and Comment/Manifestation to the Motion for Production and Examination of Evidence" on July 5, 1995 (p. 6, Petition), a "Comment and Manifestation" on July 7, 1995 (p. 6, Petition), his "Counter-Affidavit" on July 14, 1995 (pp. 6-7, Petition) and a "Motion to Resolve" on August 1, 1995. Numerous letter-requests were also sent by the petitioner Webb's counsel to the DOJ Panel requesting the latter to furnish him a copy of the reports prepared by the FBI concerning the petitioner's whereabouts during the material period (Annexes "L", "L-1" and "L-2" of the Supplemental Petition dated August 14, 1995). In fact, not satisfied with the decision of the DOJ Panel not to issue subpoena duces tecum to Atty. Arturo L. Mercader, Jr., petitioner Webb filed a "Petition for Injunction, Certiorari, Prohibition and Mandamus" with the Regional Trial Court, Branch 63 of Makati in order to compel said Atty. Mercader, Jr. to produce the first sworn statement of Alfaro for submission to the DOJ Panel. (p. 4, Petition) The said court dismissed the petition after Mercader produced and submitted to the DOJ Panel the first sworn statement of Alfaro, without ruling on the admissibility and credence of the two (2) conflicting and inconsistent sworn statements of the principal witness, Alfaro. (Attached hereto is a copy of the order of Judge Ruben A. Mendiola, RTC-Makati, Branch 63 dated July 28, 1995) marked as Annex "F." It must also be pointed out that despite the declaration by the DOJ Panel that the preliminary investigation was to be terminated after the hearing held on July 14, 1995, the panel continued to conduct further proceedings, e.g. comparison of the photo-copies of the submitted documents with the originals on July 17, 1995. (p. 7, Petition) The panel even entertained the "Response" submitted by accused Miguel Rodriguez on July 18, 1995. (p. 17 Resolution) In addition to these, the panel even announced that any party may submit additional evidence before the resolution of the case. (p. 8, Petition) From the time the panel declared the termination of the preliminary investigation on July 14, 1995, twenty-seven (27) days elapsed before the resolution was promulgated, and the information eventually filed in the Regional Trial Court of Paraaque on August 10, 1995.

This notwithstanding the directive of Section 3(f) Rule 112 of the Revised Rules of Court that the investigating officer shall resolve the case within ten (10) days from the termination of the preliminary investigation. The DOJ Panel precisely allowed the parties to adduce more evidence in their behalf and for the panel to study the evidence submitted more fully. This directly disputes the allegation of the petitioners that the resolution was done with indecent haste in violation of the rights of the petitioners. During the period of twenty-seven (27) days, the petitioners were free to adduce and present additional evidence before the DOJ Panel. Verily, petitioners cannot now assert that they were denied due process during the conduct of the preliminary investigation simply because the DOJ Panel promulgated the adverse resolution and filed the Information in court against them.

Petitioners cannot also assail as premature the filing of the Information in court against them for rape with homicide on the ground that they still have the right to appeal the adverse resolution of the DOJ Panel to the Secretary of Justice. The filing of said Information is in accord with Department of Justice Order No. 223, series of 1993, dated June 25, 1993. We quote its pertinent sections, viz.:
Sec. 4. Non-Appealable Cases; Exceptions. No appeal may be taken from a resolution of the Chief State Prosecutor/Regional State Prosecutor/Provincial or City Prosecutor finding probable cause except upon showing of manifest error or grave abuse of discretion. Notwithstanding the showing of manifest error or grave abuse of discretion, no appeal shall be entertained where the appellant had already been arraigned. If the appellant is arraigned during the pendency of the appeal, said appeal shall be dismissed motu propio by the Secretary of Justice. An appeal/motion for reinvestigation from a resolution finding probable cause, however, shall not hold the filing of the information in court. Sec. 2. When to appeal. The appeal must be filed within a period of fifteen (15) days from receipt of the questioned resolution by the party or his counsel. The period shall be interrupted only by the filing of a motion for reconsideration within ten (10) days from receipt of the resolution and shall continue to run from the time the resolution denying the motion shall have been received by the movant or his counsel. (Emphasis supplied)

Without doubt then, the said DOJ Order No. 223 allows the filing of an Information in court after the consummation of the preliminary investigation even if the accused can still exercise the right to seek a review of the prosecutor's recommendation with the Secretary of Justice. Next, petitioners fault the DOJ Panel for not including Alfaro in the Information considering her alleged conspiratorial participation in the crime of rape with homicide. The non-inclusion of Alfaro is anchored on Republic Act No. 6981, entitled "An Act Providing For A Witness Protection, Security And Benefit Program And For Other Purposes" enacted on April 24, 1991. Alfaro qualified under its Section 10, which provides:
xxx xxx xxx

Sec. 10. State Witness. Any person who has participated in the commission of a crime and desires to a witness for the State, can apply and, if qualified as determined in this Act and by the Department, shall be admitted into the Program whenever the following circumstances are present: (a) the offense in which his testimony will be used is a grave felony as defined under the R.P.C. or its equivalent under special laws; (b) there is absolute necessity for his testimony; (c) there is no other direct evidence available for the proper prosecution of the offense committed; (d) his testimony can be substantially corroborated on its material points; (e) he does not appear to be most guilty; and (f) he has not at anytime been convicted of any crime involving moral turpitude. An accused discharged from an information or criminal complaint by the court in order that he may be a State Witness pursuant to Sections 9 and 10 of Rule 119 of the Revised Rules of Court may upon his petition be admitted to the Program if he complies with the other requirements of this Act. Nothing in this Act shall prevent the discharge of an accused so that he can be used as a Witness under Rule 119 of the Revised Rules of Court.

Upon qualification of Alfaro to the program, Section 12 of the said law mandates her non-inclusion in the criminal Complaint or Information, thus:
xxx xxx xxx Sec. 12. Effect of Admission of a State Witness into the Program. The certification of admission into the Program by the Department shall be given full faith and credit by the provincial or city prosecutor who is required NOT TO INCLUDE THE WITNESS IN THE CRIMINAL COMPLAINT OR INFORMATION and if included therein, to petition the court for his discharge in order that he can be utilized as a State Witness. The court shall order the discharge and exclusion of the said accused from the information. Admission into the Program shall entitle such State Witness to immunity from criminal prosecution for the offense or offenses in which his testimony will be given or used and all the rights and benefits provided under Section 8 hereof.

The validity of these provisions is challenged by petitioner Webb. It is urged that they constitute ". . . an intrusion into judicial prerogative for it is only the court which has the power under the Rules on Criminal Procedure to discharge an accused as a state witness." The argument is based on Section 9, Rule 119 38 which gives the court the prerogative to approve the discharge of an accused to be a state witness. Petitioner's argument lacks appeal for it lies on the faulty assumption that the decision whom to prosecute is a judicial function, the sole prerogative of courts and beyond executive and legislative interference. In truth, the prosecution of crimes appertains to the executive department of government

whose principal power and responsibility is to see that our laws are faithfully executed. A necessary component of this power to execute our laws is the right to prosecute their violators. The right to prosecute vests the prosecutor with a wide range of discretion the discretion of whether, what and whom to charge, the exercise of which depends on a smorgasbord of factors which are best appreciated by prosecutors. We thus hold that it is not constitutionally impermissible for Congress to enact R.A. No. 6981 vesting in the Department of Justice the power to determine who can qualify as a witness in the program and who shall be granted immunity from prosecution. 39 Section 9 of Rule 119 does not support the proposition that the power to choose who shall be a state witness is an inherent judicial prerogative. Under this provision, the court, is given the power to discharge a state witness only because it has already acquired jurisdiction over the crime and the accused. The discharge of an accused is part of the exercise of jurisdiction but is not a recognition of an inherent judicial function. Moreover, the Rules of Court have never been interpreted to be beyond change by legislation designed to improve the administration of our justice system. R.A. No. 6981 is one of the much sought penal reform laws to help government in its uphill fight against crime, one certain cause of which is the reticence of witnesses to testify. The rationale for the law is well put by the Department of Justice, viz.: "Witnesses, for fear of reprisal and economic dislocation, usually refuse to appear and testify in the investigation/prosecution of criminal complaints/cases. Because of such refusal, criminal complaints/cases have been dismissed for insufficiency and/or lack of evidence. For a more effective administration of criminal justice, there was a necessity to pass a law protecting witnesses and granting them certain rights and benefits to ensure their appearance in investigative bodies/courts." 40 Petitioner Webb's challenge to the validity of R.A. No. 6981 cannot therefore succeed. Further, petitioners charge the NBI with violating their right to discovery proceedings during their preliminary investigation by suppressing the April 28, 1995 original copy of the sworn statement of Alfaro and the FBI Report. The argument is novel in this jurisdiction and as it urges an expansive reading of the rights of persons under preliminary investigation it deserves serious consideration. To start with, our Rules on Criminal Procedure do not expressly provide for discovery proceedings during the preliminary investigation stage of a criminal proceeding. 41 Sections 10 and 11 of Rule 117 do provide an accused the right to move for a bill of particulars and for production or inspection of material evidence in possession of the prosecution. 42 But these provisions apply after the filing of the Complaint or Information in court and the rights are accorded to the accused to assist them to make an intelligent plea at arraignment and to prepare for trial. 43 This failure to provide discovery procedure during preliminary investigation does not, however, negate its use by a person under investigation when indispensable to protect his constitutional right to life, liberty and property. Preliminary investigation is not too early a stage to guard against any significant erosion of

the constitutional right to due process of a potential accused. As aforediscussed, the object of a preliminary investigation is to determine the probability that the suspect committed a crime. We hold that the finding of a probable cause by itself subjects the suspect's life, liberty and property to real risk of loss or diminution. In the case at bar, the risk to the liberty of petitioners cannot be understated for they are charged with the crime of rape with homicide, a non-bailable offense when the evidence of guilt is strong. Attuned to the times, our Rules have discarded the pure inquisitorial system of preliminary investigation. Instead, Rule 112 installed a quasi-judicial type of preliminary investigation conducted by one whose high duty is to be fair and impartial. 44 As this Court emphasized in Rolito Go vs. Court of Appeals, 45 "the right to have a preliminary investigation conducted before being bound over for trial for a criminal offense, and hence formally at risk of incarceration or some other penalty, is not a mere formal or technical right; it is a substantive right." A preliminary investigation should therefore be scrupulously conducted so that the constitutional right to liberty of a potential accused can be protected from any material damage. We uphold the legal basis of the right of petitioners to demand from their prosecutor, the NBI, the original copy of the April 28, 1995 sworn statement of Alfaro and the FBI Report during their preliminary investigation considering their exculpatory character, and hence, unquestionable materiality to the issue of their probable guilt. The right is rooted on the constitutional protection of due process which we rule to be operational even during the preliminary investigation of a potential accused. It is also implicit in section (3) (a) of Rule 112 which requires during the preliminary investigation the filing of a sworn complaint, which shall ". . . state the known address of the respondent and be accompanied by affidavits of the complainant and his witnesses as well as other supporting documents . . ." In laying down this rule, the Court is not without enlightened precedents from other jurisdictions. In the 1963 watershed case of Brady v. Maryland 46 the United States Supreme Court held that "suppression of evidence favorable to an accused upon request violates due process where the evidence is material to guilt or punishment, irrespective of the good faith or bad faith of the prosecution." Its progeny is the 1935 case of Mooney v. Holohan 47 which laid down the proposition that a prosecutor's intentional use of perjured testimony to procure conviction violates due process. Thus, evolved jurisprudence firming up the prosecutor's duty to disclose to the defense exculpatory evidence in its possession. 48 The rationale is well put by Justice Brennan in Brady 49 "society wins not only when the guilty are convicted but when criminal trials are fair." Indeed, prosecutors should not treat litigation like a game of poker where surprises can be sprung and where gain by guile is not punished. But given the right of petitioners to compel the NBI to disclose exculpatory evidence in their favor, we are not prepared to rule that the initial non-production of the original sworn statement of Alfaro dated April 28, 1995 could have resulted

in the reasonable likelihood that the DOJ Panel would not have found probable cause. To be sure, the NBI, on July 4, 1995, upon request of petitioners, submitted a photocopy of Alfaro's April 28, 1995 sworn statement. It explained it cannot produce the original as it had been lost. Fortunately, petitioners, on July 28, 1995, were able to obtain a copy of the original from Atty. Arturo Mercader in the course of the proceedings in Civil Case No. 951099. 50 As petitioners admit, the DOJ Panel accepted the original of Alfaro's April 28, 1995 sworn statement as a part of their evidence. 51 Petitioners thus had the fair chance to explain to the DOJ Panel then still conducting their preliminary investigation the exculpatory aspects of this sworn statement. Unfortunately for petitioners, the DOJ Panel still found probable cause to charge them despite the alleged material discrepancies between the first and second sworn statements of Alfaro. For reasons we have expounded, this finding of probable cause cannot be struck down as done with grave abuse of discretion. 52 On the other hand, the FBI Report while corroborative of the alibi of petitioner Webb cannot by itself reverse the probable cause finding of the DOJ Panel in light of the totality of evidence presented by the NBI. Finally, we come to the argument of petitioner that the DOJ Panel lost its impartiality due to the prejudicial publicity waged in the press and broadcast media by the NBI. Again, petitioners raise the effect of prejudicial publicity on their right to due process while undergoing preliminary investigation. We find no procedural impediment to its early invocation considering the substantial risk to their liberty while undergoing a preliminary investigation. In floating this issue, petitioners touch on some of the most problematic areas in constitutional law where the conflicting demands of freedom of speech and of the press, the public's right to information, and an accused's right to a fair and impartial trial collide and compete for prioritization. The process of pinpointing where the balance should be struck has divided men of learning as the balance keeps moving either on the side of liberty or on the side of order as the tumult of the time and the welfare of the people dictate. The dance of balance is a difficult act to follow. In democratic settings, media coverage of trials of sensational cases cannot be avoided and oftentimes, its excessiveness has been aggravated by kinetic developments in the telecommunications industry. For sure, few cases can match the high volume and high velocity of publicity that attended the preliminary investigation of the case at bar. Our daily diet of facts and fiction about the case continues unabated even today. Commentators still bombard the public with views not too many of which are sober and sublime. Indeed, even the principal actors in the case the NBI, the respondents, their lawyers and their sympathizers have participated in this media blitz. The possibility of media abuses and their threat to a fair trial notwithstanding, criminal trials cannot be

completely closed to the press and the public. In the seminal case of Richmond Newspapers, Inc. v. Virginia, 53 it was wisely held:
xxx xxx xxx (a) The historical evidence of the evolution of the criminal trial in Anglo-American justice demonstrates conclusively that at the time this Nation's organic laws were adopted, criminal trials both here and in England had long been presumptively open, thus giving assurance that the proceedings were conducted fairly to all concerned and discouraging perjury, the misconduct of participants, or decisions based on secret bias or partiality. In addition, the significant community therapeutic value of public trials was recognized: when a shocking crime occurs, a community reaction of outrage and public protest often follows, and thereafter the open processes of justice serve an important prophylactic purpose, providing an outlet for community concern, hostility, and emotion. To work effectively, it is important that society's criminal process "satisfy the appearance of justice," Offutt v. United States, 348 US 11, 14, 99 L Ed 11, 75 S Ct 11, which can best be provided by allowing people to observe such process. From this unbroken, uncontradicted history, supported by reasons as valid today as in centuries past, it must be concluded that a presumption of openness inheres in the very nature of a criminal trial under this Nation's system of justice, Cf., e.g., Levine v. United States, 362 US 610, 4 L Ed 2d 989, 80 S Ct 1038. (b) The freedoms of speech, press, and assembly, expressly guaranteed by the First Amendment, share a common core purpose of assuring freedom of communication on matters relating to the functioning of government. In guaranteeing freedoms such as those of speech and press, the First Amendment can be read as protecting the right of everyone to attend trials so as to give meaning to those explicit guarantees; the First Amendment right to receive information and ideas means, in the context of trials, that the guarantees of speech and press, standing alone, prohibit government from summarily closing courtroom doors which had long been open to the public at the time the First Amendment was adopted. Moreover, the right of assembly is also relevant, having been regarded not only as an independent right but also as a catalyst to augment the free exercise of the other First Amendment rights with which it was deliberately linked by the draftsmen. A trial courtroom is a public place where the people generally and representatives of the media have a right to be present, and where their presence historically has been thought to enhance the integrity and quality of what takes place. (c) Even though the Constitution contains no provision which by its terms guarantees to the public the right to attend criminal trials, various fundamental rights, not expressly guaranteed, have been recognized as indispensable to the enjoyment of enumerated rights. The right to attend criminal trials is implicit in the guarantees of the First Amendment; without the freedom to attend such trials, which people have exercised for centuries, important aspects of freedom of speech and of the press could be eviscerated.

Be that as it may, we recognize that pervasive and prejudicial publicity under certain circumstances can deprive an accused of his due process right to fair trial. Thus, in Martelino, et al. vs. Alejandro, et al., 54 we held that to warrant a finding of prejudicial publicity there must be allegation and proof that the judges have been unduly influenced, not simply that they might be, by the barrage of publicity. In the case at bar, we find nothing in the records that will prove that the tone and content, of the publicity that attended the investigation of petitioners fatally infected the fairness and impartiality of the DOJ Panel. Petitioners cannot just rely on the subliminal effects of publicity on the sense of fairness of the DOJ

Panel, for these are basically unbeknown and beyond knowing. To be sure, the DOJ Panel is composed of an Assistant Chief State Prosecutor and Senior State Prosecutors. Their long experience in criminal investigation is a factor to consider in determining whether they can easily be blinded by the klieg lights of publicity. Indeed, their 26-page Resolution carries no indubitable indicia of bias for it does not appear that they considered any extra-record evidence except evidence properly adduced by the parties. The length of time the investigation was conducted despite its summary nature and the generosity with which they accommodated the discovery motions of petitioners speak well of their fairness. At no instance, we note, did petitioners seek the disqualification of any member of the DOJ Panel on the ground of bias resulting from their bombardment of prejudicial publicity. It all remains to state that the Vizconde case will move to a more critical stage as petitioners will now have to undergo trial on the merits. We stress that probable cause is not synonymous with guilt and while the light of publicity may be a good disinfectant of unfairness, too much of its heat can bring to flame an accused's right to fair trial. Without imposing on the trial judge the difficult task of supervising every specie of speech relating to the case at bar, it behooves her to be reminded of the duty of a trial judge in high profile criminal cases to control publicity prejudicial to the fair administration of justice. 55 The Court reminds judges that our ability to dispense impartial justice is an issue in every trial and in every criminal prosecution, the judiciary always stands as a silent accused. More than convicting the guilty and acquitting the innocent, the business of the judiciary is to assure fulfillment of the promise that justice shall be done and is done and that is the only way for the judiciary to get an acquittal from the bar of public opinion. IN VIEW WHEREOF, the petitions are dismissed for lack of showing of grave abuse of discretion on the part of the respondents. Costs against petitioners. SO ORDERED. ** Alvarez v CFI 64 PHIL 33 (1937)
Facts: On 3 June 1936, the chief of the secret service of the Anti-Usury Board, of the Department of Justice, presented to Judge Eduardo Gutierrez David then presiding over the Court of First Instance of Tayabas, an affidavit alleging that according to reliable information, Narciso Alvarez kept in his house in Infanta, Tayabas, books, documents, receipts, lists, chits and other papers used by him in connection with his activities as a moneylender, charging usurious rates of interest in violation of the law. In his oath at the end of the affidavit, the chief of the secret service stated that his answers to the questions were correct to the best of his knowledge and belief. He did not swear to the truth of his statements upon his own knowledge of the facts but upon the information received by him from a reliable person. Upon the affidavit the judge, on said date, issued the warrant which is the subject matter of the petition, ordering the search of the Alvarezs house at any time of the day or night, the seizure of the books and documents and the immediate delivery thereof to him to be disposed of in accordance with the law. With said warrant, several agents of the Anti-Usury Board entered Alvarezs store and residence at 7:00 p.m. of 4

June 1936, and seized and took possession of the following articles: internal revenue licenses for the years 1933 to 1936, 1 ledger, 2 journals, 2 cashbooks, 9 order books, 4 notebooks, 4 check stubs, 2 memorandums, 3 bankbooks, 2 contracts, 4 stubs, 48 stubs of purchases of copra, 2 inventories, 2 bundles of bills of lading, 1 bundle of credit receipts, 1 bundle of stubs of purchases of copra, 2 packages of correspondence, 1 receipt book belonging to Luis Fernandez, 14 bundles of invoices and other papers, many documents and loan contracts with security and promissory notes, 504 chits, promissory notes and stubs of used checks of the Hongkong & Shanghai Banking Corporation (HSBC). The search for and seizure of said articles were made with the opposition of Alvarez who stated his protest below the inventories on the ground that the agents seized even the originals of the documents. As the articles had not been brought immediately to the judge who issued the search warrant, Alvarez, through his attorney, filed a motion on 8 June 1936, praying that the agent Emilio L. Siongco, or any other agent, be ordered immediately to deposit all the seized articles in the office of the clerk of court and that said agent be declared guilty of contempt for having disobeyed the order of the court. On said date the court issued an order directing Siongco to deposit all the articles seized within 24 hours from the receipt of notice thereof and giving him a period of 5 days within which to show cause why he should not be punished for contempt of court. On 10 June, Attorney Arsenio Rodriguez, representing the Anti-Usury Board, filed a motion praying that the order of the 8th of said month be set aside and that the Anti-Usury Board be authorized to retain the articles seized for a period of 30 days for the necessary investigation. On June 25, the court issued an order requiring agent Siongco forthwith to file the search warrant and the affidavit in the court, together with the proceedings taken by him, and to present an inventory duly verified by oath of all the articles seized. On July 2, the attorney for the petitioner filed a petition alleging that the search warrant issued was illegal and that it had not yet been returned to date together with the proceedings taken in connection therewith, and praying that said warrant be cancelled, that an order be issued directing the return of all the articles seized to Alvarez, that the agent who seized them be declared guilty of contempt of court, and that charges be filed against him for abuse of authority. On September 10, the court issued an order holding: that the search warrant was obtained and issued in accordance with the law, that it had been duly complied with and, consequently, should not be cancelled, and that agent Siongco did not commit any contempt of court and must, therefore, be exonerated, and ordering the chief of the AntiUsury Board in Manila to show cause, if any, within the unextendible period of 2 days from the date of notice of said order, why all the articles seized appearing in the inventory should not be returned to Alvarez. The assistant chief of the Anti-Usury Board of the Department of Justice filed a motion praying, for the reasons stated therein, that the articles seized be ordered retained for the purpose of conducting an investigation of the violation of the Anti-Usury Law committed by Alvarez. On October 10, said official again filed another motion alleging that he needed 60 days to examine the documents and papers seized, which are designated on pages 1 to 4 of the inventory by Nos. 5, 10, 16, 23, 25-27, 30-31 , 34, 36-43 and 45, and praying that he be granted said period of 60 days. In an order of October 16, the court granted him the period of 60 days to investigate said 19 documents. Alvarez, herein, asks that the search warrant as well as the order authorizing the agents of the Anti-Usury Board to retain the articles seized, be declared illegal and set aside, and prays that all the articles in question be returned to him. Issue: Whether the search warrant issued by the court is illegal because it has been based upon the affidavit of agent Almeda in whose oath he declared that he had no personal knowledge of the facts which were to serve as a basis for the issuance of the warrant but that he had knowledge thereof through mere information secured from a person whom he considered reliable, and that it is illegal as it was not supported by other affidavits aside from that made by the applicant. Held: Section 1, paragraph 3, of Article III of the Constitution and Section 97 of General Orders 58 require that there be not only probable cause before the issuance of a search warrant but that the search warrant must be based upon an application supported by oath of the applicant and the witnesses he may produce. In its broadest sense, an oath includes any form of attestation by which a party signifies that he is bound in conscience to perform an act faithfully and truthfully; and it is sometimes defined as an outward pledge given by the person taking it that his attestation or promise is made under an immediate sense of his responsibility to God. The oath required must refer to the truth of the facts within the personal

knowledge of the petitioner or his witnesses, because the purpose thereof is to convince the committing magistrate, not the individual making the affidavit and seeking the issuance of the warrant, of the existence of probable cause. The true test of sufficiency of an affidavit to warrant issuance of a search warrant is whether it has been drawn in such a manner that perjury could be charged thereon and affiant be held liable for damages caused. The affidavit, which served as the exclusive basis of the search warrant, is insufficient and fatally defective by reason of the manner in which the oath was made, and therefore, the search warrant and the subsequent seizure of the books, documents and other papers are illegal. Further, it is the practice in this jurisdiction to attach the affidavit of at least the applicant or complainant to the application. It is admitted that the judge who issued the search warrant in this case, relied exclusively upon the affidavit made by agent Almeda and that he did not require nor take the deposition of any other witness. Neither the Constitution nor General Orders 58 provides that it is of imperative necessity to take the depositions of the witnesses to be presented by the applicant or complainant in addition to the affidavit of the latter. The purpose of both in requiring the presentation of depositions is nothing more than to satisfy the committing magistrate of the existence of probable cause. Therefore, if the affidavit of the applicant or complainant is sufficient, the judge may dispense with that of other witnesses. Inasmuch as the affidavit of the agent was insufficient because his knowledge of the facts was not personal but merely hearsay, it is the duty of the judge to require the affidavit of one or more witnesses for the purpose of determining the existence of probable cause to warrant the issuance of the search warrant. When the affidavit of the applicant or complainant contains sufficient facts within his personal and direct knowledge, it is sufficient if the judge is satisfied that there exists probable cause; when the applicants knowledge of the facts is mere hearsay, the affidavit of one or more witnesses having a personal knowledge of the facts is necessary. Thus the warrant issued is likewise illegal because it was based only on the affidavit of the agent who had no personal knowledge of the facts.

January 29, 1937 G.R. No. L-45358 NARCISO ALVAREZ, petitioner, vs. THE COURT OF FIRST INSTANCE OF TAYABAS and THE ANTI-USURY BOARD, respondents. Godofredo Reyes for petitioner. Adolfo N. Feliciano for respondents Anti-Usury Board. Imperial, J.: The petitioner asks that the warrant of June 3, 1936, issued by the Court of First Instance of Tayabas, ordering the search of his house and the seizure, at any time of the day or night, of certain accounting books, documents and papers belonging to him in his residence situated in Infanta, Province of Tayabas, as well as the order of a later date, authorizing the agents of the Anti-Usury Board to retain the articles seized, be declared illegal and set aside, and prays that all the articles in question be returned to him. On the date above-mentioned, the chief of the secret service of the Anti-Usury Board, of the Department of Justice, presented to Judge Eduardo Gutierrez David then presiding over the

Court of First Instance of Tayabas, an affidavit alleging that according to reliable information, the petitioner kept in his house in Infanta, Tayabas, books, documents, receipts, lists, chits and other papers used by him in connection with his activities as a money-lender charging usurious rates of interest in violation of the law. In his oath at the and of the affidavit, the chief of the secret service stated that his answers to the questions were correct to the best of his knowledge and belief. He did not swear to the truth of his statements upon his own knowledge of the facts but upon the information received by him from a reliable person. Upon the affidavit in question the Judge, on said date, issued the warrant which is the subject matter of the petition, ordering the search of the petitioners house at nay time of the day or night, the seizure of the books and documents above-mentioned and the immediate delivery thereof to him to be disposed of in accordance with the law. With said warrant, several agents of the Anti-Usury Board entered the petitioners store and residence at seven oclock on the night of June 4, 1936, and seized and took possession of the following articles: internal revenue licenses for the years 1933 to 1936, one ledger, two journals, two cashbooks, nine order books, four notebooks, four checks stubs, two memorandums, three bankbooks, two contracts, four stubs, forty-eight stubs of purchases of copra, two inventories, two bundles of bills of lading, one bundle of credit receipts, one bundle of stubs of purchases of copra, two packages of correspondence, one receipt book belonging to Luis Fernandez, fourteen bundles of invoices and other papers many documents and loan contracts with security and promissory notes, 504 chits, promissory notes and stubs of used checks of the Hongkong & Shanghai Banking Corporation. The search for and a seizure of said articles were made with the opposition of the petitioner who stated his protest below the inventories on the ground that the agents seized even the originals of the documents. As the articles had not been brought immediately to the judge who issued the search warrant, the petitioner, through his attorney, filed a motion on June 8, 1936, praying that the agent Emilio L. Siongco, or any other agent, be ordered immediately to deposit all the seized articles in the office of the clerk of court and that said agent be declared guilty of contempt for having disobeyed the order of the court. On said date the court issued an order directing Emilio L. Siongco to deposit all the articles seized within twenty-four hours from the receipt of notice thereof and giving him a period of five (5) days within which to show cause why he should not be punished for contempt of court. On June 10th, Attorney Arsenio Rodriguez, representing the Anti-Usury Board, filed a motion praying that the order of the 8th of said month be set aside and that the Anti-Usury Board be authorized to retain the articles seized for a period of thirty (30) days for the necessary investigation. The attorney for the petitioner, on June 20th, filed another motion alleging that, notwithstanding the order of the 8th of said month, the officials of the Anti-Usury Board had failed to deposit the articles seized by them and praying that a search warrant be issued, that the sheriff be ordered to take all the articles into his custody and deposit of the AntiUsury Board be punished for contempt of court. Said attorney, on June 24th, filed an ex parte petition alleging that while agent Emilio L. Siongco had deposited some documents and papers in the office of the clerk of court, he had so far failed to file an inventory duly verified by oath of all the documents seized by him, to return the search warrant together with the affidavit it presented in support thereof, or to present the report of the proceedings taken by him; and prayed that said agent be directed to filed the documents in question immediately. On the 25th of said month the court issued an order requiring agent Emilio L. Siongco forthwith to file the search warrant and the affidavit in the court, together with the proceedings taken by him, and to present an inventory duly verified by oath of all the articles seized. On July 2d of said year, the attorney for the petitioner filed another petition alleging that the search warrant issue was illegal and that

it had nit yet been returned to date together with the proceedings taken in connection therewith, and praying that said warrant be cancelled, that an order be issued directing the return of all the articles seized to the petitioner, that the agent who seized them be declared guilty of contempt of court, and that charges be filed against him for abuse of authority. On September 10, 1936, the court issued an order holding: that the search warrant was obtained and issued in accordance with the law, that it had been duly complied with and, consequently, should not be cancelled, and that agent Emilio L. Siongco did not commit any contempt of court and must, therefore, be exonerated, and ordering the chief of the Anti-Usury Board in Manila to show case, if any, within the unextendible period of two (2) days from the date of notice of said order, why all the articles seized appearing in the inventory, Exhibit 1, should not be returned to the petitioner. The assistant chief of the Anti-Usury Board of the Department of Justice filed a motion praying, for the reasons stated therein, that the articles seized be ordered retained for the purpose of conducting an investigation of the violation of the Anti-Usury Law committed by the petitioner. In view of the opposition of the attorney for the petitioner, the court, on September 25th, issued an order requiring the Anti-Usury Board to specify the time needed by it to examine the documents and papers seized and which of them should be retained, granting it a period of five (5) days for said purpose. On the 30th of said month the assistant chief of the Anti-Usury Board filed a motion praying that he be granted ten (10) days to comply with the order of September 25th and that the clerk of court be ordered to return to him all the documents and papers together with the inventory thereof. The court, in an order of October 2d of said year, granted him the additional period of ten(10) days and ordered the clerk of court to send him a copy of the inventory. On October 10th, said official again filed another motion alleging that he needed sixty (60) days to examine the documents and papers seized, which are designated on pages 1 to 4 of the inventory by Nos. 5, 1016, 23, 25, 26, 27, 30, 31, 34, 36, 37, 38, 39, 40, 41, 42, 43 and 45, and praying that he be granted said period of sixty (60) days. In an order of October 16th, the court granted him the period of sixty (60) days to investigate said nineteen (19) documents. The petitioner alleges, and it is not denied by the respondents, that these nineteen (19)documents continue in the possession of the court, the rest having been returned to said petitioner. I. A search warrant is an order in writing, issued in the name of the People of the Philippine Islands, signed by a judge or a justice of the peace, and directed to a peace officer, commanding him to search for personal property and bring it before the court (section 95, General Orders. No. 58, as amended by section 6 of Act No. 2886). Of all the rights of a citizen, few are of greater importance or more essential to his peace and happiness than the right of personal security, and that involves the exemption of his private affairs, books, and papers from the inspection and scrutiny of others (In re Pacific Railways Commission, 32 Fed., 241; Interstate Commerce Commission vs Brimson, 38 Law. ed., 1047; Broyd vs. U. S., 29 Law. ed., 746; Caroll vs. U. S., 69 Law. ed., 543, 549). While the power to search and seize is necessary to the public welfare, still it must be exercised and the law enforced without transgressing the constitutional rights or citizen, for the enforcement of no statue is of sufficient importance to justify indifference to the basis principles of government (People vs. Elias, 147 N. E., 472). II. As the protection of the citizen and the maintenance of his constitutional right is one of the highest duties and privileges of the court, these constitutional guaranties should be given a liberal construction or a strict construction in favor of the individual, to prevent stealthy encroachment upon, or gradual depreciation on, the rights secured by them(State vs. Custer County, 198 Pac.,

362; State vs. McDaniel, 231 Pac., 965; 237 Pac., 373). Since the proceeding is a drastic one, it is the general rule that statutes authorizing searches and seizure or search warrants must be strictly construed (Rose vs. St. Clair, 28 Fed., [2d], 189; Leonard vs. U. S., 6 Fed. [2d], 353; Perry vs. U. S. 14 Fed. [2d],88; Cofer vs. State, 118 So., 613). III. The petitioner claims that the search warrant issued by the court is illegal because it has been based upon the affidavit of agent Mariano G. Almeda in whose oath he declared that he had no personal knowledge of the facts which were to serve as a basis for the issuance of the warrant but that he had knowledge thereof through mere information secured from a person whom he considered reliable. To the question What are your reason for applying for this search warrant, appearing in the affidavit, the agent answered: It has been reported to me by a person whom I consider to be reliable that there are being kept in said premises, books, documents, receipts, lists, chits, and other papers used by him in connection with his activities as a money-lender, charging a usurious rate of interest, in violation of the law and in attesting the truth of his statements contained in the affidavit, the said agent states that he found them to be correct and true to the best of his knowledge and belief. Section 1, paragraph 3, of Article III of the Constitution, relative to the bill of rights, provides that The right of the people to be secure in their persons, houses, papers, and effects against unreasonable searches and seizures shall not be violated, and no warrants shall issue but upon probable cause, to be determined by the judge after examination under oath or affirmation of the complainant and the witnesses he may produce, and particularly describing the place top be searched, and the persons or things to be seized. Section 97 of General Orders, No. 58 provides that A search warrant shall not issue except for probable cause and upon application supported by oath particularly describing the place to be searched and the person or thing to be seized. It will be noted that both provisions require that there be not only probable cause before the issuance of a search warrant but that the search warrant must be based upon an application supported by oath of the applicant ands the witnesses he may produce. In its broadest sense, an oath includes any form of attestation by which a party signifies that he is bound in conscience to perform an act faithfully and truthfully; and it is sometimes defined asan outward pledge given by the person taking it that his attestation or promise is made under an immediate sense of his responsibility to God (Bouviers Law Dictionary; State vs. Jackson, 137 N. W., 1034; In re Sage, 24 Oh. Cir. Ct. [N. S.], 7; Pumphery vs. State, 122 N. W., 19; Priest vs. State, 6 N. W., 468; State vs. Jones, 154 Pac., 378; Atwood vs. State, 111 So., 865). The oath required must refer to the truth of the facts within the personal knowledge of the petitioner or his witnesses, because the purpose thereof is to convince the committing magistrate, not the individual making the affidavit and seeking the issuance of the warrant, of the existence of probable cause (U. S. vs. Tureaud, 20 Fed., 621; U. S. vs. Michalski, 265 Fed., 8349; U. S. vs. Pitotto, 267 Fed., 603; U. S. vs. Lai Chew, 298 Fed., 652). The true test of sufficiency of an affidavit to warrant issuance of a search warrant is whether it has been drawn in such a manner that perjury could be charged thereon and affiant be held liable for damages caused (State vs. Roosevelt Country 20th Jud. Dis. Ct., 244 Pac., 280; State vs. Quartier, 236 Pac., 746). It will likewise be noted that section 1, paragraph 3, of Article III of the Constitution prohibits unreasonable searches and seizure. Unreasonable searches and seizures are a menace against which the constitutional guarantee afford full protection. The term unreasonable search and

seizure is not defined in the Constitution or in General Orders No. 58, and it is said to have no fixed, absolute or unchangeable meaning, although the term has been defined in general language. All illegal searches and seizure are unreasonable while lawful ones are reasonable. What constitutes a reasonable or unreasonable search or seizure in any particular case is purely a judicial question, determinable from a consideration of the circumstances involved, including the purpose of the search, the presence or absence or probable cause, the manner in which the search and seizure was made, the place or thing searched, and the character of the articles procured (GoBart Importing Co. vs. U. S. 75 Law. ed., 374; Peru vs. U. S., 4 Fed., [2d], 881;U. S. vs. Vatune, 292 Fed., 497; Angelo vs. U. S. 70 Law, ed., 145; Lambert vs. U. S. 282 Fed., 413; U. S. vs. Bateman, 278 Fed., 231; Mason vs. Rollins, 16 Fed. Cas. [No. 9252], 2 Biss., 99). In view of the foregoing and under the above-cited authorities, it appears that the affidavit, which served as the exclusive basis of the search warrant, is insufficient and fatally defective by reason of the manner in which the oath was made, and therefore, it is hereby held that the search warrant in question and the subsequent seizure of the books, documents and other papers are illegal and do not in any way warrant the deprivation to which the petitioner was subjected. IV. Another ground alleged by the petitioner in asking that the search warrant be declared illegal and cancelled is that it was not supported by other affidavits aside from that made by the applicant. In other words, it is contended that the search warrant cannot be issued unless it be supported by affidavits made by the applicant and the witnesses to be presented necessity by him. Section 1, paragraph 3, of Article III of the Constitution provides that no warrants shall issue but upon probable cause, to be determined by the judge after examination under oath or affirmation of the complainant and the witnesses he may produce. Section 98 of General Orders, No. 58 provides that the judge or justice must, before issuing the warrant, examine under oath the complainant and any witnesses he may produce and take their depositions in writing. It is the practice in this jurisdiction to attach the affidavit of at least the applicant or complainant to the application. It is admitted that the judge who issued the search warrant in this case, relied exclusively upon the affidavit made by agent Mariano G. Almeda and that he did not require nor take the deposition of any other witness. Neither the Constitution nor General Orders. No. 58 provides that it is of imperative necessity to take the deposition of the witnesses to be presented by the applicant or complainant in addition to the affidavit of the latter. The purpose of both in requiring the presentation of depositions is nothing more than to satisfy the committing magistrate of the existence of probable cause. Therefore, if the affidavit of the applicant or complainant is sufficient, the judge may dispense with that of other witnesses. Inasmuch as the affidavit of the agent in this case was insufficient because his knowledge of the facts was not personal but merely hearsay, it is the duty of the judge to require the affidavit of one or more witnesses for the purpose of determining the existence of probable cause to warrant the issuance of the search warrant. When the affidavit of the applicant of the complaint contains sufficient facts within his personal and direct knowledge, it is sufficient if the judge is satisfied that there exist probable cause; when the applicants knowledge of the facts is mere hearsay, the affidavit of one or more witnesses having a personal knowledge of the fact is necessary. We conclude, therefore, that the warrant issued is likewise illegal because it was based only on the affidavit of the agent who had no personal knowledge of the facts.

V. The petitioner alleged as another ground for the declaration of the illegality of the search warrant and the cancellation thereof, the fact that it authorized its execution at night. Section 101 of General Orders, No. 58 authorizes that the search be made at night when it is positively asserted in the affidavits that the property is on the person or in the place ordered to be searched. As we have declared the affidavits insufficient and the warrant issued exclusively upon it illegal, our conclusion is that the contention is equally well founded and that the search could not legally be made at night. VI. One of the grounds alleged by the petitioner in support of his contention that the warrant was issued illegally is the lack of an adequate description of the books and documents to be seized. Section 1, paragraphs 3, of Article III of the Constitution, and section 97 of General Orders, No. 58 provide that the affidavit to be presented, which shall serve as the basis for determining whether probable cause exist and whether the warrant should be issued, must contain a particular description of the place to be searched and the person or thing to be seized. These provisions are mandatory and must be strictly complied with (Munch vs. U. S., 24 Fed. [2d], 518; U. S. vs. Boyd, 1 Fed. [2d], 1019; U. S. vs. Carlson, 292 Fed., 463; U. S. vs. Borkowski, 268 Fed., 408; In re Tri-State Coal & Coke Co., 253 Fed., 605; People vs. Mayen, 188 Cal., 237; People vs. Kahn, 256 Ill. App., 4125); but where, by the nature of the goods to be seized, their description must be rather generally, it is not required that a technical description be given, as this would mean that no warrant could issue (People vs. Rubio, 57 Phil. 284; People vs. Kahn, supra). The only description of the articles given in the affidavit presented to the judge was as follows: that there are being kept in said premises books, documents, receipts, lists, chits and other papers used by him in connection with his activities as money-lender, charging a usurious rate of interest, in violation of the law. Taking into consideration the nature of the article so described, it is clear that no other more adequate and detailed description could have been given, particularly because it is difficult to give a particular description of the contents thereof. The description so made substantially complies with the legal provisions because the officer of the law who executed the warrant was thereby placed in a position enabling him to identify the articles, which he did. VII. The last ground alleged by the petitioner, in support of his claim that the search warrant was obtained illegally, is that the articles were seized in order that the Anti-Usury Board might provide itself with evidence to be used by it in the criminal case or cases which might be filed against him for violation of the Anti-usury Law. At the hearing of the incidents of the case raised before the court it clearly appeared that the books and documents had really been seized to enable the Anti-Usury Board to conduct an investigation and later use all or some of the articles in question as evidence against the petitioner in the criminal cases that may be filed against him. The seizure of books and documents by means of a search warrant, for the purpose of using them as evidence in a criminal case against the person in whose possession they were found, is unconstitutional because it makes the warrant unreasonable, and it is equivalent to a violation of the constitutional provision prohibiting the compulsion of an accused to testify against himself (Uy Kheytin vs. Villareal, 42 Phil,, 886; Brady vs. U. S., 266 U. S., 620; Temperani vs. U. S., 299 Fed., 365; U. S. vs. Madden, 297 Fed., 679; Boyd vs. U. S.,116 U. S., 116; Caroll vs. U. S., 267 U. S., 132). Therefore, it appearing that at least nineteen of the documents in question were seized for the purpose of using them as evidence against the petitioner in the criminal proceeding or proceedings for violation against him, we hold that the search warrant issued is illegal and that the documents should be returned to him.

The Anti-Usury Board insinuates in its answer that the petitioner cannot now question the validity of the search warrant or the proceedings had subsequent to the issuance thereof, because he has waived his constitutional rights in proposing a compromise whereby he agreed to pay a fine of P200 for the purpose of evading the criminal proceeding or proceedings. We are of the opinion that there was no such waiver, first, because the petitioner has emphatically denied the offer of compromise and, second, because if there was a compromise it reffered but to the institution of criminal proceedings fro violation of the Anti-Usury Law. The waiver would have been a good defense for the respondents had the petitioner voluntarily consented to the search and seizure of the articles in question, but such was not the case because the petitioner protested from the beginning and stated his protest in writing in the insufficient inventory furnished him by the agents. Said board alleges as another defense that the remedy sought by the petitioner does not lie because he can appeal from the orders which prejudiced him and are the subject matter of his petition. Section 222 of the Code of Civil Procedure in fact provides that mandamus will not issue when there is another plain, speedy and adequate remedy in the ordinary course of law. We are of the opinion, however, that an appeal from said orders would have to lapse before he recovers possession of the documents and before the rights, of which he has been unlawfully deprived, are restored to him (Fajardo vs. Llorente, 6 Phil. 426; Manotoc vs. McMicking and Trinidad, 10 Phil. 119; Cruz Herrera de Lukban vs. McMicking, 14 Phil. 641; Lamb vs. Phipps, 22 Phil. 456). Summarizing the foregoing conclusions, we hold: 1. That the provisions of the Constitution and General Orders, No. 58, relative to search and seizure, should be given a liberal construction in favor of the individual in order to maintain the constitutional guaranties whole and in their full force; 2. That since the provisions in question are drastic in their form and fundamentally restrict the enjoyment of the ownership, possession and use of the personal property of the individual, they should be strictly construed; 3. That the search and seizure made are illegal for the following reasons: (a) Because the warrant was based solely upon the affidavit of the petitioner who had no personal knowledge of the facts of probable cause, and (b) because the warrant was issued for the sole purpose of seizing evidence which would later be used in the criminal proceedings that might be instituted against the petitioner, for violation of the Anti-Usury Law; 4. That as the warrant had been issued unreasonably, and as it does not appear positively in the affidavit that the articles were in the possession of the petitioner and in the place indicated, neither could the search and seizure be made at night; 5. That although it is not mandatory to present affidavits of witnesses to corroborate the applicant or a complainant in cases where the latter has personal knowledge of the facts, when the applicants or complainants knowledge of the facts is merely hearsay, it is the duty of the judge to require affidavits of other witnesses so that he may determine whether probable cause exists;

6. That a detailed description of the person and place to be searched and the articles to be seized is necessary, but whereby, by the nature of the articles to be seized, their description must be rather general, but is not required that a technical description be given, as this would mean that no warrant could issue; 7. That the petitioner did not waive his constitutional rights because the offer of compromise or settlement attributed to him, does not mean, if so made, that he voluntarily tolerated the search and seizure; and 8. That an appeal from the orders questioned by the petitioner, if taken by him, would not be an effective, speedy or adequate remedy in the ordinary course of law, and, consequently, the petition for mandamus filed by him, lies. For the foregoing considerations, the search warrant and the seizure of June 3, 1936, and the orders of the respondent court authorizing the relation of the books and documents, are declared illegal and are set aside, and it is ordered that the judge presiding over the Court of First Instance of Tayabas direct the immediate return to the petitioner of the nineteen (19) documents designated on pages 1 to 4 of the inventory by Nos. 5, 10, 16, 23, 25,26, 27, 30, 31, 34, 36, 37, 38, 39, 40, 41, 42, 43 and 45, without special pronouncement as to costs. So ordered.

G.R. No. L-27511

November 29, 1968

IN THE MATTER OF THE APPLICATION FOR A WRIT OF HABEAS CORPUS, SIMON LUNA, petitioner-appellant, vs. HON. LORENZO M. PLAZA, as Judge of the Municipal Court of Tandag, Surigao del Sur; HON. SANTOS B. BEBERINO as Provincial Fiscal of Surigao del Sur; and THE PROVINCIAL WARDEN of Surigao del Sur, respondents- appellees. Sisenando Villaluz and Juan T. David for petitioner-appellant. Office of the Assistant Solicitor General Pacifico P. de Castro and Solicitor Augusto M. Amores for other respondents-appellees. Provincial Fiscal Santos B. Beberno in his own behalf as respondent-appellee. ZALDIVAR, J.: Appeal from the decision of the Court of First Instance of Surigao del Sur, dated April 20, 1967, dismissing the petition for a writ of habeas corpus, filed by herein petitioner-appellant Simon Luna hereinafter referred to simply as petitioner who was charged with murder in Criminal Case No. 655-New of the same court. The criminal action was commenced by T-Sgt. Candido Patosa, PC investigator of Tandag, Surigao del Sur, by filing with respondent Municipal Judge Lorenzo M. Plaza, of the Municipal Court of Tandag, criminal case No. 1138 charging the accused, herein petitioner, with the crime

of murder. Supporting the complaint were sworn statements of the witnesses for the prosecution, in the form of questions and answers taken by T-Sgt. Patosa, and subscribed and sworn to before the respondent Judge at the time of the filing of the complaint. The respondent Judge examined the prosecution witnesses by reading to them "all over again the questions and answers" in their statements in writing, and the witnesses-affiants declared before said Judge that the questions were propounded by T-Sgt. Candido Patosa, and that the answers were made by them. The affiants further declared before respondent Judge that their answers were true, and were freely and voluntarily made; that they fully understood the questions and answers, and that they were willing to sign their respective affidavits. The affiants signed their respective affidavits in the presence of the respondent Judge, who also signed after the usual procedure of administering the oath. Considering the answers of the affiants to the questions contained in their sworn statements, together with the post-mortem and autopsy report on the dead body of the victim Jaime Diaz Ng, the certificate of death, the sketch showing the position of the victim and the accused, and Exhibits 6, 7, 8, 12, and 13 of herein respondents, the respondent Judge opined that there was reasonable ground to believe that the crime of murder had been committed and the accused was probably guilty thereof. Respondent Judge issued the order and warrant of arrest, specifying therein that no bail should be accepted for the provisional release of the accused. On February 20, 1967, upon motion of petitioner that he be admitted to bail upon the ground that the evidence of guilt was not strong, respondent Judge issued an order granting bail, fixing it at P30,000.00; which order, however, respondent Judge later revoked, and petitioner was denied bail. The case was subsequently remanded to the Court of First Instance of Surigao del Sur, after petitioner filed a waiver of his right to preliminary investigation. On March 9, 1967 respondent Provincial Fiscal filed an information charging herein petitioner with the crime of murder. The petitioner was detained in the provincial jail of Surigao del Sur under the custody of respondent Provincial Warden. On April 5, 1967, petitioner filed a petition for a writ of habeas corpus with the Court of First Instance of Surigao del Sur, therein docketed as Special Proceedings No. 105-New, claiming that he was being deprived of liberty without the due process of law, on the ground that the imprisonment and detention was the result of a warrant of arrest issued by respondent Judge in violation of Republic Act No. 3828, and praying for the annulment of the order for his arrest and his discharge from confinement. Herein respondents filed their answer, alleging that Republic Act No. 3828 had been substantially complied with; that a motion to quash, and not a petition for habeas corpus was the proper remedy; and that petitioner's application for bail constituted a waiver of the right to question the validity of the arrest. After trial, the Court of First Instance of Surigao del Sur rendered its decision, dated April 20, 1967, holding that respondent Municipal Judge had substantially complied with Republic Act No. 3828, and consequently denied the application for the writ of habeas corpus, and dismissed the case. Hence this appeal.

Petitioner, in his assignment of errors, claims that the trial court erred, as follows: 1. In giving absolute credence to the oral testimony of the respondent Judge to the effect that he adopted and made his own the questions and answers taken by T-Sgt. Patosa, PC Investigator, one of the prosecution witnesses, because the records show the contrary; 2. In denying the writ of habeas corpus and in dismissing the petition. 1. In support of his first assignment of error, petitioner contends that Republic Act No. 3828 imposes on a municipal judge, before he can issue a warrant of arrest, two specific duties, to wit: (1) personally examine the complainant and witnesses with "searching questions and answers", which means that the judge must cross-examine them in case their affidavits are presented; and (2) said examination must be reduced to writing and form part of the records of the case. The record of the instant case, according to petitioner, does not show said examination was performed by respondent Judge. Petitioner urges that the absence of any document in the record that shows that respondent Judge had performed the examination is positive proof that respondent Judge did not perform his duty, notwithstanding his testimony before the Court of First Instance of Surigao del Sur, during the hearing of this case, to the effect that he adopted the questions propounded to each of the prosecution witnesses by T-Sgt. Patosa. Petitioner maintains that this testimony, being self-serving intended to cover up the failure to comply with the law, should not have been believed by the Court of First Instance, and said court thereby committed errors when, believing said testimony, it found that there had been substantial compliance with the requirement that the municipal judge should personally examine the witnesses. Petitioner further maintains that assuming that the adoption of the questions made by T-Sgt. Patosa constituted substantial compliance with the requirement that the judge should examine the witnesses by asking searching questions, still the second requirement, that of reducing to writing the said procedure of adoption, has not been complied with; and so, Republic Act No. 3828 was still violated, and the issuance of the warrant of arrest was in violation of said Act and the Constitution and constituted denial of due process. Petitioner contends that the trial court erred in giving absolute credence to the testimony of respondent Municipal Judge. Regarding credibility of witnesses, this Court has consistently held that, as a general rule, the lower court's findings as to the credibility of witnesses will not be interfered with by appellate courts. Thus, in the case of People vs. Sinaon1 this Court said: Time and again, we have held that as a rule where the issue is one of credibility of witnesses, appellate courts will not generally disturb the findings of the trial court, considering that it is in a better position to decide the question, having seen and heard the witnesses themselves and observed their deportment and manner of testifying during the trial, unless there is a showing that it has overlooked certain facts of substance and value, that if considered, might affect the result of the case. Petitioner has appealed "from the decision/order" of the trial court "to the Honorable Supreme Court of the Philippines, on the ground that the same is contrary to law and the Philippine Constitution" and prayed that "all the records of the proceeding and the evidence, oral and documentary, be transmitted or forwarded to the Honorable Supreme Court ...".2 Since petitioner

appealed directly to this Court he must, therefore, raise only questions of law and he has thereby waived the right to raise any question of fact,3 and the findings of facts of the trial court, under the rules and precedents, must be deemed final and binding upon this Court.4 The findings of facts of the trial court are found in the following portion of the decision appealed from, to wit: There is no dispute that there is a valid complaint charging the accused Simon Luna, the herein petitioner with the crime of Murder filed with the respondent Judge authorized to conduct the examination of the witnesses for the prosecution for the purpose of determining the existence of probable cause before the issuance of the corresponding warrant of arrest; that the complaint is supported by the statements of the witnesses under oath in writing in the form of questions and answers and other documents attached to the complaint; that before the issuance of the corresponding warrant of arrest, the respondent judge personally examined the witnesses for the prosecution on their statements taken by T-Sgt. Candido Patosa by reading the questions and answers all over again to the affiants who confirmed to the respondent Judge that the statements contained in their sworn statements are true; that being satisfied that the questions and answers contained in the sworn statements taken by T-Sgt Patosa partake of the nature of his searching questions and answers as required by law, the respondent Judge adopted them as his own personal examination of the witnesses for the purpose of determining the existence of probable cause, the order and the warrant of arrest were issued to take the accused into custody for the commission of the offense charged (Exhibits "H", "H-1", "I", and "I-1"-petitioner); and that the petitioner waived his right to the preliminary investigation (Exhibit "12"respondent) and applied to be admitted to bail. Petitioner, however, claims that the failure of respondent Judge to put in writing that he adopted the questions asked by T-Sgt. Patosa and his failure to ask "searching questions" violated Republic Act No. 3828. Republic Act No. 3828, approved June 22, 1963, inserted in section 87 (e) of the Judiciary Act of 1948 the following paragraph: No warrant of arrest shall be issued by any justice of the peace in any criminal case filed with him unless he first examines the witness or witnesses personally, and the examination shall be under oath and reduced to writing in the form of searching questions and answers. Before a municipal judge may issue a warrant of arrest, the following conditions must first be fulfilled: (1) he must examine the witnesses personally; (2) the examination must be under oath; (3) the examination must be reduced to writing in the form of searching questions and answers. Were these conditions fulfilled in the instant case? The first condition was fulfilled. The trial court found as a fact that "the respondent judge personally examined the witnesses for the prosecution ...;" that respondent judge adopted as his own personal examination the questions asked by T-Sgt. Patosa as appearing in the written

statements, which he read over again to the witnesses together with the answers given therein, asking the witnesses whether said answers were theirs, and whether the same answers were true, to which the witness answered in the affirmative. Republic Act No. 3828 does not prohibit the municipal Judge from adopting the questions asked by a previous investigator. It appears that the sworn statements5 of the witnesses state at the beginning that the sworn statement was "taken by T-Sgt. Candido L. Patosa", and does not state that it was taken by the respondent municipal Judge himself. This circumstance is explained by the fact that said written statements already taken by T-Sgt. Patosa were delivered to respondent Municipal Judge who adopted the questions therein in his examination, because he considered them searching questions. Respondent Judge presumably did not consider it necessary to change the introductory remarks in each of the written statements. But that he made the examination personally cannot be doubted; it is so stated in the order dated February 18, 1967, which recites: After examining the witness personally and under oath there is reasonable ground to believe that an offense for murder has been committed and that the accused, Simon Luna, is probably guilty thereof. (Exh. H) The ruling in Doce vs. Branch II of the Court of First Instance of Quezon, et al.,6 wherein this Court held that the warrant of arrest issued therein was irregularly issued is not applicable to the case at bar for the simple reason that the facts are different. This Court in that case said: There is merit in the assertion that the warrant of arrest was irregularly issued. Section 87 of the Judiciary Act as amended by Republic Act 3828 requires that the Municipal Judge issuing the same, personally examine under oath the witnesses, and by searching questions and answers which are to be reduced to writing. Here, instead of searching questions and answers, we have only the affidavits of respondent and her one witness. Moreover, said affidavits were sworn to before Judge Cabungcal, not before Judge Juntereal who issued the warrant of arrest. In the instant case, as stated above, the respondent Municipal Judge personally examined under oath the witnesses by asking questions, that were adopted from a previous investigation, and considered by him as sufficiently searching and which questions and the answers thereto were in writing and sworn to before him prior to his issuance of the order of arrest. The second condition required by Republic Act No. 3828 for the issuance of a warrant of arrest was also fulfilled. The trial court found that the complaint was "supported by statements of the witnesses under oath." The record also shows the following documents to have been subscribed and sworn to before respondent Judge, namely: Exhibit B, sworn statement of herein petitioner Simon Luna y Albay; Exhibit C, sworn statement of Eusebio Corpuz; Exhibit D, sworn statement of Bruno M. Zafra; Exhibit E, sworn statement of Martiliano J. Bautista; Exhibit F, sworn statement of Janedina Diaz y Bandoy. The third condition required by Republic Act No. 3828 was likewise fulfilled. The examination of the witnesses was written down, in the form of searching questions and answers. The term "searching questions and answers" means only, taking into consideration the purpose of the

preliminary examination which is to determine "whether there is a reasonable ground to believe that an offense has been committed and the accused is probably guilty thereof so that a warrant of arrest may be issued and the accused held for trial",7 such questions as have tendency to show the commission of a crime and the perpetrator thereof. What would be searching questions would depend on what is sought to be inquired into, such as: the nature of the offense, the date, time, and place of its commission, the possible motives for its commission; the subject, his age, education, status, financial and social circumstances, his attitude toward the investigation, social attitudes, opportunities to commit the offense; the victim, his age, status, family responsibilities, financial and social circumstances, characteristics, etc. The points that are the subject of inquiry may differ from case to case. The questions, therefore, must to a great degree depend upon the Judge making the investigation. At any rate, the court a quo found that respondent judge was "satisfied that the questions and answers contained in the sworn statements taken by T-Sgt. Patosa partake of the nature of his searching questions and answers as required by law," so the respondent Judge adopted them. Petitioner's further contention that the issuance of the warrant of arrest was a violation of the constitution and of procedural due process is likewise untenable. The Constitution, in Section 1(3), Article III, provides that no warrant shall issue but upon probable cause, to be determined by the judge after examination under oath or affirmation of the complainant and the witnesses he may produce. The constitutional requirement of examination of witnesses under oath was, as shown above, fulfilled. The existence of probable cause depended to a large degree upon the finding or opinion of the judge conducting the examination. Respondent judge found that there was probable cause, as stated in his order of arrest, that "after examining the witnesses personally and under oath there is a reasonable ground to believe that an offense of murder has been committed and that the accused, Simon Luna, is probably guilty thereof." Petitioner's last contention that the warrant of arrest issued was a violation of procedural due process because of the alleged defective preliminary examination has no leg to stand on, in view of what we have hereinbefore stated. Moreover, this Court has held that preliminary examination is not an essential part of due process of law.8 Preliminary examination may be conducted by the municipal judge, prior to the issuance of the warrant of arrest, either in the presence, or in the absence, of the accused. The record shows that herein petitioner waived the preliminary investigation before respondent Municipal Judge, and instead, he filed a petition for bail. The petition for bail was at first granted by respondent Judge, but later the order granting bail was revoked. This conduct of petitioner indicates that he had waived his objection to whatever defect, if any, in the preliminary examination conducted by respondent Judge prior to the issuance of the warrant of arrest. Indeed, petitioner has no substantial much less legal ground to complain that he was denied the due process of law. We find that the trial Judge committed no error when he held that, based upon the facts shown during the hearing of this case, respondent Municipal Judge had substantially complied with the requirements of the law specifically Republic Act 3828 before issuing the warrant of arrest in this case.

2. In the light of what has been said above, it appears clear that petitioner's second assignment of error, that the trial court erred in denying the writ of habeas corpus, is untenable. Moreover, Section 4 of Rule 102; of the Rules of Court provides in part, as follows: SEC. 4. When writ not allowed or discharge authorized. If it appears that the person alleged to be restrained of his liberty is in the custody of an officer under process issued by a court or judge ... and that the court or judge had jurisdiction to issue the process ... or make the order the writ, shall not be allowed.... All the conditions, in the afore-quoted Section 4, set forth to deny the writ, are present in the instant case. It is shown that petitioner is detained and is in the custody of the respondent Provincial Warden by virtue of the order of arrest dated February 18, 1967, and the order dated February 21, 1967, of respondent Judge, to confine petitioner in the provincial jail. It is not disputed by petitioner that respondent Judge had jurisdiction to issue the warrant of arrest and the order of commitment under the provisions of Section 47, Republic Act No. 409, as amended by Republic Act No. 1201, although petitioner did question the validity of the warrant of arrest for allegedly having been issued in violation of Republic Act No. 3828 which claim We have found to be untenable. Consequently, the trial Judge did not commit an error in denying the writ of habeas corpus prayed for. At any rate, we believe that, if at all, the remedy available to the petitioner herein, under the circumstances stated in this opinion, is not a petition for a writ of habeas corpus but a petition to quash the warrant of arrest or a petition for a reinvestigation of the case by the respondent Municipal Judge or by the Provincial Fiscal. We wish to stress, however, that what has been stated in this opinion is certainly not intended to sanction the return to the former practice of municipal judges of simply relying upon affidavits or sworn statements that are made to accompany the complaints that are filed before them, in determining whether there is a probable cause for the issuance of a warrant of arrest. That practice is precisely what is sought to be voided by the amendment of Section 87 (c) of Republic Act 296 (Judiciary Act of 1948) which requires that before a municipal judge issues a warrant of arrest he should first satisfy himself that there is a probable cause by examining the witnesses personally, and that the examination must be under oath and reduced to writing in the form of searching questions and answers. It is obvious that the purpose of this amendment is to prevent the issuance of a warrant of arrest against a person based simply upon affidavits of witnesses who made, and swore to, their statements before a person or persons other than the judge before whom the criminal complaint is filed. We wish to emphasize strict compliance by municipal or city judges of the provision of Section 87 (c) of the Judiciary Act of 1948, as amended by Republic Act 3828, in order to avoid malicious and/or unfounded criminal prosecution of persons.9 In the case now before Us, while it is true that the respondent Municipal Judge did not himself personally cause to be reduced to writing in the form of questions and answers the examination of witnesses presented before him by the person who filed the criminal complaint, We are satisfied that, as shown by the evidence, respondent Judge had personally examined the witnesses under oath and that the questions asked by the Judge and the answers of the witnesses

were reflected in writings which were actually subscribed and sworn to before him. Moreover, We are of the considered view that no substantial right of the petitioner had been violated because, as hereinbefore adverted to, petitioner waived his right to preliminary investigation after he was arrested, and he took the step of applying for bail before respondent Municipal Judge. These acts of the petitioner subsequent to his arrest, constitute an implied admission on his part that here was a probable cause for the issuance of the warrant of arrest against him. Those acts of the petitioner constitute a waiver of whatever irregularity, if any there was, that attended his arrest.10 WHEREFORE, the decision of the trial court dated April 20, 1967, appealed from, is affirmed. Costs against petitioner-appellant. It is so ordered. A.M. No. 71-MJ November 29, 1974 SOLEDAD AVILLAR DE MULATA, ANTONIO ESTEBAN, and BUENAVENTURA BAYO, complainants, vs. JUDGE ELIAS C. IRIZARI, respondent. RESOLUTION

ANTONIO, J.:p These are two administrative complaints filed by Soledad Avillar and Antonio Esteban (Administrative Case No. L-2) and Buenaventura Bayo (Administrative Case No. L-3), respectively, against respondent Municipal Judge Elias C. Irizari of Hinatuan, Surigao del Sur. Both cases advert to respondent's conduct of the preliminary examination in two criminal cases filed with his court. In Administrative Case No. L-2, respondent is charged with "grave abuse of discretion, favoritism and bias" for having conducted the preliminary examination in Criminal Case No. 1074 without giving the accused therein an opportunity to be heard because they were not granted the occasion to cross-examine the complainant and her witnesses before issuing the warrant for their arrest; that they were arrested and detained in jail for fifteen (15) days without knowing why they were arrested as neither of them were shown any warrant of arrest; and that they were "treated like criminals of high degree" because they were first required to post bail for P20,000.00, "although later on thru the intervention of a brother of theirs in the Iglesia ni Cristo," they were bailed out for P10,000.00. According to District Judge Otilio G. Abayan of the Court of First Instance, Branch II, Lianga, Surigao del Sur, who was designated to investigate these administrative cases, Criminal Case No. 1074, which involved a complaint for "Assault Upon a Person in Authority with Physical Injuries", was filed by the Chief of Police of Hinatuan on July 19,

1969 against Soledad Avillar and Antonio Esteban. The Investigator found that respondent municipal judge issued the order for the arrest of the accused "after conducting a preliminary examination consisting in the adoption of the affidavits of the prosecution witnesses and the propounding of additional questions to them." The requirement that the investigating judge must examine the witnesses personally, which examination shall be under oath and reduced to writing in the form of searching questions and answers, 1 is fulfilled where the municipal judge examined under oath the witnesses by asking questions that were adopted from a previous investigation and considered by him as sufficiently searching and, which questions and the answers thereto, were in writing and sworn to before him prior to his issuance of the order of arrest. 2 Under Section 5, Rule 112, of the Revised Rules of Court, the accused is not entitled as a matter of right to be present during the preliminary examination or to crossexamine the witnesses presented against him before his arrest. 3 In Criminal Case No. 1074, complainants not only failed to question in the aforesaid case the validity of the preliminary examination, but also expressly waived their right to the second stage of the preliminary investigation. This case was subsequently elevated to the Court of First Instance and docketed as Criminal Case No. L-125. On motion of the Provincial Fiscal, the case against Antonio Esteban was dismissed, while Soledad Avillar was found, after trial, guilty of the crime of Slight Physical Injuries. Contrary to the claim of the complainants that they were first required to post bail of P20,000.00 each, the amount of the bail fixed for their provisional liberty as per Order of respondent dated June 19, 1969 in Criminal Case No. 1074 (Exhibit "VII-A") is P5,000.00. We agree, therefore, with the recommendation of the Investigating Judge that on the basis of these facts, this charge should be dismissed. In Administrative Case No. L-3, respondent is charged with having arbitrarily dismiss a "Grave Coercion" case filed by Buenaventura Bayo, a member of the Iglesia ni Cristo, against Barrio Captain Monica Peas and fourteen (14) others. Complainant asserts that he was notified only two days before the preliminary examination set on July 22, 1969; that after he was asked a few questions, he was informed that the case was dismissed; that he was deprived of his right to be represented by his lawyer; and that the act of respondent in dismissing this case while ordering the arrest of the accused in Criminal Case No. 1074 (subject of Administrative Case No. L-2) shows his bias and prejudice against members of the Iglesia ni Cristo. It was shown, however, that although the complaint was filed on July 3, 1969, the preliminary examination was set for July 21, 1969, but it was postponed to the following day. Therefore, complainant had sufficient time and opportunity to consult his lawyers if he so desired. The record also disclose that the dismissal of the case was made after respondent had propounded searching questions to complainant Buenaventura Bayo (Exhibit "26") and his witnesses (Exhibits "19", "20", "21" and "22"). This dismissal was predicated upon respondent's finding that the accused therein had acted without malice or criminal intent. This conclusion is borne out by the admission of the complaining witness,

Buenaventura Bayo, a member of the Iglesia ni Cristo, during the preliminary examination to the effect that he did not bury his dead child at the Roman Catholic Cemetery of Barrio San Juan on the morning of June 14, 1969, because he was told by Monica Peas and Pelegrino Malinao that said cemetery is exclusively for the use of the members of said church. However, the same Monica Peas, with the assistance of Felix Sereno, chartered at their own personal expense a pump boat to bring the cadaver of his daughter to the Poblacion of Hinatuan for burial at the public cemetery on the afternoon of the same day. It was also shown that Atty. Fermin B. Quejada, counsel of the complaining witness, received a copy of this order of dismissal from respondent's court. Said counsel never bothered to file a motion for reconsideration or take any action for the reinstatement of the case. The purpose of the preliminary examination is to determine whether or not there is sufficient reason to issue a warrant of arrest. Section 6 of Rule 112 of the Revised Rules of Court requires that the warrant of arrest shall be issued only when the judge conducting the preliminary examination is satisfied that the offense has in fact been committed and that there is a reasonable ground to believe that the accused committed the same. This is in conformity with the constitutional requirement that "no warrants shall issue but upon probable cause, to be determined by the judge after examination under oath or affirmation of the complainant and the witnesses he may produce." 4 The question as to whether "probable cause" exists or not, must, therefore, depend upon the judgment and discretion of the judge issuing the warrant. We are not satisfied that in the circumstances attendant to the case, respondent, in dismissing the complaint for "grave coercion", has acted arbitrarily. We do note, however, that respondent, upon accepting the complaint, failed to enter the same in his general docket book, so that the case had no corresponding docket number, before conducting the preliminary examination. The rules specifically require that the municipal judge must keep a docket in which he shall enter all civil and criminal cases or proceedings commenced before him. 5 It is a public record available during office hours for examination by any person, upon his reasonable request, to ascertain the status of any given case pending therein. 6 Respondent is, therefore, admonished to keep a faithful record of all proceedings before him in the dockets required by law to be kept for that purpose. WHEREFORE, respondent is exonerated from the aforementioned charges, with the above admonition. G.R. Nos. L-25707&25753-25754 May 14, 1981 ANTONIO MARINAS, ANTONIO MONTANO and GREGORIO RUPISAN petitioners, vs. HON. ANDRES S. SIOCHI, Presiding Judge of the Municipal Court of Pasig, Rizal, VICTORIA LASIN VDA. DE ATIENZA and ROSARIO L. ATIENZA, respondents.

MELENCIO-HERRERA, J.:1wph1.t Before us is a Petition for certiorari with Preliminary injunction seeking to annul the proceedings held in Criminal Cases Nos. 12943 and 12945 for Theft, and Criminal Case No. 12944 for Grave Coercion, before the Municipal Court of Pasig, Rizal; to annul the warrants of arrest issued in the said cases; and to declare as unconstitutional and void Section 5, Rule l l2 of the Rules of Court in so far as it denies the accused the right of notice and opportunity to be heard in the preliminary examination. The present controversy arose out of the issuance by the Municipal Court of Pasig, Rizal, of a Writ of Execution in Civil Case No. 938 for Ejectment, entitled Jose C. Zulueta vs. Gregorio Atienza. On December 13, 1965, petitioner Antonio Marinas, Deputy Sheriff of Rizal, with his co-petitioners Antonio Montano and Gregorio Rupisan enforced said Writ of Execution by levying upon the personal properties and chattels of private respondents Victoria Lasin Vda. de Atienza and] Rosario L. Atienza, and taking out said properties from their (respondents') rented house at #23 General Malvar St., Antonio Village, Pasig, Rizal. Respondents were also ejected from said house. On the same date, respondent Victoria Lasin Vda, de Atienza reported to the police authorities of Pasig that her jewelry worth P590.00 had been taken by petitioners without issuing any receipt therefor, 1 and in connection therewith, she executed a written Statement which was sworn to before Special Counsel Lucila P. Alcoba. 2 On January 28, 1966, respondents re-entered the house they had been ejected from after securing a Court Order for that purpose. Respondent Rosario L. Atienza then discovered that several pieces of her jewelry and other personal items, with a total value of P1,018.00, were missing. She reported the loss to the authorities on February 2, 1966, and her Statement was taken. She subscribed and swore to the same before respondent Municipal Judge Andres S. Siochi. 3 On February 3, 1966, respondents, armed with a Court Order authorizing them to enter the premises of the said house, did so again to get their remaining unlevied properties. They claimed, however, that on the said date petitioners and their companions forcibly compelled them to deliver the unlevied personal properties found therein, hauled said articles into a truck and left. Private respondents reported the incident to the police authorities at Pasig. 4 Victoria Lasin executed a Statement 5 alleging that the personal properties forcibly taken from them by petitioners, amounting to P2,645.00, were not included in the levy. Her son, Tranquilino Atienza, also executed an Affidavit corroborating her declaration. 6 Both Statements were subscribed and sworn to before respondent Judge. On February 7, 1966, two separate charges for Theft, docketed as Criminal Cases Nos. 12943 and 12945, were filed against petitioners and Carlos Quintana before the Municipal Court of Pasig, Rizal, respondent Judge, presiding. 7 A Complaint for Grave Coercion Crime Case No. 12944) was also lodged against petitioners and three Does

on the same date. 8 The three Complaints were filed by Lt. Jose S. Lontoc, Chief of the Criminal Investigation Section of the Police Department of Pasig, Rizal, for and on behalf of the Chief of Police. These Complaints contained an annotation on the lower left hand corner reading: "APPROVED AFTER PRELIMINARY EXAMINATION: (SGD) Lucila P. Alcoba, Special Counsel." The Complaints in Criminal Cases Nos. 12943 and 12944 for Theft and Grave Coercion, respectively, were subscribed and sworn to by Lt. Jose S. Lontoc before respondent Judge. The Complaint in Criminal Case No. 12945 for Theft does not show the jurat on its face, but respondents state that it was also attested to by Lt. Lontoc before respondent Judge and that this appears on the dorsal side of the Complaint. On February 8, 1966, warrants for the arrest of petitioners were issued by respondent Judge in all three cases 9 after preliminary examination conducted by him in Criminal Cases Nos. 12943 and 12944, and by Special Counsel Lucila P. Alcoba in Criminal Case No. 12945. Petitioners took exception to the issuance of the warrants of arrest against them and instituted the present Petition raising the following issues: 1wph1.t
1. When Section 87, Republic Act No. 296, as amended by Republic Act No. 3828, provides that when the penalty provided by law does not exceed prision correccional, then the Municipal Judge in the capitals of the provinces shall have "like jurisdiction as the Court of First Instance" to try the offense, does the Municipal Court in such cases follow the procedure for Municipal Courts or that for Courts of First Instance? 2. Is preliminary investigation a part of due process? 3. Can there be due process without the presence of the accused during the preliminary 10 investigation.

On February 23, 1966, we required respondents to file an Answer, and ordered the issuance of a Writ of Preliminary injunction restraining respondent Judge from enforcing the warrants of arrest issued in Criminal Cases Nos. 12943, 12944 and 12945. Section 87, paragraph 4 of the Judiciary Act of 1948 (R.A. 296), as amended by Republic Acts Nos. 2613 and 3828, provides. 1wph1.t
Municipal judges in the capitals of provinces and judges of city courts shall have like jurisdiction as the Court of First Instance to. try parties charged with an offense committed within their respective jurisdiction, in which the penalty provided by law does not exceed prision correccional or imprisonment for not more than six years or fine not exceeding six thousand pesos or both, and in the absence of the district judge, shall have like jurisdiction within the province as the Court of First Instance to hear application for bail.

Pursuant to the foregoing provision, both Criminal Cases Nos. 12943 and 12945, for Theft of P590.00 and P1,018.00, respectively, fall under the concurrent jurisdiction of the Municipal Court of Pasig and the Court of First Instance of Rizal, as the penalty provided for said crimes, pursuant to Article 309 (3) of the Revised Penal Code, is prision correccional in its minimum and medium periods.

Criminal Case No. 12944 for Grave Coercion, with a penalty, under Article 286 of the Revised Penal Code, of arresto mayor and a fine not exceeding P500.00, also falls under the concurrent jurisdiction of the Municipal Court of Pasig and the Court of First Instance of Rizal. 11 It is petitioners' submission that because of this concurrent jurisdiction, a Municipal Court acts in reality as a Court of First Instance and, consequently, it cannot issue warrants of arrest without first giving the accused a chance to be heard; and that the Information filed should carry a certification under oath that defendant was given a chance to appear in person at said examination and investigation. Continuing, petitioners argue that since Special Counsel Lucila P. Alcoba of the Office of the Provincial Fiscal of Rizal, in Criminal Cases Nos. 12943 and 12945, merely signed the Complaints for these two cases below the notation, "Approved after preliminary examination", her failure to make the certification under oath to the effect that the accused were given a chance to appear in person or by counsel at said examination and investigation, was violative of the due process clause, and, therefore, the warrants of arrest issued thereafter should be quashed. Section 14, Rule 112 of the Rules of Court, relied upon by petitioners, provides: 1wph1.t
Section 14. Preliminary examination and investigation by provincial or city fiscal or by state attorney in cases cognizable by the Court of First Instance. -Except when an investigation has been conducted by a judge of first instance, justice of the peace or other officer in accordance with the provisions of the preceding sections, no information for an offense cognizable by the Court of First Instance shall be filed by the provincial or city fiscal, or state attorney, without first giving the accused a chance to be heard in a preliminary investigation conducted by him or by his assistant by issuing a corresponding subpoena. lf the accused appears, the investigation shall be conducted in his presence and he shall have the right to be heard, and to cross-examine the complainant and his witnesses. and to adduce evidence in his favor. If he cannot be subpoenaed, or if subpoenaed he does not appear before the fiscal, the investigation shall proceed without him. The fiscal or state attorney shall certify under oath in the information to be filed by him that the defendant was given a chance to appear in person or by counsel at said investigation and examination.

On the other hand, respondents contend that the governing proviso is the second paragraph of Section 10, Rule 112, referring to the right of an accused to preliminary. investigation after arrest, reading: "in cases triable in the municipal or city courts. the accused shall not be entitled as a matter of right to a preliminary investigation in accordance with this section" and that this rule applies whether the case is within the exclusive original jurisdiction of the Municipal Court or within its concurrent jurisdiction with the Court of First Instance. The issue of whether or not an accused is entitled to appear and present evidence in a preliminary investigation in cases falling within the concurrent jurisdiction of the Municipal Court and the Court of First Instance has been squarely resolved in the

negative by this Court, speaking through Mr. Justice Claudio Teehankee, in the cases of People vs. Abejuela and People vs. Endan, 12 reiterated in the case of Banzon vs. Cabato, 64 SCRA 419 (1975), which decisively held, that even though the offense be one falling within the concurrent jurisdiction of the City Courts and Courts of First Instance, the accused is not entitled as a matter of right to be heard in a preliminary investigation under section 10, Rule 112. The reason is because the case goes to trial already after the arrest of the accused and his delivery to the Court. 13 "The ensuing trial on the merits takes the place of preliminary investigation, without needless waste or duplication of time and effort, and a final verdict on the innocence (or guilt) of the accused is thereupon rendered, rather than an inconclusive dismissal of the charge by the fiscal in a preliminary investigation which would not constitute jeopardy." 14 To reiterate and to re-state the rule, therefore, there is no right of preliminary investigation in cases triable by inferior Courts, without distinction as to whether such case be of their exclusive or concurrent jurisdiction. 15 What was conducted by the respondent Judge in these cases is the preliminary examination before the issuance of a warrant of arrest pursuant to section 1, Rule 112. The 1935 Constitution, in section l (3), Article III provides that no warrant shall be issued but upon probable cause to be determined by the Judge after examination of witnesses under oath or affirmation of the complaint and the witnesses he may produce. Conformably thereto, Section 87, paragraph 3, of the Judiciary Act, as amended by Republic Act No. 3828, provides that: before a Municipal Judge may issue a warrant of arrest, the following conditions must first be fulfilled: (1) he must examine the witness or witnesses personally; (2) the examination must be under oath; and (3) the examination must be reduced to writing in the form of searching questions and answers. These requirements have been met in the three criminal cases involved herein. As explained by respondent Judge in his Answer: 1wph1.t
Before the warrants of arrest were issued by the respondent Judge in Criminal Cases Nos. 12944 and 12945 (actually 12943 and 12944), he first conducted, on February 8, 1966, the necessary preliminary examination required by Section l of Rule 112 by adopting, as his own questions, and by asking the complainants and their witnesses, the same or Identical questions asked of them by the Investigating Police Officer in their written statements before the said Police Investigator, Annexes "4", "5", and '7' hereof, and thereafter the respondent Judge required them (the complainants and their witnesses) to subscribe before and make oath to him as to the truth of the answers given by them to the Police Investigator as shown by the fact that in said Annexes "4", "5", and "7", the deponents signed their respective names twice, once before the Investigating Police Officer and the second time before the respondent Judge who also required them to take the jurat to the oath, thereby complying to the requirements of Section 87 of the Judiciary Act of 1948, as amended, providing therein that 'no warrant of arrest shall be issued by any justice of the peace in any criminal case filed with him unless he first examines the witness or witnesses personally, and the examination shall be under oath 16 and reduced to writing in the form of searching questions and answers.

By "searching questions and answers" is meant: 1wph1.t

the term "searching questions and answers" means only, taking into consideration the purpose of the preliminary examination which is to determine 'whether there is a reasonable ground to believe that an offense has been committed and the accused is probably guilty thereof so that a warrant of arrest may be issued and the accused held for trial', such questions as have tendency to show the commission of a crime and the perpetrator thereof. What would be searching questions would depend on what is sought to be inquired into, such as: the nature of the offense, the date, time, and place of its commission, the possible motives for its commission the subject, his age, education, status, financial and social circumstances, his attitude toward the investigation, social attitudes, opportunities to commit the offense; the victim, his age, status, family responsibilities, financial and social circumstances, characteristics, etc. The points that are the subject of inquiry may differ from case to case. The questions, therefore, must to a great degree depend upon the Judge making the investigation. At any rate, the court a quo found that respondent Judge was satisfied that the questions and answers contained in the sworn statements taken by T-Sgt. Patosa partake of the nature of his searching 17 questions and answers as required by law,' so the respondent Judge adopted them.

In the language of this Court in De Mulata vs. Irizari, 61 SCRA 210, 213 (1974): 1wph1.t
The requirement that the investigating judge must examine the witnesses personally, which examination shall be under oath and reduced to writing in the form of searching questions and answers, is fulfilled where the municipal judge examined under oath the witnesses by asking questions that were adopted from a previous investigation and considered by him as sufficiently searching and, which questions and the answers thereto, were in writing and sworn to before him prior to his issuance of the order of arrest.

In regards Criminal Case No. 12945 for Theft, respondent Judge had this to say: 1wph1.t
As regards Criminal Case No. 12943 (actually 12945), although, the respondent Judge did not take the oath of the complainant and her witness on the statement given by them to the Police investigator, Annexes "I" and "2" hereof, Special Counsel Lucile P. Alcoba of the Office of the Provincial Fiscal of Rizal conducted the necessary preliminary examination required by Section l of Rule l l 2 in that, as can be seen from said Annexes 'I' and '2', she asked the same or Identical questions appearing in said annexes to the deponents and adopted the questioning of the Police Investigator as her own interrogations of the complainant and her witness, and thereafter she required them to subscribe their respective names and to swear before her as to the truth of the answers given by them to each and every question appearing in said Annexes '1' and '2' and, although there was no certification, in the exact form required by law, by Special Counsel Lucile P. Alcoba that she conducted the required preliminary examination of the complainant and her witness, it is admitted that her certification in the body of the complaint stating "Approved after preliminary examination", accompanied by the oath taken by her before the respondent Judge after making such certification, is a substantial compliance to the requirements of the law although it can be said that the same is somewhat defective in form. (pp. 46-47, Rollo) xxx xxx xxx

From the foregoing explanation, lt may be deduced that respondent Judge was satisfied that the questions and answers in a previous investigation by Special Counsel Alcoba

partook of the nature of his searching questions and answers and made them his own. As held in Luna vs. Plaza, supra, the Judiciary Act as amended by Republic Act No. 3828, does not prohibit the Municipal Judge from adopting the questions asked by a previous investigator. For, in the final analysis, whether or not probable cause exists or not depends upon the judgment and discretion of the Judge issuing the warrant of arrest (De Mulata vs. Irizari, supra). In Criminal Case No. 12945 below, respondent Judge had convinced himself that probable cause existed before he issued the warrant of arrest. Under the attendant circumstances, respondent Judge may not be said to have acted arbitrarily. We reiterate, however, the reminder in the Luna case (supra), reading: 1wph1.t
We wish to stress, however, that what has been stated in this opinion is certainly not intended to sanction the return to the former practice of municipal judges of simply relying upon affidavits or sworn statements that are made to accompany the complaints that are filed before them, in determining whether there is a probable cause for the issuance of a warrant of arrest. That practice is precisely what is sought to be voided by the amendment of Section 87 (c) of Republic Act. 296 (Judiciary Act of 1948) which requires that before a municipal judge issues a warrant of arrest he should first satisfy himself that there is a probable cause by examining the witnesses personally, and that the examination must be under oath and reduced to writing in the form of searching questions and answers. It is obvious that the purpose of this amendment is to prevent the issuance of a warrant of arrest against a person based simply upon affidavits of witnesses who made, and swore to, their statements before a person or persons other than the judge before whom the criminal complaint is filed. We wish to emphasize strict compliance by municipal or city judges of the provision of Section 87(c) of the Judiciary Act of 1948, as amended by Republic Act 3828, in order to avoid malicious and/or unfounded criminal prosecution of persons. (Luna vs. Plaza, supra p. 323)

Petitioners further maintain that Section 5 of Rule 112 of the Rules of Court, in so far as it authorizes the Municipal Court to conduct a preliminary examination before the issuance of a Warrant of Arrest without previous notice to the accused, is unconstitutional as it violates the guarantee of equal protection of the laws, and Section l (15), Art. III of the 1935 Constitution which states, "No person shall be held to answer for a criminal offense without due process of law." Section 5, Rule l l 2 provides: 1wph1.t
The municipal, the city judge, the fiscal or the municipal mayor who conducts the preliminary examination as provided in these rules must take under oath, either in the presence or in the absence of the accused, the testimony of the complainant and his witnesses. The testimony of the complainant and his witnesses shall be reduced to writing and signed by them.

The preliminary examination referred to is defined, under Section l of Rule 112, as a previous inquiry or examination made before the arrest of the accused by a Judge or officer authorized to conduct the same, with whom a Complaint or Information has been filed imputing the commission of an offense cognizable by the Court of First Instance, for the purpose of determining whether there is a reasonable ground to believe that an offense has been committed and the accused is probably guilty thereof, so that a warrant of arrest may be issued and the accused held for trial. This section does not

refer to the preliminary investigation proper provided for under Section 10, Rule 112, in which the accused is given access to the testimony and evidence presented against him at the preliminary examination, and to present evidence if he so desires. From Section 5 of Rule 112, supra, it is clear that, unlike in the preliminary investigation proper, an accused is not entitled as a matter of right to be present, during the preliminary examination nor to cross-examine the witnesses presented against him before his arrest, the purpose of said examination being merely to determine whether or not there is sufficient reason to issue a warrant of arrest. 18 Section l (3), Article III of the 1935 Constitution commanding the determination of probable cause prior to issuance of a warrant arrest, requires no notice to an accused. A preliminary examination is generally a proceeding ex-parte in which the person charged has no right to participate or be present. The right to confrontation of witnesses neither applies to a preliminary hearing. The reason therefor has been explained thus: 1wph1.t
... It can not be seriously contended that an accused person has a right to be present during this stage of the proceedings. To hold that he had such a right and to reverse a judgment of conviction on this ground would have the effect of destroying the very purpose of that part of the criminal law. lt would be against public policy. lt is frequently essential that such investigations be kept secret and that the accused should have no suspicion of any complaint against him, otherwise he might avoid punishment for his crime by escaping before arrest. (U.S. vs. Grant, et al., 18 Phil. 122, 147) ... it is often the only means of discovering the persons who may reasonably be charged with the crime so as to enable the fiscal to prepare his complaint or information, ... (People vs. Badilla, 48 Phil., 719, 731)

While section l (3) Art. III of the 1935 Constitution does require, before the issuance of a warrant of arrest, the determination of probable cause by the Judge after examination of witnesses he may produce, the curtailment of the presence of an accused during that preliminary examination entails no infringement of the constitutional right to due process of law nor to equal protection of the laws. Thus, in Manzano vs. Villa, 19 this Court categorically held: 1wph1.t
The preliminary examination conducted by the municipal judge was essentially a procedural matter and no substantial rights of the accused were violated just because he had not been given an opportunity to examine the witnesses against him. The first stage of the preliminary investigation is 'not the occasion for full and exhaustive presentation of parties' evidence but only such as may engender well-grounded belief that an offense has been committed and that the accused is probably guilty thereof' The proceeding is usually held ex-parte, for under section 5 of Rule 112 all that is required is for the judge conducting such examination to 'take under oath, either in the presence or absence of the accused, the testimony of the complainant and his witnesses,' said testimony to be reduced to writing and signed by them. Hence, the absence of the accused during the preliminary examination was not a denial of due process of law.

Neither can the withholding of the right of preliminary investigation from the accused in cases triable by inferior Courts be termed ' an unjust or unfair distinction, as explained in People vs. Abejuela, supra: 1wph1.t

... The loss of time entailed in the conduct of preliminary investigations, with the consequent extension of deprivation of the accused's liberty, in case he fails to post bail, which at times out- lasts the period of the penalty provided by law for the offense, besides the mental anguish suffered in protracted litigations, are eliminated with the assurance of a speedy and expeditious trial for the accused, upon his arraignment (without having to undergo the second stage of the preliminary investigation), and of a prompt verdict on his guilt or innocence. On the other hand, the so-called first stage of preliminary investigation or the preliminary examination, conducted by the duly authorized officer, as borne out by the examination and sworn written statements of the complainants and their witnesses, generally offices to establish the existence of reasonable ground to charge the accused with having committed the offense complained of.

Attention should also be called to the fact that neither the 1935 nor the 1973 Constitution requires the holding of a preliminary investigation. lt is settled doctrine that the right hereto is of statutory character and may be invoked only when specifically created by statute. 20 lt is not a fundamental right and may be waived expressly or by silence. 21 In a nutshell, the proceedings in these three criminal cases conformed to law and jurisprudence. But even conceding that petitioners were entitled to a preliminary investigation, the proper forum before which absence thereof should have been raised and ventilated was in the trial Court, not in an appellate Court because the absence of preliminary investigation does not go to the jurisdiction of the Court but merely to the regularity of the proceedings, and bearing in mind that preliminary investigation can be waived, as in fact, it is frequently waived. 22 WHEREFORE, the Petition is hereby denied and the Writ of Preliminary Injunction heretofore issued is hereby lifted. Costs against petitioners. SO ORDERED.

Roan v. Gonzales, 145 SCRA 687 (1986)


Roan v. Gonzales, 145 SCRA 687 (1986) F: The challenged SW was issued by the resp. judge on 5/10/84. The petitioner''s house was searched 2 days later but none of the articles listed in the warrant was discovered. The officers conducting the search found 1 colt Magnum revolver & 18 live bullets w/c they confiscated. They are now the bases of the charge against the petitioner. RULING: Search warrant issued by resp. judge is hereby declared null and void and accordingly set aside. The petitioner claims that no depositions were taken by the resp. judge in accordance w/ Rule 126, Sec. 4 of the ROC, but this is not entirely true. Depositions were taken of the complainant''s 2 witnesses in addition to the affidavit executed by them. It is correct to say, however, that the complainant himself was not subjected to a similar interrogation. By his own accounts, all that resp. judge did was question Capt. Quillosa on the contents of his affidavit only "to ascertain among others, if he knew and understood the same," and only bec. "the application was not yet subscribed and sworn to." The suggestion is that he would not have asked any questions at all if the affidavit had already been completed when it was submitted to him. In any case, he did not ask his own searching questions. He limited himself to the contents of the affidavit. He did not take the applicant''s deposition in writing and attach them to the record, together

w/ the affidavit presented to him. Such written deposition is necessary in order that the Judge may be able to properly determine the existence or non-existence of the probable cause, to hold liable for perjury the person giving it if it will be found later that his declarations are false. (Mata v. Bayona.) The applicant was asking for the issuance of the SW on the basis of mere hearsay and not of info. personally known to him. His application, standing alone, was insufficient to justify the issuance of the warrant sought. It was, therefore, necessary for the witnesses themselves, by their own personal info., to establish the applicant''s claims. Even assuming then that it would have suffied to take the deposition only of the witnesses and not of the applicant himself, there is still the question of the sufficiency of their depositions. A study of the deposition taken from witnesess Esmael Morada and Jesus Tohilida, who both claimed to be "intelligence informers," shows that they were in the main a mere restatement of their allegations in their affidavits, except that they were made in the form of answers to the questions put to them by the resp. judge. One may well wonder why it did not occur to the resp. judge to ask how the witness could be so certain even as to the caliber of the guns, or how far he was from the window, or whether it was on the first floor or second floor, or why his presence was not noticed at all, or if the acts related were really done openly, in the full view of the witnesses, considering that these acts were against the law. These would have been judicious questions but they were injudiciously omitted. Instead, the declaration of the witnesses were readily accepted and the warrant sought was issued forthwith. SOL-GEN ARGUES THAT THE PETITIONER WAIVED WHATEVER DEFECT WHEN THE PETITIONER VOLUNTARILY SUBMITTED TO THE SEARCH AND MANIFESTED HIS CONFORMITY IN WRITING. We do not agree. What we see here is pressure exerted by the military authorities, who practically coerced the petitioner to sign the supposed waiver as guaran

Roan v. Gonzales, 145 SCRA 687 ( 1986) F: The challenged SW was issued by the resp. judge on 5/10/84. The petitioners house was searched 2 days later but none of the articles listed in the warrant was discovered. The officers conducting the search found 1 colt Magnum revolver & 18 live bullets w/c they confiscated. They are now the bases of the charge against the petitioner. RULING: Search warrant issued by resp. judge is hereby declared null and void and accordingly set aside. The petitioner claims that no depositions were taken by the resp. judge in accordance w/ Rule 126, Sec. 4 of the ROC, but this is not entirely true. Depositions were taken of the complainants 2 witnesses in addition to the affidavit executed by them. It is correct to say, however, that the complainant himself was not subjected to a similar interrogation. By his own accounts, all that resp. judge did was question Capt. Quillosa on the contents of his affidavit only to ascertain among others, if he knew and understood the same, and only bec. the application was not yet subscribed and sworn to. The suggestion is that he would not have asked any questions at all if the affidavit had already been completed when it was submitted to him. In any case, he did not ask his own searching questions. He limited himself to the contents of the affidavit. He did not take the applicants deposition in writing and attach them to the record, together w/ the affidavit presented to him. Such written deposition is necessary in order that the Judge may be able to properly determine the existence or non-existence of the probable cause, to hold liable for perjury the person giving it if it will be

found later that his declarations are false. (Mata v. Bayona.) The applicant was asking for the issuance of the SW on the basis of mere hearsay and not of info. personally known to him. His application, standing alone, was insufficient to justify the issuance of the warrant sought. It was, therefore, necessary for the witnesses themselves, by their own personal info., to establish the applicants claims. Even assuming then that it would have suffied to take the deposition only of the witnesses and not of the applicant himself, there is still the question of the sufficiency of their depositions. A study of the deposition taken from witnesess Esmael Morada and Jesus Tohilida, who both claimed to be intelligence informers, shows that they were in the main a mere restatement of their allegations in their affidavits, except that they were made in the form of answers to the questions put to them by the resp. judge. One may well wonder why it did not occur to the resp. judge to ask how the witness could be so certain even as to the caliber of the guns, or how far he was from the window, or whether it was on the first floor or second floor, or why his presence was not noticed at all, or if the acts related were really done openly, in the full view of the witnesses, considering that these acts were against the law. These would have been judicious questions but they were injudiciously omitted. Instead, the declaration of the witnesses were readily accepted and the warrant sought was issued forthwith. SOL-GEN ARGUES THAT THE PETITIONER WAIVED WHATEVER DEFECT WHEN THE PETITIONER VOLUNTARILY SUBMITTED TO THE SEARCH AND MANIFESTED HIS CONFORMITY IN WRITING. We do not agree. What we see here is pressure exerted by the military authorities, who practically coerced the petitioner to sign the supposed waiver as guaran

** Mata v Bayona 128 SCRA 388 (1984)


Facts: Soriano Mata was accused under Presidential Decree (PD) 810, as amended by PD 1306, the information against him alleging that Soriano Mata offered, took and arranged bets on the Jai Alai game by selling illegal tickets known as Masiao tickets without any authority from the Philippine Jai Alai & Amusement Corporation or from the government authorities concerned. Mata claimed that during the hearing of the case, he discovered that nowhere from the records of the said case could be found the search warrant and other pertinent papers connected to the issuance of the same, so that he had to inquire from the City Fiscal its whereabouts, and to which inquiry Judge Josephine K. Bayona, presiding Jufe of the City Court of Ormoc replied, it is with the court. The Judge then handed the records to the Fiscal who attached them to the records. This led Mata to file a motion to quash and annul the search warrant and for the return of the articles seized, citing and invoking, among others, Section 4 of Rule 126 of the Revised Rules of Court. The motion was denied by the Judge on 1 March 1979, stating that the court has made a thorough investigation and examination under oath of Bernardo U. Goles and Reynaldo T. Mayote, members of the Intelligence Section of 352nd PC Co./Police District II INP; that in fact the court made a certification to that effect; and that the fact that documents relating to the search warrant were not attached immediately to the record of the criminal case is of no moment, considering that the rule does not specify when these documents are to be attached to the records. Matas motion for reconsideration of the aforesaid order having been denied, he came to the Supreme Court, with the petition for certiorari, praying, among others, that the Court declare the search warrant to be invalid for its alleged failure to comply with the requisites of the Constitution and the Rules of Court, and that all the articles confiscated under such warrant as inadmissible as evidence in the case, or in any proceedings on the matter.

Issue: Whether the judge must before issuing the warrant personally examine on oath or affirmation the complainant and any witnesses he may produce and take their depositions in writing, and attach them to the record, in addition to any affidavits presented to him. Held: Under the Constitution no search warrant shall issue but upon probable cause to be determined by the Judge or such other responsible officer as may be authorized by law after examination under oath or affirmation of the complainant and the witnesses he may produce. More emphatic and detailed is the implementing rule of the constitutional injunction, The Rules provide that the judge must before issuing the warrant personally examine on oath or affirmation the complainant and any witnesses he may produce and take their depositions in writing, and attach them to the record, in addition to any affidavits presented to him. Mere affidavits of the complainant and his witnesses are thus not sufficient. The examining Judge has to take depositions in writing of the complainant and the witnesses he may produce and to attach them to the record. Such written deposition is necessary in order that the Judge may be able to properly determine the existence or nonexistence of the probable cause, to hold liable for perjury the person giving it if it will be found later that his declarations are false. We, therefore, hold that the search warrant is tainted with illegality by the failure of the Judge to conform with the essential requisites of taking the depositions in writing and attaching them to the record, rendering the search warrant invalid.

Corro v. Lising 137 SCRA 341 (1985)


Corro v. Lising 137 SCRA 341 (1985) F: Respondent Judge issued a search warrant for the seizure of articles allegedly used by petitioner in committing the crime of sedition. Seized were printed copies of the Philippine Times, newspaper dummies, typewriters, mimeographing machines and tape recorders, video machines and tapes. The petitioner moved to quash the warrant but his motion was denied. HELD: The statements made in the affidavits are mere conclusions of law and do not satisfy the requirement of probable cause. The language used is all embracing as to include all conceivable words and equipment of petitioner regardless of whether they are legal or illegal. The search warrant under consideration was in the nature of a general warrant which is objectionable.

Corro v. Lising Philippine Times conclusions of law of military officers will not satisfy probable cause requirement for issuance of search warrants. G.R. No. L-69899 July 15, 1985 ROMMEL CORRO, petitioner, vs. HON. ESTEBAN LISING Presiding Judge, Regional Trial Court, Quezon City, Branch XCV HON. REMIGIO ZARI Regional Trial Court, Quezon City, Branch 98; CITY FISCAL'S OFFICE, Quezon City; LT. COL. BERLIN A. CASTILLO and 1ST LT. GODOFREDO M. IGNACIO, respondents, Reynaldo L. Bagatsing for petitioner. RELOVA, J.: On September 29, 1983, respondent Regional Trial Court judge Esteban Lising of Quezon City, upon application filed by Lt. Col. Berlin Castillo of the Philippine

Constabulary Criminal Investigation Service, issued Search Warrant No. Q-00002 authorizing the search and seizure of
1. Printed copies of Philippine Times; 2. Manuscripts/drafts of articles for publication in the Philippine Times; 3. Newspaper dummies of the Philippine Times; 4. Subversive documents, articles, printed matters, handbills, leaflets, banners; 5. Typewriters, duplicating machines, mimeographing and tape recording machines, video machines and tapes

which have been used and are being used as instrument and means of committing the crime of inciting to sedition defined and penalized under Article 142 of the Revised Penal Code, as amended by PD 1835 ... (p. 24, Rollo) On November 6, 1984, petitioner filed an urgent motion to recall warrant and to return documents/personal properties alleging among others that:
2. ... the properties seized are typewriters, duplicating machines, mimeographing and tape recording machines, video machines and tapes which are not in any way, inanimate or mute things as they are, connected with the offense of inciting to sedition. 3. More so, documents or papers seized purporting to do the body of the crime has been rendered moot and academic due to the findings of the Agrava Board that a military conspiracy was responsible for the slaying of the late Senator Benigno Aquino, Jr. on August 21, 1983 at the Manila International Airport. The Agrava Board which has the exclusive jurisdiction to determine the facts and circumstances behind the killing had virtually affirmed by evidence testamentary and documentary the fact that soldiers killed Benigno Aquino, Jr. 4. More so, the grave offense of libel, RTC, Q.C. Branch XCV has dismissed said case against the accused on all documents pertinent and more so as we repeat, rendered moot and academic by the recent Agrava Report. (p. 27, Rollo)

On January 28, 1985, respondent Judge Lising denied the motion in a resolution, pertinent portions of which state:
... The said articles presently form part of the evidence of the prosecution and they are not under the control of the prosecuting arm of the government. Under these circumstances, the proper forum from which the petition to withdraw the articles should be addressed, is the Office of the City Fiscal, Quezon City and not with this Branch of the Court. It is to be further noted that it is not even with this Branch of the Court that the offense of inciting to sedition is pending. (p 29, Rollo)

Hence, this petition for certiorari and mandamus, with application for preliminary injunction and restraining order to enjoin respondent Regional Trial Court, National Capital Region, Branch 98 from proceeding with the trial of Criminal Case No. S3-Q-

29243, praying (a) that Search Warrant No. Q-00002 issued by respondent Judge Esteban M. Lising be declared null and void ab initio and that a mandatory injunction be issued directing respondents City Fiscal's Office of Quezon City and Lt. Col. Berlin Castillo and 1st Lt. Godofredo Ignacio jointly and severally to return immediately the documents/properties illegally seized from herein petitioner and that final injunction be issued enjoining respondents City Fiscal's Office of Quezon City, Lt. Col. Castillo and 1st Lt. Ignacio from utilizing said documents/properties as evidence in Criminal Case No. 29243; and (b) that respondent PC-CIS officers Lt. Col. Berlin A. Castillo and lst Lt. Godofredo Ignacio be directed to reopen the padlocked office premises of the Philippine Times at 610 Mezzanine Floor, Gochengco Building, T.M., Kalaw, Ermita, Manila. In Our Resolution of February 19, 1985, respondents were required to file their comment. The plea for temporary restraining order was granted and respondents City Fiscal's Office of Quezon City, Lt. Col. Berlin Castillo and 1st Lt. Godofredo Ignacio were enjoined from introducing as evidence for the state the documents/properties seized under Search Warrant No. Q-00002 in Criminal Cage No. Q-29243 (Sedition case against petitioner), pending before the Regional Trial Court of Quezon City, Branch 98, effective immediately and continuing until further orders from the Court. Respondents would have this Court dismiss the petition on the ground that (1) the present action is premature because petitioner should have filed a motion for reconsideration of respondent Judge Lising's order of January 28, 1985; (2) probable cause exists justifying the issuance of a search warrant; (3) the articles seized were adequately described in the search warrant; (4) a search was conducted in an orderly manner; (5) the padlocking of the searched premises was with the consent of petitioner's wife; (6) the findings of the Agrava Board is irrelevant to the issue of the validity of the search warrant; (7) press freedom is not an issue; and, (8) the petition is barred by laches. There is merit in the petition. Respondents contend that petitioner should have filed a motion for reconsideration of the order in question before coming to Us. This is not always so. When the questions raised before the Supreme Court are the same as those which were squarely raised in and passed upon by the lower court, the filing of the motion for reconsideration in said court before certiorari can be instituted in the Supreme Court is no longer a prerequisite. As held in Bache & Co. (Phil.), Inc. vs. Ruiz, 37 SCRA 823, (t)he rule requiring the filing of a motion for reconsideration before an application for a writ of certiorari can be entertained was never intended to be applied without considering the circumstances. The rule does not apply where, the deprivation of petitioners' fundamental right to due process taints the proceeding against them in the court below not only with irregularity but also with nullity." Likewise, in Pajo, et al. vs. Ago, et al., 108 Phil. 905 and in Gonzales vs. Court of Appeals, 3 SCRA 465, this Court ruled that "it is only when questions are raised for the first time before the high court in a certiorari case that the writ shall not issue, unless the lower court had first been given an opportunity to pass upon the same." Further, in the case of Matute vs. Court of Appeals, 26 SCRA 768, We

held that "while as a matter of policy a motion for reconsideration in the lower court has often been considered a condition sine qua non for the granting of a writ of certiorari, this rule does not apply where the proceeding in which the error occurred is a patent nullity or where 'the deprivation of petitioner's fundamental right to due process ... taints the proceeding against him in the court below not only with irregularity but with nullity (Luzon Surety Co. v. Marbella et al., L-16038, Sept. 30, 1960), or when special circumstances warrant immediate and more direct action. ..." The records of this petition clearly disclose that the issues herein raised have already been presented to and passed upon by the court a quo. Section 3, Article IV of the 1973 Constitution provides:
SEC. 3. ...no search warrant or warrant of arrest issue except upon probable cause to be determined by the judge, or such other responsible officer as may be authorized by law, after examination under oath or affirmation of the complainant and the witnesses he may produce, and particularly describing the place to be searched and the persons or things to be seized.

and, Section 3, Rule 126 of the New Rules of Court, states that:
SEC. 3. Requisites for issuing search warrant. A search warrant shall not issue but upon probable cause in connection with one specific offense to be determined by the judge or justice of the peace after examination under oath or affirmation of the complainant and the witnesses he may produce, and particularly describing the place to be searched and the persons or things to be seized.

Probable cause may be defined as "such reasons, supported by facts and circumstances, as will warrant a cautious man in the belief that his actions, and the means taken in prosecuting it, are legally just and proper (Burton vs. St. Paul, M & M. Ry. Co., 33 Minn. 189, cited in U.S. vs. Addison, 28 Phil. 566)." Thus, an application for search warrant must state with particularly the alleged subversive materials published or intended to be published by the publisher and editor of the Philippine Times, Rommel Corro. As We have stated in Burgos, Sr. vs. Chief of Staff of the Armed Forces of the Philippines, 133 SCRA 800, "mere generalization will not suffice." A search warrant should particularly describe the place to be searched and the things to be seized. "The evident purpose and intent of this requirement is to limit the things to be seized to those, and only those, particularly described in the search warrant- to leave the officers of the law with no discretion regarding what articles they should seize, to the end that unreasonable searches and seizures may not be committed, that abuses may not be committed Bache & Co. Phil. Inc. vs, Ruiz, supra)." The affidavit of Col. Castillo states that in several issues of the Philippine Times:
... we found that the said publication in fact foments distrust and hatred against the government of the Philippines and its duly constituted authorities, defined and penalized by Article 142 of the Revised Penal Code as amended by Presidential Decree No. 1835; (p. 22, Rollo)

and, the affidavit of Lt. Ignacio reads, among others

... the said periodical published by Rommel Corro, contains articles tending to incite distrust and hatred for the Government of the Philippines or any of its duly constituted authorities. (p. 23, Rollo)

The above statements are mere conclusions of law and will not satisfy the requirements of probable cause. They can not serve as basis for the issuance of search warrant, absent of the existence of probable cause. In fact, as a consequence of the search warrant issued, the items confiscated from the premises of the office of the Philippine Times at 610 Mezzanine Floor, Gochengco Bldg., T.M. Kalaw, Ermita, Manila were the following:
1. One bundle of assorted negative; 2. One bundle of assorted lay out; 3. Three folders of assorted articles/writings used by Philippine Times news and other paraphernalias; 4. Four tape alleged speech of Mayor Climaco, two alleged speeches of Aquino and a speech of one various artist; 5. One bundle Dummies; 6. Ten bundles of assorted copies of Philippine Times issued on different dates (Nos. 6, 7, 8, 9, 10, 11, 12, 13, 14 & 15): 7. One Typewriter Remington Brand Long Carriage with No. J-2479373; 8. OneTypewriterAdler-short with No. 9003011; 9. Three (3) bundles of Philippine Times latest issue for Baguio City (p. 26, Rollo)

In Stonehill vs. Diokno, 20 SCRA 383, this Court held that search warrants authorizing the seizure of books of accounts and records "showing all the business transactions" of certain persons, regardless of whether the transactions were legal or illegal, contravene the explicit comment of the Bill of Rights that the things to be seized should be particularly described and defeat its major objective of eliminating general warrants. In the case at bar, the search warrant issued by respondent judge allowed seizure of printed copies of the Philippine Times, manuscripts/drafts of articles for publication, newspaper dummies, subversive documents, articles, etc., and even typewriters, duplicating machines, mimeographing and tape recording machines. Thus, the language used is so all embracing as to include all conceivable records and equipment of petitioner regardless of whether they are legal or illegal. The search warrant under consideration was in the nature of a general warrant which is constitutionally objectionable. Respondents do not deny the fact that the business office of the "Philippine Times" of which petitioner was the publisher-editor was padlocked and sealed. The consequence is, the printing and publication of said newspaper were discontinued. In Burgos, Sr. vs.

Chief of Staff of the Armed Forces of the Philippines, supra, We held that "[sluch closure is in the nature of previous restraint or censorship abhorrent to the freedom of the press guaranteed under the fundamental law, and constitutes a virtual denial of petitioners' freedom to express themselves in print. This state of being is patently anathematic to a democratic framework where a free, alert and even militant press is essential for the political enlightenment and growth of the citizenry." Finally, respondents argue that while the search warrant was issued on September 29, 1983 and was executed on the very same day, it was only on November 6, 1984, or one (1) year, one (1) month and six (6) days when petitioner filed his motion for the recall of the warrant and the return of the documents/personal properties. Having failed to act seasonably, respondents claim that petitioner is guilty of laches. Laches is the failure or neglect, for an unreasonable and unexplained length of time, to do that which by exercising due diligence, could or should have been done earlier. The negligence or omission to assert a right within a reasonable time, warranting a presumption that the party entitled to assert it either has abandoned it or declined to assert it (Tijam vs. Sibonghanoy, L-21450, April 15, 1968, 23 SCRA 35). In his petition, Corro alleged that on October 1, 1983, less than forty-two (42) hours after the military operatives shut down his newspaper on September 29, 1983, he was invited by the Director-General PC/INP, and subsequently detained. Thereafter, he was charged with the crime of inciting to sedition before the City Fiscal's Office in Quezon City, and on October 7, 1983, a preventive detention action was served upon him. Consequently, he had to file a petition for habeas corpus. It was only on November 8, 1984 when this Court issued its Resolution in G.R. No. 68976, entitled: In the Matter of the Petition for Habeas Corpus of Rommel Corro Angle Corro vs. Minister Juan Ponce Enrile, et al., releasing Rommel Corro on recognizance of his lawyers, Attys. Humberto B. Basco, Reynaldo Bagatsing and Edilberto Balce, In the same month, November 1984, petitioner filed his motion to recall warrant and to return the seized documents. When respondent judge denied the motion, he came to Us. Considering the above circumstances, the claim that petitioner had abandoned his right to the possession of the seized properties is incorrect. WHEREFORE, Search Warrant No. Q-00002 issued by the respondent judge on September 29, 1983 is declared null and void and, accordingly, SET ASIDE. The prayer for a writ of mandatory injunction for the return of the seized articles is GRANTED and all properties seized thereunder are hereby ordered RELEASED to petitioner. Further, respondents Lt. Col. Berlin A. Castillo and lst Lt. Godofredo M. Ignacio are ordered to RE-OPEN the padlocked office premises of the Philippine Times at 610 Mezzanine Floor, Gochengco Bldg., T.M. Kalaw, Ermita, Manila. SO ORDERED.

NOLASCO V. PAO - 147 SCRA 509 FACTS:


The case at bar is for the motion for partial reconsideration of both petitioners and respondents of the SCs decision that the questioned search warrant by petitioners is null and void, that respondents are enjoined from introducing evidence using such search warrant, but such personalities obtained would still be retained, without prejudice to petitioner AguilarRoque. Respondents contend that the search warrant is valid and that it should be considered in the context of the crime of rebellion, where the warrant was based. Petitioners on the other hand, on the part of petitioner Aguilar-Roque, contend that a lawful search would be justified only by a lawful arrest. And since there was illegal arrest of Aguilar-Roque, the search was unlawful and that the personalities seized during the illegal search should be returned to the petitioner. The respondents, in defense, concede that the search warrants were null and void but the arrests were not.

HELD:
"Any evidence obtained in violation of this . . . section shall be inadmissible for any purpose in any proceeding" (Sec. 4[2]). This constitutional mandate expressly adopting the exclusionary rule has proved by historical experience to be the only practical means of enforcing the constitutional injunction against unreasonable searches and seizures by outlawing all evidence illegally seized and thereby removing the incentive on the part of state and police officers to disregard such basic rights. What the plain language of the Constitution mandates is beyond the power of the courts to change or modify. All the articles thus seized fag under the exclusionary rule totally and unqualifiedly and cannot be used against any of the three petitioners. Nolasco vs. Pao 139 SCRA 152 (1985) FACTS: The crime alleged is rebellion. Aguilar-Roque was one of the accused of rebellion in a criminal case before a Special Military Commission. At that time, she was at large. The military authorities arrested her and Nolasco, who had no standing arrest against him, while the latter were onboard a public vehicle. Consequently, the military authorities searched the residence of Aguilar-Roque. They seized 428 documents and written materials, and additionally a portable typewriter, and 2 wooden boxes. ISSUE: Whether or not the search and seizure fall under the rule of warrantless search incidental to a lawful arrest. HELD: Yes, the search and seizure fall under the rule of warrantless search incidental to a lawful arrest. Considering that Aguilar-Roque has been charged with rebellion, which is a crime against public order, that the warrant for her arrest has not been served for a considerable period of time; that she was arrested within the general vicinity of her dwelling; and that the search of her

dwelling was made within a half hour of her arrest, the search and seizure conducted did not need a search warrant, for possible effective results in the interest of public order. G.R. No. L-64261 December 26, 1984 JOSE BURGOS, SR vs. THE CHIEF OF STAFF- AFP, ET AL Facts: Assailed in this petition for certiorari prohibition and mandamus with preliminary mandatory and prohibitory injunction is the validity of two [2] search warrants issued on December 7, 1982 by Judge Ernani Cruz-Pano of the then CFI of Rizal [Quezon City], under which the premises of the "Metropolitan Mail" and "We Forum" newspapers, respectively, were searched, and office and printing machines, equipment, paraphernalia, motor vehicles and other articles used in the printing, publication and distribution of the said newspapers, as well as numerous papers, documents, books and other written literature alleged to be in the possession and control of petitioner Jose Burgos, Jr. publisher-editor of the "We Forum" newspaper, were seized. Petitioners further pray that a writ of preliminary mandatory and prohibitory injunction be issued for the return of the seized articles, and that respondents be enjoined from using the articles thus seized as evidence against petitioner Jose Burgos, Jr. and the other accused in Criminal Case No. Q- 022782 of the Regional Trial Court of Quezon City, entitled People v. Jose Burgos, Jr. et al. Issue: Was the closure of WE Forum a case of prior restraint? Ruling: Yes. As heretofore stated, the premises searched were the business and printing offices of the "Metropolitan Mail" and the "We Forum newspapers. As a consequence of the search and seizure, these premises were padlocked and sealed, with the further result that the printing and publication of said newspapers were discontinued. Such closure is in the nature of previous restraint or censorship abhorrent to the freedom of the press guaranteed under the fundamental law, and constitutes a virtual denial of petitioners' freedom to express themselves in print. This state of being is patently anathematic to a democratic framework where a free, alert and even militant press is essential for the political enlightenment and growth of the citizenry.
BURGOS, SR. V. CHIEF OF STAFF, AFP [133 SCRA 800; G.R. NO. 64261; 26 DEC 1984]

Facts: Petitioners assail the validity of 2 search warrants


issued on December 7, 1982 by respondent Judge CruzPano of the then Court of First Instance of Rizal, under which the premises known as No. 19, Road 3, Project 6, Quezon City, and 784 Units C & D, RMS Building, Quezon Avenue, Quezon City, business addresses of the "Metropolitan Mail" and "We Forum" newspapers, respectively, were searched, and office and printing machines, equipment, paraphernalia, motor vehicles and other articles used in the printing, publication and distribution of the said newspapers, as well as numerous

papers, documents, books and other written literature alleged to be in the possession and control of petitioner Jose Burgos, Jr. publisher-editor of the "We Forum" newspaper, were seized. As a consequence of the search and seizure, these premises were padlocked and sealed, with the further result that the printing and publication of said newspapers were discontinued. Respondents contend that petitioners should have filed a motion to quash said warrants in the court that issued them before impugning the validity of the same before this Court. Respondents also assail the petition on ground of laches (Failure or negligence for an unreasonable and unexplained length of time to do that which, by exercising due diligence, could or should have been done earlier. It is negligence or omission to assert a right within a reasonable time, warranting a presumption that the party entitled to assert it either has abandoned it or declined to assert it). Respondents further state that since petitioner had already used as evidence some of the documents seized in a prior criminal case, he is stopped from challenging the validity of the search warrants. Petitioners submit the following reasons to nullify the questioned warrants: 1. Respondent Judge failed to conduct an examination under oath or affirmation of the applicant and his witnesses, as mandated by the above-quoted constitutional provision as well as Sec. 4, Rule 126 of the Rules of Court. 2. The search warrants pinpointed only one address which would be the former abovementioned address. 3. Articles belonging to his co-petitioners were also seized although the warrants were only directed against Jose Burgos, Jr.

4. Real properties were seized. 5. The application along with a joint affidavit, upon which the warrants were issued, from the Metrocom Intelligence and Security Group could not have provided sufficient basis for the finding of a probable cause upon which a warrant may be validly issued in accordance with Section 3, Article IV of the 1973 Constitution. Respondents justify the continued sealing of the printing machines on the ground that they have been sequestered under Section 8 of Presidential Decree No. 885, as amended, which authorizes sequestration of the property of any person engaged in subversive activities against the government in accordance with implementing rules and regulations as may be issued by the Secretary of National Defense.

Issue: Held:

Whether or Not the 2 search warrants were validly issued and executed. In regard to the quashal of warrants that petitioners should have initially filed to the lower court, this Court takes cognizance of this petition in view of the seriousness and urgency of the constitutional Issue raised, not to mention the public interest generated by the search of the "We Forum" offices which was televised in Channel 7 and widely publicized in all metropolitan dailies. The existence of this special circumstance justifies this Court to exercise its inherent power to suspend its rules. With the contention pertaining to laches, the petitioners gave an explanation evidencing that they have exhausted other

extra-judicial efforts to remedy the situation, negating the presumption that they have abandoned their right to the possession of the seized property. On the enumerated reasons: 1. This objection may properly be considered moot and academic, as petitioners themselves conceded during the hearing on August 9, 1983, that an examination had indeed been conducted by respondent judge of Col. Abadilla and his witnesses. 2. The defect pointed out is obviously a typographical error. Precisely, two search warrants were applied for and issued because the purpose and intent were to search two distinct premises. It would be quite absurd and illogical for respondent judge to have issued two warrants intended for one and the same place. 3. Section 2, Rule 126, of the Rules of Court, does not require that the property to be seized should be owned by the person against whom the search warrant is directed. It may or may not be owned by him. 4. Petitioners do not claim to be the owners of the land and/or building on which the machineries were placed. This being the case, the machineries in question, while in fact bolted to the ground, remain movable property susceptible to seizure under a search warrant. 5. The broad statements in the application and joint affidavit are mere conclusions of law and does not satisfy the requirements of probable cause. Deficient of such particulars as would justify a finding of the existence of probable cause, said allegation cannot serve as basis for the issuance of a search warrant and it was a grave error

for respondent judge to have done so. In Alvarez v. Court of First Instance, this Court ruled that "the oath required must refer to the truth of the facts within the personal knowledge of the petitioner or his witnesses, because the purpose thereof is to convince the committing magistrate, not the individual making the affidavit and seeking the issuance of the warrant, of the existence of probable cause." Another factor which makes the search warrants under consideration constitutionally objectionable is that they are in the nature of general warrants. The description of the articles sought to be seized under the search warrants in question are too general. With regard to the respondents invoking PD 885, there is an absence of any implementing rules and regulations promulgated by the Minister of National Defense. Furthermore, President Marcos himself denies the request of military authorities to sequester the property seized from petitioners. The closure of the premises subjected to search and seizure is contrary to the freedom of the press as guaranteed in our fundamental law. The search warrants are declared null and void.
PEOPLE V. BURGOS - 144 SCRA 1

FACTS:
Due to an information given by a person, who allegedly was being forcibly recruited by accused to the NPA, the members of the Constabulary went to the house of accused, asked about his firearm and documents connected to subversive activities. Accused pointed to where his firearm was as well as his other documents allegedly.

HELD:
The right of the person to be secure against any unreasonable seizure of his body and any deprivation of liberty is a most basic and fundamental one. The statute or rule, which

allows exceptions to the requirement of warrants of arrest is strictly construed. Any exception must clearly fall within the situations when securing a warrant would be absurd or is manifestly unnecessary as provided by the Rule. We cannot liberally construe the rule on arrests without warrant or extend its application beyond the cases specifically provided by law. To do so would infringe upon personal liberty and set back a basic right so often violated and so deserving of full protection. G.R.No. 74869 July 6, 1988 PEOPLE OF THE PHILIPPINES, plaintiff-appellee, vs. IDEL AMINNUDIN y AHNI, defendant-appellant. The Solicitor General for plaintiff-appellee. Herminio T. Llariza counsel de-officio for defendant-appellant.

CRUZ, J.: The accused-appellant claimed his business was selling watches but he was nonetheless arrested, tried and found guilty of illegally transporting marijuana. The trial court, disbelieving him, held it was high time to put him away and sentenced him to life imprisonment plus a fine of P20,000.00. 1 Idel Aminnudin was arrested on June 25, 1984, shortly after disembarking from the M/V Wilcon 9 at about 8:30 in the evening, in Iloilo City. The PC officers who were in fact waiting for him simply accosted him, inspected his bag and finding what looked liked marijuana leaves took him to their headquarters for investigation. The two bundles of suspect articles were confiscated from him and later taken to the NBI laboratory for examination. When they were verified as marijuana leaves, an information for violation of the Dangerous Drugs Act was filed against him. 2 Later, the information was amended to include Farida Ali y Hassen, who had also been arrested with him that same evening and likewise investigated. 3 Both were arraigned and pleaded not guilty. 4 Subsequently, the fiscal filed a motion to dismiss the charge against Ali on the basis of a sworn statement of the arresting officers absolving her after a 'thorough investigation." 5 The motion was granted, and trial proceeded only against the accused-appellant, who was eventually convicted . 6 According to the prosecution, the PC officers had earlier received a tip from one of their informers that the accused-appellant was on board a vessel bound for Iloilo City and was carrying marijuana. 7 He was Identified by name. 8 Acting on this tip, they waited for him in the evening of June 25, 1984, and approached him as he descended from the gangplank after the informer had pointed to him. 9 They detained him and inspected the bag he was carrying. It was found to contain three kilos of what were later analyzed as

marijuana leaves by an NBI forensic examiner, 10 who testified that she conducted microscopic, chemical and chromatographic tests on them. On the basis of this finding, the corresponding charge was then filed against Aminnudin. In his defense, Aminnudin disclaimed the marijuana, averring that all he had in his bag was his clothing consisting of a jacket, two shirts and two pairs of pants. 11 He alleged that he was arbitrarily arrested and immediately handcuffed. His bag was confiscated without a search warrant. At the PC headquarters, he was manhandled to force him to admit he was carrying the marijuana, the investigator hitting him with a piece of wood in the chest and arms even as he parried the blows while he was still handcuffed. 12 He insisted he did not even know what marijuana looked like and that his business was selling watches and sometimes cigarettes. 13 He also argued that the marijuana he was alleged to have been carrying was not properly Identified and could have been any of several bundles kept in the stock room of the PC headquarters. 14 The trial court was unconvinced, noting from its own examination of the accused that he claimed to have come to Iloilo City to sell watches but carried only two watches at the time, traveling from Jolo for that purpose and spending P107.00 for fare, not to mention his other expenses. 15 Aminnudin testified that he kept the two watches in a secret pocket below his belt but, strangely, they were not discovered when he was bodily searched by the arresting officers nor were they damaged as a result of his manhandling. 16 He also said he sold one of the watches for P400.00 and gave away the other, although the watches belonged not to him but to his cousin, 17 to a friend whose full name he said did not even know. 18 The trial court also rejected his allegations of maltreatment, observing that he had not sufficiently proved the injuries sustained by him. 19 There is no justification to reverse these factual findings, considering that it was the trial judge who had immediate access to the testimony of the witnesses and had the opportunity to weigh their credibility on the stand. Nuances of tone or voice, meaningful pauses and hesitation, flush of face and dart of eyes, which may reveal the truth or expose the lie, are not described in the impersonal record. But the trial judge sees all of this, discovering for himself the truant fact amidst the falsities. The only exception we may make in this case is the trial court's conclusion that the accused-appellant was not really beaten up because he did not complain about it later nor did he submit to a medical examination. That is hardly fair or realistic. It is possible Aminnudin never had that opportunity as he was at that time under detention by the PC authorities and in fact has never been set free since he was arrested in 1984 and up to the present. No bail has been allowed for his release. There is one point that deserves closer examination, however, and it is Aminnudin's claim that he was arrested and searched without warrant, making the marijuana allegedly found in his possession inadmissible in evidence against him under the Bill of Rights. The decision did not even discuss this point. For his part, the Solicitor General dismissed this after an all-too-short argument that the arrest of Aminnudin was valid

because it came under Rule 113, Section 6(b) of the Rules of Court on warrantless arrests. This made the search also valid as incidental to a lawful arrest. It is not disputed, and in fact it is admitted by the PC officers who testified for the prosecution, that they had no warrant when they arrested Aminnudin and seized the bag he was carrying. Their only justification was the tip they had earlier received from a reliable and regular informer who reported to them that Aminnudin was arriving in Iloilo by boat with marijuana. Their testimony varies as to the time they received the tip, one saying it was two days before the arrest, 20 another two weeks 21 and a third "weeks before June 25." 22 On this matter, we may prefer the declaration of the chief of the arresting team, Lt. Cipriano Querol, Jr., who testified as follows:
Q You mentioned an intelligence report, you mean with respect to the coming of Idel Aminnudin on June 25, 1984? A Yes, sir. Q When did you receive this intelligence report? A Two days before June 25, 1984 and it was supported by reliable sources. Q Were you informed of the coming of the Wilcon 9 and the possible trafficking of marijuana leaves on that date? A Yes, sir, two days before June 25, 1984 when we received this information from that particular informer, prior to June 25, 1984 we have already reports of the particular operation which was being participated by Idel Aminnudin. Q You said you received an intelligence report two days before June 25, 1984 with respect to the coming of Wilcon 9? A Yes, sir. Q Did you receive any other report aside from this intelligence report? A Well, I have received also other reports but not pertaining to the coming of Wilcon 9. For instance, report of illegal gambling operation. COURT: Q Previous to that particular information which you said two days before June 25, 1984, did you also receive daily report regarding the activities of Idel Aminnudin A Previous to June 25, 1984 we received reports on the activities of Idel Aminnudin. Q What were those activities? A Purely marijuana trafficking. Q From whom did you get that information? A It came to my hand which was written in a required sheet of information, maybe for security reason and we cannot Identify the person. Q But you received it from your regular informer?

A Yes, sir. ATTY. LLARIZA: Q Previous to June 25, 1984, you were more or less sure that Idel Aminnudin is coming with drugs? A Marijuana, sir. Q And this information respecting Idel Aminnudin's coming to Iloilo with marijuana was received by you many days before you received the intelligence report in writing? A Not a report of the particular coming of Aminnudin but his activities. Q You only knew that he was coming on June 25,1984 two days before? A Yes, sir. Q You mean that before June 23, 1984 you did not know that minnudin was coming? A Before June 23,1984, I, in my capacity, did not know that he was coming but on June 23, 1984 that was the time when I received the information that he was coming. Regarding the reports on his activities, we have reports that he was already consummated the act of selling and shipping marijuana stuff. COURT: Q And as a result of that report, you put him under surveillance? A Yes, sir. Q In the intelligence report, only the name of Idel Aminnudin was mentioned? A Yes, sir. Q Are you sure of that? A On the 23rd he will be coming with the woman. Q So that even before you received the official report on June 23, 1984, you had already gathered information to the effect that Idel Aminnudin was coming to Iloilo on June 25, 1984? A Only on the 23rd of June. Q You did not try to secure a search warrant for the seizure or search of the subject mentioned in your intelligence report? A No, more. Q Why not? A Because we were very very sure that our operation will yield positive result. Q Is that your procedure that whenever it will yield positive result you do not need a search warrant anymore? A Search warrant is not necessary.
23

That last answer is a cavalier pronouncement, especially as it comes from a mere lieutenant of the PC. The Supreme Court cannot countenance such a statement. This is still a government of laws and not of men. The mandate of the Bill of Rights is clear:
Sec. 2. The right of the people to be secure in their persons, houses, papers and effects against unreasonable searches and seizures of whatever nature and for any purpose shall be inviolable, and no search warrant or warrant of arrest shall issue except upon probable cause to be determined personally by the judge after examination under oath or affirmation of the complainant and the witnesses he may produce, and particularly describing the place to be searched and the persons or things to be seized.

In the case at bar, there was no warrant of arrest or search warrant issued by a judge after personal determination by him of the existence of probable cause. Contrary to the averments of the government, the accused-appellant was not caught in flagrante nor was a crime about to be committed or had just been committed to justify the warrantless arrest allowed under Rule 113 of the Rules of Court. Even expediency could not be invoked to dispense with the obtention of the warrant as in the case of Roldan v. Arca, 24 for example. Here it was held that vessels and aircraft are subject to warrantless searches and seizures for violation of the customs law because these vehicles may be quickly moved out of the locality or jurisdiction before the warrant can be secured. The present case presented no such urgency. From the conflicting declarations of the PC witnesses, it is clear that they had at least two days within which they could have obtained a warrant to arrest and search Aminnudin who was coming to Iloilo on the M/V Wilcon 9. His name was known. The vehicle was Identified. The date of its arrival was certain. And from the information they had received, they could have persuaded a judge that there was probable cause, indeed, to justify the issuance of a warrant. Yet they did nothing. No effort was made to comply with the law. The Bill of Rights was ignored altogether because the PC lieutenant who was the head of the arresting team, had determined on his own authority that a "search warrant was not necessary." In the many cases where this Court has sustained the warrantless arrest of violators of the Dangerous Drugs Act, it has always been shown that they were caught red-handed, as a result of what are popularly called "buy-bust" operations of the narcotics agents. 25 Rule 113 was clearly applicable because at the precise time of arrest the accused was in the act of selling the prohibited drug. In the case at bar, the accused-appellant was not, at the moment of his arrest, committing a crime nor was it shown that he was about to do so or that he had just done so. What he was doing was descending the gangplank of the M/V Wilcon 9 and there was no outward indication that called for his arrest. To all appearances, he was like any of the other passengers innocently disembarking from the vessel. It was only when the informer pointed to him as the carrier of the marijuana that he suddenly became suspect and so subject to apprehension. It was the furtive finger that triggered his arrest. The Identification by the informer was the probable cause as determined by the officers (and

not a judge) that authorized them to pounce upon Aminnudin and immediately arrest him. Now that we have succeeded in restoring democracy in our country after fourteen years of the despised dictatorship, when any one could be picked up at will, detained without charges and punished without trial, we will have only ourselves to blame if that kind of arbitrariness is allowed to return, to once more flaunt its disdain of the Constitution and the individual liberties its Bill of Rights guarantees. While this is not to say that the accused-appellant is innocent, for indeed his very own words suggest that he is lying, that fact alone does not justify a finding that he is guilty. The constitutional presumption is that he is innocent, and he will be so declared even if his defense is weak as long as the prosecution is not strong enough to convict him. Without the evidence of the marijuana allegedly seized from Aminnudin, the case of the prosecution must fall. That evidence cannot be admitted, and should never have been considered by the trial court for the simple fact is that the marijuana was seized illegally. It is the fruit of the poisonous tree, to use Justice Holmes' felicitous phrase. The search was not an incident of a lawful arrest because there was no warrant of arrest and the warrantless arrest did not come under the exceptions allowed by the Rules of Court. Hence, the warrantless search was also illegal and the evidence obtained thereby was inadmissible. The Court strongly supports the campaign of the government against drug addiction and commends the efforts of our law-enforcement officers against those who would inflict this malediction upon our people, especially the susceptible youth. But as demanding as this campaign may be, it cannot be more so than the compulsions of the Bill of Rights for the protection of the liberty of every individual in the realm, including the basest of criminals. The Constitution covers with the mantle of its protection the innocent and the guilty alike against any manner of high- handedness from the authorities, however praiseworthy their intentions. Those who are supposed to enforce the law are not justified in disregarding the rights of the individual in the name of order. Order is too high a price for the loss of liberty. As Justice Holmes, again, said, "I think it a less evil that some criminals should escape than that the government should play an ignoble part." It is simply not allowed in the free society to violate a law to enforce another, especially if the law violated is the Constitution itself. We find that with the exclusion of the illegally seized marijuana as evidence against the accused-appellant, his guilt has not been proved beyond reasonable doubt and he must therefore be discharged on the presumption that he is innocent. ACCORDINGLY, the decision of the trial court is REVERSED and the accusedappellant is ACQUITTED. It is so ordered.

G.R. No. 72301

July 31, 1987

ROLANDO PONSICA, ROGELIO ARNAIZ, FR. NICO HOFSTEDE, BERNARDINO PATIGAS, ZACHEUS ROJO, GODOFREDO RETIRACTON, LORETO BERING, ROGELIO ARTAJO, JOVITO MARATAS, CARLOS ALLEONES, MILO PICCIO, ADOLFO MAGUATE, GONZALO CASTILLA, RONNIE DESUYO, FEDERICO AYO, ROLANDO BERNABE, MARIANO REYES, DANIEL GEMPESALA, WILFREDO SARATOBIAS, MILDRED SAGUIRE, CRESENCIO ENCARGUEZ, JOHN BUSTAMANTE, JOHN DOE and RICHARD DOE, petitioners, vs. HON. EMILIO M. IGNALAGA, Presiding Judge, Municipal Trial Court of Escalante, Negros Occidental, MAYOR BRAULIO LUMAYNO, CAPT. MODESTO SAN-SON, CAPT. RAFAEL JUGAN, respondents. NARVASA, J.: The chief issue raised by the petitioners in this case is whether or not Section 143 of the Local Government Code1 granting power to the municipal mayor to conduct preliminary investigations and order the arrest of the accused, was repealed by the 1985 Rules on Criminal Procedure promulgated by this Court; and is, in addition, unconstitutional as vesting the power to conduct preliminary investigations in an official who cannot be deemed a "neutral and detached magistrate" within the contemplation of Section 3, Article IV of the 1973 Constitution. The issue is hereby resolved adversely to the petitioners, with the stressed qualification that the mayor's power to order arrest ceased to exist as of February 2, 1987 when the new Constitution was ratified by the Filipino people, and that, in any event, the investigation actually conducted by respondent mayor in the case at bar was fatally defective. Shortly after noon on September 20, 1985, an attempt was made by firemen and soldiers to disperse a crowd of demonstrators massed in front of the Municipal Building of Escalante, Negros Occidental, with the use first, of water spewed from fire hoses, and later, tear gas. Eventually there was gunfire. Within moments, rallyists lay dead on and by the National Road. The fatalities numbered fifteen (15), according to the military officers; twenty-nine (29), according to the demonstrators. In the afternoon of that day, Escalante Town Mayor Braulio Lumayno, in view of the absence of the Municipal Circuit Court Judge (Emilio Ignalaga), took cognizance of a complaint filed by the Military Station Commander charging some of the rallyists with the felony of inciting to sedition, and after avowedly conducting an investigation of the witnesses presented by the complainant, issued an order for the arrest of certain of the demonstrators. His order reads as follows: It appearing that the Presiding Municipal Circuit Court Judge, Hon. Emilio M. Ignalaga, is on official leave of absence and, in the interest of justice, the undersigned has to urgently act on the complaint filed by the Station Commander, against the above-named accused for "INCITING TO SEDITION" and, on the basis of the evidence submitted after a searching question and answer were conducted and, being satisfied that said crime

has been committed, in order not to frustrate the ends of justice, it is necessary that the above-named accused be placed under custody. Let therefore, a warrant of arrest be issued for said above-named accused. Bail recommended: P12,000.00. 2 In the record of the Court a quo appear the following inter alia: 1) Complaint for "Inciting to Sedition" (RPC 142, as and by PD 183 and PD 1974), signed by a Capt. Jugan, and sworn to before Mayor Lumayno on Sept. 20, 1985, bearing the stamped notation of filing with the MTC: "9/24/85, 4:00 PM:" 3 2) 2-page Affidavit of Capt. Sanson, dated, and sworn to before Actg.. City Fiscal Abros (Cadiz City) on Sept. 20, 1985; 4 3) 3-page document, "Searching Questions and Answers" signed by Capt. Sanson, dated. and sworn to before Mayor Lumayno on, September 20, 1985; 5 4) 3-page sworn statement of Godofredo Hoyo-A y Jayme, General Manager of the Balintawak-Escalante Water District; 6 5) Affidavit of Leopoldo Villalon; 7 6) Affidavit of Elpidio Carbajosa; 8 and 7) Affidavit of Eduardo Flores. 9 The gist of the testimony of Capt. Sanson and the other affiants is that on September 20, 1985, the demonstrators, numbering "about 1,000, " had blockaded the main highway in front of the Escalante Municipal Building, by massing themselves on the road as well as by piling stones, coconut trunks and pieces of wood in the middle of the highway. They were also "shouting invectives, seditious and scurrilous words against the government." Negotiations with Ponsica, Chairman of the Escalante Chapter of "BAYAN" (Bagong Alyansang Makabayan), to have the road cleared having been unavailing, firemen on firetrucks began to train a "torrent of water" from their fire hoses on the demonstrators. The rallyists retaliated by hurling stones at the firemen. One of them "approached the security of the firetruck and stabbed him." Others climbed aboard the trucks and tried to grab the firehoses and firearms of the officers. At this point, on Capt. Sanson's orders, his "back-up teams" of soldiers commenced to throw tear gas at the crowd. One of the demonstrators picked up a tear gas canister and hurled it back at the soldiers. At the same time gunfire from "different assorted firearms" emanated from the rallyists; and some of the shots hit the blinker of a firetruck and the headlight of another. The soldiers shot back. This exchange of gunfire resulted in "fourteen (14) demonstrators killed on the spot. " Recovered at the scene were a rifle; a U.S. 45 cal. pistol; 2 "homemade" pistols; 14 steel arrows and 4 assorted slings; 30 assorted knives; a "pogakhang" with 2 live cartridges; 2 grenades; and several empty shells of different caliber. Hours later, another corpse, Identified as that of a demonstrator, too, was brought to the PC Headquarters.

The petitioners however give a different version of the facts. What happened, according to them, was that at 9 o'clock in the morning on that day, a group of demonstrators, "composed mostly of laid-off sugar field workers," gathered in front of Escalante Municipal Hall "in the exercise of their constitutionally guaranteed right to freedom of expression and to assemble peacefully to petition the government for redress of grievances."10 About an hour afterwards, fire trucks arrived one after another, as well as jeeploads of soldiers and CHDF members, in full combat gear, Shortly after noon, after "going thru the motions of negotiating with the demonstrators," the military officers ordered the crowd to disperse; but without warning, fire hoses were trained on and sprayed water at the demonstrators. When the rallyists did not budge, tear gas canisters were thrown at them. A demonstrator picked up a canister and threw it at an "empty space in the plaza" The soldier and CHDF members thereupon fired indiscriminately at the crowd, killing 29 and injuring at least 30 persons.11 After the Mayor had referred the case to Municipal Trial Court Judge Ignalaga on September 24, 1985,12 an "URGENT MOTION TO QUASH WARRANT OF ARREST" was filed on September 26, 1985 by petitioners' counsel on the ground that a mayor no longer has authority to conduct preliminary investigations or issue warrants of arrest that authority having been "withheld in the 1985 New Rules on Criminal Procedure."13 This was opposed by the Station Commander14 who invoked Section 143 of the Local Government Code providing that "(i)n case of temporary absence of the Judge assigned to the municipality, the mayor may conduct the preliminary examination in criminal cases when, in his opinion the investigation cannot be delayed without prejudice to the interest of justice." A reply was filed by the petitioners after their receipt of the opposition "only last October 8, 1985."15 They contended that the "power of the municipal mayor to conduct preliminary investigation and issue a warrant of arrest under the 1964 Revised Rules of Court** (had been) impliedly repealed by the 1985 New Rules on Criminal Procedure;" that "the 1985 New Rules on Criminal Procedure being a special law, controls over provisions of the Local Government Code (BP 337, 1983), which is a general law;" and in any case, "subject t warrants of arrest should be reviewed and revoked as done without observance of legal requisites." By Resolution dated Oct. 11, 1985,16 the Judge confirmed the mayor's arrest order. He opined that in the absence of the judge, the mayor still has authority to conduct preliminary investigations and issue arrest warrants, since Rule 112, Sec. 2 (d), of the 1985 Rules, includes as among those authorized to conduct preliminary investigations, "Such other officers as may be authorized by law;" and the Local G government Code of 1983, Section 143, grants a town mayor authority to conduct preliminary examinations in case of the temporary absence of the judge when such investigation cannot be delayed without prejudice to the interest of justice. The Judge declared that in the case at bar, the mayor had conducted the examination personally, and having in the exercise of his discretion found probable cause, issued the warrants of arrest in question; and conceding arguendo irregularity in that the preliminary examination was conducted without according the parties the assistance of lawyers (contrary to par. 2, Sec. 143, Chap. 3, Title 2, Book 11, Local Government Code), this does not render the proceedings void because at any rate, the mayor had duly observed the uniform procedure under PD 91 (citing: People v. Paran, 52 Phil. 712; Hashim v. Boncan, 71 Phil. 216; Lino v. Fugoso, 77 Phil. 933).

It is Mayor Lumayno's order for the petitioners' arrest of September 20, 1985, and Jude Ignalaga's Order of October 11, 1985 "validating that order of arrest" that the petitioners would have this Court nullify and perpetually enjoin.17 In their petition for certiorari and prohibition filed on October 15, 1985, and their Memorandum of January 21, 1986,18 they assert that: 1) while Section 3,Rule 112 of the l964 Rules of Court,empowers the municipal mayor, "in case of temporary absence of both the municipal and the auxiliary municipal judges from the municipality, town or place wherein they exercise their jurisdiction, to make the preliminary examination in criminal cases when such examination cannot be delayed without prejudice to the interest of justice," that power has been removed from him by the 1985 New Rules on Criminal Procedure which "do not mention the Municipal Mayor as among the officers authorized to conduct preliminary investigation (Section 2, Rule 112), much less to issue an order of arrest (Section 6, Ibid.)19 2) Section 143 of BP Blg. 337 (Local Government Code) under which the order of arrest in question is presumably made to rest-cannot withstand the "constitutional test" of Section 3, Article IV,20 which safeguards the right against unreasonable searches and seizures, and requires the interposition of a "judge, or such other responsible officer as may be authorized by law," meaning "a neutral and detached magistrate competent to determine probable cause (Shadwick v. City of Tampa, 40 LW 4758; Castillo v. Jias, 62 SCRA 124; Ang Tibay v. CIR, 69 Phil. 635; Zambales Chromite, etc. v. C.A., 934 SCRA 2617);" and since a mayor is obviously not such a magistrate (U.S. v. Chadwick, 433 U.S. 197), the orders of arrest at bar are constitutionally infirm;21 moreover, said Section 143 of the Local Government Code is "merely a rule of procedure** (and is thus) deemed to have been superseded by the New Rules of Criminal Procedure;22 3) the Mayor's examination "falls short of the requirements of "searching questions and answers;" the statements of the witnesses supposedly interrogated by the mayor, are either merely conclusions of law or sterile as regards seditious utterances, hence, probable cause was non-existent23 and 4) Article 142 of the Revised Penal Code, as amended-under which the petitioners are charged is based on the US Sedition Act of 1978, which has been declared by the US Supreme Court as "repugnant to the constitutional guarantee of freedom of speech and expression (New York Times Co. v. Sullivan, 376 U.S. 254);" hence, said Article 142 is also fatally flawed and therefore, the warrant. of arrest in question was in effect issued for a "crime which in the context of the constitutionally guaranteed freedom of speech and expression does not exist."24 In his comment filed with this Court,25 Judge Ignalaga argues that 1) The validity of the preliminary examination on the basis of which the arrest warrants were issued, may not be raised for the first time in the Supreme Court, but should first be ventilated before the RTC having cognizance of the crime.26 In any case, upon the

considerations set out in his resolution of October 11, 1985, the preliminary examination in question is valid.27 2) The petitioners raise factual issues which are best left for determination by the RTC, the Supreme Court not being a trier of facts.28 3) The Mayor in fact followed the procedure in the constitution prior to issuing the arrest warrants.29 4) The constitutionality of PD 1974 should be assailed by separate petition.30 The Solicitor General at the time, Estelito Mendoza, also filed a comment on December 6, 1985.31 The comment addressed itself only to the issue of "the validity of the law authorizing municipal mayors to issue warrants of arrest and the law punishing inciting to sedition," in view of the fact that the respondents had presented their own separate comments on the petition.32 The comment points out that: 1) Section 3, Article IV, of the 1973 Constitution, which mentions a "judge, or such other responsible officer as may be authorized by law" as vested with competence to conduct preliminary investigations, is an innovation. In implementation thereof, BP Blg. 337 was enacted on February 10, 1983, empowering mayors to conduct preliminary investigations (Sec. 143). This is a recognition of the truism that the determination of probable cause is but a quasi judicial function Ocampo v. US, 58 LED 1231).33 Petitioners' American authorities are not applicable: the mayor is the highest official in the municipality; he exercises only general supervision over the police but is not directly involved in police work; the old rules precisely expressed a recognition of the capability of i mayors to determine probable cause, and the omission of mayors in the 1985 Rules simply means that the determination of officers who may be authorized to conduct preliminary investigations was deemed best left to legislation.34 2) The Philippine sedition law is not akin to the US Sedition Law; in any event our own sedition law has passed the test of constitutionality (Espuelas v. People, 90 Phil. 524).35 Solicitor General Sedfrey A. Ordonez subsequently declared that he "stands by the constitutionality of the statutes the petitioners question and, therefore, sustains the position taken by his predecessor in office;"36 this, in response to this Court's resolution dated May 15, 1986, requiring the parties to state whether supervening events had transpired materially affecting the case.37 It is clear from the outset that the issue before this Court does not involved the adjudgment of the guilt or innocence of the soldiers in the tragic and regrettable killings in front of the Escalante Town Hall in the early afternoon of that day, the twentieth of September, 1985, an event that caused a great outcry of lamentation and condemnation throughout the land. This is a matter that should be and is in fact now subject of a separate criminal proceeding. Neither is the propriety of the victims' exercise of their constitutional rights of free speech and free assembly for redress of grievances in the premises at issue here. The basic question before the Court is divorced of the

drama and the passion of those issues; it deals mainly with the dry, unexciting, but nonetheless important matter of whether or not the municipal mayor has the power to conduct preliminary investigations in the light of the 1985 amendments of the rules governing criminal procedure in the Rules of Court. The answer to the question entails a re-examination and analysis of the relevant legal provisions. The 1964 Rules of Court explicitly gave the mayor authority to conduct preliminary investigations. SEC. 3. Preliminary examination by the municipal mayor. In case of temporary absence of both the municipal and the auxiliary municipal judges from the municipality, town, or place wherein they exercise their jurisdiction, the municipal mayor shall make the preliminary examination in criminal cases when such examination cannot be delayed without prejudice to the interest of justice. He shall make a report of any preliminary examination so made to the municipal or to the auxiliary municipal judge immediately upon the return of one or the other. He shall have authority in such cases to order the arrest of the defendant and to grant him bail in the manner and cases provided for in Rule 114. 38 The Rules of 1940 contained an Identical provision, in Section 3, Rule 108. The 1985 Rules on Criminal Procedure did not reproduce this provision, and did not include the mayor in the enumeration of the officers authorized to conduct preliminary investigations, those listed being judges of municipal trial courts and municipal circuit trial courts; city or provincial fiscals and their assistants; national and regional state prosecutors; and "such other officers as may be authorized by law." 39 SEC. 6. When warrant of arrest may issue. (a) By the Regional Trial Court. Upon the filing of an information, the Regional Trial Court may issue a warrant for the arrest of the accused (b) By the Municipal Trial Court. If the municipal trial judge conducting the preliminary investigation is satisfied after an examination in writing and under oath of the complainant and his witnesses in the form of searching questions and answers, that a probable cause exists and that there is a necessity of placing the respondent under immediate custody in order not to frustrate the ends of justice, he shall issue a warrant of arrest. (n). 40 It is of course the deletion in the 1985 rules of reference to the municipal mayor as among those authorized to conduct preliminary investigations and order arrests upon which the petitioners chiefly rely as basis for their impugnation of Mayor Lumayno's preliminary investigation and order of arrest based thereon. The matter is however treated of not solely by the Rules of Court but also by the 1973 Constitution, Section 3, Article IV whereof reads:

SEC. 3. The right of the people to be secure in their persons, houses, papers, and effects against unreasonable searches and seizures of whatever nature and for any purpose shall not be violated, and no search warrant or warrant of arrest shall issue except upon probable cause to be determined by the judge, or such other responsible officer as may be authorized by law, after examination under oath or affirmation of the complainant and the witnesses he may produce, and particularly describing the place to be searched, and the persons or things to be seized. 41 Parenthetically, it may be noted that Section 3, Rule III of the 1935 Constitution mentioned only "the judge" as having power to determine probable cause and issue search and arrest warrants. The matter is further dealt with in Section 143 of Batas Pambansa Bilang 337, otherwise known as the Local Government Code, which took effect on February 10, 1983. This section evidently deems the mayor a "responsible officer" in contemplation of the cited constitutional provision, and explicitly authorizes him to conduct preliminary examination in criminal cases and order the arrest of the accused upon probable cause. SEC. 143. Authority of the Mayor to Conduct Preliminary Examination. (1) In case of temporary absence of the judge assigned to the municipalities the mayor may conduct the preliminary examination in criminal cases when, in his opinion the investigation cannot be delayed without prejudice to the interest of justice. (2) No examination shall be conducted unless the parties are assisted by lawyers. (3) In cases where he may conduct preliminary examination, the mayor shall upon probable cause after examination of witnesses, have authority order the arrest of the accused and to grant him bail in the manner and cases provided in the Revised Rules of Court and order his provisional release. (4) The mayor shall make a report of any preliminary examination so made immediately after the return of the judge assigned in the area, or upon the designation of his replacement. 42 The 1973 Constitution plainly and unmistakably grants to the legislature the power to determine which " responsible officers, " aside from judges, may issue warrants of arrest after examination under oath or affirmation of the complainant and the witnesses he may produce. And as plainly and unmistakably, the legislature, the Batas Pambansa, has in the Local Government Code made the determination that the mayor is such a "responsible officer "and has in consequence authorized him to conduct preliminary investigations in criminal cases and order the arrest of the accused upon a finding of probable cause. The first issue raised by the petitioners that in September ,1985 the mayor no longer had power to conduct preliminary investigations and issue arrest warrants43 must therefore be resolved against them. The argument that Section 143 of the Local Government Code is just "a rule of procedure merely having incorporated Rule 112, Section 2 of the old rules of Criminal

Procedure," and should therefore be deemed "superseded by the New Rules of Criminal Procedure pursuant to the power of this ** Court to promulgate rules of procedure (Article X, Section 5 151, Constitution **)"44 cannot be sustained. However superior the Supreme Court may be to the other branches of government in the realm of adjudication, its power to ordain rules of court was at the time inferior to the lawmaking power of the legislature.45 It is true that the 1935 Constitution repealed all procedural laws then in force "as statutes" and declared them to be "rules of court;" but this was only so that they could be subject to repeal or modification by the Supreme Court, which was given the power to promulgate rules of procedure,46 and has since been exercising such power by promulgating the Rules of Court of 1940 and of 1964, and the 1985 Rules on Criminal Procedure, etc. But the 1935 and 1973 Constitutions explicitly conferred on the legislature the power to repeal, alter or supplement those rules of court, although it would appear that that power is no longer granted to it by the 1987 Charter.47 Now, the conditions under which the mayor could conduct preliminary investigations are also clearly indicated by the law, to wit: 1) in case of the temporary absence of the judge assigned to the municipality; and 2) in his (the mayor's) opinion, the investigation cannot be delayed without prejudice to the interests of justice. On these occasions, the mayor may order the arrest of the accused upon a finding of probable cause after searching examination of the complainant and the witnesses the latter may present. In the case at bar, there is no showing that these conditions have not been met. Indeed, quite apart from the presumption that official duty has been regularly performed.48 and the affirmative statements of respondent official that on that fateful 20th day of September, 1985, Judge Ignalaga was in Manila"49 on official leave or absence,"50 the petitioners have not put the fulfillment of those conditions at issue at all, or the matter of whether or not the accused were "assisted by lawyers" in the course of the investigation.51 The petitioners also challenge the constitutionality of Section 143 of the Local Government Code, it being claimed that the mayor has been thereby authorized to conduct pre investigations and issue warrants of arrest, although he can not be deemed a "neutral and detached magistrate" within the contemplation of Section 3, Article IV of the 1973 Constitution.52 The competence of the Batasang Pambansa to decide and declare by statute which "responsible officers," aside from judges, should be entrusted with the authority to conduct preliminary investigations and issue arrest warrants cannot be doubted in view of the clear language of the Constitution.53 And the determination by the Batasan that a municipal mayor is one such "responsible officer" who may properly be entrusted with the function of conducting preliminary investigations and ordering arrests of suspects upon probable cause, can not be subject of judicial review, absent any indication that the legislative proceedings leading to that statutory determination are void on account of some grave cause. Certainly, the wisdom of the statute, or the validity of the reasons underlying it, or the adequacy of the statistics, facts and circumstances considered by the legislature in its enactment, are beyond the sphere of inquiry of the courts54

The Batasan was apparently of the belief that a municipal mayor could be sufficiently objective and impartial as to be relied upon to conduct preliminary investigations and issue orders of arrest in the exceptional situation when the judge assigned in the municipality was absent. The petitioners disagree. They contend that the mayor cannot in the very nature of things be "neutral and detached." The disagreement, and the fact that plausible reasons may be adduced by one side or the other on the proposition does not make the question a justiciable one. The theory advocated by the petitioners that the mayor's "deep involvement in law enforcement functions is likely to color his judgment as a trier of probable cause,"55 does not induce persuasion. In the first place the premise cannot be conceded. While it is true that the mayors do "exercise general supervision over units and elements of the INP stationed or assigned in their respective jurisdictions," they are not themselves directly involved in police work and cannot in any sense be described, as the petitioners do, as being deeply involved in law enforcement functions. And even if that "deep involvement" be conceded, it does not follow that this would necessarily preclude their assuming "the cold neutrality of an impartial judge" in conducting preliminary investigations of persons suspected of crimes. But it must be emphasized here and now that what has just been described is the state of the law as it was in September, 1985. The law has since been alter. No longer does the mayor have at this time the power to conduct preliminary investigations, much less issue orders of arrest. Section 143 of the Local Government Code, conferring this power on the mayor has been abrogated, rendered functus officio by the 1987 Constitution which took effect on February 2, 1987, the date of its ratification by the Filipino people. Section 2, Article III of the 1987 Constitution pertinently provides that "no search warrant or warrant of arrest shall issue except upon probable cause to be determined personally by the judge after examination under oath or affirmation of the complainant and the witnesses he may produce, and particularly describing the place to be searched and the person or things to be seized. " The constitutional proscription has thereby been manifested that thenceforth, the function of determining probable cause and issuing, on the basis thereof, warrants of arrest or search warrants, may be validly exercised only by judges, this being evidenced by the elimination in the present Constitution of the phrase, "such other responsible officer as may be authorized by law" found in the counterpart provision of said 1973 Constitution,56 who, aside from judges, might conduct preliminary investigations and issue warrants of arrest or search warrants. As the law now stands, the mayor may no longer conduct preliminary investigation, the authority to do so being limited under Section 2, Rule 1 1 2 of the Rules of Court to (1) provincial or city fiscals and their assistants; (2) judges of the Municipal Trial Courts and Municipal Circuit Trial Courts; (3) national and regional state prosecutors; and (d) such other officers as may be authorized by law.57 But only "the judge" may issue search and arrest warrants after due determination of probable cause.58 The petitioners' assault, on the other hand, upon the constitutionality of Article 141 of the Revised Penal Code, defining and penalizing the felony of inciting to sedition, upon the claim that it was "borrowed" from the U.S. Sedition Act of 1798 which in turn has been struck down as inconsistent with the First Amendment of the American Constitution,59 is repelled by the Solicitor General's arguments.60

In the first place, ** our law on inciting to sedition is not akin to the US Sedition Act of 1798 which was imposed on the American colonies by their British ruler. With the success of the American revolution, the 1798 Sedition Act naturally ceased to have effect as it would be utterly incongrous to punish those who sought the overthrow of the British government in America. xxx xxx xxx xxx xxx xxx

To annul our law on sedition is to give license to those who seek the application of lawless methods in the advancement of their political views. Our constitution surely does not contemplate this. Finally, the petitioners postulate that in the determination of the existence of probable cause from the constitutional aspect, it is required that: "(1) The judge (or) officer must examine the ** witnesses personally; (2) The examination must be under oath; and (3) The examination must be reduced to writing in the form of searching questions and answers. (Marinas v. Siochio, 104 SCRA 403)." In the light of these principles they contend: firstly, that the mayor's questioning of the witnesses was not "searching" enough; and secondly, that the witnesses' testimony does not establish prima facie the commission of the felony of inciting to sedition. The fact is that is shown by the record, questions about the material events were in truth propounded by Mayor Lumayno to the chief witness. Capt. Sanson;61 and no proof to the contrary has been submitted. The circumstance that the answers given by Capt. Sanson to the mayor's questions are closely reflective of the contends of his affidavit should not come as a surprise and cannot, without more, be taken as debilitating or nullifying the interrogation. The petitioner's second point is much more substantial and is decisive of the controversy. The petitioners are correct in their claim that Capt. Sanson's testimony62 does not in truth contain any facts demonstrating the actuality of the crime of inciting to sedition, which is the crime for which arrest warrants were issued by Mayor Lumayno. The facts recited by Capt. Sanson may perhaps warrant a charge of traffic obstruction, or grave coercion, or malicious mischief, or illegal possession of firearms or deadly weapons, or maybe, attempted murder or homicide. But it is barren of facts to support an accusation of inciting to sedition. The other evidence on record exhibits the same barreness. Two witnesses, Godofredo Hoyo-A and Elpidio Carbajosa, advert in their affidavits63 to the shouting of "seditious words," etc. Hoyo-A is quoted as deposing that "the demonstrators were already unruly ** (and kept) on shouting anti-government slogans, ** invectiues and seditious words against the government,"64 that certain of the petitioners were Identified as being "against the government, ** and using the (Mt. Carmel) School in doctrining (sic) the students against the government,"65 that petitioner Rogelio Arnaiz had delivered a speech in which he had said,

"Rumpagon ang dictatorial na regimen ni Pres. Marcos, pamatukan ang NSL, ibagsak ang military kagwalaon ang CHDF, suklan nato ang gobyerno," a woman speaker had said the same thing, and a third speaker, petitioner Daniel Gempesala, also stated: Lumpagon and dictatorial na regimen ni Marcos, kag ibagsak ang Gobyerno;"66 and they had elicited a response from the people there who had raised their right hands and repeated the shouted "slogan."67 Carbajosa in his turn declared that the demonstrators ** were shouting for trouble and ready to make revolution with the use of arms."68 Another affiant, Eduardo Flores, stated that the demonstrators were unruly, "tumultuously shouting seditious words against the government and shouting for revolution."69 The sworn statement of the only other witness, Leopoldo Villalon,70 is totally innocuous as far as proving the elements of inciting to sedition is concerned. The evidence can not justify the action taken by the respondent Mayor and Judge. The Court thus declares as sorely inadequate and mortally defective the avowed evidentiary foundation for Mayor Lumayno's finding of probable cause respecting the commission by the petitioners of the crime of inciting to sedition. The affiants declarations in their sworn statements which might otherwise be pertinent to the offense, are generalities, mere conclusions of their, not positive averments of particular facts within their personal knowledge. 'They do not Identify the specific persons supposed to have perpetrated the crime charged, except two. But even the Identification of these two is of no moment. For except as regards Capt. Sanson, whose testimony, to repeat, is in any case ineffectual to prove the precise offense ascribed to the petitioners, there had been no searching interrogation by Mayor Lumayno of the witnesses as required by the Constitution. Hence, whatever credit could possibly have been accorded to the affidavit of Hoyo-A which ventures to quote the exact words allegedly shouted by petitioners Arnaiz and Gempesala, and an unidentified woman was thereby effectively foreclosed. WHEREFORE, the writs of certiorari and prohibition are granted. The order of respondent Mayor Lumayno issued on September 20, 1985 and the resolution promulgated by respondent Judge Ignalaga on October 11, 1985 are annulled and set aside, and the respondents are perpetually forbidden to enforce or in any way implement the orders for the arrest of any of the petitioners. No costs. ROGELIO ABERCA, et al. vs. FABIAN VER, et al. L-69866 April 15, 1988 FACTS: Sometime in the early 1980s, various Intelligence units of the AFP known as Task Force Makabansa (TFM) were ordered by respondents then Maj. Gen. Fabian Ver to conduct pre-emptive strikes against known communist-terrorist (CT) underground houses in view of increasing reports about CT plans to sow disturbances in Metro Manila. In compliance thereof,

the TFM raided several places, employing in most cases defectively issued judicial search warrants. During these raids, certain members of the raiding TFM confiscated a number of purely personal items belonging to the 20 petitioners. Petitioners were arrested without proper arrest warrants issued by the courts. For some period after their arrest, they were arrested without denied visits of relatives and lawyers; interrogated in violation of their rights to silence and counsel, through threats, torture and other forms of violence in order to obtain incriminatory information or confessions and in order to punish them. Plaintiffs then filed an action for damages before the RTC of Quezon City against respondents-officers of the AFP headed by Ver. Respondents, in their motion to dismiss, claimed that (1) the wrti of habeas corpus was suspended, thus giving credence to petitioners detention; (2) respondents were immune from liability for acts done in the performance of their official duties, and that (3) the complaint did not state a cause of action against respondents. On November 8, 1983, the RTC granted the motion to dismiss the case. A motion to set aside the order dismissing the complaint, and a supplemental motion for reconsideration were filed by petitioners. On May 11, 1984, the trial court, without acting on the motion to set aside the Order of Nov. 8, 1983, declared the finality of said Order against petitioners. After their motion for reconsideration was denied by the RTC, petitioners then filed the instant petition for certiorari, on March 15, 1985, seeking to annul and set aside the respondent courts resolutions and order. ISSUES: (1) Whether or not the suspension of the privilege of the writ of habeas corpus bars a civil action for damages for illegal searches conducted by military personnel and other violations of rights and liberties guaranteed under the Constitution; (2) Whether or not respondents may invoke state immunity from suit for acts done in the performance of official duties and functions; (3) Whether or not a superior officer, under the notion of respondeat superior, be answerable for damages jointly and severally with his subordinates, to the person whose constitutional rights and liberties have been violated. HELD: (1) The suspension of the privilege of the writ of habeas corpus (PWHC) does not destroy petitioners right and cause of action for damages for illegal arrest and detention and other violations of their constitutional rights. The suspension does not render valid an otherwise illegal arrest or detention. What is suspended is merely the right of the individual to seek release from detention through the writ of habeas corpus as a speedy means of obtaining his liberty. Moreover, as pointed out by petitioners, their right and cause of action for damages are explicitly recognized in PD 1755 which amended Art. 1146 of the Civil Code by adding the following text: However, when the action (for injury to the rights of the plaintiff or for quasi-

delict) arises from or out of any act, activity or conduct of any public officer involving the exercise of powers or authority arising from martial law including the arrest, detention and/or trial of the plaintiff, the same must be brought within one year. Even assuming that the suspension of the PWHC suspends petitioners right of action for damages for illegal arrest and detention, it does not and cannot suspend their rights and causes of action for injuries suffered because of respondents confiscation of their private belongings, the violation of their right to remain silent and to counsel and their right to protection against unreasonable searches and seizures and against torture and other cruel and inhuman treatment. The question became moot and academic since the suspension of the PWHC had been lifted with the issuance of then Pres. Corazon Aquino of Proclamation No. 2 on March 25, 1986. (2) It may be that the respondents, as members of the AFP, were merely responding to their duties, as they claim, to prevent or suppress lawless violence, insurrection, rebellion and subversion in accordance with Proclamation No. 2054 of Pres. Marcos, despite the lifting of Martial Law on January 27, 1981, and in pursuance of such objective, to launch pre-emptive strikes against alleged CT underground houses. But this cannot be construed as a blanket license or roving commission untrammeled by any constitutional restraint, to disregard or transgress upon the rights and liberties of the individual citizen enshrined and protected by the Constitution. Article 32 of the Civil Code, which renders any public officer or employees, or any private individual, liable in damages for violating the constitutional rights and liberties of another, does not exempt the respondents from responsibility. Only judges are excluded from liability under the said article, provided their acts or omissions do not constitute a violation of the Revised Penal Code or other penal statute. This is not say that military authorities are restrained from pursuing their assigned task or carrying out their mission with vigor, to protect the Philippines from its enemies, whether of the left or of the right, or from within or without, seeking to destroy or subvert our democratic institutions and imperil their very existence. What is meant is that in carrying out their task and mission, constitutional and legal safeguards must be observed; otherwise, the very fabric of our faith will start to unravel. In the battle of competing ideologies, the struggle of mind is just as vital as the struggle of arms. The linchpin in that psychological struggle is faith in the rule of law. Once that faith is lost or compromised, the struggle may well be abandoned. (3) The doctrine of respondeat superior is not applicable in this case. It has been generally limited in its application to principal and agent or to master and servant relationships. No such relationship exists superiors of the military and their subordinates. However, the decisive factor in this case is the language of Art. 32, Civil Code; the law speaks of an officer or employee or person directly or indirectly responsible for the violation of the constitutional rights and liberties of another. Thus, it is not the actor alone who must answer for damages under Art. 32; the person indirectly responsible has also to answer for the damages or injury

caused to the aggrieved party. Art. 32 makes the persons who are directly as well as indirectly responsible for the transgression joint tortfeasors.
Valmonte v. De Villa, 170 SCRA 256 (1989) F: On 1/20/87, the NCRDC was activated w/ the mission of conducting security operations w/in its area or responsibility and peripheral areas, for the purpose of establishing an effective territorial defense, maintaining peace and order, and providing an atmosphere conducive to the social, economic and political dev''t of the NCR. As part of its duty to maitain peace and order, the NCRDC installed checkpoints in various parts of Valenzuela and MM. Petitioners aver that, bec. of the institution of said checkpoints, the Valenzuela residents are worried of being harassed and of their sarety being placed at the arbitrary, capricious and whimsical disposition of the military manning the checkpoints, considering that their cars and vehicles are being subjected to regular searches and check-ups, especially at night or at dawn, w/o a SW and/ or court order. Their alleged fear for their safety increased when Benjamin Parpon, was gaunned down allegedly in cold blood by members of the NCRDC for ignoring and/ or continuing to speed off inspite of warning shots fired in the air. HELD: Petitioner''s concern for their safety and apprehension at being harassed by the military manning the checkpoints are not sufficient grounds to declare the checkpoints per se, illegal. No proof has been presented before the Court to show that, in the course of their routine checks, the military, indeed, committed specific violations of petitioners'' rights against unlawful search and seizure of other rights. The constitutional right against unreasonable searches and seizures is a personal right invocable only by those whose rights have been infringed, or threatened to be infringed. Not all searches and seizures are prohibited. Those w/c are reasonable are not forbidden. The setting up of the questioned checkpoints may be considered as a security measure to enable the NCRDC to pursue its mission of establishing effective territorial defense and maintaining peace and order for the benfit of the public. Checkpoints may not also be regarded as measures to thwart plots to destabilize the govt, in the interest of public security. Between the inherent right of the state to protect its existence and promote public welfare and an individual''s right against a warrantless search w/c is, however, reasonably conducted, the former should prevail. True, the manning of checkpoints by the military is susceptible of abuse by the military in the same manner that all governmental power is susceptible of abuse. But, at the cost of occasional inconveninece, discomfort and even irritation to the citizen, the checkpoints during these abnormal times, when conducted w/in reasonable limits, are part of the price we pay for an orderly society and a peaceful community.

** Valmonte v De Villa 185 SCRA 665 (1989)F:

On 1/20/87, the NCRDC was activated w/ the mission of conducting security operations w/in its area or responsibility and peripheral areas, for the purpose of establishing an effective territorial defense, maintaining peace and order, and providing an atmosphere conducive to the social, economic and political dev't of the NCR. As part of its duty to maitain peace and order, the NCRDC installed checkpoints in various parts of Valenzuela and MM. Petitioners aver that, bec. of the institution of said checkpoints, the Valenzuela residents are worried of being harassed and of their sarety being placed at the arbitrary, capricious and whimsical disposition of the military manning the checkpoints, considering that their cars and vehicles are being subjected to regular searches and check-ups, especially at night or at dawn, w/o a SW and/ or court order. Their alleged fear for their safety increased when Benjamin Parpon, was gaunned down allegedly in cold

blood by members of the NCRDC for ignoring and/ or continuing to speed off inspite of warning shots fired in the air. HELD: Petitioner's concern for their safety and apprehension at being harassed by the military manning the checkpoints are not sufficient grounds to declare the checkpoints per se, illegal. No proof has been presented before the Court to show that, in the course of their routine checks, the military, indeed, committed specific violations of petitioners' rights against unlawful search and seizure of other rights. The constitutional right against unreasonable searches and seizures is a personal right invocable only by those whose rights have been infringed, or threatened to be infringed. Not all searches and seizures are prohibited. Those w/c are reasonable are not forbidden. The setting up of the questioned checkpoints may be considered as a security measure to enable the NCRDC to pursue its mission of establishing effective territorial defense and maintaining peace and order for the benfit of the public. Checkpoints may not also be regarded as measures to thwart plots to destabilize the govt, in the interest of public security. Between the inherent right of the state to protect its existence and promote public welfare and an individual's right against a warrantless search w/c is, however, reasonably conducted, the former should prevail. True, the manning of checkpoints by the military is susceptible of abuse by the military in the same manner that all governmental power is susceptible of abuse. But, at the cost of occasional inconveninece, discomfort and even irritation to the citizen, the checkpoints during these abnormal times, when conducted w/in reasonable limits, are part of the price we pay for an orderly society and a peaceful community.

** People v Malmstedt 198 SCRA 401 (1991)


Facts: In an information filed against the accused- appellant Mikael Malmstead was charged before the RTC of La Trinidad, Benguet, for violation of Section 4, Art. II of Republic Act 6425, as amended, otherwise known as the Dangerous Drugs Act of 1972, as amended. Accused Mikael Malmstedt, a Swedish national, entered the Philippines for the third time in December 1988 as a tourist. He had visited the country sometime in 1982 and 1985. In the evening of 7 May 1989, accused left for Baguio City. Upon his arrival thereat in the morning of the following day, he took a bus to Sagada and stayed in that place for two (2) days. Then in the 7 in the morning of May 11, 1989, the accused went to Nangonogan bus stop in Sagada. At about 8: 00 o'clock in the morning of that same day (11 May 1989), Captain Alen Vasco, the Commanding Officer of the First Regional Command (NARCOM) stationed at Camp Dangwa, ordered his men to set up a temporary checkpoint at Kilometer 14, Acop, Tublay, Mountain Province, for the purpose of checking all vehicles coming from the Cordillera Region. The order to establish a checkpoint in the said area was prompted by persistent reports that vehicles coming from Sagada were transporting marijuana and other prohibited drugs. Moreover, information was received by the Commanding Officer of NARCOM, that same morning that a Caucasian coming from Sagada had in his possession prohibited drugs. The group composed of seven (7) NARCOM officers, in coordination with Tublay Police Station, set up a checkpoint at the designated area at about 10:00 o'clock in the morning and inspected all vehicles coming from the Cordillera Region. The two (2) NARCOM officers started their inspection from the front going towards the rear of the bus. Accused who was the sole foreigner riding the bus was seated at the rear thereof.

During the inspection, CIC Galutan noticed a bulge on accused's waist. Suspecting the bulge on accused's waist to be a gun, the officer asked for accused's passport and other identification papers. When accused failed to comply, the officer required him to bring out whatever it was that was bulging on his waist. The bulging object turned out to be a pouch bag and when accused opened the same bag, as ordered, the officer noticed four (4) suspicious-looking objects wrapped in brown packing tape, prompting the officer to open one of the wrapped objects. The wrapped objects turned out to contain hashish, a derivative of marijuana. Thereafter, accused was invited outside the bus for questioning. But before he alighted from the bus, accused stopped to get two (2) travelling bags from the luggage carrier. Upon stepping out of the bus, the officers got the bags and opened them. A teddy bear was found in each bag. Feeling the teddy bears, the officer noticed that there were bulges inside the same which did not feel like foam stuffing. It was only after the officers had opened the bags that accused finally presented his passport. Accused was then brought to the headquarters of the NARCOM at Camp Dangwa, La Trinidad, Benguet for further investigation. At the investigation room, the officers opened the teddy bears and they were found to also contain hashish. Representative samples were taken from the hashish found among the personal effects of accused and the same were brought to the PC Crime Laboratory for chemical analysis. In the chemistry report, it was established that the objects examined were hashish. a prohibited drug which is a derivative of marijuana. Thus, an information was filed against accused for violation of the Dangerous Drugs Act. ACCUSEDS DEFENSE During the arraignment, accused entered a plea of "not guilty." For his defense, he raised the issue of illegal search of his personal effects. He also claimed that the hashish was planted by the NARCOM officers in his pouch bag and that the two (2) travelling bags were not owned by him, but were merely entrusted to him by an Australian couple whom he met in Sagada. He further claimed that the Australian couple intended to take the same bus with him but because there were no more seats available in said bus, they decided to take the next ride and asked accused to take charge of the bags, and that they would meet each other at the Dangwa Station. The trial court found the guilt of the accused Mikael Malmstedt established beyond reasonable doubt. Seeking the reversal of the decision of the trial court finding him guilty of the crime charged, accused argues that the search of his personal effects was illegal because it was made without a search warrant and, therefore, the prohibited drugs which were discovered during the illegal search are not admissible as evidence against him.

Issue: Whether or Not the contention of the accused is valid, and therefore the RTC ruling be reversed.

Held: The Constitution guarantees the right of the people to be secure in their persons, houses, papers and effects against unreasonable searches and seizures. However, where the search is made pursuant to a lawful arrest, there is no need to obtain a search warrant. A lawful arrest without a warrant may be made by a peace officer or a private person under the following circumstances. Sec. 5 Arrest without warrant; when lawful. A peace officer or a private person may, without a

warrant, arrest a person: (a) When, in his presence, the person to be arrested has committed is actually committing, or is attempting to commit an offense; (b) When an offense has in fact just been committed, and he has personal knowledge of facts indicating that the person to be arrested has committed it; and (c) When the person to be arrested is a prisoner who has escaped from a penal establishment or place where he is serving final judgment or temporarily confined while his case is pending, or has escaped while being transferred from one confinement to another. Accused was searched and arrested while transporting prohibited drugs (hashish). A crime was actually being committed by the accused and he was caught in flagrante delicto. Thus, the search made upon his personal effects falls squarely under paragraph (1) of the foregoing provisions of law, which allow a warrantless search incident to a lawful arrest. While it is true that the NARCOM officers were not armed with a search warrant when the search was made over the personal effects of accused, however, under the circumstances of the case, there was sufficient probable cause for said officers to believe that accused was then and there committing a crime. Probable cause has been defined as such facts and circumstances which could lead a reasonable, discreet and prudent man to believe that an offense has been committed, and that the objects sought in connection with the offense are in the place sought to be searched. Warrantless search of the personal effects of an accused has been declared by this Court as valid, because of existence of probable cause, where the smell of marijuana emanated from a plastic bag owned by the accused, 10 or where the accused was acting suspiciously, 11 and attempted to flee. The appealed judgment of conviction by the trial court is hereby affirmed. Costs against the accusedappellant.

Alih vs Castro GL. 69401 This case was in explaining Sec 3 of Art 2 of the 1987 Constitution regarding the supremacy of the civilians.

Facts; Zona was conducted by the men of Maj. Gen Castro in a compoud where petioners reside and conducted illegal search and thereafter seized guns from them. The order was carried on by his Castro's men with the contention that the petitioners are involved in the latest killing of the town's mayor Cesar Climaco. Issue; Is the warrantless search and seizure legal?

Held; The Supreme Court declared those seized in custodia legis and declared that the operation conducted by Maj. Gen. Castro was ILLEGAL. The respondents have all the time to obtain a search warrant granted that they have about 10 trial courts. The SC also held the protection of the petitioner's human rights as stated in Art IV Sec 3 and 4 of the 1973 Constitution regarding illegal search and seizure. The presumption of innocence of the petitioners should be observed and that they cannot be subjected to self-incriminating instances like paraffin tests, photographing and finger printing. As penned by J. Cruz in this case, "The Constitution is a law for rulers and people, equally in war and in peace, and covers with the shield of its protection all classes of men, at all times and under all circumstances. No doctrine, involving more pernicious consequences, was ever invented by the wit of man than that any of its provisions can be suspended during any of the great exigencies of government." G.R. No. 84715 October 17, 1990 THE PEOPLE OF THE PHILIPPINES, plaintiff-appellee, vs. ARNULFO CENDANA y REYES, accused-appellant. The Solicitor General for plaintiff-appellee. Edmundo M. Manaois for accused-appellant.

CORTES, J.: In the early morning of November 24, 1986, the Police Station at Sta. Barbara, Pangasinan received a report that a man was found dead on the field near the ricemill of a Mrs. Thelma Bautista at Barangay Ventinilla West, Sta. Barbara, Pangasinan. Upon investigation, policemen found the cadaver, Identified to be that of Dominador Manongdo, lying prostrate on the ground with one gunshot wound on the head. Subsequently, accused-appellant Arnulfo Cendana y Reyes was apprehended and later charged with the crime of murder "thru illegally possessed firearm" on the basis of the following information:
That on or about November 23, 1986, in the evening at a ricefield in barangay Ventinilla West, municipality of Sta. Barbara, province of Pangasinan, Philippines and within the jurisdiction of this Honorable Court, the above-named accused, armed with an unlicensed shotgun, with intent to kill, with treachery and evident premeditation, did then and there, wilfully, unlawfully and feloniously shoot one Dominador Manongdo y Loresco, inflicting upon him a gunshot wound which caused his instantaneous death, to the damage and prejudice of his heirs.

Contrary to Article 248 of the Revised Penal Code. [Record, p. 1].

Upon arraignment, accused-appellant pleaded not guilty. The pre-trial conference was terminated on November 2, 1987 after which, trial proceeded. The trial judge convicted accused-appellant in a decision dated June 3, 1988, the dispositive portion reading as follows:
WHEREFORE, the Court finds accused Arnulfo Cendana y Reyes guilty beyond reasonable doubt of the crime of Murder thru Illegally Possessed Firearm, and considering that the crime of Murder was committed with the use of an unlicensed firearm (shotgun, Exhibit "H"), the accused is sentenced to suffer the penalty of Reclusion Perpetua with all the accessory penalties provided by law, to indemnify the heirs of the deceased Dominador Manongdo y Loresco the sum of P30,000.00, and to pay the costs. The firearm is ordered confiscated and forfeited in favor of the Government, and the Clerk of this Court is ordered to turn over the firearm to the nearest Constabulary Command. [Record, p. 121].

Not agreeing with the findings of the trial court, accused-appellant appeals to this Court raising the following assignment of errors:
I. The trial court erred in convicting the accused even without proof beyond reasonable doubt. II. The trial court erred in not appreciating the defense of the accused. III. The trial court erred in convicting the accused based on shaky and crude circumstantial evidence. [Rollo, p. 43].

At the outset, it is noted that the Solicitor General filed a manifestation recommending the acquittal of accused-appellant. He asserts that the alleged admission made by the accused-appellant and the gun seized from him are both inadmissible in evidence for having been obtained in violation of accused-appellant's constitutional rights. He further asserts that after the exclusion of such evidence, the remaining evidence of the prosecution is clearly insufficient to support a conviction based on proof beyond reasonable doubt [Rollo, pp. 84-85]. We agree. To support its judgment of conviction, the trial court relied mainly on the testimonies of the Police Station Commander Sgt. Amadeo Asuncion, Pat. Alden Poserio, and Pat. Fernando Quinto, who were the police officers investigating the case. Their testimonies sought to prove the following: After the cadaver was brought to the morgue and later to a funeral parlor for autopsy, they went back to the place where the body was found and after interviewing some people, received information that accused-appellant was seen before the incident carrying a gun [TSN, November 3, 1987, p. 8]. When they went to

accused-appellant's house, he was not around and so they picked up one of his brothers, Antonio Cendana, who informed them that accused-appellant was at Pogo District in Dagupan City [TSN, November 3, 1987, pp. 8-9; February 1, 1988, pp. 5-6]. Not knowing where such place was, they forced Antonio Cendana to accompany them to the house where accused-appellant could be found [TSN, November 3, 1987, p. 9; February 1, 1988, p. 6]. Upon their arrival, Sgt. Asuncion and Pat. Quinto entered the house while Pat. Poserio remained outside [TSN, November 3, 1987, p. 10]. They were able to arrest accused-appellant who afterwards admitted to them that he was the one who shot the victim [TSN, February 1, 1988, p. 7]. They also recovered a homemade shotgun which, according to Pat. Quinto, was voluntarily handed to them by accusedappellant [TSN, November 4, 1987, p. 4] but according to Sgt. Asuncion was handed to them by a woman relative of accused-appellant upon the latter's instructions [TSN, February 1, 1988, p. 7]. They then proceeded to the NBI, Dagupan City where accusedappellant was subjected to a paraffin test and the recovered firearm surrendered for ballistics examination [TSN, November 3, 1987, p. 11; February 1, 1988, p. 8]. From the NBI, they proceeded to the Sta. Barbara Police Station where accused was thereafter detained [TSN, November 3, 1987, p. 12; February 1, 1988, p. 8]. No written statement was taken from accused-appellant [TSN, November 3, 1987, p. 12]. From the foregoing narration of events, we note the following: (1) That there was no eye witness to the killing of the victim Dominador Manongdo; (2) That the accused-appellant was apprehended by the police investigators on the basis of information obtained from unidentified persons that accused-appellant was seen carrying a gun before the incident. (3) That the accused-appellant was apprehended by the police officers without any warrant of arrest; (4) That the shotgun was recovered without a search warrant from the house where accused-appellant was arrested; and (5) That the alleged statement made by accused-appellant to the police officers admitting to the commission of the offense and made after his arrest, was used as the main basis for his conviction. I Section 5, Rule 113 of the 1985 Rules of Criminal Procedure enumerates the instances when a peace officer or a private person may arrest a person without a warrant:
(a) When, in his presence, the person to be arrested has commited is actually committing, or is attempting to commit an offense; (b) When an offense has in fact just been committed, and he has personal knowledge of facts indicating that the person to be arrested has committed it; and

(c) When the person to be arrrested is a prisoner who has escaped from a penal establishment or place where he is serving final judgment or temporarily confined while his case is pending, or has escaped while being transferred from one confinement to another.

The facts of the case do not warrant the applicability of paragraphs (a) and (c). Moreover, in paragraph (b), the only instance under which accused-appellant's case could possibly fall, what is essential is that the person making the arrest has personal knowledge of the facts indicating that the arrestee is responsible for an offense which has just been committed [People v. Burgos, G.R. No. 68955, September 4, 1986, 144 SCRA 1]. Accused-appellant was arrested one day after the killing of the victim and only on the basis of information obtained by the police officers from unnamed sources. These abovementioned circumstances clearly belie a lawful warrantless arrest. Considering that the arrest of accused-appellant herein was unlawful, any search conducted on his person or place of arrest which is an incident thereof, was also unlawful [People v. Burgos, supra]. Perforce, any evidence recovered during the unlawful search, being made without a warrant, becomes inadmissible in evidence against accused-appellant and the shotgun which was allegedly the fatal weapon cannot be presented against him [Nolasco v. Pano, G.R. No. 69803, January 30, 1987, 147 SCRA 509]. Section 20, Article IV of the 1973 Constitution ordains that:
No person shall be compelled to be a witness against himself Any person under investigation for the commission of an offense shall have the right to remain silent and to counsel, and to be informed of such right. No force, violence, threat, intimidation, or any other means which vitiates the free will shall be used against him. Any confession obtained in violation of this section shall be inadmissible in evidence.

The Court elaborated on the scope of this right in the case of Morales, Jr. v. Enrile [G.R. No. 61016, April 26, 1983, 121 SCRA 538], thus:
At the time a person is arrested, it shall be the duty of the arresting officer to inform him of the reason for the arrest and he must be shown the warrant of arrest, if any. He shall be informed of his constitutional rights to remain silent and to counsel, and that any statement he might make could be used against him. The person arrested shall have the right to communicate with his lawyer, a relative, or anyone he chooses by the most expedient means-by telephone if possible-or by letter or messenger. It shall be the responsibility of the arresting officer to see to it that this is accomplished. No custodial investigation shall be conducted unless it be in the presence of counsel engaged by the person arrested, by any person on his behalf, or appointed by the court upon petition either of the detainee himself or by anyone on his behalf. The right to counsel may be waived but the waiver shall not be valid unless made with the assistance of counsel. Any statement obtained in violation of the procedure herein laid down, whether exculpatory or inculpatory, in whole or in part, shall be inadmissible in evidence. [At p. 554]

This procedure served as the guideline in subsequent cases [People v. Ramos, G.R. No. 59318, May 16, 1983, 122 SCRA 312; People v. Galit, G.R. No. 51770, March 20, 1985, 135 SCRA 465]. In the present case, if is clear from the record that at the time

that accused-appellant was arrested, he was not apprised of the right to remain silent and to counsel, and to be informed of such rights, before he supposedly admitted to the killing of the deceased. As can be gleaned from the testimony of Sgt. Asuncion:
Q Were you able to locate Arnulfo Cendana? A Yes, sir. Q Where? A At the house of one of his relatives, sir. Q What happened when you arrived in that house? A When we arrived in the house I asked one of the occupants of the house the whereabouts of Arnulfo Cendana and I told one of my men to surround the house. I tried to went (sic) up to the house but I saw Arnulfo Cendana trying to escape so I told him "agka ombabatik ta paltogen taka" which means don't run or else I will shoot you''. Q What happened when you warned Arnulfo Cendana not to escape? A He stopped and he raised his hands. Q What happened next? A I asked him if he was the one who is responsible about the incident, the killing incident. At first he denied it but later on when I talked to him in calm manner he admitted that he was responsible and then I asked him where is the gun that he used. Q What did he tell you? A At first he denied it but later on he told to one of his relatives to get the gun, sir. [TSN, February 1, 1988, pp. 6-7; Emphasis supplied]

The failure of the police investigators to apprise accused-appellant of his constitutional rights makes inadmissible their testimonies that the accused-appellant admitted to the commission of the offense and pointed to the location of the shotgun [Art. III, sec. 12, par. 3, Constitution]. With the exclusion of these alleged admission of the accused-appelant, we must look into the other evidence taken against the accused-appellant which led to a judgment of conviction against him by the trial court. Only if this is sufficient to convince the Court without any reasonable doubt that he committed the offense charged can the judgment of conviction rendered against him be affirmed. II. The trial court took into account the following reports as clear indications that it was accused-appellant who fired the shotgun which killed Dominador Manongdo: (1) the postmortem report, as testified to by the Municipal Health Officer, Dr. Leonard Carbonell, to the effect that "the cause of death of the deceased is intracranial injuries with intracranial hemorrhage secondry to gunshot wound" [TSN, November 5, 1987, p. 43]; (2) Chemistry Report No. C-86-1205, as testified to by Ma. Carina Javier, an NBI

Forensic Chemist, to the effect that the examination of the paraffin casts of accusedappellant yielded positive results for specks in both hands, indicating that gunpowder nitrates were present [TSN, December 22, 1987, pp. 53-54]; and (3) the ballistics report, as testified to by Irineo Ordiano, Jr., an NBI Senior Ballistician, to the effect that the homemade shotgun allegedly recovered from the accused-appellant is serviceable [TSN, January 20, 1988, p. 64]. A careful analysis of the postmortem report will show that the gunshot wound sustained by the deceased Dominador Manongdo is indeed compatible with what could be inflicted by a shotgun. However, such a finding assumes no significance in connection with a finding of the guilt of accused-appellant unless the firing of the shotgun could positively be linked to him. Neither is the ballistics report any support to the conclusion that accused-appellant is guilty of the commission of the offense. All that the NBI ballistics report establishes is that the shotgun which was submitted to it for examination is still serviceable. Although this could have been done facilely by the crime laboratory, there was no finding as to whether or not the shotgun has just been recently fired. Therefore, instead of bolstering the case for the prosecution, it instead raises the doubt that the tested shotgun may not have been the same shotgun which was used to shoot at and kill the deceased Manongdo. The only other evidence left with which to establish the guilt of accused-appellant is the positive result obtained from the paraffin casts taken from his hands. If this was supported by other evidence before the Court, then the presence of gunpowder nitrates on the hands of accused-appellant could have been a strong indication that it was he who fired the gun which killed the deceased. However, this does not obtain in the case at bar. Likewise of note is the plausible reason preferred by the accused-appellant that he tested positive for gunpowder burns because he took a turn at firing at coconut fruits with an armalite of a military man who was likewise a guest at a party given by his kumpare [TSN, April 6, 1988, p. 7]. The absence of any eyewitness to the commission of the offense and the exclusion of the admissions allegedly made by accused-appellant and the fruits thereof, calls into application Rule 133, section 5 of the Rules of Court which states:
Sec. 5. Circumstantial evidence, when sufficient. Circumstantial evidence is sufficient for conviction if: (a) There is more than one circumstance; (b) The facts from which the inferences are derived are proven; and (c) The combination of all the circumstances is such as to produce a conviction beyond reasonable doubt.

There being only one circumstance indicative of the guilt of the accused, i.e. the paraffin casts of accused-appellant's hands yielded for gunpowder burns the above requisites are not satisfied. The case of the prosecution definitely fails. WHEREFORE, finding that the guilt of the accused-appellant has not been proved beyond reasonable doubt, the decision of the lower court is reversed and the accusedappellant is hereby ACQUITTED. SO ORDERED. G.R. No. 87783 August 6, 1990 THE PEOPLE OF THE PHILIPPINES, plaintiff-appellee, vs. ADELINA CASTILLER y CASTRO, defendant-appellant. The Solicitor General for plaintiff-appellee. Public Attorney's Office for defendant-appellant.

CORTES, J.: Appellant Adelina Castiller y Castro was charged with and convicted of violation of Section 4, Art. II of Rep. Act No. 6425, as amended, otherwise known as the Dangerous Drugs Act, under an information which read:
That on or about the 17th day of April, 1988, in the Municipality of Taguig, Metro Manila, Philippines and within the jurisdiction of this Honorable Court, the said accused, without having been authorized by law, did then and there wilfully, unlawfully and feloniously sell, deliver and give away to another two (2) foils of dried marijuana fruiting tops and was found to be in possession of the following: one (1) Ajinomoto Plastic bag containing 545 grams of dried marijuana fruiting tops, ten (10) sticks of marijuana cigarette wrapped with white paper and five (5) newspaper wrappers each with dried marijuana fruiting tops having a total weight of 20.77 grams, which is a prohibited drug. Contrary to law [Rollo, p. 4].

Upon arraignment, appellant, assisted by counsel, entered a plea of "not guilty" to the offense charged. Trial ensued with the prosecution and the defense presenting their own witnesses and evidence to support their respective versions of the events leading to the arrest of the accused-appellant. According to the prosecution witnesses, composed mainly of police officers, at around nine o'clock in the morning of April 18, 1988, the anti-narcotics intelligence division of the Taguig Police Station received information from an undisclosed caller that marijuana was being sold by an old woman in a small store in North Daang Hari Street, Taguig

[TSN, June 29, 1988, p. 4; TSN, October 7, 1988, p. 2]. Immediately, Capt. Ferdinand Santos organized a team composed of police officers Felixberto Maog (team leader), Jesus Chan, Joselito Lintad, Jessie Pasion, Ruel Viring, and Carlos Mendibel in order to conduct a buy-bust operation [TSN, June 29, 1988, p. 4]. Pat. Mendibel, assigned to act as poseur-buyer, was briefed by Capt. Santos separately from the other members of the team who were to serve as backup [TSN, August 2, 1988, pp. 3-4], and was later handed a twenty-peso bill to be used to purchase marijuana in the buybust operation [TSN, June 29, 1988, p. 3; TSN, August 10, 1988, p. 14]. Around noontime, the plain clothes team arrived at North Daang Hari. Indeed, they found an old woman in a small store in No. 303 North Daang Hari, which Pat. Jessie Pasion pointed out to Mendibel as the subject of the operation. The other members of the team in the meantime positioned themselves some fifteen (15) meters away from the store [TSN, October 7, 1988, p. 2]. Pat. Mendibel went to the store, and said to the appellant: "La paiskor ng dalawang foil" [TSN, July 15, 1988, p. 3]. Appellant went inside the store and came back with two (2) foils of marijuana which she gave to Pat. Mendibel. Taking the marijuana foils, Pat. Mendibel introduced himself as a police officer [TSN, July 15, 1988, pp. 3-4]. Appellant, upon learning that her "customer" was in fact a police officer, scampered into the back of her store and locked herself inside even before Pat. Mendibel could hand over the twenty-peso bill in payment of the foils of marijuana [TSN, August 10, 1988, p. 15]. Pat. Mendibel immediately gave the prearranged signal to inform his teammates that the operation had indeed yielded marijuana, and to get their assistance in arresting the appellant [TSN, October 7, 1988, p. 3]. The police officers knocked at the door but appellant refused to open the door. Later, however, she voluntarily opened the door when the police officers, still knocking, asked permission to be allowed inside her store [TSN, August 10, 1988, pp. 16-17]. When the police officers went inside, appellant pointed to a large gray container placed beside some cases of softdrink [TSN, August 10, 1988, p. 4]. They found a hole at the bottom of the large gray plastic container, and stored inside were a plastic "Ajinomoto" bag containing substances which appeared to be marijuana leaves, ten (10) sticks of what appeared to be marijuana cigarettes, five (5) foils of what appeared to be marijuana leaves wrapped in newspaper, six (6) packs of "Capitol" brand rolling paper and a red leatherette bag [Exhibits "G", "C", "D", "E", "H"-1" to "H-6" and "H" respectively]. Appellant and the confiscated evidence were brought to the police station. Pat. Santiago Villa, the officer assigned to conduct the investigation, informed appellant of her constitutional rights. Stating only her name and her desire to consult her lawyer [TSN, August 25, 1988, p. 5], appellant opted to remain silent. The seized specimen, along with a letter-request dated April 18, 1988 signed by Capt. Ferdinand Santos, were then brought to the PC Crime Laboratory at Camp Crame so that tests could be conducted on the substances which appeared to be marijuana.

The laboratory analysis conducted by P/Capt. Lina C. Sarmiento found the specimens to be "positive for marijuana, a prohibited drug" [Chemistry Report No. D-348-88, Exhibit "A"; and Certificate of Laboratory Result, Exhibit "J"] i.e., a plastic bag containing 545 grams dried marijuana fruiting tops, ten (10) sticks of marijuana cigarettes wrapped with white paper, and the five (5) newspaper wrappers each with dried marijuana fruiting tops having a total weight of 20.77 grams. On the other hand, the defense presented a different version of the events that transpired. According to appellant, she lived in a squatter area where the only source of water was an artesian well located one house away from her store [TSN, February 14, 1989, p. 41. People usually had to queue up in order to get water from said well. Those living nearby habitually left their water containers at appellant's store which they later claimed when the need arose [TSN, February 14, 1989, p. 4]. Appellant testified that on the night before the alleged buy-bust operation took place, one "Magda" was the last person to leave her water container at the store [TSN, January 10, 1989, p. 2]. The next day, at around 11:30 in the morning, a group of men, strangers to appellant, approached her in her store. One of them suddenly held her at her right side and informed her that she would be brought to the police precinct. Appellant protested and asked if he had a warrant, but the man did not answer her [TSN, January 10, 1989, p. 3]. The others went inside her store and began searching the place. Appellant was brought to the police precinct while the others continued searching her store [TSN, January 10, 1989, pp. 34]. Appellant denied knowledge of the articles inside the gray container which was seized from her store but she admitted that she recognized the gray container to be Magda's [TSN, January 10, 1989, p. 4]. Two other witnesses were presented for the defense. Carmelita Ramos testified that, like some five other neighbors of appellant, she usually left her water container at the latter's store, and claimed it when she needed to fetch water from the well again [TSN, February 14, 1989, pp. 3-4]. On the other hand, Rebecca De los Santos testified that in the evening of April 17, 1988, she saw a certain woman leave her water container at appellant's store, and heard the name "Magda" mentioned [TSN, February 14, 1989, p. 2]. The trial court, giving credence to the evidence of the prosecution, found that appellant was caught in flagrante delicto delivering marijuana. Furthermore, the trial court held that appellant's possession of considerable quantities of marijuana indicates the intention to sell, distribute or deliver the same, and that she was "really engaged in the illicit trade of marijuana." The dispositive portion of the judgment of conviction reads:
WHEREFORE, the Court hereby sentences the accused, ADELINA CASTILLER Y CASTRO to suffer the penalty of reclusion perpetua with all its accessory penalties, to pay a fine of P20,000 and to pay the costs. In the service of her sentence, the accused shall be credited in full with the period of her preventive imprisonment.

Pursuant to Section 20, Article II of Republic Act No. 6425, as amended, let the 545 grams of dried marijuana fruiting tops, ten (10) sticks of marijuana cigarette and another 20.77 grams of dried marijuana fruiting tops in five (5) newspaper wrappers subject matter of this case be turned over to the Dangerous Drugs Board Custodian, NBI, to be disposed of according to law. Pursuant to Article 45 of the Revised Penal Code, the gray plastic container (Exhibit "G"), the red leatherette bag (Exhibit "H") and six (6) packs of rolling paper (Exhibits "H-1" to "H-6") are hereby ordered confiscated and forfeited in favor of the Government. SO ORDERED.

Appellant assails the decision of the trial court in this appeal, assigning the following errors:
I THE TRIAL COURT ERRED IN GIVING CREDENCE TO THE TESTIMONIES OF THE PROSECUTION WITNESSES AND IN TOTALLY DISREGARDING THE VERSION OF THE DEFENSE. II THE TRIAL COURT ERRED IN CONVICTING THE ACCUSED DESPITE HER UNLAWFUL ARREST AND ILLEGAL SEARCH AND SEIZURE UPON HER PREMISES. III THE TRIAL COURT ERRED IN FINDING THE ACCUSED-APPELLANT GUILTY BEYOND REASONABLE DOUBT OF THE CRIME OF VIOLATION OF SECTION 4, ARTICLE II OF R.A. 6425, AS AMENDED. [Appellants Brief, p. 1.]

Ultimately, this case presents only one issue: whether or not appellant was proven guilty of the crime charged beyond reasonable doubt. After a thorough perusal of the evidence of both parties, we find that indeed, as sufficiently established by the prosecution, appellant is guilty of the crime charged. The testimonies of the prosecution witnesses substantially covered the circumstances of each and every stage of the crime, and the resulting arrest, search and seizure. The buy-bust operation team leader (Pfc Felixberto Maog) testified as to the circumstances leading to and the formation of the entrapment scheme. The poseur- buyer (Pat. Mendibel) positively identified appellant as the woman who delivered two (2) foils of marijuana to him, and along with a member of the "back-up" (Pat. Jesus Chan), testified as to the incidental search conducted on the store, and the prohibited drugs thereon. The case investigator (Pat. Santiago Villa) testified on the investigation made on appellant and the examination of the seized articles after the arrest, search and seizure, and the forensic chemist (P/Capt. Lina Sarmiento) found the seized articles submitted for examination to be positive for marijuana.

We find their testimonies to be clear, lucid, straightforward and uncontradicted on all material points. There is nothing in the record to suggest that the police officers were compelled by any motive other than to accomplish their mission to capture appellant in the execution of the crime. The presumption being that police officers perform their duties regularly in the absence of any evidence to the contrary [Rule 13], Section 5 (m), Rules of Court; People v. Natipravat, G.R. No. 69876, November 13, 1986, 145 SCRA 483; People v. De Jesus, G.R. Nos. 71942-3, November 13, 1986, 145 SCRA 521; People v. Claudia G.R. No. 72564, April 15, 1988, 160 SCRA 6461, their testimonies are entitled to full faith and credence [People v. Patog, G.R. No. 69620, September 24, 1986, 144 SCRA 429]. On the other hand, the defense did not put a substantial defense other than the denial of the accused. It is well-settled that mere denials cannot prevail against positive identification of appellant as the seller of the prohibited substances [People v. Khan, G.R. No. 71863, May 23, 1988, 161 SCRA 406; People v. Paco, G.R. No. 76893, February 27, 1989, 170 SCRA 681]. Furthermore, the testimonies of defense witnesses Rebecca de los Santos and Carmelita Ramos do not buttress appellant's case. At best, they present minor details which neither substantiate appellant's denial of the charges nor strengthen the claim of an arrest and search totally wanting in legal basis. Curiously, appellant never made any effort to locate "Magda", the alleged owner of the container filled with the prohibited substances, in order to have her arrested. Certainly in a case where her own life and liberty were at stake, the victim of a wrongful accusation would have earnestly, nay even desperately, sought ways to vindicate herself by at least assisting the police in order to have the true offender apprehended. Appellant's inaction becomes all the more baffling considering that, by her own admission, she had known "Magda" for about a year, they lived on the same street (North Daang Hari), "Magda" fetched water at the artesian well just one house away from appellant's store everyday, and even bought things from the store [TSN, January 10, 1989, pp. 2, 6]. Collaterally, appellant would now attack the decision by putting at issue the validity of the buy-bust operation, the failure of prosecution to produce the "buy-bust money", and the legality of the arrest and the incidental search and seizure. Appellant underscores in its arguments the admission by the police officers that at the time they set out to conduct the operation, they did not know who their exact "target" was, having acted solely on a tip given by a caller who merely described the marijuana seller to be an old woman in a store on North Daang Hari, but gave no specific name. From this allegation, the defense concluded that no buy-bust operation had in fact taken place, and that the prosecution merely concocted the story in order to justify an unlawful arrest resulting from an "intense enthusiasm to respond to the unidentified caller" [Appellant's Brief, pp. 7-8].

This contention is devoid of merit. The buy-bust operation is a recognized means of entrapment for the apprehension of drug pushers [People v. Gatong-o G.R. No. 78698, December 29, 1988, 168 SCRA 716]. This is so because the very characteristic of prohibited drugs i.e. their being easily concealed and transferred without threat of detection in small and handy quantities, allows its sale, use and delivery with relative facility. Contrary to appellant's contention, no surveillance of the area or the subject of the entrapment is necessary where the police officers had, as in this case, reasonable ground to believe that the informer and the information given were reliable, and that a crime is indeed being perpetrated. The buy-bust operation was formed by the police officers precisely to test the veracity of the tip and in order to apprehend the perpetrator, if he in fact commits the offense, before he further endangers society. Besides, the tip given describing an old woman keeping a store at North Daang Hari as a drug pusher sufficiently and accurately points to the appellant. There was no arbitrariness on the part of the poseur-buyer in approaching appellant, who exactly fit every detail of the description given. The defense further belabors its theory that no entrapment was effected by pointing to the fact that the twenty-peso bill allegedly used in the buy-bust operation and later submitted in evidence was not even marked, thereby rendering the alleged buy-bust operation all the more dubious [Appellant's Brief, p. 8]. Again, we find no merit in this argument. Although the alleged buy-bust money was indeed unmarked, nevertheless, this fact alone will not suffice to weaken the prosecution's case. Pat. Mendibel testified that he was informed by his immediate superior who handed him the money that it was marked [TSN, June 29, 1988, p. 5; TSN, August 10, 1988, p. 14]. Apparently however, Mendibel, who joined the police service barely four months earlier [TSN, June 29, 1988], failed to personally check whether it was marked. As correctly noted by the Solicitor General, Mendibel was still a "tyro" in such operations, this being his first drugs case [Appellee's Brief, p. 14; Rollo, p. 84], and he was obviously unfamiliar with the manner in which buy-bust operations are conducted. Be that as it may, the failure to mark the bill is not fatal to the case because the Dangerous Drugs Act punishes "any person who, unless authorized by law, shall sell, administer, deliver, give away to another, distribute, dispatch in transit or transport any prohibited drug, or shall act as a broker in any such transactions. ... [Emphasis supplied]." In this case, the police officer sought to buy two sticks of marijuana and his offer was accepted by appellant who produced and delivered the same. The crime was consummated by the mere delivery of the prohibited goods even without money changing hands. As to the issue of the legality of appellant's warrantless arrest and the warrantless search, we find that both arrest and incidental search were made well within the bounds of the law.

The 1985 Rules on Criminal Procedure provide that:


Sec. 5. Arrest without warrant; when lawful. A peace officer or a private person may, without a warrant, arrest a person: a) When, in his presence the person to be arrested has committed, is actually committing, or is attempting to commit an offense; b) When an offense has in fact just been committed and he has personal knowledge of facts indicating that the person to be arrested has committed it; xxx xxx xxx In cases falling under paragraphs (a) and (b) hereof, the person arrested without a warrant shall be forthwith delivered to the nearest police station or jail, and he shall be proceeded against in accordance with Rule 112, Section 7. [Rule 113, Section 5; 1985 Rules on Criminal Procedure.]

Both of the above-cited instances of a lawful warrantless arrest are attendant in this case. Appellant was caught in flagrante delicto delivering to the poseur-buyer two (2) sticks of marijuana. The offense was committed in the presence of the police officer, and therefore the latter had personal knowledge of the commission of the offense. Under the circumstances, appellant's arrest was lawfully effected without need of a warrant. Furthermore, where, as in this case, the person to be arrested attempts to evade the same, the Rules on Criminal Procedure allow a peace officer, in order to make a lawful warrantless arrest, to "... break into any building or enclosure in which the person to be arrested is or is reasonably believed to be, if he is refused admittance thereto, after he has announced his authority and purpose" [Section 11, Rule 113, 1985 Rules on Criminal Procedure]. In the case at bar, the police officers did not have to break into the premises since appellant voluntarily allowed the law officer inside the store. Therefore, the entry by the law enforcers into the store in order to effect appellant's arrest was perfectly lawful. As to the validity of the search conducted upon the premises of appellant's store and the consequent seizure of incriminating evidence found therein, this Court finds that both search and seizure were valid. That searches and seizures must be supported by a valid warrant is not an absolute rule [Manipon Jr. v. Sandiganbayan, G.R. No. 58889, July 31, 1986, 143 SCRA 267]. Among the exceptions granted by law is a search incidental to a lawful arrest under Section 12, Rule 126 of the Rules on Criminal Procedure, which provides that "[a] person lawfully arrested may be searched for dangerous weapons or anything which may be used as proof of the commission of an offense, without a search warrant." In such an instance, a contemporaneous search may be conducted upon the person of the arrestee and the immediate vicinity where the arrest was made [Nolasco v. Pao, G.R. No. 69803, January 30, 1987, 147 SCRA 5091, as was done in this case. The inclusion

of the seized items in the evidence for the prosecution cannot be challenged as they were seized in conformity with the provision on lawful searches. All told, this Court is convinced that appellant Adelina Castiller had indeed committed the offense as charged. The assailed decision must be upheld. The trial court aptly quoted the dictum of this Court in the case of People v. Abedes [G.R. No. 73399, November 28, 1986, 146 SCRA 132]: "A drug pusher is a killer without mercy. He poisons the mind and deadens the body. He deserves no mercy." Our society has very right to be protected from the potential and actual harm wrought by prohibited drugs. WHEREFORE, the decision appealed from is hereby AFFIRMED. SO ORDERED. G.R. No. 76547 July 30, 1990 PEOPLE OF THE PHILIPPINES, plaintiff-appellee, vs. ALBERT OLAES y AMOROSO, accused-appellant. The Solicitor General for plaintiff-appellee. Romeo C. Alinea for accused-appellant.

REGALADO, J.: The decision 1 of the Regional Trial Court, Branch LXXII in Olongapo City, in Criminal Case No. 3602 involving a violation of Section 4, Article II of Republic Act No. 6425 (Dangerous Drugs Act of 1972) has been brought to us on appeal 2 by accusedappellant who was sentenced to suffer the penalty of life imprisonment and to pay a fine of P30,000.00 and the costs. The confiscation and immediate destruction of six (6) lids of marijuana subject matter of the case was also ordered. 3 In an information dated March 27, 1978, appellant was charged with unlawfully selling six (6) lids of marijuana. 4 On March 16, 1979, appellant, assisted by his counsel de oficio, was duly arraigned and, having pleaded not guilty, 5 trial on the merits ensued. Based on the evidence for the prosecution, the court a quo narrated the case for the People as follows:
That at 12:50 in the afternoon of June 15, 1977, said officers were at their office at the CANU investigating Manuelito Bernardo, whom they arrested earlier at No. 41 Harris St., East Bajac-Bajac, Olongapo City for possession of several lids of marijuana. Bernardo informed them that the lids of marijuana confiscated from him came from one alias "Abet" and his brother alias "Bonjing". They made Bernardo agree to act as buyer and to go to

the house which, according to Bernardo, was the source of the confiscated marijuana. After giving instructions to Bernardo about the pre-arranged signal, they all proceeded near the house of "Abet" located at No. 116 Jones Street, Olongapo City. Bernardo entered the compound bringing with him marked money amounting to P300.00 (These were earlier xeroxed after the signature of Pacifico Mugar was affixed).itc-asl After consummating the deal on the marijuana, Bernardo came out and surrendered the same to the officers. Immediately, they entered "Abet's" house and found him in the living room with some other persons. "Abet" was confronted, searched, and in his person was found the P300.00 marked money and lids of marijuana. "Abet" was interrogated on the spot as to the whereabouts of other marijuana if any. They proceeded next door at his brother Benjamin Olaes' kitchen where they recovered sixty more lids of marijuana. Thereafter, "Abet", which (sic) was later identified as accused Albert Olaes was brought to the CANU office for investigation. Albert Olaes was finger-printed by C2C Armando Cases and the booking sheet and arrest report were prepared (Exhibit "G"). The sworn statement of Olaes was taken by Pacifico Mugar (Exhibit "K"). Pfc. Abello together with Capitulo, Elgar, Macomb and Cases executed a Joint Affidavit in connection with this case (Exhibit "H"). The sixty lids of marijuana which were confiscated from the kitchen of 6 Benjamin Olaes are covered and stated in the Receipt for Property Seized (Exhibit "I").

On the other hand, the decision makes the factual recital that
The defense version per testimony of accused Alberto Olaes tends to show that on June 15, 1977, at around two o'clock in the afternoon, he was at their house located at 116 Jones Street Kalalake, Olongapo City. While at Room 5 of said address, a certain Eling carrying a bag came to their house asking for his brother Benjamin Olaes who was not home at that time. Eling was carrying a bag of dog food. Upon being told that Benjamin was out, Eling asked to be accompanied to Room 2 also located at 116 Jones Street, Olongapo City, where he left the bag in his brother's room, particularly in the "lababo". Thereafter, he and Eling went out of the room and he went home to his residence at Room 5, 116 Jones Street, Olongapo City. He executed a sworn statement in connection with this case. He does not know any person by the name of Lito. He affixed his signature on his statement because he was mauled by the investigator and was threatened to be killed. On cross-examination, he stated that he does not know who among the investigators beat him up and neither does he know who arrested him. All his personal circumstances stated in his statement are true. He also declared that Eling came from Saletran, 7 Dasmarinas. Cavite.

Opining that the testimony of appellant is incredible and relying heavily on the testimonies of the prosecution witnesses, as well as the extrajudicial confession executed by appellant "corroborated by evidence of corpus delicti," the court below rendered its judgment of conviction stated at the outset of this decision. Hence, this appeal where appellant submits as alleged reversible errors (1) the conviction based on an extrajudicial confession extracted in violation of due process and the bill of rights of the fundamental law: (2) the conviction under Section 4, Article II of Republic Act No. 6425 (selling) even if one of the elements of the offense is missing; and (3) the resolution of doubt on the defense theory by reason of non-corroboration. 8 We find for appellant and decree his acquittal.

Firstly, the extrajudicial confession 9 executed by appellant on June 15, 1977 is inadmissible in evidence. An examination thereof shows that he was informed of his constitutional rights to remain silent and to be assisted by counsel during said investigation. He was also asked if he was waiving his right to be assisted by counsel and he answered in the affirmative. However, while the rights of a person under custodial investigation may be waived, nonetheless the waiver must be made not only voluntarily, knowingly and intelligently but in the presence and with the assistance of counsel. 10 In the case at bar, the waiver was made without the assistance of counsel. 11 This omission alone is sufficient to invalidate the confession. 12 While the trial court observed that the narration of the accused in his extrajudicial confession is complete in every detail and did not show any sign of suspicious circumstance to indicate that there was pressure of restraints exerted upon his person, 13 having been made without the mandatory assistance of counsel the same is inadmissible in evidence regardless of the absence of coercion or even if it had been voluntarily given. 14 Secondly, this case exemplifies the instance where the non-presentation of the supposed poseur-buyer is fatal to the prosecution's case. The records show that the alleged sales transaction took place inside the house of appellant. In other words, the transaction was supposedly witnessed only by the poseur-buyer, Manuelito Bernardo. Only he has personal knowledge of such transaction which is the subject matter of this prosecution. In People vs. Ramos, 15 where the alleged informant and poseur-buyer was one and the same person, we stressed that without the testimony of said poseur-buyer, there is no convincing evidence pointing to the accused as having sold marijuana. In this case, the police officers did not see the actual sale of marijuana. For the culprit to be convicted, the element of sale must be unequivocably established. Yet, the alleged poseur-buyer in the "buy-bust" operation, the only one who allegedly dealt directly with appellant in the purchase of marijuana, was not presented at all at the trial. Under such circumstances, we have repeatedly held that the failure of the prosecution to present the alleged buyer was a fatal blow to the case against the accused. 16 The other prosecution witnesses admitted that it was only when the alleged poseurbuyer left the house of appellant and gave the signal indicating the consummation of the transaction that they started to approach the house of appellant and entered the same. Thereafter, they allegedly searched the person of appellant and proceeded to the adjoining room where they claimed to have recovered other prohibited drugs. 17 We have carefully reviewed the records and find that the testimonies of the prosecution witnesses do not inspire belief. While suspiciously dovetailing on certain aspects in their testimonies, the members of said law enforcement team, despite the fact that they claim to have been together before, during and after the operation, enmeshed themselves in significant inconsistencies and contradictions.

Thus, for instance, Cpl. Ernesto Abello claimed that the marked money which was used to purchase the marijuana was zeroxed before it was given to the poseur-buyer who later allegedly paid it to appellant. 18 On the other hand, Pfc. Jaime Capitulo who allegedly recovered the marked money from appellant testified that it was after such money was retrieved and the team had returned to their office that he turned over the money to their superior, Capt. Aldaba, and it was then that zerox copies of the money were made presumably for purposes of evidence, 19 only to subsequently reverse himself by claiming that the same was zeroxed before they were given to the poseurbuyer. 20 It will be noted that it is this evidence, with the confusing testimony thereon, by which the prosecution seeks to link appellant to the supposed sale. Now, the poseur-buyer, according to the prosecution witnesses, was supposed to give them a pre-arranged signal after he had made the purchase of the marijuana. He is alleged to have done so but, as to the signal pre-arranged and given, the versions of said witnesses are in hopeless disarray. According to Cpl. Abello, Bernardo "would be coming out of the backdoor and he will be combing his hair. 21 On the other hand, Sgt. Glenn Logan testified that the prearranged signal was that Bernardo would put out his handkerchief and wipe his face. 22 These were all contradicted in the testimony of Pfc. Jaime Capitulo who claimed that the informant was able to give the pre-arranged signal because the investigating team had with them an electric device with a receiver that received Bernardo's signal which could be heard by everyone in the group. 23 How this would be possible and why the witnesses are at loggerheads hereon, the prosecution does not say. Again, Sgt. Logan declared that the search was conducted in the residence of appellant resulting in the seizure of sixty (60) lids of marijuana. 24 On the contrary, Pat. Abella clearly explained that said sixty (60) lids of marijuana were hidden and recovered in the kitchen of the house of Benjamin Olaes, a brother of appellant, which was a separate residence. 25 These conflicting testimonies, which further discredit said witnesses, are apart from the consideration that appellant herein is charged not with illegal possession but with the sale of marijuana. We also do not lose sight of the fact that without the testimony of the supposed poseurbuyer proving the alleged sale of marijuana inside appellant's house, the unlawful intrusion into the sanctity of appellant's abode and the unreasonable search and seizure proscribed by the Constitution are clearly established. It is undisputed that the police operatives did not have either a search warrant or a warrant of arrest. 26 The searches on the person of appellant and of his house were not also incidental to a lawful arrest. The police officers admittedly did not have personal knowledge at all of what actually transpired inside the appellant's house. They only learned of the alleged consummation of the illicit transaction when they were supposedly given a signal by their so-called poseur-buyer after the latter left appellant's house. However, what they reportedly learned from said poseur-buyer was indubitably hearsay as the latter was never called to appear and testify at the trial. Since what was conducted was a warrantless search and the arrest of appellant was unlawful, any evidence obtained from him is also inadmissible in evidence.

It is further undisputed that the six (6) lids of marijuana supposedly bought by the poseur-buyer was taken by the police officers not from appellant but from said poseurbuyer. 27 Appellant's disclaimer of ownership of the prohibited drug should have cautioned and alerted the prosecution to the fact that the testimony of their alleged poseur-buyer was not merely corroborative nor cumulative but direct and material to the defense of appellant who claims innocence of the offense imputed to him. 28 It is also significant that the identity of the informer and/or poseur-buyer was already known during the trial. 29 Despite all these, the prosecution opted not to present him for reasons which remain unknown. Such unexplained failure to present this vital witness gives rise to the presumption that, if he had been presented, his testimony would probably not have supported the case of the prosecution. 30 The foregoing considerations cannot but rule out a verdict of guilty, there being an exiguity of any other independent incriminating evidence, aside from appellant's uncounselled and inadmissible extrajudicial confession. While the theory of the defense is not totally convincing, the Court cannot tip the scales of justice against him in the face of the cardinal and long entrenched rule that the prosecution must rely on the strength of its own evidence and not on the weakness of that of the defense. One final observation. What cannot escape our attention is the penalty imposed by the trial court, that is, life imprisonment and a fine of P30,000.00. The court a quo obviously failed to consider that the alleged crime was committed on June 15, 1977. Under Section 4 of Republic Act No. 6425, prior to its amendment on February 17, 1980 by Presidential Decree No. 1675, the act of selling marijuana was punishable by imprisonment ranging from twelve (12) years and one (1) day to twenty (20) years and a fine ranging from P12,000.00 to P20,000.00. 31 Due to the urgent necessity of reinforcing the drive against dangerous drugs by making "drug-pushing" a capital offense, Presidential Decree No. 1675 was subsequently passed providing for the penalty of life imprisonment to death and a fine ranging from P20,000.00 to P30,000.00. Obviously, this amendment having taken effect after the alleged commission of the crime in the case at bar, it could not have been given retroactive effect. WHEREFORE, the guilt of accused-appellant not having been proved with the requisite quantum of evidence, the appealed decision is REVERSED and he is hereby ACQUITTED on reasonable doubt, with costs de officio. SO ORDERED.

Papa vs. M ago Facts:Petitioner Martin Alagao, head of the counter-intelligence unit of the Manila Police Department, acting upona reliable information received on November 3, 1 966 to theeffect that a certain shipment of personal effects, allegedlymisdeclared and undervalued, would be released thefollowing day from the customs zone of the port of Manilaand loaded on two trucks, and upon orders of petitioner Ricardo Papa, Chief of Police of Manila and a duly deputizedagent of the Bureau of Customs, conducted surveillance atgate No. 1 of the customs zone.When the trucks left gate No. 1

at about 4:3 0 in theafternoon of November 4, 1 966, elements of the counter-intelligence unit went after the trucks and intercepted them atthe Agrifina Circle, Ermita, Manila. The load of the two trucks,consisting of nine bales of goods, and the two trucks, wereseized on instructions of the Chief of Police. Uponinvestigation, a person claimed ownership of the goods andshowed to the policemen a "Statement and Receipts of Duties Collected on Informal Entry No. 1 47-55 01 ", issued bythe Bureau of Customs in the name of a certain BienvenidoNaguit.The respondent Mago, filed a petition for mandamusand certiorari before the CFI Manila contending that thesearch and seizure is illegal for lack of a valid warrant.Moreover, she also contends that such articles sought fromher is not included by the law for prohibited importation andthat it no longer under the control of the Tariff and Customscode for it (articles) were already sold to the petitioner.She also contends that the search seizureconducted by the respondents are illegally being madeoutside the jurisdiction of the BOC and that the subsequentsearch warrant issued by the collector of customs is not validbeing not issued by a judge.The respondent Mago filed an ex-parte motion torelease the confiscated articles upon her posting a bond.This motion was then granted by the respondent JudgeJarencio.Issue:Was the seizure of the goods unlawful? And that theBOC has no jurisdiction over the articles sought?Held:No. it is a valid seizure.The Chief of the Manila Police Department, RicardoG. Papa, having been deputized in writing by theCommissioner of Customs, could, for the purposes of theenforcement of the customs and tariff laws, effect searches,seizures, and arrests, and it was his duty to make seizure,among others, of any cargo, articles or other movableproperty when the same may be subject to forfeiture or liablefor any fine imposed under customs and tariff laws. He couldlawfully open and examine any box, trunk, envelope or other container wherever found when he had reasonable cause tosuspect the presence therein of dutiable articles introducedinto the Philippines contrary to law; and likewise to stop,search and examine any vehicle, beast or person reasonablysuspected of holding or conveying such article as aforesaid.It cannot be doubted, therefore, that petitioner Ricardo G.Papa, Chief of Police of Manila, could lawfully effect thesearch and seizure of the goods in question. The Tariff andCustoms Code authorizes him to demand assistance of anypolice officer to effect said search and seizure, and the latter has the legal duty to render said assistance. This was whathappened precisely in the case of Lt. Martin Alagao who, withhis unit, made the search and seizure of the two trucksloaded with the nine bales of goods in question at the Agrifina Circle. He was given authority by the Chief of Policeto make the interception of the cargo.Petitioner Martin Alagao and his companionpolicemen had authority to effect the seizure without anysearch warrant issued by a competent court.The Tariff and Customs Code does not require saidwarrant in the instant case. The Code authorizes personshaving police authority under Section 22 0 3 of the Tariff andCustoms Code to enter, pass through or search any land,inclosure, warehouse, store or building, not being a dwellinghouse; and also to inspect, search and examine any vesselor aircraft and any trunk, package, box or envelope or anyperson on board, or stop and search and examine anyvehicle, beast or person suspected of holding or conveyingany dutiable or prohibited article introduced into thePhilippines contrary to law, without mentioning the need of asearch warrant in said cases.It is our considered view, therefore, that except inthe case of the search of a dwelling house, personsexercising police authority under the customs law may effectsearch and seizure without a search warrant in theenforcement of customs laws. Note: The Bureau of Customs has the duties, powers and jurisdiction, among others, (1) to assess and collect all lawfulrevenues from imported articles, and all other dues, fees,charges, fines and penalties, accruing under the tariff andcustoms laws; (2) to prevent and suppress smuggling andother frauds upon the customs; and (3) to enforce tariff andcustoms laws.The goods in question were imported from Hongkong, asshown in the "Statement and Receipts of Duties Collected onInformal Entry." As long as the importation has not beenterminated the imported goods remain under the jurisdictionof the Bureau of Customs. Importationis deemed terminated only upon thepayment of the duties, taxes and other charges upon thearticles, or secured to be paid, at the port of entry and thelegal permit for withdrawal shall have been granted.. the payment of the duties, taxes, fees and other charges must be paid in full.

Roldan vs. Arca [GR L-25434, 25 July 1975] First Division, Makasiar (J): 4 concur, 1 took no part Facts: On 3 April 1964, Morabe, De Guzman & Company filed with the Court of First Instance(CFI) of Manila a civil case (56701) against Fisheries Commissioner Arsenio N. Roldan, Jr., for the recovery of fishing vessel Tony Lex VI which had been seized and impounded by theFisheries Commissioner through the Philippine Navy. On 10 April 1964, the company prayed for a writ of preliminary mandatory injunction with the CFI, but said prayer was denied. On 28 April1964, the CFI set aside its order of 10 April 1964 and granted the companys motion for reconsideration praying for preliminary mandatory injunction. Thus, the company took possession of the vessel Tony Lex VI from the Philippine Fisheries Commission adn thePhilippine Navy by virtue of the said writ. On 10 December 1964, the CFI dismissed Civil Case56701 for failure of the company to prosecute as well as for failure of the Commission and the Navy to appear on the scheduled date of hearing. The vessel, Tony Lex VI or Srta. Winniehowever, remained in the possession of the company.On 20 July 1965, the Fisheries Commissioner requested the Philippine Navy to apprehendvessels Tony Lex VI and Tony Lex III, also respectively called Srta. Winnie and Srta. Agnes, for alleged violations of some provisions of the Fisheries Act and the rules and regulations promulgated thereunder. On August 5 or 6, 1965, the two fishing boats were actually seized for illegal fishing with dynamite. Fish caught with dynamite and sticks of dynamite were then foundaboard the two vessels. On 18 August 1965, the Fisheries Commissioner requested the PalawanProvincial Fiscal to file criminal charges against the crew members of the fishing vessels. On 30September 1965, there were filed in the CFI of Palawan a couple of informations, one against thecrew members of Tony Lex III, and another against the crew members of Tony Lex VI bothfor violations of Act 4003, as amended by Commonwealth Acts 462, 659 and 1088, i.e., for illegal fishing with the use of dynamite. On the same day, the Fiscal filed an ex parte motion tohold the boats in custody as instruments and therefore evidence of the crime, and cabled theFisheries Commissioner to detain the vessels. On October 2 and 4, likewise, the CFI of Palawanordered the Philippine Navy to take the boats in custody. On 2 October 1965, the company fileda

complaint with application for preliminary mandatory injunction (Civil Case 62799) with theCFI of Manila against the Commission and the Navy. Among others, it was alleged that at thetime of the seizure of the fishing boats in issue, the same were engaged in legitimate fishingoperations off the coast of Palawan; that by virtue of the offer of compromise dated 13September 1965 by the company to the Secretary of Agriculture and Natural Resources, thenumerous violations of the Fishery Laws, if any, by the crew members of the vessels weresettled. On 18 October 1965, Judge Francisco Arca issued an order granting the issuance of thewrit of preliminary mandatory injunction and issued the preliminary writ upon the filing by thecompany of a bond of P5,000.00 for the release of the two vessels. On 19 October 1965, theCommission and the Navy filed a motion for reconsideration of the order issuing the preliminarywrit on 18 October 1965 on the ground, among others, that on 18 October 1965 the Philippine Navy received from the Palawan CFI two orders dated October 2 and 4, 1965 requiring thePhilippine Navy to hold the fishing boats in custody and directing that the said vessels should not be released until further orders from the Court, and that the bond of P5,000.00 is grosslyinsufficient to cover the Governments losses in case the two vessels, which are worthP495,000.00, are placed beyond the reach of the Government, thus frustrating their forfeiture asinstruments of the crime. On 23 November 1965, Judge Arca denied the said motion for reconsideration. The Commission and the Navy filed a petition for certiorari and prohibition with preliminary injunction to restrain Judge Arca from enforcing his order dated 18 October 1965,and the writ of preliminary mandatory injunction thereunder issued. Issue: Whether the Fisheries Commissioner and the Navy can validly direct and/or effect theseizure of the vessels of the company for illegal fishing by the use of dynamite and without therequisite licenses. Held: Section 4 of Republic Act 3512 approved on 20 March 1963 empowers the FisheriesCommissioner to carry out the provisions of the Fisheries Act, as amended, and all rules andregulations promulgated thereunder, to make searches and seizures personally or through hisduly authorized representatives in accordance with the Rules of Court, of explosives such asdynamites and the like; including fishery products, fishing equipment, tackle and other thingsthat are subject to seizure under existing fishery laws; and to effectively implement theenforcement of existing fishery laws on illegal fishing. Paragraph 5 of Section 4 of the sameRepublic Act 3512 likewise transferred to and vested in the Philippine Fisheries Commission allthe powers, functions and duties heretofore exercised by the Bureau of Customs, Philippine Navy and Philippine Constabulary over fishing vessels and fishery matters. Section 12 of theFisheries Act, otherwise known as Republic Act 4003, as amended, prohibits fishing withdynamites or other explosives which is penalized by Section 76 thereof by a fine of not lessthan P1,500.00 nor more than P5,000.00, and by imprisonment for not less than one (1) year andsix (6) months nor more than five (5) years, aside from the confiscation and forfeiture of allexplosives, boats, tackles, apparel, furniture, and other apparatus used in fishing in violation of said Section 12 of this Act. Section 78 of the same Fisheries Law provides that in case of asecond offense, the vessel, together with its tackle, apparel, furniture and stores shall be forfeitedto the Government. The second paragraph of Section 12 also provides that the possessionand/or finding, of dynamite, blasting caps and other explosives in any fishing boat shallconstitute a presumption that the said dynamite and/or blasting caps and explosives are beingused for fishing purposes in violation of this Section, and that the possession or discover in anyfishing boat or fish caught or killed by the use of dynamite or other explosives, under experttestimony, shall constitute a presumption that the owner, if present in the fishing boat, or thefishing crew have been fishing with dynamite or other explosives. Under Section 78 of theFisheries Act, as amended, any person, association or corporation fishing in deep sea fisherywithout the corresponding license prescribed in Sections 17 to 22 Article V of the Fisheries Actor any other order or regulation deriving force from its provisions, shall be punished for eachoffense by a fine of not more than P5,000.00, or imprisonment, for not more than one year, or both, in the discretion of the Court; Provided, That in case of an association or corporation, thePresident or manager shall be directly responsible for the acts of his employees or laborers if it is proven that the latter acted with his knowledge; otherwise the responsibility shall extend only asfar as fine is concerned: Provided, further, That in the absence of a known owner of the vessel,the master, patron or person in charge of such vessel shall be responsible for any violation of thisAct: and Provided, further, That in case of a second offense, the vessel together with its tackle,apparel, furniture and stores shall be forfeited to the Government. Under Section 13 of Executive Order 389 of 23 December 1950, reorganizing the Armed Forces of the Philippines,the Philippine Navy has the function, among others, to assist the proper governmental agenciesin the enforcement of laws and regulations pertaining to Fishing. Section 2210 of the Tariff andCustoms Code, as amended by PD 34 of 27 October 1972, authorized any official or person exercising police authority under the provisions of the Code, to search and seize any vessel or air craft as well as any trunk, package, bag or envelope on board and to search any person on boardfor any breach or violation of the customs and tariff laws. Herein, when the Philippine Navy,upon request of the Fisheries Commissioner, apprehended on August 5 or 6, 1965 the fishing boats Tony Lex III and Tony Lex VI, otherwise known respectively as Srta. Agnes and Srta.Winnie, these vessels were found to be without the necessary license in violation of Section 903of the Tariff and Customs Code and therefore subject to seizure under Section 2210 of the sameCode, and illegally fishing with explosives and without fishing license required by Sections 17and 18 of the Fisheries Law. Search and seizure without search warrant of vessels and air craftsfor violations of the customs laws have been the traditional exception to the constitutionalrequirement of a search warrant, because the vessel can be quickly moved out of the locality or jurisdiction in which the search warrant must be sought before such warrant could be secured;hence it is not practicable to require a search warrant before such search or seizure can beconstitutionally effected. The

same exception should apply to seizures of fishing vessels breaching our fishery laws: They are usually equipped with powerful motors that enable them toelude pursuing ships of the Philippine Navy or Coast Guard.

G.R. No. L-25434 July 25, 1975 HONORABLE ARSENIO N. ROLDAN, JR., in his capacity as Acting Commissioner, Philippine Fisheries Commission, and THE PHILIPPINE NAVY, petitioners, vs. HONORABLE FRANCISCO ARCA, as Presiding Judge of the Court of First Instance of Manila (Branch 1) and MORABE, DE GUZMAN & COMPANY, respondents. Office of the Solicitor General Arturo A. Alafriz and Solicitor Augusto M. Amores for petitioners. J. C. Yuseco and A.R. Narvasa for private respondent.

MAKASIAR, J.: A petition for certiorari and prohibition with preliminary injunction to restrain respondent Judge from enforcing his order dated October 18, 1965, and the writ of preliminary mandatory injunction thereunder issued. On April 3, 1964, respondent company filed with the Court of First Instance of Manila a civil case docketed as No. 56701 against petitioner Fisheries Commissioner Arsenio N. Roldan, Jr., for the recovery of fishing vessel Tony Lex VI (one of two fishing boats in question) which had been seized and impounded by petitioner Fisheries Commissioner through the Philippine Navy. On April 10, 1964, respondent company prayed for a writ of preliminary mandatory injunction with respondent court, but said prayer was, however, denied. On April 28, 1964, the Court of First Instance of Manila set aside its order of April 10, 1964 and granted respondent company's motion for reconsideration praying for preliminary mandatory injunction. Thus, respondent company took Possession of the vessel Tony Lex VI from herein petitioners by virtue of the abovesaid writ. On December 10, 1964, the Court of First Instance of Manila dismissed Civil Case No. 56701 for failure of therein petitioner (respondent company herein) to prosecute as well as for failure of therein defendants (petitioners herein)to appear on the scheduled date of hearing. The vessel, Tony Lex VI or Srta. Winnie however, remained in the possession of respondent company.

On July 20, 1965, petitioner Fisheries Commissioner requested the Philippine Navy to apprehend vessels Tony Lex VI and Tony Lex III, also respectively called Srta. Winnie and Srta. Agnes, for alleged violations of some provisions of the Fisheries Act and the rules and regulations promulgated thereunder. On August 5 or 6, 1965, the two fishing boats were actually seized for illegal fishing with dynamite. Fish caught with dynamite and sticks of dynamite were then found aboard the two vessels. On August 18, 1965, the Fisheries Commissioner requested the Palawan Provincial Fiscal to file criminal charges against the crew members of the fishing vessels. On September 30, 1965, there were filed in the court of First Instance of Palawan a couple of informations, one against the crew members of Tony Lex III, and another against the crew members of Tony Lex VI both for violations of Act No. 4003, as amended by Commonwealth Acts Nos. 462, 659 and 1088, i.e., for illegal fishing with the use of dynamite. On the same day, the Fiscal filed an ex parte motion to hold the boats in custody as instruments and therefore evidence of the crime (p. 54, rec.), and cabled the Fisheries Commissioner to detain the vessels (p. 56, rec.). On October 2 and 4, likewise, the Court of First Instance of Palawan ordered the Philippine Navy to take the boats in custody. On October 2, 1965, respondent company filed a complaint with application for preliminary mandatory injunction, docketed as Civil Case No. 62799 with the Court of First Instance of Manila against herein petitioners. Among others, it was alleged that at the time of the seizure of the fishing boats in issue, the same were engaged in legitimate fishing operations off the coast of Palawan; that by virtue of the offer of compromise dated September 13, 1965 by respondent company to the Secretary of Agriculture and Natural Resources, the numerous violations of the Fishery Laws, if any, by the crew members of the vessels were settled. On October 9, 1965, petitioners, represented by the Solicitor General, opposed the above-mentioned complaint, alleging among others, that: (1) the issuance of the writ would disrupt the status quo of the parties and would render nugatory any decision of the respondent court favorable to the defendant; (2) that the vessels, being instruments of a crime in criminal cases Nos. 3416 and 3417 filed with the Court of First Instance of Palawan, the release of the vessels sans the corresponding order from the abovementioned court would deprive the same of its authority to dispose of the vessels in the criminal cases and the Provincial Fiscal would not be able to utilize said vessels as evidence in the prosecution of said cases; (3) that as petitioners herein were in possession of one of the vessels in point, they cannot now be deprived of the legal custody thereof by reason of the dismissal of Civil Case No. 56701; (4) that petitioner Fisheries Commissioner has the power to seize and detain the vessels pursuant to Section 5 of Republic Act No. 3215 in relation to Sections 903 and 2210 of the Revised Tariff and Customs Code; (5) that respondents herein have not exhausted

administrative remedies before coming to court; (6) that the compromise agreement approved by the Secretary of Agriculture and Natural Resources and indorsed to the Fisheries Commissioner is never a bar to the prosecution of the crime perpetrated by the crew members of the vessels belonging to respondent company. And again, on October 15, 1965, herein petitioners filed their memorandum praying for the denial of the application for preliminary mandatory injunction. On the same day, October 15, 1965, herein petitioners filed an urgent motion to submit additional documentary evidence. On October 18, 1965, herein petitioners, as defendants in said Civil Case No. 62799, filed their answer to the complaint with affirmative defenses, reiterating the grounds in their opposition to the issuance of a writ of preliminary mandatory injunction and adding that herein private respondent admitted committing the last violation when it offered in its letter dated September 21, 1965 to the Acting Commissioner of Fisheries, to compromise said last violation (Exh. 12, pp. 60-61, rec.). On said day, October 18, 1965, the respondent Judge issued the challenged order granting the issuance of the writ of preliminary mandatory injunction and issued the preliminary writ upon the filing by private respondent of a bond of P5,000.00 for the release of the two vessels(pp. 95-102, rec.). On October 19, 1965, herein petitioners filed a motion for reconsideration of the order issuing the preliminary writ on October 18, 1965 on the ground, among others, that on October 18, 1965 the Philippine Navy received from the Palawan Court of First Instance two orders dated October 2 and 4, 1965 requiring the Philippine Navy to hold the fishing boats in custody and directing that the said vessels should not be released until further orders from the Court, and that the bond of P5,000.00 is grossly insufficient to cover the Government's losses in case the two vessels, which are worth P495,000.00, are placed beyond the reach of the Government, thus frustrating their forfeiture as instruments of the crime (pp. 103-109, rec.).1wph1.t On November 23, 1965, respondent Judge denied the said motion for reconsideration (p. 110, rec.). WE rule that the respondent Judge of the Manila Court of First Instance acted without jurisdiction and with grave abuse of discretion when he issued on October 18, 1965 the order directing the issuance of a writ of preliminary mandatory injunction and when he refused to reconsider the same. I When the respondent Judge issued the challenged order on October 18, 1965 and the writ of preliminary mandatory injunction pursuant thereto, the fishing vessels were already under the jurisdiction of the Court of First Instance of Palawan by virtue of its orders of October 2 and 4, 1965, upon motion of the Provincial Fiscal (pp. 54, 55, rec.),

directing the Philippine Navy to detain (pp. 108, 109, rec.) said vessels, which are subject to forfeiture as instruments of the crime, to be utilized as evidence in Criminal Cases Nos. 3416 and 3417 for illegal fishing pending in said court (pp. 54-55, rec.). The said vessels were seized while engaging in prohibited fishing within the territorial waters of Palawan (pp. 45, 48,-53, rec.) and hence within the jurisdiction of the Court of First Instance of Palawan, in obedience to the rule that "the place where a criminal offense was committed not only determines the venue of the action but is an essential element of jurisdiction"(Lopez vs. Paras, L-25795, Oct. 29, 1966, 18 SCRA 616, 619). The jurisdiction over the vessels acquired by the Palawan Court of First Instance cannot be interfered with by another Court of First Instance. The orders of October 2 and 4, 1965 by the Palawan Court of First Instance expressly direct the Philippine Navy "to hold in custody" the two vessels and that "same should not be released without prior order or authority from this Court" (pp. 108, 109, rec.). Only the Palawan court can order the release of the two vessels. Not even the Secretary of Agriculture and Natural Resources nor the Fisheries Commissioner can direct that the fishing boats be turned over to private respondent without risking contempt of court. The grave abuse of discretion committed by the respondent Judge was heightened by the fact that he did not reconsider his order of October 18, 1965 after he was informed by petitioners in their motion for reconsideration filed on October 19, 1965 that the Palawan Court of First Instance had already issued the two orders dated October 2 and 4, 1965 directing the Philippine Navy to hold in custody the fishing boats until further orders. It is basic that one court cannot interfere with the judgments, orders or decrees of another court of concurrent or coordinate jurisdiction having equal power to grant the relief sought by injunction; because if coordinate courts were allowed to interfere with each other's judgments, decrees or injunctions, the same would obviously lead to confusion and might seriously hinder the administration of justice (Ongsinco, etc. vs. Tan, et al., 97 Phil. 330; PNB vs. Javellana, 92 Phil. 525; Montesa vs. Manila Cordage Company, 92 Phil. 25; Hubahib vs. Insular Drug Company, 64 Phil. 119; Hacbang, et al. vs. The Leyte Auto Bus Company, et al., G.R. No. L-17907, May 30, 1963, 8 SCRA, 103, 107-109; NPC vs. Hon. Jesus de Vera, G.R. No. L-15763, Dec. 22, 1961, 3 SCRA, 646, 648; Cabigao vs. del Rosario, 44 Phil. 182; Araneta & Uy vs. Commonwealth Insurance Company, 55 OG 431; Moran, Comments on the Rules of Court, Vol. III, 1970 ed., p. 64). As early as October 2 and 4, 1965, the two boats were already in custodia legis under the sole control of the Palawan Court of First Instance. The Manila Court of First Instance cannot interfere with and change that possession (Hacbang vs. Leyte Bus Co., Inc., supra; NPC vs. Hon. Jesus de Vera, supra). It is immaterial that the vessels were then in the Philippine Navy basin in Manila; for the same in no way impugns the jurisdiction already vested in the Palawan court, which has custody thereof through the Philippine Navy. This is analogous to the situation in Colmenares versus Villar (L-27124, May 29, 1970, 33 SCRA 186, 188-9), wherein We

ruled "where the illegal possession of firearms was committed in the town where the Court sits, the fact that the firearms were confiscated from the accused in another town does not affect the jurisdiction of the Court" (pp. 186, 189). It is likewise of no moment that the herein respondents were not notified by the herein petitioners of the seizure of the questioned vessels by the Philippine Navy, because such previous notice is not required by law. II The dismissal on December 10, 1964 of the first Civil Case No. 56701 by the Court of First Instance of Manila had the necessary effect of automatically dissolving the writ of preliminary mandatory injunction issued therein on April 28, 1964, directing the return of fishing vessel Tony Lex VI (pp. 156-157, rec.). Such a preliminary writ, like any other interlocutory order, cannot survive the main case of which it was but an incident; because "an ancillary writ of preliminary injunction loses its force and effect after the dismissal of the main petition" (National Sugar Workers' Union, etc., vs. La Carlota Sugar Central, et al., L-23569, May 25, 1972, 45 SCRA 104, 109; Lazaro vs. Mariano, 59 Phil. 6Z7, 631; Saavedra vs. Ibaez, 56 Phil. 33, 37; Hi Caiji vs. Phil. Sugar Estate and Development Company, 50 Phil. 592, 594).1wph1.t Moreover, the writ of preliminary injunction issued on April 28, 1964 in Civil Case No. 56701 was directed against the detention of the vessel Tony Lex VI for violations committed prior to August 5, 1965, and therefore cannot and does not extend to the seizure and detention of said vessel for violations on August 5 or 6, 1965, which violations were not and could not possibly be the subject-matter of said Civil Case No. 56701 which was filed on April 3, 1964 (p. 12, rec.). III Herein petitioners can validly direct and/or effect the seizure of the vessels of private respondent for illegal fishing by the use of dynamite and without the requisite licenses. Section 4 of Republic Act No. 3512 approved on March 20, 1963 empowers the Fisheries Commissioner to carry out the provisions of the Fisheries Act, as amended, and all rules and regulations promulgated thereunder, to make searches and seizures personally or through his duly authorized representatives in accordance with the Rules of Court, of "explosives such as ... dynamites and the like ...; including fishery products, fishing equipment, tackle and other things that are subject to seizure under existing fishery laws"; and "to effectively implement the enforcement of existing fishery laws on illegal fishing." Paragraph 5 of Section 4 of the same Republic Act 3512 likewise transferred to and vested in the Philippine Fisheries Commission "all the powers, functions and duties heretofore exercised by the Bureau of Customs, Philippine Navy and Philippine Constabulary over fishing vessels and fishery matters ..."

Section 12 of the Fisheries Act, otherwise known as Republic Act No. 4003, as amended, prohibits fishing with dynamites or other explosives which is penalized by Section 76 thereof "by a fine of not less than P1,500.00 nor more than P5,000.00, and by imprisonment for not less than one (1) year and six (6) months nor more than five (5) years, aside from the confiscation and forfeiture of all explosives, boats, tackles, apparel, furniture, and other apparatus used in fishing in violation of said Section 12 of this Act." Section 78 of the same Fisheries Law provides that "in case of a second offense, the vessel, together with its tackle, apparel, furniture and stores shall be forfeited to the Government." The second paragraph of Section 12 also provides that "the possession and/or finding, of dynamite, blasting caps and other explosives in any fishing boat shall constitute a presumption that the said dynamite and/or blasting caps and explosives are being used for fishing purposes in violation of this Section, and that the possession or discovery in any fishing boat or fish caught or killed by the use of dynamite or other explosives, under expert testimony, shall constitute a presumption that the owner, if present in the fishing boat, or the fishing crew have been fishing with dynamite or other explosives." (Emphasis supplied). Under Section 78 of the Fisheries Act, as amended, any person, association or corporation fishing in deep sea fishery without the corresponding license prescribed in Sections 17 to 22 Article V of the Fisheries Act or any other order or regulation deriving force from its provisions, "shall be punished for each offense by a fine of not more than P5,000.00, or imprisonment, for not more than one year, or both, in the discretion of the Court; Provided, That in case of an association or corporation, the President or manager shall be directly responsible for the acts of his employees or laborers if it is proven that the latter acted with his knowledge; otherwise the responsibility shall extend only as far as fine is concerned: Provided, further, That in the absence of a known owner of the vessel, the master, patron or person in charge of such vessel shall be responsible for any violation of this Act: and Provided, finally, That in case of a second offense, the vessel together with its tackle, apparel, furniture and stores shall be forfeited to the Government" (Emphasis supplied). Under Section 13 of Executive Order No. 389 of December 23, 1950, reorganizing the Armed Forces of the Philippines, the Philippine Navy has the function, among others, "to assist the proper governmental agencies in the enforcement of laws and regulations pertaining to ... fishing ..." (46 OG 5905, 5911). Section 2210 of the Tariff and Customs Code, as amended by PD No. 34 of October 27, 1972, authorized any official or person exercising police authority under the provisions of the Code, to search and seize any vessel or air craft as well as any trunk, package, bag or envelope on board and to search any person on board for any breach or violation of the customs and tariff laws. When the Philippine Navy, upon request of the Fisheries Commissioner, apprehended on August 5 or 6, 1965 the fishing boats Tony Lex III and Tony Lex VI, otherwise known

respectively as Srta. Agnes and Srta. Winnie, these vessels were found to be without the necessary license in violation of Section 903 of the Tariff and Customs Code and therefore subject to seizure under Section 2210 of the same Code, and illegally fishing with explosives and without fishing license required by Sections 17 and 18 of the Fisheries Law (pp. 46-47, rec.).1wph1.t The operation of the fishing boat Tony Lex III was suspended pursuant to the order dated January 28, 1964 issued by the Commissioner of Fisheries pending the final determination of the case against it for illegal fishing with explosives on January 21, 1964 (p. 34, rec.) and remained suspended until its apprehension on August 5 or 6, 1965 (p. 46, rec.). For illegal fishing with explosives on March 23, 1963, the renewal of the fishing boat license of Tony Lex VI was suspended for one year from the time said boat was moored at Pier 14 at North Harbor, Manila, without prejudice to the institution of a criminal case against its owner and/or operator, pursuant to the order dated May 19, 1964 issued by the Commissioner of Fisheries (pp. 35-36, rec.), the motion for reconsideration of which order was denied by the Commissioner of Fisheries in an order dated August 17, 1964 (pp. 41-42, rec.). For illegal fishing with dynamite on March 28, 1963, the operation of Tony Lex VI was suspended by the Commissioner of Fisheries in an order dated April 1, 1963 (p. 62, rec.). For illegal fishing again with explosives on April 25, 1963, the fishing boat Tony Lex VI together with its tackle, apparel, furniture and all other apparatus used in fishing was ordered confiscated and forfeited in favor of the Government and a fine in the amount of P5,000.00 was imposed on its owners-operators, without prejudice to the filing of the necessary criminal action, pursuant to the order of June 2, 1964 of the Commissioner of Fisheries(pp. 37-38, rec.). Again, for comitting the same violation on June 19, 1963, a fine in the amount of P5,000.00 was imposed on the owners-operators of fishing boat Tony Lex VI pursuant to the order of June 4, 1964 issued by the Commissioner of Fisheries (pp. 39-40, rec.).. It appears, therefore, that since January 28, 1964, the fishing boat Tony Lex III was suspended from operating and was ordered moored at Pier 14, North Harbor, Manila (pp. 34, 46-47, rec.); and that the fishing vessel Tony Lex VI was suspended for one year from May 24, 1964 and was actually ordered forfeited to the Government pursuant to the order of June 2, 1964 for repeated violations of Section 12 of the Fisheries Act (pp. 37- 38. rec.).1wph1.t As a matter of fact, when apprehended on August 5 or 6, 1965, both vessels were found to be without any license or permit for coastwise trade or for fishing and unlawfully fishing with explosives, for which reason their owners and crew were accordingly indicted by the Provincial Fiscal of Palawan for illegal fishing with dynamite and without the requisite license (pp. 48-53, rec.).

As heretofore intimated, the two fishing boats were apprehended on numerous occasions for fishing with dynamite from March 28, 1963 to March 11, 1964, which violations private respondent, as owner-operator, sought to compromise by offering to pay a fine of P21,000.00 for all said prior violations. Such previous violations of Sections 12, 17 and 18 of the Fisheries Act committed by the two fishing boats, Tony Lex III and Tony Lex VI, from March 28, 1963 until August 5 or 6, 1965, rendered the said vessels subject to forfeiture under Sections 76 and 78 of the Fisheries Act, as amended. Search and seizure without search warrant of vessels and air crafts for violations of the customs laws have been the traditional exception to the constitutional requirement of a search warrant, because the vessel can be quickly moved out of the locality or jurisdiction in which the search warrant must be sought before such warrant could be secured; hence it is not practicable to require a search warrant before such search or seizure can be constitutionally effected (Papa vs. Mago, L-27360, Feb. 28, 1968, 22 SCRA 857, 871-74; Magoncia vs. Palacio, 80 Phil. 770, 774; Caroll vs. U.S. 267, pp. 132, 149, 158; Justice Fernando, The Bill of Rights, 1972 ed., p. 225; Gonzales, Philippine Constitutional Law, 1966 ed., p. 300). The same exception should apply to seizures of fishing vessels breaching our fishery laws. They are usually equipped with powerful motors that enable them to elude pursuing ships of the Philippine Navy or Coast Guard. Another exception to the constitutional requirement of a search warrant for a valid search and seizure, is a search or seizure as an incident to a lawful arrest (Alvero vs. Dizon, 76 Phil. 637; Justice Fernando, The Bill of Rights, 1972 ed., p. 224). Under our Rules of Court, a police officer or a private individual may, without a warrant, arrest a person (a) who has committed, is actually committing or is about to commit an offense in his presence; (b) who is reasonably believed to have committed an offense which has been actually committed; or (c) who is a prisoner who has escaped from confinement while serving a final judgment or from temporary detention during the pendency of his case or while being transferred from one confinement to another (Sec. 6, Rule 113, Revised Rules of Court). In the case at bar, the members of the crew of the two vessels were caught in flagrante illegally fishing with dynamite and without the requisite license. Thus their apprehension without a warrant of arrest while committing a crime is lawful. Consequently, the seizure of the vessel, its equipment and dynamites therein was equally valid as an incident to a lawful arrest. The alleged compromise approved by the Secretary of Agriculture and Natural Resources on September 13, 1965 (pp. 63-64, 158-159, rec.) cannot be invoked by the respondents because the said compromise referred to about thirty violations of the fisheries law committed by the private respondent from March 28, 1963 to March 11, 1964. The violations by the two vessels of private respondent by reason of which these vessels were apprehended and detained by the Philippine Navy upon request of the Commissioner of Fisheries, were committed on August 5 or 6, 1965.

Moreover, the power to compromise would exist only before a criminal prosecution is instituted; otherwise the Department Secretary or any of his sub-alterns can render criminal prosecutions for violations of the fisheries law a mere mockery. It is not in the public interest nor is it good policy to sustain the viewpoint that the Department Secretary can compromise criminal cases involving public, not private, offenses after the indictment had been instituted in court. The fishing vessels together with all their equipment and the dynamites found therein are not only evidence of the crime of illegal fishing but also subject to forfeiture in favor of the Government as instruments of the crime (Art. 45, Revised Penal Code, Sec. 78, Act No. 4003, as amended). Section 80(j) of Act No. 4003, as amended, precludes such a compromise the moment the Fisheries Commissioner decides to prosecute the criminal action in accordance with Sections 76 and 78 of the other penal provisions of the fisheries law. Furthermore, any compromise shall be upon the recommendation of the Fisheries Commission (Section 80[i], Act No. 4003), which did not recommend such a compromise for the violation on August 5 or 6, 1965 of Section 12 in relation to Sections 76 and 78 of Act No. 4003, as amended. On the contrary, the Fisheries Commissioner requested the Provincial Fiscal to institute the criminal cases (pp. 43-45, rec.) and the Provincial Fiscal filed the corresponding informations docketed as Criminal Cases Nos. 3416 and 3417 on September 30, 1965 against the owners and the members of the crew of the vessels (pp. 48-53, rec.). It should be noted that in the first indorsement dated September 13, 1965 of the Secretary of Agriculture and Natural Resources approving the compromise fine of P21,000.00 for the various violations committed previous to August 5 or 6, 1965 (pp. 3442, 47, 58-64, 149-155, 158-159, rec.), the Department Secretary "believes that the offer made by the company was an implied admission of violations of said provisions of the Fisheries Law and regulations, ..." (pp. 63, 158, rec.). The said approval was granted after the private respondent filed a motion for reconsideration of the indorsement dated March 5, 1965 of the Secretary of Agriculture and Natural Resources disapproving the offer by private respondent to pay the fine by way of compromise. There can be no dispute that the term fishing boat (employed in the second paragraph of Section 12 of the Fisheries Act applies to the vessels Tony Lex III and Tony Lex VI. Even private respondent refers to said fishing boats as fishing vessels "engaged in fishing operations" or "in commercial fishing" in paragraph IV of its complaint in Civil Case No. 62799 (p. 18, rec.), as well as in its various communications to the Fisheries Commissioner (pp. 60-61, 65, 82, rec.).1wph1.t The two fishing vessels Tony Lex III and Tony Lex VI likewise fall under the term vessel used in Sections 17, 76 and 78, as well as the term boats utilized in the second paragraph of Section 76 of the Fisheries Act. They can also fall under the term fishing equipment employed in Section 4 of Republic Act No. 3512; because a fishing equipment is never complete and cannot be effectively used in off-shore or deep-sea fishing without the fishing boat or fishing vessel itself. And these two vessels of private respondent certainly come under the term fishing vessels employed in paragraph 5 of Section 4 of the same Republic Act 3512 creating the Fisheries Commission.

Hence, no useful purpose can be served in trying to distinguish between boat and vessel with reference to Tony Lex III and Tony Lex VI. As a matter of fact, the accepted definition of vessel includes "every description of water craft, large or small, used or capable of being used as a means of transportation on water" (Cope versus Vallete, etc., 199 U.S. 625; U.S. vs. Holmes, 104 Fed. 884; Charles Barnes Co. vs. One Dredge Boat, 169 Fed. 895; and Yu Con vs. Ipil, 41 Phil. 780). The word boat in its ordinary sense, means any water craft (Monongahela River Construction, etc. vs. Hardsaw, 77 NE 363, 365). The fishing boats Tony Lex III and Tony Lex VI are likewise vessels within the meaning of the term vessel used in Sections 903 and 2210 of the Tariff and Customs Code. WHEREFORE, THE PETITION IS HEREBY GRANTED AND THE ORDER OF RESPONDENT JUDGE DATED OCTOBER 18, 1965, THE WRIT OF PRELIMINARY MANDATORY INJUNCTION ISSUED THEREUNDER AND THE ORDER DATED NOVEMBER 23, 1965, ARE HEREBY SET ASIDE AS NULL AND VOID, WITH COSTS AGAINST PRIVATE RESPONDENT. People v. CFI of Rizal search of moving vehicle Carroll doctrine search of moving vehicles or automobiles no search warrant needed G.R. No. L-41686 November 17, 1980 PEOPLE OF THE PHILIPPINES, petitioner, vs. COURT OF FIRST INSTANCE OF RIZAL, BRANCH IX, QUEZON CITY, presided by HON. ULPIANO SARMIENTO, JESSIE HOPE and MONINA MEDINA, respondents.

GUERRERO, J.: This original petition for certiorari seeks to nullify the Order dated August 20, 1975 issued by District Judge Ulpiano Sarmiento in Criminal Case No. Q-3781 which stalled the prosecution of respondents Sgt. Jessie C. Hope and Monina Medina for the alleged violation of section 3601 1 of the Tariff and Customs Code. The order declared as inadmissible in evidence the allegedly smuggled articles obtained by apprehending agents in the course of a warrantless search and seizure. Dispositively, the order decreed:
WHEREFORE, in accordance with Article IV, Sec. 4, paragraph 2 of the present Constitution, the boxes and the watches and bracelets contained therein seized from the car of the accused Sgt. Jessie C. Hope, are hereby declared inadmissible in evidence in this case; likewise, the pictures taken of said items attempted to be presented as evidence in the instant case is hereby declared in admissible as evidence against the accused.

SO ORDERED.

The records disclose that one week before February 9, 1974, the Regional AntiSmuggling Action Center (RASAC) was informed by an undisclosed Informer that a shipment of highly dutiable goods would be transported to Manila from Angeles City on a blue Dodge car. Spurred by such lead, RASAC Agents Arthur Manuel and Macario Sabado, on the aforesaid date and upon order of the Chief of Intelligence and Operations Branch, RASAC-MBA, Col. Antonio Abad, Jr., stationed themselves in the vicinity of the toll gate of the North Diversion Road at Balintawak, Quezon City. At about 6:45 A.M. of the same day, a light blue Dodge car with Plate No. 21-87-73, driven by Sgt. Jessie Hope who was accompanied by Monina Medina approached the exit gate and after giving the toll receipt sped away towards Manila. The RASAC agents gave a chase and overtook Sgt. Hope's car. Agent Sabado blew his whistle and signaled Sgt. Hope to stop but the latter instead of heeding, made a U-turn back to the North Diversion Road, but he could not go through because of the buses in front of his car. At this point, the agents succeeded in blocking Sgt. Hope's car and the latter stopped. Manuel and Sabado who were in civilian clothes showed their Identification cards to respondents and introduced themselves as RASAC agents. The Agents saw four (4) boxes on the back seat of the Dodge and upon inquiry as to what those boxes were, Sgt. Hope answered "I do not know." Further, respondents were asked where they were bringing the boxes, to which respondent Medina replied that they were bringing them (boxes) to the Tropical Hut at Epifanio de los Santos. Agent Sabado boarded the Dodge car with respondents while Agent Manuel took their own car and both cars drove towards Tropical Hut making a brief stop at the Bonanza where Agent Manuel called up Col. Abad by telephone. Arriving at the Tropical Hut, the party, together with Col. Abad who had joined them waited for the man who according to Monina Medina was supposed to receive the boxes. As the man did not appear, Col. Abad "called off the mission" and brought respondents and their car to Camp Aguinaldo arriving there at about 9:00 A.M. (Respondents' Memorandum, records, pp. 180-183). An inspection of Sgt. Hope's car at Camp Aguinaldo yielded eleven (11) sealed boxes, four (4) on the rear seat and seven (7) more in the baggage compartment which was opened on orders of Col. Abad. On the same order of the intelligence officer, the boxes were opened before the presence of respondents Hope and Medina, representatives of the Bureau of Internal Revenue, Bureau of Customs, P.C., COSAC and photographers of the Department of National Defense. The contents of the boxes revealed some "4,441 more or less wrist watches of assorted brands; 1,075 more or less watch bracelets of assorted brands" (based on a later inventory), supposedly untaxed. As consequence, thereof, ASAC Chairman General Pelagio Cruz requested the Bureau of Customs to issue a Warrant of Seizure and Detention against the articles including the Dodge car. The Collector of Customs did issue the same on February 12, 1974. It was admitted, however, that when the apprehending agents arrested respondents and

brought them together with the seized articles to the ASAC Office in Camp Aguinaldo, the former were not armed with a warrant of arrest and seizure. In conjunction with the Warrant of Seizure and Detention issued by the Collector of Customs, seizure proceedings were instituted and docketed as Seizure Identification No. 14281 against the wrist watches and watch bracelets pursuant to Section 2530 (m) 1 of the Tariff and Customs Code, and Seizure Identification No. 14281-A against the Dodge car pursuant to Section 2530(k) of the same Code. 2 During the hearing of the aforesaid cases, respondents disclaimed ownership of the seized articles. Ownership was instead claimed by one Antonio del Rosario who intervened in the proceedings. The claimant-intervenor testified that he bought the watches and bracelets from Buenafe Trading as evidenced by a sales invoice certified to be authentic by the BIR Revenue Regional Office No. 6 of Quezon City, which transaction was entered in the book of accounts of aforesaid claimant; that the same articles were brought to a buyer in Angeles City, but when the sale failed to materialize, claimant contracted respondent Monina Medina to transport back the boxes to Manila for a consideration of P1,000.00 without disclosing the contents thereof which claimant simply represented as PX goods; that when he bought the watches from Buenafe, he presumed that the corresponding duties have already been paid, only to be surprised later on when he was informed that the same were seized for non-payment of taxes. On the other hand, respondent Hope testified to the effect that at the time of apprehension, he had no knowledge of the contents of the boxes, and granting that he had such knowledge, he never knew that these are untaxed commodities that he consented to transport said boxes from Angeles City to Manila in his car upon request of his girl friend Monina as a personal favor; that he was not present when the boxes were loaded in his car nor was he ever told of their contents on the way. On the part of respondent Monina Medina, she testified that what she did was only in compliance with the agreement with Mr. Del Rosario to transport the boxes and deliver them to a certain Mr. Peter at the Tropical Hut who will in turn give her the contracted price; that Mr. Del Rosario did not reveal the contents of the boxes which she came to know of only when the boxes were opened at Camp Aguinaldo. As there was not enough evidence to controvert the testimonies of respondents and the narration of claimant Antonio del Rosario, the Collector of Customs issued his decision in the seizure cases on April 1, 1975 declaring that the seized articles including the car are not subject of forfeiture. The dispositive portion of this decision reads:
WHEREFORE, by virtue of Section 2312 of the Tariff and Customs Code, it is hereby ordered and decreed that the subject motor vehicle, one (1) Dodge, Model 1965, Motor No. 33859, Serial No. W357348361, File No. 2B-1884, with Plate No. EH 21-87, '73 covered by Seizure Identification No. 14281-A be, as it is hereby declared released to its registered owner, Jessie C. Hope, upon proper Identification. Relative to Seizure Identification No. 14281, it is further ordered and decreed that the subject matter thereof to wit: 4,606 pcs. of assorted brands of wrist watches, 1,399 pieces of assorted brands of wrist bracelets and 100 pcs. of tools be, as they are hereby likewise declared released to the rightful owner thereof, Antonio del Rosario, upon payment of the levitable duties, taxes and other charges due thereon plus a fine equivalent to 100% of the duties and

taxes thereof. Furthermore, should claimant-intervenor fail to pay the assessable duties, taxes and other charges owing from the aforestated articles within 30 days from the time this decision becomes final and unappealable, the same shall be deemed abandoned in favor of the government to be disposed of in the manner provided for by law.

Meanwhile, on March 14, 1974, after the requisite preliminary investigation, the City Fiscal of Quezon City, finding the existence of a prima facie case against respondents Hope and Medina, filed Criminal Case No. Q-3781 in the Court of First Instance of Rizal (Quezon City). Upon arraignment on April 23, 1974, respondents pleaded not guilty. Trial commenced on January 28, 1975 and while the prosecution through its first witness, Agent Macario Sabado, was adducing as evidence the pictures of the eleven (11) boxes containing the assorted watches and watch bracelets, counsel for respondents objected to the presentation of the pictures and the subject articles on the ground that they were seized without the benefit of warrant, and therefore inadmissible in evidence under Section 4(2), Article IV of the New Constitution. After the parties have argued their grounds in their respective memoranda, respondent trial court issued the questioned order of August 20, 1975 as cited earlier. The prosecutions motion for reconsideration was denied on September 30, 1975. Hence, this petition which was treated as a special civil action in Our Resolution of May 5, 1976. The substantive issue as urged in the petition is whether or not the seizure of the merchandise in a moving vehicle by authorized agents commissioned to enforce customs laws without warrant of seizure breaches the constitutional immunity against unreasonable search and seizure and therefore, such merchandise are inadmissible in evidence. Corollary to the issue is, has the trial court gravely abused its discretion in finding the affirmative? The State holds on the proposition that the rules governing search and seizure had been liberalized when a moving vehicle is the object of the search and the necessity of a prior warrant has been relaxed on the ground of practicality, considering that before a warrant could be obtained, the place, things and persons to be searched must be described to the satisfaction of the issuing judge a requirement which borders on impossibility in the case of smuggling effected by the use of a moving vehicle that can transport contraband from one place to another with impunity. Petitioner vigorously contends that contraband may be seized without necessity of a search warrant since the Constitution does not guaranty immunity to smugglers and that a warrantless seizure of contraband in a moving vehicle is justified by the traditional exception attached to the Fourth Amendment of the U.S. Constitution, and such exception must be adopted in interpreting the relevant provision in the new Philippine Constitution. As counter argument, respondents maintain that the decision of the Collector of Customs in their seizure cases which has now become final and unappealable has made no pronouncement that the subject articles are smuggled items. More so, the decision has entirely cleared respondents of any liability or responsibility in the alleged smuggling activity and as a consequence, the decision has the direct effect of deciding finally that the watches and bracelets are not smuggled and that respondents have not violated the customs and tariff laws as charged in the criminal complaint. Respondents

argue further that the interception of accused Jessie Hope's car by RASAC Agents while in the course of a normal trip without any order of the court and without having shown that the interception was necessary in the interest of national security, public safety or public health, is an impairment of the liberty of travel under section 5, Article IV of the 1973 Constitution. Finally, they claim that the agents had one week's time before the date of apprehension to secure the necessary warrant but since they failed to get this court order, the search of Hope's car and the spontaneous seizure of the boxes loaded therein and the contents thereof is a violation of the constitutional guarantee against "unreasonable searches and seizure of whatever nature and for any purpose" under section 3, Article IV of the fundamental law. We find for petitioner. The opposing counsel's attempt to draw an Identity between the seizure cases and the present criminal action to the ultimate end that the decision in the former should be made decisive of the issue of criminal liability must be overruled. It is not accurate to say that the Collector of Customs made no findings that the articles were smuggled. In fact, what the Collector stated was that the prosecution failed to present the quantum of evidence sufficient to warrant the forfeiture of the subject articles (Pages 128 and 130 of Annex "E", Records, p. 109). In a general sense, this does not necessarily exclude the possibility of smuggling. But if the aim of a confirmation that the goods are indeed smuggled, is to draw an inference to tie up respondents' criminal liability, the Collector is not duty bound, nor is there any need for him to arrive at such a conclusion. It is quite clear that seizure and forfeiture proceedings under the tariff and customs laws are not criminal in nature as they do not result in the conviction of the offender nor in the imposition of the penalty provided for in section 3601 of the Code 3. As can be gleaned from Section 2533 of the code, seizure proceedings, such as those instituted in this case, are purely civil and administrative in character, the main purpose of which is to enforce the administrative fines or forfeiture incident to unlawful importation of goods or their deliberate possession. The penalty in seizure cases is distinct and separate from the criminal liability that might be imposed against the indicted importer or possessor and both kinds of penalties may be imposed.
4

In the case at bar, the decision of the Collector of Customs, as in other seizure proceedings, concerns the res rather than the persona. The proceeding is a probe on contraband or illegally imported goods. These merchandise violated the revenue law of the country, and as such, have been prevented from being assimilated in lawful commerce until corresponding duties are paid thereon and the penalties imposed and satisfied either in the form of fines or of forfeiture in favor of the government who will dispose of them in accordance with law. The importer or possessor is treated differently. The fact that the administrative penalty befalls on him is an inconsequential incidence to criminal liability. By the same token, the probable guilt cannot be negated simply because he was not held administratively liable. The Collector's final declaration that the articles are not subject to forfeiture does not detract his findings that untaxed goods were transported in respondents' car and seized from their possession by agents of the law. Whether criminal liability lurks on the strength of the provision of the Tariff and Customs Code adduced in the information can only be determined in a separate criminal action. Respondents' exoneration in the administrative cases cannot deprive

the State of its right to prosecute. But under our penal laws, criminal responsibility, if any, must be proven not by preponderance of evidence but by proof beyond reasonable doubt. Considering now the critical area of the dispute, under the law, the authority of persons duly commissioned to enforce tariff and customs laws is quite exceptional when it pertains to the domain of searches and seizures of goods suspected to have been introduced in the country in violation of the customs laws. This Court had occasion to recognize this power granted to persons having police authority under Section 2203 of the Code, who in order to discharge their official duties more effecttively
... may at anytime enter, pass through, or search any land or inclosure of any warehouse, store or other building not being a dwelling house. (Section 2208, emphasis supplied) ... (to) go aboard any vessel or aircraft within the limits of any collection district, and to inspect, search and examine said vessel or aircraft and any trunk, package, box or envelope on board, and search any person on board the said vessel or aircraft and to this end to hail and stop such vessel or aircraft if under way. to use all necessary force to compel compliance; and if it shall appear that any breach or violation of the customs and tariff laws of the Philippines has been committed, whereby or in consequence of which such vessels or aircrafts, or the article, or any part thereof, on board of or imported by such vessel or aircrafts, is hable to forfeiture to make seizure of the same or any part thereof. The power of search herein above given shall extend to the removal of any false bottom, partition, bulkhead or other obstruction, so far as may be necessary to enable the officer to discover whether any dutiable or forfeitable articles may be concealed. (Section 2210)

or,
... (to) open and examine any box, trunk, envelope or other container wherever found when he has reasonable cause to suspect the presence therein of dutiable or prohibited article or articles introduced into the Philippines contrary to law, and likewise to stop, search and examine any vehicle, beast or person reasonably suspected of holding or conveying such article as aforesaid (Section 2211, emphasis supplied)

As enunciated in the leading case of Papa v. Mago 5, in the exercise of the specific functions aforecited, the Code does not mention the need of a search warrant unlike Section 2209 which explicitly provides that a "dwelling house may be entered and searched only upon warrant issued by a judge (or justice of the peace), upon swom application showing probable cause and particularly describing the place to be searched and person or thing to be seized." Aware of this delineation, the Court in that case expressed the considered view that "except in the case of the search of a dwelling house, persons exercising police authority under the customs law may effect search and seizure without a search warrant in the enforcement of customs laws. The rationale of the Mago ruling was nurtured by the traditional doctrine in Carroll v. United States 6 wherein an imprimatur against constitutional infirmity was stamped in favor of a warrantless search and seizure of such nature as in the case at bar. On this stable foundation We refute the constitutional charge of respondents that the

warrantless seizure violated Article IV, Section 3 of the 1973 Constitution, which finds origin in the Fourth Amendment of the American Constitution 7 The Carroll doctrine arose from the indictment and conviction of George Carroll and partner for transporting in an automobile intoxicating liquor in violation of the National Prohibition Act. They assailed the conviction on the ground that the trial court admitted in evidence two of the sixty-eight bottles found by searching the automobile and eventual seizure of the same allegedly in violation of the 4th Amendment, and therefore that the use of the liquor as evidence was improper. 8 To paraphrase the significant views of Mr. Chief Justice Taft, the legislative history of the Act clearly established the intent of Congress to make a distinction between the necessity for a search warrant in the search of private dwellings and that of automobiles and other road vehicles in the enforcement of the Act. This distinction is consistent with the 4th Amendment since the latter does not denounce an searches or seizures, but only such as are unreasonable. Searches and seizures without warrant are valid if made upon probable cause, that is, upon a belief reasonably arising out of circumstances known to the seizing officer, that an automobile or other vehicle contains that which by law is subject to seizure and destruction. 9 Similarly, other statutes of the Union such as the Act of 1789, Act of August 4, 1790, and Act of March 3, 1815, among others, construed in the light of the 4th Amendment had recognized the distinctive feature of a warrantless search of a ship motorboat, wagon, or automobile for contraband goods where it is not practicable to secure a warrant because the vehicle can be quickly moved out of the locality or jurisdiction in which the warrant must be sought. 10 In such a situation, what appears to the measure of legality of the seizure was formulated in this sense: "that the seizing officer shall have reasonable or probable cause for believing that the automobile which he stops and seizes has contraband liquor therein which is being illegally transported. " Therein the guarantee of the 4th Amendment was fulfilled. Where seizure is impossible except without warrant, the seizing officer acts unlawfully and at his peril unless he can show the court probable cause. 11 The counsel for the State is candid enough to admit that the Anti-Smuggling Action Center tries its best to follow-up the more promising tips and information from informers, but ever often, the information proves false or the smugglers are forewarned. 12 It is quite true the ASAC received one such information several days or a week before the encounter; but the fact that its agents failed to obtain a warrant in spite of the time allowance is not a sign that they have been remiss in their duty. The records hardly reveal anything certain and confirmatory of the report during the said period except the general knowledge that some highly dutiable goods would be transported from Angeles City to Manila in a blue Dodge automobile. Not even the trial court has made any findings that ASAC has established with exactitude the place to be searched and the person or thing to be seized. Lacking this essential determination, the agents could not have possibly secured a valid warrant even if they had foreseen its compelling necessity. For one thing, the information could have been just another false alarm. Providentially, however, things turned out differently when in the morning of February 9, 1974, the undisclosed Informer himself went along with the agents to the rendezvous point where at the appointed time he positively Identified an approaching car as the one

described by him a week earlier to be the suspected carrier of untaxed merchandise. Clearly therefore, the agents acted not on the basis of a mere hearsay but on a confirmed information worthy of belief and probable cause enough for them to adopt measures to freeze the fleeting event. We need not argue that the subjective phase of the police action taken by the ASAC Agents to effect the apprehension of the suspected violators can be anything less than the ensuing interception and stoppage of respondents' vehicle after a short chase. Neither can We sustain the argument that in doing so, the agents violated respondents' constitutional "liberty of travel". To recall again Mr. Chief Justice Taft: "(B)ut those lawfully within the country, entitled to use the public highways, have a right to free passage without interruption or search unless there is known to a competent official authorized to search, probable cause for believing that their vehicles are carrying contraband or illegal merchandise." 13 What followed next in the scene was a simple inquiry as to the contents of the boxes seen inside the car. Respondents' baffled denial of knowledge thereof could not but only heighten the suspicion of a reasonable and inquisitive mind. Thus, the probable cause has not been any less mitigated. The purpose of the constitutional guarantee against unreasonable searches and seizures is to prevent violations of private security in person and property and unlawful invasion of the sanctity of the home by officers of the law acting under legislative or judicial sanction and to give remedy against such usurpation when attempted. 14 The right to privacy is an essential condition to the dignity and happiness and to the peace and security of every individual, whether it be of home or of persons and correspondence. 15 The constitutional inviolability of this great fundamental right against unreasonable searches and seizures must be deemed absolute as nothing is more closer to a man's soul than the serenity of his privacy and the assurance of his personal security. Any interference allowable can only be for the best of causes and reasons. We draw from the context of the Constitution that an intended search or seizure attains a high degree of propriety only when a probable cause duly determined is branded on a warrant duly issued by a judge or other responsible person as may be authorized by law. Not invariably, however, the reasonableness or unreasonableness of the interference is not wholly defendent on the presence of a warrant or the lack of it. In the ordinary cases where warrant is indispensably necessary, the mechanics prescribed by the Constitution and reiterated in the Rules of Court must be followed and satisfied. But We need not argue that there are exceptions. Thus, in the extraordinary events where warrant is not necessary to effect a valid search or seizure, or when the latter cannot be performed except without warrant, what constitutes a reasonable or unreasonable search or seizure becomes purely a judicial question, determinable from the uniqueness of the circumstances involved, including the purpose of the search or seizure, the presence or absence of probable cause, the manner in which the search and seizure was made, the place or thing searched and the character of the articles procured. 16 The ultimate question then, if any, that should confront the actuations of the ASAC Agents in this case is whether the warrantless search and seizure conducted by them is lawful or not. We have already seen that what they did was a faithful performance of a

duty authorized under the Tariff and Customs Code directing them as authorized agents to retrieve articles reasonably suspected of having been possessed, issued or procured in violation of the tariff laws for which the government has a direct interest. The official capacity of the agents has never been questioned by respondents. Neither did respondents raise an issue on the constitutionality of the law giving the agents the power to act as mandated. There 'is no question that the Agents have not exceeded their authority nor have they acted so licentiously to bear upon respondents moral embarrassment or substantial prejudice beyond what is necessary. The purpose of the search and seizure is more than clear to Us, hence, We rule out the suspicion that the intention is only to elicit evidence to be used against respondents. We do not see strong justification for the trial court's failure to recognize the circumstances at bar as among the "rare cases" which it admittedly conceded to be exempted from the requirement of a warrant. 17 The lapse lies on the dismal gap in the trial court's developmental treat- ment of the law on arrest, search and seizure. It missed the vital distinction emphatically laid down in Boyd v. United States 18 which was cited in Carroll with "particular significance and applicability." Thus, We quote Mr. Justice Bradley in Boyd:
... The search and seizure of stolen or forfeited goods, or goods liable to duties and concealed to avoid the payment thereof, are totally different things from a search for and seizure of a man's private books and papers for the purpose of obtaining information therein contained, or of using them as evidence against him, The two things differ in toto coelo. In the one case, the government is entitled to the possession of the property; in the other it is not. The seizure of stolen goods is authorized by the common law; and the seizure of goods forfeited for a breach of the revenue laws or concealed to avoid the duties payable on them, has been authorized by English statutes for at least two centuries past; and the like seizure have been authorized by our revenue acts from the commencement of the government. The first statute passed by Congress to regulate the collection of duties, the Act of July 31, 1789. 1 State at L. 29, 43, chap. 5, contains provisions to this effect. As this act was passed by the same Congress which proposed for adoption the original Amendments to the Constitution, it is clear that the members of that body did not regard searches and seizures of this kind as 'unreasonable' and they are not embraced within the prohibition of the Amendment. So also the supervision authorized to be exercised by officers of the revenue over the manufacture of custody of excisable articles, and the entries thereof in books required by law to be kept for their inspection, are necessarily excepted out of the category of unreasonable searches and seizures. So also the laws which provide for the search and seizure of articles and things which it is unlawful for a person to have in his possession for the purpose of issue or disposition, such as counterfeit coin, lottery tickets, implements of gambling, etc. are not within this category. Commonwealth v. Dana, 2 Met 329. Many other things of this character might be enumerated. (Emphasis supplied).

Recently, in Viduya v. Berdiago 19 " this Court reiterated the controlling force of the Papa v. Mago ruling hereinbefore cited and the persuasive authority of the leading decision in Carroll v. U.S., supra, and in explaining the rationale of the doctrine significantly said that "(i)t is not for this Court to do less than it can to implement and enforce the mandates of the customs and revenue laws. The evils associated with tax evasion must be stamped out without any disregard, it is to be affirmed, of any constitutional right ...

The circumstances of the case at bar undoubtedly fall squarely within the privileged area where search and seizure may lawfully be effected without the need of a warrant. The facts being no less receptive to the applicability of the classic American ruling, the latter's force and effect as well as the Mago decision must be upheld and reiterated in this petition. the find that the constitutional guarantee has not been violated and the respondent court gravely erred in issuing the order of August 20, 1975 declaring as inadmissible evidence the items or articles obtained and seized by the apprehending agents without any search warrant, as well as the pictures of said items attempted to be presented as evidence against the accused. Notwithstanding the reversal and setting aside of the order of respondent judge assailed herein, thereby allowing the introduction and admission of the subject prohibited articles in the trial of the accused Jessie C. Hope and Monina Medina for alleged smuggling, in the interest of speedy justice, the prosecution is directed forthwith to re-assess and reevaluate the evidence at its disposal, considering the lapse of time since the trial commenced on June 28, 1975 and was thus delayed due to the filing of the instant certiorari petition and that on April 1, 1975, after seizure proceedings initiated by the Collector of Customs, the said articles were ordered released upon payment of the leviable duties, taxes and other charges due thereon plus a fine equivalent to 100% of the duties and taxes thereof. After such re-assessment and re-evaluation, the prosecution must promptly take the necessary action on the premises for the protection of the rights and interests of all parties concerned. WHEREFORE, the Order appealed from is hereby set aside and the case is ordered remanded for further trial and reception of evidence without excluding the articles subject of the seizure or for such action as the prosecution may take after the reassessment and re-evaluation of its evidence as hereinabove directed. This judgment is immediately executory. SO ORDERED.

Pacis vs. Pamaran


56 SCRA 16 (1974)

Power of Acting Commissioner of Customs to issue a warrant of seizure and detention

FACTS: Respondent Ricardo Santos is the owner of a Mercury automobile, model 1957, brought into the country without payment of customs duty and taxes because its original owner, Donald James Hatch,

was tax-exempt. Santos later on paid P311.00 for customs duty and taxes. On July 22, 1964, Acting Collector of Customs Pedro Pacis was informed by the General Affairs Administration of the Department of National Defense that the automobile was a hot car. By virtue thereof, Pacis, through his subordinates, looked into the records of his office and found that although the amount of P311.00 was already paid for customs duty, the amount collectible on the said car should be P2,500.00, more or less. Based on such discrepancy, he instituted seizure proceedings and issued a warrant of seizure and detention. The automobile was also taken by the Department of National Defense agents and brought to the General Affairs Administration for compound. In answer, Santos filed a criminal complaint against Pacis for usurpation of judicial functions with the City Fiscal of Manila, Manuel Pamaran, alleging that Pacis did not have authority to issue such warrant of seizure and detention. ISSUE:

W/N petitioner, in the discharge of his official function, lay himself open to a criminal prosecution for usurpation of judicial functions

HELD: It is undeniable that petitioner, as Acting Collector of Customs for the Port of Manila, had the requisite authority for the issuance of the contested warrant of seizure and detention for the automobile owned by respondent Ricardo Santos. What was done by him certainly could not be the basis of a prosecution for the usurpation of judicial functions. The remedy of prohibition lies.

Lopez and Velasco vs. Commissioner of Customs


Confusing case where it is adverse to later jurisprudence that a waiver of right can only be waived by the person whoseright against unreasonable search and seizure was invaded. Facts: After the surveillance conducted by the respondentsNBI officers, NBI and PC Davao officers went to the room(Rm 22 0 ) rented by the petitioner, Tomas Velasco, to searchand seized articles papers and documents including a .45 calpistol, that became evidence that commodities confiscated inthe wharf (MV Jojo Lema) allegedly smuggled fromIndonesia to the country (sacks of coffee beans and copra).The search was without a search warrant, however,the officers have successfully confiscated the articles byvirtue of the consent of the petitioners wife (Teofila ibanez)who also is an occupant of the room rented by the petitioner.The petitioner contends that the consent given byIbanez cannot be regarded since she was not the legal wifeof the petitioner Velasco, but a certain Corazon Velasco.The decision of CTA is adverse by the petitioner,affirmed by the SC.Issue:Was the consent valid to justify the warrantlesssearch and seizure?Held: As far as the decision is concerned, yes, theconsent given by Ibanez is a valid so as to dispense thenecessity of a search warrant.The court ruled that the mere fact that Ibanez ispresent in a room rented by the petitioner, her consent wouldlead to belief that her consent as an alleged wife of thepetitioner and that it would be an act on behalf of thepetitioner. Note: Exact decision penned by J. LaurelThere was an attempt on the part of petitioners tocounteract the force of the above recital by an affidavit of oneCorazon Y. Velasco, who stated that she is the legal wife of petitioner Tomas Velasco, and another by such petitioner himself reiterating such a fact and that the person who waspresent at his hotel room was one Teofila Ibaez, "amanicurist by occupation." Their effort appurtenant thereto isdoomed to failure. If such indeed were the case, then it ismuch more easily understandable why that person, TeofilaIbaez, who could be aptly described as the wrong person atthe wrong place and at the wrong time, would have signifiedher consent readily and immediately.Under the circumstances, that was the most prudentcourse of action. It would save her and even petitioner Velasco himself from any gossip or innuendo. Nor could theofficers of the law be blamed if they would act on theappearances. There was a person inside who from allindications was ready to accede to their request. Evencommon courtesy alone would have precluded them frominquiring too closely as to why she was there. Under all thecircumstances, therefore, it can readily be concluded thatthere was consent sufficient in law to dispense with the needfor a search warrant. The petition cannot, therefore, prevail.

Lopez vs. Commissioner of Customs [GR L-27968, 3 December 1975] Second Division, Fernando (J): 4 concur, 1 took no part Facts:

M/V Jolo Lema had been under strict surveillance by the combined team of agents of the NBI, PC, RASAC, and City Police of Davao prior to its apprehension at a private wharf inBatjak, Sasa, Davao City. M/V [Jolo Lema] was skippered (sic) by Capt. Aquilino Pantinopleand chartered by Mr. Tomas Velasco. During the period from the latter part of August toSeptember 18, 1966, the said vessel was in Indonesian waters where it loaded copra and coffee beans from Taruna, Pitta, and Mangenito, all of Indonesia. In its trip to Indonesia it broughtvarious merchandise from the Philippines which were exchanged and/or bartered for copra andcoffee beans and subsequently taken to Davao City. Said vessel passed Marore, Indonesia on 18 September 1966 on its a way to Tahuna, Indonesia before proceeding to Davao City where it wasapprehended on 19 September 1966. At about 3:00 p.m. of the said day, when the vessel wassearched and after Captain Pantinople informed the team that Velasco, the charterer of the vessel,had other documents showing that vessel came from Indonesia carrying smuggled copra andcoffee, a combined team of Constabulary and Regional Anti-Smuggling Center operativesheaded by Earl Reynolds, Senior NBI Agent of Davao, proceeded to the Velascos room at theSkyroom Hotel in Davao City, to ask for said document. Velasco was not inside the hotel roomwhen they entered the room. There are conficting claims whether the manicurist Teofila Ibaezor whether Velascos wife, who was allegedly inside the room at that time, voluntarily allowedthe police officers to enter; and whether the police officers forcibly opened luggages and boxesfrom which only several documents and papers were found, then seized, confiscated and took away the same, or whether Mrs. Velasco volunteered to open the suitcases and baggages of Velasco and delivered the documents and things contained therein to Reynolds. The Collector of Customs of Davao seized 1,480 sacks of copra and 86 sacks of coffee from the M/V motor vesselJolo Lema. The seizure was declared lawful by the Court of Tax Appeals, and its decision wasaffirmed by the Supreme Court on 29 November 1974 in Nasiad vs. Court of Tax Appeals (GR L29318, November 29, 1974, 61 SCRA 238). In the present special civil action for certiorari, prohibition and mandamus; the only question left then is whether the search conducted by a partyheaded by Reynolds without the search warrant for the hotel room of Velasco, who entered into acontract with Jose G. Lopez, the awardee of such Philippine Reparations Commission vessel, for its operation and use ostensibly for fishing, is violative of such constitutional provision. Issue: Whether there was consent on the part of the person who was the occupant of the hotelroom then rented by Velasco. Held: There was an attempt on the part of Lopez and Velasco to counteract the force of therecital of the written statement of Teofila Ibaez (allegedly wife of Tomas Velasco) by anaffidavit of one Corazon Y. Velasco, who stated that she is the legal wife of Velasco, and another by Velasco himself; reiterating that the person who was present at his hotel room was oneTeofila Ibaez, a manicurist by occupation. If such indeed were the case, then it is much moreeasily understandable why that person, Teofila Ibaez, who could be aptly described as thewrong person at the wrong place and at the wrong time, would have signified her consent readilyand immediately. Under the circumstances, that was the most prudent course of action. It wouldsave her and even Velasco himself from any gossip or innuendo. Nor could the officers of thelaw be blamed if they would act on the appearances. There was a person inside who from allindications was ready to accede to their request. Even common courtesy alone would have precluded them from inquiring too closely as to why she was there. Under all the circumstances,therefore, it can readily be concluded that there was consent sufficient in law to dispense with theneed for a search warrant.

People vs. dela Cruz [GR 83260, 18 April 1990]


Second Division, Regalado (J): 4 concur Facts: After receiving a confidential report from Arnel, their informant, a buy-bust operationwas conducted by the 13th Narcotics Regional Unit through a team composed of T/Sgt. JaimeRaposas as Team Leader, S/Sgt. Rodelito Oblice, Sgt. Dante Yang, Sgt. Vicente Jimenez, P/Pfc.Adolfo Arcoy as poseur-buyer and Pat. Deogracias Gorgonia at Maliclic St., Tondo, Manila ataround 2:30 p.m. of 4 May 1987 to catch the pusher/s. P/Pfc. Adolfo Arcoy acted as the poseur- buyer with Arnel as his companion to buy marijuana worth P10.00 from the two accused, Juande la Cruz and Reynaldo Beltran. At the scene, it was Juan de la Cruz whom Arcoy firstnegotiated with on the purchase and when Arcoy told De la Cruz that he was buying P10.00worth of marijuana, De la Cruz instructed Reynaldo Beltran to give one aluminum foil of marijuana which Beltran got from his pants pocket and delivered it to Arcoy. After ascertainingthat the foil of suspected marijuana was really marijuana, Arcoy gave the prearranged signal tohis teammates by scratching his head and his teammates who were strategically positioned in thevicinity, converged at the place, identified themselves as NARCOM agents and effected thearrest of De la Cruz and Beltran. The P10.00 marked bill used by Arcoy was found in the possession of Juan de la Cruz together with two aluminum foils and containing marijuana. Juande la Cruz y Gonzales and Reynaldo Beltran y Aniban were charged in Criminal Case 87-54417of the Regional Trial Court (RTC) of Manila with violation of Section 4, Art. II, in relation toSection 21, Article IV of Republic Act 6425, as amended. The court, on 15 March 1988, foundDela Cruz and Beltran guilty beyond reasonable doubt and sentenced each of them to suffer the penalty of reclusion perpetua, with the accessory penalties provided by law; to pay a fine of P20,000.00, without subsidiary imprisonment in case of insolvency, and each to pay one-half of the costs. From this decision, de la Cruz and Beltran appealed. In a letter of the Warden, ManilaCity Jail, dated 3 March 1989, the Court was informed of the death of de la Cruz on 21 February1989. Thus, the criminal case against de la Cruz was dismissed in the Supreme Court resolutionof 25 September 1989. The present appellate proceeding is limited only to Beltran. Issue: Whether the warrantless seizure incidental to the buy-bust operation violates Beltransconstitutional rights against unreasonable search and seizure.

Held: A buy-bust operation is the method employed by peace officers to trap and catch amalefactor in flagrante delicto. It is essentially a form of entrapment since the peace officer neither instigates nor induces the accused to commit a crime. Entrapment is the employment of such ways and means for the purpose of trapping or capturing a lawbreaker from whose mind thecriminal intent originated. Oftentimes, it is the only effective way of apprehending a criminal inthe act of the commission of the offense. While it is conceded that in a buy-bust operation, thereis seizure of evidence from ones person without a search warrant, needless to state a searchwarrant is not necessary, the search being incident to a lawful arrest. A peace officer may,without a warrant, arrest a person when, in his presence, the person to be arrested has committed,is actually committing or is attempting to commit an offense. It is a matter of judicial experiencethat in the arrest of violators of the Dangerous Drugs Act in a buy-bust operation, the malefactorswere invariably caught red-handed. There being no violation of the constitutional right againstunreasonable search and seizure, the confiscated articles are admissible in evidence.

NOLASCO VS PANO147 SCRA 509 (1987)Facts


: The case at bar is for the motion for partial reconsideration of both petitioners and respondents of the SCs decision that the questioned search warrant by petitioners is null and void, that respondents areenjoined from introducing evidence using such search warrant, but such personalities obtained wouldstill be retained, without prejudice to petitioner Aguilar-Roque. Respondents contend that the searchwarrant is valid and that it should be considered in the context of the crime of rebellion, where thewarrant was based. Petitioners on the other hand, on the part of petitioner Aguilar-Roque, contend thata lawful search would be justified only by a lawful arrest. And since there was illegal arrest of AguilarRoque, the search was unlawful and that the personalities seized during the illegal searchshould be returned to the petitioner. The respondents, in defense, concede that the search warrants werenull and void but the arrests were not. Issue : WON the articles seized were illegally obtained. Ruling : Yes. RD : "Any evidence obtained in violation of this . . . section shall be inadmissible for any purpose in anyproceeding" (Sec. 4[2]). This constitutional mandate expressly adopting the exclusionary rule hasproved by historical experience to be the only practical means of enforcing the constitutionalinjunction against unreasonable searches and seizures by outlawing all evidence illegally seized andthereby removing the incentive on the part of state and police officers to disregard such basic rights.What the plain language of the Constitution mandates is beyond the power of the courts to change ormodify. All the articles thus seized fag under the exclusionary rule totally and unqualifiedly and cannotbe used against any of the three petitioners.

People vs Anita Claudio G.R. No. 72564 April 15, 1988 Facts: On or about 21 July 1981, in the Olongapo City, Philippines, the above-named ACCUSED without being lawfully authorized, did then and there willfully, unlawfully and knowingly transport 1.1 kilos of Marijuana dried leaves, which are prohibited drugs for the purpose of selling the same from Baguio City to Olongapo City. Issues: a)Whether or not the accused is also liable Sec. 4, Art. II of R.A. 6425 aside from Sec. 8, Art. II of the same Act? b)Whether warrantless search, seizure, and apprehension is unlawful under Rule 126, Sec.12? Held:

Yes. In the case at bar, alibi does not deserve much credit as it was established only by the accused herself. Moreover, it is a well-established rule that alibi cannot prevail over positive testimony. The judgment appealed from is AFFIRMED. Sec. 4, Art II of R.A. 5425 The provision provides the Sale, Administration, Delivery Distribution and Transportation of Prohibited Drugs where the penalty of life imprisonment to death and a fine ranging from twenty thousand to thirty thousand pesos shall be imposed upon any person who, unless authorized by law, shall sell, administer, deliver, give away to another, distribute, dispatch in transit or transport any prohibited drug, or shall act as a broker in any of such transactions. Although the accused contends that she may not be convicted of this provision, the court held that contention is without merit. A closer perusal of the subject provision shows that it is not only delivery which is penalized but also the sale, administration, distribution and transportation of prohibited drugs. Claudio was caught transporting 1.1 kilos of marijuana, thus the lower court did not err in finding her guilty of violating Sec. 4. As held in the case of People v. Toledo, (140 SCRA 259, 267) "the possession of such considerable quantity as three plastic bags of marijuana leaves and seeds coupled with the fact that he is not a user of prohibited drugs cannot indicate anything except the intention of the accused to sell, distribute and deliver said marijuana. Rule 126, Sec. 12 The provision provides the Search incident to lawful arrest where a person lawfully arrested may be searched for dangerous weapons or anything which may be used as proof of the commission of an offense, without a search warrant in paragraph (12a). Thus, appellant Claudio was caught transporting prohibited drugs. Pat. Daniel Obia did not need a warrant to arrest Claudio as the latter was caught in flagrante delicto. The warrantless search being an incident to a lawful arrest is in itself lawful. (Nolasco v. Pano, 147 SCRA 509). Therefore, there was no infirmity in the seizure of the 1.1 kilos of marijuana.

PEOPLE VS. DEL ROSARIO [234 SCRA 246; G.R. NO. 109633; 20 JUL 1994] Wednesday, February 04, 2009 Posted by Coffeeholic Writes Labels: Case Digests, Political Law

Facts: Accused

was charged and convicted by the trial

court of illegal possession of firearms and illegal possession and sale of drugs, particularly methamphetamine or shabu.

After the issuance of the search warrant, which authorized the search and seizure of an undetermined quantity of methamphetamine and its paraphernalias, an entrapment was planned that led to the arrest of del Rosario and to the seizure of the shabu, its paraphernalias and of a .22 caliber pistol with 3 live ammunition.

Issue: Whether
proper.

or Not the seizure of the firearms was

Held: No.

Sec 2 art. III of the constitution specifically

provides that a search warrant must particularly describe the things to be seized. In herein case, the only objects to be seized that the warrant determined was the methamphetamine and the paraphernalias therein. The seizure of the firearms was unconstitutional. Wherefore the decision is reversed and the accused is acquitted.

** Harvey v Santiago 162 SCRA 840 (1988)


Facts: This is a petition for Habeas Corpus. Petitioners are the following: American nationals Andrew Harvey, 52 and Jonh Sherman 72. Dutch Citizen Adriaan Van Den Elshout, 58. All reside at Pagsanjan Laguna respondent Commissioner Miriam Defensor Santiago issued Mission Orders to the Commission of Immigration and Deportation (CID) to apprehended petitioners at their residences. The Operation Report read that Andrew Harvey was found together with two young boys. Richard Sherman was found with two naked boys inside his room. While Van Den Elshout in the after Mission Report read that two children of ages 14 and 16 has been under his care and subjects confirmed being live-in for sometime now. Seized during the petitioners apprehension were rolls of photo negatives and photos of suspected child

prostitutes shown in scandalous poses as well as boys and girls engaged in sex. Posters and other literature advertising the child prostitutes were also found. Petitioners were among the 22 suspected alien pedophiles. They were apprehended 17 February1988 after close surveillance for 3 month of the CID in Pagsanjan, Laguna. 17 of the arrested aliens opted for selfdeportation. One released for lack of evidence, another charged not for pedophile but working with NO VISA, the 3 petitioners chose to face deportation proceedings. On 4 March1988, deportation proceedings were instituted against aliens for being undesirable aliens under Sec.69 of Revised Administrative Code. Warrants of Arrest were issued 7March1988 against petitioners for violation of Sec37, 45 and 46 of Immigration Act and sec69 of RevisedAdministrative Code. Trial by the Board of Special Inquiry III commenced the same date. Petition for bail was filed 11March 1988 but was not granted by the Commissioner of Immigration. 4 April1988 Petitioners filed a petition for Writ of Habeas Corpus. The court heard the case on oral argument on 20 April 1988. Issues: (1) Whether or Not the Commissioner has the power to arrest and detain petitioners pending determination of existence of probable cause. (2) Whether or Not there was unreasonable searches and seizures by CID agents. (3) Whether or Not the writ of Habeas Corpus may be granted to petitioners. Held: While pedophilia is not a crime under the Revised Penal Code, it violates the declared policy of the state to promote and protect the physical, moral, spiritual and social well being of the youth. The arrest of petitioners was based on the probable cause determined after close surveillance of 3 months. The existence of probable cause justified the arrest and seizure of articles linked to the offense. The articles were seized as an incident to a lawful arrest; therefore the articles are admissible evidences (Rule 126, Section12 of Rules on Criminal Procedure). The rule that search and seizures must be supported by a valid warrant of arrest is not an absolute rule. There are at least three exceptions to this rule. 1.) Search is incidental to the arrest. 2.) Search in a moving vehicle. 3.) Seizure of evidence in plain view. In view of the foregoing, the search done was incidental to the arrest. The filing of the petitioners for bail is considered as a waiver of any irregularity attending their arrest and estops them from questioning its validity. Furthermore, the deportation charges and the hearing presently conducted by the Board of Special Inquiry made their detention legal. It is a fundamental rule that habeas corpus will not be granted when confinement is or has become legal, although such confinement was illegal at the beginning. The deportation charges instituted by the Commissioner ofImmigration are in accordance with Sec37 (a) of the PhilippineImmigration Act of 1940 in relation to sec69 of the RevisedAdministrative code. Section 37 (a) provides that aliens shall be arrested and deported upon warrant of the Commissioner ofImmigration and Deportation after a determination by the Board of Commissioners of the existence of a ground for deportation against them. Deportation proceedings are administrative in character and never construed as a punishment but a preventive measure. Therefore, it need not be conducted strictly in accordance with ordinary Court proceedings. What is essential is that there should be a specific charge against the alien intended to be arrested and deported. A fair hearing must also be conducted with assistance of a counsel if desired.

Lastly, the power to deport aliens is an act of the State and done under the authority of the sovereign power. It a police measure against the undesirable aliens whose continued presence in the country is found to be injurious to the public good and tranquility of the people.
EXC: determination by administrative officials CASES: Morano vs, Vivo, 20 SCRA 562 Facts: Petitioners are Chinese nationals (Chan Sau Wah from Fukein, with a minor childfrom prior marriage, Fu Yan Fun) who were granted a temporary visitors visa as mom-immigrant for 2 months upon posting P4k cash bond to visit a cousin in the Philippines.She soon married to Esteban Morano, a Filipino Citizen, on January 24, 1962 and gaveb i r t h t o a c h i l d , E s t e b a n M o r a n o , J r . A f t e r s e v e r a l e x t e n s i o n s t o p r o l o n g s t a y i n Philippines, their visas expired on Sept. 10, 1962 and were ordered by Commissioner of Immigration (COI) on Aug. 31, 1962 thru a letter, to leave the country on or before Sept.1 0 , 1 9 6 2 w i t h w a r n i n g o f i s s u a n c e o f w a r r a n t o f a r r e s t f o r f a i l u r e t o l e a v e a n d confiscation of bond.Petitioners then filed with the CFI of Manila for Mandamus to compel COI to cancel their ACR, to stop issuing arrest warrant, and preliminary injunction from confiscatingtheir bond. They argue that Chan Sau Wah became a Filipino Citizen upon marriage toE s t e b a n M o r a n o b y v i r t u e o f S e c t i o n 1 5 o f C o m m o n w e a l t h A c t N o . 4 7 3 ( R e v i s e d Naturalization Act). Likewise, it argues that Section 37 of the Naturalization Law is unconstitutional for allowing the COI to issue warrant of arrest and effecting deportationwithout judicial intervention enshrined in the Constitution. CFI decided partly againstpetitioners, thus, COI and petitioners both appealed to SC. Issues: Whether or not the marriage of Chan Sau Wah to Esteban Moranomakes her a Filipino citizen. Whether or not Section 37 of the Naturalization Law empowering theCOI to issue a warrant of arrest, and deport upon a warrant on deportation casesis unconstitutional for are covered by the Constitutional mandate on searchesand seizures without judicial intervention required under the Constitution onsearches and seizures. Ruling:Citizenship. Marriage to a Filipino citizen does not ipso facto make her a Filipinocitizen. She must show that she possess all the qualifications, and none of thedisqualifications required by the Naturalization Law requiring as follows: Valid marriage; and Alien woman herself might be lawfully naturalizedIn the additional stipulation of facts of July 3, 1963, petitioners admit that Chan Sau Wahis not possessed of all the qualifications required by the Naturalization Law. Thus, shedid not become a Filipino citizen. Searches and seizures. Power to deport aliens is an attribute of sovereignty planted onthe accepted maxim of international law, that every sovereign nation has the power, asinherent in sovereignty, and essential to self -preservation, to for bid the entrance of foreigners within its dominions. Section 1 (3), Article III of the Constitution, does notrequire judicial intervention in the execution of a final order of deportation issued in accordance with law. The constitutional limitation contemplates an order of arrest in theexercise of judicial power as a step preliminary or incidental to prosecution or proceedingfor a given offense or administrative action, not as a measure indispensable to carry out avalid decision by a competent official, such as a legal order of deportation, issued by theCommissioner of Immigration, in pursuance of a valid legislation.Petition for mandamus and prohibition with respect to petitioners Chan Sau Wah is hereby denied; and judgment declaring her a citizen of the Philippines, directing COI tocancel her Alien Certificate of Registration and other immigration papers, and declaringthe preliminary injunction with respect to her permanent, are all hereby set aside; and inall other respects, the decision appealed is hereby affirmed.

G.R. No. L-22554 August 29, 1975 DELFIN LIM and JIKIL TAHA, plaintiffs-appellants, vs. FRANCISCO PONCE DE LEON AND ORLANDO MADDELA, defendants-appellees. Ricardo L. Manalilig for plaintiffs-appellants. Iigo R. Pea for defendants-appellees.

MARTIN, J.: Appeal on a question of law from the decision of the Court of First Instance of Palawan in Civil Case No. 416, entitled "Delfin Lim and Jikil Taha vs. Francisco Ponce de Leon and Orlando Maddela", dismissing the complaint of the plaintiffs and ordering them to pay each of the defendants jointly and severally the sum of P500.00 by way of actual damages; P500.00 by way of attorney's fees; and P1,000.00 by way of exemplary damages. On April 29, 1961, plaintiff-appellant Jikil Taha sold to a certain Alberto Timbangcaya of Brooke's Point, Palawan a motor launch named M/L "SAN RAFAEL". A year later or on April 9, 1962 Alberto Timbangcaya filed a complaint with the Office of the Provincial Fiscal of Palawan alleging that after the sale Jikil Taha forcibly took away the motor launch from him. On May 14, 1962, after conducting a preliminary investigation, Fiscal Francisco Ponce de Leon in his capacity as Acting Provincial Fiscal of Palawan, filed with the Court of First Instance of Palawan the corresponding information for Robbery the Force and Intimidation upon Persons against Jikil Taha. The case was docketed as Criminal Case No. 2719. On June 15, 1962, Fiscal Francisco Ponce de Leon, upon being informed that the motor launch was in Balabac, Palawan, wrote the Provincial Commander of Palawan requesting him to direct the detachment commander-in Balabac to impound and take custody of the motor launch. 1 On June 26, 1962, Fiscal Ponce de Leon reiterated his request to the Provincial Commander to impound the motor launch, explaining that its subsequent sale to a third party, plaintiff-appellant Delfin Lim, cannot prevent the court from taking custody of the same. 2 So, on July 6, 1962 upon order of the Provincial Commander, defendantappellee Orlando Maddela, Detachment Commander of Balabac, Palawan, seized the motor launch "SAN RAFAEL" from plaintiff-appellant Delfin Lim and impounded it.

On July 15, 1962 plaintiff-appellant Delfin Lim pleaded with Orlando Maddela to return the motor launch but the latter refused. Likewise, on September 20, 1962, Jikil Taha through his counsel made representations with Fiscal Ponce de Leon to return the seized property to plaintiff-appellant Delfin Lim but Fiscal Ponce de Leon refused, on the ground that the same was the subject of a criminal offense. All efforts to recover the motor launch going to naught, plaintiffs-appellants Delfin Lim and Jikil Taha, on November 19, 1962, filed with the Court of First Instance of Palawan a complaint for damages against defendants-appellees Fiscal Francisco Ponce de Leon and Orlando Maddela, alleging that on July 6, 1962 Orlando Maddela entered the premises of Delfin Lim without a search warrant and then and there took away the hull of the motor launch without his consent; that he effected the seizure upon order of Fiscal Ponce de Leon who knew fully well that his office was not vested with authority to order the seizure of a private property; that said motor launch was purchased by Delfin Lim from Jikil Taha in consideration of Three Thousand Pesos (P3,000.00), Two Thousand Pesos (P2,000.00) of which has been given to Jikil Taha as advance payment; that as a consequence of the unlawful seizure of the motor launch, its sale did not materialize; and that since July 6, 1962, the said motor launch had been moored at the Balabac Bay, Palawan and because of exposure to the elements it had become worthless and beyond repair. For the alleged violation of their constitutional rights, plaintiffs-appellants prayed that defendants-appellees be ordered to pay jointly and severally each of them the sum of P5,750.00 representing actual, moral and exemplary damages and attorney's fees. In their answer, defendants-appellees denied the material allegations of the complaint and as affirmative defenses alleged that the motor launch in question which was sold by Jikil Taha to Alberto Timbangcaya on April 29, 1961 was sometime in April 1962, forcibly taken with violence upon persons and with intent to gain by Jikil Taha from Alfredo Timbangcaya without the latter's knowledge and consent, thus giving rise to the filing of a criminal charge of robbery against Jikil Taha; that Fiscal Ponce de Leon, in his capacity as Acting Provincial Fiscal of Palawan ordered Orlando Maddela to seize and impound the motor launch "SAN RAFAEL", for being the corpus delicti of the robbery; and that Orlando Maddela merely obeyed the orders of his superior officer to impound said launch. By way of counterclaim, defendants-appellees alleged that because of the malicious and groundless filing of the complaint by plaintiffs-appellants, they were constrained to engage the services of lawyers, each of them paying P500.00 as attorney's fees; and that they suffered moral damages in the amount of P5,000.00 each and actual damages in the amount of P500.00 each. They also prayed that each of them awarded exemplary damages in the amount of P1,000.00. On September 13, 1965, the trial court rendered its decision, upholding the validity of the seizure of the motor launch on the ground that "the authority to impound evidences or exhibits or corpus delicti in a case pending investigation is inherent in the Provincial Fiscal who controls the prosecution and who introduces those exhibits in the court." Accordingly, the trial court dismissed the complaint of plaintiffs-appellants and ordered them to pay jointly and severally each of the defendants-appellees the amount of

P500.00 by way of actual damages another amount of P500.00 for attorney's fees and P1,000.00 as exemplary damages. Hence, this appeal. Two vital issues call for resolution by this Court. First, whether or not defendantappellee Fiscal Ponce de Leon had the power to order the seizure of the motor launch in question without a warrant of search and seizure even if the same was admittedly the corpus delicti of the crime. Second, whether or not defendants-appellees are civilly liable to plaintiffs-appellants for damages allegedly suffered by them granting that the seizure of the motor launch was unlawful. The gravamen of plaintiffs-appellants' argument is that the taking of the motor launch on July 6, 1962 by Orlando Maddela upon the order of Fiscal Ponce de Loon was in violation of the constitutional guarantee against unreasonable searches and seizures since it was done without a warrant. The pertinent provision of the Constitution then in force reads:
3) The right of the people to be secure in their persons, houses, papers and effects against unreasonable searches and seizures shall not be violated, and no warrants shall issue but upon probable cause, to be determined by the judge after examination under oath or affirmation of the complainant and the witnesses he may produce, and particularly 3 describing the place to be searched, and the persons or things to be seized.

A cursory reading of the above provision easily brings into focus the unreasonableness of the seizure of the aforementioned motor launch. A search and seizure to be reasonable, must be effected by means of a valid search warrant. And for a search warrant to be valid: (1) it must be issued upon probable cause; (2) the probable cause must be determined by the judge himself and not by the applicant or any other person; (3) in the determination of probable cause, the judge must examine, under oath or affirmation, the complainant and such witnesses as the latter may produce; and (4) the warrant issued must particularly describe the place to be searched and persons or things to be seized. 4 Thus in a long line of decisions, this Court has declared invalid search warrants which were issued in utter disregard of the constitutional injunction. 5 Defendants-appellees admitted that when Orlando Maddela entered the premises of Delfin Lim and impounded the motor launch he was not armed with a search warrant; that he effected the seizure of the motor launch in the absence of and without the consent of Delfin Lim. There can be no question that without the proper search warrant, no public official has the right to enter the premises of another without his consent for the purpose of search and seizure. 6 And since in the present case defendantsappellees seized the motor launch without a warrant, they have violated the constitutional right of plaintiffs-appellants against unreasonable search and seizure. Defendants-appellees however would want to justify the seizure of the motor launch even without a warrant because of Fiscal Ponce de Leon's alleged inherent power to

order the seizure of a personal property which is the corpus delicti of a crime, he being a quasi judicial officer who has the control of the prosecution and the presentation of the evidence in the criminal case. They argue that inasmuch as the motor launch in question was allegedly stolen by Jikil Taha from Timbangcaya, Fiscal Ponce de Leon could order its seizure even without a search warrant. We cannot agree. Under the old Constitution 7 the power to issue a search warrant is vested in a judge or magistrate and in no other officer and no search and seizure can be made without a proper warrant. At the time the act complained of was committed, there was no law or rule that recognized the authority of Provincial Fiscals to issue a search warrant. In his vain attempt to justify the seizure of the motor launch in question without a warrant Fiscal Ponce de Leon invoked the provisions of Republic Act No. 732, which amended Sections 1674 and 1687 of the Revised Administrative Code. But there is nothing in said law which confers upon the provincial fiscal; the authority to issue warrants, much less to order without warrant the seizure of a personal property even if it is the corpus delicti of a crime. True, Republic Act No. 732 has broadened the power of provincial fiscals to conduct preliminary investigations, but said law did not divest the judge or magistrate of its power to determine, before issuing the corresponding warrant, whether or not probable cause exists therefor. 8 Moreover, under Sections 2 and 3 of Rule 122 of the Rules of Court 9 which complement the constitutional provision earlier cited, two principles are made clear, namely: (1) that in the seizure of a stolen property search warrant is still necessary; and (2) that in issuing a search warrant the judge alone determines whether or not there is a probable cause. The fact that a thing is a corpus delicti of a crime does not justify its seizure without a warrant. As held in U.S. v. de los Reyes and Esguerra, 10 citing McClurg v. Brenton: 11
The mere fact that a man is an officer, whether of high or low degree, gives him no more right than is possessed by the ordinary private citizen to break in upon the privacy of a home and subject its occupant to the indignity of a search for the evidence of crime, without a legal warrant procured for that purpose. No amount of incriminating evidence whatever its source, will supply the place of such warrant. At the closed door of the home be it palace or hovel even bloodhounds must wait till the law, by authoritative process, bids it open. (Emphasis supplied.)

Defendant-appellee Fiscal Ponce de Leon would also invoke lack of time to procure a search warrant as an excuse for the seizure of the motor launch without one. He claimed that the motor launch had to be seized immediately in order to preserve it and to prevent its removal out of the locality, since Balabac, Palawan, where the motor launch was at the time, could only be reached after three to four days' travel by boat. 12 The claim cannot be sustained. The records show that on June 15, 1962 13 Fiscal Ponce de Leon made the first request to the Provincial Commander for the impounding of the motor launch; and on June 26, 1962 14 another request was made. The seizure was not effected until July 6, 1962. In short, Fiscal Ponce de Leon had all the time to procure a search warrant had he wanted to and which he could have taken in less than a day, but he did not. Besides, there is no basis for the apprehension that the motor launch might be moved out of Balabac because even prior to its seizure the motor launch was

already without its engine. 15 In sum, the fact that there was no time to secure a search warrant would not legally justify a search without one. 16 As to whether or not they are entitled to damages, plaintiffs-appellants anchor their claim for damages on Articles 32 and 2219 of the New Civil Code which provide in part as follows:
ART. 32. Any public officer or employee, or any private individual, who directly or indirectly obstructs, defeats, violates or in any manner impedes or impairs any of the following rights and liberties of another person shall be liable to the latter for damages. xxx xxx xxx (9) The rights to be secure in one's person, house, papers, and effects against unreasonable searches and seizures. xxx xxx xxx The indemnity shall include moral damages. Exemplary damages may also be adjudicated. ART. 2219. Moral damages may be recovered in the following and analogous cases: xxx xxx xxx (6) Illegal search; xxx xxx xxx (1) Acts and action referred to in Articles 21, 26, 27, 28, 29, 30, 32, 34 and 35.

Pursuant to the foregoing provisions, a person whose constitutional rights have been violated or impaired is entitled to actual and moral damages from the public officer or employee responsible therefor. In addition, exemplary damages may also be awarded. In the instant case, plaintiff-appellant Delfin Lim claimed that he purchased the motor launch from Jikil Taha in consideration of P3,000.00, having given P2,000.00 as advanced payment; that since or seizure on July 6, 1962 the motor launch had been moored at Balabac Bay and because of exposure to the elements it has become worthless at the time of the filing of the present action; that because of the illegality of the seizure of the motor launch, he suffered moral damages in the sum of P1,000.00; and that because of the violation of their constitutional rights they were constrained to engage the services of a lawyer whom they have paid P1,500.00 for attorney's fees. We find these claims of Delfin Lim amply supported by the evidence and therefore should be awarded the sum of P3,000.00 as actual damages; P1,000.00 as moral damages and P750.00 for attorney's fees. However, with respect co plaintiff Jikil Taha, he is not entitled to recover any damage which he alleged he had suffered from the unlawful seizure of the motor launch inasmuch as he had already transferred the ownership and possession of the motor launch to Delfin Lim at the time it was seized and therefore, he has no legal standing to question the validity of the seizure. Well settled is the rule that

the legality of a seizure can be contested only by the party whose rights have been impaired thereby, and that the objection to an unlawful search and seizure is purely personal and cannot be availed of by third parties. 17 Consequently, one who is not the owner, lessee, or lawful occupant of the premise searched cannot raise the question of validity of the search and seizure. 18 Jikil Taha is not without recourse though. He can still collect from his co-plaintiff, Delfin Lim the unpaid balance of P1,000.00. Defendant-appellee Fiscal Ponce de Leon wanted to wash his hands of the incident by claiming that "he was in good faith, without malice and without the slightest intention of inflicting injury to plaintiff-appellant, Jikil Taha" 19 when he ordered the seizure of the motor launch. We are not prepared to sustain his defense of good faith. To be liable under Article 32 of the New Civil Code it is enough that there was a violation of the constitutional rights of the plaintiffs and it is not required that defendants should have acted with malice or bad faith. Dr. Jorge Bocobo, Chairman of the Code Commission, gave the following reasons during the public hearings of the Joint Senate and House Committees, why good faith on the part of the public officer or employee is immaterial. Thus:
DEAN BOCOBO. Article 32, regarding individual rights; Attorney Cirilo Paredes proposes that Article 32 be so amended as to make a public official liable for violation of another person's constitutional rights only if the public official acted maliciously or in bad faith. The Code Commission opposes this suggestion for these reasons: The very nature of Article 32 is that the wrong may be civil or criminal. It is not necessary therefore that there should be malice or bad faith. To make such a requisite would defeat the main purpose of Article 32 which is the effective protection of individual rights. Public officials in the past have abused their powers on the pretext of justifiable motives or good faith in the performance of their duties. Precisely, the object of the Article is to put an end to official abuse by the plea of good faith. In the United States this remedy is in he nature of a tort. Mr. Chairman, this article is firmly one of the fundamental articles introduced in the New Civil Code to implement democracy. There is no real democracy if a public official is abusing, and we made the article so strong and so comprehensive that it concludes an abuse of individual rights even if done in good faith, that official is liable. As a matter of fact, we know that there are very few public officials who openly and definitely abuse the individual rights of the citizens. In most cases, the abuse is justified on a plea of desire to enforce the law to comply with one's duty. And so, if we should limit the scope of this article, that would practically nullify the object of the article. Precisely, the opening object of the article is to put an end to abuses which are justified by a plea of good faith, which 20 is in most cases the plea of officials abusing individual rights.

But defendant-appellee Orlando Maddela cannot be held accountable because he impounded the motor launch upon the order of his superior officer. While a subordinate officer may be held liable for executing unlawful orders of his superior officer, there are certain circumstances which would warrant Maddela's exculpation from liability. The records show that after Fiscal Ponce de Leon made his first request to the Provincial Commander on June 15, 1962 Maddela was reluctant to impound the motor launch despite repeated orders from his superior officer. 21 It was only after he was furnished a copy of the reply of Fiscal Ponce de Leon, dated June 26, 1962, to the letter of the

Provincial Commander, justifying the necessity of the seizure of the motor launch on the ground that the subsequent sale of the launch to Delfin Lim could not prevent the court from taking custody of the same, 22 that he impounded the motor launch on July 6, 1962. With said letter coming from the legal officer of the province, Maddela was led to believe that there was a legal basis and authority to impound the launch. Then came the order of his superior officer to explain for the delay in the seizure of the motor launch. 23 Faced with a possible disciplinary action from his Commander, Maddela was left with no alternative but to seize the vessel. In the light of the above circumstances. We are not disposed to hold Maddela answerable for damages. IN VIEW OF THE FOREGOING, the decision appealed from is hereby reversed and another one entered declaring the seizure illegal and ordering defendant-appellee Fiscal Francisco Ponce de Leon to pay to plaintiff-appellant Delfin Lim the sum of P3,000.00 as actual damages, plus P1,000.00 moral damages, and, in addition, P750.00 for attorney's fees. With costs against defendant-appellee Fiscal Ponce de Leon. SO ORDERED.
SALAZAR VS. ACHACOSO [183 SCRA 145; G.R. NO. 81510; 14 MAR 1990] Wednesday, February 04, 2009 Posted by Coffeeholic Writes Labels: Case Digests, Political Law

Facts:

Rosalie

Tesoro

of

Pasay

City

in

sworn

statement filed with the POEA, charged petitioner with illegal recruitment. Public respondent Atty. Ferdinand Marquez sent petitioner a telegram directing him to appear to the POEA regarding the complaint against him. On the same day, after knowing that petitioner had no license to operate a recruitment agency, public respondent Administrator Tomas Achacoso issued a Closure and Seizure Order No. 1205 to petitioner. It stated that there will a seizure of the documents and paraphernalia being used or intended to be used as the means of committing illegal recruitment, it having verified that petitioner has (1) No valid license or authority from the Department of Labor and Employment to recruit and deploy workers for

overseas employment; (2) Committed/are committing acts prohibited under Article 34 of the New Labor Code in relation to Article 38 of the same code. A team was then tasked to implement the said Order. The group, accompanied by mediamen and Mandaluyong policemen, went to petitioners residence. They served the order to a certain Mrs. For a Salazar, who let them in. The team confiscated assorted costumes. Petitioner filed with POEA a letter requesting for the return of the seized properties, because she was not given prior notice and hearing. The said Order violated due process. She also alleged that it violated sec 2 of the Bill of Rights, and the properties were confiscated against her will and were done with unreasonable force and intimidation.

Issue:

Whether

or

Not

the

Philippine

Overseas

Employment Administration (or the Secretary of Labor) can validly issue warrants of search and seizure (or arrest) under Article 38 of the Labor Code

Held:

Under the new Constitution, . . . no search

warrant or warrant of arrest shall issue except upon probable cause to be determined personally by the judge after examination under oath or affirmation of the complainant and the witnesses he may produce, and particularly describing the place to be searched and the persons or things to be seized. Mayors and prosecuting

officers cannot issue warrants of seizure or arrest. The Closure and Seizure Order was based on Article 38 of the Labor Code. The Supreme Court held, We reiterate that the Secretary of Labor, not being a judge, may no longer issue search or arrest warrants. Hence, the authorities must go through the judicial process. To that extent, we declare Article 38, paragraph (c), of the Labor Code, unconstitutional and of no force and effect The power of the President to order the arrest of aliens for deportation is, obviously, exceptional. It (the power to order arrests) cannot be made to extend to other cases, like the one at bar. Under the Constitution, it is the sole domain of the courts. Furthermore, the search and seizure order was in the nature of a general warrant. The court held that the warrant is null and void, because it must identify specifically the things to be seized. WHEREFORE, the petition is GRANTED. Article 38, paragraph (c) of the Labor Code is declared UNCONSTITUTIONAL and null and void. The respondents are ORDERED to return all materials seized as a result of the implementation of Search and Seizure Order No. 1205.
Salazar v. Achacoso, 183 SCRA 145 F: Pursuant to the powers vested by PD 1920 and EO 1022, POEA Administrator Achacoso ordered the closure of the recruitment agency of Horty Salazar, having verified that she hadno license to operate a recruitment agency. He further ordered the seizure of the documents and paraphernalias,being used or intended to be used as the means of commiting illegal recruitment. This order was enforced on 26January 1988. Petitioner filed this suit for prohibition. Issue: May the POEA (or the Sec. of Labor) validly issuewarrants of serach and seizure (or arrest ) under Art. 38 of the Labor Code?HELD: NO.The provisions of PD 1920and EO 1022, now embodied in Art. 38 of the Labor Code, are the dying vestiges of authoritarian rule in its twilightsmoments. Under Art. III, Sec 2 of the 1987 Constitution, it is only judges and no other, who may issue warrants of arrest and search. The exception is in cases of deportation of illegal and undesirable aliens, whom the President of the Commissioner of Immigration may order arrested, following a final order of deportation, for the purpose of deportation. The Sec. of Labor , not being a judge. may no longer issue search or arrest warrants. Hence, theauthorities must go through the judicial process. To that extent, we declare Art. 38, par. C of the Labor Code,unconstitutional and of no force and effect.&n bsp; a. Existence of probable cause. Probable cause is such facts andcircumstances as would reasonably make a prudent man believe that a crime have been committed and that thedocuments or things sought to be searched and seized are in the possession of the person against whom the warrantis sought. Without probable cause, there can be no valid search warrant. See Pasionvda. de Garcia v. Locsin, 65Phil. 689, (1938)

G.R. No. 83578 March 16, 1989 THE PRESIDENTIAL ANTI-DOLLAR SALTING TASK FORCE, petitioner, vs. HONORABLE COURT OF APPEALS, HONORABLE TEOFILO L, GUADIZ, JR.,Presiding Judge, REGIONAL TRIAL COURT, Branch 147: NCR (MAKATI), and KARAMFIL IMPORT-EXPORT CO., INC., respondents. K. V. Faylona & Associates for respondents.

SARMIENTO, J.: The petitioner, the Presidential Anti-Dollar Salting Task Force, the President's arm assigned to investigate and prosecute so-called "dollar salting" activities in the country (per Presidential Decree No. 1936 as amended by Presidential Decree No. 2002), asks the Court to hold as null and void two Resolutions of the Court of Appeals, dated September 24, 1987 1 and May 20, 1988, 2 reversing its Decision, dated October 24, 1986. 3 The Decision set aside an Order, dated April 16, 1985, of the Regional Trial Court, 4 as well as its Order, dated August 21, 1985. The Resolution, dated September 24, 1987 disposed of, and granted, the private respondent Karamfil Import-Export Co., Inc.'s motion for reconsideration of the October 24, 1986 Decision; the Resolution dated May 20, 1988, in turn, denied the petitioner's own motion for reconsideration. The facts are not in controversy. We quote:
On March 12, 1985, State Prosecutor Jose B. Rosales, who is assigned with the Presidential Anti-Dollar Salting Task Force hereinafter referred to as PADS Task Force for purposes of convenience, issued search warrants Nos. 156, 157, 158, 159, 160 and 161 against the petitioners Karamfil Import-Export Co., Inc., P & B Enterprises Co., Inc., Philippine Veterans Corporation, Philippine Veterans Development Corporation, Philippine Construction Development Corporation, Philippine Lauan Industries Corporation, Inter-trade Development (Alvin Aquino), Amelili U. Malaquiok Enterprises and Jaime P. Lucman Enterprises.

The application for the issuance of said search warrants was filed by Atty. Napoleon Gatmaytan of the Bureau of Customs who is a deputized member of the PADS Task Force. Attached to the said application is the affidavit of Josefin M. Castro who is an operative and investigator of the PADS Task Force. Said Josefin M. Castro is likewise the sole deponent in the purported deposition to support the application for the issuance of the six (6) search warrants involved in this case. The application filed by Atty. Gatmaytan, the affidavit and deposition of Josefin M. Castro are all dated March 12, 1985. 5 Shortly thereafter, the private respondent (the petitioner below) went to the Regional Trial Court on a petition to enjoin the implementation of the search warrants in question.

6 On

March 13, 1985, the trial court issued a temporary restraining order [effective "for a period of five (5) days notice " 7 ] and set the case for hearing on March 18, 1985. In disposing of the petition, the said court found the material issues to be:
1) Competency of this Court to act on petition filed by the petitioners; 2) Validity of the search warrants issued by respondent State Prosecutor; 3) Whether or not the petition has become moot and academic because all the search warrants sought to be quashed had already been implemented and executed. 8

On April 16, 1985, the lower court issued the first of its challenged Orders, and held:
WHEREFORE, in view of all the foregoing, the Court hereby declares Search Warrant Nos. 156, 157, 158, 159, 160, and 161 to be null and void. Accordingly, the respondents are hereby ordered to return and surrender immediately all the personal properties and documents seized by them from the petitioners by virtue of the aforementioned search warrants.

SO ORDERED. 9 On August 21, 1985, the trial court denied reconsideration. On April 4, 1986, the Presidential Anti-Dollar Salting Task Force went to the respondent Court of Appeals to contest, on certiorari, the twin Order(s) of the lower court. In ruling initially for the Task Force, the Appellate Court held:
Herein petitioner is a special quasi-judicial body with express powers enumerated under PD 1936 to prosecute foreign exchange violations defined and punished under P.D. No. 1883. The petitioner, in exercising its quasi-judicial powers, ranks with the Regional Trial Courts, and the latter in the case at bar had no jurisdiction to declare the search warrants in question null and void. Besides as correctly pointed out by the Assistant Solicitor General the decision of the Presidential Anti-Dollar Salting Task Force is appealable to the Office of the President. 10

On November 12, 1986, Karamfil Import-Export Co., Inc. sought a reconsideration, on the question primarily of whether or not the Presidential Anti-Dollar Salting Task Force is "such other responsible officer' countenanced by the 1973 Constitution to issue warrants of search and seizure. As we have indicated, the Court of Appeals, on Karamfil's motion, reversed itself and issued its Resolution, dated September 1987, and subsequently, its Resolution, dated May 20, 1988, denying the petitioner's motion for reconsideration.

In its petition to this Court, the petitioner alleges that in so issuing the Resolution(s) above-mentioned, the respondent Court of Appeals "committed grave abuse of discretion and/or acted in excess of its appellate jurisdiction," 11 specifically:
a) In deviating from the settled policy and rulings of the Supreme Court that no Regional Trial Courts may countermand or restrain the enforcement of lawful writs or decrees issued by a quasi-judicial body of equal and coordinate rank, like the PADS Task Force; b) For resorting to judicial legislation to arrive at its erroneous basis for reconsidering its previous Decision dated October 24, 1986 (see Annex "I") and thus promulgated the questioned Resolutions (Annexes "A" and "B"), which violated the constitutional doctrine on separation of powers; c) In not resolving directly the other important issues raised by the petitioner in its Petition in CA-G.R. No. 08622-SP despite the fact that petitioner has demonstrated sufficiently and convincingly that respondent RTC, in issuing the questioned Orders in Special Proceeding No. M-624 (see Annexes "C" and 'D"), committed grave abuse of discretion and/or acted in excess of jurisdiction: 1. In ruling that (a) the description of the things to be seized as stated in the contested search warrant were too general which allegedly render the search warrants null and void; (b) the applications for the contested search warrants actually charged two offenses in contravention of the 2nd paragraph, Section 3, Rule 126 of the Rules of Court; and (c) this case has not become moot and academic, even if the contested search warrants had already been fully implemented with positive results; and 2. In ruling that the petitioner PADS Task Force has not been granted under PD 1936 'judicial or quasi-judicial jurisdiction. 12

We find, upon the foregoing facts, that the essential questions that confront us are- (i) is the Presidential Anti-Dollar Salting Task Force a quasi-judicial body, and one co-equal in rank and standing with the Regional Trial Court, and accordingly, beyond the latter's jurisdiction; and (ii) may the said presidential body be said to be "such other responsible officer as may be authorized by law" to issue search warrants under the 1973 Constitution questions we take up seriatim.** In submitting that it is a quasi-judicial entity, the petitioner states that it is endowed with "express powers and functions under PD No. 1936, to prosecute foreign exchange violations as defined and punished under PD No. 1883." 13 "By the very nature of its express powers as conferred by the laws," so it is contended, "which are decidedly quasi-judicial or discretionary function, such as to conduct preliminary investigation on the charges of foreign exchange violations, issue search warrants or warrants of arrest, hold departure orders, among others, and depending upon the evidence presented, to dismiss the charges or to file the corresponding information in court of Executive Order No. 934, PD No. 1936 and its Implementing Rules and Regulations effective August 26, 1984), petitioner exercises quasi-judicial power or the power of adjudication ." 14 The Court of Appeals, in its Resolution now assailed, 15 was of the opinion that "[t]he grant of quasi-judicial powers to petitioner did not diminish the regular courts' judicial power of interpretation. The right to interpret a law and, if necessary to declare one

unconstitutional, exclusively pertains to the judiciary. In assuming this function, courts do not proceed on the theory that the judiciary is superior to the two other coordinate branches of the government, but solely on the theory that they are required to declare the law in every case which come before them." 16 This Court finds the Appellate Court to be in error, since what the petitioner puts to question is the Regional Trial Court's act of assuming jurisdiction over the private respondent's petition below and its subsequent countermand of the Presidential AntiDollar Salting Task Force's orders of search and seizure, for the reason that the presidential body, as an entity (allegedly) coordinate and co-equal with the Regional Trial Court, was (is) not vested with such a jurisdiction. An examination of the Presidential Anti-Dollar Salting Task Force's petition shows indeed its recognition of judicial review (of the acts of Government) as a basic privilege of the courts. Its objection, precisely, is whether it is the Regional Trial Court, or the superior courts, that may undertake such a review. Under the Judiciary Reorganization Act of 1980,
17

the Court of Appeals exercises:

(3) Exclusive appellate jurisdiction over all final judgments, decisions, resolutions, orders or awards of Regional Trial Court and quasi-judicial agencies, instrumentalities, boards or commissions, except those falling within the appellate jurisdiction of the Supreme Court in accordance with the Constitution, the provisions of this Act, and of subparagraph (1) of the third paragraph and subparagraph (4) of the fourth paragraph of Section 17 of the Judiciary Act of 1948. 18

xxx xxx xxx Under the present Constitution, with respect to its provisions on Constitutional Commissions, it is provided, in part that:
... Unless otherwise provided by this Constitution or by law, any decision, order, or ruling of each Commission may be brought to the Supreme Court on certiorari by the aggrieved party within thirty days from receipt of a copy thereof. 19

On the other hand, Regional Trial Courts have exclusive original jurisdiction:
(6) In all cases not within the exclusive jurisdiction of any court, tribunal, person or body exercising judicial or quasi-judicial functions. 20

xxx xxx xxx Likewise:


... The Supreme Court may designate certain branches of the Regional Trial Court to handle exclusively criminal cases, juvenile and domestic relations cases, agrarian case, urban land reform cases which do not fall under the jurisdiction of quasi- judicial bodies and agencies and/or such other special cases as the Supreme Court may determine in the interest of a speedy and efficient administration of justice. 21

xxx xxx xxx Under our Resolution dated January 11, 1983:
22

... The appeals to the Intermediate Appellate Court [now, Court of Appeals] from quasijudicial bodies shall continue to be governed by the provisions of Republic Act No. 5434 insofar as the same is not inconsistent with the provisions of B.P. Blg. 129. 23

The pertinent provisions of Republic Act No. 5434 are as follows:


SECTION 1. Appeals from specified agencies. Any provision of existing law or Rule of Court to the contrary notwithstanding, parties aggrieved by a final ruling, award, order, decision, or judgment of the Court of Agrarian Relations; the Secretary of Labor under Section 7 of Republic Act Numbered Six hundred and two, also known as the "Minimum Wage Law"; the Department of Labor under Section 23 of Republic Act Numbered Eight hundred seventy-five, also known as the "Industrial Peace Act"; the Land Registration Commission; the Securities and Exchange Commission; the Social Security Commission; the Civil Aeronautics Board; the Patent Office and the Agricultural Inventions Board, may appeal therefrom to the Court of Appeals, within the period and in the manner herein provided, whether the appeal involves questions of fact, mixed questions of fact and law, or questions of law, or all three kinds of questions. From final judgments or decisions of the Court of Appeals, the aggrieved party may appeal by certiorari to the Supreme Court as provided in Rule 45 of the Rules of Court. 24

Because of subsequent amendments, including the abolition of various special courts, jurisdiction over quasi-judicial bodies has to be, consequently, determined by the corresponding amendatory statutes. Under the Labor Code, decisions and awards of the National Labor Relations Commission are final and executory, but, nevertheless, 'reviewable by this Court through a petition for certiorari and not by way of appeal." 26 Under the Property Registration Decree, decisions of the Commission of Land Registration, en consults, are appealable to the Court of Appeals. 27 The decisions of the Securities and Exchange Commission are likewise appealable to the Appellate Court, 28 and so are decisions of the Social Security Commission.29 As a rule, where legislation provides for an appeal from decisions of certain administrative bodies to the Court of Appeals, it means that such bodies are co-equal with the Regional Trial Courts, in terms of rank and stature, and logically, beyond the control of the latter.

25

As we have observed, the question is whether or not the Presidential Anti-Dollar Salting Task Force is, in the first place, a quasi-judicial body, and one whose decisions may not be challenged before the regular courts, other than the higher tribunals the Court of Appeals and this Court. A quasi-judicial body has been defined as "an organ of government other than a court and other than a legislature, which affects the rights of private parties through either

adjudication or rule making." 30 The most common types of such bodies have been listed as follows:
(1) Agencies created to function in situations wherein the government is offering some gratuity, grant, or special privilege, like the defunct Philippine Veterans Board, Board on Pensions for Veterans, and NARRA, and Philippine Veterans Administration. (2) Agencies set up to function in situations wherein the government is seeking to carry on certain government functions, like the Bureau of Immigration, the Bureau of Internal Revenue, the Board of Special Inquiry and Board of Commissioners, the Civil Service Commission, the Central Bank of the Philippines. (3) Agencies set up to function in situations wherein the government is performing some business service for the public, like the Bureau of Posts, the Postal Savings Bank, Metropolitan Waterworks & Sewerage Authority, Philippine National Railways, the Civil Aeronautics Administration. (4) Agencies set up to function in situations wherein the government is seeking to regulate business affected with public interest, like the Fiber Inspections Board, the Philippine Patent Office, Office of the Insurance Commissioner. (5) Agencies set up to function in situations wherein the government is seeking under the police power to regulate private business and individuals, like the Securities & Exchange Commission, Board of Food Inspectors, the Board of Review for Moving Pictures, and the Professional Regulation Commission. (6) Agencies set up to function in situations wherein the government is seeking to adjust individual controversies because of some strong social policy involved, such as the National Labor Relations Commission, the Court of Agrarian Relations, the Regional Offices of the Ministry of Labor, the Social Security Commission, Bureau of Labor Standards, Women and Minors Bureau. 31

As may be seen, it is the basic function of these bodies to adjudicate claims and/or to determine rights, and unless its decision are seasonably appealed to the proper reviewing authorities, the same attain finality and become executory. A perusal of the Presidential Anti-Dollar Salting Task Force's organic act, Presidential Decree No. 1936, as amended by Presidential Decree No. 2002, convinces the Court that the Task Force was not meant to exercise quasi-judicial functions, that is, to try and decide claims and execute its judgments. As the President's arm called upon to combat the vice of "dollar salting" or the blackmarketing and salting of foreign exchange, 32 it is tasked alone by the Decree to handle the prosecution of such activities, but nothing more. We quote:
SECTION 1. Powers of the Presidential Anti-Dollar Salting Task Force.-The Presidential Anti-Dollar Salting Task Force, hereinafter referred to as Task Force, shall have the following powers and authority: a) Motu proprio or upon complaint, to investigate and prosecute all dollar salting activities, including the overvaluation of imports and the undervaluation of exports; b) To administer oaths, summon persons or issue subpoenas requiring the attendance and testimony of witnesses or the production of such books, papers, contracts, records,

statements of accounts, agreements, and other as may be necessary in the conduct of investigation; c) To appoint or designate experts, consultants, state prosecutors or fiscals, investigators and hearing officers to assist the Task Force in the discharge of its duties and responsibilities; gather data, information or documents; conduct hearings, receive evidence, both oral and documentary, in all cases involving violation of foreign exchange laws or regulations; and submit reports containing findings and recommendations for consideration of appropriate authorities; d) To punish direct and indirect contempts with the appropriate penalties therefor under Rule 71 of the Rules of Court; and to adopt such measures and take such actions as may be necessary to implement this Decree.

xxx xxx xxx


f. After due investigation but prior to the filing of the appropriate criminal charges with the fiscal's office or the courts as the case may be, to impose a fine and/or administrative sanctions as the circumstances warrant, upon any person found committing or to have committed acts constituting blackmarketing or salting abroad of foreign exchange, provided said person voluntarily admits the facts and circumstances constituting the offense and presents proof that the foreign exchange retained abroad has already been brought into the country. Thereafter, no further civil or criminal action may be instituted against said person before any other judicial regulatory or administrative body for violation of Presidential Decree No. 1883. The amount of the fine shall be determined by the Chairman of the Presidential AntiDollar Salting Task Force and paid in Pesos taking into consideration the amount of foreign exchange retained abroad, the exchange rate differentials, uncollected taxes and duties thereon, undeclared profits, interest rates and such other relevant factors. The fine shall be paid to the Task Force which shall retain Twenty percent (20 %) thereof. The informer, if any, shall be entitled to Twenty percent (20 %) of the fine. Should there be no informer, the Task Force shall be entitle to retain Forty percent (40 %) of the fine and the balance shall accrue to the general funds of the National government. The amount of the fine to be retained by the Task Force shall form part of its Confidential Fund and be utilized for the operations of the Task Force . 33

The Court sees nothing in the aforequoted provisions (except with respect to the Task Force's powers to issue search warrants) that will reveal a legislative intendment to confer it with quasi-judicial responsibilities relative to offenses punished by Presidential Decree No. 1883. Its undertaking, as we said, is simply, to determine whether or not probable cause exists to warrant the filing of charges with the proper court, meaning to say, to conduct an inquiry preliminary to a judicial recourse, and to recommend action "of appropriate authorities". It is not unlike a fiscal's office that conducts a preliminary investigation to determine whether or not prima facie evidence exists to justify haling the respondent to court, and yet, while it makes that determination, it cannot be said to be acting as a quasi-court. For it is the courts, ultimately, that pass judgment on the accused, not the fiscal.

It is not unlike the Presidential Commission on Good Government either, the executive body appointed to investigate and prosecute cases involving "ill-gotten wealth". It had been vested with enormous powers, like the issuance of writs of sequestration, freeze orders, and similar processes, but that did not, on account thereof alone, make it a quasi-judicial entity as defined by recognized authorities. It cannot pronounce judgement of the accused's culpability, the jurisdiction to do which is exclusive upon the Sandiganbayan. 34 If the Presidential Anti-Dollar Salting Task Force is not, hence, a quasi-judicial body, it cannot be said to be co-equal or coordinate with the Regional Trial Court. There is nothing in its enabling statutes that would demonstrate its standing at par with the said court. In that respect, we do not find error in the respondent Court of Appeal's resolution sustaining the assumption of jurisdiction by the court a quo. It will not do to say that the fact that the Presidential Task Force has been empowered to issue warrants of arrest, search, and seizure, makes it, ergo, a "semi-court". Precisely, it is the objection interposed by the private respondent, whether or not it can under the 1973 Charter, issue such kinds of processes. It must be observed that under the present Constitution, the powers of arrest and search are exclusive upon judges. 35 To that extent, the case has become moot and academic. Nevertheless, since the question has been specifically put to the Court, we find it unavoidable to resolve it as the final arbiter of legal controversies, pursuant to the provisions of the 1973 Constitution during whose regime the case was commenced. Since the 1973 Constitution took force and effect and until it was so unceremoniously discarded in 1986, its provisions conferring the power to issue arrest and search warrants upon an officer, other than a judge, by fiat of legislation have been at best controversial. In Lim v. Ponce de Leon, 36 a 1975 decision, this Court ruled that a fiscal has no authority to issue search warrants, but held in the same vein that, by virtue of the responsible officer" clause of the 1973 Bill of Rights, "any lawful officer authorized by law can issue a search warrant or warrant of arrest.37 Authorities, however, have continued to express reservations whether or not fiscals may, by statute, be given such a power. 38 Less than a year later, we promulgated Collector of Customs v. Villaluz, 39 in which we categorically averred: Until now only the judge can issue the warrant of arrest." 40 "No law or presidential decree has been enacted or promulgated vesting the same authority in a particular responsible officer ." 41 Apparently, Villaluz had settled the debate, but the same question persisted following this Courts subsequent rulings upholding the President's alleged emergency arrest powers .42 [Mr. Justice Hugo Gutierrez would hold, however, that a Presidential Commitment Order (PCO) is (was) not a species of "arrest" in its technical sense, and

that the (deposed) Chief Executive, in issuing one, does not do so in his capacity as a "responsible officer" under the 1973 Charter, but rather, as Commander-in-Chief of the Armed Forces in times of emergency, or in order to carry out the deportation of undesirable aliens.43 In the distinguished Justice's opinion then, these are acts that can be done without need of judicial intervention because they are not, precisely, judicial but Presidential actions.] In Ponsica v. Ignalaga,44 however, we held that the mayor has been made a "responsible officer' by the Local Government Code, 45 but had ceased to be one with the approval of the 1987 Constitution according judges sole authority to issue arrest and search warrants. But in the same breath, we did not rule the grant under the Code unconstitutional based on the provisions of the former Constitution. We were agreed, though, that the "responsible officer" referred to by the fundamental law should be one capable of approximating "the cold neutrality of an impartial judge." 46 In striking down Presidential Decree No. 1936 the respondent Court relied on American jurisprudence, notably, Katz v. United States, 47 Johnson v. United States, 48 and Coolidge v. New Hampshire 49 in which the American Supreme Court ruled that prosecutors (like the petitioner) cannot be given such powers because of their incapacity for a "detached scrutiny" 50 of the cases before them. We affirm the Appellate Court. We agree that the Presidential Anti-Dollar Salting Task Force exercises, or was meant to exercise, prosecutorial powers, and on that ground, it cannot be said to be a neutral and detached "judge" to determine the existence of probable cause for purposes of arrest or search. Unlike a magistrate, a prosecutor is naturally interested in the success of his case. Although his office "is to see that justice is done and not necessarily to secure the conviction of the person accused," 51 he stands, invariably, as the accused's adversary and his accuser. To permit him to issue search warrants and indeed, warrants of arrest, is to make him both judge and jury in his own right, when he is neither. That makes, to our mind and to that extent, Presidential Decree No. 1936 as amended by Presidential Decree No. 2002, unconstitutional. It is our ruling, thus, that when the 1973 Constitution spoke of "responsible officer" to whom the authority to issue arrest and search warrants may be delegated by legislation, it did not furnish the legislator with the license to give that authority to whomsoever it pleased. It is to be noted that the Charter itself makes the qualification that the officer himself must be "responsible". We are not saying, of course, that the Presidential AntiDollar Salting Task Force (or any similar prosecutor) is or has been irresponsible in discharging its duty. Rather, we take "responsibility", as used by the Constitution, to mean not only skill and competence but more significantly, neutrality and independence comparable to the impartiality presumed of a judicial officer. A prosecutor can in no manner be said to be possessed of the latter qualities. According to the Court of Appeals, the implied exclusion of prosecutors under the 1973 Constitution was founded on the requirements of due process, notably, the assurance to

the respondent of an unbiased inquiry of the charges against him prior to the arrest of his person or seizure of his property. We add that the exclusion is also demanded by the principle of separation of powers on which our republican structure rests. Prosecutors exercise essentially an executive function (the petitioner itself is chaired by the Minister, now Secretary, of Trade and Industry), since under the Constitution, the President has pledged to execute the laws. 52 As such, they cannot be made to issue judicial processes without unlawfully impinging the prerogative of the courts. At any rate, Ponsica v. Ignalaga should foreclose all questions on the matter, although the Court hopes that this disposition has clarified a controversy that had generated often bitter debates and bickerings. The Court joins the Government in its campaign against the scourge of "dollar- salting", a pernicious practice that has substantially drained the nation's coffers and has seriously threatened its economy. We recognize the menace it has posed (and continues to pose) unto the very stability of the country, the urgency for tough measures designed to contain if not eradicate it, and foremost, the need for cooperation from the citizenry in an all-out campaign. But while we support the State's efforts, we do so not at the expense of fundamental rights and liberties and constitutional safeguards against arbitrary and unreasonable acts of Government. If in the event that as a result of this ruling, we prove to be an "obstacle" to the vital endeavour of stamping out the blackmarketing of valuable foreign exchange, we do not relish it and certainly, do not mean it. The Constitution simply does not leave us much choice. WHEREFORE, the petition is DISMISSED. No costs. SO ORDERED.

Rule 126, Sec. 2. Personal property to be seized. A search warrant may be issued for the search and seizure of personal property: (a) Subject of the offense; (b) Stolen or embezzled and other proceeds or fruits of the offense; and (c) Use or intended to be used as the means of committing an offense.

E spano vs. C ourtof A ppeals Facts:Sometime July of 1 99 1 , the narcotics division of theWPD conducted a by bust operation in Zamora andPandacan Streets in Manila after the confirmation of drugpushing reports in the same area.During the operation in the area, after the policeofficers saw the accused selling something to a buyer, theyapproached Espano and frisked him. The officers seized twoplastic tea bags of marijuana from the accused. They thenlater asked the accused if there are more marijuana in him,the Espano replied that there are more in his house. Thus,the arresting officers went to Espanos house andsubsequently confiscated ten more plastic tea bags of marijuana.Espano was brought to the police, and charged inviolation with RA 6425. The RTC Br. 1 of Manila convictedhim in the charges whish was later affirmed in toto by therespondent appellate court. Hence, this petition for review. As defense, the petitioner contends that the drugsconfiscated are inadmissible evidence against him beingsearch and seized illegally.Issue:Was the contention correct?Held:Partly, the drugs confiscated in his person isadmissible, however, the articles sought in his residencecannot be admitted as evidence by the prosecution beingunlawfully seized.Rule 11 3 Section 5(a) of the Rules of Court providesthat an officer may without a warrant arrest a person when inhis presence:(a) Has committed(b) Is actually committing(c) Or is about to commit a crimeIn the case at bar, petitioner's arrest falls squarely under the aforecited rule. He was caught in flagranti as a result of abuy-bust operation conducted by police officers on the basisof information received regarding the illegal trade of drugswithin the area of Zamora and Pandacan Streets, Manila.The police officer saw petitioner handing over something toan alleged buyer. After the buyer left, they searched him anddiscovered two cellophanes of marijuana. His arrest was,therefore, lawful and the two cellophane bags of marijuanaseized were admissible in evidence, being the fruits of thecrime. As for the ten cellophane bags of marijuana found atpetitioner's residence, however, the same inadmissiblein evidence.The 1 9 8 7 Constitution guarantees freedom againstunreasonable searches and seizures under Article III,Section 2 which provides: The right of the people to be secure in their persons,houses, papers and effects against unreasonablesearches and seizures of whatever nature and for anypurposes shall be inviolable, and no search warrant or warrant of arrest shall issue except upon probable causeto be determined personally by the judge after examination under oath or affirmation of the complainantand the witnesses he may produce, and particularlydescribing the place to be searched and the persons or things to be seized." An exception to the said rule is a warrantless searchincidental to a lawful arrest for dangerous weapons or anything, which may be used as proof of the commission of an offense. It may extend beyond the person of the onearrested to include the premises or surroundings under hisimmediate control. In this case, the ten cellophane bags of marijuana seized at petitioner's house after his arrest atPandacan and Zamora Streets do not fall under the saidexceptions.The articles seized from petitioner during his arrestwere valid under the doctrine of search made incidental to alawful arrest. The warrantless search made in his house,however, which yielded ten cellophane bags of marijuanabecame unlawful since the police officers were not armedwith a search warrant at the time. Moreover, it was beyondthe reach and control of petitioner.

People vs. Bati (G.R. No. 87429, August 27, 1990) Facts: By the word of their civilian informer, Patrolmen Jose Luciano, Angelito Caraan, Nelson Dimatulac and Democrito Cuenca immediately proceeded to the vicinity an alleged buy-and-sell of marijuana was taking place. They saw Marquez giving something to Bati, who, thereafter, handed a wrapped object, which turned out to be marijuana worth P190, to Marquez who then inserted the object inside the front of his pants in front of his abdomen while Bati, on his part, placed the thing

given to him inside his pocket. Marquez was arrested on the spot. Both Bati and Marquez were brought to the Police station where they admitted they were in the buying and selling of the confiscated marijuana.

Issue: Appellant contends that the arrest was not valid as the requirements for a warrantless arrest were not complied with.

Held: This contention is without merit.

Section 5 Rule 113 of the Rules in Criminal Procedure clearly provides:

Sec. 5. Arrest without warrant, when lawful. A peace officer or private person may, without warrant, arrest a person: (a) When in his presence, the person to be arrested has committed, is actually committing, or is attempting to commit an offense; (b) When an offense has in fact just been committed, and he has personal knowledge of facts indicating that the person to be arrested has committed it x x x It is the considered view of the Court that there was no need for Luciano and Caraan to be armed with a warrant of arrest when they arrested Marquez and the accused since they had personal knowledge of the actual commission of the crime viz: They were eyewitnesses to the illegal exchange of marijuana and P190.00 between Marquez and accused who were caught in flagrante delicto. The facts and circumstances attendant precisely fall under Sec. 5, (a), Rule 113 of the Rules on Criminal Procedure. The subsequent arrest of Marquez and accused were made under the principle of "hot pursuit". The recovery of the marijuana from Marquez and the P190.00 from accused by the said police officers were not violative of their constitutional rights since Marquez and the accused voluntarily surrendered them to the police officers. But even for the sake of argument that the recovery of the marijuana and peso bills were against the consent of Marquez and accused, still, the search on their persons was incidental to their valid warrantless arrest. For, the rule that searches and seizures must be supported by a valid warrant is not an absolute rule. There are at least three exceptions: (1) search incidental to an arrest, (2) search of a moving vehicle and (3) seizure of evidence in plain view. In the case at bar, the searches made on Marquez and accused were incidental to their valid arrest.

Having caught the appellant in flagrante as a result of the buy-bust operation, the policemen were not only authorized but were also under obligation to apprehend the drug pusher even without a warrant of arrest And since appellant's arrest was lawful, it follows that the search made incidental to the arrest was also valid.

Supreme Court Poetry: The law is severe because those who are caught in the strangle hold of prohibited drugs not only slide into the ranks of the living dead, what is worse, they become a grave menace to the safety of the law-abiding members of society.

Josefina Garcia-Padilla vs Minister of Defense Juan Ponce Enrile et al


Reversal of the Lansang Doctrine & Reinstatement of the Montenegro Doctrine In July 1982, Sabino Padilla, together w/ 8 others who were having a conference in a house in Bayombong, NV, were arrested by members of the PC. The raid of the house was authorized by a search warrant issued by Judge Sayo. Josefina, mother of Sabino, opposed the arrest averring that no warrant of arrest was issued but rather it was just a warrant of arrest hence the arrest of her son and the others was w/o just cause. Sabino and companions together with 4 others were later transferred to a facility only the PCs know. Josefina petitioned the court for the issuance of the writ of habeas corpus. ISSUE: Whether or not the arrests done against Sabino et al is valid. HELD: In a complete about face, the SC decision in the Lansang Case was reversed and the ruling in the Barcelon Case & the Montenegro Case was again reinstated. The questioned power of the president to suspend the privilege of the WoHC was once again held as discretionary in the president. The SC again reiterated that the suspension of the writ was a political question to be resolved solely by the president. It was also noted that the suspension of the privilege of the writ of habeas corpus must, indeed, carry with it the suspension of the right to bail, if the governments campaign to suppress the rebellion is to be enhanced and rendered effective. If the right to bail may be demanded during the continuance of the rebellion, and those arrested, captured and detained in the course thereof will be released, they would, without the least doubt, rejoin their comrades in the field thereby jeopardizing the success of government efforts to bring to an end the invasion, rebellion or insurrection.

NOTE: This ruling was abrogated by Sec 18, Art 7 of the 1987 Constitution which expressly constitutionalized the Lansang Doctrine. Note as well that under Art 3 (Sec 13) of the Constitution it is stated that the right to bail shall not be impaired even if the privilege of the writ of habeas corpus is suspended. GARCIA PADILLA VS ENRILE

Facts:

The records show that nine (9) of the fourteen (14) detainees herein were arrested on July 6, 1982 at about 1:45 p.m. when three (3) teams of the PC/INP of Bayombong, Nueva after securing a Search Warrant conducted a raid at the residence of Dra. Aurora Parong. Apprehended during the said raid were Dra. Aurora Parong, Benjamin Pineda, Sabino Padilla, Francisco Divinagracia, Zenaida Mallari, Letty Ballogan, Norberto Portuguese, and Mariano Soriano who were then having a conference in the dining room of Dra. Parong's residence which had been doing on since 10:00 a.m. of that same day. The other four (4) detainees herein, namely: Imelda de los Santos, Eufronio Ortiz, Jr., Juanita Granada, and Bienvenida Garcia, were arrested on the following day, July 7, 1982 by the same PC teams. On July 15, 1982, Tom Vasquez was arrested, and his Volkswagen car, bearing Plate No. DAP 347, was seized by the PC authorities. The herein fourteen (14) detainees were all detained at the PC/INP Command Headquarters, Bayombong, Nueva Vizcaya from July 6, 1982 until their transfer on the morning of August 10, 1982 to an undisclosed place reportedly to Camp Crame, Quezon City, to Echague, Isabela, and to Tuguegarao, Cagayan. Hence, this petition for the writ of habeas corpus and mandamus filed by Josefina Garcia-Padilla, mother of detained petitioner Sabino G. Padilla, Jr. on August 13, 1982. The mandamus aspect of the instant petition has, however, become moot and academic, and whereabouts of petitioners having already become known to petitioner Josefina Garcia-Padilla.

Ruling: The last question relates to the legality of the Presidential Commitment Order (PCO) issued by the President on July 12, 1982, tested by the conformity of its issuance to the procedure laid down under LOI 1211, petitioners insisting that the LOI limits the authority of the President to cause the arrest and detention of persons engaged in or charged with, the crimes mentioned in Proclamation No. 2045. They contend that the procedure prescribed in the LOI not having been observed, the PCO issued thereunder did not validate the initial illegal arrest of the herein petitioners as wen as their continued detention. By its very nature, and clearly by its language, LOI 1211 is a mere directive of the President as Commander-in-Chief of the Armed Forces of the Philippines to his subordinates or implementing officers for the ultimate objective of providing guidelines in the arrest and detention of the persons covered by Presidential Proclamation No. 2045. The purpose is "to insure protection to individual liberties without sacrificing the requirements of public order and safety and the effectiveness of the campaign against those seeking the forcible overthrow of the government and duty constituted authorities. " LOI 1211 does not, in any manner, limit the authority of the President to cause the arrest and detention of persons engaged in, or charged with the crimes or offenses mentioned in said Proclamation in that he (President) would subject himself to the superior authority of the judge who, under normal judicial processes in the

prosecution of the common offenses, is the one authorized to issue a judicial warrant after a preliminary investigation is conducted with a finding of probable cause. Those who would read such an intention on the part of the President in issuing LOI 1211 seems to do so in their view that LOI forms part of the law of the land under the 1976 amendment of the Constitution. They would then contend that a PCO issued not in compliance with the provisions of the LOI would be an illegality and of no effect. To form part of the law of the land, the decree, order or LOI must be issued by the President in the exercise of his extraordinary power of legislation as contemplated in Section 6 of the 1976 amendments to the Constitution, whenever in his judgment, there exists a grave emergency or a threat or imminence thereof, or whenever the interim Batasan Pambansa or the regular National Assembly fails or is unable to act adequately on any matter for any reason that in his judgment requires immediate action. There can be no pretense, much less a showing, that these conditions prompted the President to issue LOI 1211. Verily, not all LOI issued by the President should be dignified into forming part of the law of the land.
People vs. Maspil, the police had also been tipped off by an informant thatt h e a c c u s e d M aspil and Basking would be transporting a large volume of marijuana to Baguio City. The operatives intercepted a Sarao type jeep driven by Maspil andBagking as his companion. Upon inspection, the jeep was found loaded with two plastic bags., which when opened contained several bundles of suspected dried marijuana leaves.Maspil and Basking were arrested and the suspected marijuana leaves were confiscated. The Supreme Court held that this case falls squarely to the exceptions of requiring a searchwarrant. It also falls to the exceptions enumerated in Rule 113, Sec. 5 which state:Sec. 5. Arrest without warrant; when lawful. A peace officer or a private person may, withouta warrant, arrest a person:(a) When, in his presence, the person to be arrested has committed, is actually committing,or is attempting to commit an offense.It is because according to the Court, the appellants (Maspil and Basking) were caught inflagrante delicto since they were transporting the prohibited drugs at the time of the arrest.(Emphasis Supplied)There are also several cases such People vs Ramos, Amelia Roque vs de Villa, Anonuevovs Ramos and People vs Tangliben, where in the court rulings did not strictly observed therule on personal knowledge of facts constituting a probable cause under Rule 113, Sec. 5 (a)and (b). The fact that the search yielded possession of illegal articles was included as a justification for a warrantless arrest under Section 5(a) although the arresting officer at thetime of arrest has no personal knowledge of a crime being committed.In the case at hand, the Makati Anti-illegal Drugs Unit caught the respondents in flagrantedelicto and that is delivering or transporting the prohibited drugs at the time of the arrest. Theitems confiscated were inside the white crosswind, Respondent 1 was driving whererespondent 2 is his companion. The fact that the respondents were caught in flagrante delicto is already a sufficient ground for the Makati Anti-Illegal Drug Unit to arrest them.And thus, there is a probable cause for the Makati Anti-illegal Drugs Unit to arrest therespondents.

G.R. No. 85177 August 20, 1990 PEOPLE OF THE PHILIPPINES, plaintiff-appellee, vs. MOISES MASPIL, JR. y WAYWAY and SALCEDO BAGKING y ALTAKI, defendantsappellants.

The Solicitor General for plaintiff-appellee. Peter C. Fianza for defendants-appellants.

GUTIERREZ, JR., J.: This petition is an appeal from the decision of the Regional Trial Court of Baguio City, Branch 5, the dispositive portion of which reads:
WHEREFORE, the Court finds and declares the accused MOISES MASPIL, JR. y WAYWAY and SALCEDO BAGKING y ALTAKI, guilty beyond reasonable doubt of the crime of illegal transportation of marijuana as charged and hereby sentences EACH of them to suffer LIFE IMPRISONMENT; to pay a fine of P20,000.00, without subsidiary imprisonment in case of insolvency; and to pay their proportionate shares in the costs. The confiscated marijuana (Exhibits "B", "B-1 " to "B-23"; "C" to "C-16", "D", "D-1" to "D20"; "E", "E-1", to "E-14", "F", "F-1", "G", "G-l") are hereby declared forfeited in favor of the Government and upon the finality of this decision, the Branch Clerk of Court is directed to turn over the same to the Dangerous Drugs Board (NBI), through the Chief, PC Crime Laboratory, Regional Unit No. 1, Camp Dangwa, La Trinidad, Benguet, for disposition in accordance ' with law. (Rollo, pp. 25-26)

In Criminal Case No. 4263-R, the information filed against the two accused alleged:
That on or about the 1st day of November, 1986, at Sayangan, Municipality of Atok, Province of Benguet, Philippines, and within the jurisdiction of this Honorable Court, the above-named accused, conspiring, confederating and mutually aiding each other, and without any authority of law, did then and there willfully, unlawfully and knowingly transport and carry in transit from Sinto, Bauko, Mt. Province to Atok, Benguet One Hundred Eleven Kilos and Nine Grams (111.9 kilos), more or less, of dried marijuana leaves which are sources of dangerous and prohibited drugs and from which dangerous and prohibited drugs may be derived and manufactured, in violation of the said law. (Rollo, p. 11)

The narration of facts by the trial court is as follows:


According to Jerry Valeroso, Sgt. Amador Ablang and Sgt. Florentino Baillo, all members of the First Narcotics Regional Unit of the Narcotics Command stationed in Baguio City, (See also Exhibit "I") on October 30, 1986, they established a checkpoint in front of the Municipal Hall at Sayangan, Atok, Benguet, which is along the Halsema Highway, to check on vehicles proceeding to Baguio City because their Commanding Officer, Maj. Basilio Cablayan had been earlier tipped off by some confidential informers that the herein accused Maspil and Basking would be transporting a large volume of marijuana to Baguio City. The informers went along with the operatives to Sayangan. At about 2:00 o'clock in the early morning of November 1, the operatives intercepted a Sarao type jeep driven by Maspil with Bagking as his companion. Upon inspection, the jeep was found loaded with two (2) plastic sacks (Exhibits "B" and "D"), one (1) jute sack (Exhibit "C") and three (3) big round tin cans (Exhibits "E", "F" and "G") which, when

opened contained several bundles of suspected dried marijuana leaves (Exhibits "B-1", to "B-23", "C-1" to "C-16", D-1" to "D-20", "E-1" to "E-14","F-1" and "G-1"). Maspil and Basking were arrested and the suspected marijuana leaves were confiscated. The confiscated items were later on referred to the PC Crime Laboratory, Regional Unit 1, for examination (Exhibit "A"). Forensic Chemist Carlos V. Figuerroa performed the requested examination and determined that the specimen, with an aggregate weight of 115.66 kilos, were positive to the standard tests for marijuana. The accused admitted that the marijuana dried leaves were indeed confiscated from the jeep being then driven by Maspil with Bagking as his helper. However, they claimed that the prohibited drugs belonged to two of their passengers who loaded them in the jeep as paying cargo for Baguio City without the accused knowing that they were marijuana. The accused declared that on October 31, 1986, at the burned area along Lakandula Street, Baguio City, a certain Mrs. Luisa Mendoza hired the jeep of Maspil to transport her stock of dried fish and canned goods contained in cartons to Abatan, Buguias, Benguet, because her own vehicle broke down. They left Baguio City at about 1:00 o'clock in the afternoon (11:30 in the morning, according to Bagking) with Mrs. Mendoza, her helper and salesgirls on board the jeep with Maspil as driver and Bagking as his own helper. They arrived at Abatan at about 6:00 o'clock in the evening. After unloading their cargo, Maspil and Bagking repaired to a restaurant for their dinner before undertaking the trip back to Baguio City. While thus eating, they were approached by two persons, one of whom they would learn later on to be a certain Danny Buteng. Buteng inquired if they were going to Baguio City and upon being given an affirmative answer, he said that he would ride with them and that he has some cargo. Asked what the cargo was, Buteng replied that they were flowers in closed tin cans and sealed sacks for the commemoration of All Souls Day in Baguio City. After Buteng had agreed to Maspil's condition that he would pay for the space to be occupied by his cargo, Buteng himself and his companion loaded the cargo and fixed them inside Maspil's jeep. Maspil and Bagking left Abatan at about 7:00 o'clock that same evening of October 31. Aside from Buteng and companion they had four other passengers. These four other passengers alighted at Natubleng, Buguias, Benguet. Upon reaching Sayangan, Atok, Benguet, Maspil stopped at the Marosan Restaurant where they intended to take coffee. Their remaining passengers-Buteng and companion alighted and went to the restaurant. However, a soldier waved at Maspil to drive to where he was, which Maspil did. The soldier secured Maspil's permission to inspect their cargo after which he grabbed Maspil on the latter's left shoulder and asked who owned the cargo. Maspil told the soldier that the cargo belonged to their passengers who went to the restaurant. The soldier called for his companions and they went to look for Maspil's passengers in the restaurant. Later on, they returned and placed Maspil and Bagking under arrest since their cargo turned out to be marijuana. Lawrence Balonglong, alias Banawe, a radio reporter of DZWX Bombo Radio who was invited by Lt Valeroso to witness the operation, affirmed the unsuccessful pursuit of the alleged two companions of Maspil and Bagking. He recalled that he was awakened from his sleep at the townhall in Sayangan after the arrest of Maspil and Bagking. When he went to the scene, the NARCOM operatives boarded the jeep of Maspil to chase the two companions of Maspil and Bagking. Balonglong climbed on top of the jeep with his

camera to join the chase. They proceeded toward the direction of Bontoc but failed to catch anyone. Hence, they returned. Thereupon, Maspil and Bagking were taken to the townhall where they were allegedly maltreated to admit ownership of the confiscated marijuana. At about 4:00 o'clock in the afternoon of November 1, the soldiers took them away from Sayangan to be transferred to their station at Baguio City. On their way, particularly at Km. 32 or 34, they met Mike Maspil, an elder brother of Moises Maspil, and the soldiers called for him and then Lt. Valeroso and his men mauled him on the road. Mike testified that between 3:00 and 4:00 o'clock in the afternoon of November 1, he was informed by a neighbor that his brother Moises was detained at the Atok Municipal Jail. So he called for Jose Pos-el and James Longages, his driver and helper, respectively, to go along with him to see Moises. They rode in his jeep. On the way, they met the group of Lt. Valeroso. For no apparent reason, Lt. Valeroso boxed and kicked him several times. Thereafter, Lt. Valeroso placed him under arrest together with his driver and helper. They were all brought to a shoe store on Gen. Luna Road, Baguio City, together with Moises and Bagking. There, Lt. Valeroso got his wallet containing P21000 and Seiko wrist watch but the receipt (Exhibit "3") was issued by a certain Miss Pingil, a companion of Valeroso. He was released after nine days. He then went to Lt. Valeroso to claim his wallet, money and watch but he was told that they were with Miss Pingil. However, when he went to Miss Pingil, the latter said that the items were with Lt. Valeroso. He sought the assistance of then Tourism Deputy Minister Honorato Aquino who assigned a lawyer to assist him. The lawyer advised him to file a case against Lt. Valeroso but because of the intervening congressional elections, the matter has never been pursued. (Rollo, p. 21-24)

The appellants raise the following assignment of errors in their appeal, to wit:
I THAT THE TRIAL COURT ERRED IN NOT FINDING THAT THE ALLEGED MARIJUANA AS CHARGED IN THE INFORMATION IS DIFFERENT FROM THAT PRESENTED FOR LABORATORY EXAMINATION. II THAT THE TRIAL COURT ERRED IN FINDING THAT THERE WERE ONLY TWO OCCUPANTS, THE APPELLANTS, IN THE VEHICLE WHERE THE ALLEGED MARIJUANA WAS CONFISCATED III. THAT THE TRIAL COURT ERRED IN FINDING THAT THE ACCUSED KNEW THAT THE CARGO THEY WERE TRANSPORTING WAS MARIJUANA. IV THAT THE TRIAL COURT ERRED IN ADMITTING IN EVIDENCE THE ALLEGED CONFISCATED MARIJUANA. V

THAT THE TRIAL COURT ERRED IN SHIPPING FROM THE PROSECUTION THE BURDEN OF PROVING THE COMMISSION OF THE OFFENSE CHARGED TO THE APPELLANTS TO PROVE THEIR INNOCENCE." (Rollo, p. 40)

The main defense of the appellants is their claim that the prohibited drugs belonged to their two passengers who loaded them in the jeep as paying cargo without the appellants knowing that the cargo was marijuana. In the second and third assignment of errors, the appellants claim that the trial court erred in not appreciating their version of the facts. The appellants state that the trial court's reliance on Sgt. Baillo's testimony that they were the only ones in the jeep cannot be given credence as Sgt. Baillo's testimony is full of inconsistencies. The appellants cite Sgt. Baillo's inconsistencies as to the time of the arrest whether morning or afternoon, the time the checkpoint was removed and the person who were with him at the time of arrest. It has been ruled that inconsistencies in the testimonies of the prosecution witnesses not on material points is not fatal. Moreover, minor inconsistencies are to be expected but must be disregarded if they do not affect the basic credibility of the evidence as a whole. (People v. Marcos, G. R. No. 83325, May 8, 1990). The defense even state that there were l lot of policemen (T.S.N., December 1, 1987, p. 22) and it was but natural that there would be confusion on who was there at the time of the arrest. The trial court gave credence to the positive and categorical statement of Sgt. Baillo that there were only two occupants, and these were the appellants inside the jeepney at the time (T.S.N., June 30, 1987, p. 18). We see no cogent reason to reverse this finding of fact. There is nothing in the records to suggest that the arrest was motivated by any reason other than the desire of the police officers to accomplish their mission. Courts generally give full faith and credit to police officers when the facts and circumstances surrounding their acts sustain the presumption that they have performed their duties in a regular manner. (Rule 131, Section 5 (m), Rules of Court; People v. Marcos, supra; People v. Yap and Mendoza, G.R. Nos. 87088-89, May 9,1990) The appellants put forward the testimony of Lawrence Balonglong which corroborates and affirms their stand that there were, indeed, passengers in the jeepney. However, a close perusal of said testimony reveals no such corroboration. The pertinent portions of Balonglong's testimony is as follows:
xxx xxx xxx

Q Where were you when these two persons were apprehended? A I was in the Municipal Hall asleep, sir. Q How did you know then that these people were apprehended? A It is like this, sir, on the night of October 31, 1 was then asleep and at around 11:00, 1guess, p.m., they awakened me so I went and I saw these two guys being apprehended by the Narcom operative. Court: Q You saw them being apprehended? A No, sir ... I saw them there. Q Already apprehended? A Already apprehended. Atty. Fianza: Q And when you saw these persons, what did you do, if any? A What I recall is that when I went to the road, where these two guys were apprehended, the operatives boarded the same jeep and I even climbed the jeep . . . on top of the jeep holding my camera and tape recorder and we . . . I don't know ... they chased, according to the operatives, they chased two companions of the two arrested guys. (T.S.N., May 11, 1988, p. 4)

In their brief, the appellants even admit that "he (Balonglong) did not see the passengers" and it was just his impression that there were other people present. (Appellant's Brief, p. 7) The appellants maintain that they did not know what was in the cargo. Their main concern was in going back to Baguio City and they saw no need to question their two passengers on why flowers were being kept in closed cans and sacks. They were apprehended after midnight. They traversed a lonely and reputedly dangerous portion of the mountain highway. The appellants' version is not believable. It is inconceivable that the appellants would not even bother to ask the names of the strangers who approached them in a restaurant at night wanting to hire their jeepney, considering that they were familiar with the Identity of the passenger, Luisa Mendoza, who hired them to transport her goods to Abatan, Buguias, Benguet. It is likewise incredible that the appellants did not show the slightest curiousity as to why flowers were being kept in closed tin cans and sealed sacks and cellophane. On the

other hand, the appellants had clear knowledge that Luisa Mendoza was transporting cartons containing dried fish and canned goods on the trip out of Baguio. It is contrary to human experience that the appellants would inquire about the name of the passenger and the cargo she was loading on their jeep and not doing the same about another who would transport goods on a midnight trip. Well-settled is the rule that evidence to be believed, must not only proceed from the mouth of a credible witness but it must be credible itself. No better test has yet been found to measure the value of a witness than its conformity to the knowledge and common experience of mankind. (People v. Maribung, 149 SCRA 292, 297 [1987] ; People v. Aldana, G.R. No. 81817, July 27, 1989; People v. Pascua, G.R. No. 82303, December 21, 1989). The appellants further allege that if, indeed they knew about the contents of their cargo, they would have adopted means to prevent detection or to evade arrest. At the time the appellants were being motioned by the policemen to come nearer the checkpoint, there was no way that the appellants could have evaded the arrest without putting their lives in jeopardy. They decided to just brazen it out with police and insist on their version of the story. As for the other assigned errors, the appellants in the first assigned error, contend that since there is a discrepancy of 3.76 between the number of kilos stated in the information (111.9 kilos) and in the report (115.66 kilos) of the forensic chemist, it is very likely that the marijuana presented as evidence was not the one confiscated from the appellants or even if they were the same, it could have already been tampered with. The appellants conclude that the marijuana then, cannot be admitted as evidence. The marijuana examined by the forensic chemist, which was contained in three big round tin cans, two jute sacks (there was really only one jute sack colored light green which was confiscated but since one of the plastic sacks [green] appeared to be tattered, some of its contents were transferred to a white jute sack), (T.S.N., June 23, 1987, p. 5) and two plastic bags colored yellow and green (T.S.N., June 23, 1987, p. 3), was positively Identified to be the same as those confiscated from the appellants. This is very clear from the testimony of Lt. Valeroso who stated:
xxx xxx xxx
Q When you went down, where were these two suspects, as you said? A They were sitted (sic) at the front seat. Q Front seat of what? A The jeep, sir. Q And did you ask or see what was inside the jeep? A Yes.

Q And what were those? A It was all suspected marijuana dried leaves contained in three big cans, one sack colored green, two sacks colored yellow and green. (Italics supplied, T.S.N., September 16, 1987, P. 4)

Lt. Valeroso further testified that Exhibits "B" (yellow plastic big), "C" light green jute sack, "D" (green plastic bag), "E" (one big can), "F" (second can), "G" (third can) were, indeed, the same articles which he saw at the back of the jeepney of the appellants. (T.S.N., September 16, 1987, p. 5) One of the appellants, Moises Maspil, even admitted that the articles Identified by Lt. Valeroso in his testimony were indeed, the same articles confiscated from their jeepney at Sayangan, Atok, Benguet. (T.S.N., February 24, 1988, pp. 34-35) Moreover, the words "more or less" following the weight in kilos of the marijuana in the questioned information declare that the number of kilos stated therein is just an approximation. It can therefore be a little lighter or heavier. The slight discrepancy is not material. Another ground stated by the appellants for the inadmissibility in evidence of the confiscated marijuana is that the marijuana allegedly seized from them was a product of an unlawful search without a warrant. In the case of Valmonte v. de Villa, G.R. No. 83988, September 29, 1989, the Court held that:
xxx xxx xxx True, the manning of checkpoints by the military is susceptible of abuse by the men in uniform, in the same manner that all government power is susceptible of abuse. But at the cost of occasional inconvenience, discomfort and even irritation to the citizen, the checkpoints during these abnormal times, when conducted within reasonable limits are part of the price we pay for an orderly society and a peaceful community.

The search was conducted within reasonable limits. There was information that a sizeable volume of marijuana will be transported to take advantage of the All Saints Day holiday wherein there will be a lot of people going to and from Baguio City (T.S.N., September 16, 1987, p. 6). In fact, during the three day (October 30, 1986 to November 1, 1986) duration of the checkpoint, there were also other drug related arrests made aside from that of the two appellants. But even without the Valmonte ruling, the search would still be valid. This case involves a search incident to a lawful arrest which is one of the exceptions to the general rule requiring a search warrant. This exception is embodied in Section 12 of Rule 126 of the 1985 Rules on Criminal Procedure which provides:
Sec. 12. Search incident to lawful arrest. A person lawfully arrested may be searched for dangerous weapons or anything which may be used as proof of the commission of an offense, without a search warrant.

and Rule 113, Section 5 (11) which state:


Sec. 5. Arrest without warrant; when lawful. A peace officer or a private person may, without a warrant, arrest a person: (a) When, in his presence, the person to be arrested has committed, is actually committing, or is attempting to commit an offense.

This case falls squarely within the exceptions. The appellants were caught in flagrante delicto since they were transporting the prohibited drugs at the time of their arrest. (People v. Tangliben, G.R. No. 63630, April 6, 1990) A crime was actually being committed. The appellants, however, cite the case of People V. Aminnudin, (163 SCRA 402 [1988]. In said case, the PC officers received information that the accused-appellant, on board a vessel bound for Iloilo City, was carrying marijuana. When the accused-appellant was descending the gangplank, the PC officers detained him and inspected the bag that he was carrying and found marijuana. The Court ruled that since the marijuana was seized illegally, it is inadmissible in evidence. There are certain facts of the said case which are not present in the case before us. In the Aminnudin case, the records showed that there was sufficient time and adequate information for the PC officers to have obtained a warrant. The officers knew the name of the accused, that the accused was on board M/V Wilcon 9, bound to Iloilo and the exact date of the arrival of the said vessel. On the other hand, in this case there was no information as to the exact description of the vehicle and no definite time of the arrival. A jeepney cannot be equated with a passenger ship on the high seas. The ruling in the Aminnudin case, is not applicable to the case at bar. As for the fifth and last assigned error we agree with the Solicitor General that:
Examination of the testimonies of appellants show that they admit the fact that the confiscated marijuana was taken from their jeep while they were transporting it from Abatan, Buguias, Benguet to Baguio City. This being so, the burden of the prosecution to prove illegal transportation of prohibited drugs punished under Section 4 of RA 6425, as amended, has been satisfactorily discharged. The rule in civil as well as in criminal cases is that each party must prove his own affirmative allegations. The prosecution avers the guilt of the accused who is presumed to be innocent until the contrary is proved. Therefore, the prosecution must prove such guilt by establishing the existence of all the elements of the crime charged. But facts judicially known, presumed, admitted or confessed need not be proved. (Rule 129, Sec. 4, Rules on Evidence) (Appellee's Brief, p. 26-27)

WHEREFORE, the guilt of the appellants having been proved beyond reasonable doubt, the appealed decision is hereby AFFIRMED. SO ORDERED.

POSADAS VS. COURT OF APPEALS [188 SCRA 288; G.R. NO. 89139; 2 AUG 1990] Friday, February 06, 2009 Posted by Coffeeholic Writes Labels: Case Digests, Political Law

Facts: Members of the Integrated National Police (INP) of the Davao Metrodiscom assigned with the Intelligence Task Force, Pat. Ursicio Ungab and Pat. Umbra Umpar conducted surveillance along Magallanes Street, Davao City. While in the vicinity of Rizal Memorial Colleges they spotted petitioner carrying a "buri" bag and they noticed him to be acting suspiciously. They approached the petitioner and identified themselves as members of the INP. Petitioner attempted to flee but his attempt to get away was unsuccessful. They then checked the "buri" bag of the petitioner where they found one (1) caliber .38 Smith & Wesson revolver with Serial No. 770196, two (2) rounds of live ammunition for a .38 caliber gun, a smoke (tear gas) grenade, and two (2) live ammunitions for a .22 caliber gun. They brought the petitioner to the police station for further investigation. In the course of the same, the petitioner was asked to show the necessary license or authority to possess firearms and ammunitions found in his possession but he failed to do so. He was then taken to the Davao Metrodiscom office and the prohibited articles recovered from him were indorsed to M/Sgt. Didoy the officer then on duty. He was prosecuted for illegal possession of firearms and ammunitions in the Regional Trial Court of Davao City.

Issue:

Whether

or

Not

the

warantless

search

is

valid.

Held: In justifying the warrantless search of the buri bag then carried by the petitioner, argues that under Section 12, Rule 136 of the Rules of Court a person lawfully arrested may be searched for dangerous weapons or anything used as proof of a commission of an offense without a search warrant. It is further alleged that the arrest without a warrant of the petitioner was lawful under the circumstances. in the case at bar, there is no question that, indeed, it is reasonable considering that it was effected on the basis of a probable cause. The probable cause is that when the petitioner acted suspiciously and attempted to flee with the buri bag there was a probable cause that he was concealing something illegal in the bag and it was the right and duty of the police officers to inspect the same. It is too much indeed to require the police officers to search the bag in the possession of the petitioner only after they shall have obtained a search warrant for the purpose. Such an exercise may prove to be useless, futile and much too late. Clearly, the search in the case at bar can be sustained under the exceptions heretofore discussed, and hence, the constitutional guarantee against unreasonable searches and seizures has not been violated.

People vs. dela Cruz [GR 83260, 18 April 1990]


Second Division, Regalado (J): 4 concur Facts: After receiving a confidential report from Arnel, their informant, a buy-bust operationwas conducted by the 13th Narcotics Regional Unit through a team composed of T/Sgt. JaimeRaposas as Team Leader, S/Sgt. Rodelito Oblice, Sgt. Dante Yang, Sgt. Vicente Jimenez, P/Pfc.Adolfo Arcoy as poseur-buyer and Pat. Deogracias Gorgonia at Maliclic St., Tondo, Manila ataround 2:30 p.m. of 4 May 1987 to catch the pusher/s. P/Pfc. Adolfo Arcoy acted as the poseur- buyer with Arnel as his companion to buy marijuana worth P10.00 from the two accused, Juande la Cruz and Reynaldo Beltran. At the scene, it was Juan de la Cruz whom Arcoy firstnegotiated with on the purchase and when Arcoy told De la Cruz that he was buying P10.00worth of marijuana, De la Cruz instructed Reynaldo Beltran to give one aluminum foil of marijuana which Beltran got from his pants pocket and delivered it to Arcoy. After ascertainingthat the foil of suspected marijuana was really marijuana, Arcoy gave the prearranged signal tohis teammates by scratching his head and his teammates who were strategically positioned in thevicinity, converged at the place, identified themselves as NARCOM agents and effected thearrest of De la Cruz and Beltran. The P10.00 marked bill used by Arcoy was found in the possession of Juan de la Cruz together with two aluminum foils and containing marijuana. Juande la Cruz y Gonzales and Reynaldo Beltran y Aniban were charged in Criminal Case 8754417of the Regional Trial Court (RTC) of Manila with violation of Section 4, Art. II, in relation toSection 21, Article IV of Republic Act 6425, as amended. The court, on 15 March 1988, foundDela Cruz and Beltran guilty beyond reasonable doubt and sentenced each of them to suffer the penalty of reclusion perpetua, with the accessory penalties provided by law; to pay a fine of P20,000.00, without subsidiary imprisonment in case of insolvency, and each to pay one-half of the costs. From this decision, de la Cruz and Beltran appealed. In a letter of the Warden, ManilaCity Jail, dated 3 March 1989, the Court was informed of the death of de la Cruz on 21 February1989. Thus, the criminal case against de la Cruz was dismissed in the Supreme Court resolutionof 25 September 1989. The present appellate proceeding is limited only to Beltran. Issue: Whether the warrantless seizure incidental to the buy-bust operation violates Beltransconstitutional rights against unreasonable search and seizure. Held: A buy-bust operation is the method employed by peace officers to trap and catch amalefactor in flagrante delicto. It is essentially a form of entrapment since the peace officer neither instigates nor induces the accused to commit a crime. Entrapment is the employment of such ways and means for the purpose of trapping or capturing a lawbreaker from whose mind thecriminal intent originated. Oftentimes, it is the only effective way of apprehending a criminal inthe act of the commission of the offense. While it is conceded that in a buy-bust operation, thereis seizure of evidence from ones person without a search warrant, needless to state a searchwarrant is not necessary, the search being incident to a lawful arrest. A peace officer may,without a warrant, arrest a person when, in his presence, the person to be arrested has committed,is actually committing or is attempting to commit an offense. It is a matter of judicial experiencethat in the arrest of violators of the Dangerous Drugs Act in a buy-bust operation, the malefactorswere invariably caught red-handed. There being no violation of the constitutional right againstunreasonable search and seizure, the confiscated articles are admissible in evidence.

GO VS. COURT OF APPEALS [206 SCRA 138; G.R. NO. 101837; 11 FEB 1992] Wednesday, February 04, 2009 Posted by Coffeeholic Writes Labels: Case Digests, Political Law

Facts:

Petitioner, while traveling in the wrong direction

on a one-way street, almost had a collision with another vehicle. Petitioner thereafter got out of his car, shot the driver of the other vehicle, and drove off. An eyewitness of the incident was able to take down petitioners plate number and reported the same to the police, who subsequently ordered a manhunt for petitioner. 6 days after the shooting, petitioner presented himself in the police station, accompanied by 2 lawyers, the police detained him. Subsequently a criminal charge was brought against him. Petitioner posted bail, the prosecutor filed the case to the lower court, setting and commencing trial without preliminary investigation. Prosecutor reasons that the petitioner has waived his right to preliminary investigation as bail has been posted and that such situation, that petitioner has been arrested without a warrant lawfully, falls under Section 5, Rule 113 and Section 7, Rule 112 of The 1985 Rules of Criminal Procedure which provides for the rules and procedure pertaining to situations of lawful warrantless arrests. Petitioner in his petition for certiorari assails such procedure and actions undertaken and files for a preliminary investigation.

Issues:
(1) Whether or Not warrantless arrest of petitioner was lawful. (2) Whether or Not petitioner effectively waived his right to

preliminary

investigation.

Held: Petitioner

and prosecutor err in relying on Umil v.

Ramos, wherein the Court upheld the warrantless arrest as valid effected 1 to 14 days from actual commission of the offenses, which however constituted continuing crimes, i.e. subversion, membership in an outlawed organization, etc. There was no lawful warrantless arrest under Section 5, Rule 113. This is because the arresting officers were not actually there during the incident, thus they had no personal knowledge and their information regarding petitioner were derived from other sources. Further, Section 7, Rule 112, does not apply. Petitioner was not arrested at all, as when he walked in the police station, he neither expressed surrender nor any statement that he was or was not guilty of any crime. When a complaint was filed to the prosecutor, preliminary investigation should have been scheduled to determine probable cause. Prosecutor made a substantive error, petitioner is entitled to preliminary investigation, necessarily in a criminal charge, where the same is required appear thereat. Petition granted, prosecutor is ordered to conduct preliminary investigation, trial for the criminal case is suspended pending result from preliminary investigation, petitioner is ordered released upon posting a bail bond.
Go vs. Court of Appeals [GR 101837, 11 February 1992] En Banc, Feliciano (J): 5 concur Facts:

On 2 July 1991, Eldon Maguan was driving his car along Wilson St., San Juan, MetroManila, heading towards P. Guevarra St. Rolito Go y Tambunting entered Wilson St., where it isa one-way street and started traveling in the opposite or wrong direction. At the corner of Wilson and J. Abad Santos Sts., Gos and Maguans cars nearly bumped each other. Go alightedfrom his car, walked over and shot Maguan inside his car. Go then boarded his car and left thescene. A security guard at a nearby restaurant was able to take down Gos car plate number. The police arrived shortly thereafter at the scene of the shooting and there retrieved an empty shelland one round of live ammunition for a 9mm caliber pistol. Verification at the LandTransportation Office showed that the car was registered to one Elsa Ang Go. The following day,the police returned to the scene of the shooting to find out where the suspect had come from;they were informed that Go had dined at Cravings Bake Shop shortly before the shooting. The police obtained a facsimile or impression of the credit card used by Go from the cashier of the bake shop. The security guard of the bake shop was shown a picture of Go and he positivelyidentified him as the same person who had shot Maguan. Having established that the assailantwas probably Go, the police launched a manhunt for Go. On 8 July 1991, Go presented himself before the San Juan Police Station to verify news reports that he was being hunted by the police;he was accompanied by two (2) lawyers. The police forthwith detained him. An eyewitness tothe shooting, who was at the police station at that time, positively identified Go as the gunman.That same day, the police promptly filed a complaint for frustrated homicide against Go with theOffice of the Provincial Prosecutor of Rizal. First Assistant Provincial Prosecutor Dennis VillaIgnacio (Prosecutor) informed Go, in the Presence of his lawyers. that he could avail himself of his right to preliminary investigation but that he must first sign a waiver of the provisions of Article 125 of the Revised Penal Code. Go refused to execute any such waiver. On 9 July 1991,while the complaint was still with the Prosecutor, and before an information could be filed incourt, the victim, Eldon Maguan, died of his gunshot wound(s). Accordingly, on 11 July 1991,the Prosecutor, instead of filing an information for frustrated homicide, filed an information for murder before the Regional Trial Court. No bail was recommended. At the bottom of theinformation, the Prosecutor certified that no preliminary investigation had been conducted because the accused did not execute and sign a waiver of the provisions of Article 125 of theRevised Penal Code. In the afternoon of 11 July 1991, Gos counsel filed with the prosecutor an omnibus motion for immediate release and proper preliminary investigation, alleging that thewarrantless arrest of Go was unlawful and that no preliminary investigation had been conducted before the information was filed. On 12 July 1991, Go filed an urgent ex-parte motion for specialraffle in order to expedite action on the Prosecutors bail recommendation. The case was raffledto the sala of Judge Benjamin V. Pelayo (Branch 168, RTC of Pasig City), who, on the samedate, approved the cash bond posted by Go and ordered his release. Go was in fact released thatsame day. On 16 July 1991, the Prosecutor filed with the Regional Trial Court a motion for leaveto conduct preliminary investigation and prayed that in the meantime all proceedings in the court be suspended. On the said date, the trial court issued an Order 9 granting leave to conduct preliminary investigation and cancelling the arraignment set for 15 August 1991 until after the prosecution shall have concluded its preliminary investigation. On 17 July 1991, however, theJudge motu proprio issued an Order, (1) recalling the 12 July 1991 Order which granted bail: petitioner was given 48 hours from receipt of the Order to surrender himself: (2) recalling andcancelling the 16 July 1991 Order which granted leave to the Prosecutor to conduct preliminaryinvestigation: (3) treating Gos omnibus motion for immediate release and preliminaryinvestigation dated 11 July 1991 as a petition for bail and set for hearing on 23 July 1991. On 19July 1991, Go filed a petition for certiorari, prohibition and mandamus before the Supreme Courtassailing the 17 July 1991 Order. Go also moved for suspension of all proceedings in the case pending resolution by the Supreme Court of his petition: this motion was, however, denied byJudge Pelayo. On 23 July 1991, Go surrendered to the police. By a Resolution dated 24 July1991, the Supreme Court remanded the petition for certiorari, prohibition and mandamus to theCourt of Appeals. On 16 August 1991, Judge Pelayo issued an order in open court setting Gosarraignment on 23 August 1991. On 19 August 1991, Go filed with the Court of Appeals amotion to restrain his arraignment. On 23 August 1991, Judge Pelayo issued a CommitmentOrder directing the Provincial Warden of Rizal to admit Go into his custody at the RizalProvincial Jail. On the same date, Go was arraigned. In view, however, of his refusal to enter a plea, the trial court entered for him a plea of not guilty. The trial court then set the criminal casefor continuous hearings on 19, 24 and 26 September; on 2, 3, 11 and 17 October; and on 7, 8, 14,15, 21 and 22 November 1991. On 27 August 1991. Go filed a petition for habeas corpus in theCourt of Appeals. On 30 August 1991, the Court of Appeals issued the writ of habeas corpus.The petition for certiorari, prohibition and mandamus, on the one hand, and the petition for habeas corpus, upon the other, were subsequently consolidated in the Court of Appeals. TheCourt of Appeals, on 2 September 1991, issued a resolution denying Gos motion to restrain hisarraignment on the ground that motion had become moot and academic. On 19 September 1991,trial of the criminal case commenced. On 23 September 1991, the Court of Appeals rendered aconsolidated decision dismissing the 2 petitions on the grounds that Gos warrantless arrest wasvalid and Gos act

of posting bail constituted waiver of any irregularity attending his arrest,among others. On 3 October 1991, the prosecution presented three (3) more witnesses at the trial.Gos Counsel also filed a Withdrawal of Appearance with the trial court, with Gosconformity. On 4 October 1991, Go filed the present petition for Review on Certiorari. On 14October 1991, the Court issued a Resolution directing Judge Pelayo to held in abeyance thehearing of the criminal case below until further orders from the Supreme Court. Issue: Whether Go was arrested legally without warrant for the killing of Maguan, and is thusnot entitled to be released pending the conduct of a preliminary investigation. Held: Gos warrantless arrest or detention does not fall within the terms of Section 5 of Rule113 of the 1985 Rules on Criminal Procedure which provides that A peace officer or a private person may, without a warrant, arrest a person: (a) When, in his presence, the person to becreated has committed, is actually committing, or is attempting to commit an offense; (b) Whenan offense has in fact just been committed, and he has personal knowledge of facts indicatingthat the person to be arrested has committed it; and (c) When the person to be arrested is a prisoner who has escaped from a penal establishment or place where he is serving final judgmentor temporarily confined while his case is pending or has escaped while being transferred fromone confinement to another. In cases falling under paragraphs (a) and (b) hereof, the personarrested without a warrant shall be forthwith delivered to the nearest police station or jail, and heshall be proceeded against in accordance with Rule 112, Section 7. Gos arrest took place 6days after the shooting of Maguan. The arresting officers obviously were not present, withinthe meaning of Section 5(a), at the time Go had allegedly shot Maguan. Neither could thearrest effected 6 days after the shooting be reasonably regarded as effected when [theshooting had] in fact just been committed within the meaning of Section 5 (b). Moreover, noneof the arresting officers had any personal knowledge of facts indicating that Go was thegunman who had shot Maguan. The information upon which the police acted had been derivedfrom statements made by alleged eyewitnesses to the shooting one stated that Go was thegunman another was able to take down the alleged gunmans cars plate number which turnedout to be registered in Gos wifes name. That information did not, however, constitute personalknowledge. It is thus clear to the Court that there was no lawful warrantless arrest of Go withinthe meaning of Section 5 of Rule 113. It is clear too that Section 7 of Rule 112 is also notapplicable. Indeed, Go was not arrested at all. When he walked into the San Juan Police Station,accompanied by two (2) lawyers, he in fact placed himself at the disposal of the policeauthorities. He did not state that he was surrendering himself, in all probability to avoid theimplication he was admitting that he had slain Eldon Maguan or that he was otherwise guilty of acrime. When the police filed a complaint for frustrated homicide with the Prosecutor, the latter should have immediately scheduled a preliminary investigation to determine whether there was probable cause for charging Go in court for the killing of Eldon Maguan. Instead, as notedearlier, the Prosecutor proceeded under the erroneous supposition that Section 7 of Rule 112 wasapplicable and required Go to waive the provisions of Article 125 of the Revised Penal Code as acondition for carrying out a preliminary investigation. This was substantive error, for Go wasentitled to a preliminary investigation and that right should have been accorded him without anyconditions. Moreover, since Go had not been arrested; with or without a warrant, he was alsoentitled to be released forthwith subject only to his appearing at the preliminary investigation.

PEOPLE VS. MARTI [193 SCRA 57; G.R. NO. 81561; 18 JAN 1991] Tuesday, February 03, 2009 Posted by Coffeeholic Writes Labels: Case Digests, Political Law

Facts:

Accused-appellant went to a forwarding agency

to send four packages to a friend in Zurich. Initially, the accused was asked by the proprietress if the packages can

be examined. However, he refused. Before delivering said packages to the Bureau of Customs and the Bureau of Posts, the husband of the proprietress opened said boxes for final inspection. From that inspection, included in the standard operating procedure and out of curiosity, he took several grams of its contents. He brought a letter and the said sample to the National Bureau of Investigation. When the NBI was informed that the rest of the shipment was still in his office, three agents went back with him. In their presence, the husband totally opened the packages. Afterwards, the NBI took custody of said packages. The contents , after examination by forensic chemists, were found to be marijuana flowering tops. The appellant, while claiming his mail at the Central Post Office, was invited by the agents for questioning. Later on, the trial court found him guilty of violation of the Dangerous Drugs Act.

Issues:
(1) Whether or Not the items admitted in the searched illegally searched and seized. (2) Whether or Not custodial investigation properly applied. (3) Whether or Not the trial court not give credence to the explanation of the appellant on how said packages came to his possession.

Held:

No. The case at bar assumes a peculiar character

since the evidence sought to be excluded was primarily discovered and obtained by a private person, acting in a private capacity and without the intervention and participation of State authorities. Under the circumstances, can accused/appellant validly claim that his constitutional right against unreasonable searches and seizure has been violated. Stated otherwise, may an act of a private individual, allegedly in violation of appellant's constitutional rights, be invoked against the State. In the absence of governmental interference, the liberties guaranteed by the Constitution cannot be invoked against the State. It was Mr. Job Reyes, the proprietor of the forwarding agency, who made search/inspection of the packages. Said inspection was reasonable and a standard operating procedure on the part of Mr. Reyes as a precautionary measure before delivery of packages to the Bureau of Customs or the Bureau of Posts. Second, the mere presence of the NBI agents did not convert the reasonable search effected by Reyes into a warrantless search and seizure proscribed by the Constitution. Merely to observe and look at that which is in plain sight is not a search. Having observed that which is open, where no trespass has been committed in aid thereof, is not search. No. The law enforcers testified that accused/appellant was informed of his constitutional rights. It is presumed that they have regularly performed their duties (See. 5(m),

Rule 131) and their testimonies should be given full faith and credence, there being no evidence to the contrary. No. Appellant signed the contract as the owner and shipper thereof giving more weight to the presumption that things which a person possesses, or exercises acts of ownership over, are owned by him (Sec. 5 [j], Rule 131). At this point, appellant is therefore estopped to claim otherwise.
PEOPLE OF THE PHILIPPINES, plaintiff-appellee vs A NDRE M A RTI, accused-appellant . G . R . No . 81561 January 18, 1991 F AC TS: Accused-appellant went to a forwarding agency to send four packages to a friend inZurich. Initially, the accused was asked by the proprietress if the packages can beexamined. However, he refused. Before delivering said packages to the Bureau of Customs and the Bureau of Posts, the husband of the proprietress opened said boxes for final inspection. From that inspection, included in the standard operating procedure andout of curiosity, he took several grams of its contents.He brought a letter and the said sample to the National Bureau of Investigation. When the NBI was informed that the rest of the shipment was still in his office, three agents went back with him. In their presence, the husband totally opened the packages. Afterwards,the NBI took custody of said packages. The contents, after examination by forensicchemists, were found to be marijuana flowering tops.The appellant, while claiming his mail at the Central Post Office, was invited by theagents for questioning. Later on, the trial court found him guilty of violation of theDangerous Drugs Act. ISSUES: 1. Whether or not the items admitted in the searched illegally searched and seized.2. Whether or not custodial investigation was not properly applied.3.

Whether or not the trial court did not give credence to the explanation of the appellanton how said packages came to his possession. HELD: 1. No. The case at bar assumes a peculiar character since the evidence sought to beexcluded was primarily discovered and obtained by a private person, acting in a private capacity and without the intervention and participation of State authorities.Under the circumstances, can accused/appellant validly claim that his constitutionalright against unreasonable searches and seizure has been violated. Stated otherwise,may an act of a private individual, allegedly in violation of appellant's constitutionalrights, be invoked against the State. In the absence of governmental interference, theliberties guaranteed by the Constitution cannot be invoked against the State. It wasMr. Job Reyes, the proprietor of the forwarding agency, who made search/inspectionof the packages. Said inspection was reasonable and a standard operating procedureon the part of Mr. Reyes as a precautionary measure before delivery of packages tothe Bureau of Customs or the Bureau of Posts. Second, the mere presence of the NBIagents did not convert the reasonable search effected by Reyes into a warrantlesssearch and seizure proscribed by the Constitution. Merely to observe and look at thatwhich is in plain sight is not a search. Having observed that which is open, where notrespass has been committed in aid thereof, is not search.2. No. The law enforcers testified that accused/appellant was informed of hisconstitutional rights. It is presumed that they have regularly performed their duties(See. 5(m), Rule 131) and their testimonies should be given full faith and credence,there being no evidence to the contrary.3. No. Appellant signed the contract as the owner and shipper thereof giving moreweight to the presumption that things which a person possesses, or exercises acts of ownership over, are owned by him (Sec. 5 [j], Rule 131). At this point, appellant istherefore estopped to claim otherwise.

Horacio Morales Jr vs Minister of Defense Juan Ponce Enrile et al


17 11 2010

Habeas Corpus The Right to Bail In April 1982, Morales and some others were arrested while driving a motor vehicle in LaongLaan St, QC. They were charged in CFI Rizal for rebellion punishable under the RPC. Morales alleged that they were arrested without any warrant of arrest; that their constitutional rights were violated, among them the right to counsel, the right to remain silent, the right to a speedy and public trial, and the right to bail. Respondents countered that the group of Morales were already under surveillance for some time before they were arrested and that the warrantless arrest done is valid and at the same time the privilege of the writ of habeas corpus was already suspended. ISSUE: Whether or not Morales et al can post bail. HELD: Normally, rebellion being a non-capital offense is bailable. But because the privilege of the writ of habeas corpus remains suspended with respect to persons at present detained as well as other who may hereafter be similarly detained for the crimes of insurrection or rebellion,

subversion, conspiracy or proposal to commit such crimes, and for all other crimes and offenses committed by them in furtherance of or on the occasion thereof, or incident thereto, or in connection therewith, the natural consequence is that the right to bail for the commission of anyone of the said offenses is also suspended. To hold otherwise would defeat the very purpose of the suspension. Therefore, where the offense for which the detainee was arrested is anyone of the said offenses he has no right to bail even after the charges are filed in court. The crimes of rebellion, subversion, conspiracy or proposal to commit such crimes, and crimes or offenses committed in furtherance thereof or in connection therewith constitute direct attacks on the life of the State. Just as an individual has right to self-defense when his life is endangered, so does the State. The suspension of the privilege of the writ is to enable the State to hold in preventive imprisonment pending investigation and trial those persons who plot against it and commit acts that endanger the States very existence. For this measure of self-defense to be effective, the right to bail must also be deemed suspended with respect to these offenses. However, there is a difference between preventive and punitive imprisonment. Where the filing of charges in court or the trial of such charges already filed becomes protracted without any justifiable reason, the detention becomes punitive in character and the detainee regains his right to freedom. Quite notable in this case however is that the 2nd division of the SC reiterated the Lansang Doctrine as opposed to what they ruled in the Garcia-Padilla Case. PEOPLE OF THE PHILIPPINES, Plaintiff-Appellee, vs.RUBEN BURGOS y
TITO, Defendant-Appellant G . R . No.L-68955 September 4, 1986 FACTS: Cesar Masamlok surrendered to the authorities at the Davao del Sur Constabulary Head Quarters.He testified that Ruben Burgos forcibly recruited him as member of NPA. Burgos threatened himwith the use of a firearm. Masamlok attended the seminar where Burgos spoke about hismembership with the NPA and the organizations desire to overthrow the government.Pursuant to this information, PC-INP members went to house of accused. Accused was plowingthe field when they arrived. Pat. Bioco called the accused and asked him about the firearm.Accused denied possession of said firearm but later, his wife pointed to a place below their housewhere a gun was buried in the ground. After the recovery of said firearm, accused pointed to astock pile of cogon where the officers recovered:1.maroon notebook 2. pamphlets: Ang Bayan, Pahayagan ng Paritdo Komunista ng Pilipinas etc.Accused admitted that firearm was issued to him by Nestor Jimines, team leader of sparrow unit. ISSUE: 1. Whether or not the warrantless arrest of Ruben Burgos was lawful?2. Whether or not the search of his house and the subsequent confiscation of a firearm anddocuments allegedly found therein conducted in a lawful and valid manner?3. Whether or not the evidence sustaining the crime charged meet the test of proving guilt beyond reasonable doubt? HELD: 1. No. Under Section 6(a) of Rule 113, the officer arresting a person who has just committed, iscommitting, or is about to commit an offense must have personal knowledge of that fact. Theoffense must also be committed in his presence or within his view. In this case, the accusedwas arrested on the sole basis of Masamlok's verbal report. Masamlok led the authorities tosuspect that the accused had committed a crime.2.

No. If an arrest without a warrant is unlawful at the moment it is made, generally nothing thathappened or is discovered afterwards can make it lawful. Neither can it be presumed thatthere was a waiver, or that consent was given by the accused to be searched simply becausehe failed to object.3. No. The Constitution itself mandates that any evidence obtained in violation of right isinadmissible in evidence. With the extra-judicial confession, the firearm, and the allegedsubversive documents inadmissible in evidence against the accused-appellant, the onlyremaining proof to sustain the charge of Illegal Possession of Firearm in Furtherance of Subversion is the testimony of Cesar Masamlok. In the instant case, Masamlok's testimonywas totally uncorroborated. Considering that Masamlok surrendered to the military certainlyhis fate depended on how eagerly he cooperated with the authorities

G.R. Nos. 95122-23 May 31, 1991 BOARD OF COMMISSIONERS (COMMISSION ON IMMIGRATION AND DEPORTATION), BOARD OF SPECIAL INQUIRY, COMMISSIONER ANDREA D. DOMINGO, ASSOCIATE COMMISSIONER JORGE V. SARMIENTO, ACTING ASSOCIATE COMMISSIONER REGINO R. SANTIAGO, MEMBERS OF THE BOARD OF SPECIAL INQUIRY, ESTANISLAO CANTA, LEO MAGAHOM and BENJAMIN KALAW, petitioners, vs. HON. JOSELITO DELA ROSA, Presiding Judge, RTC Manila, Branch 29, WILLIAM T. GATCHALIAN, respondents. BOARD OF COMMISSIONERS (COMMISSION ON IMMIGRATION AND DEPORTATION), BOARD OF SPECIAL INQUIRY, COMMISSIONER ANDREA D. DOMINGO, ASSOCIATE COMMISSIONER JORGE V. SARMIENTO, ACTING ASSOCIATE COMMISSIONER REGINO R. SANTIAGO, MEMBERS OF THE BOARD OF SPECIAL INQUIRY, ESTANISLAO CANTA, LEO MAGAHOM and BENJAMIN KALAW, petitioners, vs. HON. TERESITA DIZON CAPULONG, Presiding Judge, RTC Branch 172, Valenzuela, Metro Manila, DEE HUA T. GATCHALIAN, SHERWING T. GATCHALIAN, KENNETH T. GATCHALIAN, REXLON T. GATCHALIAN, and WESLIE T. GATCHALIAN, respondents. G.R. Nos. 95612-13 May 31, 1991 WILLIAM T. GATCHALIAN, petitioner, vs. BOARD OF COMMISSIONERS (COMMISSION ON IMMIGRATION AND DEPORTATION), et al., respondents. The Solicitor General for petitioners. edesma, Saludo & Associates for respondent William Gatchalian. Cervo and Tanay Law Office for respondent T.D. Capulong, D.H.T. Gatchalian, et al.

BIDIN, J.:p This is a petition for certiorari and prohibition filed by the Solicitor General seeking 1) to set aside the Resolution/Temporary Restraining Order dated September 7, 1990, issued by respondent Judge de la Rosa in Civil Case No. 90-54214 which denied petitioners' motion to dismiss and restrained petitioners from commencing or continuing with any of the proceedings which would lead to the deportation of respondent William Gatchalian, docketed as D.C. No. 90-523, as well as the Order of respondent Judge Capulong dated September 6, 1990 in Civil Case No. 3431-V-90 which likewise enjoined petitioners from proceeding with the deportation charges against respondent Gatchalian, and 2) to prohibit respondent judges from further acting in the aforesaid civil cases. On October 23, 1990, respondent Gatchalian filed his Comment with Counter-Petition, docketed as G.R. Nos. 96512-13, alleging lack of jurisdiction on the part of respondent Board of Commissioners, et al., over his person with prayer that he be declared a Filipino citizen, or in the alternative, to remand the case to the trial court for further proceedings. On December 13, 1990, petitioners filed their comment to respondent Gatchalian's counter-petition. The Court considers the comment filed by respondent Gatchalian as answer to the petition and petitioners' comment as answer to the counter-petition and gives due course to the petitions. There is no dispute as to the following facts: On July 12, 1960, Santiago Gatchalian, grandfather of William Gatchalian, was recognized by the Bureau of Immigration as a native born Filipino citizen following the citizenship of his natural mother, Marciana Gatchalian (Annex "1", counter-petition). Before the Citizenship Evaluation Board, Santiago Gatchalian testified that he has five (5) children with his wife Chu Gim Tee, namely: Jose Gatchalian, Gloria Gatchalian, Francisco Gatchalian, Elena Gatchalian and Benjamin Gatchalian (Annex "2", counterpetition). On June 27, 1961, William Gatchalian, then a twelve-year old minor, arrived in Manila from Hongkong together with Gloria, Francisco, and Johnson, all surnamed Gatchalian. They had with them Certificates of Registration and Identity issued by the Philippine Consulate in Hongkong based on a cablegram bearing the signature of the then Secretary of Foreign Affairs, Felixberto Serrano, and sought admission as Filipino citizens. Gloria and Francisco are the daughter and son, respectively, of Santiago Gatchalian; while William and Johnson are the sons of Francisco. After investigation, the Board of Special Inquiry No. 1 rendered a decision dated July 6, 1961, admitting William Gatchalian and his companions as Filipino citizens (Annex "C",

petition). As a consequence thereof, William Gatchalian was issued Identification Certificate No. 16135 by the immigration authorities on August 16, 1961 (Annex "D", petition). On January 24, 1962, the then Secretary of Justice issued Memorandum No. 9 setting aside all decisions purporting to have been rendered by the Board of Commissioners on appeal or on review motu proprio of decisions of the Board of Special Inquiry. The same memorandum directed the Board of Commissioners to review all cases where entry was allowed on the ground that the entrant was a Philippine citizen. Among those cases was that of William and others. On July 6, 1962, the new Board of Commissioners, after a review motu proprio of the proceedings had in the Board of Special Inquiry, reversed the decision of the latter and ordered the exclusion of, among others, respondent Gatchalian (Annex "E", petition). A warrant of exclusion also dated July 6, 1962 was issued alleging that "the decision of the Board of Commissioners dated July 6, 1962 . . . has now become final and executory (Annex "F", petition). The actual date of rendition of said decision by the Board of Commissioners (whether on July 6, 1962 or July 20, 1962) became the subject of controversy in the 1967 case of Arocha vs. Vivo (21 SCRA 532) wherein this Court sustained the validity of the decision of the new Board of Commissioners having been promulgated on July 6, 1962, or within the reglementary period for review. Sometime in 1973, respondent Gatchalian, as well as the others covered by the July 6, 1962 warrant of exclusion, filed a motion for re-hearing with the Board of Special Inquiry where the deportion case against them was assigned. On March 14, 1973, the Board of Special Inquiry recommended to the then Acting Commissioner Victor Nituda the reversal of the July 6, 1962 decision of the then Board of Commissioners and the recall of the warrants of arrest issued therein (Annex "5", counter-petition). On March 15, 1973, Acting Commissioner Nituda issued an order reaffirming the July 6, 1961 decision of the Board of Special Inquiry thereby admitting respondent Gatchalian as a Filipino citizen and recalled the warrant of arrest issued against him (Annex "6", counter-petition). On June 7, 1990, the acting director of the National Bureau of Investigation wrote the Secretary of Justice recommending that respondent Gatchalian along with the other applicants covered by the warrant of exclusion dated July 6, 1962 be charged with violation of Sec. 37 (a), pars. 1 and 2, in relation to Secs. 45 (c), and (d) and (e) of Commonwealth Act No. 613, as amended, also known as the Immigration Act of 1940 (Annex "G", petition).

On August 1, 1990, the Secretary of Justice indorsed the recommendation of the NBI to the Commissioner of Immigration for investigation and immediate action (Annex "20", counter-petition). On August 15, 1990, petitioner Commissioner Domingo of the Commission of Immigration and Deportation * issued a mission order commanding the arrest of respondent William Gatchalian (Annex "18", counter-petition). The latter appeared before Commissioner Domingo on August 20, 1990 and was released on the same day upon posting P200,000.00 cash bond. On August 29, 1990, William Gatchalian filed a petition for certiorari and prohibition with injunction before the Regional Trial Court of Manila, Br. 29, presided by respondent Judge dela Rosa, docketed as Civil Case No. 90-54214. On September 4, 1990, petitioners filed a motion to dismiss Civil Case No. 90-54214 alleging that respondent judge has no jurisdiction over the Board of Commissioners and/or the Board of Special Inquiry. Nonetheless, respondent judge dela Rosa issued the assailed order dated September 7, 1990, denying the motion to dismiss. Meanwhile, on September 6, 1990, respondent Gatchalian's wife and minor children filed before the Regional Trial Court of Valenzuela, Metro Manila, Br. 172, presided by respondent judge Capulong Civil Case No. 3431-V-90 for injunction with writ of preliminary injunction. The complaint alleged, among others, that petitioners acted without or in excess of jurisdiction in the institution of deportation proceedings against William. On the same day, respondent Capulong issued the questioned temporary restraining order restraining petitioners from continuing with the deportation proceedings against William Gatchalian. The petition is anchored on the following propositions: 1) respondent judges have no jurisdiction over petitioners (Board of Commissioners, et al.,) and the subject matter of the case, appellate jurisdiction being vested by BP 129 with the Court of Appeals; 2) assuming respondent judges have jurisdiction, they acted with grave abuse of discretion in preempting petitioners in the exercise of the authority and jurisdiction to hear and determine the deportation case against respondent Gatchalian, and in the process determine also his citizenship; 3) respondent judge dela Rosa gravely abused his discretion in ruling that the issues raised in the deportation proceedings are beyond the competence and jurisdiction of petitioners, thereby disregarding the cases of Arocha vs. Vivo and Vivo vs. Arca (supra), which put finality to the July 6, 1962 decision of the Board of Commissioners that respondent Gatchalian is a Chinese citizen; and 4) respondent judge Capulong should have dismissed Civil Case No. 3431-V-90 for forumshopping. In his counter-petition, William Gatchalian alleges among others that: 1) assuming that the evidence on record is not sufficient to declare him a Filipino citizen, petitioners have no jurisdiction to proceed with the deportation case until the courts shall have finally resolved the question of his citizenship; 2) petitioners can no longer judiciously and

fairly resolve the question of respondent's citizenship in the deportation case because of their bias, pre-judgment and prejudice against him; and 3) the ground for which he is sought to be deported has already prescribed. For purposes of uniformity, the parties herein will be referred to in the order the petitions were filed. Petitioners argue that under Sec. 9 (3) of BP 129, it is the Court of Appeals which has exclusive appellate jurisdiction over all final judgments or orders of quasi-judicial agencies, boards or commissions, such as the Board of Commissioners and the Board of Special Inquiry. Respondent, on the other hand, contends that petitioners are not quasi-judicial agencies and are not in equal rank with Regional Trial Courts. Under Sec. 21 (1) of Batas Pambansa Blg. 129, the Regional Trial Courts have concurrent jurisdiction with this Court and the Court of Appeals to issue "writs of certiorari, prohibition, mandamus, quo warranto, habeas corpus and injunction which may be enforced in any part of their respective regions, . . ." Thus, the RTCs are vested with the power to determine whether or not there has been a grave abuse of discretion on the part of any branch or instrumentality of the government. It is true that under Sec. 9 (3) of Batas Pambansa Blg. 129, the Court of Appeals is vested with
(3) Exclusive appellate jurisdiction over all final judgments, decisions, resolutions, order, or awards of Regional Trial Courts and quasi-judicial agencies, instrumentalities, board or commission, except those falling within the appellate jurisdiction of the Supreme Court in accordance with the Constitution, the provisions of this Act, and of sub-paragraph (1) of the third paragraph of and sub-paragraph (4) of the fourth paragraph of Section 17 of the Judiciary Act of 1948.

It does not provide, however, that said exclusive appellate jurisdiction of the Court of Appeals extends to all quasi-judicial agencies. The quasi-judicial bodies whose decisions are exclusively appealable to the Court of Appeals are those which under the law, Republic Act No. 5434, or their enabling acts, are specifically appealable to the Court of Appeals (Presidential Anti-Dollar Salting Task Force vs. Court of Appeals, 171 SCRA 348 [1989]; Lupangco vs. Court of Appeals, 160 SCRA 848 [1988]). Thus, under Republic Act No. 5434, it is specifically provided that the decisions of the Land Registration Commission (LRC), the Social Security Commission (SSC), Civil Aeronautics Board (CAB), the Patent Office and the Agricultural Invention Board are appealable to the Court of Appeals. In the Presidential Anti-Dollar Salting Task Force (supra), this Court clarified the matter when We ruled:
Under our Resolution dated January 11, 1983:

. . . The appeals to the Intermediate Appellate Court (now Court of Appeals) from quasi-judicial bodies shall continue to be governed by the provisions of Republic Act No. 5434 insofar as the same is not inconsistent with the provisions of B.P. Blg. 129. The pertinent provisions of Republic Act No. 5434 are as follows: Sec. 1. Appeals from specified agencies. Any provision of existing law or Rules of Court to the contrary notwithstanding, parties aggrieved by a final ruling, award, order, or decision, or judgment of the Court of Agrarian Relations; the Secretary of Labor under Section 7 of Republic Act Numbered Six hundred and two, also known as the "Minimum Wage Law"; the Department of Labor under Section 23 of Republic Act Numbered Eight hundred seventy-five, also known as the "Industrial Peace Act"; the Land Registration Commission; the Social Security Commission; the Civil Aeronautics Board; the Patent Office and the Agricultural Inventions Board, may appeal therefrom to the Court of Appeals, within the period and in the manner herein provided, whether the appeal involves questions of fact, mixed questions of fact and law, or questions of law, or all three kinds of questions. From final judgments or decisions of the Court of Appeals, the aggrieved party may appeal by certiorari to the Supreme Court as provided under Rule 45 of the Rules of Court. Because of subsequent amendments, including the abolition of various special courts, jurisdiction over quasi-judicial bodies has to be, consequently, determined by the corresponding amendatory statutes. Under the Labor Code, decisions and awards of the National Labor Relations Commission are final and executory, but, nevertheless, reviewable by this Court through a petition for certiorari and not by way of appeal. Under the Property Registration Decree, decision of the Commission of Land Registration, en consulta, are appealable to the Court of Appeals. The decisions of the Securities and Exchange Commission are likewise appealable to the Appellate Court, and so are decisions of the Social Security Commission. As a rule, where legislation provides for an appeal from decisions of certain administrative bodies to the Court of Appeals, it means that such bodies are co-equal with the Regional Trial Courts, in terms of rank and stature, and logically, beyond the control of the latter. (Emphasis supplied)

There are quasi-judicial agencies, as the National Labor Relations Commissions, whose decisions are directly appealable to this Court. It is only when a specific law, as Republic Act No. 5434, provides appeal from certain bodies or commissions to the Court of Appeals as the Land Registration Commission (LRC), Securities and Exchange Commission (SEC) and others, that the said commissions or boards may be considered co-equal with the RTCs in terms of rank, stature and are logically beyond the control of the latter. However, the Bureau of Immigration (or CID) is not among those quasi-judicial agencies specified by law whose decisions, orders, and resolutions are directly appealable to the Court of Appeals. In fact, its decisions are subject to judicial review in accordance with

Sec. 25, Chapter 4, Book VII of the 1987 Administrative Code, which provides as follows:
Sec. 25. Judicial Review.(1) Agency decisions shall be subject to judicial review in accordance with this chapter and applicable laws. xxx xxx xxx (6) The review proceeding shall be filed in the court specified in the statute or, in the absence thereof, in any court of competent jurisdiction in accordance with the provisions on venue of the Rules of Court.

Said provision of the Administrative Code, which is subsequent to B.P. Blg. 129 and which thus modifies the latter, provides that the decision of an agency like the Bureau of Immigration should be subject to review by the court specified by the statute or in the absence thereof, it is subject to review by any court of competent jurisdiction in accordance with the provisions on venue of the Rules of Court. B.P. Blg. 129 did not intend to raise all quasi-judicial bodies to the same level or rank of the RTC except those specifically provided for under the law as aforestated. As the Bureau of Immigration is not of equal rank as the RTC, its decisions may be appealable to, and may be reviewed through a special civil action for certiorari by, the RTC (Sec. 21, (1) BP 129). True, it is beyond cavil that the Bureau of Immigration has the exclusive authority and jurisdiction to try and hear cases against an alleged alien, and in the process, determine also their citizenship (Lao Gi vs. Court of Appeals, 180 SCRA 756 [1989]). And a mere claim of citizenship cannot operate to divest the Board of Commissioners of its jurisdiction in deportation proceedings (Miranda vs. Deportation Board, 94 Phil. 531 [1954]). However, the rule enunciated in the above-cases admits of an exception, at least insofar as deportation proceedings are concerned. Thus, what if the claim to citizenship of the alleged deportee is satisfactory? Should the deportation proceedings be allowed to continue or should the question of citizenship be ventilated in a judicial proceeding? In Chua Hiong vs. Deportation Board (96 Phil. 665 [1955]), this Court answered the question in the affirmative, and We quote:
When the evidence submitted by a respondent is conclusive of his citizenship, the right to immediate review should also be recognized and the courts should promptly enjoin the deportation proceedings. A citizen is entitled to live in peace, without molestation from any official or authority, and if he is disturbed by a deportation proceeding, he has the unquestionable right to resort to the courts for his protection, either by a writ of habeas corpus or of prohibition, on the legal ground that the Board lacks jurisdiction. If he is a citizen and evidence thereof is satisfactory, there is no sense nor justice in allowing the deportation proceedings to continue, granting him the remedy only after the Board has finished its investigation of his undesirability.

. . . And if the right (to peace) is precious and valuable at all, it must also be protected on time, to prevent undue harassment at the hands of ill-meaning or misinformed administrative officials. Of what use is this much boasted right to peace and liberty if it can be availed of only after the Deportation Board has unjustly trampled upon it, besmirching the citizen's name before the bar of public opinion? (Emphasis supplied)

The doctrine of primary jurisdiction of petitioners Board of Commissioners over deportation proceedings is, therefore, not without exception (Calacday vs. Vivo, 33 SCRA 413 [1970]; Vivo vs. Montesa, 24 SCRA 155 [1967]). Judicial intervention, however, should be granted only in cases where the "claim of citizenship is so substantial that there are reasonable grounds to believe that the claim is correct. In other words, the remedy should be allowed only on sound discretion of a competent court in a proper proceeding (Chua Hiong vs. Deportation Board, supra; Co. vs. Deportation Board, 78 SCRA 107 [1977]). It appearing from the records that respondent's claim of citizenship is substantial, as We shall show later, judicial intervention should be allowed. In the case at bar, the competent court which could properly take cognizance of the proceedings instituted by respondent Gatchalian would nonetheless be the Regional Trial Court and not the Court of Appeals in view of Sec. 21 (1), BP 129, which confers upon the former jurisdiction over actions for prohibition concurrently with the Court of Appeals and the Supreme Court and in line with the pronouncements of this Court in Chua Hiong and Co cases. Ordinarily, the case would then be remanded to the Regional Trial Court. But not in the case at bar. Considering the voluminous pleadings submitted by the parties and the evidence presented, We deem it proper to decide the controversy right at this instance. And this course of action is not without precedent for "it is a cherished rule of procedure for this Court to always strive to settle the entire controversy in a single proceeding leaving no root or branch to bear the seeds of future litigation. No useful purpose will be served if this case is remanded to the trial court only to have its decision raised again to the Court of Appeals and from there to this Court" (Marquez vs. Marquez, 73 Phil. 74; Keramic Industries, Inc. vs. Guerrero, 61 SCRA 265 [1974]) Alger Electric, Inc. vs. Court of Appeals (135 SCRA 37 [1985]), citing Gayos vs. Gayos (67 SCRA 146 [1975]). In Lianga Bay Logging Co., Inc. vs. Court of Appeals (157 SCRA 357 [1988]), We also stated:
Remand of the case to the lower court for further reception of evidence is not necessary where the court is in a position to resolve the dispute based on the records before it. On many occasions, the Court, in the public interest and the expeditious administration of justice, has resolved actions on the merits instead of remanding them to the trial court for further proceedings, such as where the ends of justice would not be subserved by the remand of the case or when public interest demands an early disposition of the case or where the trial court had already received all the evidence of the parties (Quisumbing vs. CA, 112 SCRA 703; Francisco, et al., vs. The City of Davao, et al., supra; Republic vs. Security Credit & Acceptance Corp., et al., 19 SCRA 58; Samal vs. CA, supra; Republic vs. Central Surety & Insurance Co., 25 SCRA 641).

Likewise in Tejones vs. Gironella (159 SCRA 100 [1988]), We said:


Sound practice seeks to accommodate the theory which avoids waste of time, effort and expense, both to the parties and the government, not to speak of delay in the disposal of the case (cf. Fernandez vs. Garcia, 92 Phil. 592, 297). A marked characterstic of our judicial set-up is that where the dictates of justice so demand . . . the Supreme Court should act, and act with finality (Li Siu Liat vs. Republic, 21 SCRA 1039, 1046, citing Samal vs. CA, 99 Phil. 230 and US vs. Gimenez, 34 Phil. 74.) (Beautifont, Inc. vs. Court of appeals, et al., Jan. 29, 1988; See also Labo vs. Commission on Elections, 176 SCRA 1 [1989]).

Respondent Gatchalian has adduced evidence not only before the Regional Trial Court but also before Us in the form of public documents attached to his pleadings. On the other hand, Special Prosecutor Renato Mabolo in his Manifestation (dated September 6, 1990; Rollo, p. 298, counter-petition) before the Bureau of Immigration already stated that there is no longer a need to adduce evidence in support of the deportation charges against respondent. In addition, petitioners invoke that this Court's decision in Arocha vs. Vivo and Vivo vs. Arca (supra), has already settled respondent's alienage. Hence, the need for a judicial determination of respondent's citizenship specially so where the latter is not seeking admission, but is already in the Philippines (for the past thirty [30] years) and is being expelled (Chua Hiong vs. Deportation Board, supra). According to petitioners, respondent's alienage has been conclusively settled by this Court in the Arocha and Vivo cases, We disagree. It must be noted that in said cases, the sole issue resolved therein was the actual date of rendition of the July 6, 1962 decision of the then board of Commissioners, i.e., whether the decision was rendered on July 6, 1962 or on July 20, 1962 it appearing that the figure (date) "20" was erased and over it was superimposed the figure "6" thereby making the decision fall within the one-year reglementary period from July 6, 1961 within which the decision may be reviewed. This Court did not squarely pass upon any question of citizenship, much less that of respondent's who was not a party in the aforesaid cases. The said cases originated from a petition for a writ of habeas corpus filed on July 21, 1965 by Macario Arocha in behalf of Pedro Gatchalian. Well settled is the rule that a person not party to a case cannot be bound by a decision rendered therein. Neither can it be argued that the Board of Commissioners' decision (dated July 6, 1962) finding respondent's claim to Philippine citizenship not satisfactorily proved, constitute res judicata. For one thing, said decision did not make any categorical statement that respondent Gatchalian is a Chinese. Secondly, the doctrine of res judicata does not apply to questions of citizenship (Labo vs. Commission on Elections (supra); citing Soria vs. Commissioner of Immigration, 37 SCRA 213; Lee vs. Commissioner of Immigration, 42 SCRA 561 [1971]; Sia Reyes vs. Deportation Board, 122 SCRA 478 [1983]). In Moy Ya Lim vs. Commissioner of Immigration (41 SCRA 292 [1971]) and in Lee vs. Commissioner of Immigration (supra), this Court declared that:
(e)verytime the citizenship of a person is material or indispensable in a judicial or administrative case, whatever the corresponding court or administrative authority decides

therein as to such citizenship is generally not considered as res adjudicata, hence it has to be threshed out again and again as the occasion may demand.

An exception to the above rule was laid by this Court in Burca vs. Republic (51 SCRA 248 [1973]), viz:
We declare it to be a sound rule that where the citizenship of a party in a case is definitely resolved by a court or by an administrative agency, as a material issue in the controversy, after a full-blown hearing with the active participation of the Solicitor General or his authorized representative, and this finding or the citizenship of the party is affirmed by this Court, the decision on the matter shall constitute conclusive proof of such party's citizenship in any other case or proceeding. But it is made clear that in no instance will a decision on the question of citizenship in such cases be considered conclusive or binding in any other case or proceeding, unless obtained in accordance with the procedure herein stated.

Thus, in order that the doctrine of res judicata may be applied in cases of citizenship, the following must be present: 1) a person's citizenship must be raised as a material issue in a controversy where said person is a party; 2) the Solicitor General or his authorized representative took active part in the resolution thereof, and 3) the finding or citizenship is affirmed by this Court. Gauged by the foregoing, We find the pre-conditions set forth in Burca inexistent in the Arocha and Vivo cases relied upon by petitioners. Indeed, respondent William Gatchalian was not even a party in said cases. Coming now to the contention of petitioners that the arrest of respondent follows as a matter of consequence based on the warrant of exclusion issued on July 6, 1962, coupled with the Arocha and Vivo cases (Rollo, pp. 33), the Court finds the same devoid of merit. Sec. 37 (a) of Commonwealth Act No. 613, as amended, otherwise known as the Immigration Act of 1940, reads:
Sec. 37. (a) The following aliens shall be arrested upon the warrant of the Commissioner of Immigration or of any other officer designated by him for the purpose and deported upon the warrant of the Commissioner of Immigration after a determination by the Board of Commissioner of the existence of the ground for deportation as charged against the alien. (Emphasis supplied)

From a perusal of the above provision, it is clear that in matters of implementing the Immigration Act insofar as deportation of aliens are concerned, the Commissioner of Immigration may issue warrants of arrest only after a determination by the Board of Commissioners of the existence of the ground for deportation as charged against the alien. In other words, a warrant of arrest issued by the Commissioner of Immigration, to be valid, must be for the sole purpose of executing a final order of deportation. A warrant of arrest issued by the Commissioner of Immigration for purposes of investigation only, as in the case at bar, is null and void for being unconstitutional (Ang Ngo Chiong vs. Galang, 67 SCRA 338 [1975] citing Po Siok Pin vs. Vivo, 62 SCRA 363

[1975]; Vivo vs. Montesa, 24 SCRA 155; Morano vs. Vivo, 20 SCRA 562; Qua Chee Gan vs. Deportation Board, 9 SCRA 27 [1963]; Ng Hua To vs. Galang, 10 SCRA 411; see also Santos vs. Commissioner of Immigration, 74 SCRA 96 [1976]). As We held in Qua Chee Gan vs. Deportation Board (supra), "(t)he constitution does not distinguish warrants between a criminal case and administrative proceedings. And if one suspected of having committed a crime is entitled to a determination of the probable cause against him, by a judge, why should one suspected of a violation of an administrative nature deserve less guarantee?" It is not indispensable that the alleged alien be arrested for purposes of investigation. If the purpose of the issuance of the warrant of arrest is to determine the existence of probable cause, surely, it cannot pass the test of constitutionality for only judges can issue the same (Sec. 2, Art. III, Constitution). A reading of the mission order/warrant of arrest (dated August 15, 1990; Rollo, p. 183, counter-petition) issued by the Commissioner of Immigration, clearly indicates that the same was issued only for purposes of investigation of the suspects, William Gatchalian included. Paragraphs 1 and 3 of the mission order directs the Intelligence Agents/Officers to:
xxx xxx xxx 1. Make a warrantless arrest under the Rules of Criminal Procedure, Rule 113, Sec. 5, for violation of the Immigration Act, Sec. 37, para. a; Secs. 45 and 46 Administrative Code; xxx xxx xxx 3. Deliver the suspect to the Intelligence Division and immediately conduct custodial interrogation, after warning the suspect that he has a right to remain silent and a right to counsel; . . .

Hence, petitioners' argument that the arrest of respondent was based, ostensibly, on the July 6, 1962 warrant of exclusion has obviously no leg to stand on. The mission order/warrant of arrest made no mention that the same was issued pursuant to a final order of deportation or warrant of exclusion. But there is one more thing that militates against petitioners' cause. As records indicate, which petitioners conveniently omitted to state either in their petition or comment to the counter-petition of respondent, respondent Gatchalian, along with others previously covered by the 1962 warrant of exclusion, filed a motion for re-hearing before the Board of Special Inquiry (BSI) sometime in 1973. On March 14, 1973, the Board of Special Inquiry, after giving due course to the motion for re-hearing, submitted a memorandum to the then Acting Commissioner Victor Nituda (Annex "5", counter-petition) recommending 1 the reconsideration of the July 6, 1962 decision of the then Board of Commissioners which reversed the July 6, 1961 decision of the then Board of Special Inquiry No. 1 and 2 the lifting of the warrants of arrest

issued against applicants. The memorandum inferred that the "very basis of the Board of Commissioners in reversing the decision of the Board of Special Inquiry was due to a forged cablegram by the then Secretary of Foreign Affairs, . . ., which was dispatched to the Philippine Consulate in Hong Kong authorizing the registration of applicants as P.I. citizens." The Board of Special Inquiry concluded that "(i)f at all, the cablegram only led to the issuance of their Certificate(s) of Identity which took the place of a passport for their authorized travel to the Philippines. It being so, even if the applicants could have entered illegally, the mere fact that they are citizens of the Philippines entitles them to remain in the country." On March 15, 1973, then Acting Commissioner Nituda issued an Order (Annex "6", counter-petition) which affirmed the Board of Special Inquiry No. 1 decision dated July 6, 1961 admitting respondent Gatchalian and others as Filipino citizens; recalled the July 6, 1962 warrant of arrest and revalidated their Identification Certificates. The above order admitting respondent as a Filipino citizen is the last official act of the government on the basis of which respondent William Gatchalian continually exercised the rights of a Filipino citizen to the present. Consequently, the presumption of citizenship lies in favor of respondent William Gatchalian. There should be no question that Santiago Gatchalian, grandfather of William Gatchalian, is a Filipino citizen. As a matter of fact, in the very order of the BOC of July 6, 1962, which reversed the July 6, 1961 BSI order, it is an accepted fact that Santiago Gatchalian is a Filipino. The opening paragraph of said order states:
The claim to Filipino citizenship of abovenamed applicants is based on the citizenship of one Santiago Gatchalian whose Philippine citizenship was recognized by the Bureau of Immigration in an Order dated July 12, 1960. (Annex "37", Comment with CounterPetition).

Nonetheless, in said order it was found that the applicants therein have not satisfactorily proven that they are the children and/or grandchildren of Santiago Gatchalian. The status of Santiago Gatchalian as a Filipino was reiterated in Arocha and Arca (supra) where advertence is made to the "applicants being the descendants of one Santiago Gatchalian, a Filipino." (at p. 539). In the sworn statement of Santiago Gatchalian before the Philippine Consul in Hongkong in 1961 (Annex "1" to the Comment of petitioners to Counter-Petition), he reiterated his status as a Philippine citizen being the illegitimate child of Pablo Pacheco and Marciana Gatchalian, the latter being a Filipino; that he was born in Manila on July 25, 1905; and that he was issued Philippine Passport No. 28160 (PA-No. A91196) on November 18, 1960 by the Department of Foreign Affairs in Manila. In his affidavit of January 23, 1961 (Annex "5", counter-petition), Santiago reiterated his claim of Philippine citizenship as a consequence of his petition for cancellation of his alien registry which was granted on February 18, 1960 in C.E.B. No. 3660-L; and that on July 20, 1960, he was recognized by the Bureau of Immigration as a Filipino and was issued Certificate No. 1-2123.

The dissenting opinions of my esteemed brethrens, Messrs. Justices F.P. Feliciano and H.G. Davide, Jr., proposing to re-open the question of citizenship of Santiago Gatchalian at this stage of the case, where it is not even put in issue, is quite much to late. As stated above, the records of the Bureau of Immigration show that as of July 20, 1960, Santiago Gatchalian had been declared to be a Filipino citizen. It is a final decision that forecloses a re-opening of the same 30 years later. Petitioners do not even question Santiago Gatchalian's Philippine citizenship. It is the citizenship of respondent William Gatchalian that is in issue and addressed for determination of the Court in this case. Furthermore, petitioners' position is not enhanced by the fact that respondent's arrest came twenty-eight (28) years after the alleged cause of deportation arose. Section 37 (b) of the Immigration Act states that deportation "shall not be effected . . . unless the arrest in the deportation proceedings is made within five (5) years after the cause of deportation arises." In Lam Shee vs. Bengzon (93 Phil. 1065 [1953]), We laid down the consequences of such inaction, thus:
There is however an important circumstance which places this case beyond the reach of the resultant consequence of the fraudulent act committed by the mother of the minor when she admitted that she gained entrance into the Philippines by making use of the name of a Chinese resident merchant other than that of her lawful husband, and that is, that the mother can no longer be the subject of deportation proceedings for the simple reason that more than 5 years had elapsed from the date of her admission. Note that the above irregularity was divulged by the mother herself, who in a gesture of sincerity, made an spontaneous admission before the immigration officials in the investigation conducted in connection with the landing of the minor on September 24, 1947, and not through any effort on the part of the immigration authorities. And considering this frank admission, plus the fact that the mother was found to be married to another Chinese resident merchant, now deceased, who owned a restaurant in the Philippines valued at P15,000 and which gives a net profit of P500 a month, the immigration officials then must have considered the irregularity not serious enough when, inspire of that finding, they decided to land said minor "as a properly documented preference quota immigrant" (Exhibit D). We cannot therefore but wonder why two years later the immigration officials would reverse their attitude and would take steps to institute deportation proceedings against the minor. Under the circumstances obtaining in this case, we believe that much as the attitude of the mother would be condemned for having made use of an improper means to gain entrance into the Philippines and acquire permanent residence there, it is now too late, not to say unchristian, to deport the minor after having allowed the mother to remain even illegally to the extent of validating her residence by inaction, thus allowing the period of prescription to set in and to elapse in her favor. To permit his deportation at this late hour would be to condemn him to live separately from his mother through no fault of his thereby leaving him to a life of insecurity resulting from lack of support and protection of his family. This inaction or oversight on the part of immigration officials has created an anomalous situation which, for reasons of equity, should be resolved in favor of the minor herein involved. (Emphasis supplied)

In the case at bar, petitioners' alleged cause of action and deportation against herein respondent arose in 1962. However, the warrant of arrest of respondent was issued by Commissioner Domingo only on August 15, 1990 28 long years after. It is clear that

petitioners' cause of action has already prescribed and by their inaction could not now be validly enforced by petitioners against respondent William Gatchalian. Furthermore, the warrant of exclusion dated July 6, 1962 was already recalled and the Identification certificate of respondent, among others, was revalidated on March 15, 1973 by the then Acting Commissioner Nituda. It is also proposed in the dissenting opinions of Messrs. Justices Feliciano and Davide, Jr., that the BOC decision dated July 6, 1962 and the warrant of exclusion which was found to be valid in Arocha should be applicable to respondent William Gatchalian even if the latter was not a party to said case. They also opined that under Sec. 37 (b) of the Immigration Act, the five (5) years limitation is applicable only where the deportation is sought to be effected under clauses of Sec. 37 (b) other than clauses 2, 7, 8, 11 and 12 and that no period of limitation is applicable in deportations under clauses 2, 7, 8, 11 and 12. The Court disagrees. Under Sec. 39 of the Immigration Act, it is reiterated that such deportation proceedings should be instituted within five (5) years. Section 45 of the same Act provides penal sanctions for violations of the offenses therein enumerated with a fine of "not more than P1,000.00 and imprisonment for not more than two (2) years and deportation if he is an alien." Thus:
Penal Provisions Sec. 45. Any individual who (a) When applying for an immigration document personates another individual, or falsely appears in the name of deceased individual, or evades the immigration laws by appearing under an assumed name; fictitious name; or (b) Issues or otherwise disposes of an immigration document, to any person not authorized by law to receive such document; or (c) Obtains, accepts or uses any immigration document, knowing it to be false; or (d) Being an alien, enters the Philippines without inspection and admission by the immigration officials, or obtains entry into the Philippines by wilful, false, or misleading representation or wilful concealment of a material fact; or (e) Being an alien shall for any fraudulent purpose represent himself to be a Philippine citizen in order to evade any requirement of the immigration laws; or (f) In any immigration matter shall knowingly make under oath any false statement or representations; or (g) Being an alien, shall depart from the Philippines without first securing an immigration clearance certificates required by section twenty-two of this Act; or (h) Attempts or conspires with another to commit any of the foregoing acts, shall be guilty of an offense, and upon conviction thereof, shall be fined not more than one thousand

pesos, and imprisoned for not more than two years, and deported if he is an alien. (Emphasis supplied)

Such offenses punishable by correctional penalty prescribe in 10 years (Art. 90, Revised Penal Code); correctional penalties also prescribe in 10 years (Art. 92, Revised Penal Code). It must be noted, however, that under Sec. 1, Act No. 3326 [1926], as amended, (Prescription for Violations Penalized by Special Acts and Municipal Ordinances) "violations penalized by special acts shall, unless otherwise provided in such acts, prescribe in accordance with the following rules: . . .c) after eight years for those punished by imprisonment for two years or more, but less than six years; . . ." Consequently, no prosecution and consequent deportation for violation of the offenses enumerated in the Immigration Act can be initiated beyond the eight-year prescriptive period, the Immigration Act being a special legislation. The Court, therefore, holds that the period of effecting deportation of an alien after entry or a warrant of exclusion based on a final order of the BSI or BOC are not imprescriptible. The law itself provides for a period of prescription. Prescription of the crime is forfeiture or loss of the rights of the State to prosecute the offender after the lapse of a certain time, while prescription of the penalty is the loss or forfeiture by the government of the right to execute the final sentence after the lapse of a certain time (Padilla, Criminal Law, Vol. 1, 1974, at p. 855). "Although a deportation proceeding does not partake of the nature of a criminal action, however, considering that it is a harsh and extraordinary administrative proceeding affecting the freedom and liberty of a person, the constitutional right of such person to due process should not be denied. Thus, the provisions of the Rules of Court of the Philippines particularly on criminal procedure are applicable to deportation proceedings." (Lao Gi vs. Court of Appeals, supra). Under Sec. 6, Rule 39 of the Rules of Court, a final judgment may not be executed after the lapse of five (5) years from the date of its entry or from the date it becomes final and executory. Thereafter, it may be enforced only by a separate action subject to the statute of limitations. Under Art. 1144 (3) of the Civil Code, an action based on judgment must be brought within 10 years from the time the right of action accrues. In relation to Sec. 37 (b) of the Immigration Act, the rule, therefore, is: 1. Deportation or exclusion proceedings should be initiated within five (5) years after the cause of deportation or exclusion arises when effected under any other clauses other than clauses 2, 7, 8, 11 and 12 and of paragraph (a) of Sec. 37 of the Immigration Act; and 2. When deportation or exclusion is effected under clauses 2, 7, 8, 11 and 12 of paragraph (a) of Sec. 37, the prescriptive period of the deportation or exclusion proceedings is eight (8) years.

In the case at bar, it took petitioners 28 years since the BOC decision was rendered on July 6, 1962 before they commenced deportation or exclusion proceedings against respondent William Gatchalian in 1990. Undoubtedly, petitioners' cause of action has already prescribed. Neither may an action to revive and/or enforce the decision dated July 6, 1962 be instituted after ten (10) years (Art. 1144 [3], Civil Code). Since his admission as a Filipino citizen in 1961, respondent William Gatchalian has continuously resided in the Philippines. He married Ting Dee Hua on July 1, 1973 (Annex "8", counter-petition) with whom he has four (4) minor children. The marriage contract shows that said respondent is a Filipino (Annex "8"). He holds passports and earlier passports as a Filipino (Annexes "9", "10" & "11", counter-petition). He is a registered voter of Valenzuela, Metro Manila where he has long resided and exercised his right of suffrage (Annex 12, counter-petition). He engaged in business in the Philippines since 1973 and is the director/officer of the International Polymer Corp. and Ropeman International Corp. as a Filipino (Annexes, "13" & "14", counter-petition). He is a taxpayer. Respondent claims that the companies he runs and in which he has a controlling investment provides livelihood to 4,000 employees and approximately 25,000 dependents. He continuously enjoyed the status of Filipino citizenship and discharged his responsibility as such until petitioners initiated the deportation proceedings against him. "The power to deport an alien is an act of the State. It is an act by or under the authority of the sovereign power. It is a police measure against undesirable aliens whose presence in the country is found to be injurious to the public good and domestic tranquility of the people" (Lao Gi vs. Court of Appeals, supra). How could one who has helped the economy of the country by providing employment to some 4,000 people be considered undesirable and be summarily deported when the government, in its concerted drive to attract foreign investors, grants Special Resident Visa to any alien who invest at least US$50,000.00 in the country? Even assuming arguendo that respondent is an alien, his deportation under the circumstances is unjust and unfair, if not downright illegal. The action taken by petitioners in the case at bar is diametrically opposed to settled government policy. Petitioners, on the other hand, claim that respondent is an alien. In support of their position, petitioners point out that Santiago Gatchalian's marriage with Chu Gim Tee in China as well as the marriage of Francisco (father of William) Gatchalian to Ong Chiu Kiok, likewise in China, were not supported by any evidence other than their own selfserving testimony nor was there any showing what the laws of China were. It is the postulate advanced by petitioners that for the said marriages to be valid in this country, it should have been shown that they were valid by the laws of China wherein the same were contracted. There being none, petitioners conclude that the aforesaid marriages cannot be considered valid. Hence, Santiago's children, including Francisco, followed the citizenship of their mother, having been born outside of a valid marriage. Similarly, the validity of the Francisco's marriage not having been demonstrated, William and Johnson followed the citizenship of their mother, a Chinese national.

After a careful consideration of petitioner's argument, We find that it cannot be sustained. In Miciano vs. Brimo (50 Phil. 867 [1924]; Lim and Lim vs. Collector of Customs, 36 Phil. 472; Yam Ka Lim vs. Collector of Customs, 30 Phil. 46 [1915]), this Court held that in the absence of evidence to the contrary, foreign laws on a particular subject are presumed to be the same as those of the Philippines. In the case at bar, there being no proof of Chinese law relating to marriage, there arises the presumption that it is the same as that of Philippine law. The lack of proof of Chinese law on the matter cannot be blamed on Santiago Gatchalian much more on respondent William Gatchalian who was then a twelve-year old minor. The fact is, as records indicate, Santiago was not pressed by the Citizenship Investigation Board to prove the laws of China relating to marriage, having been content with the testimony of Santiago that the Marriage Certificate was lost or destroyed during the Japanese occupation of China. Neither was Francisco Gatchalian's testimony subjected to the same scrutiny by the Board of Special Inquiry. Nevertheless, the testimonies of Santiago Gatchalian and Francisco Gatchalian before the Philippine consular and immigration authorities regarding their marriages, birth and relationship to each other are not self-serving but are admissible in evidence as statements or declarations regarding family reputation or tradition in matters of pedigree (Sec. 34, Rule 130). Furtheremore, this salutary rule of evidence finds support in substantive law. Thus, Art. 267 of the Civil Code provides:
Art. 267. In the absence of a record of birth, authentic document, final judgment or possession of status, legitimate filiation may be proved by any other means allowed by the Rules of Court and special laws. (See also Art. 172 of the Family Code)

Consequently, the testimonies/affidavits of Santiago Gatchalian and Francisco Gatchalian aforementioned are not self-serving but are competent proof of filiation (Art. 172 [2], Family Code). Philippine law, following the lex loci celebrationis, adheres to the rule that a marriage formally valid where celebrated is valid everywhere. Referring to marriages contracted abroad, Art. 71 of the Civil Code (now Art. 26 of the Family Code) provides that "(a)ll marriages performed outside of the Philippines in accordance with the laws in force in the country where they were performed, and valid there as such, shall also be valid in this country . . ." And any doubt as to the validity of the matrimonial unity and the extent as to how far the validity of such marriage may be extended to the consequences of the coverture is answered by Art. 220 of the Civil Code in this manner: "In case of doubt, all presumptions favor the solidarity of the family. Thus, every intendment of law or facts leans toward the validity of marriage, the indissolubility of the marriage bonds, the legitimacy of children, the community of property during marriage, the authority of parents over their children, and the validity of defense for any member of the family in case of unlawful aggression." (Emphasis supplied). Bearing in mind the "processual presumption" enunciated in Miciano and other cases, he who asserts that the marriage is not valid under our law bears the burden of proof to present the foreign law.

Having declared the assailed marriages as valid, respondent William Gatchalian follows the citizenship of his father Francisco, a Filipino, as a legitimate child of the latter. Francisco, in turn is likewise a Filipino being the legitimate child of Santiago Gatchalian who (the latter) is admittedly a Filipino citizen whose Philippine citizenship was recognized by the Bureau of Immigration in an order dated July 12, 1960. Finally, respondent William Gatchalian belongs to the class of Filipino citizens contemplated under Sec. 1, Article IV of the Constitution, which provides:
Sec. 1. The following are citizens of the Philippines: (1) Those who are citizens of the Philippines at the time of the adoption of this Constitution. . . .

This forecloses any further question about the Philippine citizenship of respondent William Gatchalian. The Court is not unaware of Woong Woo Yiu vs. Vivo (13 SCRA 552 [1965]) relied upon by petitioners. The ruling arrived thereat, however, cannot apply in the case at bar for the simple reason that the parties therein testified to have been married in China by a village leader, which undoubtedly is not among those authorized to solemnize marriage as provided in Art. 56 of the Civil Code (now Art. 7, Family Code). Premises considered, the Court deems it unnecessary to resolve the other issues raised by the parties. WHEREFORE, G.R. Nos. 95122-23 is DISMISSED for lack of merit; G.R. Nos. 9561213 is hereby GRANTED and respondent William Gatchalian is declared a Filipino citizen. Petitioners are hereby permanently enjoined from continuing with the deportation proceedings docketed as DC No. 90-523 for lack of jurisdiction over respondent Gatchalian, he being a Filipino citizen; Civil Cases No. 90-54214 and 3431V-90 pending before respondent judges are likewise DISMISSED. Without pronouncement as to costs. SO ORDERED.
PEOPLE VS. SUCRO [195 SCRA 388; G.R. No. 93239; 18 Mar 1991] Wednesday, February 04, 2009 Posted by Coffeeholic Writes Labels: Case Digests, Political Law

Facts:

Pat. Fulgencio went to Arlie Regalados house at

C. Quimpo to monitor activities of Edison SUCRO (accused). Sucro was reported to be selling marijuana at a

chapel 2 meters away from Regalados house. Sucro was monitored to have talked and exchanged things three times. These activities are reported through radio to P/Lt. Seraspi. A third buyer was transacting with appellant and was reported and later identified as Ronnie Macabante. From that moment, P/Lt.Seraspi proceeded to the area. While the police officers were at the Youth Hostel in Maagama St. Fulgencio told Lt. Seraspi to intercept. Macabante was intercepted at Mabini and Maagama crossing in front of Aklan Medical center. Macabante saw the police and threw a tea bag of marijuana on the ground. Macabante admitted buying the marijuana from Sucro in front of the chapel. The police team intercepted and arrested SUCRO at the corner of C. Quimpo and Veterans. Recovered were 19 sticks and 4 teabags of marijuana from a cart inside the chapel and another teabag from Macabante.

Issues:
(1) Whether or Not arrest without warrant is lawful. (2) Whether admissible. or Not evidence from such arrest is

Held: Search

and seizures supported by a valid warrant

of arrest is not an absolute rule. Rule 126, Sec 12 of Rules of Criminal Procedure provides that a person lawfully

arrested may be searched for dangerous weapons or anything, which may be used as proff of the commission of an offense, without a search warrant.(People v. Castiller) The failure of the police officers to secure a warrant stems from the fact that their knowledge required from the surveillance was insufficient to fulfill requirements for its issuance. However, warantless search and seizures are legal as long as PROBABLE CAUSE existed. The police officers have personal knowledge of the actual commission of the crime from the surveillance of the activities of the accused. As police officers were the ones conducting the surveillance, it is presumed that they are regularly in performance of their duties.

** People v Sucro 195 SCRA 388 (1991) Facts: Pat. Fulgencio went to Arlie Regalados house at C. Quimpo to monitor activities of Edison SUCRO (accused). Sucro was reported to be selling marijuana at a chapel 2 meters away from Regalados house. Sucro was monitored to have talked and exchanged things three times. These activities are reported through radio to P/Lt. Seraspi. A third buyer was transacting with appellant and was reported and later identified as Ronnie Macabante. From that moment, P/Lt.Seraspi proceeded to the area. While the police officers were at the Youth Hostel in Maagama St. Fulgencio told Lt. Seraspi to intercept. Macabante was intercepted at Mabini and Maagama crossing in front of Aklan Medical center. Macabante saw the police and threw a tea bag of marijuana on the ground. Macabante admitted buying the marijuana from Sucro in front of the chapel. The police team intercepted and arrested SUCRO at the corner of C. Quimpo and Veterans. Recovered were 19 sticks and 4 teabags of marijuana from a cart inside the chapel and another teabag from Macabante. Issues: (1) Whether or Not arrest without warrant is lawful. (2) Whether or Not evidence from such arrest is admissible.

Held: Search and seizures supported by a valid warrant of arrest is not an absolute rule. Rule 126, Sec 12 of Rules of Criminal Procedure provides that a person lawfully arrested may be searched for dangerous weapons or anything, which may be used as proff of the commission of an offense, without a search warrant.(People v. Castiller) The failure of the police officers to secure a warrant stems from the fact that their knowledge required from the surveillance was insufficient to fulfill requirements for its issuance. However, warantless search and seizures are legal as long as PROBABLE CAUSE existed. The police officers have personal knowledge of the actual commission of the crime from the surveillance of the activities of the accused. As police officers were the ones conducting the surveillance, it is presumed that they are regularly in performance of their duties.

PEOPLE VS. SOLAYAOGR 119220, SEPT. 20, 1996, SECOND DIVISIONJUSTICE


ROMEROFACTS: Nilo Solayao was charged before the Regional Trial Court of Naval, Biliran, Branch 16, with the crime of illegal possession of firearm and ammunition defined and penalized under Presidential Decree No. 1866.- SPO3 Jose Nio stated that he and other operatives went to Barangay Caulangohan, Caibiran, Biliran.They were to conduct an intelligence patrol as required of them by their intelligence officer to verifyreports on the presence of armed persons roaming around the barangays of Caibiran.The team of Police Officer Nio proceeded to Barangay Onion where they met the group of accused-appellant Nilo Solayao numbering five. The former became suspicious when they observed that thelatter were drunk and that scompanions, upon seeing the government agents, fled.- Confiscated from Solayao is a homemade firearm called Latong. Solayao admitted that he had nopermission to possess the same. Thereupon, SPO3 Nio confiscated the firearm and turned him over tothe custody of the policeman of Caibiran who subsequently investigated him and charged him withillegal possession of firearm.Solayao was found guilty, then he appealed to the court against the admissibility of the firearm asevidence as it was the product of an unlawful warrantless search. ISSUE: WON t he trial court erred in admitting in evidence the homemade firearm . HELD: Firearm is admissible as evidence. RATIO: The case at bar constitutes an instance where a search and seizure may be effected without firstmaking an arrest. There was justifiable cause to "stop and frisk" accused-appellant when his companionsfled upon seeing the government agents. Under the circumstances, the government agents could notpossibly have procured a search warrant first. Suspicion also arouse when the group was spotteddressed in camouflage.Also the officers were precisely in the area to conduct an operation to verify the intelligencereport and stop any potential disturbance in the area.

G.R. No. L-54562 August 6, 1987 THE PEOPLE OF THE PHILIPPINES, plaintiff-appellee, vs. DANILO PUNZALAN, VGET ISON and GUILBERT CUISON, defendants-appellants.

GANCAYCO, J.: This is an automatic review of the decision dated September 7, 1979, rendered by Hon. Judge Onofre L. Vinaluz, Circuit Criminal Court, Pasig, Metro Manila in Criminal Case No. CCC-VII-2639-Rizal, convicting all three (3) accused-appellants of the crime of murder as charged in the information and imposing upon them the death penalty for the fatal shooting of Cpl. Maximo de los Santos of the Paraaque police force in Paranaque, Metro Manila, on June 11, 1978. The dispositive portion of the decision in the vernacular reads in full as follows:
Sa dahilang napatunayan ng Hukuman na sina Danilo Punzalan, Vget Ison at Guilbert Cuison ay nagkasala sa salang sinadyang pagpatay, ayon sa Articulo 248 ng Binagong Kodigo Penal sila ay pinapatawan ng parusang KAMATAYAN. Magbabayad sila ng danyos perwisyong P12,000.00 sa mga naulila ng nasawi at P5,000.00 bayad pinsalang moral at P5,000.00 bayad pinsalang di dapat pamarisan at hati-hati sila sa gastos ng usapin. IPINAGUUTOS. 1

The original information, dated July 19, 1978 and filed wih the trial court on July 27, 1978 charged only three (3) accused, namely, Ramon Jumawan, Danilo Punzalan and Vget Ison, with murder qualified by treachery, abuse of superior strength and cruelty. About four (4) months later, on November 21, 1978, the information was amended to include Guilbert Cuison as a fourth accused as a result of a preliminary investigation conducted by the Fiscal pursuant to the Order of the trial court dated August 11, 1978 issued upon motion of the private prosecutor on the strength of a supplementary sworn statement of one Arsenio Telmo dated August 9, 1978 (Exh. 1) claiming that Guilbert Cuison was one of those he saw hitting the late Cpl. Maximo de los Santos with a chair. Upon arraignment, the three (3) accused, Danilo Punzalan, Vget Ison and Guilbert Cuison pleaded not guilty to the charge and thereafter entered into trial. The other accused, Ramon Jumawan, was never arrested and remained at large. After trial, the lower court returned a verdict of guilty and sentenced the three (3) accused to death as earlier stated. As set forth in the People's brief, the facts attendant to the commission of the crime charged are as follows:
In the early evening of June 11, 1978, Cpl. Maximo delos Santos of the Paraaque Police Department, was inside the Nordel Restaurant in Sucat Road, Paranaque, Metro Manila. While he was there, a group of men composed of Ramon Jumawan, who remains at large, Danilo Punzalan, Vget Ison, Guilbert Cuison, Reynaldo Dandan, Angelito Bernardo, Cesar Llamas, Wally or Walfrido Punzalan, Benjamin Arciaga and Leon Allan: gue occupied a round table at the restaurant drinking beer. Sometime later, Cpl. delos Santos approached the group saying: "Mga padres, pulis ako, ibig ko lang alamin sa inyo kung sino ang may baril." Whereupon, Wally Punzalan who was seated across the table, pulled out a .45 cal. pistol which his brother, Danilo, had earlier slipped to him under the

table. Cpl. delos Santos drew his service revolver and fired twice at Wally Punzalan before the latter could fire at him. Accused Ramon Jumawan then hit the policeman on the head with a chair. The policeman fell to the floor, face down. Appellants Danilo Punzalan and Guilbert Cuison followed suit hitting Cpl. delos Santos repeatedly with chairs on the head, back and arms. Appellant Vget Ison joined in and hit their victim with a beer bottle (Tsn, Jan. 10, 1979, pp. 6-20; Tsn, July 19, 1979, pp. 14-16; Exhs. E, F). The attack was so violent that the chairs and beer bottles which appellants used in hitting the policeman were broken. The broken chairs and fragments of broken glass were strewn on the floor (Tsn, Dec. 7, 1978, pp. 67-76; Exhs. C to C-16). As the policeman lay prostrate and helpless on the floor, Ramon Jumawan grabbed the victim's service revolver and shot the latter four (4) times. All shots found their mark (Tsn, Jan. 10, 1979, pp. 16.17; Tsn, Dec. 7,1978, pp. 28-33; Exh. A). After the group saw that Cpl. Santos was already dead the group left together (Tsn, Jan. 10, 1979, p. 19). They helped each other in bringing the wounded Wally Punzalan to the hospital where he expired five days later (Tsn, July 17, 1979, pp. 75-78, 151-152). The 2 jacket of Danilo Punzalan was left on the floor (Tsn, Dec. 7, 1978, pp. 80-81; Exh. C-4.

Three separate briefs by different lawyers were filed for the appellants: the first for all the appellants; the second for appellants Vget Ison and Guilbert Cuison; and the third, for appellant Vget Ison only. The defense advanced the following version of the incident:
The undisputed facts are that at about 9:30 P.M. June 11, 1978, a group of ten (10) male persons was * at the Nordel Restaurant, Dr. Santos Street, Sucat, Paraaque, Metro Manila, for food and beer. The group included the four accused, Danilo Punzalan, Ramon Jumawan Vget Ison, and Guilbert Cuison. Wally Punzalan, brother of Danilo Punzalan was also there. The owner of the restaurant, Adelaida Borinaga, was also present as was one Elizabeth Mendoza, waitress, who waited on the group. The ten persons seated themselves about a round table at the center of the eating place as they partook of the food and drinks. Cpl. Maximo de los Santos, a traffic policeman of Paranaque, was also in the restaurant, in civilian clothes, drinking beer, seated at the counter some five meters away from the table of the group. He ordered a bottle of beer from Adelaida Borinaga. All went well until Cpl. de los Santos suddenly left his place at the counter, went to the table occupied by the accused-appellants and their companions, and with his .38 caliber pistol shot Wally Punzalan twice at close range felling the latter. In the ensuing rapid succession of action Ramon Jumawan hit Cpl. de los Santos on the head with a chair and gained possession of the policeman's gun and used it in shooting the policeman in turn three or four times. The policeman sustained four gunshot wounds three of which caused his death. The fourth one was a mere grazing wound. He also had a lacerated wound on the head and other wounds and some contusions and abrasions. He died on the spot where he fell. The necropsy report (Exhibit A) gave the cause of death as severe hemorrhage secondary to gunshot wounds. He was brought to the Olivares General Hospital nearby where he expired six days later, on June 17, 1978, from peritonitis generalized, 3 secondary to gunshot wounds according to the necropsy report (Exhibit 6).

It appears from the foregoing narration that contrary to the trial court's findings that the gunplay was started by one of the members of the group. ("isa sa mga nakaupo ang

nagbunot ng .45 cal. at binaril ang pulis ... ) what actually transpired was that it was the policeman (victim) who first shot Wally Punzalan. With that backdrop, We shall now discuss the various assignments of errors raised by the defense which are summarized as follows:
1. The trial court erred in finding the existence of treachery and conspiracy among the accused; 2. The trial court erred in not crediting the defendants with the justifying circumstance of self-defense or defense of relative; 3. The trial court erred in giving credence to the testimony of the prosecution witness, Arsenio Telmo; 4. The trial court erred in failing to acquit all the defendants for failure of the prosecution to establish their guilt beyond reasonable doubt.

Anent the first assigned error, appellants maintain that the trial court erred in finding the existence of conspiracy and treachery among them. As provided in paragraph 2, Article 8 of the Revised Penal Code, conspiracy exists when two or more persons come to an agreement concerning the commission of a felony and decide to commit it. Since in conspiracy, the act of one becomes the act of all, all persons taking part in the crime are held guilty as principals. Because of its farreaching consequences, the same degree of proof required for establishing the crime is required to support a finding of the presence of conspiracy. Thus, it must be shown to exist as clearly and convincingly as the commission of the offense itself, 4 in order to uphold the fundamental principle that no one shall be found guilty of a crime except upon proof beyond reasonable doubt. 5 While it may be true that direct proof is not essential to prove conspiracy, 6 and it may be shown by acts and circumstances from which may logically be inferred the existence of a common design among the accused to commit the offense charged, nonetheless, the evidence to prove the same must be positive and convincing. 7 A close scrutiny of the records in the present case shows that there is not a shred of evidence to establish the existence of conspiracy between appellants. From the evidence presented by the prosecution it is clear that the incident at the Nordel Restaurant on June 11, 1978 was unplanned and spontaneous. First, admittedly, appellants were together drinking and eating in the company of other persons before the incident happened. This circumstance alone does not suffice to prove the existence of a common criminal design. Mere suspicion, speculation, relationship or association and companionship do not establish conspiracy, for proof thereof must be positive and convincing. 8 In the instant case, it is neither alleged nor shown that appellants' meeting at the Nordel Restaurant at that particular time was planned. Nor is there evidence that any of the appellants knew that the victim was going

to be in that particular place or that such meeting was purposely sought for by them. No doubt the encounter was purely accidental. In fact, had the victim not approached the appellants, no untoward incident could have happened. Second, it appears that it was the deceased Maximo de los Santos who approached the table of appellants and was the first to fire the two shots at Wally Punzalan, when he attempted to draw his gun. The fact that the attack was commenced by Maximo de los Santos negates the theory that appellants conspired to kill him. lt was only after Wally Punzalan was gunned down that appellants reacted and ganged up on the victim. Ramon Jumawan hit the victim at the back so his gun fell. Jumawan then picked up the gun which he used in killing the victim. 9 The fact that the appellants struck and hit the victim with the chairs and beer bottles did not make them conspirators thereby. The evidence shows that appellants acted on impulse, without prior deliberation, planning or design. The appellants must have felt threatened by the deceased who killed their companion. Hence their spontaneous response to the real and determined acts of the victim must have been motivated by a desire to repel the aggression if not to prevent any further harm the victim may inflict on any of them. The simultaneous attack of appellants cannot be considered as amounting to a conspiracy to kill the deceased. Neither joint and simultaneous action nor relationship is per se a sufficient indicium of conspiracy. 10 A common design must further be shown to have motivated such action. 11 Indeed, it is the gunshot wounds that led to the death of the victim. The injuries inflicted by appellants were superficial to serious but not fatal. 12 In the absence of clear proof that the killing was in fact envisaged by them, they cannot be held responsible for the death of the victim. With respect to treachery, the attendance of this aggravating circumstance is found in the concurrence of two conditions: (1) the employment of means, method or manner of execution which would insure the offender's safety from any defensive or retaliatory act on the part of the offended party, which means that no opportunity is given to the latter to do so, 13 and (2) that such means, method or manner of execution was deliberately or consciously chosen. 14 Thus, it is not enough that the means, method or form of execution tends directly and specially to facilitate the commission of the offense without danger to the offender arising from the defense or retaliation that might be made by the offended party. It is further required, for treachery to be appreciated, that such means, method or form was deliberated upon or consciously adopted by the offender. 15 The first of the two conditions aforestated appears to be present in the instant case. It has been adequately established that appellants delivered blows to the victim when the latter was down on the floor after being struck by Ramon Jumawan which fact insured their safety from any defensive or retaliatory act from the victim. However, the second condition is wanting.

From the rapid succession of events, it appears that the means employed were not deliberated upon by appellants. The act of the appellants in assaulting the victim appears to be impelled by the instinct of self-preservation if not retaliation for the shooting of their comrade. Such deliberate or conscious choice of the means was held non-existent where, as in this case, the attack was the product of an impulse of the moment, 16 and where the defendants did not make any preparation to assault the victim. 17 The decision to attack the victim was accidental. 18 The attack arose from a chance encounter and quarrel. 19 Treachery therefore is not present in this case. Anent the second assigned error, appellants maintain that the trial court erred in not crediting the appellants with the justifying circumstance of self-defense or defense of relative. Self-defense 20 and defense of the rights of another 21 are recognized circumstances justifying an offense and exempting the perpetrator from criminal liability. It appears that all the elements of self-defense and defense of a stranger are present under the facts and circumstances of the case at bar. It was shown that the policeman was the aggressor. The assault was sudden and totally unexpected so that he succeeded in pumping two bullets into the chest and stomach of his unsuspecting victim Wally Punzalan, giving the latter no chance or time to defend himself or evade the aggression. After gunning down Wally Punzalan, the policeman's aggression did not stop there. While the policeman stood there menacingly with his gun, the peril to the life of Wally Punzalan and all his companions continued. Faced with this real and imminent danger to their own life, and in obedience to the dictates of the instinct of self- preservation, appellants together with Ramon Jumawan, mauled the policeman to immobilize him. And when the latter was disarmed, Jumawan picked the gun and shot him four times. Although this Court cannot in all reason condone the shooting of a person who is already helpless and lying prostrate on the floor, so that Jumawan who is at large must be held to account for his acts, as to the appellants, this Court is mindful of the following discourse it made on the subject, thus:
The law does not require, and it would be too much to ask of the ordinary man, that when he is defending himself from a deadly assault, in the heat of an encounter at close quarters, he should so mete out his blows that upon a calm and deliberate review of the incident, it will not appear that he exceeded the precise limits of what was absolutely necessary to put his antagonist hors de combat; or that he struck one blow more than was absolutely necessary to save his own life; or that he failed to hold his hand so as to avoid inflicting a fatal wound where a less severe stroke might have served his purpose. Of course, the victim of an unlawful aggression may not lawfully exceed the bounds of rational necessity in repelling the assault. But the measure of rational necessity in cases of this kind is to be found in the situation as it appears to the victim of the assault at the time when the blow is struck; and the courts should not and will not, in the light of after events or fuller knowledge, hold the victims of such deadly assaults at close quarters, to so strict a degree of accountability that they will hesitate to put forth their utmost effort in 22 their own defense when that seems to them to be reasonably necessary.

The reasonableness of the means employed to prevent an aggression depends upon the nature and quality of the weapon used by the aggressor, his physical condition, his size, his character and the surrounding circumstances vis-a-vis those of the person defending himself. It is also well-settled that in emergencies which imperil the life and limb of a person, human nature acts not upon processes of formal reason but in obedience to the imperious dictates of the instinct of self-preservation ... the protective mantle of the law shields not only him who repels actual aggression but as well as him who prevents an aggression that is real and imminent. And the killing of the aggressor would be justified at 23 a time when all the elements of self-defense are present. In emergencies of this kind, human nature does not act upon processes of formal reason but in obedience to the instinct of self-preservation, and when it is apparent that a person has reasonably acted upon this instinct, it is the duty of the courts to sanction the act and 24 to hold the actor irresponsible in law for the consequences.

Except for the shooting of the victim by Jumawan, We find that the appellants employed reasonable means to repel the aggression of the policeman. The final requisite for complete self-defense, lack of sufficient provocation of the person defending himself, is also present. The evidence shows that Wally Punzalan and his companions had not done anything to invite the ire of the policeman and provoke the latter's aggression. It also appears that the companions of the slain Wally Punzalan were not motivated by revenge, resentment or other evil motive in defending themselves and Wally Punzalan. The appellants having acted in self-defense and/or in defense of their companion, acted justifiably and are hereby exempt from criminal liability. As to the third assigned error, the testimony of prosecution witness Arsenio Telmo is sought to be discredited because he gave his statement implicating appellants to the police only four (4) days after the incident. The initial reluctance of witnesses in this country to volunteer information about a criminal case and their unwillingness to be involved in or dragged into a criminal investigation is common and has been judicially declared not to affect credibility. 25 Telmo, a witness to the incident, was actually asked to go to the police headquarters on the night of June 11, 1978 but he refrained because he was scared. However, he readily gave his name, address and residence certificate to the police. There is nothing in the record that would prove he had an ax to grind against appellants. He is therefore a disinterested witness. Also, his presence at the night of the incident was confirmed by the restaurant owner. 26 The fact that it was only on August 9, 1978 that Telmo executed a supplemental affidavit implicating Guilbert Cuison for the first time is explained by the fact that it was only on that date that he was made to confront Cuison when the latter gave himself up to the authorities at Camp Crame. On that occasion, Telmo readily Identified Cuison as one of those who hit the victim. 27

Anent the last assigned error, from our view of the evidence, the participation of herein appellants in mauling the policeman cannot be denied. However, as earlier discussed, their acts were justified. From the evidence on record, this Court is convinced that herein appellants did not intend to kill the victim; that the circumstances where they found themselves provoked the action they took; their friend dead, shot by another who happened to be a policeman, who at that time was in plain clothes but was holding a gun. It is easy to imagine and feel the intense passion, fear and apprehension in everyone's heart at a time like that. In appellants' eyes, he was a common enemy, killer of their friend and a possible threat to their lives. Accused appellants cannot therefore be faulted since selfpreservation is still the first law of nature. Considering the above disquisitions, this Court is of the opinion that appellants should be acquitted of the charges against them. WHEREFORE, the judgment of conviction is hereby set aside and appellants are acquitted of the crime charged with costs de oficio. Their immediate release from custody is ordered, unless they are otherwise detained for some other legal cause. However, the authorities must exert all efforts for the apprehension and prosecution of Ramon Jumawan for this killing. SO ORDERED.

People vs. Damaso


Facts: After a sequence of arrest among members of theNPA, the Philippine Constabulary officers of LingayenPangasinan, the apprehended NPA (Luz Tancianco)members pointed out to the PC the house of the AppellantDamaso where the same is leasing.Being pointed out that the appellant is the lessee of the house, the police went inside and eventually saw M1 4rifles, radio sets, subversive materials and pamphlets, maps,computer machines, bullets and ammunitions. These articlesare confiscated to use as evidence against the accused inthe crime of Subversion.He was then convicted by the RTC Dagupan in thecrime of subversion, hence this appeal.The appellant contends that the seizure in his houseis illegal for absence of a search warrant.Issue:Was the search unlawful?Held:Yes. The search n the house of the appellant isillegal because of absence of search warrant and evenprobable cause for the issuance of the same.In the testimony of the witness by the prosecution,he has no personal knowledge making its testimonieshearsay and weak to establish the existence of a probablecause.Moreover, the search conducted is not one beingqualified as a valid search without warrant( 1 ) Search incidental to an arrest.(2) Customs search.(3) Consented search(4) Search of a moving vehicle(5) Stop frisk(6) Plain view search. ( A pply the M endoza doctrine- incidental discovery of incri m inating evidence to qualify as a plain view search )Moreover, the rights granted under the Bill of Rights ispersonal and that cannot be waived by anyone else rather than the person whose rights is invaded or one whoexpressly to do so in his behalf.In the case at bar, the prosecution did notestablished that Luz Tancianco was authorized by theappellant to allow the officers to enter the appellants houseand seized the effects of the appellant upon seeing it inpresumption that it is connected with the crime of subversion.Thereby violating the appellants rights of privacy and securityof house and effects.Thus, the

prosecutions evidences are weak andthat the search being illegal, the articles sought areinadmissible as evidences being a fruit of poisonous tree.

People vs. Damaso [GR 93516, 12 August 1992] First Division, Medialdea (J): 3 concur Facts: On 18 June 1988, Lt. Candido Quijardo, a Philippine Constabulary officer connected withthe 152nd PC Company at Lingayen, Pangasinan, and some companions were sent to verify the presence of CPP/NPA members in Barangay Catacdang, Arellano-Bani, Dagupan City. In said place, the group apprehended Gregorio Flameniano, Berlina Aritumba, Revelina Gamboa andDeogracias Mayaoa. When interrogated, the persons apprehended revealed that there was anunderground safehouse at Gracia Village in Urdaneta, Pangasinan. After coordinating with theStation Commander of Urdaneta, the group proceeded to the house in Gracia Village. Theyfound subversive documents, a radio, a 1 x 7 caliber .45 firearm and other items. After the raid,the group proceeded to Bonuan, Dagupan City, and put under surveillance the rented apartmentof Rosemarie Aritumba, sister of Berlina Aritumba whom they earlier arrested. They interviewedLuzviminda Morados, a visitor of Rosemarie Aritumba. She stated that she worked with BernieMendoza/Basilio Damaso. She guided the group to the house rented by Damaso(@Mendoza).When they reached the house, the group found that it had already vacated by the occupants.Since Morados was hesitant to give the new address of Damaso (@Mendoza), the group lookedfor the Barangay Captain of the place and requested him to point out the new house rented byDamaso (@Mendoza). The group again required Morados to go with them. When they reachedthe house, the group saw Luz Tanciangco outside. They told her that they already knew that shewas a member of the NPA in the area. At first, she denied it, but when she saw Morados sherequested the group to go inside the house. Upon entering the house, the group, as well as theBarangay Captain, saw radio sets, pamphlets entitled Ang Bayan, xerox copiers and acomputer machine. They also found persons who were companions of Luz Tanciangco (namely,Teresita Calosa, Ricardo Calosa, Marites Calosa, Eric Tanciangco and Luzviminda Morados).The group requested the persons in the house to allow them to look around. When LuzTanciangco opened one of the rooms, they saw books used for subversive orientation, one M-14rifle, bullets and ammunitions, Kenwood radio, artificial beard, maps of the Philippines,Zambales, Mindoro and Laguna and other items. They confiscated the articles and brought themto their headquarters for final inventory. They likewise brought the persons found in the house tothe headquarters for investigation. Said persons revealed that Damaso (@Mendoza) was thelessee of the house and owned the items confiscated therefrom. Thus, Basilio Damaso, wasoriginally charged in an information filed before the Regional Trial Court of Dagupan City withviolation of Presidential Decree 1866 in furtherance of, or incident to, or in connection with thecrime of subversion, together with Luzviminda Morados y Galang @ Ka Mel, Teresita Calosa yMacabangon @ Ka Tessie, Ricardo Calosa y Perez @ Ka Ric, Marites Calosa y Evangelista @Ka Tess, Eric Tanciangco y Capira @ Ka Ric and Luz Tanciangco y Pencial @ Ka Luz. Suchinformation was later amended to exclude all other persons except Damaso from the criminalcharge. Upon arraignment, Damaso pleaded not guilty to the crime charged. Trial on the meritsensued. The prosecution rested its case and offered its exhibits for admission. The defensecounsel interposed his objections to the admissibility of the prosecutions evidence on grounds of its being hearsay, immaterial or irrelevant and illegal for lack of a search warrant; and thereafter,manifested that he was not presenting any evidence for the accused. On 17 January 1990, thetrial court rendered its decision, finding Damaso guilty beyond reasonable doubt, sentencing thelatter to suffer the penalty of Reclusion Perpetua and to pay the costs of the proceedings. Damasoappealed. Issue: Whether there was waiver on the part of Damaso to allow the warrantless search of hishouse. Held: Damaso was singled out as the sole violator of PD 1866, in furtherance of, or incident to,or in connection with the crime of subversion. There is no substantial and credible evidence to establish the fact that the appellant is allegedly the same person as the lessee of the house wherethe M-14 rifle and other subversive items were found or the owner of the said items. Evenassuming for the sake of argument that Damaso is the lessee of the house, the case against himstill will not prosper, the reason being that the law enforcers failed to comply with therequirements of a valid search and seizure proceedings. The constitutional immunity fromunreasonable searches and seizures, being a personal one cannot he waived by anyone except the person whose rights are invaded or one who is expressly authorized to do so in his or her . Therecords show that Damaso was not

in his house at that time Luz Tanciangco and Luz Morados,his alleged helper, allowed the authorities to enter it. There is no evidence that would establishthe fact that Luz Morados was indeed Damasos helper or if it was true that she was his helper,that Damaso had given her authority to open his house in his absence. The prosecution likewisefailed to show if Luz Tanciangco has such an authority. Without this evidence, the authoritiesintrusion into Damasos dwelling cannot be given any color of legality. While the power tosearch and seize is necessary to the public welfare, still it must be exercised and the law enforcedwithout transgressing the constitutional rights of the citizens, for the enforcement of no statute isof sufficient importance to justify indifference to the basic principles of government. As aconsequence, the search conducted by the authorities was illegal. It would have been different if the situation here demanded urgency which could have prompted the authorities to dispense witha search warrant. But the record is silent on this point. The fact that they came to Damasoshouse at nighttime, does not grant them the license to go inside his house.

[G.R. No. 104378. August 20, 1996] PEOPLE OF THE PHILIPPINES, plaintiff-appellee, vs. DANILO JUATAN y CAPSA, accusedappellant. DECISION VITUG, J.: In its decision,i[1] dated 03 February 1992, in Criminal Case No. 91-96313, the Regional Trial Court of Manila, Branch 45, pronounced the accused, DANILO JUATAN Y CAPSA, "guilty beyond reasonable doubt" of violating Section 15 of Republic Act No. 6425, as amended by Presidential Decree 1683, and sentenced him to suffer the penalty of life imprisonment and to pay a fine of P20,000.00. Juantan was charged, on 09 July 1991, with the commission of the above offense in an information that read: "That on or about July 5, 1991, in the City of Manila, Philippines, the said accused, not having been authorized by law to sell, dispense, deliver, transport or distribute any regulated drug, did then and there wilfully, unlawfully and feloniously sell or offer for sale white crystalline substance known as `Shabu' containing methamphetamine hydrochloride, which is a regulated drug. "Contrary to law."ii[2] The accused entered a plea of not guilty;iii[3] forthwith, the trial proceeded. The prosecution made the following rendition of what it submitted to be the facts. The Western Police District Command, based in the City of Manila, received word from a confidential informant and some barangay officials that Danilo Juatan had been dealing in prohibited drugs. On orders of P/Maj. Cipriano Herrera, Jr., of the Narcotics Section, a police team, led by Lt. Enrique Sy, was organized to conduct a one-week surveillance on Juatan. When its surveillance showed positive results, the police team decided to conduct a buy-bust operation.

On 05 July 1991, at around one o'clock in the morning, the 8-member police teamiv[4] proceeded to Instruccion Street, Sampaloc, Manila. Pat. Ernesto Yamson was designated to be the poseur-buyer, while Pat. Eduardo Sison and the others were assigned to secure the area. Pat. Amelito Lopez placed himself around seven meters away from the house of the accused.v[5] The actual operation started with the informant calling the accused and telling him that Yamson wanted to buy "drugs." Juatan met Yamson and the informant at the corner of an alley, near Maceda and Instruccion streets, around ten meters away from Juatan's house. Yamson gave Juatan a P500-bill marked with his initials on the upper right hand portion of the bill;vi[6] in turn, Juatan handed over to Yamson a plastic container measuring 2" x 2 1/2."vii[7] At that precise moment, Yamson raised his right hand to signal his companions that the deal had been made. Sensing that something was amiss, Juatan fled. He was about to get into his house when the police apprehended him. Upon being searched, Juatan's right side pocket yielded the marked P500-bill.viii[8] Juatan was accompanied by his wife to the police headquarters.ix[9] The booking and information sheet bearing Juatan's signaturex[10] described him to be a 5'6" tall taxi driver, with tattoo marks of the "Sigue-Sigue" commando.xi[11] At around nine o'clock that same morning, the members of police team executed a joint affidavit of arrest.xii[12] Sgt. Antonio T. Taca, signing for Maj. Cipriano Herrera, Jr., sent a letter-request to the Chemistry Section of the Criminal Investigation Laboratory for the laboratory examination of the contents of the plastic bag taken from Juatan.xiii[13] The request, along with the specimen, was received in the late afternoon of 05 July 1991.xiv[14] On 08 July 1991,xv[15] the item was turned over to the forensic chemist, Renee Eric Checa, of the Chemistry Section. Measuring the specimen, Checa found it to weigh 0.395 gram.xvi[16] Using the thin-layer chromatography, Checa specifically identified the article to be shabu or methamphetamine hydrochloride.xvii[17] At the police headquarters, Juatan admitted that he was a drug user but, after being apprised of his constitutional rights to counsel and to remain silent, appellant decided not to make any further statement.xviii[18] On 08 July 1991, Sgt. Taca referred the case to the inquest fiscalxix[19] who suggested that the accused be detained.xx[20] In his testimony, Juatan said that he was arrested by policemen Edwin Sison, Ernesto Yamson and Amelito Lopez. Surprised by the arrest, Juatan asked the police officers what the problem was. He was told to produce a certain Boy Chua whom he did not know at all, thus prompting him to remark, "Edwin (referring to Pat. Sison), personal na yata ang ginagawa mo sa akin."xxi[21] When Juatan asked to be shown either a search warrant or a warrant of arrest, he, instead, got a punch on the face from Sison. The other police officers stopped Sison from doing any possible further harm on the accused. The group made a search. Finding nothing, they dragged Juatan out of his house and brought him, along with his wife, to the police headquarters.xxii[22] Juatan's wife, 38-year-old Aurora, declared that she was doing her laundry, at about one o'clock in the morning of 05 July 1991, when armed men forcibly entered their house. When she started to complain, the men simply told her to keep quiet. She followed the group upstairs; she was told that they were looking for her husband and a certain Boy Chua. When she told them that her

husband was asleep in the room, the men went in and immediately handcuffed him. It only angered the policemen when she demanded to be shown a warrant. At the police headquarters, she recognized one of the apprehending police officers to be Pat. Sison.xxiii[23] The defense presented three other witnesses. Jesus Lingat, a 32-year-old driver who resided just a house away from the Juatan residence, testified that at around 1:30 a.m. of 05 July 1991, he and his brother saw around nine armed men entering Juatan's house and later taking appellant with them. The witness did not see Juatan's wife.xxiv[24] Ludovico Munsayac, a 35-year old merchant and member of the barangay council, said that he personally knew Juatan to be just a taxi driver. Nonilon Reyes, barangay captain of Barangay 524, Zone 52, Sampaloc, Manila, stated that Juatan used to be a neighbor, and that, in connection with the latter's application with the "Operation Tulong of DZRH," he had issued a certification to the effect that Juatan was a taxi driver "ready to lend a helping hand to his neighbor."xxv[25] On rebuttal, the prosecution presented Pat. Eduardo Sison who denied the allegations made against him by Juatan and his wife. The primary issue in this appeal, as so aptly pointed out by appellant's counsel himself, is one of credibility.xxvi[26] It has long been the entrenched rule in criminal jurisprudence that appellate courts should defer to the findings of the trial court considering the latter's vantage position in ascertaining that question of credibility.xxvii[27] Unless it is fairly evident that the trial court, we consistently have said before, overlooked certain facts of substance and value that, if considered, could affect the outcome of the case, the trial court's own appraisal on the issue should be held to stand.xxviii[28] We have examined the areas suggested by appellant but, in sum, they appear to us to be matters peripheral in nature more than of substance. For one, appellant ascribes ulterior motives on the part of the arresting officers; however, he did not give any clear account or details of this charge. Appellant underscores the absence of a warrant of arrest or search warrant despite the fact that the police has had him subjected, prior to his apprehension, to a week-long surveillance. In fine, he contends that the police could have easily procured a "warrant before proceeding with the buy-bust operation."xxix[29] A buy-bust operation is far variant from an ordinary arrest; it is a form of entrapment which has repeatedly been accepted to be a valid means of arresting violators of the Dangerous Drugs Law. In a buy-bust operation the violator is caught in flagrante delicto and the police officers conducting the operation are not only authorized but duty-bound to apprehend the violator and to search him for anything that may have been part of or used in the commission of the crime.xxx[30] Appellant has a violated Section 15 of the Dangerous Drugs Law, as amended, which prescribes the penalty of reclusion perpetua to death. Pursuant, however, to the Court's ruling in People vs. Simon,xxxi[31] the penalty, considering that only 0.395 gram of shabu is involved in the prohibited sale, should now be prision correccional conformably with the amendatory law. Applying the Indeterminate Sentence Law, and there being neither an aggravating nor mitigating circumstance, the imposable penalty could be within the range of arresto mayor, as minimum, to prision correccional in its medium period, as maximum.

WHEREFORE, the Decision of the trial court finding appellant Danilo Juatan y Capsa guilty beyond reasonable doubt of violating Section 15 of the Dangerous Drugs Law, as amended, is hereby affirmed subject to the modification that he, instead, is meted the indeterminate sentence of six (6) months of arresto mayor, as minimum penalty, to four (4) years and two (2) months of prision correccional medium, as maximum penalty. Considering that appellant has been imprisoned for more than five (5) years or beyond the maximum penalty herein imposed upon him, his IMMEDIATE RELEASE from confinement is hereby ordered unless his continued detention can be justified by any other valid reason or cause. Costs against appellant.

G.R. No. 135053 PEOPLE OF THE PHILIPPINES, appellee, vs. BENJAMIN GALVEZ, appellant. PANGANIBAN, J.: The mandatory procedure laid down by jurisprudence and the Rules of Court should be meticulously observed by trial courts in accepting a plea of guilt in a case involving a capital offense. There should be no doubt that the accused might have misunderstood the nature of the charges and the consequences thereof. Otherwise, the plea would be set aside for having been improvidently made. , J.: The mandatory procedure laid down by jurisprudence and the Rules of Court should be meticulously observed by trial courts in accepting a plea of guilt in a case involving a capital offense. There should be no doubt that the accused might have misunderstood the nature of the charges and the consequences thereof. Otherwise, the plea would be set aside for having been improvidently made. The Case For automatic review is the July 30, 1998 Decision1 of the Regional Trial Court (RTC) of Bayombong, Nueva Vizcaya (Branch 27) in Criminal Case No. 3299, convicting Benjamin Galvez of qualified rape and sentencing him to death. The assailed Decision disposed as follows: "WHEREFORE, finding the accused Benjamin Galvez y Domingo GUILTY beyond reasonable doubt of committing rape against his own daughter, 16 years old at the time of the rape, he is hereby sentenced to die by lethal injection; to pay the victim the sum of P50,000.00 as civil indemnity and to pay the costs of the suit."2 The Facts In its Brief,3 the Office of the Solicitor General presents the following narration of facts:

"Cristina Galvez was born on May 10, 1981. She, her father, Benjamin Galvez, the appellant herein, together with her five (5) siblings, resided at Tuao South, Bagabag, Nueva Vizcaya. "Sometime in the third week of April 1997, about 6:00 oclock in the evening, Cristina, who was then sixteen (16) years old, and her youngest sister, Melowin, were sleeping beside each other on the bed. Their bed was beside an aparador and Cristina slept on the side nearest to the aparador. Appellants bed was beside the door and was just near the bed where Cristina and Melowin were sleeping. Three (3) of their brothers were also asleep in the house, namely, Melchor, 13 years old, Alvin, 12 years old, and Jesus, 9 years old, while their 19 year-old brother was at a neighbors house watching television. Their mother, Marilyn Galvez, was not with them as she was in Hongkong at that time. "That night, appellant, who was drunk, arrived and then ate. After eating, he went to lie down on his bed. After a lapse of thirty (30) minutes, appellant went to lie down on the bed where Cristina and Melowin were sleeping. He positioned himself on the right side of Cristina, slowly lifting and moving Melowin, who was asleep beside Cristina, away from her sisters side. Then, he slowly removed Cristinas shorts and shirt and began touching her on all parts of her body. He positioned himself on top of her but she kicked him, causing him to be thrown against the aparador. Apparently hurt and angered, he pulled his samurai from under his mat and pointed it towards the left front portion between her breast and her armpit. Scared, she was immobilized. She knew of her fathers capacity to kill her because she had previously witnessed how he almost killed their mother when the latter was still with them before she left for Hongkong. Appellant completely undressed himself, started to kiss her, and forcibly inserted his sexual organ into hers and did a pumping motion. Before leaving her to sleep on his bed, he threatened to kill her if she would report what he did to her, warning that he would also include the one whom she would report to. She felt pain on her body, including her private parts. She also saw a small quantity of blood coming from her vagina. "On April 28, 1997, appellant repeated the dastardly act on his daughter, Cristina. This time, he already held the samurai, pointing it towards her neck when he went near her. Afraid, she had to allow appellant to undress her without a struggle and he successfully had sexual intercourse with her. He thereafter regularly raped her about 3 to 4 times a week, usually after a one (1) day interval. This went on until Cristina became pregnant and gave birth to a baby boy on January 23, 1998. The baby boy was named Christian."4 (Citations omitted) In an Information dated February 20, 1998, appellant was charged with multiple rape, defined and penalized under Republic Act No. 8353. He allegedly committed the crime as follows: "That sometime in the third week of April, 1997, in the evening and several times thereafter, in Barangay Tuao South, Municipality of Bagabag, Province of Nueva Vizcaya, Philippines and within the jurisdiction of this Honorable Court, the above-named accused, with lewd design, with the use of force, threat and intimidation, did then and there wilfully, unlawfully and feloniously have carnal knowledge of his daughter Cristina Galvez y Tomboc, 16 years old at the time, against the latters will and without her consent, to her damage and prejudice including her parents."5

During the arraignment, read and explained to appellant in Ilocano a dialect he spoke and understood were the charges for ten counts of rape. Assisted by Atty. Renato Mercado, he pleaded not guilty to the charges.6 However, on May 14, 1998, appellant, this time with the assistance of Atty. Ruby Rosa Espino,7 changed his plea to that of guilt. In accordance with the RTCs Order dated May 14, 1998, an inquiry into the voluntariness and full comprehension of his plea was conducted. After hearing evidence for the prosecution, the lower court rendered the assailed Decision. Appellant did not present any evidence on his behalf. Neither did his counsel present in his Brief any counter-statement of the facts.8 Ruling of the Trial Court The automatically appealed Decision states that the trial was conducted pursuant to People v. Alicando,9 which held that "a conviction in capital offenses cannot rest alone on a plea of guilt after a free and intelligent plea of guilt, the trial court must require the prosecution to prove the guilt of the appellant and the precise degree of his culpability beyond reasonable doubt."10 As appellant absolutely refused to offer any evidence in his own favor, the trial court decided the case solely on the basis of the evidence presented by the prosecution. According full faith and credence to the testimony of the victim, the RTC found that she had been raped by her father. It based its conclusion on the following: (1) she cried several times during her testimony; (2) no daughter, especially one as young as she was, would have charged her own father with so serious an offense that prescribed the death penalty, if she had not indeed been raped; (3) appellant was accorded the chance to refute the claim of his daughter, yet he did not; besides, a young girl like her would not have submitted to the advances of her own father, unless these were made through force and intimidation; and (4) appellant entered a plea of guilt.11 However, the RTC convicted him of only one count of rape perpetrated in the third week of April, 1997, because the Information had alleged only one incident of the crime. The allegation that the victim was raped several times after the third week of April 1997 was deemed "too indefinite to give the accused the opportunity to prepare his defense." Hence, this automatic appeal.12 The Issue Appellant prays for the remand of the case to the court of origin for proper arraignment and trial based on this sole assignment of error: "The court a quo gravely erred in not applying the safeguards set forth under Rule 116, 1985 Rules on Criminal Procedure."13 This Courts Ruling

We agree with appellant his plea of guilt was improvident. Main Issue: Proper Procedure When the Accused Pleads Guilty in a Case Involving a Capital Offense Citing People v. Bello,14 appellant argues that the trial court failed to observe the mandatory procedure for accepting a positive plea to a charge punishable by death. The stringent procedure governing the reception of a plea of guilt, especially in a case involving the death penalty, is imposed upon the trial judge in order to leave no room for doubt on the possibility that the accused might have misunderstood the nature of the charge and the consequences of the plea.15 In People v. Aranzado,16 the Court, citing Section 3, Rule 11617 of the Rules of Court, set the following guidelines for receiving a plea of guilt in a case involving a capital offense: "(1) The court must conduct a searching inquiry into the voluntariness and full comprehension of the consequences of the plea; (2) The court must require the prosecution to present evidence to prove the guilt of the accused and the precise degree of his culpability; and (3) The court must ask the accused if he desires to present evidence in his behalf and allow him to do so if he desires."18 Moreover, as prescribed in Aranzado, the searching inquiry to be conducted by the trial court should consist of the following: "(1) Ascertain from the accused himself (a) how he was brought into the custody of the law; (b) whether he had the assistance of a competent counsel during the custodial and preliminary investigations; and (c) under what conditions he was detained and interrogated during the investigations. These the court shall do in order to rule out the possibility that the accused has been coerced or placed under a state of duress either by actual threats of physical harm coming from malevolent or avenging quarters. (2) Ask the defense counsel a series of questions as to whether he had conferred with, and completely explained to, the accused the meaning and consequences of a plea of guilty. (3) Elicit information about the personality profile of the accused, such as his age, socioeconomic status, and educational background, which may serve as a trustworthy index of his capacity to give a free and informed plea of guilty. (4) Inform the accused [of] the exact length of imprisonment or nature of the penalty under the law and the certainty that he will serve such sentence. Not infrequently indeed an accused pleads guilty in the hope of a lenient treatment or upon bad advice or because of promises of the

authorities or parties of a lighter penalty should he admit guilt or express remorse. It is the duty of the judge to see to it that the accused does not labor under these mistaken impressions. (5) Require the accused to fully narrate the incident that spawned the charges against him or make him reenact the manner in which he perpetrated the crime, or cause him to supply missing details of significance."19 Appellants re-arraignment on May 14, 1999 miserably fell short of these guidelines, as shown by the pertinent portion of the transcript of stenographic notes, which we quote hereunder: "PROS. CASTILLO: By way of dialogue with the defense counsel the accused is willing to enter a plea of guilty for the ten (10) counts of rape, your Honor. COURT: Why dont you arraign him? Alright, the previous plea of not guilty is now withdrawn to give way to the plea of guilty by the accused for 10 counts of rape but as the Court had already observed we have to conduct the re-arraignment of this case. Will you please arraign him. (The accused was arraigned by reading to him the Information in Ilocano dialect which the accused speaks and understands.) INTERPRETER: The accused pleaded guilty. COURT: I would like to ask the accused if he understands from his counsel, the circumstances in this case because the victim here is his own daughter and she is below 18 years of age. In accordance with the heinous offense law, the Court will have to impose on him the penalty of death. Is this clear to the accused? At any rate we will conduct a trial to find out if there is sufficient evidence to convict you so that your rights will be protected you are given a chance to prove your innocence latter to refute the evidence of guilt beyond reasonable doubt."20 It is clear from the foregoing that the trial judge did not conduct a "searching inquiry" into the voluntariness of appellants plea of guilt and full comprehension thereof. He asked no questions on the subjects mentioned in Aranzado. His purported compliance with Alicando was more like a monologue, or a warning at best, rather than a searching inquiry. He did not inquire into appellants personality profile age, socio-economic status or educational background.21 His Honor did not even require an answer to his question on whether appellant realized that the death penalty would result from the latters plea. No response from appellant was given or recorded.

Moreover, there is no showing that the lawyer explained to appellant the consequences of the latters plea probable conviction and death sentence. Equally important, the trial judge should have asked why the plea of appellant was changed. The former obtained none of the information required in Aranzado. Hence, there is no basis to conclude that the latter voluntarily and intelligently pleaded guilty to the charges against him. In Bello, the Court remarked that there were cases when the accused would plead guilty in the hope of a lenient treatment or because of promises from the authorities or parties that an expression of remorse would result in a lighter penalty.22 Where the punishment to be inflicted is death, it is not enough that the information be read to the accused or even translated into the dialect they speak. This is because the implementation of such penalty is irrevocable, and experience has shown that innocent persons have at times pleaded guilty.23 The trial court must avoid improvident pleas of guilt, since the accused might be admitting their guilt and thus forfeiting their lives and liberties without having fully understood the meaning, significance or consequences of their pleas.24 What is apparent here is that appellant was not properly advised by his counsel. In People v. Sevilleno,25 the Court remanded the case for re-arraignment of the accused who had been charged with the rape and murder of a nine-year-old girl, because his counsel had declined to present evidence for his client, banking on the mitigating circumstance of the plea of guilt. This Court clarified that under no circumstance would an admission of guilt in that case affect or reduce the death sentence because it was a single indivisible penalty which is applied regardless of any mitigating or aggravating circumstance attending the crime. In the instant case, the Court also notes that "guilty" was not the original plea of appellant; hence, careful effort should have been exerted by the court below to inquire into why he changed his plea. In addition, he refused to present evidence in his defense. This should have again prompted the trial judge to probe more deeply, following the guidelines in Aranzado. A plea of guilt is improvidently accepted where no effort is made to explain to the accused that, in a case involving a capital offense, such plea may result in the imposition of the death penalty.26 The same is true when the requirements in Aranzado are not satisfied.27 Recently, in People v. Bernas,28 the Court set aside a death sentence and remanded the case to the trial court, because the Aranzado guidelines on how to conduct a "searching inquiry" had not been followed. WHEREFORE, the automatically appealed Decision is SET ASIDE. Criminal Case No. 3299 is REMANDED to the court of origin for re-arraignment and further proceedings to be conducted with all deliberate speed, in accordance with this Decision. No costs. SO ORDERED.
CALLANTA VS. VILLANUEVA [77 SCRA 377; G.R. NOS. 24646 & L-24674; 20 JUN 1977] Thursday, February 12, 2009 Posted by Coffeeholic Writes Labels: Case Digests, Political Law

Facts:

Two complaints for grave oral defamation were

filed against Faustina Callanta. The City Judge of Dagupan City, Felipe Villanueva, denied the motions to quash the complaints. Thus, petitioner Callanta brought the suits for certiorari in the Supreme Court. Petitioner questions the validity of the issuance of warrant of arrest by respondent, arguing that the City Fiscal should have conducted the preliminary investigation. According to petitioners counsel, there was jurisdictional infirmity. After the issuance of the warrants of arrest and the bail fixed at P600, petitioner posted the bail bond, thus obtaining her provisional liberty. The City Fiscal in this case did not disagree with the judges investigation, and agreed with the complaints filed.

Issue:
given

Whether or Not petitioners contentions are to be merit.

Held:

Based on many precedent cases of the Supreme

Court, where the accused has filed bail and waived the preliminary investigation proper, he has waived whatever defect, if any, in the preliminary examination conducted prior to the issuance of the warrant of arrest. In the case at bar, it is futile for the petitioner to question the validity of the issuance of the warrant of arrest, because she posted the bail bond. Petitioner also erred in arguing that only the City Fiscal can conduct a preliminary investigation.

According to the Charter of the City of Dagupan, the City Court of Dagupan City may also conduct preliminary investigation for any offense, without regard to the limits of punishment, and may release, or commit and bind over any person charged with such offense to secure his appearance before the proper court. Petition for certiorari is denied. Restraining order issued by the Court is lifted and set aside.

Callanta v Villanueva (77 SCRA 377)


November 10, 2010

Facts: 1 Respondent Judge denies the motions to quash two complaints for grave oral defamation against petitioner. 2 Petitioners contest the validity of the arrest warrants issued by respondent Judge on the ground that it should have been the City Fiscal who should have conducted the preliminary investigation. 3 After the warrants were issued however, petitioner posted bail thus obtaining her provisional liberty. Issue: WON warrant of arrest may be invalidated after posting of bail Held: NO 1 Zacarias v. Cruz: Posting of a bail bond constitutes waiver of any irregularity attending the arrest of a person, estops him from discussing the validity of his arrest. 2 People v. Obngayan: Where the accused has filed bail and waived the preliminary investigation proper, he has waived whatever defect, if any, in the preliminary examination conducted prior to the issuance of the warrant of arrest. PETITION DISMISSED

CALLANTA v VILLANUEVA 77 SCRA 377FERNANDO; June 20, 1977 NATURE Original petitions in the Supreme Court, certiorari with preliminary injunction FACTS - Judge Villanueva of Dagupan refused to grant the motions to quash two complaints for oral defamationagainst Callanta.- Callantas counsel argued that there w as an issuewith regard to the validity of Villanuevas issuance of the warrants of arrest on the ground that it should h a v e b e e n t h e C i t y F i s c a l w h o c o n d u c t e d t h e preliminary investigation.- After the warrants were issued (with bail pegged atP600), Callanta posted the required bail bonds and was granted her provisional liberty.- T h e C i t y F i s c a l h a d m a n i f e s t e d h i s i n t e n t t o prosecute the case.- February 25, 1965 After the Court had conductedp r e l i m i n a r y i n v e s t i g a t i o n a n d h a d a c q u i r e d jurisdiction over the case, the Court referred the caseto the Fiscal.- March 4, 1965 The arraignment was postponed because the Fiscal was still doing his investigation.- I n t h e p r o c e e d i n g s o f A p r i l 2 0 , 1 9 6 5 , t h e F i s c a l e n t e r e d h i s a p p e a r a n c e f o r t h e g o v e r n m e n t a n d manifested that he was ready for trial. ISSUE WON Callanta can contest the validity of his arrest HELD NO Ratio Posting of a bail bond constitutes waiver of any irregularity attending the arrest of a person andestops him from discussing the validity of his arrest. Reasoning - In the case of Luna vs. Plaza , the Court held that where petitioner has filed an application for bail andw a i v e d t h e p r e l i m i n a r y i n v e s t i g a t i o n p r o p e r , h e waived his objection to whatever defect, if any, in thepreliminary examination conducted, prior to theissuance of a warrant of arrest. - This doctrine has been upheld in a number of casesincluding People vs. Olandar, Zacarias vs. Cruz,Bermejo vs. Barrios, People vs. La Caste, Manzano vsVilla andPeople vs. Obngayan which stated that w h e r e t h e a c c u s e d h a s f i l e d b a i l a n d w a i v e d t h e p r e l i m i n a r y investigation proper, he has waived w h a t e v e r d e f e c t , i f a n y , i n t h e p r e l i m i n a r y examination conducted prior to the issuance of the warrant of arrest.- T h e c i t y f i s c a l h a d b e e n q u i t e a c t i v e i n t h e investigation and in the prosecution of the accused.It was he who manifested his readiness to appear inthe trial. Obiter - With regard to the issue of whether or not the onlyp e r s o n v e s t e d w i t h a u t h o r i t y t o c o n d u c t a preliminary investigation is the city fiscal, the Charterof the City of Dagupan provides that the City Courto f D a g u p a n C i t y m a y a l s o c o n d u c t p r e l i m i n a r y investigation for for any offense, without regard tothe limits of punishment and may release or commitany person charged with such offense to secure hisappearance before the proper court. Dispositive W H E R E F O R E , t h e s e p e t i t i o n s f o r certiorari are dismissed. The restraining order issuedby this Court is lifted and set aside. Costs against petitioner. SEPARATE OPINIONAQUINO [concurring] - S e c . 7 7 o f t h e D a g u p a n C i t y c h a r t e r e x p r e s s l y empowers its city court (formerly municipal court) toconduct preliminary investigation for any offense, without regard to the limits of punishment.- E v e r y j u s t i c e o f t h e p e a c e , m u n i c i p a l j u d g e (meaning city judge), city or provincial fiscal, shallhave authority to conduct preliminary examination orinvestigation in accordance with these rules of alloffenses alleged to have been committed within hismunicipality, city or province, cognizable by theCourt of First Instance (Sec. 87 of the Judiciary Law and Sec. 2, Rule 112)

G.R. No. 103964 August 1, 1996 PEOPLE OF THE PHILIPPINES, plaintiff-appellee, vs. NARCISO NAZARENO, RAMIL REGALA, ORLANDO HULAR and MANUEL LAUREAGA, accused-appellants.

MENDOZA, J.:p This is an appeal from the decision, 1 dated May 28, 1991, of the Regional Trial Court of Makati (Branch 136), finding accused-appellants Narciso Nazareno and Ramil Regala guilty of murder for the killing of Romulo "Molet" Bunye II in Muntinlupa, Metro Manila on December 14, 1988 and sentencing them to suffer the penalty of reclusion perpetua. In addition, the two were ordered to pay jointly and severally to the heirs of the deceased the amount of P50,000.00. Two others, accused with them, Manuel Laureaga and Orlando Hular, were acquitted. The evidence for the prosecution shows that on December 14, 1988, between 8:00 a.m. and 9:00 a.m., Romulo Bunye II took a tricycle (referred to in the record as "stainless" tricycle evidently because its body was made of stainless steel), which was driven by Fernando Hernandez. Unknown to Bunye was that two men were waiting outside his house and that the two hailed another tricycle in order to follow him. Bunye alighted at the corner of T. Molina and Mendiola Streets in Alabang, Muntinlupa and crossed to the left side of the street . Shortly after, the tricycle, driven by Rogelio de Limos, arrived and stopped in front of Hernandez's "stainless" tricycle. One of the men jumped out of the tricycle and shot Bunye at the back of the head. When Bunye fell face down, the assailant fired another shot at Bunye's head. Then, the other man approached Bunye and shot him also in the head. Rogelio de Limos and Fernando Hernandez, the tricycle drivers, executed sworn affidavits relating what they had witnessed. 2 The two described the assailants and stated that they could recognize the killers if they saw them again. There was another witness, a woman, who was also a passenger of the "stainless" tricycle on which Bunye rode but her identity had remained unknown. The autopsy report on the victim showed that he died of gunshot wounds in the head. 3

On December 28, 1988, Ramil Regala, Narciso Nazareno, Orlando Hular and Manuel Laureaga were arrested. Regala and Nazareno were put in a police lineup. They were identified and pointed to as the assailants by the tricycle drivers Hernandez and de Limos. Hernandez and de Limos executed additional sworn affidavits. Ramil Regala executed affidavits, dated December 28, 1988 4 and January 2, 1989, 5 admitting participation in the slaying of Bunye and pointing to Narciso Nazareno and a certain Rey Taling as his co-conspirators. He claimed that they had been hired by Orlando "Boy" Hular to kill the victim and told that they would be paid P30,000.00 by Manuel Laureaga. His affidavits were corroborated by Orlando Hular who, in an affidavit, executed on the same day, December 28, 1988, 6 stated that it was Laureaga who wanted Bunye killed, apparently in connection with Bunye's job as administrator of the public market in Alabang. However, Regala and Hular subsequently recanted. Regala claimed that he had been tortured. 7 On the other hand, Hular claimed that, although he was not tortured, he admitted to the crime and signed the affidavit because he was afraid he would also be tortured. 8 Narciso Nazareno also claimed to have been tortured to admit to the crime but refused to sign any written statement. 9 The trial court ruled the confessions of Regala and Hular to be in admissible. However, it held Regala and Nazareno guilty on the basis of their positive identification by Hernandez and de Limos during the police line-up on December 28, 1988 and their testimony in court. The trial court stated:
As between the aforecited testimonies of Rogelio de Limos and Hernandez on one hand and the testimonies of Narciso Nazareno and Ramil Regala on the other, the Court would place its reliance on the testimonies of the prosecution witnesses, because firstly, there is no showing in the record that Rogelio de Limos and Fernando Hernandez are manufactured evidence. As a matter of fact, none of the defense witnesses had ever made such an imputation; and neither did the defense lawyers do so in their extensive memoranda. Secondly, it is a well-settled doctrine in this jurisdiction that as between positive testimonies and denials, the Court should place more weight on the former (People v. Mostoles, Jr. 124 SCRA 906). Thirdly, the testimonies of Narciso Nazareno and Ramil Regala are in the nature of alibis, and it is also settled that because they can easily be concocted, the Courts should exercise extreme caution in accepting them as defense (People vs. Bagsica, 65 SCRA 400).

Orlando Hular and Manuel Laureaga were acquitted for lack of evidence against them. 10 Hence this appeal by Nazareno and Regala. In his brief, accused-appellant Narciso Nazareno assigns the following errors:
THE LOWER COURT ERRED IN FAILING TO CONSIDER THE VIOLATION OF THE ACCUSED-APPELLANT'S CONSTITUTIONAL RIGHT TO DUE PROCESS AS A FATAL FLAW IN HIS PROSECUTION AND SUBSEQUENT CONVICTION.

THE LOWER COURT ERRED IN FAILING TO ACQUIT THE ACCUSED-APPELLANT ON REASONABLE DOUBT.

Accused-appellant Ramil Regala, on the other hand, contends:


THE TRIAL COURT ERRED IN NOT CONSIDERING THE UNLAWFUL ARREST OF RAMIL REGALA AS A GROSS VIOLATION OF HIS CONSTITUTIONAL RIGHT TO DUE PROCESS. THE TRIAL COURT ERRED IN NOT CONSIDERING THE TESTIMONY OF THE EXPERT WITNESS FROM THE NATIONAL BUREAU OF INVESTIGATION IN DETERMINING THE PROBABILITY OF GUILT OF APPELLANT. THE TRIAL COURT ERRED IN RELYING ON THE INCREDIBLE TESTIMONIES OF FERNANDO HERNANDEZ AND ROGELIO DE LIMOS IN CONVICTING HEREIN APPELLANT.

We have reviewed the record and the evidence, and we find accused-appellants' contentions to be without merit. First. Accused-appellants claim that their arrests without warrant were illegal and justify the nullification of the proceedings of the trial court. The contention is untenable. The warrantless arrest of accused-appellant Narciso Nazareno was upheld by this Court in 1990 in a petition for habeas corpus. It appears that, on January 9, 1989, Nazareno filed a motion for bail. 11 As the trial court denied his motion, a petition for habeas corpus was filed on his behalf with this Court. It was alleged that Nazareno's arrest was illegal because it was made without warrant fourteen days after the killing of Romulo Bunye II. This Court dismissed the petition in its decision of July 9, 1990. 12 He filed a motion for reconsideration which the Court also denied on the ground that the warrantless arrest was in accordance with Rule 113, 5(b) of the Revised Rules of Criminal Procedure. 13 The question which Nazareno raises has thus been settled long ago in a final decision of this Court. Furthermore, Nazareno and Regala waived objections based on the alleged irregularity of their arrest, considering that they pleaded not guilty to the charges against them and participated in the trial. Any defect in their arrest must be deemed cured when they voluntarily submitted to the jurisdiction of the court. 14 For the legality of an arrest affects only the jurisdiction of the court over the person of the accused. 15 Consequently, if objections based on this ground are waived, the fact that the arrest was illegal is not a sufficient cause for setting aside an otherwise valid judgment rendered after a trial, free from error. 16 The technicality cannot render the subsequent proceedings void and deprive the State of its right to convict the guilty when all the facts on record point to the culpability of accused. 17 Second. Accused-appellants argue that the trial court erred in giving credence to the testimony of Hernandez and de Limos. Accused-appellant Nazareno claims

that the decision of the trial court does not contain an analysis of the testimonies of Hernandez and de Limos and suggests that the killing of Bunye was executed by professionals and not by a simple fruit vendor. Nazareno claims that the witnesses were reluctant, evasive and fearful and that they never had the opportunity to fully observe the incident as there was traffic and they merely had a side view of the assailants. Nazareno claims that the evidence of torture and maltreatment and the other circumstances were indications of manipulation and manufacture of evidence to frame him. 18 Similarly, accused-appellant Regala claims that the testimonies of Hernandez and de Limos were confused and confusing. He then suggests that the manner the killing was perpetuated shows that it was done by professional assassins. 19 These arguments are without merit. Far from being confused, the testimonies of Hernandez and de Limos were straightforward and unwavering and justified the trial court in giving them full faith and credit. The accused-appellants were positively identified by Hernandez and de Limos under circumstances which were ideal for identification. The incident happened in daylight and only two meters away from them. 20 They did not only see the assailants but they also witnessed the whole incident. The testimonies of Hernandez and de Limos during direct and cross examinations corroborate each other on the material facts. A summary of Hernandez's direct examination 21 reveals that on December 14, 1988, between 8:00 to 9:00 in the morning, a woman took a ride in his tricycle, followed by a man whom he identified as Molet Bunye, who asked to be taken to Purok 6, T. Molina St. Upon reaching his destination, Bunye alighted and walked across the street. Hernandez said that just then he noticed a man approach Bunye, point a gun and fire at him. He saw the face of the assailant. He then saw the assailant position himself near Bunye's head and fire another shot when Bunye fell. Hernandez said he got confused and afraid and, as his other passenger was screaming, he tried to turn his tricycle around to leave. As he was doing so, he noticed accused-appellant Ramil Regala on his right approach Bunye and fire a shot at the victim. When asked if he could identify the assailants, Hernandez answered "yes." When asked to point to the assailants, he identified accusedappellant Narciso Nazareno as the first one who shot Bunye and Ramil Regala as the second one. In his cross-examination, defense counsel tried to show that Hernandez did not see what really happened. This is not so. His testimony on cross examination, slightly edited, follows: 22
Q On Dec. 14, 1988, you were driving your tricycle. Is that correct? A Yes, sir. Q You know that Rogelio de Limos was also driving his tricycle on the morning of Dec. 28, 1988?

A Yes, sir. Q While you were driving your tricycle on that faithful morning, who was ahead you or Mr. de Limos? A I. Q After you took the woman and the man who was shot, did you know that Mr. de Limos was following you? A I don't know. Q How do you know that you were ahead? A I learn[ed] it later on when Mr. de Limos overtook my tricycle? Q In what place, when [did] Mr. de Limos overtake your tricycle? A Also at T. Molina Street, purok 6. Q When Mr. de Limos overtook your tricycle, where you on stop position or were you still running? A Already stopped. Q In other words you stopped your tricycle and that was the time when Mr. de Limos went ahead or overtook your tricycle? A Yes, sir. Q To what side of you, left side or right side, did Mr. de Limos pass? A To my left. Q He stopped his tricycle in front of yours? A Yes, sir. Q When Mr. de Limos stopped his tricycle in front of yours, how far was his tricycle from the front side of your tricycle? A Almost touching the front side of my tricycle. Q Can you tell us, how many minutes after you have stopped when Mr. de Limos overtook you? A I don't remember. Q Maybe five (5) minutes after you have stopped that Mr. de Limos arrived?

A I can not estimate. Q Was it very short period or was it in reasonable period of time after you stopped when Mr. de Limos arrived? A Only for a while. Q When Mr. de Limos stopped his tricycle, what was the position of his tricycle in relation to yours? Was it directly in front of you or was it towards your left side? A His tricycle stopped directly in front of me, but a little sway to the right. Q Would you say that the distance of your tricycle when both of you stopped was about one (1) foot only? A About two (2) to three (3) feet. Q Is it not correct to say that the tricycle touched each other at that time? A I was surprised when he suddenly overtook me. Q So your previous statement is not correct that the tricycle almost touched each other, because they were between two (2) to three (3) feet? A Because I don't remember the distance because the incident happened a long time ago. Q When the two (2) tricycles [were] already on stopped position, was there enough space for a person to pass? A Yes, sir. Q Your two (2) passengers, the man and the woman who alighted your tricycle first? A The man (Bunye). Q After Molet Bunye alighted from your tricycle, he walked to the left, crossed the street crossing in front of your tricycle? A After Molet Bunye alighted from my tricycle, he walked in front of me and paid his fare. After that he walked across the street towards the left. xxx xxx xxx Q Was Molet Bunye able to reach the other side of T. Molina Street? A Yes, sir.

Q This tricycle of Mr. de Limos, tell us when the tricycle arrived was it after Molet Bunye paid his fare to you or after? A While Molet Bunye was paying the fare, that was the time when Mr. de Limos overtook me. Q Molet Bunye was still near you or beside you when the tricycle of Mr. de Limos arrived? A Yes, sir. Q After paying his fare, you said, Molet Bunye walked to the other side of the street. How long did it take Molet Bunye to go to the other side of the street? A I don't remember. Q When the tricycle of Mr. de Limos overtook you, you did not know that that was Mr. de Limos? A I noticed him when he was already in front of me, and he was at the back. Q When Molet Bunye paid his fare to you, you back-tract your tricycle because you were preparing to turn around? A Yes, sir. I back-up my tricycle a little so that I'll be prepared to leave the place. Q Were you supposed to leave the place after embarking Molet Bunye, because you were to bring another lady to another destination? A Yes, sir. Q When you back-up to which direction were you looking? A To the direction of Molet Bunye. I was looking to the direction of Molet Bunye, because I was about to call him because he still had change, and I was about to return it back to Molet Bunye. Q When you were backing-up your tricycle preparing to leave the place, you were looking at Molet Bunye? A Yes, sir. Q Did you not look behind, so that you will be able to see him? A I look[ed] at my side mirror. Q Did you call Molet Bunye, so that he can hear you, because you wanted to give his change?

A Yes, sir, but he did not hear me. Q How far was he when you call[ed] him? A About three (3) to four (4) meters. Q And he was walking away from you? A Yes, sir. xxx xxx xxx Q How much [did] Molet Bunye give you, when he paid his fare to you? A Two pesos (2.00). Q Why did you not give his change when he was still near you? A Because he was in hurry. Q In other words, when he alighted he did not approach you anymore but he just handed to you his two (2.00) pesos? A Yes, sir, he handed his two (2.00) pesos to me. He said "BOY BAYAD KO." (Witness demonstrating by stretching his left hand forward.) Q And you immediately received the money? A Yes, sir. Q And Molet Bunye proceeded to walk away from you? A Yes, sir. Q When was it when the tricycle of Mr. de Limos overtook you, was it when Molet Bunye paid his fare to you or after Molet Bunye had paid? A I can not remember. Q You said that the first person who shot Molet Bunye was Narciso Nazareno, is that correct? A Yes, sir. Q Did you see that? A Yes, sir. I saw that the gun was poked at the head. Q And you heard the shot? A Yes, sir.

Q Can you demonstrate how the gun was pointed to the head of Molet Bunye? A Like this (witness demonstrating stood up stretching his right hand forward as if holding a gun and poked it at the back of the head of the interpreter). Q Assuming that the Court Interpreter is Molet Bunye, please demonstrate which part of the head was the gun pointed [at]? A (Witness stood up demonstrating by pointing somewhere above the neck or maybe center of the neck of the interpreter.) Q Can you tell us, how tall is Molet Bunye in relation to our Court Interpreter here? A Shorter than the interpreter. Q When the gun which you said was pointed at the head of Molet Bunye, Molet Bunye was walking away from you? Is that correct? A Yes, sir. Q He was walking. Right? A Yes, sir. Q The man whom you said pointed a gun at the head of Molet Bunye, he was also walking away from you, is that correct? A Yes, sir. Q He was not walking towards you? A No, sir. Away from me. Q So that you were from the position from where you were, you were still in your tricycle? A Yes, sir. Q From that position you can see Molet Bunye and the person whom you said fired a shot? A Yes, sir. Q The man whom you said fired the first shot was also walking when he fired the first shot? A The man who was holding a gun stopped first and he fired it.

Q Tell us how far was this man who fired the first shot to Molet Bunye when the first shot was fired? A About one (1) meter. Q What did you see after the first shot was fired? A He positioned himself near the head of Molet Bunye, and he was already facing me and he fired another shot. Q At that time if your narration is correct Bunye was already lying on the ground? A Yes, sir. Q So the first shot was fired when Molet Bunye was walking? A Yes, sir. Q That was to the head? A Yes, sir. Q Do you know what part of the body of Molet Bunye was hit by the shot? A I don't remember what part of the body was hit, but he fired it at the head. xxx xxx xxx Q All the while when the gun was pointed at the head of Molet Bunye, and it was fired, Molet Bunye was lying on the ground, and the man fired another shot. During this period were you looking at Molet Bunye? A Yes, sir, because I was shocked, surprised at what had happened so I looked at Molet Bunye. Q Can you tell us in seconds or minutes, how long did this happening last? A I can not estimate. Q All the while you were looking at Molet Bunye while this was happening? A Yes, sir. Q Where you then worried about the change of Molet Bunye while you were looking at him? A Yes, sir. It is like that.

Q After the second shot did you turn your tricycle and went home? A Yes, sir. Q And you went home? A I also noticed that the other person was approaching. Q How far where you positioned from Molet Bunye when he fell to the ground? A About three (3) to four (4) meters. Q Do you know the width of T. Molina Street? A No, sir. Q At that time you claimed that you were at the right side of the road? A Yes, sir. Q And the incident happened at the edge of the left side? A Yes, sir. Q And you said that the distance between your tricycle and the edge of T. Molina is only four (4) meters? A My estimate is more or less three (3) to four (4) meters. Q The second person, did you see him approach Molet Bunye? A Yes, sir. Q What did he do with Molet Bunye? A I also noticed that the second person also pointed a gun at the head of Molet Bunye. Q The second man did he fire a shot? A Yes, sir. Q To the head of Molet Bunye? A Yes, sir. Q Do you know if he hit the head? A Yes, sir.

Q How about the face? Do you know if he hit the face? A It was pocked very near the head.

Hernandez's testimony was corroborated by Rogelio de Limos. De Limos testified that on December 14, 1988, between 8:00 to 9:00 in the morning, two men, standing in front of Bunye's house on Ilaya Street, hailed his tricycle. He identified in court the two as Narciso Nazareno and Ramil Regala. De Limos said that he was told by Nazareno to follow the "stainless" tricycle ahead and that upon overtaking it as it stopped at the corner of T. Molina St. and Mendiola St., he also stopped his vehicle. One of the passengers, Nazareno, jumped out of the tricycle, pulled something from his waist and fired at Bunye. When Bunye fell down, Nazareno fired another shot at the head of Bunye, after which Ramil Regala got off the tricycle, approached Bunye and also shot him in the head. Accused-appellants then boarded the tricycle and ordered him to take them to the rotunda, where they alighted from the tricycle and fled. De Limos was unshaken by questions asked during the cross examination. 23 His testimony, slightly edited, follows:
Q When you were operating your tricycle last Dec. 14, 1988, you were flagged by two (2) persons who became your passengers. Tell us where is this stainless tricycle at the time you were flagged down in relation to your tricycle? A The stainless tricycle was already running. Q When you stopped your tricycle and you picked up these two (2) persons, how far away was the stainless tricycle from you? A I don't remember. Q Can we say about twenty (20) meters away and still running at the time you stop your tricycle? A Not more than twenty (20). Q What would be your estimate if it is less than twenty (20) meters away? A I am not sure of my estimate. Q When these two (2) passengers got in your vehicle and at the moment you started your tricycle, how far away is this first tricycle from you? A It is not far. Q Give us your estimate in meters the distance between you when you started your tricycle and the stainless tricycle that you were to follow?

A About more or less twenty (20) meters. Q Were their other vehicles including tricycle[s] that were between you and the stainless tricycle when you commence running? A I don't remember, sir. Q So that the space between you and the first tricycle was totally clear of vehicle? A Yes, sir. Q How many meters did this first tricycle travel from where you picked up your passengers until it stopped and unloaded its passengers? A More or less 200 meters. Q What street were you traveling at that time? A Ilaya Street, Alabang Muntinlupa. Q When you were travelling along Ilaya Street following this stainless vehicle were you exactly behind? A Yes, sir. Q How far away from your starting point where you able to approach the vehicle that you are going to follow? A Near the corner of Mendiola and Timolin Street. Q When the first tricycle already stopped you were able to get near that vehicle? A Yes, sir. Q But at any time when you were running and traveling along Ilaya Street you were able to get [to] this stainless tricycle? A No, sir. Q When your first passenger alighted from your vehicle you were still running . . . your vehicle was still running. Is that correct? A Yes, sir. Q Where was this stainless vehicles situated at that moment when your first passenger alighted from your vehicle? A The tricycle was behind me.

Q You mean to say that you overtook the stainless tricycle? A Yes, sir. Q And the passenger of the stainless vehicle got off? A Yes, sir. Q Who got off their respective vehicle[s] first, your passenger or the passenger of the stainless tricycle? A The passenger of the stainless tricycle. Q When the passenger of the stainless tricycle got off, the tricycle was on your right side. Is that correct? A No, sir. The stainless tricycle was behind me. Q Directly behind your tricycle? A Yes, sir. Q How far away was it from you? A About one (1) foot behind me. Q At that moment when your first passenger alighted from your tricycle where was the victim in relation to where you were situated at that moment? A He was in front of me walking. Q So you can see him directly in front of you walking? A Yes, sir. Q Your tricycle was on the right portion of the left side of the street. Is that correct? A Yes, sir. Q And the victim was walking also on the right portion of the street of the road? A At the left side. Q How far away from you was the victim when you said he was shot for the first time? A More or less two (2) meters.

Q Will you please clarify. Was he walking in front of you or to the left of you when he was first shot? A To the left. Q At that moment, meaning, when the victim was first shot where was the stainless tricycle? A He was still behind me. Q How did you know that the vehicle was still behind you? A Because I saw and I could not leave the place. Q When did you see the stainless tricycle, was it still behind you in relation to the first shot? A At the time when I heard the first shot. Q When you heard the first shot. You want to impress [to] us that you looked back and you saw the stainless tricycle still there behind your tricycle? A Because the tricycle could not run or move. Q But when the first shot was fired you did not look back? A I did not. Q So you did not really know that the tricycle was still there? A Because the tricycle was "NAKATOTOK" behind me and I could not move my tricycle. Q It was only your impression that the stainless tricycle was still there but you did not see it actually? A No, sir. I see. Q When did you see it? A At that time because I could not overtake coz of that incident. Q After the first shot, did you see the stainless tricycle still there? A Yes, sir. Q You mean you looked back? A Yes, sir.

Q Why did you look back? A Because the passenger of the stainless tricycle scramble[d]. Q After the second shot was the stainless tricycle still there? A Yes, sir. Q Did you look back? A Yes, sir, through my side view. Q Did you look back and see that stainless tricycle still there after the second shot? A I saw the driver of the stainless tricycle moving his tricycle. Q You said there were three (3) shots fired. After the third shot where was the stainless tricycle? A After the third shot, the driver of the stainless tricycle was still moving his tricycle back and forth to get out of the place. Q Was he able to get out? A Yes, sir. Q How long after the third shot was he able to get out from where he was? A I don't remember. Q Was this stainless tricycle blocked by your tricycle so that he can not get out. A Yes, sir. Q How about to his rear was he blocked by any other vehicle? A No, sir. Q Of your knowledge he was free to move out by backing away from your vehicle? A Yes, sir. Q You claimed that immediately after the second shot was fired, this stainless tricycle was already moving his vehicle back and forth to get out of the place. Is that correct? A Yes, sir.

Q How far was this stainless vehicle from the victim when the latter fell down the street after the first shot? A I don't remember. Q When your second passenger got off you did not see him get off? A I did not. Q You felt movement of your tricycle? A Yes, sir. Q You claim that your two (2) passengers rode your vehicle again after the shooting. Is that correct? A Yes, sir. Q Who boarded your vehicle first, the first one who alighted or the second one who alighted? A I don't remember. Q How far was it after the third shot was fired that these passengers boarded your vehicle again? A Minutes only. Q How many minutes? A More or less five (5) minutes. Q What were their instructions to your when they boarded? A Before answering your question I made [a] mistake on the preceding question. Not minutes only seconds. Q Will you now answer my question? A They told me to bring them to "Rotonda." Q In what dialects or language was the instruction given to you? A Tagalog. Q Was that all they said to you during that occasion when they boarded your vehicle for the second time? A Yes, sir.

Q From the scene of the shooting to where you discharged your passengers, how far was it? A I can not estimate the distance. Q You were driving your tricycle along that road for many years already? A Yes, sir. Q Why did you say that you can not estimate the distance between these two places? A I am not sure. Q How many minutes did it take you to drive them to the place where they alighted from your vehicle? A Few minutes only. Q How many minutes? A Seconds only. Q How many seconds? A More or less ten (10) seconds. Q Did your passenger pay you his fare? A No, sir. Q Did you not run after him to be able to collect the fare? A No, sir. Q When your two (2) passengers alighted, where did you go next? A I went home.

As we have held in other cases, 24 testimonies given in simple, straightforward manner, giving details of the incident that could not have been merely concocted, indicate sincerity in the narration of events of the incident and truth as to what actually happened, specially, if those giving them have not been shown to have any improper motive to testify falsely against the accused. The weight to be given to testimonies of such witnesses depends chiefly upon their observation and means of knowing the facts testified to by them. 25 Accused-appellants point to inconsistencies in the testimonies regarding distances and time lapse as an indication that the testimonies are lacking in veracity. These are inconsistencies on minor points which do not affect the truth of the testimonies. They are discrepancies to be expected from uncoached witnesses. 26 What is important is

that the testimonies corroborate each other on important and relevant details concerning the principal occurrence. 27 Third. Accused-appellant Nazareno makes much of the fact that he was recognized by Hernandez only in the police line-up. Nazareno claims that Hernandez was his classmate in the elementary grade in one subject 28 and therefore should have been able to identify and name him on December 14, 1988, when Hernandez gave his first sworn statement. It is contended that because Hernandez did not then recognize Nazareno shows that Hernandez's testimony was "manufactured." Other than Nazareno's claim, however, there is no other evidence showing that he and Hernandez were classmates. This matter was brought up only once during Hernandez's cross examination in the hearing for bail, where the following appears: 29
ATTY. MATUNOG: Q Mr. Hernandez is it correct that you signed a statement dated 28 December 1988 before Capt. Jose Manuel? FISCAL: Admitted. Q On Exhibit "C" [which] appears to be signed by you question No. 9. The question raised to you, YONG TAONG BUMARIL DITO KAY MOLET, NAKIKILALA MO BA ITO? and your answer: YAON ISA LANG HO. (affiant pointing to the person of Narciso Nazareno y Barro 22 taong gulang, binata, walang trabaho, tubong Catanduanes at kasalukuyang naninirahan sa Prk. 6-B, Alabang Muntinlupa, MM.) Is this correct? This was the answer you gave? ATTY. BAUTISTA: May it please be placed on record that the witness refused to answer. FISCAL: May I remind the witness he is under oath. COURT: MAY ANSWER.

A Nazareno (pointing to Nazareno). Yes, sir. Q Why do you know the accused Nazareno? FISCAL: The best evidence is the statement. Court: MAY ANSWER. A When I looked to my left, I saw him. Q Since when have you known the accused Nazareno? A Only on that day. Q You do not remember you were a classmate of Nazareno sometime in 1979-1980? A I cannot remember sir. I don't remember.

There is nothing in the sworn statements of Hernandez nor in his testimony in court which shows that he ever claimed to know Nazareno because they had been classmates eight years before he identified him in the police line-up. He said he only came to know Nazareno at the police line-up on December 28, 1988. He was able to identify Nazareno because he remembered his face as that of the one who shot Bunye on December 14, 1988. Indeed, Hernandez should first have been asked whether he and Nazareno had not been classmates in the elementary grades. There was no basis for defense counsel's questions premised on this fact. Be that as it may, even if they had been classmates, it is possible Hernandez had forgotten Nazareno. After all, as Nazareno claimed, they were classmates in only one subject in the elementary grades and that had been eight years before the incident. Fourth. Accused-appellant Nazareno assails the testimony of the eyewitnesses as contrary to the evidence. He contends that Bunye was 5'6" tall, while he is only 5'4". Considering the downward angle of the bullet wound at the back of Bunye's head, he theorizes that the assailant must have been taller than Bunye or Bunye must have been shot while he was in a kneeling position. This contention is without merit. In the first place, there is no basis for the claim that Bunye was 5'6" tall and Nazareno only 5'4". Bunye's cadaver was not measured. Nothing in the record shows how the figure 5'6" was reached. His death certificate, 30 the Certificate of Post-Mortem Examination 31 and the autopsy

report 32 do not contain data as to his height. It appears that the basis of accusedappellant Nazareno in claiming that Bunye was 5'6" is the testimony of the wife of the victim, Evelyn Bunye, who said during her cross examination that barefoot, the height of her husband was 5'6." Her only basis for saying so was "because he was [her] husband." 33 Nor was there evidence as to Nazareno's height. It is only in his appellant's brief that his height is said to be 5'4." This Court then, for lack of any basis, cannot give credence to this assertion. In the second place, only accused-appellants claim that, on the basis of the autopsy report, the trajectory of the bullet was "sharply downwards" 34 and that therefore the conclusion was that the gun was fired from a certain position in such a way that "(1) the gun was aimed at the head of victim; (2) the gun was positioned with the barrel pointed downwards not upwards and neither laterally; (3) the gun was held at a point approximately five (5) feet four (4) inches from the ground; and (4) was fired while at that position." 35 The autopsy report does not say what accused-appellants say. It reads: 36
Wounds Gunshot: I. Entrance, oval, edges inverted, 0.9 x 1.0 cm. in size, located at the scalp, parietal region, along the posterior median line, 13.0 cm. above and 5.0 cm. behind the left external auditory meatus, directed forwards, downwards and laterally, involving the skin and underlying soft tissues, fracturing the parietal bone along the posterior median line, perforating the left side of the parietal bone along the posterior median line, perforating the left sides, of the parietal and temporal lobes of the brain, making an Exit, roughly oval in shape, edges averted, located at the face, left side, 1.0 cm. below and 2.5 cm. in front of the left external auditory meatus. II. Entrance, oval, edges inverted, 0.9 x 1.0 cm. in size, located at the face, right side, 3.0 cm. above and 5.5 cm. in front of the right external auditory meatus, directed backwards, downwards and from right to left, involving the skin and underlying soft tissues, fracturing the right side, of the temporal bone, perforating the right side of the temporal lobe of the brain, piercing the left side of occipital lobe of the brain, with two (2) fragments lodged and recovered in that area.

There are no findings in the report regarding the relative positions of the victim and the accused-appellants. The medico-legal officer, when asked if he could determine the position of the victim from the gunshot wound located at the back, said, "Since the entry of the gunshot wound is at the back, naturally it is in the back." 37 The answer is vague and does not shed light on the positions of those involved. No further clarification was made as accused-appellants' counsels waived the right to cross examine the medico-legal officer. Evidence could have been presented showing that the road was level and not sloping or that the angle could not have been caused by the recoil affecting the assailant. As it is, the mere assertion of disparity in height cannot persuade this Court to do away with the positive identification of the accused-appellants by the witnesses.

Moreover, the position of a victim when he was shot vis-a-vis an assailant is difficult to ascertain considering the mobility of the head. 38 It is entirely possible that the victim's head in this case was slightly inclined upward. As the autopsy report shows, the trajectory of the bullet was "directed forwards, downwards and laterally" and not, as accused-appellants claim, "sharply downwards." This downward angle could be due to the bullet hitting a specific bone (scalp) causing a slight deflection. 39 On the other hand, accused-appellant Ramil Regala contends that the failure of the prosecution to investigate Rey Taling, his alleged companion, and Mang Doming, Mang Romy, and Nick Pealosa, as those who supplied the guns, raises doubt as to accused-appellant's guilt. However, the manner by which the prosecution of a case is handled is within the sound discretion of the prosecutor and the non-inclusion of other guilty parties is irrelevant to the case against an accused. 40 Fifth. Indeed, accused-appellants' defense consists of denial and alibi. Nazareno claims that at the time of the incident he was in the market selling fruits, while Regala claims that he was at home in Cavite. 41 Accused-appellant Regala failed to present any witness to corroborate his alibi, while accused-appellant Nazareno presented his mother Gloria Nazareno who testified that she asked her son to go to the market to tend their fruit stand because she wanted to stay at home to rest. 42 His mother could not thus testify positively whether he was in the market at the time of the killing. Nor were accused-appellants strangers to each other. Regala hauled fruits for Nazareno's family. 43 This circumstance rules out the possibility raised by the defense that the accused-appellants had simply been picked up at random 44 by the police. Bare denial and alibi are insufficient to overcome the positive identification given by the prosecution witnesses. As the trial court held, between the positive declarations of the prosecution witnesses and the negative statements of the accused, the former deserve more credence and weight. 45 But we find that the qualifying circumstance of evident premeditation was not proven and therefore should not be appreciated. There is neither evidence of planning or preparation to kill nor of the time when the plot was conceived. 46 However, the qualifying circumstance of treachery was correctly appreciated by the trial court to qualify the killing to murder. The testimonies of the prosecution witnesses show that Bunye was not in a position to defend himself and that the manner by which the killing was done shows that his assailants consciously and deliberately adopted a particular method or form of attack to insure the accomplishment of their purpose. 47 The attack was sudden and unexpected, giving Bunye no chance to defend himself, thereby insuring the execution of the crime without risk to its perpetrators. WHEREFORE, the judgment of the Regional Trial Court of Makati, Branch 136, being in accord with the law and the evidence, is AFFIRMED.

SO ORDERED.

G.R. No. 79543 October 16, 1996 JOSE D. FILOTEO, JR., petitioner, vs. SANDIGANBAYAN and THE PEOPLE OF THE PHILIPPINES, respondents.

PANGANIBAN, J.:p A person under investigation for the commission of an offense is constitutionally guaranteed certain rights. One of the most cherished of these is the right "to have competent and independent counsel preferably of his choice". The 1987 Constitution, unlike its predecessors, expressly covenants that such guarantee "cannot be waived except in writing and in the presence of counsel". In the present case, petitioner claims that such proscription against an uncounselled waiver of the right to counsel is applicable to him retroactively, even though his custodial investigation took place in 1983 long before the effectivity of the new Constitution. He also alleges that his arrest was illegal, that his extrajudicial confession was extracted through torture, and that the prosecution's evidence was insufficient to convict him. Finally, though not raised by petitioner, the question of what crime - brigandage or robbery was committed is likewise motu proprio addressed by the Court in this Decision. Challenged in the instant amended petition is the Decision 1 of respondent Sandiganbayan 2 in Criminal Case No. 8496 promulgated on June 19, 1987 convicting petitioner of brigandage, and the Resolution 3 promulgated on July 27, 1987 denying his motion for reconsideration. The Facts Petitioner Jose D. Filoteo, Jr. was a police investigator of the Western Police District in Metro Manila, an old hand at dealing with suspected criminals. A recipient of various awards and commendations attesting to his competence and performance as a police officer, he could not therefore imagine that one day he would be sitting on the other side of the investigation table as the suspected mastermind of the armed hijacking of a postal delivery van. Along with his co-accused Martin Mateo, Jr. y Mijares, PC/Sgt. Bernardo Relator, Jr. y Retino, CIC Ed Saguindel y Pabinguit, Ex-PC/Sgt. Danilo Miravalles y Marcelo and civilians Ricardo Perez, Reynaldo Frias, Raul Mendoza, Angel Liwanag, Severino Castro and Gerardo Escalada, petitioner Filoteo was charged in the following Information: 4

That on or about the 3rd day of May, 1982, in the municipality of Meycauayan, province of Bulacan, Philippines, and within the jurisdiction of this Honorable Court, the said accused, two of whom were armed with guns, conspiring, confederating together and helping one another, did then and there wilfully, unlawfully and feloniously with intent of gain and by means of violence, threat and intimidation, stop the Postal Delivery Truck of the Bureau of Postal while it was travelling along the MacArthur Highway of said municipality, at the point of their guns, and then take, rob and carry away with them the following, to wit: 1) Postal Delivery Truck 2) Social Security System Medicare Checks and Vouchers 3) Social Security System Pension Checks and Vouchers 4) Treasury Warrants 5) Several Mail Matters from abroad in the total amount of P253,728.29 more or less, belonging to US Government Pensionados, SSS Pensionados, SSS Medicare Beneficiaries and Private Individuals from Bulacan, Pampanga, Bataan, Zambales and Olongapo City, to the damage and prejudice of the owners in the aforementioned amount. Contrary to law

On separate dates, accused Filoteo, Mateo, Saguindel, Relator and Miravalles, assisted by their respective counsel, pleaded not guilty. Their co-accused Perez, Frias, Mendoza, Liwanag, Castro and Escalada were never arrested and remained at large. Accused Mateo escaped from police custody and was tried in absentia in accordance with Article IV, Section 19 of the 1973 Constitution. Accused Saguindel and Relator failed to appear during the trial on February 21, 1985 and on March 31, 1986, respectively, and were thus ordered arrested but remained at large since then. Like in the case of Mateo, proceedings against them were held in absentia. 5 Only Filoteo filed this petition, after the respondent Court rendered its assailed Decision and Resolution. Before trial commenced and upon the instance of the prosecution for a stipulation of facts, the defense admitted the following: 6
The existence of the bound record of Criminal Case No. 50737-B-82, consisting of 343 pages from the Bulacan CFI (Exhibit A); in 1982 or thereabouts, accused Bernardo Relator was a PC Sergeant at Camp Bagong Diwa, Bicutan, Metro Manila; as such PC Sergeant, accused Relator was issued a service revolver, Smith & Wesson Revolver, 32 (sic), with Serial No. 11707 (Exhibit B) and holster (Exhibit B-1) with six (6) live ammo (Exhibit B-2); in 1982 or thereabouts, accused Eddie Saguindel was a PC Constable First Class; on May 30, 1982, accused Saguindel, together with accused Relator and Danilo Miravalles, a former PC Sergeant, was invited for investigation in connection with the hijacking of a delivery van by the elements of the Special Operations Group, PC, and the three availed of their right to remain silent and to have counsel of their choice, as shown by their Joint Affidavit (Exhibit A-20); and the existence of the sworn statement executed

by accused Martin Mateo (Exhibit A-11) as well as the Certification dated May 30, 1982, subject to the qualification that said document was made under duress.

The prosecution sought to prove its case with the testimonies of Bernardo Bautista, Rodolfo Miranda, Capt. Rosendo Ferrer, M/Sgt. Noel Alcazar and Capt. Samuel Pagdilao, Jr. 6-a and the submission of Exhibits A to K. In their defense, accused Filoteo and Miravalles presented their respective testimonies plus those of Gary Gallardo and Manolo Almogera. Filoteo also submitted his Exhibits 1-14-Filoteo, but Miravalles filed no written evidence. Thereafter, the prosecution proffered rebuttal evidence and rested with the admission of Exhibits A-16-a, A-31 and L. Evidence for the Prosecution At about 6:30 in the morning of May 3, 1982, Bureau of Post mail van no. MVD 02 left San Fernando, Pampanga to pick up and deliver mail matters to and from Manila. On board the vehicle were Nerito Miranda, the driver, and two couriers named Bernardo Bautista and Eminiano Tagudar who were seated beside the driver. They arrived at around 9:40 that morning at the Airmail Distribution Center of the Manila International Airport where they were issued waybills 7 for the sacks of mail they collected. They then proceeded to the Central Post Office where they likewise gathered mail matters including 737 check letters 8 sent by the United States Embassy. All the mail matters were placed inside the delivery van, and its door padlocked. As they had to deliver mail matters to several towns of Bulacan, they took the MacArthur Highway on the return trip to Pampanga. When they reached Kalvario, Meycauayan, Bulacan at about 4:30 in the afternoon, an old blue Mercedes Benz sedan 9 overtook their van and cut across its path. The car had five (5) passengers three seated in front and two at the back. The car's driver and the passenger beside him were in white shirts; the third man in front and the person immediately behind him were both clad in fatigue uniforms, while the fifth man in the back had on a long-sleeved shirt. 10 Two of the car passengers aimed an armalite and a hand gun at driver Nerito Miranda as someone uttered, "Are you not going to stop this truck?" 11 Frightened, Miranda pulled over and stopped the van's engine. Alighting from the car, the armed group identified themselves as policemen. 12 They ordered the postal employees to disembark from the van. As he stepped out of the van, Miranda took the ignition key with him, but when threatened, he surrendered it to one of the car passengers. 13 The three postal employees were then ordered to board the Benz. As he was about to enter the car, Bautista looked back and saw one of the malefactors, who turned out to be Reynaldo Frias, going up the van. Inside the car, the three delivery employees were ordered to lower their heads. They sat between two of their captors at the back of the car while two others were in front. Later, Nerito Miranda asked permission to straighten up as he was feeling dizzy for lack of air. As he stretched, he caught a glimpse of the pimply face of the man to his left. He also recognized the driver who had glanced back. These men turned out to be Angel Liwanag and Reynaldo Frias, respectively. 14

As the car started moving, Bautista complained about feeling "densely confined." We was allowed to raise his head but with eyes closed. However, he sneaked a look and recognized the driver of the car as Raul Mendoza and the fellow beside him who poked a "balisong" at him as Angel Liwanag. The man in uniform on the front seat was Eddie Saguindel. Earlier, as he was about to enter the car, Bautista looked back and recognized Frias. 15 These incidents yielded the pieces of information critical to the subsequent identification of Mendoza, Liwanag, Saguindel and Frias in the line-up of suspects at Camp Crame later on. The car seemed to move around in circles. When it finally came to a stop, the captured men discovered that they were along Kaimito Road in Kalookan City They were made to remove their pants and shoes and then told to run towards the shrubs with their heads lowered. Upon realizing that the hijackers had left, they put on their pants and reported the incident to the Kalookan Police Station. The Security and Intelligence Unit of the Bureau of Posts recovered the postal van at the corner of Malindang and Angelo Streets, La Loma, Quezon City on May 4, 1982. Discovered missing were several mail matters, 16 including checks and warrants, along with the van's battery, tools and fuel. 17 In a letter-request dated May 6, 1982 to then Col. Ramon Montao, then Postmaster General Roilo S. Golez sought the assistance of the Special Operations Group (SOG) of the Philippine Constabulary in the investigation of the hijacking incident. 18 Responding to the request, the SOG, which was tasked to detect, investigate and "neutralize" criminal syndicates in Metro Manila and adjacent provinces, organized two investigative teams. One group was led by Capt. Rosendo Ferrer and the other by 1st Lt. Samuel Pagdilao. Initially, they conducted a "massive intelligence build-up" to monitor the drop points where the stolen checks could be sold or negotiated. On May 28, 1982, the SOG received a tip from a civilian informer that two persons were looking for buyers of stolen checks. Capt. Ferrer requested the informer to arrange a meeting with them. The meeting materialized at about 9:00 P.M. of May 29, 1982 at the Bughaw Restaurant in Cubao, Quezon City. With cash on hand, Capt. Ferrer posed as the buyer. The informer introduced him to Rey Frias and Rafael Alcantara. Frias in turn showed Capt. Ferrer a sample Social Security System (SSS) pension check and told him that the bulk of the checks were in the possession of their companions in Obrero, Tondo, Manila. After some negotiations, they agreed to proceed to Tondo. Then as they boarded a car, Capt. Ferrer introduced himself and his companions as lawmen investigating the hijacking incident. Shocked and distressed, Frias calmed down only when assured that his penalty would be mitigated should he cooperate with the authorities. Frias thus volunteered to help crack the case and lead the SOG team to Ricardo Perez and Raul Mendoza. Capt. Ferrer instructed Lt. Pagdilao, his assistant operations officer who was in another car during the mission, to accompany Frias to Obrero Tondo while he escorted Alcantara to their headquarters at Camp Crame. On the way to the headquarters,

Alcantara denied participation in the hijacking although he admitted living with Martin Mateo who allegedly was in possession of several checks. Alcantara was turned over to the investigation section of the SOG for further questioning. Meanwhile, Lt. Pagdilao's group was able to corner Ricardo Perez in his house in Tondo. Confronted with the hijacking incident, Perez admitted participation therein and expressed disappointment over his inability to dispose of the checks even after a month from the hijacking. He surrendered the checks in his possession to Lt. Pagdilao.'s. 19 An hour and a half later, Capt. Ferrer received information over their two-way radio that Ricardo Perez and Raul Mendoza were in Lt. Pagdilao's custody. Capt. Ferrer ordered that, instead of returning to headquarters, Lt. Pagdilao and his companions should meet him in Quirino, Novaliches to apprehend Martin Mateo. They met at the designated place and proceeded to Gulod, Novaliches arriving there at about 10:30 P.M. of May 29, 1982. Walking atop a ricefield dike to the house of Mateo, they noticed two men heading in their direction. Perez identified them as Martin Mateo and Angel Liwanag. The latter threw something into the ricefield which, when retrieved, turned out to be bundles of checks wrapped in cellophane inside a plastic bag. 20 As the two were about to board the SOG teams's car, Mateo said, "Sir, Kung baga sa basketball, talo na kami. Ibibigay ko yong para sa panalo. Marami pa akong tseke doon sa bahay ko, sir, kunin na natin para di na natin babalikan." 21 Capt. Ferrer accompanied Mateo to his house where they retrieved several other checks in another plastic bag. On the way to the SOG headquarters in Camp Crame, Mateo and Liwanag admitted participation in the postal hijacking. At a confrontation with Perez and Mendoza, all four of them pointed to petitioner, Jose D. Filoteo, Jr., as the mastermind of the crime. Consequently, Capt. Ferrer directed Lt. Pagdilao to accompany Mateo to the house of petitioner in Tondo, Manila. The lawmen found petitioner at home. Upon being invited to Camp Crame to shed light on his participation in the hijacking, petitioner was dumbfounded (" parang nagulat). Pursuant to standard operating procedure in arrests, petitioner was informed of his constitutional rights, 22 whereupon they proceeded to Camp Crame. However, the group, including petitioner, returned to the latter's place to recover the loot. It was "in the neighborhood," not in petitioner's house, where the authorities located the checks. 23 The authorities confronted Filoteo about his participation in the hijacking, telling him that Frias, Mendoza and Perez had earlier volunteered the information that petitioner furnished the Benz used in the hijacking. Thereupon, Filoteo admitted involvement in the crime and pointed to three other soldiers, namely, Eddie Saguindel, Bernardo Relator and Jack Miravalles (who turned out to be a discharged soldier), as his confederates. At 1:45 in the afternoon of May 30, 1982, petitioner executed a sworn

statement in Tagalog before M/Sgt. Arsenio C. Carlos and Sgt. Romeo P. Espero which, quoted in full, reads as follows:
BABALA Nais kong ipaalam sa iyo, Patrolman Filoteo, na ang dahilan ng pagsisiyasat na ito ay tungkol sa isang kasong Robbery-in-Band/Hi-Jacking na naganap noong ika-3 ng Mayo 1982 doon sa Meycauayan, Bulacan, mga bandang alas-4:00 ng hapon, humigitkumulang, kung saang maraming tsekeng US, tseke ng BIR at iba pang mga personal na tseke ang nabawi mula sa iyo. Nais ko ring ibigay sa iyo ang babala alinsunod sa mga isinasaad ng Section 20, Article IV ng Bagong Saligang Batas ng Republika ng Pilipinas, kagaya ng mga sumusunod: a. Na ikaw ay may karapatang tumahimik; b. Na ikaw ay may karapatang kumuha ng isang abugadong sarili mong pili upang may magpapayo sa iyo habang ikaw ay sinisiyasat; c. Na ikaw ay may karapatang huwag sumagot sa mga katanungang maaring makasira sa iyo sa dahilang anumang iyong isasalaysay ay maaring gamitin pabor or laban sa iyo sa kinauukulang hukuman; d. Na kung ikaw ay walang maibabayad sa isang abugado, ako mismo ang makipagugnayan sa CLAO-IBP upang ikaw ay magkaroon ng isang abugadong walang bayad. 1. TANONG: Ang mga bagay-bagay bang akin nang naipaliwanag sa iyo ay iyong lubos na naiintindihan at nauunawaan? SAGOT: Opo. 2. T: Handa mo bang lagdaan ang ilalim ng katanungan at sagot na ito bilang katibayan na iyo ngang naiintindihan ang iyong mga karapatan at gayun na rin sa dahilan ng pagsisiyasat na ito, at ikaw din ay nakahanda ngang magbigay ng isang malaya at kusang-loob na salaysay, sumagot sa mga katanungan at sumusumpang lahat ng iyong isasalaysay ay pawang mga katotohanan lamang? S: Opo, pipirma ako Ser. (Sgd.) JOSE D. FILOTE O (Affiant) MGA SAKSI:

(Sgd.) (Sgd.) ROMEO P. ESPERO THERESA L. TOLENTINO Ssg., PC C1C, WAC (PC) 3. T: Maari bang sabihin mong mull ang iyong buong pangalan, edad at iba pang bagay-bagay na maaring mapagkakikilalanan sa iyo? S: Jose Filoteo y Diendo, 30-anyos, may-asawa, isang Patrolman ng Western Police District, Metropolitan Police Force na kasalukuyang nakatalaga sa General Assignment Section, Investigation Division ng naturang Distrito ng Pulisya at kasalukuyang nakatira sa No. 810 Cabesas St., Dagupan, Tondo, Manila. 4. T: Kailan ka pa na-appoint sa service bilang isang Kabatas? S: Noon pong October 1978, hindi ko maalaala ang exactong petsa, noong ako ay mapasok sa serbisyo. 5. T: Kailan ka pa naman na-assign sa GAS, WPD, MPF? S: Noon lamang pong January 1982. 6. T: Patrolman Filoteo, ikaw ba ay tubong saang bayan, lungsod or lalawigan? S: Pagkakaalam ko sa tatay ko ay Bulacan samantalang ang aking ina naman ay Bisaya, pero ako ay ipinanganak na sa Maynila noon July 17, 1951. 7. T: Ano naman ang natapos mong kurso sa pag-aaral? S: Undergraduate ako ng BS Criminology sa PCCr, dahil hindi ko natapos ang second semester ng 4th year ko. 8. T: Ano naman ang iyong specific designation sa GAS, ID, WPDMPF? S: Sa Follow-Up Unit ako. 9. T: At bilang miyembro ng follow-up unit no GAS, ano naman ang iyong mga specific duties? S: Kami po ang magsasagawa ng follow-up kung may mga at-large sa mga suspects namin sa mga kasong hawak ng investigation. 10. T: Noong ika-3 ng Mayo 1982, mga bandang alas-4:00 ng hapon humigit-kumulang, saan ka naroroon at ano ang iyong ginagawa? S: Nasa Plaza Lawton ho kami, eh, at inaantay na namin iyong hinayjack namin na Philippine Mail delivery van.

11. T: Wika mo'y kami, sinu-sino ang tinutukoy mong mga kasamahan? S: Si Carding Perez, ho; si Junior ho (Affiant pointed to Martin Mateo, Jr. who was seated in the investigation room and asked the name and was duly answered: Martin Mateo, Jr.); si Rey Frias, Raul Mendoza; Angelo Liwanag at ang mga taga LRP ng PC Brigade na sina Sgt. Ed Saguindel, Sgt. Dan Miravales at isa pang Sergeant na ang alam ko lang sa kanya ay JUN ang tawag namin. Walo (8) (corrected and initialled by affiant to read as "SIYAM [9]") kaming lahat doon noon at ang mga gamit naman naming kotse noon ay ang kotse ng kumpare kong si Rudy Miranda na isang Mercedes Benz na may plakang NMJ-659 kung saang ang driver namin noon ay si Raul Mendoza (corrected and initialled by affiant to read as "AKO") at ang mga kasama naman naming sakay ay sina Angelo Liwanag, Sgt. Ed Saguindel at Sgt. Jun na parehong tagaLRP (affiant added and initialled this additional fact: "AT RAUL MENDOZA"). Ang isang kotse namang gamit namin ay pag-aari daw ng pinsan ni Carding Perez na kanya na rin mismong minaneho na isang Lancer na dirty-white ang kulay at ang mga sakay naman ni Carding Perez ay sina Junior Mateo, Rey Frias at Sgt. Dan Miravalles ng LRP rin. Pero may kasama pa kaming contact ni Carding Perez na taga-loob ng Post Office na sina Alias NINOY na isang dispatcher at Alias JERRY, dahil ang mastermind dito sa trabahong ito ay si Carding PEREZ at kami naman ng mga sundalong taga-LRP ay kanila lamang inimporta upang umeskort sa kaniia sa pag-hijack ng delivery van. 12. T: Anong oras naman noong umalis ang delivery van ng Post Office patungong norte? S: Kung hindi ako nagkakamali ay nasa pagitan na noon ng alas- 4:00 hanggang alas-5:00 ng hapon. 13. T: Isalaysay mo nga ng buong-buo kung ano ang mga naganap noong hapon na iyon? S: Noon pong lumakad na ang delivery van ng Central Post Office, sinundan na namin, una ang van, sumunod ang Lancer at huli ang Mercedes Benz namin. Pagdating namin sa Malinta, Valenzuela Metro Manila ay nagpalit kami ng puwesto sa pagsunod, van naman ngayon, sunod ang Mercedes Benz at huli na ang Lancer. Noong makapasok na kami ng boundary ng Meycauayan, Bulacan ay kumuha na kami ng tiyempo at noon makatiyempo kami ay kinat namin ang delivery van. Tumigil naman ito at bumaba kaagad sina Sgt. Ed Saguindel at Sgt. Jun ng LRP datiil sila noon ang may hawak ng kanilang Armalite Rifle pero may service pa silang maiksing baril. Pinababa nila ang tatlong maydala ng delivery van at pinasakay sa Mercedes Benz, habang nakatutok ang kanilang mga baril sa kanila. Ako naman ay bumaba na sa aming kotse at sumakay ng delivery van at ako na mismo ang nagmaneho at sinamahan naman ako nina Junior Mateo at si Rey Frias, tatlo (3) rin kaming pumalit sa puwesto noong tatlong (3) taga-Post Office na maydala ng delivery van. Nag-Utturn (sic) kami ngayon at ibinalik na namin sa Manila ang van. Iyong Mercedes Benz na minaneho pa rin ni Raul Mendoza ay dumeretso pa norte samantalang ang Lancer naman ay nag-U-turn din at sumunod sa amin. Noong makarating na kami sa Malinta, Valenzuela, Metro Manila ay inunahan na kami ng Lancer at

iyon na nga, parang follow the leader na dahil siya na noon ang aming guide. 14. T: Ipagpatuloy mo ang iyong pagsasalaysay? S: Dumeretso kami ngayon sa may Obrero, sa bahay mismo nina Carding Perez, at noong nakarating na kami roon ay iniyatras ko na ang van sa kaniling garahe at doon ay ibinaba namin lahat ang mga duffle bag, hindi ko na ho alam kung ilan lahat iyon, na siyang laman ng delivery van at pagkatapos ay umalis kaming muli ng mga kasama ko rin sa van papuntang Quezon City kung saan namin inabandon ang delivery van. Sa Retiro ho yata iyong lugar na iyon, kung hindi ako nagkakamali. 15. T: Ano ang mga sumunod na nangyari? S: Sumakay kami ngayon ng taksi at bumalik na kami kina Carding Perez sa may bahay nila sa Obrero, Tondo, Manila at inabutan na namin sila na nagkakarga na noong mga duffle bag sa (sic), madilim na ho noon, sa isang kotseng mamula-mula o orange na Camaro at isa pang Mercedes Benz na brown, dahil ang Lancer ay isinoli na raw nila sa mayari. Dinala nila ngayon ang mga duffle bag sa Bocaue, Bulacan, iyon kasi ang usapan namin noon dahil sumilip lamang ako noon at kasama ko si Carding Perez, kami naman ngayon ay pumunta sa bahay nina Rudy Miranda sa San Marcelino, Malate, Manila na sakay ng isang Toyota Corona na brown na si Carding Perez ang nagmaneho. Pagdating namin doon sa kina Rudy Miranda ay naroon na rin noon ang Mercedes Benz na ginamit namin, pero wala na ang crew ng delivery van dahil ibinaba at iniwanan daw nila sa Caloocan City. Ang naroroon na lamang noon ay sina Angelo Liwanag, si Raul Mendoza, si Sgt. Ed Saguindel at si Sgt. Jun na parehong taga-LRP. Naiwan na noon ang Mercedes Benz namin doon kina Rudy Miranda at iniwan na rin ang susi doon sa kamag-anak, dahil hindi nila alam ang trabahong ito. Sumakay na iyong apat naming kasama sa Toyota Corona na sakay namin at inihatid namin sina Sgt. Saguindel at Sgt. Jun doon sa tinitirhan nitong huling nabanggit na sundalo doon sa malapit sa Del Pan Bridge sa may Recto Avenue sa San Nicolas yata iyon sa Manila. Kami naman ngayong apat, sina Carding Perez, Angelo Liwanag at si Raul Mendoza ay tumuloy na sa Bocaue, Bulacan. Dumaan kami sa North Diversion Road at paglabas namin sa exit papuntang Bocaue, Bulacan ay hindi na kalayuan doon, hindi ko alam ang lugar pero alam kong puntahan. Bahay daw yata ng kamag-anak ni Carding Perez iyon pero hindi ko alam ang pangalan. Naroon na ngayon ang buong tropa, maliban sa mga dalawang sundalong naihatid na namin sa may Manila, at may mga nadagdag pang ibang mukha pero hindi ko ito mga kakilala. Si JACK o Sgt. Dan Miravalles ay naroon din noon. Kumain kami, pagkatapos ay nagbukasan na ng mga duffle bag. Iyon na nga, nakita na namin ang mga tsekeng ito, (Affiant pointed to the checks he voluntarily surrendered) at aming inihiwalay ngayon sa mga sulat na naroon na sinunog lahat pagkatapos doon sa bahay ni Junior Mateo sa Novaliches. Di magdamag ngayon ang trabaho namin, kinabukasan ay kanyakanyang uwian na, pagkatapos ay pahinga. Kinabukasan mull, gabi, inilipat na namin doon sa bahay ni Junior Mateo ang mga tsekeng ito (Affiant again referred to said checks). Isinakay namin noon sa isang cargo truck na pag-aari din daw nina Carding. lyong mga tsekeng iyan ngayon ay nakalagay noon doon sa isang sikretong compartment sa

gitna ng truck, doon ba sa may chassis. Sikretong compartment iyon, na mahirap mahalata. 16. T: Ikaw ba naman ay mayroong dalang baril noon at kung ganoon, sabihin mo nga kung anong uring baril iyon? S: Wala po akong baril, Ser. 17. T: Paano naman napunta ang mga tsekeng ito (the checks recovered from the Affiant was referred to) sa iyo? S: E, di ganoon na nga he, habang tumatagal ay umiinit ang situwasyon sa aming grupo, dahil iyong partehan sana namin ay puro pangako ang nangyari. Kaya napagpasyahan namin na hatiin na lamang iyong mga tseke upang walang onsehan sa amin. Ito ngayon ay parte namin nina Sgt. Ed Saguindel, Sgt. Dan Miravalles Alias JACK at ni Sgt. Jun, dahil noong una ay doon muna sa amin ito nakatago (The checks recovered from the Affiant was referred to). Pero habang tumatagal ay umiinit at nalaman namin pati na may alarma na, kaya't inilipat namin doon sa may Raxa Bago sa may likod ng Alhambra Cigar & Cigarette Factory sa Tondo, Manila at akin munang ipinatago sa isang kumare ko doon, pansamantala, pero hindi alam nitong kumare ko ang laman noon dahil mahigpit kong ipinagbilin na huwag nilang bubuksan. Doon na rin namin kinuha iyon noong isurender ko ang mga tsekeng ito kagabi, at hanggang sa kinuha na namin ang supot na ito (the checks placed in a plastic bag was again referred to) ay wala pa rin kamalay- malay ang kumare ko. 18. T: Iyong sinasabi mong mga kontak nina Carding Perez sa Central Post Office, mga kakilala mo rin ba ang mga ito? S: Iyong araw lamang na iyon ko sila nakita, dahil maghapon ko noon silang nakikita, itong si Alias NINOY lamang ang dispatcher, dahil palabas-labas siya noon at nakikipag-usap kina Carding Perez, Raul Mendoza at saka si Rey Frias. Makikilala ko itong si Alias NINOY kung makita ko siyang muli. 19. T: Sino naman ang kumontak sa iyo upang sumama sa trabahong ito? S: Si Junior Mateo po, ipinakilala niya ako kina Carding at sa buong tropa na namin. 20. T: Pansamantala ay wala na muna akong itatanong pa sa iyo, mayroon ka bang nais na idagdag, bawasin o palitan kaya sa salaysay na ito? S: Wala na po. 21. T: Handa mo bang lagdaan ang iyong salaysay na ito bilang patotoo sa katotohanan nito nang hindi ka pinilit, sinaktan or pinangakuan kaya ng anuman upang lumagda lamang?

S: Opo. WAKAS NG SALAYSAY: . . . ./ac (Sgd) JOSE D. FILOTE O MGA SAKSI SA LAGDA: (Sgd.) SSG ROMEO P. ESPERO PC

(Sgd.) C1C THERESA TOLENTINO WAC (PC) 24 Petitioner executed two other documents on the same day, May 30, 1982. One was a certification stating that he voluntarily surrendered "voluminous assorted US checks and vouchers," that because of the "large number of pieces" of checks, he affixed his signature upon the middle portion of the back of each check "to serve as identification in the future, prior to the completion of its proper inventory and listing conducted by elements of SOG" in his presence, and that he "guided the elements of SOG" to the residence of Rodolfo C. Miranda, the owner of the sky-blue Mercedes Benz car which was surrendered to the SOG Headquarters. 25 The other document was a sworn statement wherein petitioner attested to his waiver of the provisions of Article 125 of the Revised Penal Code and the following facts: (a) that he was apprised of his constitutional rights under Section 20, Article IV of the (1973) Constitution, that he understood all his rights thereunder, and that the investigators offered him counsel from the CLAO-IBP but he refused to avail of the privilege; (b) that he was arrested by SOG men in his house at around 11:00 p.m. of May 29, 1982" sa dahilang ako ay kasangkot sa pagnanakaw ng mga US Treasury Warrants, SSS Pension Checks and Vouchers at SSS Medicare Checks and Vouchers mula sa delivery van ng Philippine Mail;" (c) that the SOG men confiscated from him numerous checks and a Mercedes Benz 200 colored sky-blue, and (d) that he was not hurt or maltreated nor was anything taken from him which was not duly receipted for. 26 As certified to by petitioner (in the above described document), he led the SOG operatives to the house of Rodolfo Miranda on Singalong where the latter admitted that petitioner was his friend. He denied, however, having knowledge that his car was used in the hijacking until the authorities came to his house. According to Miranda, he was made to believe that his car would be used for surveillance purposes because petitioner's jeep was not available. The car was not returned until the evening following that when it was borrowed. 27 After the trip to Miranda's house, petitioner informed the investigators that some more checks could be recovered from his kumare. Said checks

were retrieved and turned over to headquarters along with the car surrendered by Miranda who later executed a sworn statement dated May 31, 1992 at the SOG. 28 Upon learning of the whereabouts of Miravalles, Eddie Saguindel and Bernardo Relator, the team of Capt. Ferrer proceeded to Taguig, Metro Manila in the afternoon of May 30, 1982. They met Miravalles along the way to his house. Informed by Capt. Ferrer that six of his companions were already under custody and that they implicated him as one of their confederates, Miravalles reacted by saying, "Sir, ang hihina kasi ng mga loob niyan, eh." 29 Capt. Ferrer later asked Miravalles to bring him to Eddie Saguindel. At the barracks of the Long Range Patrol in Bicutan, Metro Manila, Saguindel voluntarily accepted the invitation to proceed to the SOG headquarters, after Miravalles initially informed him of the facts obtained during the investigation. Saguindel was heard saying, "Hindi na kami interesado, sir, sa mga tsekeng iyan kasi isang buwan na hindi pa nabebenta." 30 With Miravalles and Saguindel, Capt. Ferrer and his team moved on to Binondo, Manila to look for Bernardo Relator. When they found him at home, Relator excused himself, went upstairs, returned with a .32 caliber revolver with six bullets 31 and said, "Sir, ito yong baril na nagamit." 32 The three suspects were brought to Camp Crame for further investigation. Thereafter, Capt. Ferrer submitted an after-operations report about their mission and executed jointly with Lt. Pagdilao an affidavit on the same matter. 33 Aside from petitioner, Liwanag, Mateo and Perez executed sworn statements. 34 Prior to doing so, they waived their right to counsel. Liwanag and Mateo admitted their participation and implicated petitioner in the crime. Perez, on the other hand, denied having driven a Lancer car in the hijacking and stated that he was implicated in the crime only because in one drinking spree with petitioner, Mateo and one alias "Buro" during that month of May, they had a heated altercation. Like petitioner, Liwanag and Mendoza certified that they voluntarily surrendered vouchers and checks which were part of their loot in the hijacking; they also executed waivers under Article 125 of the Revised Penal Code. For his part, Relator executed a certification to the effect that he voluntarily surrendered his .32 caliber Smith & Wesson service revolver used in the commission of the crime. In spite of the fact that his father-in-law was a lawyer, petitioner did not manifest that he needed the assistance of counsel. During the taking of his statement, petitioner was visited by Jimmy Victorino and another comrade from the General Assignment Section of the WPD. For their part, Relator, Saguindel and Miravalles executed a joint affidavit 35 manifesting their option to avail of their right to remain silent until such time as they would have retained a counsel of their choice. Frias and Mendoza executed a similar joint affidavit. 36 Severino Castro, the postal employee implicated, also chose to remain silent as he wanted to testify in court. However, he linked to the crime a certain Gerardo Escalada, a former clerk of the Central Post Office and son of a director of the Bureau of Posts in Region I. 37

On May 31, 1982, then Postmaster General Golez summoned postal employees Miranda, Bautista and Tagudar and directed them to proceed to Camp Crame. At the office of the SOG, they were told to go over some pictures for identification of the culprits. The three recognized and pointed to the suspects in a line-up. Tagudar identified Saguindel and Liwanag. 38 Miranda pointed at Frias and Liwanag 39 while Bautista identified Frias, Mendoza and Liwanag. 40 Petitioner himself, when told to identify his alleged cohorts, pointed to Severino Castro as their contact at the post office. 41 Five of the suspects who were not identified in the line-up were however implicated by Liwanag, Mateo and petitioner. SOG Chief Investigator Jorge C. Mercado filed a complaint for robbery-in-band (hijacking) before the Municipal Court of Meycauayan, Bulacan against petitioner and ten (10) others, namely, Mateo, Saguindel, Relator, Miravalles, Perez, Frias, Mendoza, Liwanag, Castro and Escalada (Criminal Case No. 7885). 42 On August 8, 1983, the Information previously referred to and aforequoted was filed with the Sandiganbayan and docketed as Criminal Case No. 8496. On September 20, 1983, Sandiganbayan Associate Justice Romeo M. Escareal issued orders for the arrest of the accused 43 and fixed bail at P13,000.00 each. Saguindel and Relator filed a motion to quash the Information asserting that under the Articles of War and Section 1 of P.D. 1850, they should be tried by a court martial. 44 The Sandiganbayan denied the motion on January 3, 1984 45 on the ground that courts martial could no longer exercise jurisdiction over them by virtue of their separation from military service. Evidence for the Defense Testifying in his own defense, petitioner alleged that as a patrolman since August 21, 1978 assigned to the Investigation Division or the Detective Bureau of the WPD to which the General Assignment Section belonged, he was the recipient of several awards and recognitions starting with ranking fifth in the Final Order of Merit in the basic course for police officers. 46 He also claimed to have received a loyalty medal for meritorious service above the call of duty 47 and several commendations 48 for the distinguished performance of his duties. On that fateful date of May 3, 1982, he was a member of the Special Task Force Unit covering the tourist belt area. Of the ten other accused in this case, petitioner admitted knowing only Martin Mateo whose name appeared in the initial follow-up operation he allegedly participated in regarding a P250,000 qualified theft case on May 16, 1980 at the Shemberg Marketing Corporation. 49 Although a suspect, Mateo was not charged in the information subsequently filed in that case. Sometime in March 1981, Mateo visited petitioner at the police headquarters seeking assistance in his bid to lead a new life. Considering Mateo's familiarity with underworld characters, petitioner readily made him an informer who was paid from time to time out of the police intelligence fund. Mateo proved to be

an effective informer. In fact, he allegedly supplied vital information on the identities and whereabouts of suspects in robbery cases at the La Elegancia Jewelry Store, at the Likha Antique and Crafts, 50 and in an alleged racket in Aranque Market in Manila involving jewelries. As such informer, Mateo became accustomed to borrowing petitioner's owner-type jeep whenever he was given an assignment. In one instance however, petitioner saw Mateo using his jeep with some male companions. Because Mateo denied the occurrence of the incident, petitioner from then on refused to lend his jeep to Mateo. Instead, Mateo was given an allowance to cover his traveling expenses. About a month prior to May 3, 1982, petitioner met Mateo and requested the latter to give him a good project as he was working for his transfer to the Metrocom Intelligence Security Group (MISG). On May 2, 1982, Mateo urged petitioner to lend him his jeep in order that he could follow-up a bank robbery case. That same evening, petitioner approached his kumpare, accused Rodolfo Miranda, to borrow the latter's old Mercedes Benz since, if the jeep was used, Mateo could be identified as an informer. Petitioner left his jeep with Miranda and "went around boasting of the Mercedes Benz." 51 Mateo took the Benz in the morning of May 3, 1982. Petitioner advised him to return the car between the hours of two and three in the afternoon at the Lakan Beer House at the corner of Rizal Avenue and Zurbaran Streets in Sta. Cruz, Manila where petitioner was to meet his friend Manolo Almoguera who would be celebrating his birthday there. Petitioner met Almoguera and company at around 3:30 in the afternoon. He waited for Mateo until shortly before 5:00 in the afternoon when he was constrained to leave without seeing Mateo because he had to attend a mandatory regular troop formation at 5:00 P.M. at the police headquarters. From there, petitioner proceeded to his area of responsibility in the tourist belt. He returned to the beer house at about 6:00 in the evening hoping to find Mateo and the automobile. A little before 8:00 o'clock, someone informed him that Mateo had finally arrived. Petitioner went out and scolded Mateo for being late; the latter apologized and said that his surveillance bore good results. Petitioner then returned the car to Miranda, through the latter's cousin. At around 11:00 in the evening of May 29, 1982, Mateo, escorted by a group of military men, went to petitioner's house at 810 Cabezas St., Tondo, Manila. The group refused to give any reason for their visit but arrested him. Wearing only short pants, petitioner was made to board a car where he was handcuffed. The men asked him about the Benz and the identities of his companions in an alleged hijacking incident. Petitioner admitted having knowledge of the exact location of the car but denied participation in the crime. Nobody apprised him of his constitutional rights to remain silent and to be assisted by counsel. 52 Petitioner was then instructed to accompany Lt. Pagdilao to the residence of Miranda to get the Benz. They were on board two cars. When petitioner noticed that they were not heading for Miranda's place, he clutched the hand of Lt. Pagdilao, pleading for pity and thinking that he was about to be "salvaged". Lt. Pagdilao however informed him that

they would be dropping by petitioner's house first per the investigator's information that more checks could be recovered thereat. A warrantless search was then allegedly conducted in petitioner's house but nothing was found. Suddenly, someone from the other car came out of a nearby house owned by Mateo and reported that they had recovered some checks. Thereafter, they proceeded to the house of Miranda who was also invited for questioning. The latter surrendered his Benz to the group. At the SOG headquarters in Camp Crame, petitioner was repeatedly coaxed to admit participation in the hijacking. As he vehemently denied the accusation against him, someone blindfolded him from behind, led him outside and loaded him in a car. He was taken to an unidentified place and made to lie flat on his back. An object was tied to his small finger to electrocute him. While a wet handkerchief was stuffed in his mouth, someone mounted his chest and applied the "water cure" ("tinutubig") through his nose. Because these ordeals were simultaneously carried out, petitioner felt unbearable pain. He sought permission to get in touch with his father-in-law, Atty. Felix Rosacia, but his request was denied. They urged him to cooperate otherwise something terrible would happen to him. Meanwhile, petitioner's wife reported to the WPD General Assignment Section her husband's forcible abduction by armed men whom she mistook for CIS agents. A check with the CIS yielded negative results. Thereafter, Lt. Reynaldo Dator went to the SOG where he was informed that petitioner was being investigated but no details were given thereon pending clearance with superior officers. 53 Consequently, a newspaper carried an item on the SOG's refusal to allow petitioner's co-police officers to see him in his detention cell. 54 Among his comrades, only Jimmy Victorino, formerly of the WPD who was transferred to the SOG, was able to visit him. Petitioner revealed to Victorino the maltreatment done him but the latter expressed helplessness about it. In fact, Victorino advised him to just cooperate so that the SOG would not incriminate him (" para hindi ka pag-initan dito"). 55 The advice came after petitioner was warned that he, like Pat. Serrano of the WPD, would be liquidated by the SOG, 56 should he refuse to cooperate. Later, Mateo came to petitioner's cell and confided that he had been similarly maltreated and forced to implicate petitioner. After Mateo left, a prepared statement was shown and read to petitioner. Because its contents were false, petitioner refused to sign it. Placing his arm around petitioner, a certain Capt. Lagman told petitioner that he thought they had an understanding already. Petitioner later discovered that Lagman was not member of the military but an "agent" of the SOG, and a member of the "Contreras gang". Petitioner was therefore constrained to sign the statement because of his excruciating experience ("hirap na hirap"). He however admitted having read the document before affixing his signature thereto and initialing the corrections therein. The waiver under Article 125 of the Revised Penal Code and the certification he executed were allegedly also obtained by duress. Although he picked out one Severino Castro in a police line-up, he did not even know Castro. He implicated Castro because he was threatened by a certain Boy Zapanta.

Petitioner filed a complaint for grave coercion and maltreatment against Lt. Rosendo Ferrer and several John Does. On August 4, 1982, Asst. City Fiscal Emelita H. Garayblas recommended its dismissal for petitioner's failure to appear despite subpoenas and to answer clarificatory questions as well as to authenticate his statement. 57 However, petitioner swore that he never received the subpoenas. Petitioner's alibi was supported by Manolo Almoguera whose birthday on May 3, 1995 was the reason for the celebration at the Lakan Beer House. While his baptismal certificate indicated that he was born on May 4, 1956, 58 a joint affidavit 59 also attested that his birth date was actually May 3, 1956. Gary Gallardo, the owner of the beer house, corroborated Almoguera's testimony as to petitioner's alleged presence during the birthday celebration. The Respondent Court's Decision On June 18, 1987, the Sandiganbayan rendered the herein questioned 51-page Decision, the dispositive portion of which reads:
WHEREFORE, judgment is hereby rendered finding accused Jose Filoteo, Jr. y Diendo, Martin Mateo, Jr. y Mijares, Bernardo Relator, Jr. y Retino and Eddie Saguindel y Pabinguit GUILTY as co-principals beyond reasonable doubt of the violation of Section 2 (e), in relation to Section 3 (b) of Presidential Decree No. 532, otherwise known as the Anti-Piracy and Anti-Highway Robbery Law of 1974 and hereby sentences each of said accused to suffer the indeterminate penalty ranging from TWELVE (12) YEARS and ONE (1) DAY as minimum, to THIRTEEN (13) YEARS, ONE (1) MONTH and ELEVEN (11) DAYS as maximum, both of reclusion temporal, and to pay their proportionate share of the costs of the action. Accused Danilo Miravalles y Marcelo is hereby acquitted, with costs de oficio, for insufficiency of evidence. No civil indemnity is hereby awarded due to the complete dearth of any proof as to the actual damages suffered by the Bureau of Posts or the owners of the pilfered mail matters, and it further appearing that the mail van which was hijacked had been recovered, as well as most of the checks and warrants which were surrendered by some of the accused, without prejudice to the institution of the proper civil action to recover damages should proof thereof be available. Consequently, it is hereby ordered that Exhibits B, B-l and B-2, which are the .32 Cal. Revolver, Smith and Wesson, Serial No. 11707, its holster and six (6) live ammunition respectively, which were surrendered by accused Relator, and Exhibits J, J-l to J-5, consisting of 187, 222, 215, 197, 194 and 22 pieces, respectively, of Social Security System and Medicare checks and vouchers, be returned to the Firearm and Explosives Unit (FEU), PC, Camp Crame, Quezon City and the Social Security System, respectively, upon proper receipts. Let copies of this decision be furnished the Postmaster-General, Central Post Office, Liwasang Bonifacio, Metro Manila and the Commanding General and Chief, PC-INP, Camp Crame, Quezon City for their information and guidance with respect to the other accused who are still at-large. SO ORDERED.

Petitioner's motion for reconsideration of said Decision was denied by the Sandiganbayan in its challenged Resolution of July 27, 1987. Hence, the instant alternative petition for certiorari and/or review on certiorari charging the Sandiganbayan with having gravely abused its discretion amounting to lack or excess of jurisdiction and with reversible error in arriving at said Decision. The Issues The amended petition raises the following:
Assignments of Error and/or Excess of Jurisdiction/Grave Abuse of Discretion xxx xxx xxx First The respondent court erred and gravely abused its discretion as well as exceeded its jurisdiction when it made its determination of the alleged guilt of petitioner on the basis of mere preponderance of evidence and not proof beyond reasonable doubt. Second The respondent court erred and gravely abused its discretion as well as exceeded its jurisdiction in finding that petitioner's having borrowed the Mercedes Benz car utilized by the other accused in the hijacking of the mail van indubitably established his direct participation and/or indispensable cooperation in the said hijacking, the same being in gross disregard of basic Rules of Law. Third The respondent court erred and gravely abused its discretion as well as exceeded its jurisdiction in finding that the voluminous SSS Medicare and Pension Checks were confiscated from and surrendered by petitioner and three of the other accused and in finding the testimonies and investigation reports relative thereto. "credible and unrefuted", said findings being, insofar as petitioner is concerned, absolutely without any basis in the evidence and in fact contrary to the prosecution's only evidence that has some measure of competency and admissibility. Fourth The respondent court erred and gravely abused its discretion in finding that dorsal portions of the checks and warrants allegedly taken from petitioner were signed by him to indicate his admission of accountability therefor and that his signatures thereon confirm the confiscation from and/or surrender by him of said checks, said findings being absolutely without any support in the evidence. Fifth

The respondent court erred and gravely abused its discretion as well as exceeded its jurisdiction in admitting and considering against petitioner his alleged extra judical confession, despite petitioner's uncontradicted testimony and documentary proof that he was made to give or sign the same through torture, maltreatment, physical compulsion, threats and intimidation and without the presence and assistance of counsel, his request for which was refused, in gross violation of Constitutional Provisions and the prevailing jurisprudence. Sixth The respondent court erred and gravely abused its discretion as well as exceeded its jurisdiction in finding that petitioner's participation in the hijacking of the mail van is indubitably established "by the manner by which the SOG operatives succeeded in ferreting out the members of the hijacking syndicate one by one through patient sleuthing" and in finding that they did so "without resorting to extra-legal measures" and that "no evidence having been adduced to show that they were actuated by improper motives to testify falsely against the herein accused, then their testimonies should be accorded full credence". Seventh The respondent court erred and gravely abused its discretion as well as exceeded its jurisdiction in finding that "even setting aside the inter-locking confessional statements of Filoteo, Mateo and Liwanag, . . substantial and sufficient evidence exist which indubitably prove the guilt of Filoteo" (Petitioner). Eighth Insofar as petitioner is concerned, the respondent court erred and gravely abused its discretion as well as exceeded its jurisdiction in finding that "accused Filoteo's ( petitioner's) and Mateo's [alleged] unexplained possession of the stolen checks raised the presumption that "they were responsible for the robbery in question", petitioner's alleged possession not being borne out but disputed by the prosecution's own evidence. Ninth The respondent court erred and gravely abused its discretion as well as exceeded its jurisdiction in finding that "accused Filoteo's denials and alibi cannot be entertained for being quite weak and implausible". The truth of the matter being that they should have been sustained since petitioner was not identified by direct victims-eyewitnesses as among those who participated in or were present at the hijack and none of the checks and treasury warrants were found in his possession or retrieved from him. Tenth The respondent court erred and gravely abused its discretion as well as exceeded its jurisdiction in finding that the participation of petitioner in the criminal conspiracy has been proven beyond reasonable doubt by the evidence of record and that said evidence "not only confirms the conspiracy between [him and the other accused] as easily discernible from their conduct before, during and after the commission of the offense, but also their participation and/or indispensable cooperation". Eleventh

The respondent Court erred and gravely abused its discretion as well as exceeded its jurisdiction in cavalierly rejecting, through the use of pejorative words, and without stating the legal basis of such rejection, the various vital factual points raised by petitioner, in gross violation of the express mandate of the 1987 Constitution.

The Court believes that the above "errors" may be condensed into four: (1) Are the written statements, particularly the extra-judicial confession executed by the accused without the presence of his lawyer, admissible in evidence against him? (2) Were said statements obtained through torture, duress, maltreatment and intimidation and therefore illegal and inadmissible? (3) Was petitioner's warrantless arrest valid and proper? (4) Is the evidence of the prosecution sufficient to find the petitioner guilty beyond reasonable doubt? The Court's Ruling Preliminary Issue: Rule 4 or Rule 65? Before ruling on the foregoing issues, it is necessary to dwell on the procedural aspects of the case. Petitioner, a "segurista", opted to file an (amended) "alternative petition" for certiorari under Rule 65 and for review on certiorari under Rule 45 of the Rules of Court. We however hold that the instant petition must be considered as one for review on certiorari under Rule 45. In Jariol, Jr. vs. Sandiganbayan, 60 this Court clearly ruled:
Presidential Decree No. 1486, as amended by P.D. No. 1606, which created the Sandiganbayan, specified that decisions and final orders of the Sandiganbayan shall be subject to review on certiorari by this Court in accordance with Rule 45 of the Rules of Court. And Rule 45 of the Revised Rules of Court provides, in Section 2, that only questions of law may be raised in the Petition for Review and these must be distinctly set forth. Thus, in principle, findings of fact of the Sandiganbayan are not to be reviewed by this Court in a petition for review on certiorari. There are, of course, certain exceptions to this general principle. Here, reading petitioner's Petition for Review and Memorandum in the most favorable possible light, petitioner may be seen to be in effect asserting that the Sandiganbayan misapprehended certain (f)acts in arriving at its factual conclusions.

As amended by Republic Act No. 7975, Section 7 of P.D. No. 1606 expressly provides that "(d)ecisions and final orders of the Sandiganbayan shall be appealable to the Supreme Court by petition for review on certiorari raising pure questions of law in accordance with Rule 45 of the Rules of Court." However, in exceptional cases, this Court has taken cognizance of questions of fact in order to resolve legal issues, as where there was palpable error or grave misapprehension of facts by the lower court. Criminal cases elevated by convicted public officials from the Sandiganbayan deserve the same thorough treatment by this Court as criminal cases involving ordinary citizens simply because the constitutional presumption of innocence must be overcome by proof beyond reasonable doubt. In all criminal cases, a person's life and liberty are at stake. 61

As a petition for review under Rule 45 is the available remedy, a petition for certiorari under Rule 65 would not prosper. Basic it is that certiorari is invocable only where there is no other plain, speedy or adequate remedy. For waffling on procedural matters, petitioner could have lost this battle through a summary dismissal of his "alternative" petition. But in view of the importance of the issues raised, the Court decided to take cognizance of the matter. First Issue: Uncounselled Waiver On the merits of the petition, we find that the pivotal issue here is the admissibility of petitioner's extrajudicial confession which lays out in detail his complicity in the crime. Petitioner contends that respondent Court erred in admitting his extrajudicial confession notwithstanding uncontradicted testimony and documentary proof that he was made to sign the same through torture, maltreatment, physical compulsion, threats and intimidation and without the presence and assistance of counsel. He also claims that in executing the extrajudicial confession, he was denied the right to counsel in the sameway that his waiver of the said right was likewise without the benefit of counsel. Petitioner therefore questions the respondent Court's admission evidence of his extrajudicial confession on the strength of cases 62 upholding the admissibility of extrajudicial confessions notwithstanding the absence of counsel "especially where the statements are replete with details and circumstances which are indicative of voluntariness." We shall first tackle the issue of his uncounselled waiver of his right to counsel. The pertinent provision of Article IV, Section 20 of the 1973 Constitution reads as follows:
No person shall be compelled to be a witness against himself. Any person under investigation for the commission of an offense shall have the right to remain silent and to counsel and to be informed of such rights. No force, violence, threat, intimidation, or any other means which vitiate the free will shall be used against him. Any confession obtained in violation of this section shall be inadmissible in evidence.

In comparison, the relevant rights of an accused under Article III, Section 12 of the 1987 Constitution are, inter alia, as follows:
(1) Any person under investigation for the commission of an offense shall have the right to be informed of his right to remain silent and to have competent and independent counsel preferably of his own choice. If the person cannot afford the services of counsel, he must be provided with one. These rights cannot be waived except in writing and in the presence of counsel. (2) No torture, force, violence, threat, intimidation; or any other means which vitiate the free will shall be used against him. Secret detention places, solitary, incommunicado, or other similar forms of detention are prohibited. (3) Any confession or admission obtained in violation of this or Section 17 hereof shall be inadmissible in evidence against him.

(4) The law shall provide for penal and civil sanctions for violations of this section as well as compensation to and rehabilitation of victims of torture or similar practices and their families." (emphasis supplied. Obviously, the 1973 Constitution did not contain the right against an uncounselled waiver of the right to counsel which is provided under paragraph 1, Section 12, Article III of the 1987 Constitution, above underscored.)

In the landmark case of Magtoto vs. Manguera, 63 the Court categorically held that the aforequoted provisions of the 1973 Constitution (which were not included in the 1935 Charter) must be prospectively applied. This Court said:
We hold that this specific portion of this constitutional mandate has and should be given a prospective and not a retrospective effect. Consequently, a confession obtained from a person under investigation for the commission of an offense, who has not been informed of his right (to silence and) to counsel, is inadmissible in evidence if the same had been obtained after the effectivity of the New Constitution on January 17, 1973. Conversely, such confession is admissible in evidence against the accused, if the same had been obtained before the effectivity of the New Constitution, even if presented after January 17, 1973, and even if he had not been informed of his right to counsel, since no law gave the accused the right to be so informed before that date.

By parity of reasoning, the specific provision of the 1987 Constitution requiring that a waiver by an accused of his right to counsel during custodial investigation must be made with the assistance of counsel may not be applied retroactively or in cases where the extrajudicial confession was made prior to the effectivity of said Constitution. Accordingly, waivers of the right to counsel during custodial investigation without the benefit of counsel during the effectivity of the 1973 Constitution should, by such argumentation, be admissible. Although a number of cases held that extrajudicial confessions made while the 1973 Constitution was in force and effect, should have been made with the assistance of counsel, 64 the definitive ruling was enunciated only on April 26, 1983 when this Court, through Morales, Jr. vs. Enrile, 65 issued the guidelines to be observed by law enforcers during custodial investigation. The Court specifically ruled that "(t)he right to counsel may be waived but the waiver shall not be valid unless made with the assistance of counsel. 66 Thereafter, in People vs. Luvendino, 67 the Court through Mr. Justice Florentino P. Feliciano vigorously taught:
. . . The doctrine that an uncounseled waiver of the right to counsel is not to be given legal effect was initially a judge-made one and was first announced on 26 April 1983 in Morales vs. Enrile and reiterated on 20 March 1985 in People vs. Galit. . . . While the Morales-Galit doctrine eventually became part of Section 12(1) of the 1987 Constitution, that doctrine affords no comfort to appellant Luvendino for the requirements and restrictions outlined in Morales and Galit have no retroactive effect and do not reach waivers made prior to 26 April 1983 the date of promulgation of Morales.

Pursuant to the above doctrine, petitioner may not claim the benefits of the Morales and Galit rulings because he executed his extrajudicial confession and his waiver to the right to counsel on May 30, 1982, or before April 26, 1983. The prospective application of "judge-made" laws was underscored in Co vs. Court of Appeals 68 where the Court ruled thru Chief Justice Andres R. Narvasa that in accordance with Article 8 of the Civil Code which provides that "(j)udicial decisions applying or interpreting the laws or the

Constitution shall form part of the legal system of the Philippines," and Article 4 of the same Code which states that "(l)aws shall have no retroactive effect unless the contrary is provided," the principle of prospectivity of statutes, original or amendatory, shall apply to judicial decisions, which, although in themselves are not laws, are nevertheless evidence of what the law means. 69 Petitioner's contention that Article III, Section 12 of the 1987 Constitution should be given retroactive effect for being favorable to him as an accused, cannot be sustained. While Article 22 of the Revised Penal Code provides that "(p)enal laws shall have a retroactive effect insofar as they favor the person guilty of a felony who is not a habitual criminal," what is being construed here is a constitutional provision specifically contained in the Bill of Rights which is obviously not a penal statute. A bill of rights is a declaration and enumeration of the individual rights and privileges which the Constitution is designed to protect against violations by the government, or by individuals or groups of individuals. It is a charter of liberties for the individual and a limitation upon the power of the state. 70 Penal laws, on the other hand, strictly and properly are those imposing punishment for an offense committed against the state which the executive of the state has the power to pardon. In other words, a penal law denotes punishment imposed and enforced by the state for a crime or offense against its law. 71 Hence, petitioner's vigorous reliance on People vs. Sison 72 to make his extrajudicial confession inadmissible is misplaced. In that case, the extrajudicial confession was executed on May 19, 1983, clearly after the promulgation of Morales on April 26, 1983. The admissibility of petitioner's uncounselled waiver of the right to counsel notwithstanding, the Court has still to determine whether such waiver was made voluntarily and intelligently. 73 The waiver must also be categorical and definitive, 74 and must rest on clear evidence. 75 In his affidavit of May 30, 1982 waiving the provisions of Article 125 of the Revised Penal Code, 76 petitioner stated that:
. . . matapos akong mapagpaliwanagan ng mga imbestigador ng Special Operations Group, PC/INP Central Anti-Organized Crime Task Force, Camp Crame, Quezon City ng aking mga karapatan alinsunod sa mga isinasaad ng Section 20, Article IV ng Bagong Saligang Batas ng Republika ng Pilipinas ay malaya at kusang-loob na nagsasalaysay ng mga sumusunod kahit na walang abugadong magpapayo sa akin sa pagsasagawa nito sa dahilang alam at nauunawaan ko ang aking ginagawa at wala naman akong isasalaysay kung hindi mga katotohanan lamang, bagama't ako ay inalok ng mga imbestigador na ikuha ng isang abugadong walang bayad mula sa CLAO-IBP na akin namang tinanggihan: xxx xxx xxx Na ako ay hindi sinaktan a minaltrato gayunding walang kinuha mula sa akin na hindi niresibohan; xxx xxx xxx

Sgt. Arsenio Carlos, investigating officer, testified that he apprised petitioner of his right to counsel even in waiving the same right 77 but petitioner did not even inform him that his father-in-law was a lawyer. Although allowed to talk for thirty minutes with Jimmy Victorino, who was his comrade at the WPD General Assignment Section, 78 still, petitioner did not invoke his right to counsel. It should be emphasized that petitioner could not have been ignorant of his rights as an accused. He was a fourth year criminology student and a topnotch student in the police basic course. 79 Having been in the police force since 1978, with stints at the investigation division or the detective bureau, he knew the tactics used by investigators to incriminate criminal suspects. 80 In other words, he was knowledgeable on the matterof extrajudicial confessions. The Second Issue: Confession Extracted Through Torture? Petitioner's claim that he was tortured into signing the confession appears incredible, or at least susceptible to serious doubts. The allegation of torture was negated by the medical report 81 showing no evidence of physical injuries upon his person. As correctly observed by the Solicitor General, there is no reason to maltreat him in particular when the record shows that the investigating team respected the right of the other suspects to remain silent. When he was presented before Judge Mariano Mendieta of the municipal court in Meycauayan, petitioner even waived his right to present evidence 82 instead of impugning his confession on account of the torture allegedly inflicted upon him. If indeed he had been tortured, he would have revived the case he filed against his alleged torturers upon learning of its dismissal. Furthermore, an examination of his signatures in the different documents on record bearing the same discloses an evenness of lines and strokes in his penmanship which is markedly consistent in his certification, extrajudicial confession and waiver of detention. Human experience has proven that the lines and strokes of a person's handwriting reflect his disposition at a certain given time. In the present case, no handwriting expert is needed to declare that petitioner's signatures were written voluntarily and not under compulsion of fear immediately after he had been subjected to maltreatment. In view of the foregoing, his extrajudicial confession is presumed to have been voluntarily made, in the absence of conclusive evidence showing that petitioner's consent in executing the same had been vitiated. 83 Besides, the question of whether petitioner was indeed subjected to torture or maltreatment is a factual question addressed primarily to trial courts, the findings of which are binding on this Court whose function, as afore-discussed, is principally to review only of questions of law. Moreover, we have pored over the assailed Decision and we are satisfied that respondent Court performed its duty in evaluating the evidence. More on this later. The Third Issue: Illegal Arrest?

Petitioner questions the manner of his arrest, stating that the arresting officers "invited" him without a warrant of arrest and brought him to Camp Crame where he was allegedly subjected to torture almost a month after the commission of the crime. 84 Petitioner's claim is belatedly made. He should have questioned the validity of his arrest before he entered his plea in the trial court. On this point, this Court explained in People vs. Lopez, Jr.: 85
Finally, it is much too late for appellant to raise the question of his arrest without a warrant. When accused-appellant was arrested and a case was filed against him, he pleaded not guilty upon arraignment, participated in the trial and presented his evidence. Appellant is thus estopped from questioning the legality of his arrest. It is well-settled that any objection involving a warrant of arrest or procedure in the acquisition by the court of jurisdiction over the person of an accused must be made before he enters his plea, otherwise the objection is deemed waived. Besides, this issue is being raised for the first time by appellant. He did not move for the quashal of the information before the trial court on this ground. Consequently, any irregularity attendant to his arrest, if any, was cured when he voluntarily submitted himself to the jurisdiction of the trial court by entering a plea of not guilty and by participating in the trial. Moreover, the illegal arrest of an accused is not sufficient cause for setting aside a valid judgment rendered upon a sufficient complaint after trial free from error.

The only move petitioner made in regard to his arrest was to file a complaint for "grave coercion, grave threat & maltreatment" which was docketed as I.S. No. 82-12684 before the Fiscal's Office of Quezon City. 86 The complaint was an offshoot of his alleged maltreatment in the hands of the SOG upon his arrest. However, as stated above, he did not lift a finger to revive it upon its dismissal. The Fourth Issue: Sufficiency of the Prosecution's Evidence Contrary to petitioner's claim, his culpability has been proven beyond reasonable doubt. He borrowed a car to use in the hijacking knowing fully well that his owner-type jeep would give away his identity. Hecould not be identified by the postal employees in the postal van simply because after overtaking said vehicle and forcing its driver to pull over, he gave up driving the Mercedes Benz where the postal employees were made to ride, and commandeered the van. That the checks were not found in his own home is of no moment. Before the arrest and upon learning that the authorities had begun to nail down the identities of the malefactors, hehad entrusted them to his "kumare". It was petitioner himself who led the team of Lt. Pagdilao back to his place after he had admitted to Sgt. Arsenio Carlos that his share of the checks were in the possession of his "kumare" in the neighborhood. 87 In view of these facts, it is beyond dispute that petitioner was a direct participant in the commission of the crime. His alibi has been correctly considered by the Sandiganbayan to be weak and implausible. The distance between Kalvario, Meycauayan, Bulacan and downtown Manila where petitioner claimed to have been at the crucial time was between fifteen (15) to twenty (20) kilometers, which, through first-class roads, could be negotiated during that time in approximately thirty (30) minutes. It could not therefore have been physically impossible for him to be at the crime scene or its immediate vicinity when the crime was committed. 88

Having already ruled on the admissibility of petitioner's confession, this Court holds that the full force of the totality of the prosecution's evidence proves his guilt well beyond reasonable doubt. Weighing heavily against the defense is the well-settled doctrine that findings of facts of the trial courts in this case, the Sandiganbayan itself particularly in the assessment of the credibility of witnesses, is binding upon this Court, absent any arbitrariness, abuse or palpable error.
. . . It is well-settled that this Court will not interfere with the judgment of the trial court in passing on the credibility of the witnesses, unless there appears in the record some fact or circumstance of weight and influence which has been overlooked or the significance of which has been misapprehended or misinterpreted. The reason for this is that the trial court is in a better position to decide the question, having heard the witnesses 89 themselves and observed their deportment and manner of testifying during the trial. The doctrine is firmly settled that the trial court's conclusion on issues of credibility is accorded with highest respect by the appellate courts (People v. Dominguez, 217 SCRA 170). Appellate courts will generally respect the findings of trial courts on the credibility of witnesses since trial courts are in a better position to weigh conflicting testimonies. They heard the witnesses themselves and observed their deportment and manner of testifying. 90 ...

So overwhelming is the prosecution's evidence that respondent Court opined that even without the "inter-locking confessions of Filoteo, Mateo and Liwanag" the remaining evidence would still be sufficient for conviction. 91 Said the respondent tribunal:
However, even setting aside the inter-locking confessional statements of Filoteo, Mateo and Liwanag, we are of the considered opinion that substantial and sufficient evidence exist which indubitably prove the guilt of Filoteo, Relator, Mateo and Saguindel who had submitted themselves to the jurisdiction of this Court. As above-stated, Filoteo was responsible for securing the use of the Mercedes Benz car used by the co-conspirators in the hi-jacking. Together with Mateo, Liwanag and Mendoza, he surrendered voluminous assorted checks which were part of the loot. Relator admitted that his service firearm was used by him in the hi-jacking, which firearm was identified by prosecution witnesses Miranda and Bautista. Saguindel was identified in line-ups at the SOG office as the suspect clad in fatigue uniform and carrying an Armalite rifle by prosecution witnesses Tagudar and Bautista. All three (3) accused, namely, Mateo, Relator and Saguindel also jumped bail during the trial and did not offer any evidence to refute the evidence presented by the prosecution against them. Such flight to evade prosecution constitutes an implied admission of guilt. Moreover, accused Filoteo's and Mateo's unexplained possession of the stolen checks raises the presumption that they were responsible for the robbery in question. It is a rule established by an abundance of jurisprudence that when stolen property is found in the possession of one, not the owner, without a satisfactory explanation of his possession, he will be presumed the thief. This rule is in accordance with the disputable presumption "that a person found in possession of a thing taken in the doing of a recent wrongful act is the taker and doer of the whole act." In the instant case, said accused has not given such satisfactory explanation, much more so when their possession had been positively established by the testimonies of prosecution witnesses Capt. Ferrer and Sgt. Carlos and by accused's own signatures at the back of said checks. Furthermore, accused Filoteo's denials and alibi cannot be entertained for being quite weak and implausible. His claim that he merely borrowed the Mercedes Bent car from

Rodolfo Miranda to help out his co-accused Mateo, who had been utilized by the police as an "informer" and was following up tips in certain unsolved cases, appears to be incredible and fantastic. He also claimed that he could not have participated in the hi-jack because after giving the car to Mateo in the morning of May 2, 1982, he waited at the corner of Zurbaran St. and Avenida Rizal between 2-3:00 o'clock p.m. of the same day and then went to the WPD headquarters to attend the police formation at around 5:00 o'clock p.m. when Mateo failed to show up. Thereafter, he tried to show through his witnesses Gary Gallardo and Manolo Almogera that he was with them between 3:00 o'clock to 4:45 o'clock p.m., then from 6:00 o'clock to 8:30 o'clock p.m. and, finally, from 10:45 o'clock p.m. to 11:00 o'clock of the same date. It was through said witnesses that he tried to establish his whereabouts between 4:30 o'clock to 7:30 o'clock p.m. of May 2, 1982, the period from the time the mail van was hi-jacked up to when postal employees Bautista, Miranda and Tagudar were brought to Caloocan City and freed by their captors. Such alibi, however, fails to show that it was physically impossible for him to be present at the scene of the hi-jacking. We take judicial notice that the distance between the crime scene and down-town Manila is some 15-20 kilometers and negotiable over first- class roads in some thirty (30) minutes.

We are likewise convinced that there is sufficient evidence of conspiracy as convincing as the evidence of the participation of each of the accused. As ratiocinated in the assailed Decision: 92
The participation of accused Filoteo, Mateo, Relator and Saguindel in the criminal conspiracy have (sic) been proved beyond reasonable doubt by the evidence on record and which evidence not only confirms the existence of the conspiracy between them as easily discernible from their conduct before, during and after the commission of the offense, but also their participation therein as co-principals by direct participation and/or indispensable cooperation. Their concerted efforts were performed with closeness and coordination indicating their common purpose. Hence, there being collective criminal responsibility, the act of one is the act of all, and each of the participants are responsible for what the others did in all the stages of execution of the offense.

Final Question: Brigandage or Robbery? The Court believes that, though not raised as an issue and though not argued by the parties in their pleadings, the question of which law was violated by the accused should be discussed and passed upon. In fact, petitioner should have brought up such question as it may benefit him with a reduced penalty. The respondent Court convicted the accused of brigandage punishable under Presidential Decree No. 532. 93 Justifying the above disposition, the assailed Decision ratiocinates:
Accused herein are charged with the violation of Presidential Decree No. 532, otherwise known as the Anti-Piracy and Anti-Highway Robbery Law of 1974. Under said decree, with respect to the highway robbery aspect, the offense is committed on a "Philippine Highway" which under Section 2 (c) thereof has been defined as "any road, street, passage, highway and bridges or any part thereof, or railway or railroad within the Philippines, used by persons or vehicles, or locomotives or trains for the movement or circulation of persons or transportation of goods, articles or property or both", while under Section 2 (e) thereof "Highway Robbery/ Brigandage" has been defined as the "the

seizure of any person for ransom, extortion or other unlawful purposes or the taking away of property of another by means of violence against or intimidation of persons nor force upon things or other unlawful means, committed by any person on any Philippine Highway". (Emphasis supplied) The offense described in the information and established by the evidence presented by the prosecution properly falls within the ambit of the aforesaid special law. Therein, it was conclusively proven that a postal van containing mail matters, including checks and warrants, was hi-jacked along the national highway in Bulacan by the accused, with the attendant use of force, violence and intimidation against the three (3) postal employees who were occupants thereof, resulting in the unlawful taking and asportation of the entire van and its contents consisting of mail matters. Also the evidence further showed that the crime was committed by the accused who were PC soldiers, policeman (sic) and private individuals in conspiracy with their co-accused Castro and Escalada who were postal employees and who participated in the planning of the crime. Accordingly, all the essential requisites to constitute a consummated offense under the law in point are present. (Emphasis in the original text.)

Obviously, the Court a quo labored under the belief that because the taking or robbery was perpetrated on a national highway (McArthur Highway), ergo, Presidential Decree No. 532, otherwise known as the Anti-Piracy and Anti-Highway Robbery Law of 1974, must have been the statute violated. Such reasoning has already been debunked by this Court in the case of People vs. Isabelo Puno, 94 where it was ruled in unmistakable language that it takes more than the situs of the robbery to bring it within the ambit of PD 532. Said the Court through Mr. Justice Florenz D. Regalado:
The following salient distinctions between brigandage and robbery are succinctly explained in a treatise on the subject and are of continuing validity: The main object of the Brigandage Law is to prevent the formation of bands of robbers. The heart of the offense consists in the formation of a band by more than three armed persons for the purpose indicated in art. 306. Such formation is sufficient to constitute a violation of art. 306. It would not be necessary to show, in a prosecution under it, that a member or members of the band actually committed robbery or kidnapping or any other purpose attainable by violent means. The crime is proven when the organization and purpose of the band are shown to be such as are contemplated by art. 306. On the other hand, if robbery is committed by a band, whose members were not primarily organized for the purpose of committing robbery or kidnapping, etc., the crime would not be brigandage, but only robbery. Simply because robbery was committed by a band of more than three armed persons, it would not follow that it was committed by a band of brigands. In the Spanish text of art. 306, it is required that the band "sala a los campos para dedicarse a robar." (Emphasis ours.) In fine, the purpose of brigandage, is inter alia, indiscriminate highway robbery. If the purpose is only a particular robbery, the crime is only robbery, or robbery in band if there are at least four armed participants. The martial law legislator, in creating and promulgating Presidential Decree No. 532 for the objectives announced therein, could not have been unaware of that distinction and is presumed to have adopted the same, there being no indication to the contrary. This conclusion is buttressed by the rule on contemporaneous construction, since it is one drawn from the time when and the

circumstances under which the decree to be construed originated. Contemporaneous exposition or construction is the best and strongest in the law. Further, that Presidential Decree No. 532 punishes as highway robbery or brigandage only acts of robbery perpetrated by outlaws indiscriminately against any person or persons on Philippine highways as defined therein, and not acts of robbery committed against only a predetermined or particular victim, is evident from the preambular clauses thereof, to wit: WHEREAS, reports from law-enforcement agencies reveal that lawless elements are still committing acts of depredation upon the persons and properties of innocent and defenseless inhabitants who travel from one place to another, thereby disturbing the peace, order and tranquility of the nation and stunting the economic and social progress of the people: WHEREAS, such acts of depredations constitute . . . highway robbery/brigandage which are among the highest forms of lawlessness condemned by the penal statutes of all countries: WHEREAS, it is imperative that said lawless elements be discouraged from perpetrating such acts of depredations by imposing heavy penalty on the offenders, with the end in view of eliminating all obstacles to the economic, social, educational and community progress of the people; (Emphasis supplied.) Indeed, it is hard to conceive of how a single act of robbery against a particular person chosen by the accused as their specific victim could be considered as committed on the "innocent and defenseless inhabitants who travel from one place to another," and which single act of depredation would be capable of "stunting the economic and social progress of the people" as to be considered "among the highest forms of lawlessness condemned by the penal statutes of all countries, and would accordingly constitute an obstacle "to the economic, social, educational and community progress of the people, such that said isolated act would constitute the highway robbery or brigandage contemplated and punished is said decree. This would be an exaggeration bordering on the ridiculous.

From the above, it is clear that a finding of brigandage or highway robbery involves not just the locus of the crime or the fact that more than three (3) persons perpetrated it. It is essential to prove that the outlaws were purposely organized not just for one act of robbery but for several indiscriminate commissions thereof. In the present case, there had been no evidence presented that the accused were a band of outlaws organized for the purpose of "depredation upon the persons and properties of innocent and defenseless inhabitants who travel from one place to another." What was duly proven in the present case is one isolated hijacking of a postal van. There was also no evidence of any previous attempts at similar robberies by the accused to show the "indiscriminate" commission thereof. 95 Upon the other hand, the Information did not specifically mention P.D. 532. 96 The facts alleged therein and proven by the evidence constitute the offense of robbery defined in Art. 293 in relation to Art. 295 and punished by Art. 244, par. 5, all of the Revised Penal Code. 97 From the facts, it was duly proven that:

* personal property (treasury warrants, checks, mail, van, tools, etc.) * belonging to another were * unlawfully taken by the accused * with intent to gain (animo lucrandi) * with intimidation against three persons (Art. 293) * in an uninhabited place, or * by an band, or * by attacking a moving motor vehicle * on a highway; and * the intimidation was made with the use of firearms (Art. 295) Hence, the offender shall be punished by the maximum period of the penalty provided under paragraph 5 of Art. 294, which is, " prision correccional in its maximum period to prision mayor in its medium period". Effectively, the penalty imposed by the Court a quo should be lightened. However, such lighter penalty shall benefit only herein petitioner and not his co-accused who did not contest or appeal the Sandiganbayan's Decision. WHEREFORE, the petition is DENIED, but the first paragraph of the dispositive portion of the assailed Decision is partially MODIFIED to read as follows:
WHEREFORE, judgment is hereby rendered finding accused Jose Filoteo, Jr. y Diendo GUILTY beyond reasonable doubt as co-principal in the crime of robbery as defined in Arts. 293 and 295 and penalized under Art. 294, paragraph 5, of the Revised Penal Code Code IMPOSING on him an indeterminate sentence of four (4) years and two (2) months of prision correccional, as minimum, to ten (10) years of prision mayor as maximum, and to pay his proportionate share of the costs of the action.

All other parts of the disposition are hereby AFFIRMED. SO ORDERED.

G.R. No. 94494 March 15, 1996

PEOPLE OF THE PHILIPPINES, plaintiff-appellee, vs. DIONISIO LAPURA y CAJAN, accused-appellant.

VITUG, J.:p On appeal is the decision, 1 dated 04 June 1990, of the Regional Trial Court of Manila, Branch 12, in Criminal Case No. 88-61209, convicting herein appellant Dionisio Lapura y Cajan of murder and imposing on him the penalty of reclusion perpetua. The trial court has reserved to the heirs of the victim, Petronilo Lim, the right to file a separate suit for civil indemnification. The information, dated 01 March 1988, which opened the criminal case against appellant read:
That on or about February 19, 1988, in the City of Manila, Philippines, the said accused, conspiring and confederating together with others whose true names, identities and present whereabouts are still unknown and helping one another, did then and there wiffully, unlawfully and feloniously, with intent to kill, evident premeditation and treachery, attack, assault and use personal violence upon one Petronilo Lim, by then and there shooting him several times with a .45 caliber pistol hitting him on left anterior lumbar and left thigh, thereby inflicting upon said Petronilo Lim multiple gunshot wounds which were the direct and immediate cause of his death thereafter. Contrary to law.
2

Following his arraignment on 13 April 1988, at which the accused pleaded not guilty to the charge, a petition for bail was heard. On 14 September 1988, the trial court rejected the petition. A motion for the reconsideration of the denial order, itself, was later denied. The prosecution's evidence would tend to establish that at approximately 7:30 in the morning of 19 February 1988, Petronilo Lim, said to be a special agent of the Criminal Investigation Service, 3 was on board his car, with his sister, driving along Honorio Lopez Blvd., Balut, Tondo, Manila. Just as he started slowing down the car before turning left to Infanta Street, two persons suddenly came forward and fired at him. Edgardo Samson, a 27-year-old bicycle maker, then working for the Teen's Bicycle Enterprises at Honorio Lopez Blvd., was only around ten (10) meters away from the shooting incident. Instinctively turning his head to where the sound of gunshots emanated, he saw two persons a hunchback (kuba) who was positioned at the front right side of the car and another person at the left side of the vehicle. The victim, now all bloodied, got out of the car and fired back using his "baby" armalite. Instantly, a person who was wearing a white undershirt, "maong" pants and white shoes, grabbed the armalite but one Ambet Zabala immediately grappled for its

possession. Ambet succeeded in recovering the armalite which he turned over to "Amang" Manalo. The man in "maong" pants fled towards nearby San Rafael Village. On 26 February 1988, Samson executed a sworn statement before Senior investigation Agent Jesus Caizares and Sgt. Feliciano Garcia at the Special Investigation Unit, Criminal Investigation Service Command, Camp Crame, Quezon City. Later that day, he identified appellant in a police line-up to be the person who positioned himself at the left side of the victim's car and who fired a .45 caliber pistol at the victim. He executed another statement to this effect before Caizares. The victim, 55-year-old Petronilo Lim, died of three gunshot wounds two (2) at the left anterior lumbar area which lacerated the small intestine the liver, the kidney and the ascending colon, and one (1) "thru and thru" at the left thigh. 4 According to Dr. Marcial Ceido, who conducted the autopsy, the victim must have been fired at while his body was inclined towards the right 5 and that, because there was "tattooing" on his left posterior forearm, the muzzle of the gun must have been pointed at close range. 6 The defense interposed alibi. Appellant, then 32 years of age and a combo drummer by profession, testified that at the time the shooting took place he was sleeping at his sister's house in 1039 Int. 17, P. Vargas St., Tondo, Manila. He woke up rather late that day for he had performed the night before at the North Mall in Caloocan City. His sister, Adelaida Lapura Cuison, corroborating appellant's alibi. testified that on 19 February 1988, when she left the house at about 6:30 a.m., her brother was still sleeping in their living room and, coming back to the house around thirty minutes later, she found him still asleep. He did not wake up until around ten o'clock that morning. Appellant was arrested by police officers in the afternoon of 25 February 1988 at the house of a fellow musician; Danilo Cabrera, in Mata Street, Divisoria, Manila. Appellant and Cabrera, along with another musician friend, Reynaldo Eliezer, were brought to Station 1 at North Bay, Tondo, Manila, where statements were taken. They were later brought inside the office of Col. Maganto where appellant was informed that his being a suspect in the killing of Petronilo Lim was because he resembled the cartograph of the killer. Cabrera added that they were watching television when the arresting policemen suddenly entered their house shouting, "You are NPA's, no one must move. 7 After evaluating the evidence presented before it, the trial court gave credence to the prosecution's case, particularly to the eyewitness account of Samson, and accordingly rendered judgment convicting appellant of murder. In this appeal, it is initially argued that the trial court should have outrightly dismissed the case against appellant considering (a) that the information was filed without the written authority of the city fiscal or prosecutor and (b) that the certification of the investigating fiscal appended to the information was defective for (i) not being under oath, (ii) having failed to state that a preliminary investigation under Section 3, Rule 112, of the Rules of Court was conducted, and (iii) not having stated that the accused was

duly informed of the complaint and given the opportunity to present controverting evidence. The sufficiency of the allegations found in the complaint, conformably with Section 6, Rule 110, of the Rules of Court, 8 has not been questioned; what, instead, is being assailed centers on the supposed failure of the investigating prosecutor to obtain the prior written authority of the city prosecutor in the manner required under Section 4, 9 Rule 112, of the Rules of Court, before the filing of the case. This assertion contradicts the certification of the investigating fiscal attesting to the fact that the information has been duly filed under the authority of the City Fiscal; viz:
I hereby certify that an ex-parte investigation in this case has been conducted by me in accordance with law; that there is reasonable ground to believe that the offense charged has been committed; that the accused is probably guilty thereof and that the filing of this 10 information is with the prior authority and approval of the City Fiscal. (Emphasis supplied.)

Absent convincing evidence to the contrary, the presumption of regularity in the performance of official functions has to be upheld. Moreover, this matter should have been raised below in a proper motion to quash 11 that appellant could have done but did not. Relative to the claim that the certification did not fully comply with the requirements of Section 4, 12 Rule 112, of the Rules of Court, we need merely to reiterate the settled rule that such certification is not an indispensable part of, let alone invalidate even by its absence, an information. 13 In People vs. Marquez, 14 the Court has had occasion to explain:
. . . . It should be observed that section 3 [now section 4] of Rule 110 defines an information as nothing more than "an accusation in writing charging a person with an offense subscribed by the fiscal and filed with the court." Thus, it is obvious that such certification is not an essential part of the information itself and its absence cannot vitiate it as such. True, as already stated, section 14 of Rule 112 enjoin that "no information . . . shall be filed, without first giving the accused a chance to be heard in a preliminary investigation," but, as can be seen, the injunction refers to the non-holding of the preliminary investigation, not the absence of the certification. In other words, what is not allowed is the filing of the information without a preliminary investigation having been previously conducted, and the injunction that there should be a certification is only a consequence of the requirement that a preliminary investigation should first be conducted.

As the Court has also said in Pecho vs. Sandiganbayan 15


If the absence of a certification would not even invalidate the information, then its presence, although deficient because of some missing clauses or phrases required under Section 4, Rule 112 of the Rules of Court, can do nothing worse than the former.

In passing, the question of whether or not a preliminary investigation has been properly conducted is itself one that should be interposed prior to an arraignment. 16 It does not here appear that appellant did before entering his plea of "not guilty" to the charge.

On the merits of the case, appellant faults the trial court for believing the testimony of Samson despite supposed contradictions and inconsistencies of the witness. A close look at the records betrays any validity to the allegation. First of all, in his; first sworn statement, Samson identified the three perpetrators 17 of the crime; thus:
16. T Maari mo bang masabi ang mga anyo ng tatlong lalake na tinutukoy mo? S Yon nasa harap ng kotse ay medyo kuba at katamtaman ang katawan at kulay ng balat, at iyon nasa gawing kaliwa ng kotse ay mahaba ang buhok, may bigote, mataas at regular ang lake ng kanyang katawan at katamtaman ang kulay ng balat at iyon dumamput no Armalite ni Mr. Lim 18 at (sic) payat at mababa at kayumanggi ang kulay.

Then, in his second sworn statement, Samson pointed to appellant:


04 T Ihinaharap namin sa iyo ngayon ang anim na lalaki na nadito (sic) sa loob ng tanggapan ng Special Investigation Unit ng CIS Camp Crame, Quezon City maari mo bang makilala at maituro sa mga lalaki na ihinarap sa iyo kung meron man sa kanilaang bumaril kay Petronilo Lim? S Mayroon pong isa yong lalaking may bigote na nakasuot ng puting Tshirt at maong na pantalon na ikalawa sa aking gawing kaliwa. (Affiant pointing to the person of Dionisio Lapura y Cajan in a Police line-up composed of six persons) 05 T Ito bang tao na itinuturo mo ngayon ay nasisiguro mo siya na isa sa tatlo na magkakasama na bumaril kay Petronilo Lim na tinutukoy mo sa iyong salaysay? S Opo siya ang isa sa tatlo na tinutukoy ko na bumaril kay Mr. Petronilo Lim at siya iyong bumaril na nasa gawing kaliwa ng 19 kotse. (Emphasis supplied.)

Most importantly, on the witness stand, Samson, although visibly shaken 20 and notwithstanding the vigorous and dramatic 21 cross-examination by defense counsel, still stood by his statement that appellant was the assailant who fired from the left side of the victim's car. The defense could not attach any evil motive on the part of Samson that might have impelled him to testify falsely against appellant. 22 Absent the most compelling reason, it is inconceivable why Samson would openly concoct a story that would send an innocent man to jail. His credibility has likewise been tested and favorably evaluated by the trial court, a matter which cannot just be ignored. Appellant contends that he has not been provided with the assistance of coirnsel at the "police line-up." He suggests that the Court take another look at its rulings on the right of an accused to counsel during a "police line-up." 23

Upon his arrest, appellant Lapura refused to make any statement. 24 During the investigation that followed, appellant's counsel, Atty. Orlando Salatandre, was in the investigation room, a fact which, unexpectedly, he himself elicited from Agent Caizares on cross-examination. Thus:
Q But do I recall from you that you have noticed this Samson pinpointing one of them? A Yes, sir, that is part of his statement, the identification of the suspect. Q Despite the fact you were busy during that line up? A I was busy supervising the investigation but the Special Investigation Unit is only a small room that you could notice what is happening within. Q Considering the fact you were present, did you notice the accused before he was included in this line up whether he was given a chance to look for a lawyer to assist him during that line-up as part of the investigation? FISCAL Objection, Your Honor, the question is premised on the fact; it's one of the constitutional rights of the accused to be assisted by counsel during the police line up ATTY. SALATANDRE. If Your Honor please, the line-up is already part of the investigation, Your Honor please. Then, the accused therefore, must be assisted by counsel because who knows it's done without prejudice to the procedure, Your Honor please. COURT Let the witness answer. A If I am not mistaken, I do not only remember exactly the time element. If I remember correctly on that same night you were there, you appeared there. ATTY. SALATANDRE Yes, I was there.
25

Given the above circumstances, it would be wrong to conclude that Atty. Salatandre was unable to provide legal assistance to appellant. Moreover, the inadmissibility of a police line-up identification of an uncounseled accused should not necessarily foreclose the admissibility of an independent in-court identification. In this instance, Samson, in open court, pointed to appellant and

identified him again to be the person dressed in white T-shirt, pants and shoes who had come running towards him and the bridge after shooting Mr. Lim. 26 Proof of motive, contrary to appellant's allegation, is not crucial where the identity of the accused, such as here, has been amply established. 27 Appellant contends that People vs. Opida, 28 where the Court exonerated the appellant for nonobservance of certain of his constitutional rights as an accused, should also be applied in acquiring him. He alleges that
Like accused-appellants Olpida (sic) and Marcelo in that case, the constitutional rights of herein appellant have been grossly violated. From the very time that he was arrested without warrant on February 25, 1988 and detained at WPD station, Tondo, Manila in connection with the murder of Petronilo Lim, five days after its perpetration on the flimsy ground that his face resembles that of man drawn in a cartograph prepared by the police, perhaps based on the descriptions of people who may have witnessed the crime, to the filing of the fatally flawed information on March 2, 1988 or 7 days after his unlawful arrest, he was deprived of his constitutional rights against unreasonable search and seizure, of his right against arbitrary or unlawful arrest, of his right to remain silent and to counsel and to be informed of said rights, of his right to a preliminary investigation in a crime cognizable by the Regional Trial Court and finally of his right to have a valid information against him filed within the period allowed by law implied in Article 125 of the Revised 29 Penal Code the provisions of which he had never waived.

We agree with the Solicitor General that Opida is predicated on two vital premises; viz: (a) the trial judge's palpable partiality, as well as the irregular manner in which he conducted his interrogation of the accused and their witness, and (b) the admission of an extrajudicial confession despite strong evidence of manhandling by the police. These circumstances do not obtain in this case. During trial, while the judge did propound questions to the witnesses, they clearly appear, however, to be in the main clarificatory and certainly not adversarial in character. 30 Relative to his alleged warrantless arrest, he has waived, by filing a petition for bell, any irregularity attendant thereto. 31 Indeed, by his application for bail and by entering a plea of not guilty and then submitting to the proceedings below, appellant must be deemed to have foregone his right to preliminary investigation and to question any irregularity that might have attended such investigation. 32 The trial court correctly appreciated the aggravating circumstance of treachery. Treachery attended the killing since the attack was perpetrated suddenly and without warning of any kind. 33 Evident premeditation, although alleged in the information, was not at all established. Absent any mitigating or aggravating circumstance, the penalty due for imposition would be, in the penalty of reclusion temporal maximum to death prescribed by Article 248 of the Revised Penal Code, its medium Period of reclusion perpetua. 34 WHEREFORE, the Decision in Criminal Case No. 88-61209 of the Regional Trial Court of Manila, Branch 12, convicting appellant Dionisio Lapura y Cajan of the crime of

murder for the killing of Petronilo Lim and imposing on him the penalty of reclusion perpetua is hereby AFFIRMED. Costs against appellant. SO ORDERED.

G.R. No. 116011 March 7, 1996 PEOPLE OF THE PHILIPPINES, plaintiff-appellee, vs. RHODESA* SILAN y BORQUE, alias "Dada," and VIRGILIO GARCIA, alias "Billy Garcia," accused-appellants.

REGALADO, J.:p Accused-appellants were prosecuted for the special complex crime of robbery with homicide in Criminal Case No. 93016 of the Regional Trial Court, Branch 163, of Pasig, Metro Manila upon an indictment filed on June 17, 1992 and alleging
That on or about the 1st day of June, 1992, in the Municipality of Marikina, Metro Manila, Philippines and within the jurisdiction of this Honorable Court, the above-named accused, conspiring and confederating together with John @ "Tol", whose true nam(e), identity and present whereabouts (are) still unknown and all of them mutually helping and assisting one another, by means of force, violence and intimidation employed upon the person of Evangeline Gargantos, did then and there willfully, unlawfully and feloniously take, steal and carry away complainant's personal belongings and valuables all amounting to P6,500.00, to the damage and prejudice of the offended party in the aforementioned amount of P6,500.00; that on the occasion of the said robbery, the above-named accused, conspiring and confederating together with John Doe @ "Tol", whose true name, identity and present whereabout(s) (are) still unknown and they all mutually helping and assisting one another, with evident premeditation and treachery, with the use of superior strength and during nighttime, while armed with bladed weapon, with intent to kill, did then and there willfully, unlawfully and feloniously attack assault, strangle with a cord of an electric stove, and stab said Evangeline Gargantos, thereby inflicting upon her 1 fatal injuries which caused her death.

They were both found guilty as charged in the decision of said court, dated October 21, 1993, which adjudged as follows:
WHEREFORE, foregoing premises considered, this Court finds both accused, Rhodesa Silan and Virgilio Garcia guilty as principal(s) of the crime of Robbery with homicide defined in Article 293 and penalized under Article 294. And, considering the aggravating circumstance of the crime having been committed in the dwelling of the offended party and there being no mitigating circumstance on record this Court sentences them to suffer the penalty of reclusion perpetua with its accessories provided for by law, to restitute to

the heirs of Evangeline Gargantos the value of the article stolen from her house in the amount of P6,500.00; to indemnify said heirs (in) the sum of P50,000.00 and to pay the cost. If accused has signed a written conformity to abide with the rules on convicted prisoner, their detention service if any should be deducted from this sentence computed as 2 provided for by law.

We have carefully and thoroughly reviewed the records of this case, especially the transcripts of the stenographic notes taken at the trial, the documentary evidence, and the respective presentations of the parties, and we find that these findings of the court below which were adopted in the People's brief provide us sufficient and accurate bases for this appellate review:
Investigation conducted by police investigator Ricardo Domingo revealed that Rhodesa Silan was seen by Carol Concepcion about 2:00 P.M. on either 31 May or 1 June 1992. Silan inquired from Carol as to whether or not there was anyone in the house of her auntie. Carol told her that there is nobody there because they are (o)n vacation. Silan told Carol that she would like to get something inside the house. About 7:00 o'clock in the evening of the same day Carol while fetching water, again saw Silan with a male companion going to the house of Evangeline Gargantos at the back. Carol did not recognize the male companion of Silan (TSN page 4, dated 23 Sept. 1992). Investigator Domingo took the statement of Rhodesa Silan who at the time was assisted by Atty. Salvador Navarro. (p. 5, November 4, 1992, TSN). In her extra-judicial statement (Exh. "H") Silan stated that she went to the house of her auntie Vangie in the late afternoon of 1 June 1992. (p. 4, July 1, 1993, TSN). Her companions in going to Vangie's house were Virgilio "Billy" Garcia and a certain Tol. (p. 4, ibid.) They went to Vangie's house to get her things so they can sell them as Billy needed money. (p. 4, ibid.) Upon reaching the house of Vangie they found it closed. They went to the back portion where Billy used a screw driver in destroying the padlock of the kitchen door. (p. 5, ibid.) Once the kitchen door was opened, they entered the house, Silan going to her room while Billy and Tol went upstair(s). (p. 5, ibid.) While Silan was arranging her things she heard a loud sound of objects falling. She went up the house and found that an electric fan fell on the floor. She saw Billy and Tol searching something. (p. 5, ibid.) She also notice (sic) that the room was in disarray. It was at this juncture that Billy told her to get things that she wants. She then got two jackets, coleman, colored blue dress of Remia, perfume, make-up kit, t-shirt, girdle of Remia and plate with saucer. When they went down from the second floor of the house Billy talked with her and forced her to go to her former room where he forced her to have sex with him. It was at that instance when her auntie Vangie arrived and knocked at the door. (p. 6, ibid.) Billy then ordered her to stand up in one corner and told her he will take care of things. He then put off the light and opened the door and allowed her auntie to come in. (p. 6, ibid.) After her auntie entered the house there was a commotion. (p. 6, ibid.) She heard her auntie shouting "Who are you, why did you enter the house" followed by shouts for help "Aling Lina, Aling Lina, tulungan ninyo ako, may gustong pumatay sa akin". (p. 6, ibid.) Soon thereafter silence followed. Then Billy returned for her and pulled her out the house. (p. 7, ibid.) Some of the articles taken by Silan from the house of Evangeline Gargantos were recovered from the house were she is now living with her live-in partner. Said articles were identified in Court by SPO1 Celso Cruz, the evidence custodian to whom the recovered articles were turned over after the same were recovered. (p. 3, October 22, 1992, TSN) Remia Gargantos Gillo, a niece of the victim who also lived with Vangie in #29 Sable St., SSS Village, identified the blouse, the t-shirt, a bottle of perfume and

powder case as that belongin(g) to the family of the deceased. (p. 6, October 22, 1992, TSN) Atty. Salvador Navarro affirmed before the Court the fact that he assisted Rhodesa Silan when she gave her extra-judicial statement before police investigator Ricardo Domingo. 3 (pp. 5-7, ibid) (pp. 2-3, Decision)

It is appellant Silan's position, as articulated in her lone assignment of error, that the lower court erred in convicting her of the crime of robbery with homicide without proof of conspiracy between her and appellant Garcia having been sufficiently established by the prosecution. 4 Parenthetically, appellant Silan also presented herself as the sole witness in her defense. Appellant Garcia has a more extended assignment of errors in his brief, 5 contending that the court below erred (1) in relying on the testimony and extrajudicial confession of appellant Silan in convicting him; (2) in failing to consider that appellant Silan had sufficient motive to implicate him in the crime charged; (3) in quickly dismissing his defense of alibi; and (4) in not considering the illegality of his arrest by the police. We absolve the trial court of the aforestated errors imputed to it and which we shall discuss seriatim conjointly with our reasons for rejecting the same, starting with the submissions of the lady appellant first. 1. Appellant Silan leads off with the argument that her only intention in going to the house of her aunt, the victim Evangeline Gargantos, was to get some of her things which she had left behind when she left the place. She claimed that she intended to sell the same and give the money to appellant Garcia because of his threat to kill her brother if she did not do so. She denied that there was any prior agreement between her and appellant Garcia or his companion, called "Tol," to rob and kill her aunt, and that those crimes were committed by the latter two without her participation or consent. We reject this hand washing attempt. It is established that the three of them first agreed to meet at a specific place on an appointed time and, from there, they then proceeded together to the victim's house at 29 Sable Street, SSS Village, Marikina, Metro Manila. The events that transpired after they had forced their way into the house clearly revealed how they cooperated with each other to successfully loot the house and escape undetected through the back door. When appellant Silan was in her former room, she said she heard noises coming from upstairs and that when she went up and saw Garcia and Tol taking things from the room of her aunt, she asked them to desist but they refused. According to her, this proved that she did not consent to those larcenous acts of her companions. We believe otherwise. Her simple admonition, even if true, is not evidence of a true intent to prevent her associates from taking valuables from the house. She could and should have insisted, for apparently there was no danger to her at that moment, but she did not do so. Further belying her supposed solicitude is the fact that she readily accepted the white bag containing part of the items taken from the room of the victim. If

she really had no intention to gain, she should have returned the white bag which she admittedly opened and wherein she discovered valuables belonging to her aunt and the latter's two sons. 6 On the contrary, the extrajudicial admissions in her sworn statement 7 which she voluntarily executed on June 15, 1992 at the Marikina Police Station, with the legal safeguards discussed hereinafter, clearly spell out in detail that she and her aforesaid confederates planned the robbery. Her act of taking things when she was allegedly instructed by appellant Garcia to take whatever she liked necessarily made her a principal by direct participation through confederacy in the robbery. What also caught our attention is the testimony of prosecution witness Carol Concepcion that she saw and talked to appellant Silan not only on June 1, 1992 but also prior thereto, that is, in all probability on May 31, 1992 likewise near the very house of the victim at Marikina. On May 31, 1992 when witness Concepcion saw appellant Silan at around 2:00 P.M., the latter asked her whether there was a man in the house, informing the former that she was going inside the house to get a few things which she had left behind. 8 Viewed from the events that transpired thereafter, her testimony that the reason why she went with her companions to the house of her aunt on June 1, 1992 was just to get her things is extremely doubtful and obviously contrived for she could have taken everything that she left behind the first time that witness Carol Concepcion saw her on May 31, 1992. It would, therefore, not be far-fetched to assume that during this first meeting with Carol Concepcion at the residence of Evangeline Gargantos, appellant Silan was actually conducting a surveillance of the place prior and consequent to the plan to ransack the same on June 1, 1992. It is likewise plausible to assume that she had reason to be angry with her aunt for she was practically thrown out of the house, because of her having stolen the camera of the latter and pawning the same, aside from previous similar incidents. In her testimony, she admitted that the reason why she left the house was because she was forced out due to the argument with her aunt regarding that camera. 9 It is apparent that she had every opportunity to return the things which she took from her aunt's house but she did not. With regard to the killing of her aunt, it is indeed out of the ordinary for appellant Silan not to even bother to find out what happened to her own aunt when, in fact, the last that she heard from the victim was the latter's cries for help after appellant Garcia and Tol went to intercept her at the door. Appellant Silan may not have participated in the actual killing of Evangeline Gargantos for in truth there is no evidence specifically showing who actually tied the electric cord around the victim's neck and stabbed her twice. However, the series of events prior to the June 1, 1992 incident, the testimony of witness Carol Concepcion, and the manifest concerted acts of appellant Silan and her two confederates as recounted by the appellant Silan herself, irresistibly suggest that

appellant Silan did participate in the planning and commission of the composite crime charged. Although the killing of Evangeline Gargantos may just have been an unfortunate complication and was not part of the original plan of the group to commit robbery, there being conspiracy among the three malefactors, the crime of one is the crime of all. Reiterating our previous holdings on this issue, People vs. Nunag, et al. 10 stressed the rule that whenever homicide has been committed as a consequence or on the occasion of the robbery, all those who took part as principals in the robbery will also be held guilty as principals for the special complex crime of robbery with homicide, although they did not actually take part in the homicide. 2. We are not at all impressed by appellant Garcia's exculpatory attempt to attribute his being enmeshed in this criminal case to the extrajudicial statement of appellant Silan. He assails the same for allegedly having been obtained by the police authorities without appellant Silan having been provided with counsel. Ironically, this claim was likewise belatedly made by Silan in a tragicomic deviation after she had voluntarily narrated in her aforesaid statement the details of her participation in the events that took place in the victim's residence. How appellant Silan came to give that statement, and how it was reduced to its present form as submitted in evidence, is vividly reported in the testimony of SP04 Ricardo S.L. Domingo, the police investigator. 11 The statement was taken down in the presence of this appellant's own mother, Catalina Silan, as indicated therein and whose signature is affixed to the pages thereof. Said appellant was assisted by Atty. Salvador Navarro, whose appearance and attendance during the interrogation and preparation of this document is also specifically mentioned therein and on the first page whereof he also affixed his signature. On top of that, said Atty. Navarro testified in court 12 on his assistance given to appellant Silan throughout the entire proceeding and how the constitutional injunctions mentioned in the sworn statement were duly observed. And, if that would not be enough, appellant Silan herself took the witness stand and affirmed to the trial court her participation in the crime, virtually reiterating what she stated in her extrajudicial statement, thus:
Q Ms. witness, on June 1, 1992 at about 5:30 P.M., you remember where you were? A Yes, ma'm. Q Where were you? A At Caloocan, Quezon City in Bonifacio. Q What were you doing in Bonifacio? A I was with "Billy" Garcia and his friend (whom) he used to call Tol.

xxx xxx xxx Q What happened? A He (was) asking for (a) sum of money. Q Did you give him money? A No, ma'm Q Why? A Because I (did) not have money. Q So, when you did not give him money, what did you do? A I remembered that when I left Sable Street, and I have few belongings left behind and then he asked me if we can go there? Q What is that place? A Sable St., SSS Village, Marikina, Metro Manila. Q Who owns that house at SSS Village? A My auntie. xxx xxx xxx Q Were you able to go to that house? A Yes, ma'm. Q You said you went to that SSS Village, who were your companion(s)? A Billy Garcia. Q Only? A And a certain friend of hi(s) which I don't know the name. He just called him "Tol." xxx xxx xxx Q Were you able to reach that house of Mrs. Gargantos? A Yes, ma'm. Q What time did you arrive? A At around 7:00 o'clock in the evening, ma'm.

Q Upon reaching the house, what did you do, the three of you? A When we reached the house the door was close(d) and I told them, we just leave the house. Q Did you leave the house? A No, ma'm, because he was insisting to enter the house. Q Who (was) insisting? A Billy Garcia. Q What did he tell you? A He told us, we just enter so that we can get my belongings. Q So, did you enter the house? A Not immediately, ma'm. Q Why? A Because the door (was) locked. Q (How) were you able to unlock the door? A He brought out a screw driver and unscrew(ed) the lock. Q The door was destroyed, (wasn't) it? A The padlock only was destroyed. Q Did you enter the house? A When we entered the house, I went to my room. Q How about the other 2, where did they go? A They also entered the house, and went upstair(s). Q How did you know? A Because when I (was) in my room downstair(s), I heard a noise upstair(s), as if something fell, so I went upstair(s) and saw that the door was also opened and unlocked. Q What did you notice? A Bill(y) Garcia and his friend (were) already upstair(s) and the room was in disarray.

Q What did these two person do? A As if they are searching for something. Q What did you do, when you saw them as if they are searching for something? A I tried to stop them. Q And what was their reply? A Billy Garcia said, "Don't make any noise. Huwag kang makialam dito, kami na ang bahala dito." Q What did they do? A They kept on searching and I went downstair(s), I continued arranging my belongings. Q After you were through in arranging your belongings, can you please tell us what happened next? A Billy Garcia handed me a white bag and we were about to leave. Q You were about to leave, what happened? A When we were about to leave, my auntie came. Q How did you know? A She was knocking at the door. Q Who open(ed) the door? A Before she enter(ed), Billy Garcia turned off the light in the house. Q After it was turned off, what happened next? A He told me not to panic because we can get out of the house. Q (Was) your auntie able to enter the house? A Yes, ma'm. Q What happened, if you know? A When I was in my room I heard there was a commotion, something happening. Q What is the commotion all about?

A There was something happening in the house. Q What did you hear? A My auntie shouting. Q Shouting of what, what was her statement? A She was shouting, "Aling Lina, Aling Lina, please help me." Q What did you do, whe(n) she was asking for help? A None, ma'm. Q Why? A Because I ha(d) a mixed emotion at that time. I (was) afraid and nervous. xxx xxx xxx Q You said there was a commotion and your auntie shouting for help, what happened after that? A She (was) also shouting that somebody was trying to kill her and then after a few moments, there was silence. And Billy Garcia went to my room and told me we were leaving. Q So, after Billy Garcia entered your room, you left the house? A Yes, ma'm. Q Where did you pass? A The back door. Q Do you know what happened to your auntie before leaving the house? A No, ma'm. Q You said a while ago that your auntie (was) dead, when for the first time did you learn that she was dead? A Few days before my arrest.
13

(Corrections in parentheses supplied)

Give or take a few passages, the aforequoted testimony of appellant Silan is a repetition of what she declared in her sworn statement aforementioned. The same, however, is further challenged by appellant Garcia as being inadmissible in evidence as an extrajudicial statement which did not have all the requisites for its admission as an exception to the rule of res inter alios acta. Although we do not agree with his strained

dissertation thereon, it would be pointless to further dwell on that objection since the contents of the same have been reproduced as judicial admissions by appellant Silan's testimony in court. Her admissions having been laid open for cross-examination, as in fact she was cross-examined thereon, the initial objections of appellant Garcia have been rendered academic. There being no legal obstacle, on either constitutional or evidentiary grounds, to our considering the inculpatory statements of appellant Silan both against her and appellant Garcia, we find said testimony of the former to be credible just as the trial court also lent its imprimatur thereto. Appellant Garcia's involvement and incrimination in this case, contrary to his desperate asseverations, rests on a firm basis. Significantly, the brief of appellee even confines its arguments to the representations made in appellant Garcia's brief, but does not even bother to make any substantial comment on that of appellant Silan. Appellant Garcia contends that the trial court failed to consider the fact that appellant Silan had a sufficient motive to implicate him in the offense with which they both now stand charged. He refers to the amorous relationship between them and later, between her and his brother, both of which associations "turned sour." We likewise reject this speculative hypothesis. As the prosecution points out, the trial court relied upon the testimony of appellant Silan not only because of her extrajudicial statement but also because she categorically testified that appellant Garcia was one of the culprits in the offense charged. It was convinced, and so are we, that her aforesaid testimony, minus the understandable attempts at justification of her acts and mitigation of her liability, was credible and can be fully appreciated against appellant Garcia. He completely failed to present any evidence on the alleged ill motive of appellant Silan. Surprisingly, he even admits in his brief that, on the matter of the unrequited relationships between Silan and him or his brother, "there is no direct relation that would convince any court that this could have spur(r)ed Silan in implicating (him)" in the offense charged and now under review. 14 Appellant Garcia further insists that although he was identified by appellant Silan as her co-conspirator and as one of the perpetrators of the offense, no prosecution witness made that positive identification, hence his alibi should be sustained. We do not believe that there is any logic to what he would now require in addition to the eyewitness account of his own co-conspirator declared in open court subject to such crossexamination as the defense desired to conduct. We accordingly quote with approval what the trial court observed on this point:
As to the accused Virgilio Garcia, the extra-judicial confession of her co-accused Rhodesa Silan, implicating him in the commission of the crime charged is admissible against him if such extra-judicial confession is used only as circumstantial evidence to show the probability of his participation (People vs. Victor, 181 SCRA 818). But in the case at bar, in addition to the extra-judicial confession of Rhodesa Silan, she also testified in open court pointing to her co-accused Virgilio Garcia as her companion in entering the house of the victim by opening its back door with the use of screw driver.

She also pointed to Garcia as the person who ransacked the bedroom of the house of the victim. She likewise pointed to Garcia as the person who ordered her to stand by in her former room while he opens the door of the kitchen and allowed the victim to enter the house. She further stated that after the victim entered the kitchen door, there was a commotion and she heard the victim shouting for help and that when the victim became 15 silent Garcia returned to where she was, pulled her and together they left the place.

Finally, appellant Garcia bewails and assigns as reversible error the failure of the trial court to take into account his supposed illegal arrest which later culminated in the filing of the charges against him. We note, however, that he never objected thereto nor placed that matter in issue when, instead, he entered his plea on arraignment and went to trial. As the Solicitor General points out, even assuming that he was illegally arrested, this will not affect his culpability since an allegation of a warrantless arrest cannot deprive the State of its right to convict the guilty when all the facts on record point to his culpability. 16 We also agree with the advertence to our pronouncement in the aforecited case of Briones which could very well be said of this incident in the case at bar:
Immediately after their arrest, appellants . . . could have objected to the legality thereof due to the failure of the police officer to secure first a warrant for their arrest. Not only that, without having questioned the legality of their arrest, they even pleaded, on arraignment, to the information filed against them. Appellants' acts constitute a clear waiver of their right against unlawful restraint of liberty. Besides, it would be impractical, if not ridiculous, to order the court a quo to set the appellants free then issue a warrant for their arrest, and try them all over again when appellants themselves have waived their right to object to such irregularity and when their objection is truly based on overwhelming evidence.

ON THE FOREGOING CONSIDERATIONS, the appealed judgment of the court a quo is hereby AFFIRMED in toto, with costs against accused-appellants Rhodesa Silan y Roque and Virgilio Garcia. SO ORDERED.

20thCentury Fox v CA
Date (19 August 1988) | Ponente: Gutierrez Jr. Overview: Search warrants were recalled because the NBI and witnesses misrepresented that they had personal knowledge of the piracy. Statement of the Case -The lower court later on lifted the 3 search warrants and ordered the NBI to return the properties thatwere seized.- C A d i s m i s s e d M R s . Statement of Facts -August 26, 1985: a letter -complaint by petitioner 20thCentury Fox Film Corporation through counselsought the National Bureau of Investigation's (NBI) assistance in the conduct of searches and seizuresin connection with the latter's anti-film piracy campaign.-Specifically, the letter -complaint alleged that certain videotape outlets all over Metro Manila are engaged in the unauthorized sale and renting out of copyrighted films in videotape form which constitute aflagrant violation of Presidential Decree No. 49(otherwise known as the Decree on the Protection of Intellectual Property).-Acting on the letter -complaint, the NBI

conductedsurveillance and investigation of the outlets pinpointed bythe petitioner and subsequently filed three (3) applicationsfor search warrants.- S e p t e m b e r 4 , 1 9 8 5 : t h e l o w e r c o u r t i s s u e d t h e d e s i r e d s e a r c h w a r r a n t s . -The NBI accompanied by the petitioner's agents, raided the video outlets and seized the itemsdescribed therein.-An inventory of the items seized was made and left with the private respondents. -The lower court later on lifted the 3 search warrants and ordered the NBI to return the properties that were seized. Applicable Laws: Section 2, Article Ill, 1987 Constitution. Issues: 1.Was there grave abuse of discretion on the part of the lower court when it lifted the search warrants itearlier issued against the private respondents? No. Rationale 1.In the instant case, the lower court lifted the three questioned search warrants again st the privaterespondents on the ground that it acted on the application for the issuance of the said search warrantsand granted it on the misrepresentations of applicant NBI and its witnesses that infringement of copyright or a piracy of a particular film have been committed- A s f o u n d o u t b y t h e court, the NBI agents who acted as witnesses did not havepersonal knowledge of the subject matter of their testimony which was the allegedcommission of the offense by the private respondents.Only the petitioner's counsel who was also a witness during the application for the issuance of the search warrants stated that he had personal knowledge that theconfiscated tapes owned by the private respondents were pirated tapes taken from master tapes belonging to the petitioner. However, the lower court did not give much credenceto his testimony in view of the fact that the master tapes of the allegedly piratedtapes were not shown to the court during the application. The essence of a copyright infringement is the similarity or at least substantialsimilarity of the purported pirated works to the copyrighted work . Hence, the applicant must present to the court the copyrighted films to compare them with the purchasedevidence of the video tapes allegedly pirated to determine whether the latter is anunauthorized reproduction of the former. This linkage of the copyrighted films to thepirated films must be established to satisfy the requirements of probable cause. Mereallegations as to the existence of the copyrighted films cannot serve as basis for theissuance of a search warrant.-Search warrant must contain a specific description of the articles to be seized. General warrants are constitutionally objectionable. Judgment: Petition dismissed.

Columbia Pictures Entertainment vs Court of Appeals


On February 9, 2012

Intellectual Property Law on Copyright Requirements Before a Search Warrant May Be Issued in Copyright Cases Piracy In 1986, the Video Regulatory Board (VRB) applied for a warrant against Jose Jinco (Jingco), owner of Showtime Enterprises for allegedly pirating movies produced and owned by Columbia

Pictures and other motion picture companies. Jingco filed a motion to quash the search warrant but the same was denied in 1987. Subsequently, Jinco filed an Urgent Motion to Lift the Search Warrant and Return the Articles Seized. In 1989, the RTC judge granted the motion. The judge ruled that based on the ruling in the 1988 case of 20th Century Fox Film Corporation vs CA, before a search warrant could be issued in copyright cases, the master copy of the films alleged to be pirated must be attached in the application for warrant. ISSUE: Whether or not the 20th Century Fox ruling may be applied retroactively in this case. HELD: No. In 1986, obviously the 1988 case of 20th Century Fox was not yet promulgated. The lower court could not possibly have expected more evidence from the VRB and Columbia Pictures in their application for a search warrant other than what the law and jurisprudence, then existing and judicially accepted, required with respect to the finding of probable cause. The Supreme Court also revisited and clarified the ruling in the 20th Century Fox Case. It is evidently incorrect to suggest, as the ruling in 20th Century Fox may appear to do, that in copyright infringement cases, the presentation of master tapes of the copyright films is always necessary to meet the requirement of probable cause for the issuance of a search warrant. It is true that such master tapes are object evidence, with the merit that in this class of evidence the ascertainment of the controverted fact is made through demonstration involving the direct use of the senses of the presiding magistrate. Such auxiliary procedure, however, does not rule out the use of testimonial or documentary evidence, depositions, admissions or other classes of evidence tending to prove the factum probandum, especially where the production in court of object evidence would result in delay, inconvenience or expenses out of proportion to is evidentiary value. In fine, the supposed pronouncement in said case regarding the necessity for the presentation of the master tapes of the copy-righted films for the validity of search warrants should at most be understood to merely serve as a guidepost in determining the existence of probable cause in copy-right infringement cases where there is doubt as to the true nexus between the master tape and the pirated copies. An objective and careful reading of the decision in said case could lead to no other conclusion than that said directive was hardly intended to be a sweeping and inflexible requirement in all or similar copyright infringement cases.

Das könnte Ihnen auch gefallen